Prep 2015

Download as pdf or txt
Download as pdf or txt
You are on page 1of 792

American Academy of Pediatrics PREP 2015

American academy of pediatrics 1


American Academy of Pediatrics PREP 2015

PREP & Pediatric in Review (PIR) Content Specifications


The PREP covers all Content Specifications over a 5-year period. Thus, the material presented in
PREP The Curriculum covers approximately 20% of the Content Specifications each year in
either the PREP Self-Assessment or PIR. Therefore, in any 5-year continuous cycle, PREP The
Curriculum covers the vast majority of these knowledge statements and provides participants
with an educational program that is ideal for achieving lifelong learning.

Core Competency Icons


Six core competencies considered to be the foundation of high-quality medical care.

1. I-C: Interpersonal and Communication Skills result in effective information exchange


and teaming with patients, families, and other health professionals
2. P: Professionalism manifested through a commitment to professional responsibilities,
adherence to ethical principles, and sensitivity to a diverse patient population
3. PBLI: Practice-Based Learning and Improvement involves investigation and evaluation
of one's own patient care, appraisal, and assimilation of scientific evidence, and
improvements of patient care
4. SBP: Systems-Based Practice demonstrates an awareness of and responsiveness to the
larger context and system of health care and effectively calls on system resources to
provide care that is of optimal value
5. S: Safety
6. TE: Interdisciplinary Teams

American academy of pediatrics 2


American Academy of Pediatrics PREP 2015

Item 1
At his health supervision visit, you note that a 15-month old child's weight has fallen to the third
percentile, while his height and head circumference remain at the 25th to 50th percentile. He was
born at full term with an appropriate weight for gestational age. The child has had no illnesses
other than mild upper respiratory infections; he has no history of vomiting, diarrhea, or
constipation; and he has had no wheezing or breathing problems. His mother reports he is a
picky eater who often refuses food of various textures. There is no family history of growth
problems or chronic diseases. He has lived alone with his mother since his father was deployed
with the military 2 months ago. Physical examination findings are unremarkable, except for the
low weight.
Of the following, the MOST appropriate initial management for this child is to
A. begin supplemental nasogastric feedings
B. hospitalize him to monitor weight gain
C. obtain an occupational therapy assessment
D. order thyroid-stimulating hormone and insulin-like growth factor 1 levels
E. refer him to a child psychiatrist for separation issues

American academy of pediatrics 3


American Academy of Pediatrics PREP 2015

Item 1 TE Preferred Response: C


For the child in the vignette, the first step should be a thorough assessment of the current feeding
practices and optimization of oral intake. Given the stress of parental deployment, the mother
may benefit from behavioral health or peer support, but the primary intervention for the child is
directed at improved feeding practices.
New concepts stress that failure to thrive is a "physical sign" of under nutrition, usually with
multiple contributing factors. The management of failure to thrive, therefore, is directed at
optimizing feeding, for which a multidisciplinary team is often required. Occupational therapists
(OT) play a vital role in evaluating and treating dysfunctional feeding practices and food
aversions, and should be involved early in the evaluation of a child with failure to thrive. In
addition to the OT and pediatrician, providers who may be helpful in managing these patients
include a dietician to monitor energy, protein, vitamin and mineral intake; a speech and language
pathologist to assist with swallowing problems; a social worker to address family needs; and a
behavioral health specialist to focus on psychosocial issues. Some or all of these resources may
be available in the outpatient setting, and in most cases, hospitalization is not required. Although
previously children were often admitted to the hospital to observe weight gain in a controlled
environment, cost issues and the recognition that the child is usually better served in his own
home environment now make hospitalization a less useful approach to the management of failure
to thrive in all but the most severe cases.
The most important aspect of the evaluation of failure to thrive is a thorough history and physical
examination directed at assessing adequacy of intake, problems with nutrient absorption, or
increased metabolic demand. The practitioner needs to determine both the quantity and
appropriateness of the food intake (eg, incorrect formula preparation, excessive juice intake,
inadequate protein), and when appropriate, the family's access to food resources. Abnormal stool
patterns, frequent vomiting, personal or family history of respiratory (eg, cystic fibrosis) or
gastrointestinal illness, or symptom onset with first exposure to gluten intake may all suggest
malabsorption. The presence of chronic disease or genetic disorders may indicate increased
metabolic needs, requiring large caloric intake to support growth. For young children in
particular, it is important to note whether impaired growth started prenatally because intrauterine
growth restriction often affects long-term growth prognosis.
Physical examination findings may indicate both causes and effects of under nutrition. The
practitioner should look for signs of an underlying chronic disease or genetic syndrome, as well
as signs of malnutrition such as decreased subcutaneous tissue, cheilosis, or sparse hair. While
many practitioners obtain screening laboratory tests for children with failure to thrive, studies
have shown that laboratory evaluation rarely detects problems that were not already indicated by
the history and physical examination. For the child in the vignette with no concerning history,
unremarkable physical examination, and preserved length, obtaining thyroid studies and insulin-
like growth factor 1 is unlikely to add useful information to the evaluation.

The goal of treatment for failure to thrive is to provide adequate nutrient intake to support
appropriate growth. Ideally, this is achieved by improving food choices and feeding behaviors. In
some severely affected children, adequate calorie intake for catch-up growth can only be
achieved with nasogastric feedings.

American academy of pediatrics 4


American Academy of Pediatrics PREP 2015

PREP Pearls
• A multidisciplinary team including a pediatrician, a nutritionist, an occupational therapist, a
social worker, and other practitioners is useful in the evaluation and management of
failure to thrive.
• Failure to thrive is a physical sign of under nutrition that is usually multi factorial.
• Hospitalization for evaluation and treatment of failure to thrive is reserved for severely
affected children or cases in which social factors place the child at significant risk.
• Evaluation and treatment of failure to thrive is aimed at improving nutrient quantity and
quality, optimizing feeding practices, and overcoming food aversions.

American Board of Pediatrics Content Specification(s)


• Plan the management of an infant with failure to thrive

Suggested Reading
• American Academy of Pediatrics. Failure to thrive. In: Kleinman RE, ed.
• Pediatric Nutrition Handbook_ 6th ed. Elk Grove Village. IL: American Academy of
Pediatrics; 2009:601-636.
• Jaffe AC. Failure to thrive: current clinical concepts. Pediatr Rev. 2011; 32(3):100-107.
doi:10.1542/pir.32-3-100.

American academy of pediatrics 5


American Academy of Pediatrics PREP 2015

Item 2
In order to enhance the preventive care in your office's practices, you decide to use your new
electronic health record (EHR) to identify patients with risk factors that lead to obesity and adult
diseases. You make changes to the EHR template that should help the physicians in your practice
capture the required information. When reviewing records a few months later, you notice that
many details are not recorded, despite the template prompts. In order to develop a continuous
quality improvement action plan, you need to consider the desired information in order of
importance.
Of the following, the MOST important item to record is
A. body mass index percentile
B. detailed dietary intake
C. family history of obesity
D. gestational age and birth weight
E. maternal smoking during pregnancy

American academy of pediatrics 6


American Academy of Pediatrics PREP 2015

Item 2 S Preferred Response: A


Body mass index (BMI), defined as weight in kilograms divided by height in meters squared, is a
good surrogate marker for body fat and should be documented during each routine visit. The
electronic medical record can make tracking of BMI even easier through programs that calculate
BMI automatically and record both the vital signs and BMI into the health supervision visit
record, including plotting them on a graph.
Studies indicate that children who are moving in the direction of becoming overweight (BMI >
85%), or are already there, are frequently not identified. This identification is crucial, as early
intervention has been shown to be the most effective way to prevent future obesity. Most
overweight teenagers continue to have an elevated BMI as adults, and it is estimated that 80% of
overweight adolescents become obese adults with the attendant complications.
The risk of an individual child becoming overweight is influenced by several genetic and
environmental risk factors. Those with a family history of obesity or those born with low birth
weight from maternal smoking (or any cause) are at increased risk for obesity. Although
documenting these factors will help identify those most at risk, it does not substitute for frequent
recordings of height, weight, and BMI, as all children are at risk of obesity because of
environmental factors. While a detailed dietary history is helpful to providing counseling on
healthy eating, this will not identify children who are gaining weight excessively. An important
message is that fad diets or excessive activity alone are to be avoided. Rather, adopting a healthy
lifestyle that includes regular exercise and healthy eating, and matching the calories consumed to
the amount expended with exercise to maintain a healthy weight and growth pattern is key to
avoiding obesity.

PREP Pearls
• Documenting height, weight, and body mass index frequently will allow for early
detection and intervention for children who are moving in the direction of overweight
status.
• Prevention of obesity is important, as most lifestyle interventions later on in adolescence
are less successful.

American Board of Pediatrics Content Specification(s)


• Know which interventions have been effective and ineffective in caring for patients of
various ages who are obese
• Identify the genetic and environmental risk factors for obesity, including lifestyle choices

Suggested Reading
• Barlow SE. Expert committee recommendations regarding the prevention, assessment,
and treatment of child and adolescent overweight and obesity: summary report.
Pediatrics. 2007;120(suppl 4):5164-S192. doi:10.1542/ peds.2007-2329C.
• Benson L, Baer HJ, Kaelber DC. Trends in the diagnosis of overweight and obesity in
children and adolescents: 1999-2007. Pediatrics. 2009;123(1):e153—el 58.
doi:10.1542/peds.2008-1408.
• Manco M, Dallapiccola B. Genetics of pediatric obesity. Pediatrics. 2012;130(1):123-
133. doi:10.1542/peds.2011-2717.

American academy of pediatrics 7


American Academy of Pediatrics PREP 2015

• National Center for Chronic Disease Prevention and Health Promotion, Division of
Nutrition, Physical Activity, and Obesity. US Centers for Disease Control and Prevention
website.
• Silventoinen K, Rokholm B, Kaprio J, Sorensen TI. The genetic and environmental
influences on childhood obesity: a systematic review of twin and adoption studies. Int J
Obes (Land). 2010;34(1):29-40. doi:10.1038/ ijo.2009.177.
• Waters E, de Silva-Sanigorski A, Hall 131, et al. Interventions for preventing obesity in
children. Cochrane Database Syst Rev. 2011;(12):CD001871.
doi:10.1002/14651858.CD001871.pub3.

American academy of pediatrics 8


American Academy of Pediatrics PREP 2015

Item 3
You are seeing a 15-month-old girl for the first time in your clinic. She was breastfed until 10
months of age and was given soy formula between 10 months of age and 12 months of age and
tolerated it without any reactions. She has atopic dermatitis and a concomitant zinc deficiency
recently diagnosed by an allergist at 1 year of age, when she also tested positive for milk, egg,
peanut, and wheat allergies. At that time, the parents switched her to fortified orange juice.
The girl is scratching herself and is irritable on examination. Weight and height are at the 10th
percentiles (previously at 50th percentile at her 1-year checkup). On physical examination, she
has sparse, thinning hair; generalized erythematous, dry papules, and patches with excoriations;
and lichenification on her chest and antecubital popliteal fossae. She also has a perioral,
erythematous, desquamating, scaly rash that looks different from the rash on the rest of her body
(Item Q3). The remainder of her examination is unremarkable. The mother is frustrated that her
current diet is highly restricted and seeks your help. You set her up for an appointment at the
nutrition clinic.

ITEM Q3: Rash as described for the girl in the vignette.

Of the following, the BEST recommendation for this patient is a trial of


A. almond milk
B. goat milk
C. protein hydrolysate formula
D. rice milk
E. toddler soy formula

American academy of pediatrics 9


American Academy of Pediatrics PREP 2015

Item 3 Preferred Response: C


The child in this vignette should add protein hydrolysate formula to her diet. This child has
atopic dermatitis (AD) complicated by possible allergies to multiple food allergens. In addition,
she has zinc deficiency.

In children with moderate to severe atopic dermatitis, the atopic dermatitis can be triggered or
exacerbated by food allergies. This occurs more commonly in those younger than 5 years of age.
In this sub-group of children, evaluation of allergy to milk, egg, peanut, wheat, and soy could be
considered if the child has persistent AD in spite of optimized management and topical therapy,
has a reliable history of an immediate reaction after ingestion of a specific food, or both. As the
child gets older, they often outgrow the food allergies and instead tend to develop environmental
allergies that can worsen their AD. Pruritus and eczematoid skin lesions can develop after
intranasal or bronchial inhalation challenge with aeroallergens in sensitized patients with AD
who have specific immunoglobulin E (IgE) antibodies against the challenge allergen.

Unfortunately, in children with AD undergoing evaluation for possible food allergy triggers,
false-positive specific IgE tests (either on skin or blood testing) often occur, making it important
to correlate the test results with history, food diary, and elimination and reintroduction
challenges, as indicated. This is particularly true in children with low positive levels of food-
specific IgE, a past history of ingesting the food safely multiple times without any IgE-mediated
symptoms, or flaring of the AD.

In children who do have evidence of a food allergen trigger worsening the AD, avoidance is key.
This will often result in improvement of the AD. In children with multiple food allergies,
institution of a nutritious replacement is essential to provide optimal nourishment to a child, who
already may have a poor appetite because of the irritability caused by lack of sleep secondary to
the unbearable itching, and protein loss through the skin as a result of AD. Almond milk would
not be the first choice substitute in this child for several reasons: there is a possibility the child
has a concomitant tree nut allergy (the child is allergic to peanut, therefore there is a 30%-40%
chance of cross-reactivity). Almond milk is also largely composed of strained starch rather than
the much needed protein. However, it remains an option in some children with food allergies
who would like to have a milk-like liquid to drink, and who otherwise are having a well-
balanced diet. Rice milk is similarly of lesser nutritional value than animal or soy milk and
would not be a suitable alternative for this child. Rice contains phytates (also found in grains and
legumes), which can bind and hinder the absorption of zinc. Goat's milk has more than 90%
cross-reactivity to cow's milk. This child tested positive on allergy testing to cow's milk,
therefore goat's milk would not be an appropriate choice.

Soy milk is nutritionally of high quality and would have been an appropriate substitute if the
child did not have a concomitant zinc deficiency. The soy phytates and fiber oligosaccharides in
soy milk bind zinc in the gut. In one study with radiolabeled zinc, the highest absorption of zinc
was from human milk (41%) and the lowest was from soy formula (14%). All soy protein-based
formulas are therefore zinc-fortified. Therefore consumption of soy milk in this child, while
reasonable for other children with AD, would further worsen the zinc deficiency by hindering
absorption of zinc.

American academy of pediatrics 10


American Academy of Pediatrics PREP 2015

Protein hydrolysate formula, where the protein allergens are broken down to tiny fragments too
small to be recognized as allergen epitopes by the body, are the preferred option in this child.
While more expensive than the other options listed and occasionally critiqued for their
displeasing smell and taste, they are the most well-balanced and nutritionally complete option for
this child. Amino acid formulas should be reserved for children with significant gastrointestinal
absorption issues such as short-gut and similar disorders. They are typically not necessary in
children with food allergies alone.

Due to the significant nutritional imbalance caused by food restrictions, the child with AD and
food allergies should be referred and co-managed by a specialist in food allergies and a
nutritionist experienced in food allergies.

PREP Pearls
• In protein hydrolysate formula, proteins are broken down into fragments too small to be
recognized as allergens.
• Children with food allergies and on a restricted diet need to be monitored to prevent
nutritional imbalances.
• Children younger than 5 years of age with atopic dermatitis and food allergy triggers may
be at risk for nutritional inadequacies secondary to dietary restrictions.
• Phytates found in soy and rice may impair absorption of zinc.
• Children with multiple food allergies need to be co-managed with a nutrition expert.

American Board of Pediatrics Content Specification(s)


• Recognize the effects of a restricted diet for multiple food allergies on the nutritional
adequacy of a patient's diet

Suggested Reading
• American Academy of Pediatrics, Committee on Nutrition. Soy protein-based formulas:
recommendations for use in infant feeding. Pediatrics. 1998;101(0;148-153.
http://pediatrics.aappublications.org/ content/101/ I /148.full.
• Schneider L. Tines S, LM P, et al. Atopic dermatitis: a practice parameter update 2012. I
Allergy Clin Immoral. 2013;131(2):295-299, el-e27. doi:10.1016/j.jaci.2012.12.672.

American academy of pediatrics 11


American Academy of Pediatrics PREP 2015

Item 4
A 3-month-old previously healthy female infant presents to the emergency department with 2
days of decreased oral intake and nonbloody, nonbilious vomiting. She was in her usual state of
health prior to this illness. She usually drinks 6 oz bottles 5 times a day, but during the last 2
days, she has vomited after each feed. She has had only 1 wet diaper in the last 12 hours. She has
been increasingly disinterested while feeding, and is becoming more listless. She does not have
fever, upper respiratory symptoms, rash, difficulty breathing, or diarrhea. She is a product of a
full-term, normal, spontaneous vaginal delivery without complications. She does not have
allergies and she has received her 2-month immunizations. Development is normal for age.
Vital signs show a temperature of 37°C, pulse rate of 180 beats/min, respiratory rate of 35
breaths/min, blood pressure of 80/40 mm Hg, and oxygen saturation of 99% on room air.
Physical examination shows a generally well-developed, but lethargic baby. Her anterior
fontanelle and eyes are sunken, and her oral mucous membranes are dry. Tympanic membranes
are clear, and her throat is not erythematous. She is breathing without difficulty, and her lungs
are clear to auscultation bilaterally. Heart sounds are regular, with a low-pitched, grade 2/6
systolic murmur. Abdomen is soft, nontender, nondistended, and with no organomegaly.
Extremities are cool, with capillary refill time approximately 4 seconds.

Of the following, the MOST appropriate initial step is to


A. administer 4 oz of oral rehydration solution
B. administer 20 mL/kg 0.9% sodium chloride intravenous bolus
C. administer ceftriaxone 50 mg/kg intramuscularly
D. obtain abdominal ultrasonography
E. obtain echocardiogram

American academy of pediatrics 12


American Academy of Pediatrics PREP 2015

Item 4 Preferred Response: B


The infant in the vignette is in compensated hypovolemic shock. It is very important to
distinguish this entity from simple dehydration. According to the American College of Critical
Care Medicine's algorithm for early goal-directed therapy for pediatric shock (Suggest Reading
#2), the first step after recognition of decreased mental status and tissue perfusion is to start
intravenous fluid boluses of 20 mL/kg of isotonic saline or colloid within the first 15 minutes.

Shock is a condition in which oxygen delivery is insufficient to meet the metabolic demands of
vital organs. Restoration of oxygen delivery must be initiated without delay. Evidence of shock
in this patient, as opposed to dehydration, includes listlessness, tachycardia, cool extremities, and
delayed capillary refill. The patient's blood pressure is normal, which reflects the body's
compensatory mechanisms in shock, including elevated endogenous catecholamines,
vasoconstriction, and diversion of blood away from some vascular beds such as the skin and
splanchnic circulation. Hypotension is a very late and ominous occurrence in shock, and reflects
decompensation.

In addition to hypovolemic shock, there are 4 other forms of shock. (1) Cardiogenic shock is
caused by inadequate myocardial function or structural heart disease. This can present with signs
and symptoms of right or left ventricular failure, including tachypnea, pulmonary edema, rales,
hepatosplenomegaly, ascites, jugular venous distention, and peripheral edema. Although the
infant in this vignette has signs of decreased perfusion, she did not have other signs of heart
disease. The murmur noted is likely caused by turbulent flow in a hyperdynamic heart, and has
characteristics of an innocent "flow" murmur. Thus, echocardiography is unlikely to benefit the
patient. (2) Obstructive shock is caused by a mechanical barrier that impairs cardiac output, for
example, right or left ventricular outflow tract obstructions, pericardial tamponade, tension
pneumothorax, or pulmonary embolism. (3) Distributive shock occurs because of inadequate
intravascular volume caused by increased capillary permeability. (4) Septic shock is caused by
an infection that leads to a combination of decreased myocardial function, increased capillary
permeability, and decreased intravascular volume.

Abdominal ultrasonography may be a part of the workup for an infant with vomiting. For the
infant in this vignette, the duration of vomiting is more indicative of a viral source than an
obstructive cause, and restoration of cardiac output is a higher priority. Oral rehydration therapy
is increasingly recommended for acute gastroenteritis, and should be attempted in most cases of
dehydration, even if there is a history of vomiting and not tolerating oral intake. However, if the
patient is in shock, attempting oral rehydration would delay resuscitation, thereby risking
decompensation. This infant is listless and disinterested, therefore she may not be able to take
fluids by mouth. Although this clinical presentation does not include fever or a history
suggestive of septic shock, antibiotics are not contraindicated. Indeed, infections often occur
without an obvious history or source, and antibiotics should be administered early in septic
shock. The most important first step in any patient in shock should be to restore cardiac output as
soon as possible.

One of the most important initial steps in caring for the acutely ill child is determining whether
the patient is in shock. Poor perfusion, altered mental status, and tachycardia distinguish
hypovolemic shock from dehydration. When the diagnosis is made, immediate efforts should be
American academy of pediatrics 13
American Academy of Pediatrics PREP 2015

made to restore cardiac output. In this case, establishing intravenous or intraosseous access and
initiating fluid resuscitation is warranted.

PREP Pearls
• Early goal-directed therapy aimed at restoring cardiac output should be immediately
initiated in all patients with shock.
• There are 5 forms of shock: hypovolemic, cardiogenic, distributive, obstructive, and
septic.
• Oral rehydration therapy should be attempted in most patients with dehydration, unless
they are in shock.

American Board of Pediatrics Content Specification(s)


• Recognize the clinical signs of shock due to fluid loss, and manage appropriately

Suggested Reading
• Boluyt N, Bollen CW, Bos AP, Kok 1H, Offringa M. Fluid resuscitation in neonatal and
pediatric hypovolemic shock: a Dutch Pediatric Society evidence-based clinical practice
guideline. Intensive Care Med. 2006;32(7):995-2003. doi:10.1007/s 00134 - 006 - 0188-4
.
• Brierley J, Carcillo IA, Choong K, et al. Clinical practice parameters for hemodynamic
support of pediatric and neonatal septic shock: 2007 update from the American College
of Critical Care Medicine. Crit Care Med. 2009;37(2):666-688.
doi:10.1097/CCM.0b013e31819323c6.
• Han YY, Carcillo IA, Dragotta MA, et al. Reversal of pediatric-neonatal septic shock by
community physicians is associated with improved outcome. Pediatrics.
2003;112(4):793-799. doi:10.1542/peds.112.4.793.
• McKiernan CA, Lieberman SA. Circulatory shock in children. Pediatr Rev.
2005;26(12):451-460. doi:10.1542/pir.26-12-451.
• Turner DA, Cheifetz 1M. Shock. In: Kliegman RM, Stanton BF, St. Geme JW III, Schor
NF, Behrman RE, eds. Nelson's Textbook of Pediatrics. 19th ed. Philadelphia, PA:
Saunders Elsevier; 2011:305.
• Yager P, Noviski N. Shock. Pediatr Rev. 2010;31(8):311-319. doi:10.1542/ pir.31-8-311.

American academy of pediatrics 14


American Academy of Pediatrics PREP 2015

Item 5
You are precepting a group of medical students. There have been recent news reports of ricin-
laced mailings to public officials. In discussing the events, one of the students asks about
potential infectious agents of bioterrorism. In reviewing this topic, you present them a number of
pictures of lesions.
Of the following, the image MOST consistent with potential transmission in a bioterrorism event
is
A. Item Q5A
B. Item Q5B
C. Item Q5C
D. Item Q5D
E. Item Q5E

American academy of pediatrics 15


American Academy of Pediatrics PREP 2015

Item 5 S Preferred Response: A


Bacillus anthracis is considered a leading threat among the biological agents with potential for
use in a bioterrorist attack. This concern is based on outbreaks of anthrax observed in
Sverdlovsk, USSR in 1979 when 77 people were infected and 66 died following the accidental
release of spores from a microbiology facility, and the distribution of anthrax spores through the
United States mail in 2001, leading to 22 cases and 5 deaths. The spore form of B anthracis
remains a potential biohazard for weeks to years after release in the environment. Anthrax
infection can manifest as cutaneous, pulmonary, gastrointestinal, or meningoencephalitis disease
based on route of infection.

Cutaneous anthrax begins as a pruritic papule or vesicle that progresses over 2 to 6 days to form
the depressed eschar with surrounding edema, as shown by the lesion in Item Item C5A: Lesion
as depicted in a patient with cutaneous anthrax.

Antibiotic therapy of anthrax varies with the clinical presentation. For bioterrorism- associated
cutaneous anthrax, treatment with ciprofloxacin or doxycycline is recommended until
antimicrobial sensitivity results are available. Anthrax vaccine may be used in conjunction with
antimicrobial therapy. Treatment should be continued for 60 days, as spores may remain dormant
in mediastinal lymph nodes for prolonged periods of time. Systemic and inhalational anthrax are
treated with multidrug regimens consisting of ciprofloxacin and one of several other drugs with
activity against the organism.

Other infectious agents considered as potential bioterrorism threats with highest priority for
preparedness include smallpox, plague, tularemia, botulism, and viral hemorrhagic fevers
(including Ebola, Marburg, Lassa, Junin, and other related viruses). These organisms all have
potential for large scale dissemination, can cause high rates of morbidity and mortality, could
lead to public panic and social disruption, and require special action for public health
preparedness. Children may be particularly vulnerable during a bioterrorist attack because they
have a more rapid respiratory rate, frequent hand-to-mouth behavior, increased skin
permeability; higher ratio of skin surface area to mass, and less fluid reserve compared to adults.

The lesion in Item C5B demonstrates extensive erythema without a papule, vesicle, or edema,
which is characteristic of cellulitis due to group A streptococcal infection. Item C5C
demonstrates a papular central lesion within surrounding erythema with central clearing that is
seen with erythema migrans in early Lyme disease. The lesion in Item C5D, has a central papular
component with discharge, but without the eschar or marked surrounding edema, and is
secondary to a skin and soft tissue infection with community-acquired methicillin-resistant
Staphylococcus aureus. The raised scaly circular lesion in Item C5E is most consistent with
Tinea corporis infection. None of these infections are considered potential agents of bioterrorism.

PREP Pearls
• Cutaneous anthrax presents as a pruritic papule or vesicle that progresses over 2 to 6 days
to form a typical necrotic eschar with surrounding erythema.
• Bacillus anthracis is considered a leading potential agent for a bioterrorist attack.
• Anthrax can present with cutaneous, respiratory, gastrointestinal, or neurologic symptoms
based on the route of acquisition and dissemination.
American academy of pediatrics 16
American Academy of Pediatrics PREP 2015

American Board of Pediatrics Content Specification(s)


• Recognize the characteristic skin lesions of anthrax

Suggested Reading
• American Academy of Pediatrics. Anthrax. In: Pickering LK, Baker CJ, Kimberlin DW,
Long SS, eds. Red Book: 2012 Report of the Committee on Infectious Diseases. 29th ed.
Elk Grove Village, IL: American Academy of Pediatrics; 2012:228-232.
• American Academy of Pediatrics. Biological terrorism. In: Pickering LK, Baker CI,
Kimberlin DW, Long SS, eds. Red Book: 2012 Report of the Committee on Infectious
Diseases. 29th ed. Elk Grove Village, IL: American Academy of Pediatrics; 2012:111-
114.
• Bradley JS, Peacock G, Krug SE et at and the AAP Committee on Infectious Diseases
and Disaster Preparedness Adviosry Council. Pediatric Anthrax Clinical Management.
Pediatrics. 2014; 133:e1411 -e1436 .doi: 10.1542/ peds.2014-0563.
• Wright JG, Quinn CP, Shadomy S, Messonnier N. Use of anthrax vaccine in the United
States: recommendations of the Advisory Committee on Immunization Practices (ACIP),
2009. MMWR Recomm Rep. 2010;59(RR-6):1-30.

American academy of pediatrics 17


American Academy of Pediatrics PREP 2015

Item 6
You are teaching a group of medical students the differential diagnosis of edema in children.
You take them to the bedside of a patient who has edema associated with hypoproteinemia
secondary to proteinuria.
Of the following, the feature you are MOST likely to observe in this patient is
A. cola-colored urine, oliguria, and hypertension
B. edema that improves throughout the day
C. edema that worsens throughout the day
D. genital edema
E. pitting pedal edema

American academy of pediatrics 18


American Academy of Pediatrics PREP 2015

Item 6 Preferred Response: B


A patient with hypoproteinemia as a result of proteinuria would likely give a history of the
edema improving throughout the day. Edema results from increased interstitial fluid volume that
leads to tissue swelling. This tissue swelling can be localized (as in ascites) or generalized
(anasarca). Peripheral edema preferentially locates to dependent areas and is usually
characterized as pitting because of the movement of interstitial fluid in response to pressure. A
detailed history of the onset, location, and progression of edema provides clues to the underlying
etiology.

Nephrotic syndrome, in which proteinuria leads to hypoalbuminemia and generalized anasarca,


is one of the common causes of edema in children. The edema in nephrotic syndrome is
secondary to a combination of renal retention of salt and water (ie, the overfill hypothesis) or
decreased oncotic pressure due to hypoalbuminemia (ie, the under-fill hypothesis). Among
individual patients, the relative role of the overfill or underfill process in the pathogenesis of
edema may vary. In nephrotic syndrome, the edema is usually periorbital and more prominent
early in the morning, with minimal systemic symptoms on presentation and clinical examination.
In the initial stages, the periorbital and peripheral edema of nephrotic syndrome improves during
the course of the day as the interstitial fluid is mobilized and excreted by the normal functioning
heart and kidneys. With chronic proteinuria, persistent peripheral edema and anasarca develop in
these patients.

A history of diaphoresis, dyspnea on exertion, failure to thrive (especially in younger children),


or heart failure is suggestive of cardiac failure as the underlying cause of edema. Such patients
usually present with tachycardia, tachypnea, or a cardiac gallop on physical examination.
Patients with impaired left ventricular function have pulmonary edema and present with
respiratory distress. Patients with impaired right ventricular function present with significant
pitting peripheral edema. Cardiomyopathy, involving the right and left ventricles, may present
with pulmonary and peripheral edema. Impaired cardiac function leads to worsening peripheral
edema as the day progresses and is associated with exertion and increased activity

Acute glomerulonephritis is characterized by glomerular hematuria (cola- or tea-colored urine),


hypertension, and renal failure. The edema seen in acute glomerulonephritis is secondary to renal
dysfunction associated with volume overload.

The presence of pitting peripheral edema is not pathognomonic for any underlying etiology. It
may be seen in cardiac, renal (nephritis or nephrotic syndrome), or other conditions (Item C6)
associated with increased interstitial fluid volume. Scrotal edema can also be seen with
generalized anasarca from any cause.

Item 6. Etiology of Edema in Children


1. Hypoproteinemia—decreased capillary oncotic pressure
a. Proteinuria (nephrotic syndrome)
b. Malnutrition
c. Liver failure
d. Increased intestinal protein losses
2. Increased capillary hydrostatic pressure
American academy of pediatrics 19
American Academy of Pediatrics PREP 2015

a. Heart failure
b. Acute renal failure (acute glomerulonephritis)
c. Chronic renal failure
d. Vasodilators
e. Venous obstruction (thrombosis, localized edema)
3. Increased capillary permeability
a. Sepsis
b. Allergic reaction
4. Lymphatic obstruction (edema may be nonpitting)

PREP Pearls
• Nephrotic syndrome usually presents with edema that is periorbital and more prominent
early in the morning and with minimal systemic symptoms.
• Edema is the most easily identified clinical feature associated with hypoproteinemia.
• Impaired cardiac function leads to worsening peripheral edema as the day progresses and
is associated with exertion and increased activity.
• Pitting edema is not pathognomonic for any specific underlying etiology.

American Board of Pediatrics Content Specification(s)


• Recognize the clinical features associated with hypoproteinemia

Suggested Reading
• Levy PA. Edema. In: Mclnerny TK, Adam HM, Campbell DE, Kamat DP, Kelleher KJ,
eds. American Academy of Pediatrics Textbook of Pediatric Care. 19th ed. Elk Grove
Village, IL: American Academy of Pediatrics; 2009:14794482.

American academy of pediatrics 20


American Academy of Pediatrics PREP 2015

Item 7
During a routine health supervision visit, the mother of a 10-week-old infant tells you that her
baby has been experiencing bloating and flatulence for the past week. His diet consists of 5 oz to
6 oz of a cow milk-based formula given 5 times per 24 hours. Because of frequent episodes of
emesis after feedings, rice cereal was added to each bottle at 2 months of age. The baby has 4 to
5 loose to seedy stools per day. On physical examination, the baby is alert and vigorous. His
length and weight are tracking between the 50th and 75th percentiles. The infant's mother asks
you whether switching to a soy protein-based formula will help her baby's "gassiness"

Of the following, the MOST likely cause of this infant's symptoms is


A. cow-milk protein allergy
B. excessive energy intake
C. incomplete starch digestion
D. lactose malabsorption
E. sucrase-isomaltase deficiency

American academy of pediatrics 21


American Academy of Pediatrics PREP 2015

Item 7 Preferred Response: C


The symptoms demonstrated by the infant in the vignette are most likely caused by the
incomplete digestion of starch. Development of digestive-absorptive function of the
gastrointestinal tract is not complete at birth. Although all intestinal brush-border enzyme
activities (disaccharidases, oligopeptidases) and nutrient transport functions for
monosaccharide's, peptides, and amino acids reach mature levels by 36 weeks of gestation,
maturation of pancreatic exocrine function is delayed for some time after birth. The newborn can
assimilate considerable amounts of complex carbohydrates through hydrolysis by salivary gland
amylase until pancreatic function and small intestinal intraluminal pancreatic amylase activity
mature. Nevertheless, until complete exocrine pancreatic function is achieved (between 4 and 6
months of age), and certainly in infants younger than 4 months of age, dietary starches may be
hydrolyzed incompletely. As a result, increased amounts of undigested complex carbohydrates
such as starches pass into the colon, where bacterial fermentation results both in increased solute
load and gas (hydrogen, carbon dioxide) production that may cause the symptoms described for
the infant in the vignette.

The appropriate timing for introducing solid foods to the infant diet depends on the development
of both neuromuscular function and gastrointestinal maturation. The American Academy of
Pediatrics supports exclusive breastfeeding for the first 6 postnatal months. However, from a
developmental perspective, full-term infants often are capable of accepting solids
(complementary foods) between 4 and 6 months of age. Maturational readiness to tolerate
complementary feedings is indicated by loss of the extrusion reflex (usually by 4 months) and
the ability to swallow nonliquid foods. Aspiration is the most obvious risk posed by solid food
consumption before these developmental milestones are reached. No available evidence supports
the hypothesis that the introduction of solid foods either accelerates development of oral-motor
skills or helps infants to sleep through the night. Although cereals are generally the first solids
introduced to the infant diet in the United States, no studies support any particular sequence of
solid food progression. However, common sense dictates that new foods should be offered one at
a time to confirm tolerance.

Feeding complementary foods, such as cereals, to breastfed infants appears to increase the
likelihood of gastrointestinal infection. The direct relationship between early complementary
feedings and the incidence of diarrheal illness was demonstrated in several case-control studies.
This effect may be enhanced with greater duration and exclusivity of breast-feeding. Studies
have failed to show any clear risk reduction in the prevalence of upper and lower respiratory tract
illnesses, asthma, and otitis media among exclusively breastfed infants compared with infants
who received a mixed diet of human milk and solids. Data on the effect of early introduction of
solids on the development of allergies are conflicting.

Although cow milk protein allergy (CMPA) may manifest at any time in the first few months
after birth, the coincidence of starch feeding and gastrointestinal symptoms strongly suggests
that undigested complex carbohydrate is the causative agent in the patient in the vignette. The
absence of atopic signs, including vomiting, eczema, and urticaria, or the more common
presentation of gastrointestinal bleeding, make the diagnosis of CMPA unlikely.

American academy of pediatrics 22


American Academy of Pediatrics PREP 2015

Results of obesity studies in relation to infant feeding are inconclusive. In exclusively breastfed
infants, solid food introduction before age 6 months generally is associated with reduced human
milk intake without accelerated weight gain. However, formula-fed infants may be encouraged to
consume the same amount of formula, even after complementary feedings are introduced. This
practice may lead to increased calorie consumption and excessive weight gain, which have not
been demonstrated in the child in the vignette.

Gastrointestinal symptoms during the first few months after birth are often diagnosed as
complications of lactose malabsorption. However, congenital lactase deficiency is rare and
generally associated with severe diarrhea and inanition during the newborn period. Late-onset
primary lactase deficiency is extremely common worldwide, with an onset usually towards the
end of the first decade of life or later. Secondary lactase deficiency, as a consequence of
intestinal mucosal injury, may accompany a wide range of gastrointestinal insults, including both
infectious (eg, acute gastroenteritis) and noninfectious (eg, celiac disease) causes. However, the
clinical history in the patient in the vignette does not indicate a preexisting intestinal disorder.
Sucrase-isomaltase deficiency (CSID), the most common congenital disaccharidase deficiency,
may present after the introduction of cereals or fruits to the infant diet. In CSID, the presenting
symptom is most frequently a watery, acidic (pH <6.0) diarrhea.

PREP Pearls
• Symptoms of colic, including flatulence and bloating, are often the result of bacterial
hydrolysis of unabsorbed carbohydrate.
• In infants younger than 4 months of age, dietary starches may be hydrolyzed
incompletely.
• Lactase deficiency is extremely rare during the first year after birth,
• Unless it occurs as the consequence of intestinal mucosal injury.

American Board of Pediatrics Content Specification(s)


• Identify the age-related changes in the ability to absorb and digest different nutrients
relevant to infant feeding
• Understand the consequences of introducing solid food prematurely
• Know the appropriate age and sequence for introducing solid food into an infant's diet

Suggested Reading
• Fein SB, Labiner-Wolfel, Scanlon KS, Grummer-Strawn LM. Selected complementary
feeding practices and their association with maternal education. Pediatrics.
2008;122(suppl 2):S91-S97. doi:10.1542/peds.2008-13151.
• Hall RT, Carroll RE. Infant feeding. Pediatr Rev. 2000;21(6):191-200.
doi:10.1542/pir.21-6-191.
• Kramer MS, Chalmers B, Hodnett ED, et al; PROBIT Study Group. Promotion of
Breastfeeding Intervention Trial (PROBIT): a randomized trial in the Republic of
Belarus. JAMA. 2001;285(4):413-420. doi:10.1001/ jama.285.4.413.
• American Academy of Pediatrics Committee on Nutrition. Complementary feeding. In:
Kleinman RE, ed. Pediatric Nutrition Handbook. 5th ed. Elk Grove Village, IL:
American Academy of Pediatrics; 2009:113-142.

American academy of pediatrics 23


American Academy of Pediatrics PREP 2015

Item 8
A 5-year-old boy is brought to your office by his mother for evaluation of behavioral difficulties.
The mother reports he has always had a small head, motor and speech delays, and poor growth as
compared to his peers. Additionally, he has poor coordination, low tone, impulsivity,
hyperactivity, and attention difficulties. He has problems with reciprocal play with his peers in
kindergarten. He has had significant problems with learning numbers and their usage. He has a
known history of a ventricular septal defect that self-resolved as an infant.
His weight is 14.5 kg (5th percentile), his height is 101 cm (5th percentile), and head
circumference is 46 cm (less than 3rd percentile).
Physical examination is remarkable for a shortened fifth finger, as well as the facies shown in
Item Q8.

Of the following, the MOST likely diagnosis is


A. fetal alcohol syndrome
B. fetal anticonvulsant syndrome
C. fragile X syndrome
D. Rett syndrome
E. Williams syndrome

American academy of pediatrics 24


American Academy of Pediatrics PREP 2015

Item 8 IC S Preferred Response: A


The patient in this vignette has classic fetal alcohol syndrome (FAS). To diagnose FAS, a
physician should identify 3 cardinal features: abnormal facial features, growth deficiency, and
central nervous system (CNS) problems. The possibility of prenatal alcohol exposure is not
required to make the clinical diagnosis, but should be discussed with the patient's caregivers.
Classic facial features include mid-facial hypoplasia, smooth philtrum, thin upper lip,
micrognathia, short palpebral fissures, epicanthal folds, small jaw, and microcephaly. Prenatal
growth deficiency is typical as well as continued growth problems after birth. Height and weight
are typically less than the tenth percentile. Central nervous system difficulties include attention-
deficit/hyperactivity disorder (ADHD) problems, hypotonia, decreased impulse control, poor
coordination, poor memory, learning disabilities (especially in math), speech and language
delays, intellectual disability, impaired executive function, and sleep problems. Heart defects
may occur and could include ventricular septal defect or atrial septal defect. There is no reported
"safe" level of alcohol use during pregnancy, but large amounts of alcohol usage during any
pregnancy and intake during the first 3 months of pregnancy do increase the risk of having a
child with FAS.

Children with fetal anticonvulsant syndrome typically display behavioral problems including
ADHD, autistic features, learning difficulties, speech delay, and gross and fine motor delay.
Clinical findings may include intrauterine growth retardation, microcephaly, hypertelorism,
epicanthal folds, long philtrum with thin vermilion border, small mouth, flat nasal bridge, low-
set ears, hypoplasia of the distal phalanges and nails, cleft palate, meningomyelocele, and
congenital heart disease. The prevalence of neural tube defects in neonates exposed to valproic
acid in utero is 10 to 20 times higher than in the general population. Mild to moderate intellectual
deficits are common.

Fragile X syndrome is an X-linked dominant disorder caused by the FMR1 gene, which is
composed of a series of CGG repeats (normal number of repeats < 45). Premutation carriers for
fragile X have 55 to 200 CGG repeats. Premutation carrier females can have premature ovarian
failure and premutation carrier males can have fragile X tremor ataxia syndrome, which
resembles Parkinson disease. Full mutation carriers have greater than 200 CGG repeats and are
then clinically affected. Full mutation carriers have intellectual disability and the characteristic
physical findings. It can affect boys and girls, but boys are more severely affected.
Affected boys typically display mild to moderate intellectual disability (only one-third of girls
are intellectually disabled), anxiety, hyperactivity, attention-deficit disorder, autism spectrum
disorder, and occasionally seizures. Physical features become more apparent with age and
include large ears, a prominent jaw and forehead, pes planus, long and narrow face, and macro-
orchidism after puberty. Only half of girls have the physical characteristics.

Rett syndrome is a neurodevelopmental disorder that predominantly affects girls. Patients with
Rett syndrome typically have normal early growth and development followed by a characteristic
slowing of development, loss of purposeful hand movements, distinctive hand wringing,
decreased brain growth (acquired microcephaly), loss of coordination, developmental regression,
autistic-like behaviors, seizures, and intellectual disability. Apraxia and breathing difficulties
while awake are quite common.

American academy of pediatrics 25


American Academy of Pediatrics PREP 2015

Williams syndrome is a genetic disorder caused by a deletion on the long arm of chromosome 7.
These patients display a hoarse voice, cardiovascular anomaly, mild to moderate intellectual
disability, unique personality characteristics, and distinctive facial features. Facial features
include a broad forehead, short nose with a broad tip, full cheeks, and a wide mouth with full
lips. The most common cardiovascular anomaly is supravalvular aortic stenosis. Affected
individuals have engaging, friendly personalities. They do well with language skills and music,
but have difficulty with visual-spatial tasks.

Some patients exposed to alcohol in utero can have alcohol-related neurodevelopmental disorder.
These patients lack the physical stigmata common with FAS, but have intellectual disabilities
and problems with behavior and learning. Patients typically require specialized therapies and an
individualized education plan in school. Therapies typically include speech, occupation, and
physical therapy, as well as friendship training, specialized math tutoring, executive function
training, and parenting and behavior management training. They often need involvement with a
child psychiatrist to manage the behavioral problems. It is crucial for pediatricians to recognize
the features consistent with alcohol exposure in utero in both FAS and alcohol-related
neurodevelopmental disorder.

PREP Pearls
• The classic phenotype for fetal alcohol syndrome includes growth deficiency, midfacial
hypoplasia, smooth philtrum, thin upper lip, micrognathia, short palpebral fissures,
epicanthal folds, small jaw, and microcephaly.
• There is no reported "safe" level of alcohol use during pregnancy.
• Neurologic problems in fetal alcohol syndrome include attention-deficit/hyperactivity
problems. decreased impulse control, poor coordination, poor memory, learning
disabilities (especially in math), speech and language delays, intellectual disability,
impaired executive function, and sleep problems.

American Board of Pediatrics Content Specification(s)


• Recognize the clinical and laboratory features associated with fetal alcohol syndrome,
and manage appropriately

Suggested Reading
• American Academy of Pediatrics, Committee on Substance Abuse and Committee on
Children with Disabilities. Fetal alcohol syndrome and alcohol-related
neurodevelopmental disorders. Pediatrics. 2000;106(2):358-361.
• Streissguth AP, Bookstein FL, Barr HM, Sampson PD, O'Malley K, Young JK. Risk
factors for adverse life outcomes in fetal alcohol syndrome and fetal alcohol effects. Dev
Behav Pediatr. 2004;5(4):228-238.
• Welch-Carre E. The neurodevelopmental consequences of prenatal alcohol exposure.
Adv Neonatal Care. 2005;5(4):217-229. doi:10.1016/j. adnc.2005.04.007.

American academy of pediatrics 26


American Academy of Pediatrics PREP 2015

Item 9
The parents of a healthy 15-month-old boy arrive at your office inquiring about a small lump
near his eyebrow that they have noticed for several weeks. They state this lump has never been
red or tender. There is no history of trauma. The lump seems to be getting a little larger over
time. On physical examination, you find a 1.5 cm mobile subcutaneous, rubbery nodule just
above the lateral margin of the right eyebrow (Item Q9). The remainder of the examination is
unremarkable.

ITEM Q9: Nodule as described for the child in the vignette.

Of the following, the BEST initial plan for management is to


A. begin child protective team investigation
B. obtain plain radiographs of the orbit
C. perform needle aspiration
D. provide reassurance and observation
E. refer to surgery

American academy of pediatrics 27


American Academy of Pediatrics PREP 2015

Item 9 Preferred Response: E


The young boy in the vignette has the classic history associated with a dermoid or dermoid cyst.
A dermoid is a congenital lesion resulting from inclusion of ectodermal elements during closure
of the neural tube adjacent to fetal suture lines. It is the most common orbital tumor seen in
childhood. Complete surgical excision, with care not to rupture the cyst, is the standard of care,
so referral to surgery is the correct answer.

Dermoids occur most commonly on or superior to the orbital ridge in children, typically at the
lateral corner of the eyebrow in the area of the frontozygomatic suture line. The left lateral brow
is the most common site. Orbital dermoid cysts may be superficial or deep. The superficial type
usually presents in the first 2 years after birth and tends to grow slowly. They are asymptomatic,
nontender, subcutaneous nodules that appear skin colored or slightly bluish. They may feel firm
or rubbery. In this location, they are not associated with deep extension. However, about 3% of
orbital dermoids are located in the nasal midline (glabella, dorsum of the nose, or columella) and
it is important to recognize the potential for deep extension and central nervous system
communication here. Other deep orbital dermoid cysts may not be detected until the adolescent
years or later. These deep dermoids often present with proptosis.

Although trauma and nonaccidental injury should be considered in the differential diagnosis of
bumps on the head, reporting to the child protection team is not justified in this case because the
characteristics of the lesion are benign and not indicative of trauma. These characteristics
include: present and slowly enlarging for weeks, no bruising associated, and subcutaneous
rubbery nodule. Needle biopsy is not indicated for diagnostic purposes before surgical excision.
All patients with midline lesions or those who present with lesions in adulthood should be
evaluated with imaging (computed tomography or magnetic resonance imaging) to accurately
determine the size, location, and extension of the lesion when developing the management plan.

Superficial dermoids that typically present in the first 2 years after birth do not extend deeply
into the orbit, so plain radiography or referral to ophthalmology is not warranted. Dermoids will
continue to grow slowly and there is a small risk of complications, including erosion of
underlying bone, infection, or inflammation, therefore complete surgical excision is the preferred
treatment rather than observation only.

PREP Pearls
 Complete surgical excision is the preferred treatment for a dermoid cyst.
 Superficial dermoid cysts occur most commonly on or superior to the orbital ridge in
children.
 Dermoids that occur in the midline have a greater potential for deep extension and need
evaluation for bony erosion and central nervous system communication.

American Board of Pediatrics Content Specification(s)


• Recognize the clinical findings associated with a dermoid

American academy of pediatrics 28


American Academy of Pediatrics PREP 2015

Suggested Reading
• Ahuja R, Azar NE Orbital dermoids in children. Semin Oplithalmol.2006;21(3):207-211.
doi:10.1080/08820530500353963.
• Paller AS, Mancini Al, eds. Cutaneous disorders of the newborn. Hurwitz Clinical
Pediatric Dermatology: A Textbook of Skin Disorders of Childhood and Adolescence_
4th ed. St. Louis, MO: Saunders Elsevier; 2011:26.
• Pielop IA. Skin nodules in newborns and infants. UpToDate. Available online only for
subscription.
• Putman TC. Lumps and bumps in children. Pediatr Rev. 1992:13(10}:371.
doiar10,1542/pir.13-10-371.

American academy of pediatrics 29


American Academy of Pediatrics PREP 2015

Item 10
An 18-month-old, African-American boy is brought to your clinic by his grandmother for
yellowness of the eyes. The grandmother states that he has been irritable and sleeping more over
the past 3 days. Last week, he had nasal congestion and a cough. Concerned that he might
develop a fever, she gave him a few doses of antibiotics that were left over from his sibling's
recent treatment. He is pale and quiet. His oral temperature is 37.3°C, pulse rate is 125
beats/min, respiratory rate is 22 breaths/min, and blood pressure is 100/65 mm Hg. On physical
examination, he has icteric sclerae and a 2/6 systolic ejection murmur. There is no
hepatosplenomegaly. The remainder of the physical examination findings are unremarkable. The
following laboratory tests are obtained:
• White blood cell count, 10,500/µl (10.5 X 109/L), with 30% polymorphonuclear
leukocytes, 61% lymphocytes, 6% monocytes, and 3% eosinophils
• Hemoglobin, 6.0 g/dL (60 g/L)
• Hematocrit, 18%
• Mean corpuscular volume, 80 um3 (80 fL)
• Platelet count, 250 x 103/µL (250 x 109/L)
• Reticulocyte count, 10% (0.10)
• Direct bilirubin, 0.5 mg/dL (8.6 µmol/L)
• Indirect bilirubin, 4.0 mg/dL (68.4 µmol/L)
• Direct antiglobulin (Coombs) test, negative

Of the following, the MOST likely cause of this patient's findings is


A. autoimmune hemolytic anemia
B. β-thalassemia minor
C. Gilbert syndrome (UGT1A1 mutation)
D. glucose-6-phosphate dehydrogenase deficiency
E. transient erythroblastopenia of childhood

American academy of pediatrics 30


American Academy of Pediatrics PREP 2015

Item 10 Preferred Response: D


The boy in the vignette has a clinical history and presentation most consistent with glucose-6-
phosphate dehydrogenase (G6PD) deficiency. G6PD is an important enzyme in protecting the
red blood cell (RBC) from oxidative stress. It is the primary step in the pentose-phosphate
pathway (PPP) that produces nicotinamide adenine dinucleotide phosphate (NADPH) and keeps
glutathione in its reduced state. This is the only mechanism of producing NADPH in ABCs.
Certain substances, such as sulfa drugs and fava beans, interact with oxygen and hemoglobin to
produce a high level of toxic oxygen peroxides. In the presence of normal levels of NADPH and
reduced glutathione, the oxygen peroxides are neutralized and converted to pentose sugars via
the PPP, thus protecting the RBCs against damage. In G6PD deficiency, however, these oxygen
peroxides persist, causing damage to the hemoglobin and cell membrane, and ultimately leading
to hemolysis.

As RBCs age, their G6PD content decreases. Glucose6-phosphate dehydrogenase activity may
be 5 times higher in reticulocytes than in older cells. This leads to selective destruction of older
cells with lower G6PD activity during times of hemolytic crisis. Therefore, testing for G6PD
deficiency during a time of acute hemolysis and reticulocytosis may lead to false-negative
results. Testing should be done when the patient is at baseline.

The gene encoding G6PD is on the long arm of the X chromosome (band X428). Male
hemizygotes have only the mutated copy of this gene and therefore will have decreased G6PD
levels, whereas in heterozygote females with one affected allele and one normal allele, the level
of G6PD activity can vary from near normal to severely decreased.

Glucose-6-phosphate dehydrogenase deficiency is the most common enzyme defect in humans


and is estimated to affect more than 400 million people worldwide. Mutations in this gene are
associated with resistance to Plasmodium falciparum malaria, therefore the prevalence of G6PD
deficiency is higher in Mediterranean, African, Middle Eastern, and South Asian populations.
Many mutations have been reported in the G6PD gene. The wild type gene most commonly
found in white populations is the B allele, whereas the most common wild type polymorphism in
African populations is the A+ variant. The A— variant is the most common disease-causing
variant in African populations, whereas the G6PD Mediterranean variant is seen mostly in the
Mediterranean and Middle Eastern regions.

The common clinical presentations for G6PD deficiency are as follows: (1) neonatal jaundice,
(2) acute hemolytic anemia, and (3) chronic nonspherocytic hemolytic anemia (CNSHA).
Neonatal jaundice resulting from unconjugated hyperbilirubinemia is more common in G6PD-
deficient newborns than in those with normal G6PD levels. Though the exact mechanism of this
is not entirely known, treatment with phototherapy or exchange transfusion is similar to neonatal
hyperbilirubinemia from other causes. Acute hemolytic anemia can occur after exposure to
infections, drugs, or fava beans. Infection is the most common inciting factor for acute hemolysis
in G6PD deficiency. It has been reported in association with a wide variety of viral or bacterial
infections. Patients who present with acute hemolytic crises usually have a baseline G6PD
enzyme activity of less than 10%. This form is commonly seen in Mediterranean and Asian
populations. The common presenting signs and symptoms during an acute hemolytic episode are
malaise, weakness, and abdominal and back pain. Within 2 to 3 days, patients often have
American academy of pediatrics 31
American Academy of Pediatrics PREP 2015

jaundice and dark urine from hemoglobinuria, which can lead to renal failure. Treatment is
supportive with packed RBC transfusion, fluids, and hemodialysis if necessary. CNSHA is seen
in a small population of G6PD-deficient patients. Baseline G6PD enzyme activity is usually less
than 10%. Patients have chronic anemia with a wide range of severity. Children with CNSHA
can also present with significant splenomegaly.

The classes of G6PD deficiency are summarized below.In the patient described in this vignette,
the negative direct antiglobulin test makes autoimmune hemolytic anemia an incorrect answer.
Although the indirect bilirubin is elevated, Gilbert syndrome is not associated with anemia as
seen in this child. Beta-thalassemia is associated with a microcytic anemia, reticulocytosis, and
an elevated bilirubin level, the patient in this case has a normocytic anemia. Transient
erythroblastopenia of childhood is associated with a low reticulocyte count, not the elevated
reticulocyte count as seen in this patient.

Item CM Classes of Glucose-6-Phosphate Dehydrogenase Deficiency


Class Level of Baseline Enzyme Clinical Manifestations
Deficiency Activity
I Severe <10% Chronic nonspherocytic hemolytic anemia
II Severe <10%  Usually asymptomatic
 Acute hemolytic anemia in response to
infections, drugs, fava beans
 Mediterranean and severe Asian variants
III Moderate 10-60% Usually asymptomatic
Acute hemolytic anemia in response to
infections, drugs, fava beans
African (A—) variant

PREP Pearls
• There is a high prevalence of G6PD deficiency in African, Mediterranean, and Asian
populations. The African variant generally tends to be the less severe form.
• Glucose-6-phosphate dehydrogenase (G6PD) deficiency is an X-linked disorder.
• Higher levels of G6PD activity may be detected during acute hemolysis and
reticulocytosis, and may lead to false-negative results when testing for G6PD deficiency.

American Board of Pediatrics Content Specification(s)


• Recognize the inheritance pattern associated with G6PD deficiency
• Recognize the clinical findings associated with G6PD deficiency in patients of differing
ethnic backgrounds
• Plan appropriate management of hemolysis in a patient with G6PD deficiency

Suggested Reading
• Arese P, Gallo V, Pantaleo A, Turrini F. Life and death of G6PD deficient erythrocytes-
role of redox stress and band 3 modifications. Tranfus Med Hemother. 2012;39(5):328-
334. doi:10.1159/000343123.

American academy of pediatrics 32


American Academy of Pediatrics PREP 2015

• Glader B. Genetics and pathophysiology of glucose-6-phosphate dehydrogenase


deficiency. UpToDate. Available online only for subscription.
• Manganelli G, Amsullo U, Passarelli 5, Filosa S. Glucose-6-phosphate dehydrogenase
deficiency: disadvantages and possible benefits. Cardiovasc Hematol Disord Drug
Targets. 2012;10(2):143-150. doi:10.2174/18715 29X11313010008.

American academy of pediatrics 33


American Academy of Pediatrics PREP 2015

Item 11
You are seeing a 3-year-old girl for a health supervision assessment. Her mother says that she
does not have any concerns to discuss today. The mother reports that her daughter, who is an
only child, started walking at 1 year of age and uses mostly single words when she talks. She
does not attend day care or preschool. In the office, you notice that the girl avoids making eye
contact and spends most of her time focused on a doll that she brought with her, rather than on
either you or her mother. When she talks, she clearly enunciates words such as "doll:' "boy:' and
"door," but does not interact with you in a meaningful way. She is at the 60th percentile for
weight and the 75th percentile for height. Her physical examination findings are unremarkable.

Of the following, the MOST helpful test to administer today would be the
A. Ages and Stages Questionnaire
B. Child Behavior Checklist
C. Patient Health Questionnaire
D. Pediatric Symptom Checklist
E. Wechsler Preschool and Primary Scale of Intelligence TM

American academy of pediatrics 34


American Academy of Pediatrics PREP 2015

Item 11 Preferred Response: A


The patient in the vignette has signs of developmental impairment. Both her language and social
interaction are at a significantly delayed level for a 3 year old. Possible etiologies include an
autism spectrum disorder and an intellectual disability, which may not have any discoverable
medical cause. The fact that this child's mother stated that she did not have any concerns to
discuss with the physician could reflect that this is her first child and she does not know what is
developmentally appropriate for a 3 year old, or may reflect denial that a disorder may be present
in her child.

Among the choices offered, the Ages and Stages Questionnaire (ASQ) is the most appropriate
rating scale assessment for this child. The ASQ has been well researched and validated as a
screening tool to identify developmental difficulties in a primary care practice population up to
age 5 years. There are multiple versions of the parent questionnaire based on age, so that
different questions are asked when assessing a 3 year old versus a 1 1/2 year old. Another
appropriate developmental screening instrument to use for children up to age 8 years would be
the Parents' Evaluation of Developmental Status (PEDS), which asks parents if they have
concerns for their child in different developmental domains. The Modified Checklist for Autism
in Toddlers (M-CHAT) would be an appropriate developmental assessment to screen for autism
spectrum difficulties, but the M-CHAT has only been designed and validated for use between 16
and 30 months of age. Screening is recommended at the 18- and 24-month health supervision
visits.

The Child Behavior Checklist (CBCL) is a well-validated mental health screening tool
commonly used by mental health clinics. It is fairly long, not quickly completed, and requires
computer scoring software. The CBCL has the significant benefit of multiple validated subscales
that indicate the degree of mental health impairments in several clinical areas. The CBCL is not a
practical screening tool for use in a busy primary care practice given the time investment
required to administer and score it. Because the CBCL does not assess developmental
impairment, it would not be an appropriate choice for this patient.

The Patient Health Questionnaire (PHQ-9) is a brief, free, and validated depression rating scale
for adults and adolescents. The PHQ -9 would not assess developmental impairment in a 3 year
old.

The Pediatric Symptom Checklist (PSC) is a general mental health disorder screening tool,
available in 17- and 35-item versions. It is free to use, and has been validated for use in children
in primary care settings between the ages of 6 and 16 years. Although it is practical and easy to
use, it is not valid or appropriate for use in assessing developmental impairment.

The Wechsler Preschool and Primary Scale of Intelligence" (WPPSI) is an IQ assessment tool
that can be administered by a psychologist trained in its use. It is an appropriate tool for use in
children ages 2 1/2 to 7 years, and would be appropriate to assess this child's dysfunction.
However the WPPSI is a several-hour-long assessment that would be unlikely to be
appropriately administered by a skilled administrator in a primary care practice on the same day
as a routine health supervision visit.

American academy of pediatrics 35


American Academy of Pediatrics PREP 2015

PREP Pearls
• Parent-completed developmental screening tools help practitioners to identify the
presence of a developmental disability and to know when to refer a child for intervention.
• Providers should know the valid age ranges for the different rating scale assessments.
• The Modified Checklist for Autism in Toddlers is designed and validated for use between
16 and 30 months of age. Screening is recommended at the 18- and 24-month health
supervision visits.

American Board of Pediatrics Content Specification(s)


• Understand the uses and limitations of various developmental screening tools
• Understand the factors that can cause variations in the normal developmental sequence
and recognize their associated features

Suggested Reading
• American Academy of Pediatrics. Addressing Mental Health Concerns in Primary Care:
A Clinician's Toolkit. Elk Grove Village, IL: American Academy of Pediatrics; 2010.
• Marks KP, LaRosa AC. Understanding developmental-behavioral screening measures.
Pediatr Rev. 2012;33:448-458. doi:10.1542/pir.33-10-448.

American academy of pediatrics 36


American Academy of Pediatrics PREP 2015

Item 12
A 2-month-old infant is brought to your office for a follow-up visit. She was diagnosed with a
large perimembranous ventricular septal defects (VSD) at 4 weeks of age when she weighed 4.2
kg. Her mother states she sweats and tires with feedings and that she does not seem to be gaining
weight. Her feedings take up to 50 min and she needs to rest every 5 min; one month ago, she
took her feedings in 20 min. A physical examination shows a weight of 4.4 kg, a heart rate of
154 beats/min, and a respiratory rate of 60 breaths/min. Her lungs are clear. The point of
maximum impulse is displaced to the left; a 4/6 holosystolic murmur at the left midsternal border
obscures S1 and radiates to the left upper sternal border. Her liver edge is palpable 3 cm below
the right costal margin.

Of the following, the fluid and feeding regimen that would BEST allow optimal growth and
weight gain for this child is

A. 100 mL/kg per day; 100 kcal/kg per day; 30 kcal/oz formula
B. 140 mL/kg per day: 140 kcal/kg per day; 30 kcal/oz formula
C. 200 mL/kg per day; 145 kcal/kg per day; 22 kcal/oz formula
D. 200 mL/kg per day; 170 kcal/kg per day; 26 kcal/oz formula
E. 250 mL/kg per day; 215 kcal/kg per day; 26 kcal/oz formula

American academy of pediatrics 37


American Academy of Pediatrics PREP 2015

Item 12 S TE Preferred Response: B


Infants and children with cardiac disease that causes ventricular dysfunction or excessive
pulmonary blood flow will be sensitive to volume intake. Infants with large right-to-left shunts
such as would be seen with a ventricular septal defect will have increased catecholamines and
work of breathing. This will increase their metabolic demands so that the usual caloric
requirements for an infant, if provided, are not adequate for growth. The ideal caloric intake for
an infant in congestive heart failure secondary to increased pulmonary blood flow, such as the
infant in the vignette, is 130 to 140 kcal/kg per day. On the other hand, the overall fluid intake
must be low enough to keep from exacerbating the pulmonary congestion. Minimal hydration
requirements for any infant would be at least 100 mL/kg per day; the challenge is to provide a
formula or enhanced breast milk at a caloric density that can achieve both. In order to provide
140 kcal/kg per day will require 140 mL/kg per day when the caloric density is 30 kcal/oz. A
volume of 200 mL/kg per day will cause worsening volume overload, even when infants are
treated with diuretics such as furosemide. Less than 130 to 140 kcal/ kg will not allow adequate
growth. Consultation with a dietician to help the parents mix and provide this high-calorie
feeding or institute nasogastric tube feedings, if necessary, is an important safety measure.

PREP Pearls
• The ideal caloric intake for infants in heart failure is 130 to 140 kcal/kg per day with the
minimal volume needed for hydration.
• Consultation with a dietician to help the parents mix and provide this high-calorie feeding
or institute nasogastric tube feedings, if necessary, is an important safety measure.

American Board of Pediatrics Content Specification(s)


• Plan the dietary management of cardiac disease in a patient who is receiving a fluid-
restricted diet

Suggested Reading
• Rosenthal D, Chrisant MR, Edens E, et al. International Society for Heart and Lung
Transplantation: practice guidelines for management of heart failure in children. .1 Heart
Lung Transplant. 2004;23(12):1313-1333. doi:10.1016/j.healun.2004.03.018.
• Stettler N, Bhatia J, Parish A, Stallings VA. Feeding healthy infants, children, and
adolescents. In: Kliegman RM, Stanton BF, St Geme JW III, Schor NF, eds. Nelson
Textbook of Pediatrics. 19th ed. Philadelphia, PA: Elsevier Saunders; 2011:160-169.

American academy of pediatrics 38


American Academy of Pediatrics PREP 2015

Item 13
You are discussing a clinical case with a group of medical students. The patient has a history of
arthritis that started in the ankles, moved to the knees, and then shifted to the wrists. On physical
examination, there is a fever (40°C) and a friction rub. Echocardiography reveals mitral valve
prolapse and left ventricular dysfunction consistent with carditis. The laboratory results show an
elevated erythrocyte sedimentation rate. The medical students ask how long before onset of
symptoms does the infection associated with this diagnosis typically occur.
Of the following, the MOST accurate response to the students' question is

A. less than 1 week


B. 3 weeks
C. 6 weeks
D. 9 weeks
E. 12 weeks

American academy of pediatrics 39


American Academy of Pediatrics PREP 2015

Item 13 Preferred Response: B


The patient described in the vignette has fever, polyarticular migratory arthritis, carditis, mitral
valve prolapse, and an elevated erythrocyte sedimentation rate. This constellation of features is
consistent with acute rheumatic fever (ARF). ARF is an autoimmune disease that follows a
group A streptococcal infection, usually pharyngitis, typically presenting 3 weeks after the initial
infection. ARF is diagnosed by means of the Jones criteria.

Acute rheumatic fever is a major cause of acquired heart disease worldwide, however the
incidence has been declining in the United States. ARF leads to rheumatic heart disease in
approximately 60% of patients not treated for streptococcal pharyngitis. The peak age at
presentation is between 5 and 14 years. In 60% to 80% of patients, the first clinical manifestation
is arthritis, described as a painful migratory polyarticular arthritis that affects medium and large
joints. Sydenham chorea (uncontrolled movement of the face, trunk, or upper extremities) occurs
in a smaller percentage of patients between 1 and 6 months into the disease course.

Approximately half of patients with ARF develop carditis that can manifest as valvulitis, with or
without pericarditis and myocarditis. Rheumatic valvulitis usually presents as mitral valve
regurgitation with a soft, blowing, pansystolic murmur. Pericarditis presents as chest pain, and a
pericardial friction rub. Occasionally, ARF can have a subclinical carditis. Cutaneous
manifestations, which include erythema marginatum and subcutaneous nodules, occur more
rarely.

Jones Criteria
Confirmation of preceding Group A Streptococcus by positive throat culture, positive rapid
antigen test, elevated or rising antistreptolysin O titer
AND
2 major manifestations
OR
1 major and 2 minor manifestations
Major Manifestation Minor Manifestations
 Polyarthritis  Fever
 Carditis  Arthralgia
 Subcutaneous nodules  Prolonged PR interval on electrocardiogram
 Sydenham chorea  Elevated acute phase reactants (erythrocyte
 Erythema marginatum sedimentation rate, C-reactive protein)

PREP Pearls
• Acute rheumatic fever presents approximately 3 weeks after a group A Streptococcus
infection.
• Diagnosis of acute rheumatic fever (ARE) is based on the Jones criteria, which require
evidence of a group A streptococcal infection and either 2 major criteria or 1 major and 2
minor criteria.
• Major Jones criteria for ARF are polyarthritis, carditis, Sydenham chorea, subcutaneous
nodules, and erythema marginatum.

American academy of pediatrics 40


American Academy of Pediatrics PREP 2015

• Minor Jones criteria for ARF are fever, arthralgia, prolonged PR interval on
electrocardiograph, and elevated acute-phase reactants.

American Board of Pediatrics Content Specification(s)


• Understand the natural history of rheumatic fever
• Recognize the clinical findings associated with rheumatic fever, including major and
minor criteria

Suggested Reading
• Barash J. Rheumatic fever and post-group A streptococcal arthritis in children. Curr
Infect Du Rep. 2013;15:263-268. doi:10.1007/s11908-013-0335-3.
• Langlois DM, Andreae M. Group A streptococcal infections. Pediatr Rev.
2011;32(10):423-430. doi:10.1542/pir.32-10-423.
• Marijon E, Mirabel M, Celermajer DS, Jouven X. Rheumatic heart disease. Lancet.
2012;379(9819):953-964. doi:10.1016/S0140-6736(11)61171-9.

American academy of pediatrics 41


American Academy of Pediatrics PREP 2015

Item 14
A 5 year-old girl presents to your office for a health supervision visit. Her parents have decided
to feed the family a strict vegan diet. You counsel the parents about young children who follow a
vegan diet.

Of the following, the MOST appropriate statement to include is that children on this type of diet

A. generally cannot consume the required calories and nutrients to allow for adequate
growth
B. generally require less protein than omnivores due to the increased digestibility of plant
proteins
C. require B6 supplementation since B6 is only found in animal products
D. require calcium supplementation as leafy greens cannot provide enough calcium for
optimal bone health
E. require increased quantities of iron because of the decreased bioavailability of iron found
in plant sources

American academy of pediatrics 42


American Academy of Pediatrics PREP 2015

Item 14 Preferred Response: E


Iron in plant-based foods has a lower bioavailability than iron found in meat, therefore children
who do not eat meat require increased iron intake. However, while iron-deficiency anemia is the
most common nutritional deficiency in children, this condition does not appear to be more
common among individuals who do not consume meat.

There are several types of vegetarianism. The term lactovegetarian describes an individual
following a diet that excludes meat and fish, but includes dairy products and eggs.
Semivegetarians eat chicken and fish, but do not consume red meat. Vegans do not consume any
animal products.

Children following a vegetarian diet typically have normal growth, but tend to have lower body
mass indices than their nonvegetarian peers. For the child in the vignette, the family should be
counseled that a varied diet including grains, legumes, nuts, fruits, and vegetables should provide
sufficient calories for age-appropriate growth. Plant proteins can be less digestible than animal
proteins. Consequently, vegetarians often need higher amounts of protein. However, the
digestibility of soy protein and animal proteins are comparable; therefore, soy is an excellent
source of protein for vegetarians.

Pyridoxine (vitamin B6 is found in legumes, grains, fruits, and vegetables, so a child following a
vegan diet would not require supplementation of this vitamin. VitaminB12, a cobalamin
compound, occurs naturally in animal products including fish, meat, eggs, and dairy. Therefore,
individuals following a vegan diet need to consume foods or nutritional products that are
supplemented with vitamin B12 to avoid deficiency.

Leafy greens are an excellent source of calcium; children who do not consume many greens may
need to consume calcium-fortified foods or calcium supplements. Soy milk, cereals, and orange
juice are examples of products that are commonly available in calcium-fortified forms.

PREP Pearls
• Children following a vegan diet require increased iron intake and vitamin B12
supplementation.
• A child following a vegan diet would be expected to have normal growth with a body
mass index lower than the average for nonvegan peers.

American Board of Pediatrics Content Specification(s)


• Identify the nutritional complications that can result from vegetarian or vegan diets

Suggested Reading
 Amit M. Vegetarian diets in children and adolescents. Paediatr Child Health.
2010;15(5):303-308.
 Renda M, Fischer P. Vegetarian diets in children and adolescents. Pediatr Rey.
2009;30(1):el-e8. doi:10.1542/pir.30-1-e1.
 Van Winckel M, Vande Velde 5, De Bruyne R, Van Biervliet S. Clinical practice:
vegetarian infant and child nutrition. Ear J Pediatr.

American academy of pediatrics 43


American Academy of Pediatrics PREP 2015

 2011;170(12):1489-1494. dok10.1007/s00431-011-1547-x.

American academy of pediatrics 44


American Academy of Pediatrics PREP 2015

Item 15
A 6-year-old boy arrives at your clinic with a 1-week history of a rash on his face. The parents
report that the rash initially started as "red bumps" that became large and seemed to be filled with
fluid. Then the lesions "popped" and the rash developed into brownish plaques. His temperature
is 37°C and vital signs are normal for his age. On physical examination, there are tender areas of
denuded skin with surrounding erythema and overlying brownish crusts on the left cheek and
around the mouth, nares, and under the eyes (Item Q15).

ITEM 15: Rash as described for the boy in the vignette.

The conjunctivae are not involved. You note shotty cervical lymphadenopathy. The remainder of
the physical examination is normal.

Of the following, the MOST appropriate antimicrobial therapy for this patient is
A. oral clindamycin
B. oral penicillin
C. oral trimethoprim-sulfamethoxazole
D. topical bacitracin
E. topical neomycin

American academy of pediatrics 45


American Academy of Pediatrics PREP 2015

Item 15 Preferred Response: A


The boy described in the vignette has nonbullous impetigo. He would benefit from treatment
with oral clindamycin as it has activity against most isolates of staphylococci and streptococci,
the pathogens associated with this illness. Impetigo is a highly contagious superficial
dermatologic infection that occurs most commonly in young children. It is classified as bullous
(always caused by Staphylococcus aureus) or nonbullous (caused by S aureus or streptococci,
including group A β-hemolytic Streptococcus [GAS, Streptococcus pyogenes]) impetigo. The
nonbullous form occurs in approximately 70% of cases. Less frequently, ecthyma can occur,
which is a form of impetigo characterized by ulcerative lesions extending through the epidermis
and surrounded by raised margins. Both poststreptococcal rheumatic fever and
glomerulonephritis following impetigo have been reported.

With nonbullous impetigo, children typically present with erythematous papules and vesicular
lesions on the limbs or the face that rupture and form brownish or honey-colored crusts over
superficial erosions. The lesions can be painful, but systemic symptoms such as fever are rare.
Mild lymphadenopathy adjacent to the affected area is common. The infection is highly
contagious because the bacteria can be spread from person to person via direct contact (fingers,
etc) or by contact with contaminated personal items such as clothing or towels. Autoinoculation
allows for the formation of lesions in areas adjacent to the initial infection. Risk factors for
acquisition and spread of infection include atopy, skin trauma, humid conditions, and poor
hygiene.

There is no agreement on a single best treatment approach to patients with impetigo;


management is dictated by the disease severity, and the prevalence and susceptibility pattern of
methicillin-resistant S aureus (MRSA) isolates in the community (Item C15). Clindamycin is a
lincosamide antibiotic that reversibly binds to the bacterial 50S ribosomal subunit and inhibits
bacterial protein synthesis. It is bacteriostatic or bactericidal, depending on infection site,
pathogen, and concentration of drug. It is active against most staphylococci (methicillin-
susceptible S aureus and MRSA) and streptococci. Duration of therapy is approximately 1 week.

Although penicillin would be an appropriate choice of therapy for patients with impetigo known
to be caused by GAS, it would be unlikely to be effective against infection because of S aureus.
Trimethoprim-sulfamethoxazole is effective against susceptible strains of S aureus, but is not
useful for treating GAS. While topical agents such as bacitracin and neomycin have activity
against some gram-positive organisms, they are not effective against MRSA. In addition, they
can cause an allergic contact dermatitis, and less commonly, anaphylactic reactions. A Cochrane
review revealed that topical mupirocin, an inhibitor of bacterial isoleucyl t-RNA synthetase with
activity against GAS and S aureus, is as effective as oral antibiotics in the treatment of impetigo;
however, there are reports of increasing mupirocin resistance in S aureus isolates in the United
States. The development of resistance to topical retapamulin (a pleuromutilin) is less likely
because of its threefold mechanism of action. It binds the 50S ribosomal subunit, blocks p-site
interactions, and inhibits peptidyl transfer to inhibit bacterial protein synthesis. Topical
antiseptics (ie, sodium hypochlorite, hexachlorophene), although commonly used, are of
unproven benefit in the treatment of impetigo. Untreated, the natural course of infection is
thought to be several weeks; however, patient discomfort and transmissibility of infection
usually preclude withholding treatment.
American academy of pediatrics 46
American Academy of Pediatrics PREP 2015

Antimicrobial Agents for the Treatment of Impetigo*


Antimicrobial Agent Effective Effective Effective Special Considerations
(route) Against Against Against
MSSA MRSA GAS
Clindamycin (oral) + + + GI upset, increasing resistance
(MSSA, MRSA) in some
communities
Cephalexin (oral) + - +
Doxycycline (oral) + + - Not for use in children < 8
months of age
Trimethoprim + + - Not for use in children < 2
sulfamethoxazole (oral) months of age
Linezolid (oral) + + + Expensive, adverse effects
(cytopenia, etc), drug interactions
Mupirocin (topical) + + + increasing resistance in some
communities
Retapamulin (topical) + + +
*Local antimicrobial resistance patterns and disease severity should dictate preferred agent.
Abbreviations: MSSA, methicillin-susceptible Staphylococcus aureus; MRSA, methicillin-
resistant S aureus; GAS, group A Streptococcus; GI, gastrointestinal
+, effective or most isolates will be susceptible
-, not effective or most isolates will be resistant

PREP Pearls
• Children with impetigo typically present with erythematous papules and vesicular lesions
on the limbs or the face that rupture, and form brownish or honey-colored crusts over
superficial erosions,
• Impetigo can be caused by methicillin-susceptible Staphylococcus aureus, methicillin-
resistant Staphylococcus aureus (MRSA) or hemolytic streptococci (primarily group A
Streptococcus or Streptococcus pyogenes).
• Clindamycin has activity against most isolates of staphylococci and streptococci,
including MRSA.
• Oral antibiotic therapy for the treatment of impetigo should be based on the disease
severity and local antimicrobial resistance patterns.
• Topical therapy with mupirocin or retapamulin can be considered in treating impetigo
depending on the severity of illness.

American Board of Pediatrics Content Specification(s)


• Recognize the common pathogens associated with impetigo
• Recognize the clinical manifestations of impetigo and manage appropriately

American academy of pediatrics 47


American Academy of Pediatrics PREP 2015

Suggested Reading
• American Academy of Pediatrics. Group A streptococcal infections, In: Pickering LK,
Baker CJ, Kimberlin DW, Long SS, eds. Red Book: 2012 Report of the Committee on
Infectious Diseases. 29th ed. Elk Grove Village, IL: American Academy of Pediatrics;
2012:668-680.
• American Academy of Pediatrics. Staphylococcal infections. In: Pickering LK, Baker CJ,
Kimberlin DW, Long SS, eds. Red Book: 2012 Report of the Committee on Infectious
Diseases. 29th ed. Elk Grove Village. IL: American Academy of Pediatrics; 2012:653-
668.
• Bangert 5, Levy M, Hebert AA. Bacterial resistance and impetigo treatment trends: a
review. Pediatr Dermatol. 2012;29(3):243-248. doi:10.1111/j.1525-1470.2011.01700.x.
• Jackson MA, Newland JG. Staphylococcal infections in the era of MRSA. Pediatr Rev.
2011;32(12):522-532. doi:10.1542/pir.32-12-522.
• Koning S, van der Sande R, Verhagen AP, et al. Interventions for impetigo. Cochrane
Database Syst Rev. 2012;1:CD003261. doh10.1002/14651858. CD003261.pub3.

American academy of pediatrics 48


American Academy of Pediatrics PREP 2015

Item 16
A 16-year-old adolescent girl is brought to the emergency department by emergency medical
services (EMS) because of decreased responsiveness. Approximately 30 min before arrival, the
patient's mother arrived home from work and found her lying on the bathroom floor
unresponsive, with vomitus around her mouth and on the front of her shirt. An open bottle of
acetaminophen was on the bathroom counter. After a few unsuccessful attempts to "wake up" her
daughter, the mother called 911 for help. En route to the emergency department, the transporting
EMS team secured intravenous access and administered 2 doses of intravenous naloxone with no
change in her clinical status.

As you are evaluating the patient, her mother tells you that her daughter has been very depressed
over the past 2 months. During that time, she has gone through a breakup with her boyfriend, and
her maternal grandmother, who has terminal lung cancer, moved into the family's home and has
been receiving palliative care through a local hospice program. When you question the mother
about medications available within the family home, she states that the family's medicine cabinet
contained only acetaminophen and multivitamins, but she is unsure what medications are stored
in the grandmother's bedroom.

The patient's temperature is 37.2°C, heart rate is 130 beats/ min, respiratory rate is 16
breaths/min, blood pressure is 92/60 mm Hg, and pulse oximetry is 97% on room air. On
physical examination, the patient is minimally responsive and does not respond to any
commands. She does not consistently localize to painful stimuli. She is drooling and has dried
vomitus on her face. Her pupils are 3 mm in diameter bilaterally and equally reactive. A
complete physical examination reveals no findings suggestive of traumatic injury. Her skin
appears pale and clammy, but there is no rash.

Of the following, the BEST next step in the management of this patient is

A. administration of activated charcoal by nasogastric tube


B. administration of intravenous flumazenil
C. administration of intravenous N-acetylcysteine
D. lavage of gastric contents through a nasogastric tube
E. placement of an endotracheal tube

American academy of pediatrics 49


American Academy of Pediatrics PREP 2015

Item 16 S TE Preferred Response:E


The adolescent girl described in this vignette presents with markedly decreased responsiveness,
most likely because ingestion of multiple substances in an attempt to commit suicide. The best
next step in management is placement of an endotracheal tube to protect her airway.

More than 2 million toxic exposures are reported to American Association of Poison Control
Centers Toxi Exposure Surveillance System each year. Two-thirds of these exposures occur in
individuals younger than 20 years of age. All physicians caring for children must be familiar
with the evaluation and management of poisoning. Adolescents 13 to 19 years of age constitute a
second peak (after preschool-aged children) in the incidence of poisonings, accounting for 7% of
all exposures reported to poison control centers.

Unlike toxic exposures involving young children-which generally involve small quantities of a
single substance encountered accidentally-adolescent exposures are more likely to be intentional,
involve more than one substance, and have serious medical implications. Although only about
10% of toxic exposures among young children are referred to health care facilities, almost 50%
of those involving teenagers are significant enough to require emergency department
management.

Poisonings in adolescents and young adults may arise from substance abuse, experimental risk-
taking behaviors, and depression or suicidal intent. Teenagers may also experience toxic
exposures in the workplace because many are recruited into lower-tier, entry-level jobs that
involve cooking, cleaning, painting, construction, or landscaping, often with little training or
instruction in the use of protective equipment.

The first priority in treating any child who has ingested a toxic substance is to ensure stability of
the airway and to take any necessary steps to maintain adequate ventilation and circulation. The
adolescent in this vignette is minimally responsive, has already had episodes of vomiting, and is
demonstrating an inability to maintain her own airway (drooling), so securing a definitive airway
should be an immediate next step in her treatment.

Although activated charcoal is the gastrointestinal decontamination agent of choice in pediatric


patients and may have some benefit for this patient, it should never be administered in patients
with an unprotected airway. Furthermore, placement of a nasogastric tube for charcoal
administration poses a significant risk of precipitating vomiting and should not be attempted in
the patient until the airway is secured.

Flumazenil is an antidote that helps to reverse the effects of benzodiazepines by competitive


inhibition at central nervous system γ-aminobutyric acid (GAGA) sites. Although it is a useful
option in cases of known, pure benzodiazepine overdose in benzodiazepine-naive patients,
flumazenil may lower the seizure threshold and is contraindicated in tricyclic antidepressant
overdoses, as well as unknown overdoses. The substances ingested by the adolescent in the
vignette are still unknown, therefore administration of flumazenil would not be warranted.

Although the patient was found lying near an open bottle of acetaminophen, her markedly
depressed mental status cannot fully be explained by acute toxicity from isolated acetaminophen
American academy of pediatrics 50
American Academy of Pediatrics PREP 2015

overdose. Although intravenous N-acetylcysteine administration may be warranted in this patient


if further historical and laboratory findings indicate acetaminophen toxicity, securing her airway
is a much more immediate priority.

The clinical benefit of gastric lavage has not been confirmed in controlled studies, and its routine
use in managing poisoning cases is no longer recommended. Furthermore, ingestion of a
hydrocarbon or caustic substance-which would be a contraindication for gastric lavage-cannot
yet be excluded in the obtunded, drooling adolescent featured in this vignette.

For patients such as the severely poisoned teenager in this vignette, poison control centers can
provide health care professionals with information and expert guidance that can be invaluable in
providing the most appropriate treatment. The American Association of Poison Centers
(AAPCC) was created in the late 1950s to develop educational programs for health care
providers and to standardize the operation of existing poison control centers in the United States.
In 2002, poison control centers reporting to the AAPCC responded to more than 2 million human
toxic exposures. In addition to providing recommendations for the management of human
exposures, poison control centers supply information on poisons, poison prevention, drugs and
drug identification, teratogenicity of various agents, as well as occupational, medical, and
environmental concerns. Poison control centers manage cases involving both human and animal
exposures. Regional poison control centers are accessible to health care providers 24 hours a day,
365 days per year, and are staffed at all times by clinical toxicologists who can assist providers
with expert advice regarding the management of toxic exposures.

PREP Pearls
• The first priority in treating any child who has ingested a toxic substance is to ensure
stability of the airway.
• In contrast to toxic exposures involving young children-which are generally accidental
and involve small quantities of a single substance-adolescent exposures are more likely to
be intentional, involve more than one substance, and have serious medical implications.
• Regional poison control centers are accessible to health care providers 24 hours a day,
365 days per year, and can assist providers with expert advice.

American Board of Pediatrics Content Specification(s)


• Understand the data available from poison control centers
• Plan the management of poisoning by an unknown substance or by multiple substances

Suggested Reading
• Wahl MS, Leikin JB, Thoman M. Poison control centers. In: Erickson TB, Ahrens WR,
Aks SE, Baum CR, Ling LJ, eds. Pediatric Toxicology: Diagnosis and Management of
the Poisoned Child. New York, NY: McGraw-Hill Co; 2005:10-14.
• Woolf A. Special considerations in the adolescent. In: Erickson T, Ahrens WR, Aks SE,
Baum CR, Ling LJ, eds. Pediatric Toxicology: Diagnosis and Management of the
Poisoned Child. New York, NY: McGraw-Hill Co; 2005: 60-66.

American academy of pediatrics 51


American Academy of Pediatrics PREP 2015

Item 17
A mother in your practice is planning to have her second baby delivered by home birth under the
care of a certified midwife. She is meeting with you to review the plan for her baby following
delivery. Her 3-year-old child was born vaginally at term with no complications and was
breastfed for 2 years. Her pregnancy has been unremarkable. Her prenatal screen results include
blood type, O+; antibody, negative; hepatitis B, negative; rapid plasma reagin, negative; HIV,
negative; and group B Streptococcus, negative. The estimated fetal growth has been appropriate
for gestational age. She requests no medical interventions, including shots or medications, be
done to her baby after birth unless serious harm may occur as a result of their omission.

Of the following, the intervention MOST appropriate to recommend after initial bonding of this
baby is

A. bathing with chlorhexidine gluconate


B. immunization with hepatitis B vaccine
C. injection of intramuscular vitamin K
D. screening blood glucose for hypoglycemia
E. testing of cord blood for blood type

American academy of pediatrics 52


American Academy of Pediatrics PREP 2015

Item 17 I-C Preferred Response: C


The American Academy of Pediatrics (AAP) has published a policy statement summarizing the
standards of care for infants who are delivered at home. This statement emphasizes that the
subsequent care of the infant should observe the standards outlined in the Guidelines of Perinatal
Care. Included in these guidelines is the parenteral administration of vitamin K shortly after birth
to prevent late hemorrhagic disease, acknowledging that the intramuscular injection may be
delayed until after the first breastfeeding.

The classic and late-onset forms of vitamin K deficiency bleeding (VKDB) are most commonly
seen in breastfed infants who do not receive parenteral vitamin K prophylaxis at birth. Classic
VKDB typically presents with bleeding from the umbilicus, gastrointestinal tract, skin, and
circumcision sites between 24 hours and 1 week after birth and has limited morbidity.
Conversely, late-onset VKDB presents 2 weeks to 6 months after birth with intracranial
hemorrhage in over 50% of cases and is associated with a high morbidity. The incidence of late-
onset VKDB is low, with published numbers ranging from 1 to 20 cases per 100,000 births. The
parenteral administration of vitamin K shortly after birth dramatically reduces the risk of classic
and late-onset VKDB, although the late-onset form may still occur in infants with liver disease.

A British study published in 1992 suggested a possible link between parenteral vitamin K and
childhood cancer. Although these results were refuted by a subsequent study published in 1994,
some health care providers and families choose to use oral vitamin K. Oral vitamin K requires
repeated doses over the first 2 months after birth and can often be obtained only through a
compounding pharmacy. Oral vitamin K has not been shown to be as efficacious in preventing
late-onset VKDB, therefore the AAP continues to recommend a single parenteral dose of vitamin
K shortly after birth.

The AAP and the American College of Obstetricians and Gynecologists have stated that
hospitals and birthing centers are the safest settings for birth in the United States, but both groups
support the right of women to make informed delivery decisions. The Committee on Fetus and
Newborns developed the policy statement of home birth to support pediatricians counseling
mothers considering home birth, as well as providing guidelines for the subsequent care of the
newborn. Neonates born at home, including the unborn child in the vignette, should be
monitored closely as they transition to the extrauterine environment. Bathing may be delayed and
should be performed with only water or a mild, nonmedicated soap. The use of chlorhexidine is
not indicated because of the potential risk of systemic absorption. Hepatitis B vaccine continues
to be recommended, but may be deferred until the first office visit if the maternal hepatitis B
status is negative. Screening for hypoglycemia is indicated only if the infant is small or large for
gestational age, requires resuscitation at birth, or appears to be symptomatic. The testing of cord
blood for blood type is required only if the mother is Rh negative. Neonates who are born to
mothers with blood type O may be closely monitored clinically for the development of jaundice,
with bilirubin screening performed between 24 and 48 hours after birth.

American academy of pediatrics 53


American Academy of Pediatrics PREP 2015

PREP Pearls
• Parenteral vitamin K should be administered shortly after birth to prevent vitamin K
deficiency bleeding.
• The American Academy of Pediatrics policy statement on planned home birth can assist
pediatricians in supporting women seeking to deliver at home.

American Board of Pediatrics Content Specification(s)


• Plan appropriate evaluation and management of an infant who was born at home
• Recognize the signs, symptoms, and causes of vitamin K deficiency, and manage
appropriately

Suggested Reading
• American Academy of Pediatrics, Committee on Fetus and Newborn. Controversies
concerning vitamin K and the newborn. Pediatrics. 2003;112(pt 0:191-192.
• Centers for Disease Control and Prevention (CDC). 2013 Notes from the field: late
Vitamin K deficiency bleeding in infants whose parents declined Vitamin K
prophylaxis—Tennessee, 2013. MMWR Morb Mortal Wkly Rep. 2013;62(45):901-902.
• Darlow BA, Phillips AA, Dickson NP. New Zealand surveillance of neonatal vitamin K
deficiency bleeding (VKDB): 1998-2008. J Paediatr Child Health. 2011;47(7):460-464.
doi:10.1111/j.1440-1754.2010.01995.x.
• Kiebanoff MA, Read JS, Mills I L, Shiono PH. The risk of childhood cancer after
neonatal exposure to vitamin K. N Eng J Med. 1993:329(13):905-908.
doi:10.1056/NEJM199309233291301.
• Takahashi D, Shirahata A, Itoh 5, Takahashi Y, Nishiguchi T, Matsuda Y. Vitamin K
prophylaxis and late vitamin K deficiency bleeding in infants: fifth nationwide survey in
Japan. Pediatr Int. 2011;53(6):897-901. doi:10.1111/j.1442-200X.2011.03392.x.
• Warren JB, Phillip' CA. Care of the well newborn. Pediatr Rev. 2012;33(1):4-18.
doi:10.1542/pir.33-1-4.
• Watterberg KL, Committee on Fetus and Newborn. Policy statement on planned home
birth: upholding the best interests of children and families. Pediatrics. 2013;132(5):1016-
1020. doi:10.1542/peds.2013-2596.

American academy of pediatrics 54


American Academy of Pediatrics PREP 2015

Item18
A 6-year-old girl has a 1-day history of walking like she is "tipsy" Her parents report that on
awakening this morning, she could not sit upright without falling over and that when she walks,
she looks like she is drunk. In addition, she has been "sleepy" all day and slept 12 to 16 hours
every night for the past week. She had fever and cough 2 weeks previously. Her parents deny
any head injuries or exposure to toxins or medications. Her physical examination reveals a
temperature of 37.9°C, blood pressure of 92/49 mm Hg, heart rate of 98 beats/min, and
respiratory rate of 20 breaths/min. The girl is awake, but does not make eye contact; she follows
1-step commands slowly. She has lateral nystagmus in both eyes when looking to the left; her
right arm and leg are slightly weaker than her left arm and leg. When sitting upright, she cannot
maintain her balance and when placed standing she wobbles; her gait is wide-based and
unsteady. The remainder of the physical examination is unremarkable.

Of the following, the MOST likely diagnosis is


A. acute cerebellar ataxia
B. acute disseminated encephalomyelitis
C. basilar migraine
D. Friedreich ataxia
E. Opsoclonus-myoclonus ataxia syndrome

American academy of pediatrics 55


American Academy of Pediatrics PREP 2015

Item 18 Preferred Response: B


The girl in the vignette has acute ataxia, most likely because of acute disseminated
encephalomyelitis (ADEM). The viral prodrome a few weeks before the onset of symptoms, and
the presence of encephalopathy with new neurologic deficits, differentiates ADEM from other
causes of acute ataxia. In ADEM, magnetic resonance imaging of the brain shows asymmetric
multifocal abnormalities involving the white matter and often the basal ganglia (Item C18).
Cerebrospinal fluid analysis can show normal or elevated protein and normal or slightly elevated
white blood cells, but bacterial and viral cultures and viral polymerase chain reactions studies are
always normal. Treatment is with intravenous steroids, or sometimes with immunoglobulin
intravenously. Prognosis for recovery is good, although neurocognitive deficits may persist.

Acute cerebellar ataxia presents with acute ataxia a few weeks after a viral infection. However, it
does not present with other neurologic deficits such as the right hemiparesis seen in the girl in
the vignette. Basilar migraine can present with ataxia, but headache is prominent,
encephalopathy is absent, and the typical clinical course lasts hours, not days or weeks.
Friedreich ataxia is a chronic, progressive ataxia and does not present with encephalopathy.
Opsoclonus myoclonus ataxia syndrome typically affects infants and younger toddlers, and does
not present with encephalopathy. Opsoclonus is an erratic conjugate jerking of the eyes in all
directions, and should be differentiated from nystagmus which is a rhythmic conjugate eye
jerking, typically in just one direction. Although not one of the responses, a posterior fossa tumor
can present with acute or subacute ataxia. Due to the location in the posterior fossa, these tumors
typically also present with signs of increased intracranial pressure such as worsening headaches
or lateral gaze palsy. Abnormal gait, due to spasticity or hemiparesis such as found on the
examination of the girl in the vignette, can also be seen.

American academy of pediatrics 56


American Academy of Pediatrics PREP 2015

PREP Pearls

• Acute ataxia with encephalopathy and additional neurological deficits differentiates


ADEM from other causes of acute ataxia.
• In acute cerebellar ataxia, there are no mental status changes.

American Board of Pediatrics Content Specification(s)


• Recognize the presentation of acute ataxia

Suggested Reading
• Falchek SI. Encephalitis in the pediatric population. Pediatr Rev. 2012;33(3):122-133.
doi: 10.1542/pir.33-3-12
• Dinolfo EA. In brief: evaluation of ataxia. Pediatr Rev. 2001;22(5):177-178. doi:
10.1542/pi r.22-5-177

American academy of pediatrics 57


American Academy of Pediatrics PREP 2015

Item 19
A 12-month-old boy was diagnosed with failure to thrive. The child was born at full term and
was healthy and active for the first 6 months after birth. However, he has had poor weight gain
for the last 6 months, when his mother noted his appetite had decreased and his energy level had
progressively worsened. His diet consisted of breast milk exclusively for the first 6 months. He
started solids at 6 months of age but continued to breastfeed. Physical examination shows
swollen wrists and ankles and bowing of the legs. The remainder of the examination is
unremarkable.

Of the following, the single BEST test to confirm this patient's diagnosis is
A. alkaline phosphatase level
B. 1,25-dihydroxyvitamin D level
C. 25-hydroxyvitamin D level
D. parathyroid hormone level
E. 24,25 vitamin D level

American academy of pediatrics 58


American Academy of Pediatrics PREP 2015

Item 19 Preferred Response: C


The child in this vignette has vitamin D deficiency rickets. Risk factors for vitamin D deficiency
and rickets in an infant include breastfeeding without vitamin D supplementation, dark skin
pigmentation, and maternal vitamin D deficiency. The American Academy of Pediatrics
recommends 400 IU/ day of vitamin D to prevent vitamin D deficiency in infants who are
exclusively breastfed or consuming less than 1 L formula per day.

After receiving only breast milk for many months without vitamin D supplementation, the child
in this vignette became vitamin D deficient. This led him to eventually develop failure-to-thrive,
along with distinct signs of rickets. Clinical manifestations of rickets can include genu varum or
genu valgum (bowed legs, "knocked knees"), bowing of the forearms, flaring (widening) of the
metaphysis of the long bones, prominence of the costochondral junctions (rachitic rosary),
indentation of the lower anterior thoracic wall (Harrison groove), frontal bossing, craniotabes,
and frequently short stature with suboptimal weight gain. Systemic symptoms such as weakness,
hypotonia, and anorexia can also be present. Hypocalcemia and hypophosphatemia are also
common. The radiograph of the knee showing an irregular metaphysis with fraying is consistent
with this diagnosis (Item C19).

Vitamin D deficiency is characterized by low 25 -hydroxyvitamin D levels. Although often


considered the active form of vitamin D, 1,25-dihydroxyvitamin D should not be used to assess
for vitamin D deficiency. The circulating half-life of 1,25-dihydroxyvitamin D is only 4 hours. It
circulates at 1,000 times lower concentration than 25-hydroxyvitamin D, and the blood level is
tightly regulated by serum levels of parathyroid hormone (PTH), calcium, and phosphate. 1,25
dihydroxyvitamin D does not reflect vitamin D reserves, and measurement of 1,25-

American academy of pediatrics 59


American Academy of Pediatrics PREP 2015

dihydroxyvitamin D is not useful for monitoring the vitamin D status of patients with vitamin D
deficiency. Instead, measurement of 25-hydroxyvitamin D is the appropriate test.

24,25 vitamin D is an alternate metabolite of vitamin D. There is currently no clinical role in its
measurement for patients with vitamin D deficiency.

Alkaline phosphatase will be elevated in patients with rickets secondary to vitamin D deficiency.
However, alkaline phosphatase is often elevated in disorders of high bone turnover, liver disease
(unless fractionated to differentiate between bone and liver isoenzymes), and by itself is not
enough to make the diagnosis of vitamin D deficiency. Alkaline phosphatase can also be
elevated in benign transient hyperphosphatasemia of infancy and childhood, a temporary
condition where alkaline phosphatase is often increased secondary to gastrointestinal infection or
failure-to-thrive.

Parathyroid hormone is also increased in vitamin D deficiency, but PTH elevation can also be
seen in other forms of rickets and in hyperparathyroidism.

PREP Pearls
• Measurement of 25-hydroxyvitamin D (the storage form of vitamin D) is the definitive
test for diagnosing vitamin D deficiency.
• The child who is being exclusively breastfed or consuming less than 1 L formula per day
should be prescribed 400 IU/day of vitamin D.

American Board of Pediatrics Content Specification(s)


• Recognize the presenting signs and symptoms of vitamin D-deficient rickets, and manage
appropriately
• Recognize the laboratory and radiologic features of vitamin D-deficient rickets

Suggested Reading
• Hoiick MF, Binkley NC, Bischoff-Ferrari HA, et al. Evaluation, treatment, and
prevention of vitamin D deficiency: an Endocrine Society clinical practice guideline. J
Clin Endocrinol Metals. 2011;96(7):1911-1930. doi:10.1210/jc.2011-0385.
• Root AW. Disorders of bone mineral metabolism: normal homeostasis. In: Sperling M,
ed. Pediatric Endocrinology. 2nd ed. Philadelphia. PA: Saunders Elsevier; 2002:625-654.
• Wagner CL, Greer FR, American Academy of Pediatrics Section on Breast feeding,
American Academy of Pediatrics Committee on Nutrition. Prevention of rickets and
vitamin D deficiency in infants, children, and adolescents. Pediatrics. 2008;122(5):1142-
1152. doi:10.1542/peds.2008-1862.

American academy of pediatrics 60


American Academy of Pediatrics PREP 2015

Item 20
You are examining a healthy 4-year-old child whose parents immigrated to the United States
from Nepal just before he was born. You note that his height and weight are at the 25th to 50th
percentile, while his parents are at approximately the third percentile. You compare this family's
nutrition and exercise patterns with those of the traditional Nepalese lifestyle.

Of the following, this family's nutrition and exercise patterns are MOST likely to be
characterized by

A. adherence to the traditional diet of the parents' country of origin


B. an increase in physical activity levels
C. increased consumption of lower caloric density foods such as fruits and vegetables
D. maintaining consumption of traditional foods while adding new food types
E. rapid assimilation of the typical dietary pattern of the United States

American academy of pediatrics 61


American Academy of Pediatrics PREP 2015

Item 20 Preferred Response: D


Eating habits are controlled by many factors including food availability and economics, but one
of the strongest influences is culture. For immigrants and minority populations, dietary
acculturation is the process by which the minority group adopts the dietary practices of the host
country. Rarely is this a linear process. Instead, it often involves retaining some traditional foods
while discarding others, adapting new foods to traditional dishes, and adopting some of the
dietary practices of the host country outright. For many migrants, limited accessibility and the
often high cost of traditional foods, as well as daily exposure to western foods and advertising,
makes maintenance of a strict traditional diet difficult. Migrants who come from a culturally and
linguistically similar background may adopt the host country's patterns rapidly, but those from a
more distinct culture are likely to slowly develop the bicultural pattern of retaining some
traditional foods, adapting new ones, and adopting foods of the host country.

In addition to dietary acculturation, immigrants, like the host country residents, experience
nutritional transition characterized by a shift to more energy-dense diets, a decrease in
consumption of lower-calorie fruits and vegetables, and a decrease in physical activity. While
some dietary changes migrants experience can promote health (eg, replacing lard with
unsaturated fats), most evidence indicates that adoption of the western lifestyle increases chronic
disease risk. For example, female Asian migrants who have lived in the United States for 10
years show an 80% higher risk of breast cancer compared with recent immigrants.

PREP Pearls
• Dietary acculturation is the process by which a minority group adopts the dietary
practices of the host or dominant group.
• Dietary acculturation typically involves retaining some traditional foods, adapting new
ones to traditional dishes, and adopting foods of the host country outright.
• Evidence suggests that the nutritional transition from low- to high-calorie dense foods
combined with a more sedentary lifestyle increases chronic disease risk among immigrant
populations.

American Board of Pediatrics Content Specification(s)


• Understand the family and cultural determinants that influence dietary practices and
nutrition

Suggested Reading
• American Academy of Pediatrics. Cultural considerations in feeding children. In:
Kleinman RE, ed. Pediatric Nutrition Handbook. 6th ed. Elk Grove Village. IL:
American Academy of Pediatrics; 2009:183-199. Fitzgibbon ML, Beech BM. The role of
culture in the context of school-based BM! screening. Pediatrics. 2009;124 (suppl 1):550-
562. doi:10.1542/ peds.2008-3586H.
• Satia IA. Dietary acculturation and the nutrition transition: an overview. Appi Physiol
Mar Metal). 2010:35(2):219-223. doi:10.1139/H10-007.
• Satia-Abouta I, Patterson RE, Neuhouser MI., Elder J. Dietary acculturation: applications
to nutrition research and dietetics. Am Diet Assoc. 2002;102(8)1105-1118.

American academy of pediatrics 62


American Academy of Pediatrics PREP 2015

Item 21
You are seeing a 17-year-old adolescent girl in your office for a lesion on her vulva. She is
sexually active and you diagnose her with genital warts. According to your records, she has
never received the human papillomavirus (HPV) vaccine. Her mother had not given consent,
stating that her daughter was not sexually active and did not need the vaccine. The girl reports
that she received 1 dose of the vaccine at a local clinic 1 year ago, and asks if she will need
additional doses or if she will need to restart the vaccine series.

Of the following, the BEST advice to the girl is that


A. she can receive the second dose today without restarting the series
B. the results of a Papanicolaou (Pap) test with HPV DNA are required before restarting the
series
C. the series can be restarted today after performing a Pap test with HPV DNA
D. the vaccine can be given as treatment for the genital warts
E. the vaccine will not be protective, as she is already infected with HPV

American academy of pediatrics 63


American Academy of Pediatrics PREP 2015

Item 21 Preferred Response: A


The human papillomavirus (HPV) vaccine is available in 2 forms: HPV4 and HPV2. HPV4 is
active against 4 virus types (6, 11, 16, and 18), while the HPV2 is active against 2 virus types (16
and 18). The recommendation from the Advisory Committee on Immunization Practices of the
US Centers of Disease Control and Prevention and the American Academy of Pediatrics is that
either vaccine be used for routine immunization of girls, beginning at age 9 years through 26
years of age. HPV4 is recommended for boys at 11 or 12 years of age, but can be used from 9
years to 26 years of age. Currently 3 doses of 0.5 mL are recommended with the minimum
interval between the first 2 doses being 4 weeks and between the second and third dose being 12
weeks, with at least 24 weeks between the first and third dose. The recommendation is that the
series need not be restarted if interrupted; therefore, it is recommended that the girl in the
vignette receive the second dose at this visit.

Exposure to the virus is not a contraindication to receiving the vaccine. Studies indicate that on
average about one quarter of subjects will have been exposed to 1 type of the virus, but only
0.1% will have been exposed to all 4 types. The vaccine will provide protection against the types
that have not already been acquired. Therefore, the results of a Papanicolaou (Pap) test, with or
without the presence of HPV DNA, do not negate the recommendation to give the vaccine.
Currently, the recommendation is to begin performing Pap tests at 21 years of age. Therefore, the
girl in the vignette neither requires a Pap test today, nor should she wait to get the vaccine until
results are available, if it is performed. The first Pap testing done when she is 21 years of age
should not include a request for HPV DNA. The need for this testing in the future depends on the
results of the current Pap test. The recommendations are based on the finding that in 91% of
those with low-grade lesions, the lesions will clear in 3 years, while only 3% of high-grade
lesions show progression with time. Therefore, delaying the first Pap test to 21 years of age will
prevent unnecessary procedures being performed on the cervix of an adolescent for lesions that
will most likely clear spontaneously, but will catch lesions that have persisted and are at risk for
further progression. The vaccine's role is only for prevention; it is not a treatment option.

PREP Pearls
• The human papillomavirus (HPV) vaccine series can be resumed at any time after being
interrupted.
• The HPV vaccine series should be administered irrespective of the history of previous
exposure to the virus.

American Board of Pediatrics Content Specification(s)


• Know the recommendations, limitations, and schedule for the human papillomavirus
vaccine

Suggested Reading
• American Academy of Pediatrics. Human papillomaviruses. In: Pickering LK, Baker CJ,
Kimberlin DW, Long SS, eds. Red Book: 2012 Report of the Committee on Infectious
Diseases. 29th ed. Elk Grove Village, IL:
• American Academy of Pediatrics; 2012:524-530.
• Brigham KS, Goldstein MA. Adolescent immunizations. Pediatr Rev. 2009;30(2):47-56.
doi:10.1542/pir.30-2-47.
American academy of pediatrics 64
American Academy of Pediatrics PREP 2015

• Darden PM, Thompson DM, Roberts JR, et al. Reasons for not vaccinating adolescents:
national immunization survey of teens, 2008-2010. Pediatrics. 2013;131(4):645-651.
doi:10,1542/peds.2012-2384.
• Saslow D, Solomon D, Lawson HW, et al. American Cancer Society, American Society
for Colposcopy and Cervical Pathology, and American Society for Clinical Pathology
screening guidelines for the prevention and early detection of cervical cancer. Am I an
Pathol. 2012;137(4):516-542. doi:10.1309/AJCPTGD94EVRSJCG.

American academy of pediatrics 65


American Academy of Pediatrics PREP 2015

Item 22
A 6-year-old boy is brought to your office for an urgent visit because of wheezing he has
experienced since returning from his father's house this past weekend. Apart from faint end-
expiratory wheezes bilaterally, the rest of the physical examination is unremarkable. You order
administration of nebulized albuterol in your office. You review his symptoms and medication
use with his mother, who reports that he has been using his albuterol almost daily for the past 3
months, ever since the parents separated. The child visits with his father every other weekend.
His mother voices her concerns that there are environmental triggers in the father's house that
make the child's asthma worse. The mother states that the father's girlfriend wears strong
perfumes and likes burning incense and candles. There are no pets or history of visible mold
growth in either house. The mother smokes tobacco. She states that she hopes to quit smoking
eventually, but with the stress of the separation, she has been smoking more frequently.

Of the following, the environmental trigger MOST likely to contribute to this child's symptoms
is

A. air freshener
B. cleaning agents
C. incense sticks
D. perfume
E. tobacco smoke

American academy of pediatrics 66


American Academy of Pediatrics PREP 2015

Item 22 I-C Preferred Response: E


While the child in the vignette has multiple environmental triggers, second-hand tobacco smoke
exposure is the trigger most likely responsible for the worsening of his asthma symptoms over
the past 3 months. Tobacco smoke is a common airway irritant. Multiple studies have shown that
second-hand smoke (SHS) exposure, particularly from parental smoking, is associated with
increased symptoms, airway responsiveness, medication usage, and utilization of health services
for asthma. Exposure to tobacco smoke additionally exacerbates inflammatory airway responses
to allergens. Intervention studies have demonstrated decreased morbidity and airway
hyperresponsiveness with reduction in SHS exposure. The National Asthma Education and
Prevention Program Guidelines recommend the elimination of SHS exposure in the homes of
children with asthma. Parents may not recognize tobacco smoke as a trigger if the child does not
have immediate symptoms. Clinicians should advise patients with asthma not to smoke or be
exposed to SHS. They should also discuss smoking habits with parents and caregivers of
pediatric patients, emphasizing the role of SHS as an airway irritant making the airways
vulnerable to the effects of other triggers. If a parent is unwilling or unable to quit smoking, the
option of not smoking in the same room where the children sleep or play should be suggested. If
feasible, parents and caregivers should be instructed to restrict their smoking to outside the
home.

Other environmental irritants can also trigger acute asthma episodes, particularly in children with
uncontrolled airway inflammation. The use of these irritants is typically sporadic, therefore these
irritants commonly tend to trigger acute symptoms rather than chronic worsening, and hence are
not likely responsible for this child's worsening asthma over the past 3 months. Agents such as
fireplace smoke, ashes, incense sticks, aerosol sprays (air fresheners, cleaning agents, perfumes),
fumes from unvented gas stoves, cooking Odors, musty odors, shower steam, traffic fumes, air
pollution, desert dust, and workplace irritants can trigger bronchospasm. Chlorine-based
household cleaners, cleaning fluids, and powders can also trigger asthma attacks. Newly installed
carpeting, paint, and furnishings in homes and offices can release volatile organic compounds
and formaldehyde. Increases in ambient particulate matter, elemental carbon and soot. Nitrogen
dioxide and ozone have been associated with worsening asthma outcomes. Recommended
measures include changing to a nonpolluting, more effective indoor heat system, using unscented
and non-aerosol products, and avoiding use of wood burning stoves, fireplaces, or unvented
stoves or heaters.

PREP Pearls
• Environmental tobacco smoke exposure can trigger worsening of asthma control and be
associated with poor asthma outcomes.
• Strong odors, aerosol sprays, and ambient particulate matter can trigger acute
bronchospasm.
• Avoidance of exposure to environmental irritants is recommended to gain optimal asthma
control.

American Board of Pediatrics Content Specification(s)

American academy of pediatrics 67


American Academy of Pediatrics PREP 2015

• Identify the common environmental irritants present in the home that can contribute to
respiratory disease in children
• Recognize the possible side effects of environmental tobacco smoke exposure

Suggested Reading
• Evans D, Levison MJ, Feldman CH, et al. The impact of passive smoking on emergency
room visits of urban children with asthma. Am Rev Respir Dis. 1987;135(3);567-572.
• Larsson ML, Frisk M. Hallstram I, Kiviloog J, Lundback B. Environmental tobacco
smoke exposure during childhood is associated with increased prevalence of asthma in
adults. Chest. 2001;120(3):711-717. dol.:10.1378/ chest.120.3.711.
• Martinez FD, Antognoni G, Macri F, et al. Parental smoking enhances bronchial
responsiveness in nine-year-old children. Am Rev Respir Dis. l988;138(3):518-523.
• National Heart, Lung, and Blood Institute. National asthma education and prevention
program expert panel report guidelines for the diagnosis and management of asthma.
National Institutes of Health, US Department of Health and Human Services website.

American academy of pediatrics 68


American Academy of Pediatrics PREP 2015

Item 23
You are caring for a 10-year-old boy who suffered a severe traumatic brain injury from a motor
vehicle accident 3 days ago. He does not have trauma to any other organ system. His elevated
intracranial pressure is currently being managed with mild hyperventilation, mannitol, sedation,
and continuous cerebrospinal fluid drainage. Medications include fentanyl continuous infusion,
cefazolin, ranitidine, and mannitol. He is receiving 30 kcal/oz formula at 50 mL/h by nasogastric
tube.
He has a temperature of 37°C, heart rate of 80 beats/ min, blood pressure of 120/70 mm Hg,
respiratory rate of 20 breaths/min, and oxygen saturation of 98% on the ventilator. His weight is
30 kg. On physical examination, he is intubated and sedated. He does not open his eyes and
withdraws all extremities equally upon painful stimuli. Pupils are 4 mm, equal, and sluggishly
reactive. Mucous membranes are moist, and he has periorbital and soft tissue edema. His
pulmonary and cardiovascular examinations are unremarkable. Over the last 24 hours, fluid
intake was 1,200 mL of feeds and 300 mL of medications and other fluids to maintain catheter
patency. Fluid output includes 500 mL of urine and 100 mL of cerebrospinal fluid drainage.

Serum chemistry results include:


• Sodium, 125 mEq/L (125 mmol/L)
• Potassium, 4 mEq/L (4 mmol/L)
• Chloride, 90 mEq/L (90 mmol/L)
• Carbon dioxide, 24 mEq/L (24 mmol/L)
• Blood urea nitrogen, 20 mg/dL (7.1 mmol/L)
• Creatinine, 0.6 mg/dL (53 umol/L)
• Osmolality, 270 mOsm/kg (270 mmol/kg)
Urine chemistry results include:
• Sodium, 50 mEq/L (50 mmol/L)
• Creatinine, 12 mg/dL (1061 µmol/L)
• Urine urea nitrogen, 6 mg/dL (2.1 mmol/L)
The patient's urine specific gravity is 1.020.

Of the following, the MOST likely physiologic mechanism for his hyponatremia is
A. adrenal insufficiency
B. excessive atrial natriuretic peptide secretion
C. excessive renal sodium loss
D. inadequate sodium intake
E. insufficient renal water excretion

American academy of pediatrics 69


American Academy of Pediatrics PREP 2015

Item 23 Preferred Response: E


The child in the vignette has the syndrome of inappropriate antidiuretic hormone (SIADH)
evidenced by hyponatremia, decreased urine output, a state of euvolemia or hypervolemia, a
fractional excretion of sodium (FENa) of 2%, and history of traumatic brain injury. Syndrome of
inappropriate antidiuretic hormone causes increased water reabsorption from the renal collecting
duct, and therefore decreased water excretion.

Arginine vasopressin (AVP), the antidiuretic hormone (ADH) of humans, is synthesized in the
hypothalamus as a response to high serum osmolality and volume depletion. In addition to other
functions, AVP activates the V2 receptors in the distal nephron, which results in the insertion of
the aquaporin channels in the collecting duct. This renders it more permeable to water, which
leads to more water reabsorption, increased plasma volume, decreased serum osmolality, and
decreased urine volume. Under normal conditions, AVP secretion is inhibited by low serum
osmolality and increased intravascular volume. In SIADH, AVP secretion is not inhibited, and
the patient is unable to excrete water sufficiently. Syndrome of inappropriate antidiuretic
hormone (Item C23A) is usually caused by central nervous system (CNS) disorders, such as
trauma, infection, tumor, or hemorrhage, or pulmonary disorders, such as pneumonia, positive
pressure ventilation, and asthma. Several medications (ie, some sedatives and neuroleptics) can
stimulate ADH release, and others (ie, oxytocin and vasopressin analogs) can potentiate AVP
action.
Item C23A: Causes of Syndrome of Inappropriate Antidiuretic Hormone
 Central Nervous System Disorders
o Infection: meningitis, encephalitis
o Neoplasms
o Vascular abnormalities
o Psychosis
o Hydrocephalus
o Postpituitary surgery
 Pulmonary Disorders
o Pneumonia
o Tuberculosis
o Asthma
o Positive pressure ventilation
o Pneumothorax
 Carcinomas
o Bronchogenic carcinomas
o Oat cell of the lung
o Duodenal
o Pancreatic
o Neuroblastoma
 Medications
o Vincristine
o Intravenous cyclophosphamide
o Carbamazepine
o Serotonin reuptake inhibitors

American academy of pediatrics 70


American Academy of Pediatrics PREP 2015

Reprinted with permission from Moritz ML, Ayus IC.


Disorders in water metabolism in children: hyponatremia and hypernatremia. Pedlar Rev.
2002;23:371

Complications from SIADH generally include adverse effects of hyponatremia and hypo-
osmolality. In a patient with head trauma, intracranial tumor, or meningitis, the neuronal
swelling from hyponatremia can lead to seizures, cerebral edema, and elevated intracranial
pressures, and in some cases, even death.

As described before, SIADH constitutes an inability to excrete water, leading to decreased serum
osmolality. In this setting, a net negative free water balance can be achieved if free water intake
is less than the output (urine output plus insensible water loss), which will raise the osmolality.
Thus, the mainstay of SIADH treatment is fluid restriction to approximately half of usual
maintenance fluid requirements. Furosemide has also been used to facilitate free water excretion.
If mental status deterioration, signs of impending herniation, or seizures occur, the hyponatremia
should be emergently treated with hypertonic (3%) saline. However, because irreversible
neurologic damage can occur with rapid correction of hyponatremia, the rate of correction should
not exceed 10 to 12 mEq/L in a 24-hour period.

Adrenal insufficiency can occur after traumatic brain injury and can cause hyponatremia. Cases
that are significant enough to cause hyponatremia are often associated with hyperkalemia, non-
anion gap metabolic acidosis, hemodynamic instability, or hypoglycemia, none of which was
seen in the patient in the vignette. Excessive renal sodium loss can occur in CNS insults in the
form of cerebral salt wasting. However, urine sodium levels would be higher than seen in the
vignette. Excessive atrial natriuretic peptide secretion does not typically occur in head trauma,
and will also be associated with increased urinary losses of both sodium and water. Decreased
sodium intake is not likely in this boy who was well and taking a normal diet before the head
injury. In addition, derangements in serum sodium levels usually result from imbalances between
intake and output of water, as opposed to intake of sodium (Item C23B).

Item C23B: Disorders of Impaired Renal Water Excretion


 Effective Circulating Volume Depletion
o Gastrointestinal losses: vomiting, diarrhea
o Skin losses: cystic fibrosis
o Renal losses: salt-wasting nephropathy, diuretics, cerebral salt wasting,
hypoaldosteronism
o Edematous states: heart failure, cirrhosis, nephrosis, hypoalbuminemia
 Thiazide Diuretics
 Renal Failure
o Acute
o Chronic
 Non hypovolemic States of Antidiuretic Hormone Excess
o Syndrome of inappropriate secretion of antidiuretic hormone

American academy of pediatrics 71


American Academy of Pediatrics PREP 2015

o Cortisol deficiency
o Hypothyroidism
Reprinted with permission from Moritz ML, Ayus H. Disorders in water metabolism in children:
hyponatremia and hypernatremia. Pediarr Rev. 2002;23:371

Sodium levels and fluid intake and output should be monitored in children with traumatic brain
injury. Symptomatic hyponatremia resulting from SIADH and other conditions can be life-
threatening and should be treated without delay. Once symptoms have resolved, correction
should not exceed 10 to 12 mEq/L per 24 hours. Syndrome of inappropriate antidiuretic hormone
frequently can be treated with fluid restriction and possibly diuretics.

PREP Pearls
• Disorders of sodium are usually from imbalances between intake and output of free
water.
• Syndrome of inappropriate antidiuretic hormone (SIADH) can be commonly seen in
central nervous system disorders (trauma, infection, and tumor) and pulmonary disorders
(pneumonia, asthma, pleural effusion).
• The mainstay of treatment of SIADH is fluid restriction.

American Board of Pediatrics Content Specification(s)


• Recognize the role of head trauma in the development of SIADH

Suggested Reading
• Greenbaum LA. Sodium. In: Kliegman RM, Stanton BE, St. Geme 1W Schor NE,
Behrman RE, eds. Nelson's Textbook of Pediatrics. 19th ed. Philadelphia. PA: Saunders
Elsevier; 2011:212-218.
• john CA, Day MW. Central neurogenic diabetes insipidus, syndrome of inappropriate
secretion of antidiuretic hormone, and cerebral salt-wasting syndrome in traumatic brain
injury. Crit Care Nurse. 20 /2;32(2):e1-7. doi:10.4037/ccri2012904.
• Moritz ML, Ayus IC. Disorders in water metabolism in children: hyponatremia and
hypernatremia. Pediatr Rev. 2002:23(10:371-380. doi:10.1542/pir.23-11-371.

American academy of pediatrics 72


American Academy of Pediatrics PREP 2015

Item 24
You are called to the delivery room to evaluate a newborn who is grunting, floppy, and mottled.
The mother came to the emergency department in active labor with ruptured membranes. She
received her prenatal care elsewhere; her group B Streptococcus status is unknown. She denies
any history of sexually transmitted infections or genital lesions during pregnancy, and she has a
history of anaphylaxis to penicillin. She received 1 dose of vancomycin prior to delivery.
The baby's Apgar scores were 4 at one min and 6 at five min. Gestational age is estimated at 36
weeks.
On physical examination, the vital signs show a temperature of 35.5°C, heart rate of 170
beats/min, and respiratory rate of 60 breaths/min with grunting and retractions.
Respiratory effort is shallow with coarse breath sounds throughout. The heart has a regular
rhythm without murmur, rub, or gallop; the capillary refill is about 3 seconds; and the skin
contains scattered petechiae.
A complete blood cell count includes the following results:
• White blood cell count, 2,700/µL (2.7 x 109/L), with 26% neutrophils, 17% bands, 46%
lymphocytes, 8% monocytes, and 3% metamyelocytes
• Hemoglobin, 15.8 g/dL (158 g/L)
• Hematocrit, 49% (0.49)
• Platelets, 66 x 103/µL (66 x 109/L)
Blood and urine culture specimens are obtained. The newborn is deemed too unstable for a
lumbar puncture at this time.

Of the following, the BEST choice for initial antimicrobial therapy is


A. ampicillin and gentamicin
B. ampicillin and ceftriaxone
C. clindamycin and cefotaxime
D. clindamycin and gentamicin
E. vancomycin and gentamicin

American academy of pediatrics 73


American Academy of Pediatrics PREP 2015

Item 24 S Preferred Response: A


A number of factors in the vignette presented suggest a risk for early-onset sepsis in this
newborn. These include the unknown duration of rupture of membranes, unknown group B
Streptococcus status of the mother, inadequate maternal antibiotic therapy prior to delivery (only
1 dose of a nonpenicillin agent), and the baby's prematurity. In this setting, the baby's marked
distress, poor respiratory effort, poor perfusion, and low white blood cell count with a left shift
support the diagnosis of 'sepsis syndrome" that requires rapid institution of supportive measures
and antimicrobial therapy. Although group B Streptococcus is the leading cause of infection in
this setting, Escherichia coli and other gram-negative organisms must be considered as well.
Although less common, Listeria monocytogenes is another potential pathogen in early-onset
sepsis of the newborn.

Of the regimens listed, ampicillin and gentamicin provide the best coverage for these organisms
with agents that are approved for use in neonates. Ampicillin and ceftriaxone cover the spectrum
as well, but ceftriaxone is not indicated for use in neonates because of its high protein binding
that could displace bilirubin, leading to an increased risk of kernicterus. The other regimens
listed do not provide coverage for Listeria. Clindamycin does not provide coverage for gram-
negative organisms. Vancomycin is indicated for coagulase-negative staphylococcal infection,
which can be a pathogen in neonates, but infection with this organism does not present with early
onset sepsis as seen in the infant in this vignette. Similarly, methicillin-resistant Staphylococcus
aureus infection has been seen in neonates, but is not a likely cause of early onset infection.
Ampicillin and cefotaxime would be another acceptable regimen in this setting, especially if
ampicillin-resistant E coil are prevalent. In view of concerns over development of resistant
bacteria and increased risk of candidiasis with prolonged use of cephalosporins, cefotaxime
might be reserved for those with meningitis because of its excellent central nervous system
penetration.

PREP Pearls
• Ampicillin is effective against Listeria. No cephalosporin is effective against this
organism.
• Early onset sepsis in the neonate is typically caused by group B Streptococcus or enteric
gram-negative rods such as Escherichia coll. Listeria rnonocytogenes is a less common
cause of this syndrome.

American Board of Pediatrics Content Specification(s)


• Plan appropriate antimicrobial therapy for suspected sepsis in the immediate newborn
period

Suggested Reading
• American Academy of Pediatrics. Group B streptococcal infections. In: Pickering LK,
Baker CI, Kimberlin DW, Long 55, eds. Red Book: 2012 Report of the Committee on
Infectious Diseases. 29th ed. Elk Grove Village, IL; American Academy of Pediatrics;
2012:680-685.
• Polio RA, Committee on Fetus and Newborn. Management of neonates with suspected or
proven early-onset bacterial sepsis. Pediatrics. 2012;129(5):1006-1015. doi;
10.15421peds.2012-0541.
American academy of pediatrics 74
American Academy of Pediatrics PREP 2015

Item 25
A 12-year-old girl comes to your office with the chief concern of fever for 1 day. She has had
mild abdominal pain since this morning and 1 episode of nonbilious vomiting 4 hours ago. She
denies any cough, nasal congestion, or painful urination. Her evaluation reveals a temperature of
38.9°C, heart rate of 86 beats/min, respiratory rate of 16 breaths/ min, blood pressure of 98/50
mm Hg, and normal growth parameters. Her physical examination is only significant for mild
discomfort on abdominal palpation in the periumbilical region, with no organomegaly, flank
tenderness, guarding, or rebound tenderness. Her urinalysis demonstrates a specific gravity of
1.025, pH of 6.0, 2+ protein, and no blood, leukocyte esterase, or nitrites.

Of the following, the MOST likely cause of proteinuria in this patient is

A. acute glomerulonephritis
B. chronic renal failure
C. febrile proteinuria
D. orthostatic proteinuria
E. urinary tract infection

American academy of pediatrics 75


American Academy of Pediatrics PREP 2015

Item 25 Preferred Response: C


A urine dipstick result is positive for protein when urine albumin secretion is more than 300
mg/day. Proteinuria can result from a benign cause or significant renal pathology. Therefore, the
approach to proteinuria should limit detailed evaluation to patients suspected of having a renal
injury of clinical significance.

Benign causes of proteinuria include orthostatic proteinuria and conditions associated with
transient proteinuria such as fever, stress, exercise, cold, and abdominal surgery. Urine should be
rechecked after the underlying condition resolves. For the girl in the vignette, fever is the most
likely cause of her proteinuria, and a repeat urinalysis should be performed a few weeks after the
fever subsides. The girl's normal growth parameters are associated with a low risk of underlying
chronic kidney disease. However, a repeat urinalysis is important, as persistent asymptomatic
proteinuria may be associated with underlying chronic kidney disease.

Orthostatic proteinuria (OP) occurs when the patient is active and disappears when the patient is
supine or asleep for at least 2 hours. To confirm OR a first morning urine sample is needed. It is
important that the patient collect the urine sample immediately upon waking, as even a small
amount of activity can lead to proteinuria. A urine protein-creatinine ratio of greater than 0.2 in a
first morning sample is abnormal and indicative of renal pathology requiring evaluation by a
pediatric nephrologist.

Acute glomerulonephritis is characterized by glomerular hematuria (cola-/tea-colored urine),


hypertension and renal failure. Acute glomerulonephritis may be associated with proteinuria;
however, in the absence of hypertension and cola-colored urine, nephritis is unlikely in the
patient described in the vignette.

Urinary tract infection (UTI) is unlikely in this case with the absence of dysuria, flank pain, or a
burning sensation on micturition. The absence of leukocyte esterase, nitrites and bacteria on the
urinalysis rule out UTI as the underlying cause of proteinuria in this patient.

PREP Pearls
• Benign causes of proteinuria include orthostatic proteinuria and conditions associated
with transient proteinuria such as fever, stress, exercise, cold, and abdominal surgery.
• When transient proteinuria is suspected, a urinalysis should be performed after the
inciting condition resolves.
• Persistent asymptomatic proteinuria may be associated with underlying chronic kidney
disease.

American Board of Pediatrics Content Specification(s)


• Identify the possible causes of proteinuria

Suggested Reading
• Feld LG, Schoeneman MI, Kaskel FJ. Evaluation of a child with asymptomatic
proteinuria. Pediatr Rey. 1984;5(8):248.254. doi:10.1542/ pir.5-8-248.
• Hogg Rj, Portman Rj, Milliner D, Lemley KV, Eddy A, Ingelfinger I. Evaluation and
management of proteinuria and nephrotic syndrome in children: recommendations from a
American academy of pediatrics 76
American Academy of Pediatrics PREP 2015

pediatric nephrology panel established at the National Kidney Foundation conference on


proteinuria, albuminuria, risk, assessment, detection, and elimination (PARADE).
Pediatrics. 2000;105(6):1242-1249_
• Sebestyen JF, Alon US. The teenager with asymptomatic proteinuria: think orthostatic
first. Clin Pediatr. 2011;50(3):179-182. doi:10.1177/0009922810380904.

American academy of pediatrics 77


American Academy of Pediatrics PREP 2015

Item 26
You are asked to recommend nutrition and fluid management for an 8-year-old, 30-kg boy who
has just undergone an exploratory laparotomy for a ruptured appendiceal abscess.

Of the following, based on the child's clinical status, you are MOST likely to recommend
A. central venous line placement for infusion of dextrose and amino acids
B. intravenous fluids by peripheral infusion with 5% dextrose and electrolytes
C. nasogastric tube insertion for fluid administration 24 hours postoperatively
D. oral glucose and electrolyte solution for the first 3 days postoperatively
E. peripheral venous alimentation with dextrose and amino acids

American academy of pediatrics 78


American Academy of Pediatrics PREP 2015

Item 26 Preferred Response: E


Decisions regarding nutrition support of the hospitalized patient must be based on the answers to
3 critical questions:
1. Can the gastrointestinal (GI) tract be used, either partially or totally?
2. If parenteral nutrition is required, for how long will it be used?
3. Based upon the answers to questions 1 and 2, how may the patient's nutrient and fluid
requirements best be met?
Whenever possible, the GI tract should be used to deliver nutrients, even if only a portion of total
daily requirements for energy and protein are met. Infants deprived of oral feeding stimulation
for long periods may lose the desire or the ability to feed. Intraluminal nutrients are essential for
maintaining intestinal function. Prolonged absence of oral or enteral feedings also may
compromise the mucosal barrier and increase the risk for bacteria and bacterial products in the
intestine entering the blood.

The child described in the vignette, who has undergone surgery for an appendiceal abscess, will
likely be taking nothing by mouth (nil per os [NPO]) and require parenteral fluids and nutrition
for 5 to 7 days. The parenteral route of administration is chosen because of the expected time
during which gut motility will be impaired postoperatively. Because the parenteral infusion is
estimated to be required for 7 or fewer days, a peripheral vein is the indicated route of
administration. Peripheral intravenous nutrition comprising 10% dextrose, 1.5% amino acids,
and 10 g/day lipid is best for supporting this child's basal energy and protein needs and
ameliorating the potential catabolic state during recovery. Any fluid regimen that fails to provide
a source of protein will be unable to minimize endogenous protein catabolism and may also
delay optimal wound healing.

Parenteral nutrition can be a lifesaving intervention, providing essential fluids and nutrients for
the patient in whom the GI tract cannot be used fully to meet nutritional needs for extended
periods. Parenteral nutrition administered via central venous catheter generally should be
reserved for 2 situations. The first is when no oral or enteral nutrition may be given for more
than 7 days because of either medical or postsurgical conditions. The second is when partial oral
or enteral feedings, in conjunction with peripheral intravenous nutrition, either cannot meet
nutrient needs or will be required for extended periods, thus presenting problems of prolonged
peripheral venous access. Central venous support also may be required when venous access
problems make peripheral infusions difficult to maintain, either because of patient age or clinical
condition. For the patient receiving total intravenous nutrition via central venous catheter,
dextrose concentrations generally are in the range of 20%. However, solutions containing more
than 12.5% dextrose are highly sclerosing to small veins and should be avoided in all peripheral
infusions. In most cases, a 10% dextrose solution is used for peripheral vein infusions.

Daily fluid and macronutrient requirements for a child weighing 20 kg or more who is receiving
parenteral nutrition in the hospital include the following:
• Fluid = 1,500 mL + 20 mL/kg for each kg body weight greater than 20 kg
• Carbohydrate calories = 30 to 75 kcal/kg as dextrose (3.4 kcal/g), or =10 to 20
g/kg
• Lipid calories = 1 to 3 g/kg
• Protein (as amino acids) = 1 to 2 g/kg
American academy of pediatrics 79
American Academy of Pediatrics PREP 2015

For the 30-kg boy described in the vignette, total fluid volume initially should be in the range of
1,700 mL per 24 hours. An infusate containing 10% dextrose provides 170 g of dextrose to this
child, which is equal to 578 kcal or approximately 25 kcal/kg per day. This amount of energy is
less than calculated requirements and reflects the limits posed by infusing a 10% dextrose
solution. However, based on patient tolerance, the infusion rate may be increased gradually to
enhance energy intake. A 1.5% amino acid infusion administered at the stated rate should
provide approximately 1 g/kg protein. Adding significantly greater amounts to the initial
infusate (eg, 3.0% amino acids) may result in excessive protein consumption if the rate is
increased. Excessive protein intake can potentially increase the risk for hyperammonemia and
azotemia because of protein catabolism.

Although not included in the answer choices, current standard practice in cases such as the one in
the vignette will use the added infusion of a lipid emulsion. Lipid infusions always should
commence at a lower-than-targeted amount (generally 1 to 3 g/kg/day) to allow for metabolic
adaptation to intravenous fat and to prevent hypertriglyceridemia. Indeed, for short-term
administration, 0.5 g/kg per day of intravenous lipids should be sufficient to prevent even a
relative deficiency of essential fatty acids. The decision to increase lipids beyond this level
depends on several factors, including the need to increase energy intake in a patient who has
compromised nutritional status and any fluid volume limitations.

PREP Pearls
• Provision of nutrients via the gastrointestinal tract, to the extent tolerated, should always
be attempted in hospitalized patients.
• In general, dextrose concentrations infused via a peripheral venous catheter should not
exceed 10%.
• When calculating energy intake for patients receiving parenteral nutrition, only dextrose
and lipid calories are included, since protein (amino acid) intake should be used solely for
anabolic purposes.

American Board of Pediatrics Content Specification(s)


• Know the indications for total and peripheral alimentation

Suggested Reading
• American Academy of Pediatrics. Parental nutrition. In: Kleinman RE, Pediatric
Nutrition Handbook. 6th ed. Elk Grove Village, III: American Academy of Pediatrics;
2009:519-540.
• Culebras JM, Martin-Pena G, Garcia-de-Lorenzo A, et aL Practical aspects of peripheral
parenteral nutrition. Curr Opin Clin Nutr Metab Care. 2004;7(3):303-307.
• Shulman RJ, Phillips S. Parenteral nutrition indications, administration, and monitoring.
In: Baker 55, Baker RD, Davis AM, eds. Pediatric Nutrition Support. Sudbury, MA:
Jones and Bartlett Publishers; 2006:273-286.

American academy of pediatrics 80


American Academy of Pediatrics PREP 2015

Item 27
An 8-year-old girl is brought to your office because of the mother's concerns for her poor
performance in school, attention difficulties, difficulty with speech and word pronunciation, and
anxiety tendencies. The mother reports that her daughter had decreased muscle tone, slow
development as an infant, and required therapy for motor and speech delay. She has a history of
well-controlled absence seizures for which she takes an antiepileptic medication daily. She
currently is in a mainstream classroom with an Individualized Education Plan in place for her
learning difficulties in language processing and comprehension. Her full scale IQ is 75 with a
verbal IQ of 70.
Birth history is essentially unremarkable, with the exception of the mother's advanced maternal
age of 45 years at the time of delivery.
The family history is negative for Parkinson disease, premature ovarian failure, intellectual
disability, early sudden death, recurrent miscarriages, and other birth defects. Her 2 brothers and
parents have IQs within reference range, and both parents have normal stature.
Physical examination shows her height to be at the 90th percentile for age. She has mild
hypotonia, epicanthal folds, clinodactyly, and long limbs. The remainder of the examination is
unremarkable.

Of the following, the MOST appropriate initial diagnostic test is

A. chromosomal microarray
B. fragile X testing
C. karyotype
D. serum amino acids
E. urine organic acids

American academy of pediatrics 81


American Academy of Pediatrics PREP 2015

Item 27 Preferred Response: C


When considering the genetic workup of a patient presenting like the girl in the vignette, with
developmental delays, intellectual deficits, and neurobehavioral problems, the optimal clinical
genetics diagnostic evaluation consists of a clinical history, 3-generation family history,
dysmorphologic examination, neurologic examination, and high-resolution karyotyping. If the
initial high-resolution karyotype is non-diagnostic, subsequent testing can include a
chromosomal microarray, fragile X molecular genetic testing, and consideration of brain
imaging. If these studies do not reveal a diagnostic etiology, then fluorescence in situ
hybridization studies for subtelomere chromosome rearrangements, targeted blood and urine
screening for metabolic disorders, and specific molecular testing for a suspected syndrome
should be considered, typically in the context of a genetics referral or consultation. In patients
with developmental or intellectual delays, high-resolution karyotyping can detect subtle
chromosomal anomalies from 9% to 36% of the time, thus leading to its diagnostic importance.
A child does not have to be dysmorphic to recommend a karyotype.

An initial high-resolution karyotype with a subsequent chromosomal microarray, if the karyotype


is negative, is the most cost-effective sequence of testing. The American College of Medical
Genetics and the Academy of Neurology recommend that physicians include fragile X testing in
both male and female patients with unexplained mental retardation, especially in the presence of
a positive family history, which is lacking in this case, and the absence of major anomalies.

Close attention should be given to signs of a metabolic disorder including growth failure,
recurrent unexplained illness, seizures, ataxia, regression, hypotonia, "coarse" appearance, eye
abnormalities, recurrent somnolence, abnormal sexual differentiation, hepatosplenomegaly,
metabolic acidosis, hyperammonemia, structural hair abnormalities, and bone and skin
abnormalities. If any of these signs are present, a screening metabolic workup should be initiated
or the child should be referred to a specialist.

A comprehensive dysmorphologic examination should be done to look for any major or minor
anomalies. A 3-generation pedigree may also give clues as to the specific inheritance pattern that
could be involved in the patient's clinical diagnosis.

The child's neurodevelopmental status should be addressed at the same time as the genetics
diagnostic evaluation. In addition, neurologic, audiologic, and ophthalmologic evaluations may
be potentially needed. Therapies also should be initiated as soon as possible.

For the patient in this vignette, it is important to note the lack of history in relatives for
Parkinsonian-like disease (fragile X tremor ataxia syndrome), premature ovarian failure, and
intellectual disability, which can commonly be found in family members who are premutation
and full mutation carriers for fragile X syndrome.

High-resolution karyotype would have revealed the diagnosis in the child in the vignette to be
47,XXX syndrome, otherwise known as trisomy X. This is a sex chromosome anomaly that
presents with a variable phenotype affecting 1 in 1,000 female births. It is the most common
female chromosomal abnormality. Only 10% of individuals with trisomy X are clinically
diagnosed. Most are only mildly affected or asymptomatic. Clinical features are subtle and
American academy of pediatrics 82
American Academy of Pediatrics PREP 2015

include hypertelorism, epicanthal folds, hypotonia, tall stature, clinodactyly, motor and speech
delays, and an increased risk of cognitive deficits or learning disabilities. Psychological features
are more prevalent than in the general population and include mood disorders, attention-
deficit/hyperactivity disorder, and other behavioral difficulties. Premature ovarian failure is also
a common finding, though most begin puberty at the expected age. It results most commonly
from meiotic nondisjunction, with risk increasing with advanced maternal age. Twenty percent
of cases result from postzygotic nondisjunction.

Another common sex chromosomal abnormality is 47,XYY syndrome. 47,XYY syndrome is a


frequently under-diagnosed but common sex chromosomal anomaly. Its frequency is 1 in 1,000
males. Klinefelter syndrome (47,XXY) occurs with a frequency of 1 in 500 males. 47,XYY
patients typically resemble those with Klinefelter syndrome with respect to the tall stature, verbal
learning disabilities, delayed speech, and attentional difficulties; however, those with 46,XYY
syndrome have normal pubertal development and testosterone levels, whereas males with
Klinefelter syndrome frequently experience childhood-onset testicular failure. Boys with
Klinefelter syndrome tend to be more impulsive and have behavioral difficulties. They often
require special education services, speech therapy, and occupational and physical therapy. They
have increased features of autism spectrum disorder because of their social deficits and language
disorders.

PREP Pearls
• In a patient presenting with developmental delays and learning disabilities, high-
resolution karyotype is the most appropriate first genetic test to order. Most laboratories
will perform the high-resolution karyotype first, then do a chromosomal microarray if the
karyotype is negative in a tiered panel.
• Trisomy X features include hypertelorism, epicanthal folds, hypotonia, tall stature,
clinodactyly, motor and speech delays, and an increased risk of cognitive deficits or
learning disabilities in association with attention-deficit disorder/mood disorder.
• 47,XYY patients typically resemble those with Klinefelter syndrome (47,XXY) with
respect to the tall stature, verbal learning disabilities, delayed speech, and attentional
difficulties; however patients with 47,XYY have normal pubertal development and
testosterone levels, whereas males with Klinefelter syndrome frequently experience
childhood-onset testicular failure.

American Board of Pediatrics Content Specification(s)


• 47,XYY Recognize the clinical features associated with a 47,XYY chromosome
abnormality
• 47,XXX Recognize the clinical features associated with a 47,XXX chromosome

Suggested Reading
• Leiser K, Powell BR, Wilks TM, et al Clinical utility of chromosomal microarray
analysis. Pediatrics. 2012;130(5);e1085. doi:10.1542/peds. 2012-0568.
• Miller DT, Adam MP, Aradhya S, et al. Consensus statement: chromosomal microarray
is a first-tier clinical diagnostic test for individuals with developmental disabilities or
congenital anomalies. Am J Hum Genet. 2010;86(5):749-764.
doi:10.1016/j.ajhg.2010.04.006.
American academy of pediatrics 83
American Academy of Pediatrics PREP 2015

• Moeschler JB, Shevell M; American Academy of Pediatrics, Committee on Genetics.


Clinical evaluation of the child with mental retardation or developmental delays.
Pediatrics. 2006;117(6):2304-2316. doi:10.1542/ peds.2006-1006.
• Ross JL, Roeltgen DP, Kushner H, et al. Behavioral and social phenotypes in boys with
47,XYY syndrome or 47,XXY Klinefelter syndrome. Pediatrics. 2012;129(4):769-778.
doi:10.1542/peds.2011-0719.
• Tartaglia NR, Howell S, Sutherland A, Wilson R, Wilson L. A review of trisomy X
(47,XXX). Orphanet J Rare Dis. 2010;5:8. doi:10.1186/1750- 1172-5-8.

American academy of pediatrics 84


American Academy of Pediatrics PREP 2015

Item 28
The parents of a healthy 2-month-old female infant come to your office because of concerns
about a red rash in her axilla and neckline. This rash has been present for 2 weeks and seems to
be getting worse. She is primarily breastfed and she has no constipation or diarrhea. She has been
otherwise well. They use baby skin care products only. On physical examination, you find a rash
as shown (Item Q28). The remainder of the physical examination is unremarkable.

Of the following, the BEST initial plan for management is to recommend


A. avoidance of milk in mother's diet
B. evaluation by dermatology
C. topical antifungal cream
D. topical corticosteroid
E. topical zinc oxide

American academy of pediatrics 85


American Academy of Pediatrics PREP 2015

Item 28 Preferred Response: D


The female infant in the vignette has seborrheic dermatitis. Seborrheic dermatitis is a common,
self-limited condition in infancy that is usually asymptomatic. It is characterized by greasy
yellowish scales in the scalp and eyebrows, and as in this case, a fine erythematous papular
eruption of the face, postauricular region and ears, nape of the neck, and intertriginous areas.
Often cheesy debris is evident along with erythema in the skin folds of the axilla, anterior
neckline, and groin. The location parallels the distribution and density of the sebaceous glands,
but the pathogenesis is not known.

Cradle cap is the common term for seborrheic dermatitis of the scalp in young infants. It may be
focal or diffuse on the scalp and may spread to the forehead and eyebrows. The disorder usually
begins in the first month after birth. It is usually nonpruritic and responds well to low-potency
topical corticosteroids on the skin and postauricular areas. Seborrheic dermatitis of the scalp can
be controlled usually by regular shampooing and increased efforts to brush away the softened
scale. Some believe that applying oil or emollients to the scalp for 30 min before shampooing
improves the ability to brush the scale away. If the scaling of the scalp is thicker or more
recurrent, one may recommend using a mild antiseborrheic shampoo 2 or 3 times per week.

In general, the prevalence of seborrheic dermatitis peaks at about 3 months of age and decreases
steadily thereafter. It usually resolves before 1 year of age and may return in adolescence as
dandruff or chronic blepharitis.

Typical infantile seborrheic dermatitis usually improves after treatment with a low-potency
topical corticosteroid; if not, a short course of medium potency topical corticosteroids may be
implemented. If the condition does not resolve promptly, it is important to consider other causes,
such as Langerhans cell histiocytosis, immunodeficiency, zinc deficiency, coexistent atopic
dermatitis, secondary bacterial suprainfection, psoriasis, or superimposed candidiasis. Directed
therapy for these other conditions, such as topical antifungal agents, topical zinc oxide,
antibiotics, or topical immunomodulatory agents, should be initiated as the next step in
management when indicated by the history and physical examination. It is not necessary for the
mother to avoid milk for routine seborrheic dermatitis, but this may be tried in cases of severe
infantile eczema when milk allergy is suspected. Dermatology referral is necessary for extreme
or refractory cases.

PREP Pearls
• Typically, infantile seborrheic dermatitis responds promptly to low-potency topical
corticosteroids.
• The common distribution for infantile seborrheic dermatitis involves the scalp, eyebrows,
face, postauricular region, nape of the neck, and intertriginous areas.

American Board of Pediatrics Content Specification(s)


• Recognize the clinical findings associated with seborrheic dermatitis, and manage
appropriately

American academy of pediatrics 86


American Academy of Pediatrics PREP 2015

Suggested Reading
• Morelli JG. Seborrheic dermatitis. In: Kliegman RM, Stanton BF, St Geme 1W III, Schor
NF, Behrman RE, eds. Nelson Textbook of Pediatrics. 19th ed. Philadelphia, PA:
Saunders Elsevier; 2011:2253-2254.
• Paller AS, Mancini AI, eds. Cutaneous disorders of the newborn. Hurwitz Clinical
Pediatric Dermatology: A Textbook of Skin Disorders of Childhood and Adolescence.
4th ed. St. Louis, MO: Saunders Elsevier; 2011:21-22.
• Sasseville D. Cradle cap and seborrheic dermatitis in infants. UpToDate. Available online
only for subscription.
• Williams ML. Differential diagnosis of seborrheic dermatitis. Pediatr Rev.
1986;7(7):204-211. doi:10.1542/pir.7-7-204.

American academy of pediatrics 87


American Academy of Pediatrics PREP 2015

Item 29
A 3-year-old girl presents to your clinic for pallor. The mother reports that for 2 days last week
her daughter had a fever and cough, which have now resolved. The child is now tolerating her
usual diet, which includes 32 to 40 oz of cow's milk per day. The mother states the child has
looked pale over the last several days. Her temperature is 37.0°C, pulse rate is 90 beats/min,
respiratory rate is 24 breaths/min, and blood pressure is 100/60 mm Hg. She is at the 95th
percentile for weight and 50th percentile for height. On examination, the child is alert, playful,
and in no apparent distress. She has anicteric sclerae and no hepatosplenomegaly or
lymphadenopathy. The remainder of the physical examination is unremarkable. The following
are the results of the child's laboratory tests:

Complete blood work:


• White blood cell count, 7,500/µL (7.5 x 109/L), with normal differential
• Hemoglobin, 6.0 g/dL (60 g/L)
• Mean corpuscular volume, 59 µm3 (59 fL)
• Platelet count, 495,000 x 103/L (495 x 109/L)
• Reticulocyte count, 0.5%

Of the following, the MOST appropriate next step in management is to


A. infuse intravenous iron dextran
B. reassure that the anemia is postinfectious
C. start oral ferrous sulfate
D. start oral prednisone
E. transfuse packed red blood cells

American academy of pediatrics 88


American Academy of Pediatrics PREP 2015

Item 29 Preferred Response: C


The child in the vignette has microcytic anemia consistent with iron deficiency and should be
started on oral iron replacement. Worldwide, iron deficiency anemia is the most common cause
of anemia, accounting for 50% of anemia in children ages 0 to 5 years. The highest prevalence
occurs between 2 and 5 years of age. Data suggest that untreated iron deficiency, with or without
anemia, may have an irreversible detrimental effect on neurocognitive development. In an
otherwise hemodynamically stable patient, such as the child in the vignette, the mainstay of
treatment for iron deficiency anemia is oral iron supplementation. Most patients who become
iron deficient because of dietary issues (eg, excessive cow's milk intake, lack of iron-rich foods)
gradually become anemic, and therefore their bodies have compensated over time by increasing
plasma volume.

In evaluating for the cause of anemia in a child, it can be useful to categorize the differential
diagnoses based on the red blood cell (RBC) size. Anemia with a low mean corpuscular volume
(MCV), that is, microcytic anemia, can be seen in iron deficiency anemia, lead poisoning,
thalassemia, sideroblastic anemia, or chronic inflammation. Anemia with a normal MCV can be
seen in hemolytic anemia, acute blood loss, or chronic renal disease. Anemia with an increased
MCV is often seen in vitamin B12 deficiency, folic acid deficiency, aplastic anemia, Diamond-
Blackfan anemia (pure RBC aplasia), hypothyroidism, liver disease, or bone marrow infiltration.

Excessive ingestion of cow's milk, which contains only a minimal amount of iron, can lead to
iron deficiency by decreasing the absorption from other foods and by causing small amounts of
gastrointestinal blood loss. Given this patient's age and history of excessive milk intake, the
microcytic anemia is most likely caused by iron deficiency. The patient should be prescribed a
course of oral iron at a dose of 3 to 6 mg/kg of elemental iron per day. This should be taken on
an empty stomach, if possible, to increase absorption. Iron absorption is also enhanced by an
acidic environment; therefore patients should be instructed to take their iron doses with orange
juice or lemonade. Concomitant use of prescription or over-the-counter antacid therapies may
impair absorption of iron. Combination tablets with iron and ascorbic acid are also available,
though these are more expensive. Oral iron replacement is available as iron salts such as ferrous
sulfate and ferrous gluconate, or as a polysaccharide iron complex. These formulations may vary
in cost, available dose (tablet) sizes, and side effects (most commonly gastrointestinal
discomfort). Dosing should be based on the amount of elemental iron in each iron formulation.
Even with good compliance, patients may require 3 to 6 months of oral iron replacement to treat
severe iron deficiency and replenish the body's iron stores.

The indications for parenteral iron include inability to tolerate oral iron, inability to absorb iron
in the gastrointestinal tract, or the need for rapid iron replacement. The patient in the vignette
does not meet these criteria. Furthermore, parenteral iron has been associated with anaphylaxis
and therefore patients must be monitored very closely when receiving this infusion. Infection can
cause various types of anemia, but none are usually microcytic. Autoimmune hemolytic anemia,
transient erythroblastopenia of childhood, and bone marrow suppression are examples of anemia
that are normocytic or macrocytic in most cases. Oral prednisone would be a reasonable
treatment for autoimmune hemolytic anemia, but the lack of reticulocytosis and jaundice would
make this diagnosis less likely. Transfusion of packed RBCs would be warranted if the patient
was unstable (tachycardia, lethargy, respiratory compromise). Given the history and appearance
American academy of pediatrics 89
American Academy of Pediatrics PREP 2015

of this patient, replacement of iron orally would be a better option. If the patient does require a
packed RBC transfusion, it should be done cautiously in small aliquots of 5 mL/kg over 3 to 4
hours, so as not to overload the patient who may already have a compensatory increase in plasma
volume.

PREP Pearls
• Iron deficiency is the most common cause of anemia worldwide in children younger than
5 years of age, with the highest prevalence in children 2 to 5 years of age.
• Treatment of iron deficiency anemia is oral iron supplementation at a dose of 3 to 6
mg/kg per day of elemental iron. It may take 3 to 6 months to replenish iron stores.
• Gastrointestinal iron absorption is enhanced by ingestion on an empty stomach,
accompanied by ascorbic acid. Concomitant use of prescription or over-the-counter
antacid therapies may impair absorption of Iron.

American Board of Pediatrics Content Specification(s)


• Plan the appropriate management of severe anemia of various etiologies, while
considering the risks associated with various therapies

Suggested Reading
• Janus 1, Moerschel SK. Evaluation of anemia in children. Am Fam Physician.
2010;81(12):1462-1471
• Thompson I, Biggs BA, Pasricha SR. Effects of daily iron supplementation in 2- to 5-
year-old children: systematic review and meta-analysis. Pediatrics. 2013;131(4):739 -
753. doi:10.1542/peds.2012-2256

American academy of pediatrics 90


American Academy of Pediatrics PREP 2015

Item 30
You are seeing a 15-year-old adolescent boy who has recently developed some academic
difficulty. He is getting good grades in math and biology at school, but has just started to have
significant struggles with English and social studies. The school difficulties started when
expectations in those classes moved toward reflection and creative thinking, rather than just
memorizing and following rules. When he is given something truly novel to work on, he has
difficulty coming up with strategies to address the new problem. You note from his history that
he appears to have a personal sense of invincibility, in that he does not hesitate to take significant
physical risks while playing sports or skateboarding.

Of the following, the predominant thought process that BEST describes what he is exhibiting is
A. abstract reasoning
B. concrete thinking
C. feelings of guilt
D. inferiority thoughts
E. unconscious drives

American academy of pediatrics 91


American Academy of Pediatrics PREP 2015

Item 30 Preferred Response: B


At the beginning of adolescence, children's cognitive abilities tend to be largely "concrete'
Concrete thinking involves applying logic to interpret things objectively rather than intuitively.
In concrete thinking one follows and manipulates rules, and memorizes and manipulates stored
information. Concrete reasoning typically involves manipulating ideas that could be represented
as tangible objects. Concrete thinking often involves a single level of situational analysis based
on appearances, while missing entirely that there are other ways of viewing or analyzing a
problem.

Abstract thinking is the next stage of thought development, in which one uses hypothetical ideas
more readily in reasoning. With abstract thinking one is able to consider several possibilities and
outcomes at once, and can understand and manipulate concepts as relative rather than absolute.
These are important skills in learning to solve problems in novel situations.

The 15-year-old boy in the vignette is likely developing academic difficulties in part because he
has poorly developed abstract reasoning abilities. Biology and Math focus on memorization of
facts and rules, whereas social studies and English high school courses require more concept
reflection than fact memorization. A personal sense of invincibility is not necessarily related to
concrete thinking. However, those who have well-developed abstract reasoning and reflection
skills tend to better appreciate that although they feel healthy now taking a significant physical
risk while doing something like skateboarding could create lasting negative consequences. In
working with a patient who is still using concrete thinking, healthcare providers should avoid
using strategies that require the patient to manipulate abstract concepts, such as weighing risks
and benefits of behavior change, in the manner that an adult would.

"Inferiority thoughts" refers to the Erik Erikson developmental stage of "industry versus
inferiority:' typically seen in 6 to 11 year olds. Those who struggle to develop a sense of
competence in school and social interactions during play will doubt their ability to become
successful and may socially withdraw. This young man is 15 years old, and we are not told that
he is struggling with classwork because he doubts his ability to be successful.

"Feelings of guilt" refers to the earlier Erikson stage of "initiative versus guilt; typically seen
during the preschool years. Children at this stage of development are learning to assert
themselves as individuals through play and other social interactions. Those who struggle in this
arena may feel ashamed of themselves, and become overly dependent on others.

"Unconscious drives" refers to the Freudian theory of the unconscious mind, in which
individuals make choices in their lives based on certain needs. They would only become aware
of these needs if they spent a significant amount of time reflecting on why they did something
that does not on its surface make sense. It is difficult to assert that this young man's school
performance difficulties are the result of sexual or oral sublimated unconscious urges.

American academy of pediatrics 92


American Academy of Pediatrics PREP 2015

PREP Pearls
• Adolescent psychological development is marked by a transition from concrete thinking
to abstract thinking.
• Changes in academic demands requiring more abstract manipulation of concepts can lead
some adolescents to suddenly start to struggle in their academics.

American Board of Pediatrics Content Specification(s)


• Understand the timing of and factors influencing the development of concrete thinking
and abstract reasoning in adolescents, and provide health advice accordingly

Suggested Reading
• Goldman L, Schafer Ai, ed. Goldman's Cecil Medicine. 24th ed. Philadelphia, PA: WB
Saunders; 2011.
• Hazen E, Schlozman 5, Beresin E. Adolescent psychological development: a review.
Pediatr Rev. 2008;29:161-168. doi:10.1542/pir.29-5 -161.

American academy of pediatrics 93


American Academy of Pediatrics PREP 2015

Item 31
An 8-year-old boy is seen for a health supervision visit. There is a family history of
hypertension. You have been monitoring his blood pressure (BP) annually for the last 3 years.
Last year, his BP was 90/65 mm Hg when taken in your office. He is active in sports without any
symptoms of fatigue. He has been consistently in the 50th percentile for weight. His BP is
recorded at 125/80 mm Hg today at check-in. You would like to assess whether his blood
pressure was taken optimally and if it is abnormal. You ask the staff to take the BP with the
optimal size cuff.

Of the following, true hypertension in this child would MOST likely be present if readings are:

A. 125/75 mm Hg with a blood pressure cuff bladder that is 80% of the length and 40% of
the circumference of the upper arm
B. 125/75 mm Hg with a blood pressure cuff bladder that is 100% the length of the upper
arm and 50% of the circumference
C. 140/80 mm Hg and the blood pressure cuff bladder is 50% of the length of the upper arm
and 80% of the circumference
D. 150/80 mm Hg and the blood pressure cuff bladder is equal in length and width
E. 180/90 mm Hg with a blood pressure cuff bladder completely encircling the lower leg

American academy of pediatrics 94


American Academy of Pediatrics PREP 2015

Item 31 Preferred Response: A


Early recognition of hypertension is important to mitigate risk factors for adult cardiovascular
disease and allow for early evaluation and treatment. The range of normal blood pressure (BP)
changes with age and gender, and can be found at
http://www.nhlbi.nih.gov/guidelines/hypertension/child_tbl. htm. The threshold for hypertension
has decreased. Accurate BP readings are imperative if appropriate referrals and treatment plans
are to be made. Blood pressure should be checked by 3 years of age in children with no risk
factors, and sooner in patients at higher risk of hypertension, such as those with kidney disease.
Children with obesity are more likely to have hypertension that may require ambulatory
monitoring to confirm. If left untreated, hypertension can cause left ventricular hypertrophy,
especially in obese patients.

The need for accurate BP measurement makes the choice of BP cuff even more important; the
choice is dependent on the relationship of the cuff bladder to the length and circumference of the
patient's upper arm. A cuff that is too small will falsely elevate the BP; a cuff that is too large to
close around the arm will not be accurate (Item 31A).

Approximate Ages at Which the Available Cuffs Would Cover 40% of the Mid upper Arm
Circumference (UAC)
Available BP Cuffs UAC (cm) Needed for Corresponding Age (y) for Mean UAC
(Median Width in the BP Cuff to Equal Male Female
Centimeters) 40%
Infant (6) 15 <1 <1
Child/pediatric (8.25) 20.6 10 10
Small adult (11) 27.5 16 25
Adult (14) 35 * *
Large adult (17) 42.5 ** **
*Maximum UAC for an average-size adult male and female are 32.6 cm and 30.3 cm,
respectively.
**Above the 95th percentile for any age group.

According to the 1987 Report of the Task Force on Blood Pressure Control in Children and its
update by the National High Blood Pressure Education Program Working Group on
Hypertension Control in Children and Adolescents, the BP cuff bladder should be wide enough
to cover 3 quarters of the upper arm length from the acromion to the olecranon. In its update in
1996, the working group recommended that the width of the cuff bladder should equal 40% of
the mid-upper arm circumference (Item 31B,). Therefore, for the boy in this vignette, a BP of
125/75 mm Hg with a BP cuff bladder that is 80% of the length and 40% of the circumference of
the upper arm at the midpoint is the correct answer. The other size BP cuffs would not lead to
accurate results and might prompt inappropriate referrals.

American academy of pediatrics 95


American Academy of Pediatrics PREP 2015

PREP Pearls
• Norms exist for blood pressure (BP) by age and gender and should be referred to when
evaluating BP in children.
• Early referral to a pediatric nephrologist or cardiologist is important for evaluation and
early treatment.
• A proper BP cuff bladder that is 80% of the length and 40% of the circumference of the
upper arm at the midpoint is key in accurate BP measurement.

American Board of Pediatrics Content Specification(s)


• Understand when to screen for an increased blood pressure and how to interpret the
results
• Understand the appropriate technique, including appropriate cuff size, for measuring
blood pressure

Suggested Reading
• Abuilar A, Ostrow B, De Luca F, Suarez E. Elevated ambulatory blood pressure in a
multiethnic population of obese children and adults. J Pediatr. 2010;156(6):930.935. doi:
10.1016/j.jpeds.2009.12.028.
• Arafat M, Mattoo TK. Measurement of blood pressure in children: recommendations and
perceptions on cuff selection. Pediatrics. 1999;104(3):e30.
• Maggio AB, Aggoun Y, Marchand LM, et al. Associations among obesity, BP and LV
mass. J Pediatr. 2008;152(4):489-493. doi:10.1016/jjpeds.2007.10.042.
• National High Blood Pressure Education Program Working Group on Hypertension
Control in Children and Adolescents. Update on the 1987 Task Force Report on High
Blood Pressure in Children and Adolescents: a working group report from the National
High Blood Pressure Education Program. Pediatrics. 1996;98(4):649-658.
• National High Blood Pressure Education Program Working Group on High Blood
Pressure in Children and Adolescents. The fourth report on diagnosis, evaluation and
treatment of high blood pressure in children and adolescents. Pediatrics. 2004;114(Suppl
21:555-576.
• National High Blood Pressure Education Program Working Group on High Blood
Pressure in Children and Adolescents, US Department of Health and Human Services. A
pocket guide to blood pressure measurement in children. US Department of Health and
Human Services website.
• Park MK. Blood pressure tables. Pediatrics. 2005;114(3):826-827. doi:

American academy of pediatrics 96


American Academy of Pediatrics PREP 2015

Item 32
You are seeing a 12-year-old girl in your office for complaints of fatigue and arm pain that has
lasted 2 weeks. The girl states that she feels fatigued after activity and that certain activities are
difficult because of pain and weakness. She has the most difficulty with fixing her hair and
walking up stairs. On physical examination, she has decreased strength. She is able to overcome
gravity, but not resistance in her shoulder girdle and hip flexors. She has erythematous patches
over her metacarpophalangeal joints and proximal interphalangeal joints bilaterally (Item Q32).
You also note a mild violaceous color over her eyelids bilaterally.

Of the following, the BEST next step in the evaluation of this patient is
A. creatine phosphokinase level
B. electromyography
C. magnetic resonance image of the spine
D. neurology referral
E. reassurance and symptomatic pain treatment

American academy of pediatrics 97


American Academy of Pediatrics PREP 2015

Item 32 Preferred Response: A


The best next step for the patient in this vignette would be to obtain a creatine phosphokinase
level, looking for evidence of muscle tissue break down caused by inflammation. Although
elevation of creatine phosphokinase is a nonspecific marker, elevation suggests a primary muscle
problem and can guide further workup. The girl in the vignette has rash, muscle pain, and
evidence of muscle weakness. These physical findings are consistent with juvenile
dermatomyositis.

The differential diagnosis for muscle weakness and pain is broad; therefore clues identified in the
history and physical examination are important to guide an appropriate workup for the patient.

Weakness often presents as the inability to perform certain activities, and the activities affected
can be helpful in directing the diagnostic workup. Proximal muscle weakness, such as difficulty
climbing stairs or raising the arms above the head, suggests a primary muscle disease. Muscle
enlargement, muscle wasting, and fasciculation point toward a lower motor neuron problem.
Patients should be screened for medications and drugs that can affect muscles. Patients with
chronic nutritional issues are at risk for mineral and vitamin deficiencies that can cause muscle
pain or weakness. Weakness associated with pain is often indicative of an inflammatory disorder
resulting from a bacterial, parasitic, or viral infection, or may be caused by an inflammatory
myopathy such as polymyositis or dermatomyositis. If the pain and weakness are exacerbated by
activity, a metabolic myopathy should be considered.

The approach to laboratory workup should include markers of serum muscle enzymes including
creatine phosphokinase, aspartate transaminase, alanine aminotransferase, and lactic acid
dehydrogenase concentrations.

Electromyography is useful in determining the neuromuscular level of the lesion, the areas
involved, and the activity of the disease process, as well as in differentiating myopathy from
neuropathy. Although electromyography can identify areas of weakness, a less invasive test
should usually be performed first. The use of magnetic resonance imaging to identify muscle
inflammation has largely replaced the use of electromyography in diagnosing inflammatory
myositis. Imaging of the spine would be helpful in looking for a neurologic cause of the
weakness; however, this girl's pattern of weakness and pain suggest a primary muscle disease,
not a neurologic problem. A neurology referral would not be appropriate at this time for the same
reason. As the child is having progressive weakness and difficulty with activity and rash, this
would clearly not be a viral myositis and thus reassurance would be an inappropriate response.
The gold standard diagnostic test for any muscle disease is muscle biopsy, best taken from
affected tissue identified by magnetic resonance imaging, ultrasonography, or electromyography.
The typical biopsy findings in dermatomyositis are necrosis, phagocytosis, perifascicular
atrophy, fiber size variation, and perivascular inflammation.

PREP Pearls
• Proximal muscle weakness is usually a sign of a primary muscular disorder.
• Pain as a symptom of muscle disease suggests an inflammatory or infectious process.
• A creatine phosphokinase level should be measured in patients with muscle pain and
weakness.
American academy of pediatrics 98
American Academy of Pediatrics PREP 2015

American Board of Pediatrics Content Specification(s)


• Plan the appropriate evaluation and management of myositis
• Identify the etiology of myositis

Suggested Reading
• Huber AM. Idiopathic inflammatory myopathies in childhood: current concepts. Pediatr
Clin North Am. 2012;59(2):365-380. doi:10.1016/j. pc1.2012.03.006.
• Patterson MC, Gomez MR. Muscle disease in children: a practical approach. Pediatr Rev.
1990;12(3):73-82.

American academy of pediatrics 99


American Academy of Pediatrics PREP 2015

Item 33
A 16-year-old boy presents to your office for evaluation of left thigh pain following an injury
during soccer. The boy reports that 3 weeks ago, while playing in a soccer match, he was struck
in the front of the thigh by the knee of another player. He had pain and decreased ability to bend
the left knee after the injury. These symptoms resolved about 10 days after the injury, but over
the past few days, his pain has recurred and the thigh now appears swollen. On physical
examination, you note induration and mild, diffuse tenderness to palpation over the anterior
aspect of the left thigh and limited flexion of the left knee. Strength and sensation in the affected
limb are normal.

Of the following, the MOST likely cause of this boy's symptoms is

A. compartment syndrome due to late hematoma development in the quadriceps muscle


group
B. excess bone formation following an acute hematoma in the quadriceps muscle group
C. neuropraxia due to a direct blow to the femoral nerve
D. partial tearing of the rectus femoris muscle
E. thrombosis of the femoral vein secondary to trauma

American academy of pediatrics 100


American Academy of Pediatrics PREP 2015

Item 33 Preferred Response: B


The boy in the vignette has a classic history and physical examination findings for myositis
ossificans traumatica, heterotopic bone formation that occurs at the site of a previous muscle
injury. Individuals with myositis ossificans traumatica typically have a history of a direct blow to
the affected muscle, followed by the development of bruising or swelling. The quadriceps
muscle group is the most common site of occurrence. Individuals with this condition usually
report at least some improvement in the weeks following the injury, with subsequent worsening
several weeks later.

On physical examination, children and adolescents with myositis ossificans traumatica may have
tenderness and induration at the injury site, as well as restricted motion in the joints crossed by
the affected muscle group. For example, young athletes with heterotopic ossification in the
quadriceps muscles often have decreased knee flexion.

Radiographs for this boy might show densities representing calcification of the quadriceps
muscle without disruption of adjacent bone. Radiographic changes are typically present by 3 to 6
weeks after the injury and often lag behind the development of characteristic signs and
symptoms.

Most individuals with myositis ossificans have symptom improvement with use of ice and
nonsteroidal anti-inflammatory medications. Patients with restricted joint motion should perform
range-of-motion exercises- Individuals with persistent symptoms, or restricted joint motion that
interferes with activity, are candidates for surgical treatment. Surgery involves resection of
heterotopic bone, and is performed once the ossification is complete and the bone scan shows no
increased uptake at the site of trauma, generally 6 to 12 months after injury.

Use of protective equipment during collision sports, such as thigh pads for football, decreases the
risk of muscle injury. Athletes with muscle injury should be encouraged to use ice and
compression and to rest from sports following injury to decrease the risk of hematoma formation.
Following quadriceps trauma, bracing the knee in at least 90 degrees of flexion for the first 24
hours after injury may decrease recovery time and the risk of joint contracture.

Compartment syndrome is a rare complication of quadriceps hematoma and would be expected


to occur within days of injury. Direct blow to the femoral nerve can cause neurologic symptoms
with parethesias and weakness. This would occur within days of the injury with subsequent
gradual improvement. A partial muscle tear would cause immediate pain, swelling, and bruising,
but would not explain this patient's subsequent worsening symptoms. Deep vein thrombosis is
unusual in a healthy athlete; thrombosis generally occurs in the absence of trauma.

PREP Pearls
• Myositis ossificans traumatica refers to ectopic bone formation in a muscle group
following trauma to the muscle.
• Myositis ossificans traumatica should be considered in a patient with a history of a direct
blow to a muscle that exhibits persistent pain or worsening several weeks after injury.

American academy of pediatrics 101


American Academy of Pediatrics PREP 2015

American Board of Pediatrics Content Specification(s)


• Identify complications associated with a deep hematoma of the thigh
• Plan the appropriate management of bruises and hematomas

Suggested Reading
• Beiner JM, Jokl P. Muscle contusion injury and myositis ossificans traumatica. Clin
Orthop Relat Res. 2002;(403 Suppl):S110-S119.
• El Gamma' R, Fernandez C, Abdessalam 5, Chase L, Scherr R, Karp TM. Index of
suspicion. Pediatr Rev. 2008;29(12):437-444. doi:10.1542/pir.29-12-437.
• Sarwark JF, LaBella C. Pediatric Orthopaedics and Sports Injuries: A Quick Reference
Guide. Elk Grove Village, IL: American Academy of Pediatrics; 2010.

American academy of pediatrics 102


American Academy of Pediatrics PREP 2015

Item 34
A 15-year-old, female distance runner presents to your office for a sports participation physical
examination. She reports no problems or concerns. Her temperature, vital signs, and growth
parameters are normal for age. Her physical examination findings are unremarkable, except for a
rash on both feet that she reports has "been there for a long time and itches" (Item Q34). No
other areas of skin are involved.

Of the following, the MOST appropriate test to diagnose this patient's condition is

A. potassium hydroxide test


B. serum immunoglobulin E level
C. skin biopsy
D. skin swab for bacterial culture
E. zinc level

American academy of pediatrics 103


American Academy of Pediatrics PREP 2015

Item 34 Preferred Response: A


The girl described in the vignette has tinea pedis (athletes foot), a dermatophyte infection caused
by Trichophyton or Epidermophyton species. The diagnosis should be confirmed by performing
a potassium hydroxide (KOH) wet mount examination of cutaneous scrapings from the affected
area to visualize septate hyphae. The infection is slowly progressive and characterized by pruritic
lesions with erythematous scales or erosions between the toes that sometimes are accompanied
by fissures. Patches of skin involvement can occur, usually with a sharp demarcation between
involved and uninvolved skin. There is a predilection for disease to occur in the third and fourth
interdigital spaces. Often, the lesions extend onto the sides and sole of the foot. Occasionally,
"moccasin ringworm" develops, where the top of the foot is involved as well. Patients can
develop tinea manum (involvement of the palms and flexor aspects of fingers), as well as nail
dystrophy (onychomycosis or tinea unguium).

Distinguishing extensive tinea pedis from atopic dermatitis or other dermatoses can be difficult.
Patients with atopic dermatitis can present with pruritic scaling rashes, however, atopic
dermatitis typically has a relapsing course, whereas dermatophyte infections persist without
treatment. Also, children with atopic dermatitis usually present at a younger age with eczematous
changes that vary and evolve with age compared with patients with tinea pedis.

Making the distinction between dermatophyte infections and atopic dermatitis or other
inflammatory skin conditions is important because the treatment differs. Tinea pedis should be
treated with topical antifungal agents, and numerous options are available (Item C34). In studies,
allylamines produced slightly better cure rates than azoles for the treatment of tinea pedis. Use of
topical steroids in patients with tinea pedis can complicate the treatment course and lead to
Majocchi granuloma (granuloma trichophyticum), a deep folliculitis that requires oral antifungal
therapy. Use of topical antifungal-steroid combination therapies is also inappropriate, because
this can lead to skin atrophy and excess treatment costs.

A serum immunoglobulin E test can be elevated in a patient with suspected atopic disease, but
will be normal in patients with cutaneous dermatophyte infections. A skin swab for bacterial
culture can be helpful in diagnosing impetigo, but would be negative in the patient described in
the vignette. A skin biopsy would not be the first step in the evaluation of a patient with an
unknown skin disorder, and would be more commonly used for blistering rashes of unknown
etiology or lesions more extensive than that described for the girl in the vignette. Lastly, a zinc
level can be helpful in diagnosing acrodermatitis enteropathica (caused by malabsorption of zinc
through intestinal cells), which presents with an inflammatory rash around the mouth and anus,
accompanied by hair loss and diarrhea.

American academy of pediatrics 104


American Academy of Pediatrics PREP 2015

Item C34. Topical Antifungal Agents for the Treatment of Tined Pedis
Allylamines Azoles Benzylamine Other
Naftifine Clotrimazole* Butenafine* Cilspirck
Terbinafine* Econazole
Ketoconazole*
Miconazole*
Oxiconazole
Sertaconazole
Sukonazole
*Over-the-counter formulation available

PREP Pearls
• Tinea pedis is caused by infection with Trichophyton or Epidermophyton species.
• Visualization of septate hyphae on potassium hydroxide (KOH) wet mount examination of
cutaneous scrapings from the affected area can confirm the diagnosis of tinea pedis.
• Tinea pedis should be treated with topical antifungal agents.
• Use of topical steroids in patients with tinea pedis can complicate the treatment course.
• Use of topical antifungal-steroid combination therapies can lead to skin atrophy and excess
treatment costs.

American Board of Pediatrics Content Specification(s)


• Differentiate the clinical findings of tinea pedis from those of atopic dermatitis, and
manage appropriately

Suggested Reading
• A merican Academy of Pediatrics. Tinea pedis and tinea unguium. In: Pickering LK,
Baker CJ, Kimberlin DW, Long SS, eds. Red Book: 2012 Report of the Committee on
Infectious Diseases. 29th ed. Elk Grove Village, IL: American Academy of Pediatrics;
2012:717-719.
• Crawford F, Hollis S. Topical treatments for fungal infections of the skin and nails of the
foot. Cochrane Database Syst Rev. 2007;(3):CD001434.
doi:10.1002/14651858.CD001434.pub2.
• Kelly BP. Superficial fungal infections. Pediatr Rev. 2012;33(4):e22-e37.
doi:10.1542/pir.33-4-e22.

American academy of pediatrics 105


American Academy of Pediatrics PREP 2015

Item 35
The parents of a healthy 12-year-old girl bring her to your office 4 hours after she inadvertently
swallowed 2 small magnetic balls, which she was using to mimic a tongue piercing. The girl
appears well. She reports having intermittent episodes of "my belly hurting just a little" over the
past 2 hours, but she denies any episodes of vomiting. On physical examination, her vital signs
are normal for age. Her abdomen is soft with normal bowel sounds and mild diffuse tenderness
with palpation, but there are no peritoneal signs. The remainder of your physical examination
findings are unremarkable. Plain radiography of the abdomen reveals that the 2 magnetic balls
are lined up, "side-by-side," in the proximal portion of the girl's small intestine.

Of the following, the BEST next step in management at this time is

A. immediate referral to a pediatric surgeon for operative management


B. reassurance with appropriate anticipatory guidance
C. referral to a pediatric gastroenterologist if there is no improvement within 24 hours
D. repeat abdominal radiograph within 1 week
E. repeat abdominal radiograph within 24 hours

American academy of pediatrics 106


American Academy of Pediatrics PREP 2015

Item 35 S TE Preferred Response: A


The girl in the vignette presents with episodic abdominal pain and diffuse abdominal tenderness
that began after ingestion of 2 magnetic balls, which on abdominal radiography are shown to be
adherent to one another and located in the proximal small intestine. Although she is well
appearing, the best next step is referral to a pediatric surgeon for operative management.

Ingestion of magnets has become an increasingly serious health hazard in children. Small
magnets in toys and household items are readily accessible to children. High-powered
neodymium magnets (also called "rare earth" magnets) are now common components in
household appliances. Furthermore, these magnets are widely available in the form of desk
"toys" and "stress relievers: which may consist of hundreds of small magnetic balls, cubes, or
cylinders. These products are labeled as being for adults, but they have been involved in many
cases of ingestion by children, including those with developmental delays and autism. The risk of
ingestion of these strong "toy" magnets is not limited to young children; cases of older
children—like the girl in the vignette—inadvertently swallowing them while using them to
imitate tongue piercings have been reported. In 2011, the United States Consumer Safety Product
Commission (CPSC) issued an alert regarding the safety risks of neodymium magnets.

When ingested, 2 or more strong magnets may attract across layers of bowel, and lead to
pressure necrosis, fistula, perforation, infection, or obstruction; this could result in avascular
necrosis ,mesentric vessels become trapped between attracted bowel loops. At least 1 pediatric
death resulting from ingestion of high-powered magnets has been reported.

Suspected magnet ingestion requires urgent evaluation. Plain radiographs of the neck and
abdomen should be obtained, including both a lateral and an anterior-posterior view. The need
for more than 1 radiologic view to avoid misclassification of a multiple magnet ingestion as a
single magnet ingestion should be emphasized because magnets may stick together and overlap
on a single radiographic view. Radiography typically cannot determine whether the bowel wall is
compressed between magnets, though the finding of magnets that appear to be stacked but are
slightly separated is suggestive of bowel entrapment.

Management of magnet ingestion depends on the number, location, and type of magnet ingested,
as well as on the timing of the ingestion. In May 2012, a task force of the North American
Society of Pediatric Gastroenterology, Hepatology, and Nutrition (NASPGHAN) developed an
algorithm to guide management of both single and multiple neodymium magnet ingestions
(Third suggested reading).

Single magnet ingestions can generally be managed conservatively, with serial radiographic
follow-up to confirm the progression of the magnet through the gastrointestinal tract and
"magnet precautions" (keeping any other magnetic or metallic objects away from the child) until
the ingested magnet has passed out of the gastrointestinal tract.

Ingestion of multiple magnets carries a high risk of complications and preemptive removal is
warranted. Magnets located in the esophagus or stomach should be removed promptly using
endoscopy. Treatment of children with multiple ingested magnets located beyond the stomach
depends on their symptoms and progression of the magnets through the gastrointestinal tract.
American academy of pediatrics 107
American Academy of Pediatrics PREP 2015

Asymptomatic children may be followed closely with serial examinations and radiography every
4 to 6 hours. Symptomatic children, or those with multiple magnets that are not seen to progress
on repeat radiography, should undergo operative removal of the magnets.

Pediatric health care providers should make parents aware of the serious health risks of high-
powered magnets, including the dangers of magnetic "toys" that can easily be ingested by young
children. Free educational materials for families are available on the American Academy of
Pediatrics website, www.healthychildren.org, as well as the website of the CPSC,
www.cpsc.gov.

The girl in the vignette is known to have ingested multiple magnets (which are currently located
beyond the stomach) and also has abdominal pain, therefore the options of repeating abdominal
radiography within 1 week, repeating abdominal radiography within 24 to 36 hours, or simple
reassurance would not be appropriate in her case. Although consultation with a pediatric
gastroenterologist in addition to a pediatric surgeon would not be completely inappropriate in the
management of this case, prompt referral of symptomatic patients with multiple magnets located
beyond the stomach to a pediatric surgeon is the recommended course of action.

PREP Pearls
• When ingested, 2 or more strong magnets may attract across layers of bowel and lead to
pressure necrosis, fistula, perforation, infection, or obstruction; this could result in severe
consequences, including intestinal resection.
• The need for more than 1 radiologic view to avoid misclassification of a multiple magnet
ingestion as a single magnet ingestion should be emphasized because magnets may stick
together and overlap on a single radiographic view.
• Management of magnet ingestion depends on the number, location, and type of magnet
ingested, as well as on the timing of the ingestion.

American Board of Pediatrics Content Specification(s) .


• Plan the management of a patient who has ingested a magnet

Suggested Reading
• Abbas MI, Oliva-Hemker M, Choi J, et al. Magnet ingestions in children presenting to
US emergency departments, 2002-2011.1 Pediatr
• Gastroenterol Nutr. 2013;57(1):18-22. doi:10.1097/MPG.0b013e31132952ee5.
• Gilger MA, Jain AK, McOmber ME. Foreign bodies of the esophagus and
gastrointestinal tract in children. UpToDate. Available online only for subscription.
• Hussain SZ, Bousvaros A, Gilger M, et al. Management of ingested magnets in children.
I Pediatr Gastroenterol Nutr. 201255(3):239-242. doi:10.1097/MPG.06013e3182687be0.

American academy of pediatrics 108


American Academy of Pediatrics PREP 2015

Item 36
You attend the cesarean delivery of a term neonate due to breech presentation. The mother
received good prenatal care with no history of substance abuse. General anesthesia is used due to
maternal scoliosis with rod placement. Artificial rupture of the membranes occurs at the time of
delivery revealing clear amniotic fluid. After breech extraction, the neonate emerges limp with a
weak cry. The neonate is placed on the warmer, dried, stimulated, and the airway cleared. Your
assessment at 30 seconds after birth reveals a hypotonic neonate with a heart rate of 90
beats/min, intermittent apnea, and central cyanosis.

Of the following, the BEST next management step for the neonate is to

A. administer naloxone
B. deliver blow-by oxygen
C. flick the soles of the feet
D. initiate positive-pressure ventilation
E. start chest compressions

American academy of pediatrics 109


American Academy of Pediatrics PREP 2015

Item 36 Preferred Response: D


Positive-pressure ventilation should be initiated if the assessment of a newborn at 30 seconds
after delivery reveals apnea, gasping, or a heart rate less than 100 beats/min, as seen in the
neonate described in the vignette. The establishment of air exchange by effective ventilation of
the lungs is the most important action in neonatal resuscitation.

Any decrease in blood flow or oxygen delivery in utero may lead to fetal compromise, which
often manifests as apnea at the time of delivery. Perinatal stress will initially lead to a period of
rapid breathing, followed by apnea and a falling heart rate in the fetus (Item C36A, page C-31).
This primary apnea often responds to gentle stimulation of the affected newborn by drying or
slapping of the feet immediately after birth. Ongoing perinatal stress leads to a brief period of
gasping respirations before the fetus enters secondary apnea, with continued bradycardia and loss
of blood pressure (Item C36A, page C-31). A newborn experiencing secondary apnea will not
respond to gentle stimulation and requires the initiation of positive pressure ventilation. It is
difficult to determine whether a newborn is in primary or secondary apnea at birth and delays in
initiating effective resuscitation may cause further compromise. Once effective ventilation is
established, most newborns will respond with a rise in heart rate, followed by the development of
respiratory effort.

Health care providers attending the cesarean delivery of the neonate in the vignette should be
prepared for the potential need for resuscitation because of the intrapartum risk factors of breech
presentation and general anesthesia. The neonate remains apneic with a heart rate less than 100
beats/min at 30 seconds after birth and positive-pressure ventilation should be initiated.
Continued stimulation by flicking of the feet is unlikely to initiate spontaneous respiratory effort
and will lengthen the time the neonate is compromised. Blow-by oxygen is not indicated, as the
neonate is not spontaneously breathing. It is important to note that room air resuscitation is
recommended in full-term neonates, with the placement of a pulse oximeter on the right hand at
the time positive pressure ventilation is begun to guide the future use of supplemental oxygen.
Chest compressions are indicated when the heart rate remains below 60 beats/min after 30
seconds of effective positive-pressure ventilation (Item C36B, page C-32). Naloxone is no longer
considered a primary resuscitation drug in the delivery room, but may be used in a neonate with

American academy of pediatrics 110


American Academy of Pediatrics PREP 2015

a normal heart rate and oxygen saturation who fails to breathe spontaneously and whose mother
received a narcotic drug within 4 hours of delivery and has no history of drug use. Neonates
whose respiratory effort is depressed by maternal magnesium sulfate or general anesthesia do not
respond to naloxone.

PREP Pearls
• The establishment of air exchange by effective ventilation of the lungs is the most
important action in neonatal resuscitation.
• It is difficult to determine whether a newborn has primary or secondary apnea at birth,
and delays in initiating effective resuscitation may cause further compromise.

American Board of Pediatrics Content Specification(s)


• Recognize the indications for immediate positive-pressure ventilation in a neonate

Suggested Reading
• Kattwinkel J, Perlman JM, A ziz K, et al. Part 15, neonatal resuscitation: 2010 American
Heart Association Guidelines for Cardiopulmonary Resuscitation and Emergency
Cardiovascular Care. Circulation. 2010;122(18 suppl 3):5909-5919. doi:10.1161/
C1RCULATIONAHA.110.971119.
• Kattwinkel J. Textbook of Neonatal Resuscitation. 6th ed. Elk Grove Village, IL:
American Academy of Pediatrics; 2011.

American academy of pediatrics 111


American Academy of Pediatrics PREP 2015

Item 37
You are called to evaluate a 10-hour-old full-term neonate with poor feeding. The chart
documents a normal pregnancy, labor, and delivery. The mother did not take any medications
during pregnancy and did not receive any medications during delivery. Physical examination of
the neonate reveals a high forehead with narrowing at the temples and a thin upper lip. With
crying, the facial movements are symmetric, and the palate and tongue are normal. The neonate's
limbs move symmetrically when he is stimulated, but at rest, there is very low tone in the limbs
and trunk. Deep tendon reflexes are present, but diminished. The remainder of the physical
examination is unremarkable.

Of the following, the MOST likely diagnosis is

A. Angelman syndrome
B. Duchenne muscular dystrophy
C. infant botulism
D. Prader-Willi syndrome
E. spinal muscular atrophy

American academy of pediatrics 112


American Academy of Pediatrics PREP 2015

Item 37 Preferred Response: D


The neonate described in the vignette has Prader-Willi syndrome, which presents in the neonatal
period with hypotonia, poor suck, and characteristic facial features that include bitemporal
narrowing of the head, almond-shaped eyes, elongated face, and thin upper lip. In this case, the
dysmorphic features are suggestive of a central nervous system, or neurogenetic, cause for the
hypotonia. When assessing a neonate with hypotonia, the clinical features help to determine
whether there is a central nervous system or peripheral nervous system cause. Dysmorphic facial
features, especially if characteristic of a known syndrome as in this case, suggest a central
nervous system cause. Hypotonia of just the lower extremities (sparing the trunk and upper
extremities) suggests a spinal cord disorder such as myelomeningocele. The pediatrician should
carefully examine the child for a sacral dimple, hair tuft, or club feet. Occult myelomeningocele
may have no cutaneous findings, so if the clinical examination is suggestive, then imaging is
necessary to make the diagnosis. Peripheral nervous system disorders can also present with
hypotonia in the newborn period. The clinical features and neurologic examination often provide
the best dues. The absence of reflexes in a hypotonic neonate, especially if accompanied by
tongue fasciculations, suggests spinal muscular atrophy. Contractures or low muscle bulk
suggest a muscle disorder, such as congenital myopathy.

Children with Angelman syndrome are typically asymptomatic at birth. During infancy they can
develop hypotonia and poor feeding. The typical facial features are prognathia and wide-spaced
teeth. Aside from Prader-Willi, the remainder of the response choices are disorders of the
peripheral nervous system and do not present with dysmorphic facial features. Duchenne
muscular dystrophy typically presents in the infant age or older with lower extremity weakness,
calf pseudohypertrophy, reflexes that are diminished or absent, and in some cases, toe walking.
Infant botulism presents with progressive ptosis, facial weakness, and eventually limb hypotonia
and weakness. Spinal muscular atrophy rarely presents in the newborn period. Neonates with this
disease have normal eye and facial movements, but trunk and limb weakness with hypotonia,
areflexia, and tongue fasciculations.

PREP Pearls
• In neonates with hypotonia, dysmorphic facial features suggest a central nervous system
disorder.
• The absence of reflexes in a hypotonic neonate, especially if accompanied by tongue
fasciculations, suggests a peripheral nerve disorder such as spinal muscular atrophy.

American Board of Pediatrics Content Specification(s)


• Differentiate the findings associated with central nervous system causes of hypotonia
from those of peripheral nervous system causes

Suggested Reading
• Bodamer OA, Miller G. Approach to the infant with hypotonia and weakness. UpToDate.
Available online only for subscription.
• Gunay-Aygun M, Schwartz S, Heeger S, O'Riordan MA, Cassidy SB. The changing
purpose of Prader-Willi syndrome clinical diagnostic criteria and proposed revised
criteria. Pediatrics. 2001;108(5):e92. doi:10.1542/ peds.108.5.e92

American academy of pediatrics 113


American Academy of Pediatrics PREP 2015

• Peredo DE, Hannibal MC. The floppy infant: evaluation of hypotonia. Pediatr Rev.
2009;30(9):e66-e76. doi:10.1542/pir.30-9-e66.

American academy of pediatrics 114


American Academy of Pediatrics PREP 2015

Item 38
A 4-year-old boy presents for evaluation of short stature. He was growing well until 1 year ago,
when he was noted to have a linear growth velocity of 2.5 cm (Item Q38). His weight is also low,
but tracking at the third percentile. His body mass index is normal.
History reveals he is sometimes a "picky eater" but has no other gastrointestinal symptoms.
Family history shows his mother is 160 cm (5 ft, 3 in) tall, and his father 167.6 cm (5 ft, 6 in).
Both parents had delayed puberty. His maternal uncle was diagnosed with a pituitary tumor as a
child. On physical examination, you note the child is short, but otherwise well appearing. The
child's bone age is 2.5 years at a chronological age of 4 years.

Of the following, the MOST likely diagnosis is


A. achondroplasia
B. constitutional delay of growth
C. familial short stature
D. growth hormone deficiency
E. inadequate caloric intake

American academy of pediatrics 115


American Academy of Pediatrics PREP 2015

Item 38 Preferred Response:D


Postnatal growth is normally characterized by a rapid linear growth velocity that declines
progressively after birth (approximately 25 cm, 12 cm, and 8 cm/year during the first 3 postnatal
years). Thereafter, until the onset of puberty, linear growth occurs at a relatively constant rate
about 5 cm/ year (range of 4-7 cm/year).

The child in the vignette has a growth velocity (rate of linear growth) far below the average of 5
cm/year, and thus a pathologic cause must be considered for his short stature. When height is
affected more than weight, an endocrine cause is more likely, and of the choices offered, growth
hormone deficiency is the most likely cause. Over time, the delay in bone age may progress and
a continued abnormal growth velocity is expected.

Achondroplasia is characterized by short stature, but also megalocephaly, facial dysmorphisms,


abnormal vertebrae, disproportionally short tubular bones, and abnormal fingers and hands. It is
autosomal dominantly inherited. The child in the vignette has unremarkable physical
examination findings, making achondroplasia unlikely.

Despite a family history of short stature and pubertal delay, this growth chart is not consistent
with familial short stature or constitutional delay of growth. In these conditions, linear growth
often resets to the bottom of the growth chart in the first 18 months to 2 years after birth and then
tracks at a normal growth velocity. Children with familial short stature will continue this growth
pattern until their final adult height is reached, and usually have a concordant bone age. They
would not reset their growth pattern at 4 years of age, as noted in the child in the vignette.
Children with constitutional delay of growth will continue growing at a prepubertal growth rate
beyond the expected age, have a delayed bone age, and then have catch-up growth later,
consistent with their family history.

Gastrointestinal disorders, such as inflammatory bowel disease (especially Crohn disease) or


other chronic disease-mediated causes of malnutrition, often present initially with poor weight
gain and impaired linear growth, especially if the period of inadequate nutrition is prolonged.

PREP Pearls
• An endocrine cause of poor growth is likely when linear growth velocity declines, but
weight gain is either normal or increasing.
• In the majority of cases in which poor growth is related to under nutrition, falloff in
weight gain will precede falloff in linear growth.

American Board of Pediatrics Content Specification(s)


• Distinguish among constitutional short stature, genetic (familial) short stature, and
growth hormone or thyroid deficiencies by growth chart evaluation
• Understand the natural history of genetic (familial) short stature
• Plan an appropriate diagnostic evaluation to differentiate constitutional growth delay and
other conditions causing growth delay
• Understand the natural history of constitutional growth delay

American academy of pediatrics 116


American Academy of Pediatrics PREP 2015

Suggested Reading
 American Academy of Pediatrics. EQIPP. Growth surveillance and linear growth failure.
2014. Expires 2/25/2017.
 Jones KL. Achondroplasia. In: Jones KL, ed. Smith's Recognizable Patterns of
Malformation. 6th ed. Philadelphia, PA: Saunders Elsevier; 2006:390-397.
 Rose SR, Vogiatzi MG, Copeland KC. A general pediatric approach to evaluating a short
child. Pediatr Rev. 2005;26(11):410-420. doi: 10.1542/pir.26-11-410.
 Rosenfeld RG, Cohen P. Disorders of growth hormone/insulin-like growth factor secretion
and action. In: Sperling M, ed. Pediatric Endocrinology. 3rd ed. Philadelphia, PA: Saunders
Elsevier; 2008:281-305.
 Weintraub B. Growth. Pediatr Rev. 2011;32(9):404-406. doi:10.1542/pir.32-9-404.

American academy of pediatrics 117


American Academy of Pediatrics PREP 2015

Item 39
You are seeing a healthy 1 month old for a health supervision visit. As you review the plans for
the next visit, the parents state they are considering not immunizing their child because a relative
was diagnosed with autism 2 months after receiving a vaccine.

Of the following, the MOST appropriate response to these parents is to

A. acknowledge their concerns and inform them that current evidence does not support such
a link
B. ask them to sign the American Academy of Pediatrics vaccine refusal form at this visit
C. provide them with a copy of an epidemiologic study showing the absence of correlation
between vaccines and autism
D. recommend an alternative vaccine schedule that would not overwhelm the child's
immune system
E. tell them you can no longer care for their child in your practice

American academy of pediatrics 118


American Academy of Pediatrics PREP 2015

Item 39 IC A Preferred Response: A


Most pediatricians encounter vaccine-hesitant parents, and the health care provider must be
prepared to provide accurate, science-based information while acknowledging the parents'
concerns. One survey found that 54% of parents were worried that vaccines could cause serious
adverse events, even while 90% agreed that vaccines were a good way to protect their child from
disease. In this same survey, 11.5% of parents reported refusing at least 1 recommended vaccine,
and 25% believed that some vaccines cause autism. The physician should inquire about the
parents' concerns, and then present clear, understandable, evidence-based information that
addresses those issues. Generally, the information pro-vided should be presented in lay language.
Providing copies of scientific reports is not usually recommended.

Pediatricians should be familiar with some of the common beliefs that lead to parental vaccine
hesitancy. These include a discredited, but often cited study that linked the mumps measles-
rubella vaccine to autism; the perceived potential risks of preservatives and additives,
particularly thimerosal, in vaccines; concerns that the complex and extensive vaccine schedule
will overwhelm the child's immune system; and concern that the safety of newer vaccines has not
been adequately confirmed.

Some pediatricians dismiss patients from their practice if the family refuses vaccines. Many of
these physicians are concerned about their liability should the child develop a vaccine
preventable disease. The American Academy of Pediatrics (AAP) bioethics committee does not
support the practice of "firing" patients and cautions that if this is done, all legal obligations to
assure the child has access to care must be met. The AAP Refusal to Vaccinate template
(www.aap. org/immunization/pediatricians/pdf/RefusaltoVaccinate. pdf) is a useful resource to
document the discussion about risks and benefits of vaccines when parents refuse immunization.
It most often is used at the time a vaccine is to be provided and documents counseling regarding
the specific immunization recommended at that time. It does not supplant the need to discuss the
family's concerns about immunization. The AAP does not support the use of alternative vaccine
schedules because they are not science-based and do not provide timely protection for the
youngest, most vulnerable population.

PREP Pearls
• More than half of parents have concerns about vaccine safety, even while acknowledging
immunizations' important role in disease prevention.
• Pediatricians should be prepared to present clear, evidence-based recommendations about
vaccines in language free of scientific jargon.
• The American Academy of Pediatrics (AAP) bioethics committee does not support the
practice of dismissing patients and families from a practice because of vaccine refusal.
• The AAP does not support the use of alternative vaccine schedules because they are not
science-based and do not provide timely protection for the youngest, most vulnerable
population.

American Board of Pediatrics Content Specification(s)


• Plan an appropriate approach to addressing the needs of the vaccine-hesitant family

American academy of pediatrics 119


American Academy of Pediatrics PREP 2015

Suggested Reading
• American Academy of Pediatrics. EQIPP. Immunizations: Improve your practice rates.
2013. Expires 6/13/2016.
• Freed GL, Clark Si, Butchart AT, Singer DC, Davis MM. Parental vaccine safety
concerns in 2009. Pediatrics. 2010;125(4):654-659. doi:10.1542/ peds.2009-1962.
• Gilmour ), Harrison C, Asadi L, Cohen MH, Vohra S. Childhood immunizations: when
physicians and parents disagree. Pediatrics. 2011; 128(suppl 4):S167-S174.
doi:10.1542/peds.2010-2720E.
• Healy CM, Pickering LK. How to communicate with vaccine-hesitant parents. Pediatrics.
2011;127(suppl 1):S127-S133. doi:10.1542/peds. 2010-1722S.
• The Childhood Immunization Support Program, Centers for Disease Control and
Prevention and the American Academy of Pediatrics. 2014 Immunizations website at
http://www2.aap.org/immunisation/ pediatricians/riskcommunication-VIDEOS.html.

American academy of pediatrics 120


American Academy of Pediatrics PREP 2015

Item 40
A shy, 14-year-old adolescent boy presents to your office for a physical examination. He denies
any concerns today and is not sexually active. His past medical history is significant only for
cryptorchidism that was repaired at 1 year of age. The boy's last genital examination, at 10 years
of age, was unremarkable for a prepubertal male. His height today is at the 5th percentile for his
age, and weight is at the 10th percentile, and he has normal vital signs. He has no facial hair,
axillary hair, or voice changes. He refuses a genital examination. The remainder of his physical
examination findings are unremarkable. You discuss with your medical student the importance
of performing a genital examination and how one would teach the boy about testicular self-
examination.

Of the following, the MOST important reason to do a genital examination in this case is to
A. determine that puberty is progressing normally
B. evaluate for the presence of an indirect hernia
C. inspect and palpate for the presence of varicoceles
D. inspect for signs of sexually transmitted infections
E. palpate for the presence of a testicular mass

American academy of pediatrics 121


American Academy of Pediatrics PREP 2015

Item 40 IC Preferred Response: E


It is not uncommon to have an early male adolescent refuse to undress for a genital examination.
Early puberty is a time of body changes that make adolescents very self-conscious and concerned
about their normality. The main value of an examination is usually to provide reassurance. The
adolescent in the vignette has a history of cryptorchidism. An undescended testicle is often
abnormal to start with and is at increased risk for development of masses. Therefore, this is the
primary reason to teach the adolescent boy how to do a self-examination and persuade him to
allow you to do one in the near future.

A male adolescent begins the development of secondary sexual characteristics between 9 and 14
years of age. Testicular development is the first sign of puberty in a male adolescent and a
teenager should enter sexual maturity rating 2 by 14.5 years of age at the latest. Therefore, for
the boy in the vignette, it is acceptable to wait up to 6 months to confirm the onset of puberty and
evaluate for testicular mass. In the meanwhile, educating the parents and the adolescent, along
with gaining his trust, would be very important as most patients will allow an examination at
future visits. The testicular examination can be very revealing of pathology that includes hernias,
varicoceles, and signs of sexually transmitted infections; in addition, testicular examination is
most important to evaluate for masses. While testicular cancer is not common, the peak
incidence is during adolescence and young adulthood.

PREP Pearls
• A history of cryptorchidism requires that a male adolescent be routinely screened for the
development of testicular masses.
• A number of lesions may be found on routine genital examination in a boy, despite a
negative history of symptoms.

American Board of Pediatrics Content Specification(s)


• Understand the importance of testicular self-examination

Suggested Reading
• Adelman WP, Joffe A. Consultation with the specialist: testicular masses/ cancer. Pediatr
Rev. 2005;26(9):34I-344. doi:10.1542/pir.26-9-341.
• Brenner JS, Hergenroeder AC, Kozinetz CA, and Kelder SH. Teaching testicular self-
examination: education and practices in pediatric residents. Pediatrics. 2003;111(3):e239-
e244. doi:10.1542/peds.111.3.e239.
• Cavanaugh RM Jr. Screening for genitourinary abnormalities in adolescent males. Pediatr
Rev. 2009;30(11):431-438. doi:10.1542/pir.30-11-431.

American academy of pediatrics 122


American Academy of Pediatrics PREP 2015

Item 41
A 4-year-old girl is brought to your office for follow-up of her atopic dermatitis and food
allergies to peanuts. The parents report symptoms of sneezing fits and nasal itching that occur
mostly during the spring and summer months. Their cat died recently and the girl has been
requesting a new cat. The parents did not notice her having any symptoms around the cat, but
remarked that she gets worsening symptoms when around the neighbor's cat. They also note that
the child's eczema worsens when they visit her grand-parents, who have 2 dogs. The parents
would like to get her a new pet, but worry it will worsen her allergies and they seek your
suggestions.

Of the following, your BEST recommendation is to get her


A. any pet, as long as they get a high-efficiency particulate air purifier
B. a dog, but keep it outdoors in the daytime
C. a hypoallergenic, nonshedding dog
D. nonfurry pets, such as a fish
E. the same breed of cat they had before

American academy of pediatrics 123


American Academy of Pediatrics PREP 2015

Item 41 Preferred Response: D


The child in the vignette desires to have a pet, has evidence of evolving atopic diathesis, and
worsening symptoms around cats and dogs; therefore, a pet without fur would be the preferred
option.

Immediate hypersensitivity to allergens is very common among children with an atopic diathesis
(atopic dermatitis, rhinitis, and asthma). One-third of US households own at least 1 cat and 40%
of American households own at least 1 dog. Sensitization to 1 or more of the major indoor
allergens (such as dust mites, cat, dog, or cockroach), in conjunction with significant
accumulation of pertinent allergens in the home, has been consistently found to be a major risk
factor for asthma. Further evidence for a causal relationship between allergen exposure and
asthma comes from bronchoprovocation experiments demonstrating eosinophilic airway
inflammation and increases in bronchial hyperreactivity after inhalation challenges. Relocation
of asthmatic individuals from their homes to a low allergen residential setting is associated with
significant improvement in clinical symptoms and bronchial hyperreactivity. Of note, variations
in climate, housekeeping practices, and pet ownership can result in differences in the quantity
and impact of dominant allergens.

In most epidemiologic studies, there is a strong positive correlation between sensitization to cat
or dog allergens and asthma, and to a lesser degree, with allergic rhinitis. If the patient is
sensitized to animal dander based on positive allergy skin tests or blood tests, it is very likely that
it is a contributory factor. In this case, the most effective course of action includes removal of the
pet (when feasible). However, it has been observed that even when a pet such as a cat is removed
from the house, allergens persist for many weeks or months. Aggressive cleaning measures can
accelerate the removal of allergens, but the quantity of cat allergen that accumulates in carpets,
sofas, and mattresses represents a major allergen reservoir that is difficult to remove without
aggressive cleaning, including removal of old carpeting and upholstered furniture. Therefore,
while complete removal of all cats and dogs is necessary to minimize pet allergen exposure in a
home, concomitant interventions targeted at reducing exposure to allergen reservoirs must also
be emphasized. Cat characteristics, such as length of hair, sex, reproductive status, and time
spent indoors, are not associated with levels of the dominant cat allergen (Fel d 1) in the
environment. Therefore, measures designed to alter those factors do not contribute significantly
to alleviating allergenicity. Data about the effect of neutering a dog or cat are inconsistent, so no
specific recommendations can be made at this time about the value of this procedure to reduce
allergen exposure.

Many pet-allergic patients with asthma simply refuse to remove the pet from their home. Disease
severity and personal preference may impact their choice to keep their pet. For that reason,
control of exposure to pet allergens with the pet still living in the environment is necessary This
process often requires aggressive measures, such as removing allergen reservoirs as discussed
earlier, washing the pet, and air cleaning. For instance, the cat allergen Fel d 1 is very pervasive
in indoor spaces. Approximately 60% of airborne Fel d 1 settles out within 2 days of disturbance,
leaving smaller particles that can remain airborne for 2 weeks or longer. This duration can be
reduced using high efficiency particulate air (HEPA) filtration. Dry dusting with a sticky dust
cloth is an effective cleaning method for removing Fel d 1 from hard smooth surfaces, but fabric
and carpet can represent significant reservoirs. When compared with high-efficiency vacuum
American academy of pediatrics 124
American Academy of Pediatrics PREP 2015

cleaning alone, the addition of HEPA filters significantly improved asthma symptoms after 12
months in cat-allergic individuals who were living with a cat, but showed no change in reservoir
or airborne Fel d I levels. A combination of a HEPA room air cleaner, mattress, pillow covers,
and cat exclusion from the bedroom was shown to reduce airborne Fel d 1 levels, although this
was not associated with clinical improvement in one controlled study. Washing cats by
immersion for 3 min at weekly intervals for a 1-month period produced a mean decrease in
airborne allergen of 79%. However, after repeated washing, the airborne levels before the next
wash were not consistently decreased. Therefore, for the child in this vignette, procuring a pet
dog or cat carries the risk of getting significant allergen exposure, despite the mitigating effect of
a HEPA air filter, and would not be recommended.

There is no published literature confirming "hypoallergenic" breeds of cats or dogs, though there
may be individual animals with varying allergen levels. Several approaches to creating
hypoallergenic cats have been tried, ranging from knocking out the gene for Fel d 1, to breeding
cats that have lower Fel d 1 levels, to simply selecting individual cats that have low levels. There
are no published studies demonstrating that "hypoallergenic cats" result in lower levels of Fel d 1
in the house. Preliminary studies exposing cat-allergic subjects to the animals showed lesser
symptoms, although no allergen measurements have been published and further information is
required. Similarly, there is no convincing evidence that certain breeds of dogs are less allergenic
than others, though reported variations in the immunoglobulin E (IgE) responses to species-
specific dog allergens may explain differences in individual tolerability. There also appears to be
marked variability among individual animals within each breed.

An important question often asked is whether living with a pet increases or decreases the
likelihood of sensitization and the risk of developing allergic disease, particularly in the neonate
and infant. Although most studies indicate that early exposure to pets reduces the likelihood that
a child will be sensitized to pet allergens, a 2012 meta-analysis concluded that "advice from
health care practitioners to avoid or to specifically acquire pets for primary prevention of asthma
or allergic rhinitis in children should not be given:' Although exposure to elevated cat allergen
Fel d 1 concentrations before 3 months of age may reduce the likelihood of developing cat
sensitization, the risk reduction does not appear to justify a decision to get a cat to avoid IgE
sensitization. Similarly, although exposure to elevated dog allergen (Can f 1) concentrations
before 3 months of age have been shown to reduce the likelihood of developing dog
sensitization, the risk reduction does not appear to be robust enough to justify a decision to get a
dog to avoid IgE sensitization. At present, therefore, the evidence neither supports preemptive
removal of animals from the home in the hope of reducing the unborn child's risk of allergic
disease, nor does it favor procuring pets for the singular purpose of preventing allergic disease.

PREP Pearls
• In a child sensitized to pet dander, avoidance of pet allergen exposure is recommended in
order to mitigate severity and worsening of allergic disease.
• The most effective measure to minimize allergen exposure in a pet-sensitized patient is
removal of the pet in conjunction with allergen abatement interventions at home. Less-
effective, but perhaps more realistic measures include those targeted at reducing exposure
to reservoirs and blocking pathways from reservoirs to home occupants.

American academy of pediatrics 125


American Academy of Pediatrics PREP 2015

• To date, studies have not validated claims that "hypoallergenic pets" carry a lower
allergen load or cause fewer symptoms.
• Evidence currently does not support preemptive removal of, or acquiring a pet, in the
hope of preventing allergic disease in the unborn child and infant.

American Board of Pediatrics Content Specification(s)


• Understand the role of the environment leg. indoor pets, passive exposure to cigarette
smoke) in the development and severity of allergic disease

Suggested Reading
• Ownby DR, Johnson CC, Peterson EL. Exposure to dogs and cats in the first year of life
and risk of allergic sensitization at 6 to 7 years of age_ 'AMA. 2002;288(8):963-972.
doi:10.1001/jama.288.8.963.
• Portnoy JM, Kennedy K, Sublett JL, et al. Environmental assessment and exposure
control: a practice parameter-furry animals. Ann Allergy Asthma Immunol.
2012;108(4):223.e1-223.e15. doil10.1016/j.anai.2012.02.015.
• Vredegoor DW, Willemse 7', Chapman MD, Heederik DJ, Krop EL Can f 1 levels in hair
and homes of different dog breeds: lack of evidence to describe any dog breed as
hypoallergenic. J Allergy Clin hnmunol. 2012;130(4):904-909.e7.
doi:10.1016/j.jaci.2012.05.013.

American academy of pediatrics 126


American Academy of Pediatrics PREP 2015

Item 42
You are caring for a 30-kg, 10-year-old boy with a history of severe spastic quadriplegia from a
hypoxic-ischemic event at 3 years of age. Other medical problems include subglottic stenosis
from prolonged intubation requiring tracheostomy, seizure disorder, hydrocephalus, scoliosis,
and inability to feed that required a gastrostomy tube. He was electively admitted to the hospital
for airway reconstructive surgery. Postoperative management required deep sedation with a
morphine drip and continuous enteral feeds. NOW, on postoperative day 3, he develops
abdominal distention and bilious vomiting.

Vital signs show a temperature of 37°C, heart rate of 80 beats/min, blood pressure of 100/60 mm
Hg, respiratory rate of 20 breaths/min, and oxygen saturation of 98% on the ventilator. On
physical examination, he is ventilated and sedated. He has moist mucous membranes, and his
lungs are clear to auscultation bilaterally. Cardiovascular examination shows a regular rate and
rhythm, with warm and well perfused extremities. His abdomen is soft, distended, and
tympanitic to percussion. There is no organomegaly. Bowel sounds are hypoactive. Over the last
24 hours, fluid intake was 1,200 mL of feeds and 100 mL of medications and other fluids to
maintain catheter patency. Fluid output includes 600 mL of urine (1.5 mL/kg per hour) and
approximately 500 mL of emesis. Frontal and cross table lateral abdominal radiographs are
shown (Item Q42A)(Item Q42B).

ITEM Q42A: Frontal abdominal radiograph for the boy in the vignette.
ITEM 042B: Cross Table Lateral abdominal radiograph for the boy in the vignette.

Of the following, the MOST appropriate initial step in management is to


A. consult surgery for lysis of adhesions
B. discontinue feeds and place gastrostomy tube to dependent drainage
C. discontinue sedation and administer naloxone
D. place transpyloric feeding tube and administer metoclopramide
E. place urinary catheter and measure intra-abdominal pressure

American academy of pediatrics 127


American Academy of Pediatrics PREP 2015

Item 42 Preferred Response: B


The child in the vignette, who has a history of severe spastic quadriplegia and chronic respiratory
failure and tracheostomy, has a postoperative ileus. He has not tolerated feeds, as evidenced by
emesis. Abdominal radiography shows air-filled viscera, but physical examination findings are
not consistent with a "surgical" abdomen, which often shows tenderness, guarding, or rigidity.
The correct first step in management is to discontinue feeds and attempt to decompress the
abdomen by placing the gastrostomy tube to dependent drainage.

Ileus is a condition of impaired intestinal motility and decreased gut peristalsis in the absence of
mechanical obstruction, which causes abdominal pain, distention, and vomiting. Physical
examination findings can include either hyperactive or hypoactive bowel sounds, abdominal
distention, and tympanitic bowel sounds. Ileus is most commonly associated with abdominal
surgery, infections, electrolyte disturbances, and medications, including narcotics, anesthetics,
and some chemotherapeutic agents. Other nonobstructive causes of ileus include meconium ileus
from neonatal cystic fibrosis, congenital aganglionic megacolon (Hirschsprung disease), and
chronic intestinal pseudo-obstruction.

Airway reconstructive surgery is a relatively major operative procedure. Several days of limited
movement are required to allow wound healing, and postoperative management often involves
large doses of sedatives. Although the child in this vignette did not have abdominal surgery, the
narcotics he received as part of his sedative regimen likely caused a "functional ileus."

A consultation with general surgery could be pursued because of the possibility of a small bowel
obstruction. However, obstruction caused by adhesions would more likely occur after a recent or
remote abdominal surgery. A surgical emergency would be more likely if there were rigidity,
guarding, or signs of intestinal ischemia such as hemodynamic compromise or lactic acidosis.
Discontinuing sedation and administering naloxone would not be indicated because it could
cause rebound pain and agitation, which might compromise the surgical repair. A transpyloric
tube for feeding intolerance may decrease vomiting in cases of impaired gastric emptying,
pyloric or duodenal obstruction, or gastroesophageal reflux disease. Bypassing the stomach with
a feeding tube is not likely to be helpful in this case because the underlying cause is a narcotic-
induced, postoperative "functional" ileus. Measurement of intra-abdominal pressure can be
helpful in diagnosing abdominal compartment syndrome, but this condition is often accompanied
by significant abdominal distention, rigid abdomen, and decreased urine output.

Postoperative feeding problems are common, especially after abdominal surgery, procedures
involving long duration of anesthesia, or requirement of significant analgesic medications and
sedation. A mechanical obstruction caused by adhesion is more common in abdominal surgery or
in a patient who has had previous abdominal surgery. Nonobstructive ileus causing vomiting,
abdominal distention, and discomfort can be treated with gastric drainage, and usually resolves
within 24 to 72 hours.

American academy of pediatrics 128


American Academy of Pediatrics PREP 2015

PREP Pearls
• Nonobstructive ileus can occur after surgery, and is more common in abdominal surgery
or cases requiring large doses of pain medication or sedation.
• Abdominal distention and discomfort due to functional ileus can be treated with gastric
drainage.
• A surgical consultation should be pursued if a mechanical bowel obstruction is suspected.

American Board of Pediatrics Content Specification(s)


• Plan appropriate management for a patient who has postoperative intestinal obstruction

Suggested Reading
• Bansal V, Ochoa JB. Ileus and mechanical small bowel obstruction. In: Fink MP,
Abraham E, Vincent JL, Kochanek PM, eds. Textbook of Critical Care. 5th ed.
Philadelphia, PA: Saunders Elsevier; 2005:1049-1053.
• Chu A, Liacouras CA. Ileus. In: Kliegman RM, Stanton BF, St. Geme JW III, Schor NF,
Behrman RE. eds. Nelson's Textbook of Pediatrics. 19th ed. Philadelphia, PA: Saunders
Elsevier; 2011:1287.
• Schnitzler E, lolster T, Russo RD. The acute abdomen. In: Nichols DG, ed. Rogers'
Textbook of Pediatric Intensive Care. 4th ed. Philadelphia, PA: Lippincott Williams &
Wilkins; 2008:1520-1530.

American academy of pediatrics 129


American Academy of Pediatrics PREP 2015

Item 43
You are called to the emergency department to evaluate a 5-day-old male newborn who was
brought in by ambulance because of a prolonged episode of twitching and perioral cyanosis at
home. Over the course of several hours before this episode, he was noted to be fussy, did not
feed well, and seemed less active. The parents report he acted well the day before, and they did
not note any fever.
His birth weight was 3 kg, and he had an Apgar score 8 at 1 min and 9 at 5 min. The prenatal
course and delivery are said to be unremarkable. The mother was negative for group B
Streptococcus, human immunodeficiency virus, and rapid plasma reagin. He was discharged
home at 2 days of age.
On physical examination, the baby's temperature is 35.9°C, heart rate is 160 beats/min, blood
pressure is 52/32 mm Hg, and respiratory rate is 56 breaths/min.
The newborn is floppy with little spontaneous activity. His anterior fontanelle is full, and his
skin is noted to be without lesions. A head, ears, eyes, nose, and throat examination shows
positive red reflex bilaterally. His neck is supple. His lungs have shallow respirations. His heart
has a regular rhythm without murmur, rub, or gallop. His abdomen is distended and soft, and his
liver edge is palpable. Findings on genitourinary examination are unremarkable. His extremities
are without lesions.

Laboratory test results


include the following:
• White blood cell
count, 1,800/µL (1.8
x109/L), with 37%
neutrophils, 22%
bands, 7%
metamyelocytes, 32%
lymphocytes, and 2% basophils
• Hemoglobin, 14.2 g/dL (142 g/L)
• Platelets, 122 x 103/µL (122 x 109/L)
Analysis of his cerebrospinal fluid shows the following:
• White blood cell count, 1,932/µL (1.9 x 109/L), with 78% neutrophils and 22%
lymphocytes
• Glucose, 10 mg/dL (0.6 mmol/L)
• Protein, 0.25 g/dL (2.5 g/L)
A Gram stain shows white blood cells and few gram-negative rods.
Results shown are from a computed tomography scan of the head (Item Q43).
Of the following, the MOST likely infectious agent causing this baby's illness is
A. Citrobacter koseri
B. group B Streptococcus
C. herpes simplex virus
D. Listeria monocytogenes
E. Pseudomonas aeruginosa

American academy of pediatrics 130


American Academy of Pediatrics PREP 2015

Item 43 Preferred Response: A


Brain abscesses in infants and children are very rare. As demonstrated in the computed
tomography scan of the neonate in this vignette, the presence of multiple brain abscesses is
uniquely associated with sepsis and meningitis caused by the enteric gram-negative organism
Citrobacter koseri (previously Citrobacter diversus) and some other gram-negative organisms
(eg, Cronobacter, Serratia marcescens). Salmonella species have also been associated with brain
abscesses in neonates.

Although all the other organisms listed are potentially associated with severe neonatal central
nervous system involvement, multiple abscesses are not typically seen in those infections.
Additionally, the other organisms listed are not gram-negative rods, except for Pseudomonas.
Group B Streptococcus is a gram-positive coccus, Listeria is a gram-positive rod, and herpes
simplex is a virus that would not be detected on gram stain. Pseudomonas is an extremely rare
cause of neonatal meningitis and is not specifically associated with brain abscesses.

Abscesses in the brain can arise as a complication of a number of infections through


hematogenous spread (eg, endocarditis [especially in children with cyanotic congenital disease]
or with sepsis and meningitis, as described above) or direct extension of an adjacent focus (eg,
sinusitis, mastoiditis, odontogenic infection). Trauma and surgical procedures (especially
neurosurgical intervention) are other predisposing factors for development of brain abscesses.
Initial empiric therapy depends on the focus of origin and the likely associated microbiology.

Treatment of brain abscesses generally includes a combination of antimicrobial therapy and


surgical drainage. A third generation cephalosporin such as cefotaxime or a carbapenem such as
meropenem are the usual agents of choice for invasive neonatal Citrobacter infection, although
there are reports of strains producing extended spectrum β-lactamases, which would cause them
to not be sensitive to third generation cephalosporins.

PREP Pearls
• Brain abscesses in infants and children are very rare.
• Brain abscesses can arise through hematogenous spread (eg, in endocarditis or sepsis and
meningitis) or direct extension (eg, complicating sinusitis, mastoiditis, or odontogenic
infection).
• Citrobacter koseri is relative uniquely associated with multiple brain abscesses in
neonates.

American Board of Pediatrics Content Specification(s)


• Recognize the clinical findings associated with brain abscess
• Plan the appropriate diagnostic evaluation of a brain abscess
• Plan the appropriate management of a brain abscess

Suggested Reading
• American Academy of Pediatrics. Escherichia coh and other gram-negative bacilli
(septicemia and meningitis in neonates). In: Pickering LK, Baker CJ, Kimberlin DW,
Long SS, eds. Red Book: 2012 Report of the Committee on Infectious Diseases. 29th ed.
Elk Grove Village, IL: American Academy of Pediatrics; 2012:321-324.
American academy of pediatrics 131
American Academy of Pediatrics PREP 2015

• Mann K, Jackson MA. Meningitis. Pediatr Rev. 2008;29(12):417-429.


doi:10.1542/nir.29-12-417.

American academy of pediatrics 132


American Academy of Pediatrics PREP 2015

Item 44
A 7-year-old boy presents to your office with painful urination of 1 day duration. His mother
mentions that for the last few weeks he has been frequently touching his genitalia, and for the
last few months, he has occasionally requested her to touch his genitalia. She attributes this to
age-appropriate curiosity. She denies any complaints of fever, trauma, blood in urine, increased
frequency, or suspicion of child abuse. Physical examination is significant for small vesicular
lesions on the glans.

Of the following, the MOST likely cause of this patient's condition is


A. local trauma
B. nonspecific (chemical) urethritis
C. sexual abuse
D. urinary tract infection
E. viral balanitis

American academy of pediatrics 133


American Academy of Pediatrics PREP 2015

Item 44 S Preferred Response: C


Various conditions associated with the inflammation of the genitourinary tract may present with
dysuria. The muscular contraction of the bladder and urethra, and contact of urine with inflamed
genitourinary mucosa, is the underlying cause of dysuria.

All of the response choices have dysuria as the presenting symptom. However, sexual abuse is
the most likely cause of this patient's symptoms. The boy in the vignette has both inappropriate
behavior and lesions on the penis, suggesting sexual abuse as the cause of the patient's
symptoms. A detailed history for possible sexual abuse should be obtained, despite the
caregiver's lack of suspicion.

Sexual abuse may lead to lower urinary tract infection. Urinary tract infection could explain the
patient's dysuria, but not the inappropriate behavior or the lesions on the glans.

Balanitis, inflammation of the glans penis, is usually seen in association with diaper dermatitis in
young children. Sexually transmitted infections (syphilis or herpes simplex virus infection), other
viruses (Epstein-Barr virus), and other systemic conditions (Behcet disease, Crohn disease) may
present with small vesicular lesions of the anogenital area but would not explain the
inappropriate behavior. Local trauma is not likely to produce vesicular lesions and urethritis
would not be expected to lead to the behavior described by the child in the vignette.

PREP Pearls
• Inappropriate behavior and genital lesions are suggestive of sexual abuse as the etiology
of dysuria in children.
• Balanitis, the inflammation of the glans penis, is usually seen in association with diaper
dermatitis in young children.
• Vesicular lesions of the anogenital area may be caused by sexually transmitted infections
(syphilis or herpes simplex virus infection), other viruses (Epstein-Barr virus), and other
systemic conditions (Beget disease, Crohn disease),

American Board of Pediatrics Content Specification(s)


• Recognize the etiology of dysuria in patients of various ages
• Plan the appropriate diagnostic evaluation of dysuria in patients of various ages

Suggested Reading
• Parrott TS. Cystitis and urethritis. Pediatr Rev. 1989;10(7):217-222. doi:10.1542/pir.10-
7-217.
• Roberts KB; Subcommittee on Urinary Tract infection, Steering Committee on Quality
Improvement and Management. Urinary tract infection: clinical practice guideline for the
diagnosis and management of the initial UTI in febrile infants and children 2 to 24
months. Pediatrics. 2011;128(3):595-610. doi:10.1542/peds.2011-1330.
• Supe-Markovina K, Kaskel FJ. Dysuria. In: Mclnerny TK, Adam 1-iM, Campbell DE,
Kamat DP, Kelleher KJ, eds. American Academy of Pediatrics Textbook of Pediatric
Care. 19th ed. Elk Grove Village, IL: American Academy of Pediatrics; 2009:1479-1482.

American academy of pediatrics 134


American Academy of Pediatrics PREP 2015

Item 45
A 4-year-old boy presents to your office because of poor growth. The child was diagnosed with
extrahepatic biliary atresia at 6 weeks of age and underwent a hepatoportoenterostomy (Kasai
procedure). Since that time, he has remained mildly jaundiced and has experienced 3 episodes of
presumed bacterial cholangitis. He is currently awaiting evaluation at a regional liver transplant
center. The child consumes a regular diet for age, including a daily multivitamin supplement. His
height and weight are at the 10th and 25th percentile, respectively. One year ago, his height and
weight were both at the 25th percentile. Physical examination findings show a small, active child
in no distress, icteric sclerae, a mildly protuberant abdomen, a firm liver edge palpated 1.5 cm
below the right costal margin, and a spleen tip palpated 2 cm below the Left costal margin. The
lower extremities show genu varum deformity. Laboratory study results include the following:
• Electrolytes, normal
• Calcium, 7.8 mg/dL (1.95 mmol/L)
• Phosphorus, 3.8 mg/dL (1.23 mmol/L)
• Total bilirubin, 6.5 mg/dL (111.2 mmol/L)
• Direct bilirubin, 4.0 mg/dL (68.4 mmol/L)
• Aspartate aminotransferase, 70 U/L (reference range, 20-60 U/L)
• Alanine aminotransferase, 90 U/L (reference range, 20-50 U/L)
• Alkaline phosphatase, 650 U/L (reference range,150-350 U/L)
Of the following, the MOST likely cause of this child's poor growth is
A. decreased energy intake
B. decreased intestinal absorption
C. impaired micellar solubilization
D. impaired nutrient hydrolysis
E. impaired re- esterification

American academy of pediatrics 135


American Academy of Pediatrics PREP 2015

Item 45 Preferred Response: C


The child with chronic liver diseases poses many therapeutic challenges, both for the primary
care provider and for the subspecialist. Malnutrition is common, and it may be related to
numerous problems directly related to dietary intake, macronutrient and micronutrient
absorption, and endogenous metabolism (Item C45A). Anorexia, disturbed hepatocellular
function, and cholestasis all play an important role in interfering with normal growth and weight
gain. In the vignette, this child presents with a deterioration in growth rate, a physical
examination that clearly suggests cholestatic jaundice (a common problem after biliary atresia
surgery), and a genu varum deformity characteristic of rickets. Laboratory studies confirm
hypocalcemia, and the elevated alkaline phosphatase indicates cholestasis or disturbed bone
mineralization. Although the etiology of skeletal and growth disturbances in this setting is likely
multifactorial, patients with cholestatic liver disease are at risk, particularly for calcium, fat, and
fat-soluble vitamin malabsorption as a consequence of reduced intraluminal bile salt
concentrations leading to impaired micellar solubilization.

Item C45A. Nutritional Causes of Growth Failure in Children with Cholestatic Liver Disease
• Anorexia
• Diarrhea (Zinc deficiency)
• Disturbed glycemic control
• Impaired duodenal alkalinization
• Increased energy expenditure
• Malabsorption
- Dietary fat
- Fat-soluble vitamins
- Calcium
• Portal-systemic shunting (interrupted hepatic metabolism)
• Protein loss (ascites)
• Reduced protein synthesis (chronic liver failure)

The assimilation of dietary lipids and lipid-soluble vitamins requires integrity of 4 critical phases
of intestinal absorption and transport:
• Pancreatic hydrolysis of triglyceride (intraluminal)
• Micellar solubilization (intraluminal)
• Re-esterification (intracellular)
• Lymphatic delivery

In cholestatic syndromes, phase 2 (micellar solubilization) is impaired because delivery of bile


salts to the intestine is insufficient to permit adequate micelle formation required for lipid
solubilization. The phases of intraluminal lipid hydrolysis and intracellular lipid re-esterification
are not disturbed. Similarly, intestinal epithelial cell absorptive function is normal in cholestatic
patients. Reduced energy intake may certainly contribute to impaired growth and weight gain
and represents a common problem in patients with chronic liver disease. In the vignette,
however, the clinical and laboratory findings strongly suggest that the child's poor growth is
related to malabsorption of lipid-soluble vitamin D, leading to vitamin D deficiency rickets.
Recent studies suggest that skeletal abnormalities in cholestasis also are related to impaired bone
formation (in contrast to enhanced bone resorption), despite the achievement of normal serum
American academy of pediatrics 136
American Academy of Pediatrics PREP 2015

25-dihydroxyvitamin D levels. Hypocalcemia may result from several factors, including


impaired vitamin D absorption and fecal excretion of dietary calcium as insoluble soaps formed
with malabsorbed fat.

The general dietary recommendations for children with cholestatic liver disease will depend on
the severity of hepatobiliary dysfunction. For many children with chronic liver disease, a
balanced diet, including appropriate energy, protein, and micronutrient intake for age and
activity level, will support adequate growth. Nevertheless, other patients will be hypermetabolic,
with disturbed hepatocellular metabolism in addition to fat malabsorption related to cholestasis.
For these children, estimated energy requirements in infants younger than 1 year of age may be
150% of that required by healthy children, while older children may require energy intakes
ranging from 120% to 170% of reference values.

In cholestatic patients, dietary supplementation with medium-chain triglycerides (MCTs) may be


required to maintain adequate nutritional status and growth. Because MCTs do not require
micellar solubilization, these lipid species are absorbed directly into the portal venous system.
Patients may require up to 70% of fat calories as MCTs, depending on their degree of biliary
dysfunction. However, care should be taken not to give more than 80% of total fat as MCTs, thus
increasing the risk of essential fatty acid deficiency.

Children with cholestatic liver disease will require supplements with fat-soluble vitamins A, D,
E, and K (Item C45B). All these micronutrients should be provided as water-soluble analogues.
Patients who remain vitamin D deficient despite oral supplements may require parenteral vitamin
D administration. Alternatively, orally administered 25-hydroxyvitamin D3 has been shown to
be effective in maintaining normal vitamin levels in children with chronic liver disease. Vitamin
E deficiency has conclusively been shown to cause severe neurologic dysfunction manifested by
hyporeflexia and cerebellar ataxia. In patients with cholestasis, vitamin E should be given as d-α-
tocopheryl polyethylene glycol-1000 succinate. in addition to providing water-soluble analogues
of the fat-soluble vitamins, dietary supplements of calcium and zinc are recommended for
cholestatic patients. As seen in the child in this vignette, calcium deficiency and bone
demineralization are common complications. Although calcium absorption will be improved by
normalizing vitamin D status, patients with cholestasis and osteopenia may require added dietary
calcium to fully saturate both vitamin D-dependent and vitamin D-independent intestinal trans-
port systems. Zinc deficiency is also a common problem in cholestasis, and, in addition to
causing diarrhea, may impair hepatic metabolic functions, including the urea cycle.
Item C45B. Fat Soluble Vitamin Supplementation in Children With Cholestatic Liver Disease*

Vitamin Recommended Intake (oral)


Vitamin A 5,000-25,000IU per day
Vitamin D 600-2,000 IU per day
Vitamin E 0-1 year 50-1001U per day
>1 year: 15-25 IU per kg per day
Vitamin K 2.5-10 mg daily or twice weekly (depending on coagulation status)
*All vitamins should be provided as water-soluble analogues (vitamin E as d-α-tocopheryl
polyethylene glycol-1000 succinate)

American academy of pediatrics 137


American Academy of Pediatrics PREP 2015

PREP Pearls
• Patients with cholestatic liver disease are at risk particularly for calcium, fat, and fat-
soluble vitamin malabsorption as a consequence of reduced intraluminal bile salt
concentrations leading to impaired micellar solubilization.
• In children with cholestasis, all vitamins should be provided as water-soluble analogues.
In particular, vitamin E should be provided as d-α-tocopheryl polyethylene glycol-1000
succinate.
• Infants and children with cholestasis often require 150% or greater of recommended
energy intake to maintain growth.
• Dietary fat intake of more than 80% medium-chain triglycerides will lead to essential
fatty acid deficiency.

American Board of Pediatrics Content Specification(s)


• Understand the nutritional causes of growth failure associated with chronic cholestatic
disease, including the effects of nutrient digestion and absorption
• Know the general dietary recommendations for hepatic disease
• Understand the mechanism of rickets associated with hepatic disease

Suggested Reading
• Argao EA, Heubi IE, Hollis BW, Tsang RC. d-a-tocopheryi polyethylene glycol-1000
succinate enhances the absorption of vitamin Din chronic cholestatic liver disease of
infancy and childhood. Pediarr Res. 1992;31M:146-150.
• Heubi IE. Heyman MB, Shulman RL The impact of liver disease on growth and nutrition.
J Pediatr Gastroenterol Mar. 2002:35(suppl 1)7555-S59.
• Heubi 3E, Hollis BW, Specker B, Tsang RC. Bone disease in chronic childhood
cholestasis, vitamin D absorption and metabolism. lieparalogy. 1989;9(2):258-264.
• Nightingale S, Ng VL. Optimizing nutritional management in children with chronic liver
disease. Pediarr Clin North Am, 2009;56(5)11161-1183. doi:10.1016/j.pc1.2009.06.005.
• Roberts CC, Book LS, Chan GM, Matlak ME, Herbst IL Rickets in children with
cholestatic liver disease: evaluation and treatment. Pedlar', Res. 1981;15;544.
doi:10.1203/00006450-198104001-00638.

American academy of pediatrics 138


American Academy of Pediatrics PREP 2015

Item 46
A 28-year-old woman arrives at your local hospital in active labor. She did not receive any
prenatal care. Multiple congenital anomalies are immediately noted upon delivery.
Physical examination is remarkable for a female newborn with 2 oval regions of scalp, with
noted absence of skin, bilateral cleft lip and palate, hypertelorism, small eyes, cyanosis with
mottling, and polydactyly. Cardiac examination reveals a systolic ejection murmur at the upper
left sternal border with a loud single S2. Pictures of the newborn (Item Q46A) and her hand
(Item Q46B) are shown.

Of the following, the MOST likely diagnosis is


A. CHARGE syndrome
B. holoprosencephaly
C. Meckel-Gruber syndrome
D. trisomy 13
E. trisomy 18

American academy of pediatrics 139


American Academy of Pediatrics PREP 2015

Item 46 IC P Preferred Response: D


The newborn described in the vignette has typical features of trisomy 13. These features include
skin defects of the posterior scalp, polydactyly, defects of the eye, cleft lip or palate,
holoprosencephaly, and cardiac defects. Other defects include hearing loss, microcephaly with
sloping forehead, wide fontanelles, microphthalmia, iris coloboma, low-set ears, hyperconvex
narrow fingernails, flexion of fingers with or without overlapping, hypospadias and
cryptorchidism in boys, bicornuate uterus in girls, and polycystic kidneys.

Of these infants, 82% die in the first month after birth. Most survive less than 15 days. One in 5
will survive for 3 months or more. Five percent to 10% will survive for 1 year or more. Survivors
typically have significant intellectual disability, seizures, and failure-to-thrive. Corrective
procedures for the heart and orthopedic issues are typically withheld in early infancy to see the
outcome of the first few months after birth because of the high infant mortality it is important to
emphasize the individuality of each case and acknowledge the personal feelings and wishes of
the parents.

Infants with trisomy 18 typically present with clenched hands with overlapping fingers,
increased muscle tone, micrognathia, low-set abnormal ears, congenital heart defects, small size
for gestational age, single umbilical artery, short palpebral fissures, nail hypoplasia, and short
sternum. Most have severe intellectual disability and limited capacity for survival.

CHARGE syndrome is an autosomal dominant disorder that is characterized by coloboma, heart


defects, choanal atresia, retarded growth and development, genital abnormalities, and ear
anomalies. They will often have multiple life-threatening medical conditions.

Holoprosencephaly is a structural anomaly of the brain resulting in incomplete separation of the


forebrain occur-ring in the first trimester. It is typically associated with developmental delay. it
can be the result of environmental causes, numeric chromosomal abnormalities (trisomy 13,
trisomy 18, triploidy), structural chromosomal abnormalities, copy number variants, single gene
mutations, and unknown causes.

Meckel-Gruber syndrome is typically composed of the triad of an occipital encephalocele, large


polycystic kidneys, and postaxial polydactyly. Some of these infants also have orofacial clefting,
central nervous system malformations, genitourinary anomalies, and pulmonary hypoplasia,
which is a leading cause of death in early infancy.

Recognition of a common aneuploidy and its associated anomalies is key to accurate family
counseling. Each case of trisomy 13 must be approached with a detailed counseling process with
the infant's family. This should include an accurate depiction of survival status, communication
of probable developmental outcome without the physician implying his or her own definition of
quality of life, and respect for the family's choices for interventions or comfort care.

Advanced maternal age is a well-defined risk factor for trisomy 13 and 18. While most mothers
who have infants with trisomies historically are younger than 35 years of age, the risk of having
an infant with an aneuploidy increases with advancing age.

American academy of pediatrics 140


American Academy of Pediatrics PREP 2015

PREP Pearls
• Infants with trisomy 13 typically have polydactyly, cleft lip or palate, holoprosencephaly,
scalp defects, and congenital heart defects.
• Infants with trisomy 18 typically have clenched hands with overlapping fingers, increased
muscle tone, micrognathia, low-set abnormal ears, congenital heart defects, small size for
gestational age, single umbilical artery, short palpebral fissures, nail hypoplasia, and
short sternum.
• Most infants with trisomy 13 and 18 have high infant mortality rates because of their
multiple congenital anomalies.

American Board of Pediatrics Content Specification(s)


• Recognize the clinical features associated with trisomy 13 in a neonate

Suggested Reading
• Carey JC. Perspectives on the care and management of infants with trisomy 18 and
trisomy 13: striving for balance. Curr Opin Pediatr. 2012;24(6):672-678.
doi:10.1097/M0P.0b013e3283595031.
• Carey JC, Cassidy SB, Allanson JE. The trisomy 18 and trisomy 13 syndromes. In:
Cassidy SB, Allanson JE, eds. Management of Genetic Syndromes. 3rd ed. Hoboken, NJ:
Wiley-Blackwell; 2010:807-823.
• Crider KS, Olney RS, Cragan JD. Trisomies 13 and 18: population prevalences,
characteristics, and prenatal diagnosis, metropolitan Atlanta, 1994-2003. Am J Med
Genet. 2008;146(7):820-826. doi:10.1002/ ajmg.a.32200.
• Wu J, Springett A, Morris JK. Survival of trisomy 18 (Edwards syndrome) and trisomy
13 (Patau syndrome) in England and Wales: 2004-2011. Am Med Genet Part A.
2013;161(10):2512-2518. doi:10.1002/ajmg.a.36127.

American academy of pediatrics 141


American Academy of Pediatrics PREP 2015

Item 47
On examination of a healthy term Asian male newborn, you notice a pinpoint indentation located
just anterior to the helix of his right ear. A thorough physical examination reveals no other
abnormalities. The mother reports no medical problems or medications used during her
pregnancy. Family history is negative for chronic kidney disease.

Of the following, the MOST important screening test is


A. audiologic evaluation
B. genetic testing
C. renal ultrasonography
D. serum creatinine
E. urinalysis

American academy of pediatrics 142


American Academy of Pediatrics PREP 2015

Item 47 Preferred Response: A


Similar to all newborns, infants with preauricular pits or sinuses should undergo hearing
screening and a formal audiologic evaluation should be scheduled if the screening result is
abnormal. The prevalence of hearing impairment among infants with preauricular skin tags or
pits is higher. These findings may be the first sign identifying a syndrome associated with
sensorineural hearing loss, so verifying that the result of the newborn hearing screen is normal is
the most important first step.

The incidence of renal anomalies in patients with isolated preauricular pits does not differ
significantly from that in the general population. Preauricular pits are more common in some
families and are most prevalent in the Asian population. Preauricular pits generally do not
require surgery unless they become repeatedly infected or drain. If surgery is performed, it is
important to thoroughly excise the entire pit, the squamous-lined cyst if present beneath the skin,
and the cartilage at the root to prevent recurrence.

External ear malformations and renal anomalies occur in a number of multiple congenital
anomaly syndromes. Patients with auricular anomalies should be examined carefully, specifically
looking for facial asymmetry, colobomas, choanal atresia, cardiac murmurs, branchial cysts or
sinuses, distal limb anomalies, and imperforate anus. Genetic testing and referral to specialists
would be indicated when these additional findings are present on physical examination.

Renal ultrasonography should be performed if the child has any other malformations or
dysmorphic features, family history of deafness, auricular or renal malformation or maternal
history of gestational diabetes. In the absence of these findings, renal ultrasonography is not
clearly indicated. Therefore, renal ultrasonography would not be recommended for the newborn
in the vignette who has a simple preauricular ear pit. Similarly, serum creatinine measurement
and urinalysis to evaluate the genitourinary tract are not necessary in patients with isolated
preauricular pits.

Patients with multiple or large preauricular skin tags has a higher risk for urinary tract and
hearing abnormalities than those with simple preauricular pits. Therefore, patients with
preauricular tags should undergo further evaluation.

PREP Pearls
• Infants with preauricular pits or sinuses should undergo newborn hearing screening and
receive a referral for formal audiologic evaluation if the screening result is abnormal.
• The presence of a simple preauricular pit does not warrant further evaluation with renal
ultrasonography or laboratory tests.

American Board of Pediatrics Content Specification(s)


• Know the significance of preauricular sinuses and pits

Suggested Reading
• Isaacson G. Congenital anomalies of the ear. UpToDate. Available online only for
subscription.

American academy of pediatrics 143


American Academy of Pediatrics PREP 2015

• Kohelet D, Arbel E. A prospective search for urinary tract abnormalities in infants with
isolated preauricular tags. Pediatrics. 2000;105(5):E61. doi:10.1542/peds.105.5.e61.
• Roth DA, Hildesheimer M, Bardenstein S, et al. Preauricular skin tags and ear pits are
associated with permanent hearing impairment in newborns. Pediatrics.
2008;122(4):e884-e890. doi:10.1542/peds.2008-0606.
• Wang RY, Earl DL, Ruder RO, Graham JM Jr. Syndromic ear anomalies and renal
ultrasounds. Pediatrics. 2001;108(2):E32. doi:10.1542/ peds.I08.2.e32.

American academy of pediatrics 144


American Academy of Pediatrics PREP 2015

Item 48
A 3-month-old boy is brought to your office for a rash and bloody diarrhea. This morning, the
mother noticed a pinpoint red rash on his face and body and then saw streaks of blood in his
stool. He drinks 24 oz to 32 oz of cow's milk formula per day. The baby is at 50th percentile for
height and weight. His temperature is 38.2°C, pulse rate is 120 beats/ min, respiratory rate is 24
breaths/min, and blood pressure is 90/55 mm Hg. On examination, the child is fussy but
consolable. There are petechiae on his face and waist, and erythematous dry maculopapular
patches on his cheeks and trunk. There is fluid behind an erythematous left tympanic membrane.
There are no anal fissures visible. The remainder of the physical examination is unremarkable.
The following are the results of the infant's laboratory tests:
• White blood cell count, 5,500/µL (5.5 x 109/L), with 40% polymorphonuclear leukocytes,
52% lymphocytes, 5% monocytes, and 3% eosinophils
• Hemoglobin, 13 g/dL (130 g/L)
• Mean corpuscular volume, 85 µm3 (85 fL)
• Platelet count, 45,000 x 103/µL (45 x 109/L)
• Mean platelet volume, 5 ft. (reference range, 6 fL-10 fL)

Of the following, the MOST appropriate next step in evaluating this patient is to
A. refer for allergy skin testing
B. refer for radioallergosorbent (RAST) testing
C. send gene testing for Wiskott-Aldrich syndrome
D. send platelet antibodies
E. send serum complement levels

American academy of pediatrics 145


American Academy of Pediatrics PREP 2015

Item 48 Preferred Response: C


The child in the vignette has features concerning for Wiskott-Aldrich syndrome (WAS).
Wiskott-Aldrich syndrome is an X-linked recessive disorder caused by mutations in the gene
encoding the WAS protein (WASp). The clinical features include thrombocytopenia with small
platelets (low mean platelet volume [MPV]), eczema, and immuno-deficiency. Mutations in the
WAS gene can cause a spectrum of diseases with varying severity, ranging from classic WAS
with autoimmunity and malignancy, to X-linked thrombocytopenia, a milder form with isolated
thrombocytopenia, as well as X-linked congenital neutropenia. The incidence of WAS is 1 in
100,000 live births, of which 50% have the classic WAS phenotype. Wiskott-Aldrich syndrome-
related disorders typically present in infancy with thrombocytopenia, bloody diarrhea, mucosal
bleeding, petechiae, purpura, eczema, and recurrent infections, particularly otitis media. More
than 40% of patients develop autoimmune cytopenias, arthritis, vasculitis, and kidney and
hepatic damage. Patients with WAS who are exposed to the Epstein-Barr virus have an increased
risk of developing atypical extranodal lymphomas with a very poor prognosis. For patients with
the classic WAS phenotype, bone marrow transplantation (BMT) is the only curative therapy.
Supportive care should be given as needed: topical corticosteroids and antibiotics (if infected) for
eczema, immunosuppressants for immune cytopenias, and granulocyte-colony-stimulating factor
and antibiotics for neutropenia.

The diagnosis of WAS should be considered in any male child with thrombocytopenia and small
platelet size, with any of the following diagnostic criteria: recurrent bacterial, viral or
opportunistic infection, eczema, lymphoma, autoimmune disorder, family history of a male on
the maternal side with a WAS-related disease, or decreased WASp level on flow cytometry or
Western blot. Early diagnosis is important in managing infections and bleeding symptoms, and
preparing for possible BMT. The likelihood of success with BMT declines as the patient
becomes older. Nonsteroidal anti-inflammatory agents should be avoided in any patient with an
increased risk for bleeding.

The boy in the vignette with bloody diarrhea, eczema, otitis media, and microthrombocytopenia
should be evaluated for WAS. Radioallergosorbent testing or skin testing for possible allergies
(eg, to milk protein), as the cause of his eczema and bloody stool, would not explain the
thrombocytopenia in this child. Platelet antibodies may be present in immune-mediated forms of
thrombocytopenia, however, the MPV is usually high because younger, newer platelets are larger
than older platelets. Serum complement levels may be decreased in certain autoimmune diseases
and immunodeficiencies, but this is not one of the specific diagnostic criteria for WAS.

PREP Pearls
• Wiskott-Aldrich syndrome (WAS)-related disorders typically present in infancy with
thrombocytopenia, bloody diarrhea, mucosal bleeding, petechiae, purpura, eczema, and
recurrent infections, particularly otitis media.
• Early diagnosis is important in managing infections and bleeding symptoms and
preparing for possible bone marrow transplantation.
• For patients with the classic WAS phenotype, bone marrow transplant is the only curative
therapy.

American academy of pediatrics 146


American Academy of Pediatrics PREP 2015

American Board of Pediatrics Content Specification(s)


• Recognize the clinical findings associated with Wiskott-Aldrich syndrome

Suggested Reading
• Filipovich AH, Johnson J, Zhang K. WAS-related disorders. Gene Reviews. Last updated
July 28, 2011. Accessed January 18, 2014.
• Nurden AT, Freson K, Seligsohn U. Inherited platelet disorders. Haemophilia.
2012;18(suppl 4):154-160. doi:10.1111/j.1365- 2516.2012.02856.x.

American academy of pediatrics 147


American Academy of Pediatrics PREP 2015

Item 49
A 14-year-old adolescent boy has been having a problem with recurrent headaches over the past
4 months. These headaches are occasionally associated with periumbilical abdominal pain. He
describes the headaches as typically occurring across his forehead or affecting his entire head.
The pain feels like a pressure, and is not pounding. Acetaminophen helps, but will often not
remove the headache entirely. The headaches happen as often as once a day and sometimes cause
him to miss school or not participate in other activities. There are cycles of time when the
headaches occur more or less frequently, but not an overall pattern of worsening. The patient has
otherwise been healthy, with normal development. His physical examination is within normal
limits, with a blood pressure of 115/80 mm Hg and a pulse rate of 88 beats/min.

Of the following, the BEST next step would be to


A. ask him about stressors in his life
B. ask him if he has had a history of recurrent sore throats
C. request a complete blood cell count and erythrocyte sedimentation rate
D. request computed tomography of the head
E. request Lyme disease serology

American academy of pediatrics 148


American Academy of Pediatrics PREP 2015

Item 49 IC Preferred Response: A


For the boy in the vignette, the near daily recurrence of headaches, the non-pounding nature of
the headaches, localization of headaches to his entire forehead or around his whole head, and the
association of headaches with episodes of periumbilical abdominal pain is all highly suspicious
of these being stress induced headaches. These tension variety headaches are very common
manifestation of stress.

Stress is well recognized to cause various physical symptoms. Common stress-related physical
complaints include headaches, nausea, chest pain, stomach aches, and fatigue. In only about 15%
of presentations of these physical complaints will an "organic" disorder be identified. Although
mental/ emotional stress is not a disorder in and of itself, biological pathways from feelings of
stress can lead to physical symptoms. These pathways include alterations in autonomic nervous
system functioning and changes in cortisol secretion. For instance, someone feeling intense stress
who presents with a medical complaint of intermittent numbness in the fingers and toes may
have anxiety-induced and sympathetic nerve-triggered hyperventilation, which then leads to
hypocapnea, which constricts distal blood vessels. The resulting decrease in peripheral perfusion
leads to the numbness sensations from the distal sensory nerves. Stress also causes cognitive
processing changes. Excessive stress can cause distraction and loss of concentration. Children
feeling stressed about schoolwork may decrease their performance, which further perpetuates
their stress.

Patients who present to their medical providers with stress-related physical symptoms, such as
headaches, often lack insight into stress as a possible cause. A helpful approach is to inquire
about both their physical symptoms and stressors in their environment at the same time, rather
than reserving the discussion of stress or anxious feelings until after a medical cause has been
"ruled out:' This will prevent unintentionally delivering a message that the physician only cares
about organic disorders, and the absence of an organic disorder would lead to disapproval or
dismissal from the provider's care and attention.

Although not one of the offered responses, migraine headaches are an unlikely cause of the
symptoms in the boy in the vignette, because migraine headaches in this age group are most
often associated with nausea, vomiting, phonophobia, and photophobia. Recurrent sore throats
and headaches could be a sign of infectious mononucleosis or Strep throat, however these are
unlikely to be "repeated" experiences. A complete blood count might be warranted if there was a
reason to suspect the presence of anemia or infection, but this would be an uncommon cause of
recurrent headache. The erythrocyte sedimentation rate is a nonspecific marker of inflammation
that even if found to be elevated would be unlikely to lead to a more specific diagnosis in this
scenario. Computed tomography of the head would not be warranted. The boy in this vignette
has no localized headache symptoms for which a specific structural brain lesion could be
suspected; unnecessary radiation exposure also should be avoided in a young person. Lyme
serology might be warranted if the boy lived in a geographic area where Lyme disease is
common, like the Northeast, and if he had other symptoms consistent with a Lyme disease
diagnosis such as a history of erythema migrans and flulike symptoms that preceded the onset of
joint pain.

American academy of pediatrics 149


American Academy of Pediatrics PREP 2015

PREP Pearls
• Clinical findings associated with stress include headaches, nausea, chest pain, stomach
aches, and fatigue.
• For patients presenting with recurrent headaches. nausea, chest pain, stomach aches. and
fatigue, organic causes are identified only about 15% of the time.
• When evaluating conditions that are commonly stress related, it is best to immediately
assess for stress and anxiety as contributors rather than waiting until organic causes are
ruled out.

American Board of Pediatrics Content Specification(s)


• Recognize the clinical findings associated with stress among adolescents

Suggested Reading
• Silber TJ, Pao M. Somatization disorders in children and adolescents. Pediatr Rev. w.
2003;24(8):255-264. doi:10.1542/pir.24-8-255.
• Kroenke K, Mangelsdorff AD. Common symptoms in ambulatory care: incidence,
evaluation, therapy, and outcome. Am I Med. 1989;86(3):262-266.

American academy of pediatrics 150


American Academy of Pediatrics PREP 2015

Item 50
An 11-year-old girl is brought to your office for a routine health supervision visit. She is doing
well academically, has good relations with her peers and parents, and has no complaints at this
time. She has not yet begun menses. Her weight is 43 kg (75th percentile) and her height is 145
cm (50th percentile). Her physical examination shows sexual maturity rating of 2 with all other
findings unremarkable. As part of her evaluation today, she has a screening total cholesterol
result of 220 mg/dL (5.70 mmol/L). You send her for fasting laboratory tests that show a low-
density lipoprotein cholesterol level of 120 mg/dL (3.11 mmol/L) and a high-density lipoprotein
cholesterol level of 55 mg/dL (1.42 mmol/L).

Of the following, the statement that MOST accurately describes her laboratory results is that

A. total cholesterol level is acceptable, low-density lipoprotein cholesterol level is high, and
high-density lipoprotein level is low
B. total cholesterol level is borderline, low-density lipoprotein cholesterol level is
acceptable, and high-density lipoprotein level is borderline
C. total cholesterol level is borderline, low-density lipoprotein cholesterol level is
borderline, and high-density lipoprotein level is high
D. total cholesterol level is high, low-density lipoprotein cholesterol level is acceptable, and
high-density lipoprotein level is low
E. total cholesterol level is high, low-density lipoprotein cholesterol level is borderline, and
high-density lipoprotein level is acceptable

American academy of pediatrics 151


American Academy of Pediatrics PREP 2015

Item 50 Preferred Response: E


Hyperlipidemia is a risk factor for adult-onset cardiovascular disease, including stroke and
ischemic heart disease. Other high-risk factors include family history of early atherosclerotic
heart disease (before age 55 years in men and age 65 years in women), body mass index greater
than the 97th percentile, diabetes mellitus, hypertension, smoking, renal disease, solid organ
transplantation, and Kawasaki syndrome with a history of aneurysms. According to the National
Health and Nutrition Examination Survey (NHANES) of 1999 to 2008, 16% of US children are
overweight, and 31% are at risk of becoming overweight because of decreased exercise,
increased sedentary activities, and increased consumption of sweetened drinks, which puts them
at risk for cardiovascular disease.

New guidelines from the National Heart Lung and Blood Institute and the American Academy of
Pediatrics recommend screening for hypercholesterolemia twice, once between the ages of 9 and
11 years and again between the ages of 17 and 21 years. Laboratory tests can be done without
fasting. Selective screening of children with a fasting lipid profile over age 2 years is
recommended when there is a family history of hypercholesterolemia or premature coronary
artery disease. Acceptable total cholesterol has been lowered to 170 mg/dL (4.4 mmol/L). Non-
high-density lipoprotein (HDL) cholesterol, which can also be measured without fasting, is
normal if less than 120 mg/dL (3.11 mmol/L). Acceptable triglycerides vary by age; for age 9
years and younger, it is 75 mg/dL (0.8 mmol/L) whereas for those aged 10 to 19 years, it is less
than 90 mg/dL (1.0 mmol/L). Thus, the girl in the vignette has a high total cholesterol level,
borderline low-density lipoprotein (LDL) cholesterol level, and acceptable HDL cholesterol
level.

Values that are definitely high and in need of further workup include total cholesterol of 200
mg/dL (5.18 mmol/L) and non-HDL cholesterol greater than 145 mg/dL (3.76 mmol/L) at any
age. Triglyceride levels greater than 100 mg/dL (1.1 mmol/L) in younger patients (0-9 years) and
130 mg/dL (1.5 mmol/L) in older patients (10-19 years) are high. Low-density lipoprotein
cholesterol is considered normal if less than 110 mg/dL (2.85 mmol/L) and high if greater than
130 mg/dL (3.37 mmol/L) for any age. High-density lipoprotein cholesterol is considered normal
if greater than 45 mg/ dL (1.17 mmol/L) at any age and low if less than 35 mg/dL (0.91 mmol/L)
at any age.

PREP Pearls
• Total cholesterol over 200 mg/dL (5.18 mmol/L), low-density lipoprotein cholesterol
over 130 mg/dL (3.37 mmol/L), or high-density lipoprotein cholesterol less than 35
mg/dL (0.91 mmol/L) are always considered abnormal at any age.
• Hyperlipidemia is a risk factor for adult-onset cardiovascular disease, including stroke
and ischemic heart disease.
• New guidelines from the National Heart Lung and Blood Institute and the American
Academy of Pediatrics recommend screening for hypercholesterolemia twice, once
between the ages of 9 and 11 years and again between the ages of 17 and 21 years.

American Board of Pediatrics Content Specification(s)


• Understand the importance of cholesterol/lipid screening examinations

American academy of pediatrics 152


American Academy of Pediatrics PREP 2015

• Recognize the clinical features associated with hypercholesterolemia/ hyperlipidemia,


and evaluate appropriately
• Know the risk factors associated with hypercholesterolemia/ hyperlipidemia
Suggested Reading
 Expert Panel on Integrated Guidelines for Cardiovascular Health and Risk Reduction in
Children and Adolescents. Expert panel on integrated guidelines for cardiovascular health
and risk reduction in children and adolescents: summary report. Pediatrics.
2011;128(suppl 5):5213-5256. doi:10.1542/peds.2009-2107C.
 Gidding SS. New cholesterol guidelines for children? Circulation. 2006;114:989-991.
doi:10.1161/CIRCULATIONAHA.106.647891.
 May AL, Kuklina EV, Yoon PW. Prevalence of cardiovascular disease risk factors
among US adolescents, 1999-2008. Pediatrics. 2012:1035-1041. doi:10.1542/peds.2011-
1082.
 NCEP Expert Panel on Blood Cholesterol Levels in Children and Adolescents. National
Cholesterol Education Program for Children (NCEP): highlights of the report of the
expert panel on blood cholesterol levels in children and adolescents. Pediatrics.
1992;89(3):495-501. Accessed January 15, 2014.
 Neal W. Disorders of lipoprotein metabolism and transport. In: Kleigman RM, Behrman
RE. Jenson HB, Stanton B, eds. Nelson Textbook of Pediatrics. 18th ed. Philadelphia,
PA: Saunders Elsevier; 2007:86.3.

American academy of pediatrics 153


American Academy of Pediatrics PREP 2015

Item 51
A 2-year-old boy comes in to your clinic with a new-onset rash. The rash started yesterday on
both palms and spread to the arms and the back of the hands bilaterally. The mother denies any
scratching and states that the boy has been active and playful. On physical examination, you
observe a well-appearing child with dull red, flat macules, with a targetlike appearance of
concentric circles and marginated wheals, over the palms and arms (Item Q51). He has some
mild lip swelling, crusting, and bullae on his buccal mucosa, with no gingival involvement. You
note in his chart that he had an episode of herpes labialis 2 weeks ago.

Of the following, the patient's MOST likely diagnosis is


A. erythema marginatum
B. erythema multiforme
C. herpes simplex infection
D. serum sickness
E. Stevens-Johnson syndrome

American academy of pediatrics 154


American Academy of Pediatrics PREP 2015

Item 51 Preferred Response: B


The boy in the vignette has classic dull red targetoid lesions over the extensor surfaces of his
arms associated with lip swelling and crusting, and bullae on his buccal mucosa consistent with
erythema multiforme (EM). He is well appearing. His history of recent herpetic infection is also
a clue to his diagnosis. EM is an immune-mediated hypersensitivity reaction most commonly
associated with herpes simplex virus, mycoplasma pneumonia, nonsteroidal anti-inflammatory
drugs, sulfonamides, antiepileptics, and antibiotics. EM is characterized by the classic target
lesions with or without mucous membrane involvement. The labial mucosa, buccal mucosa, and
gingivae are most commonly involved. Mucosal lesions begin as localized swelling with
progression to superficial erosions and pseudomembranes. There are no specific laboratory tests
for EM; however, patients often will have elevated white blood cell counts, erythrocyte
sedimentation rates, and liver enzymes. EM is self-limiting, so treatment includes removal of the
inciting agent if identified. Patients with Mycoplasma infection should be treated. The typical
course of the skin eruption is appearance over 3 to 5 days with resolution by 2 weeks. Mucosal
involvement may take longer to resolve and these patients should receive supportive care.

Much of the published literature discusses a spectrum of EM that includes EM minor, EM major,
Stevens-Johnson syndrome (SJS), and toxic epidermal necrolysis (TEN). More recent literature
draws a clear distinction between EM and SJS. EM major is distinguished from SJS in that EM
has typical target lesions with 3 concentric rings over the limbs with or without blistering,
whereas SJS has purpuric macules and blisters distributed over the trunk and face. EM major
usually has less severe mucous membrane involvement. SJS and TEN are a clinical spectrum and
can have targetlike lesions that have concentric erythematous rings and a central lesion, but there
are usually numerous confluent lesions. Histologically, EM major has more mononuclear cells at
the dermal interface, and less necrosis of the epidermal cells than SJS and TEN.

Stevens-Johnson syndrome and TEN are a clinical spectrum of immune-mediated


hypersensitivity reactions presenting as a febrile, erosive stomatitis, with ocular involvement,
and a widely distributed rash of dark macules with a necrotic center. Other skin lesions seen in
this spectrum include violaceous macules, flat atypical target lesions, bullae, and erosions. TEN
manifests with extensive loss of epidermis secondary to necrosis. Cases with more than 30%
body surface area affected by epidermal detachment are considered TEN, whereas SJS will have
less than 10% body surface area involvement. Mortality in SJS is less than 5%, whereas
mortality in TEN is close to 30%. Histologically, both SJS and TEN present with a lymphocytic
infiltrate at the dermal epidermal junction and superficial perivascular region. Full-thickness
epidermal necrosis and subepidermal blistering can be present, which can distinguish it from
other diagnoses.

Most cases of SJS and TEN are associated with drugs. The most common medications associated
with pediatric SJS are sulfonamides, anticonvulsants (phenobarbital, lamotrigine, valproic acid,
and carbamazepine), acetaminophen, and non-steroidal anti-inflammatory drugs. Treatment of
SJS and TEN is controversial. The most important step in treatment is discontinuation of the
inciting drug and supportive care until the mucosal lesions have healed. In TEN, early transfer to
a burn unit has been shown to improve survival. Early ophthalmologic evaluation and treatment
may help prevent vision loss. There is some evidence that the use of intravenous
immunoglobulin may be beneficial in both SJS and TEN. The use of systemic steroids in SJS has
American academy of pediatrics 155
American Academy of Pediatrics PREP 2015

been widely reported and although steroids may not decrease mortality, they may shorten the
course of the disease. Steroid use in TEN is even more controversial, with some studies showing
an increase in mortality. Other treatments used for SJS and TEN include plasmapheresis,
cyclosporine, cyclophosphamide, granulocyte colony-stimulating factor ,infliximab and
thalidomide

Serum sickness-like reactions present with pruritic urticarial rashes and arthralgias
approximately 1 week after drug exposure. These are most often associated with cefaclor, and
somewhat less often with penicillins, other cephalosporins, sulfonamides, macrolides, and
bupropion. Treatment is discontinuation of the offending agent, and symptomatic treatment with
antihistamines and nonsteroidal anti-inflammatory drugs for the pruritus and arthralgias.

Erythema marginatum is the rash associated with acute rheumatic fever. It is described as pink to
red circumscribed macules with central clearing, and would present along with the Jones criteria
and recent history of group A streptococcal infection.

PREP Pearls
• The first step in treating erythema multiforme is to discontinue the offending drug or treat
the inciting infection if still present.
• The first step in treating both Stevens-Johnson syndrome and toxic epidermal necrolysis
is discontinuation of the offending drug.
• Steroids and intravenous immunoglobulin have been shown to be effective in treating
Stevens-Johnson syndrome.

American Board of Pediatrics Content Specification(s)


• Recognize the clinical spectrum of erythema multiforme
• Recognize the clinical features of Stevens-Johnson syndrome and manage appropriately

Suggested Reading
• Bachot N. Roujeau JC. Differential diagnosis of severe cutaneous drug eruptions. Am I
Clin Dermatol. 2003;4(8):561-572. doi:10.2165/00128071- 20030080-00006.
• Worswick S, Cotliar J. Stevens-Johnson syndrome and toxic epidermal necrolysis: a
review of treatment options. Dermatol Then 2011;24(2):207218. doi:10.1111/j.1529-
8019.2.011.01396.x.

American academy of pediatrics 156


American Academy of Pediatrics PREP 2015

Item 52
During the month of August, while you are acting as one of the medical providers for a
marathon, volunteers carry a 16-year-old male adolescent into the medical tent. The volunteers
report that he collapsed, unwitnessed, right before the finish line. On initial examination, the
patient is mumbling and appears disoriented. His heart rate is 120 beats/ min, and his respiratory
rate is 20 breaths/min. His skin is cool and dry to the touch, and he is shivering.

Of the following, the BEST next step would be to

A. begin immediate rapid cooling


B. obtain a blood glucose reading
C. obtain a rectal temperature reading
D. place an intravenous line and deliver a normal saline bolus
E. place the patient in a supine position with his legs elevated

American academy of pediatrics 157


American Academy of Pediatrics PREP 2015

Item 52 Preferred Response: C


The boy in the vignette collapsed at the end of a long endurance event during the month of
August. Heat illness is the most likely cause of his collapse. The boy's heart rate and respiratory
rate do not suggest the need for cardiopulmonary resuscitation. Obtaining a rectal temperature is
a quick and accurate method of confirming an elevated core temperature and will not cause a
significant delay in initiating treatment for heat illness.

There are several reasons why an athlete may experience collapse during or immediately after
exercise. Collapse during exercise suggests a more serious cause. Symptomatic heat illness
occurs with a core temperature of 40°C (104°F) or higher. With exertional heatstroke, affected
individuals have a temperature above 40.5°C (104.9°F) and mental status changes such as
confusion and disorientation. Hypotension, tachypnea, sustained tachycardia, vomiting, and
decreased sweating with dry skin are also frequently seen.

Heat-related illness should be treated with rapid cooling to a body temperature of 38°C (100°F).
At mass participation sporting events, ice water bath immersion is the preferred method of rapid
cooling. The boy in the vignette will likely need rapid cooling but this should be implemented
only after confirming an elevated core temperature. If supplies are available, placement of an
intravenous line and subsequent fluid bolus may be considered after initiating cooling. Patients
that do not immediately respond to treatment should be transported emergently to the nearest
hospital.

Exercise-associated collapse generally occurs at the end of a long athletic event after a
participant voluntarily stops the activity. Venous pooling in the lower extremities leads to
transient hypotension and subsequent collapse. Affected individuals report feeling dizzy or faint
prior to collapse. Treatment of exercise-associated collapse consists of placing the athlete in a
supine position with the legs elevated and offering oral fluids. The boy in the vignette seems to
have collapsed during the race, therefore it is less likely that he experienced exercise-associated
collapse.

Hyponatremia is another possible cause of confusion and loss of consciousness. During a long
endurance event, excess consumption of water can lead to hyponatremia. Clinicians should also
consider arrhythmia, hypoglycemia, and seizure as possible causes of collapse after ruling out
heat illness.

PREP Pearls
• Rectal temperature is a quick and accurate method of assessing for the presence of heat-
related illness.
• A patient with heat-related illness should undergo rapid cooling to reduce body
temperature to 38°C (100°F).

American Board of Pediatrics Content Specification(s)


• Recognize the clinical findings associated with heat illness, including complications
• Plan the appropriate evaluation of heat illness, and manage appropriately
• Understand the mechanisms of heat-related illness, including age-related factors

American academy of pediatrics 158


American Academy of Pediatrics PREP 2015

Suggested Reading
• Brukner P, Khan K. Brukner 6. Khan's Clinical Sports Medicine. 4th ed. North Ryde,
Australia: McGraw-Hill Medical; 2012.
• Council on Sports Medicine and Fitness and Council on School Health, Bergeron MF,
Devore C, Rice SG. Policy statement: climatic heat stress and exercising children and
adolescents. Pediatrics. 2011;128(3):e741-e747. doi:10.1542/peds.2011-1664.

American academy of pediatrics 159


American Academy of Pediatrics PREP 2015

Item 53
A previously healthy 2-year-old boy comes to your office with a 3-day history of nonbilious
vomiting and nonbloody diarrhea. The parents report that he has vomited 4 times and had
approximately 10 watery stools in the last 24 hours. The boy's mother, father, and sibling also
have been ill with similar symptoms. On physical examination, his temperature is 37.5°C, heart
rate is 140 beats/min, respiratory rate is 30 breaths/min, and blood pressure is 98/72 mm Hg. The
boy appears tired and ill but not toxic. His mucous membranes are tacky and capillary refill is 2
to 3 seconds. He is tachycardic with strong peripheral pulses. His abdomen is soft, nondistended,
and mildly tender to palpation throughout without rebound or guarding. Bowel sounds are
hyperactive. The remainder of his physical examination is unremarkable.

Of the following, the finding MOST likely to be associated with this patient's illness is
A. a positive stool for ova and parasites
B. a positive stool toxin assay
C. a stool pH greater than 6
D. the presence of fecal leukocytes
E. the presence of stool reducing substances

American academy of pediatrics 160


American Academy of Pediatrics PREP 2015

Item 53 Preferred Response: E


The boy described in the vignette has acute gastroenteritis without bloody stools, most likely
caused by a virus. Viral gastroenteritis is generally a clinical diagnosis. It produces an osmotic
diarrhea demonstrated by positive stool-reducing substances and a stool pH of less than 6.
Although there are many possible etiologies of acute gastroenteritis in children (Item C53),
viruses are the most common in both developed and developing countries. With the
implementation of rota-virus immunization in young children in the United States, acute
gastroenteritis caused by this pathogen has become much less common than other viral etiologies
in persons of all ages.
Causes of Acute Gastroenteritis in Childs
Viruses Bacteria Protozoa Helminth
Astroviruses Campylobacter jejuni Cryptosporidium Strongyloides
stercoralis
Caliciviruses Escherichia coli Entamoeba histolytica
(Norovirus and (enteropathogenic and
Sapovirus) shiga toxin
producing)
Enteric adenoviruses Salmonella species Giardia lamblia
Enteroviruses Shigella species
Rotavirus Vibrio cholerae
Yersinia
enterocolitica
Modified and reprinted with permission from Granado-Villar D et al. Acute gastroenteritis.
Pediorr Rev. 2012;33(11):488

The symptoms of viral gastroenteritis include vomiting and diarrhea, often accompanied by
fever, abdominal cramping, anorexia, and myalgia. Diarrhea caused by viruses is watery and
nonbloody. Vomiting and diarrhea can occur up to 20 times a day. Typical duration of illness is 2
to 7 days depending on the infecting pathogen. Children young than 5 years of age, on average,
have 1 to 5 episodes of acute viral gastroenteritis each year. Viral gastroenteritis accounts for
more than 95% of hospitalizations for gastroenteritis this age group.

Acute gastroenteritis caused by bacterial pathogens can occur at any age, and usually is
accompanied by mucus; invasion of the intestines, resulting in inflammation. Blood diarrhea can
occur and a stool sample may reveal the prevalence of fecal leukocytes or red blood cells.
Clostridium difficile infection after antibiotic exposure is more common in older children and
adults. The diagnosis is made by detecting C difficile toxin or organisms in stool. Gastroenteritis
cause( by protozoa and helminths usually results in a protracted and intermittent disease,
characterized by nonspecific abdominal pain and malabsorption (foul-smelling stools,
flatulence), in addition to vomiting and diarrhea.

PREP Pearls
• Viral gastroenteritis is generally a clinical diagnosis. It produces an osmotic diarrhea
demonstrated by positive stool-reducing substances and a stool pH of less than 6.

American academy of pediatrics 161


American Academy of Pediatrics PREP 2015

• Viruses are the most common cause of acute gastroenteritis, especially in young children,
in both developed and developing countries.
• Since the introduction of routine immunization, rotavirus is a less common cause of viral
gastroenteritis than other pathogens.

American Board of Pediatrics Content Specification(s)


• Identify the pathogens commonly associated with infectious diarrhea in patients of
various ages

Suggested Reading
 American Academy of Pediatrics. Adenovirus infections. In: Pickering LK, Baker CJ,
Kimberlin OW, Long SS, eds. Red Book: 2012 Report of the Cot mrittee on Infectious
Diseases. 29th ed. Elk Grove Village, IL: American Academy of Pediatrics; 2012:220-
222.
 American Academy of Pediatrics. Astrovirus infections. In: Pickering LK, Baker CI,
Kimberlin DW, Long SS, eds. Red Book: 2012 Report of the Committee on Infectious
Diseases. 29th ed. Elk Grove Village, IL: American Academy of Pediatrics; 2012:243-
244.
 American Academy of Pediatrics. Enterovirus (nonpoliovirus) and parechovirus
infections. In: Pickering LK, Baker C), Kimberlin DW, Long SS, eds. Red Book: 2012
Report of the Committee on Infectious Diseases. 29th ed. Elk Grove Village, IL:
American Academy of Pediatrics; 2012:315-318.
 American Academy of Pediatrics. Human calicivirus infections. in: Pickering LK, Baker
CI, Kimberlin DW, Long SS. eds. Red Book: 2012 Report of the Committee on
Infectious Diseases. 29th ed. Elk Grove Village, IL: American Academy of Pediatrics;
2012:261-262.
 American Academy of Pediatrics. Rotavirus. In: Pickering LK, Baker CJ, Kimberlin DW,
Long SS, eds. Red Book: 2012 Report of the Committee on Infectious Diseases. 29th ed.
Elk Grove Village, IL: American Academy of Pediatrics; 2012:626-629.
 Cox E, Christenson JC. Rotavirus. Pediatr Rev. 2012;33(10):439-445.
doi:10.1542/pir.33-10-439.
 Granado-Villar D, Cunill-De Sautu B, Granados A. Acute gastroenteritis. Pediatr Rev.
2012;330 0:487-495. doi:10.1542/pir.33-11-487.

American academy of pediatrics 162


American Academy of Pediatrics PREP 2015

Item 54
A frantic mother brings her 3-year-old son to the emergency department after he ingested an
unknown quantity of vitamin tablets containing iron. Approximately 45 min before arrival, his
mother discovered him playing with tablets from a recently purchased bottle of prenatal
vitamins. The bottle originally contained 50 tablets. The child's mother inadvertently left the
bottle open on the bathroom counter while she ran into the nursery across the hall to check on the
boy's newborn sister, who was crying. When questioned, the boy told his mother that he did eat
some of the tablets before she discovered him playing with the bottle. She is not sure how many
tablets he ingested, but she estimates that as many as 15 tablets could be missing from the bottle.
His mother states that he has displayed no symptoms since she found him ingesting the prenatal
vitamins and has acted normally. The boy has no significant past medical history, takes no
medications, and has no allergies.

As you are interviewing the boy's concerned mother, you note that he appears very well. He is
happily pretending to read a storybook to his newborn sister. The boy's vital signs are normal for
his age, and a complete physical examination yields no remarkable findings.

Of the following, the BEST next step in management of this patient is to


A. administer activated charcoal solution orally
B. administer deferoxamine intravenously
C. administer of syrup of ipecac orally
D. discharge home with appropriate anticipatory guidance
E. obtain an abdominal radiograph of the abdomen

American academy of pediatrics 163


American Academy of Pediatrics PREP 2015

Item 54 Preferred Response: E


The young boy in the vignette seeks medical attention following suspected ingestion of multiple
prenatal vitamin tablets. Although he is asymptomatic and appears well, he is still at risk for
potentially significant toxicity from iron ingestion; obtaining an abdominal radiograph-along
with laboratory studies-is the best next step in his treatment.

Iron poisoning is one of the most common potentially lethal intoxications in children; all
pediatric providers should be familiar with its clinical manifestations and management. In the
United States, nearly 16,000 iron exposures in children younger than 6 years of age are reported
annually. The most serious of these exposures typically involve prenatal vitamins and pure iron
preparations containing ferrous sulfate tablets, which contain more elemental iron per tablet (60-
65 mg) than other iron preparations. These tablets are attractive to children because they are
often brightly colored, coated with sugar, and look like candy. One case-control study identified
the recent birth of a sibling as a risk factor for iron poisoning in children.

Toxicity arising from iron exposure depends on the amount of elemental iron ingested. The
minimum toxic dose and lethal doses of iron are not well established. Ingestions of as little as 20
mg/kg of iron have caused toxicity, whereas ingestions of more than 50 mg/kg often result in
toxicity. In many situations involving unintentional ingestion of iron-containing medications by
children, the exact number of tablets ingested cannot be determined. As few as ten 300-mg
ferrous sulfate tablets have been fatal to a young child.

Iron is toxic to numerous cellular processes in its ferric form. The main pathophysiologic
mechanism for iron toxicity is free radical production and lipid peroxidation, which results in
damage to the body's tissues. Local toxicity (manifested as abdominal pain, vomiting, diarrhea,
and gastrointestinal bleeding results from damage to intestinal mucosa

The clinical manifestations of iron poisoning in children are most often described in terms of 5
characteristic stages that are typically sequential, but can overlap.

Stage 1 (0-6 hours after ingestion): Direct injury to the GI mucosa can lead to vomiting, diarrhea,
abdominal pain, hematemesis, and hematochezia. In severe cases, hypovolemic shock, metabolic
acidosis, and even death can occur during this phase because of loss of GI fluid and blood. Many
patients with mild to moderate toxicity do not progress beyond this stage.

Stage 2 (6-12 hours after ingestion): Known as the "relative stability" or "quiescent" stage,
patients appear to be improving clinically during this period. Symptoms may fully resolve in
some cases as free iron redistributes into the reticuloendothelial system. It is crucial to
differentiate children in this phase from those with mild toxicity in whom symptoms have truly
resolved. A blood gas analysis may indicate mild metabolic acidosis.

Stage 3 (12-24 hours after ingestion): Severe systemic toxicity may develop in cases of severe
iron poisoning. Signs may include gastrointestinal hemorrhage, altered mental status, seizures,
cardiovascular collapse, renal failure, hepatic failure, coagulopathy, and profound metabolic
acidosis.

American academy of pediatrics 164


American Academy of Pediatrics PREP 2015

Stage 4 (within 48 hours after ingestion): Dose-dependent hepatotoxicity may develop, with
elevation of transaminase concentrations because of the toxic effects of iron on the liver.
Following shock, hepatic failure is the second most common cause of death from iron poisoning.

Stage 5 (2-8 weeks after ingestion): Gastrointestinal obstruction may occur because of strictures
that developed from iron-induced damage to the GI tract. Patients may present with abdominal
pain, vomiting, and findings indicative of GI obstruction, which typically occurs at the gastric
outlet.

Although most children ingesting iron-containing products develop only mild symptoms or no
symptoms at all, clinicians should consider them to be at risk for developing life-threatening
toxicity until a thorough evaluation proves otherwise. Although no minimal "safe" quantity of
ingested iron has been established, serum iron levels generally correlate with the likelihood of
developing toxicity. When measured approximately 4 hours after ingestion, iron levels below
350 µg/dl (62.6 µmol/L) generally predict an asymptomatic or very mild course. Children with
iron levels in the 350 to 500 µg/dL (62.6-89.5 µmol/L) range often show mild to moderate
gastrointestinal symptoms, but rarely develop serious complications, whereas those with levels
greater than 500 µg/dL (89.5 µmol/L) are at risk of developing serious systemic toxicity.

The management of acute iron poisoning cases begins with aggressive supportive care, with a
focus on supporting the airway, breathing, and circulation. Fluid resuscitation to help correct
hypovolemic shock is crucial in children with severe toxicity.

Key diagnostic tests for children presenting after known or suspected iron ingestion include a
serum iron level and an abdominal radiograph. Serum iron levels typically peak 2 to 6 hours after
ingestion; therefore, most experts recommend 4 hours after ingestion as an optimal time to obtain
this level.

Abdominal radiography can help to direct the initial treatment of the child presenting after iron
ingestion; if radiography findings are negative and the patient is asymptomatic, no immediate
interventions are indicated. If iron tablets are identified on radiography, however, GI
decontamination via whole bowel irrigation is generally indicated. It is important to note that
although iron is radio opaque, negative findings on radiography do not absolutely rule out iron
ingestion. An arterial blood gas determination is also useful in children known or suspected to
have ingested significant amounts of iron because the finding of metabolic acidosis in a
seemingly well-appearing child can serve as a warning sign for the development of shock.

Although the administration of activated charcoal is indicated after most pediatric ingestions, it is
not effective or indicated in poisonings with heavy metals such as iron; therefore, it would not be
the best next step for the boy in the vignette.

Administration of intravenous deferoxamine, the best known antidote to iron poisoning, would
not be the best next step for the boy in the vignette. Chelation therapy with deferoxamine is
indicated in children with significant clinical manifestations of iron poisoning, serum iron level
higher than 500 µg/dL (89.5 µmol/L), or metabolic acidosis. Deferoxamine is associated with
significant adverse effects, including hypotension, renal failure, acute respiratory distress
American academy of pediatrics 165
American Academy of Pediatrics PREP 2015

syndrome (ARDS), and Yersinia sepsis; therefore, administration would not be recommended in
this asymptomatic boy before determination of serum iron level.

Controversy exists as to whether ipecac has any place in the treatment of poisoned children.
Ipecac administration would not be the best next step for this patient.

Although discharging the boy from the hospital with appropriate anticipatory guidance may be
the ultimate resolution of this case if he remains asymptomatic and significant iron toxicity is
ruled out, a thorough evaluation should be made, including laboratory and radiologic studies as
described earlier.

PREP Pearls
• Key diagnostic tests to guide the treatment of children presenting after known or
suspected iron ingestion include a serum iron level and abdominal radiography.
• Although most children ingesting iron-containing products develop only mild symptoms
or no symptoms at all, clinicians should consider them to be at risk for developing life-
threatening toxicity until a thorough evaluation proves otherwise.
• Management of acute iron poisoning cases begins with aggressive supportive care, with a
focus on supporting the airway, breathing, and circulation. Fluid resuscitation to help
correct hypovolemic shock is crucial in children with severe toxicity.

American Board of Pediatrics Content Specification(s)


• Plan the management of a patient who has ingested iron pills

Suggested Reading
 Chang TP, Rangan C. Iron poisoning: a literature-based review of epidemiology,
diagnosis, and management. Pediatr Emerg Care. 2011;27(10):978-985.
doi:10.1097/PEC.0b013e3182302604.
 Liebelt EL, Kronfol R. Acute iron poisoning. UpToDate. Available onlit only for
subscription.
 Osterhoudt KC, Ewald MB, Shannon M, Henretig FM. Toxicologic emergencies. In:
Fleisher GR, Ludwig S. eds. Textbook of Pediatric Emergency Medicine. 6th ed.
Philadelphia, PA: Lippincott Williams & Wilkins; 2010:1171-1223.
 Woolf AD. Poisoning in children and adolescents. Pediatr Rev. 1993;14(10:411-422.
doi:10.1542/pir.14-11-411.

American academy of pediatrics 166


American Academy of Pediatrics PREP 2015

Item 55
You are called to assess a term newborn with tachypnea and temperature instability 2 hours after
delivery. The maternal history is notable for 17 hours of ruptured membranes and unknown
group B streptococcal status. Examination reveals a tachypneic infant with delayed capillary
refill, mild grunting, and weak pulses. Vital signs include a heart rate of 180 beats/min,
respiratory rate of 80 breaths/min, and a blood pressure of 50/20 mm Hg. The arterial blood
sample values include a pH of 7.25, a PCO2 of 38 mm Hg, a PO2 of 70 mm Hg, bicarbonate of
16 mEq/L (16 mmol/L), lactate of 90.09 mg/dL (10 mmol/L), hemoglobin of 12 g/dL (120 g/L),
and ionized calcium of 4 mg/dL (1 mmol/L). You initiate evaluation and treatment for possible
sepsis.

Of the following, the MOST appropriate next step in management is to


A. administer hydrocortisone
B. bolus normal saline
C. infuse calcium gluconate
D. infuse sodium bicarbonate
E. transfuse packed red blood cells

American academy of pediatrics 167


American Academy of Pediatrics PREP 2015

Item 55 Preferred Response: B


The newborn in the vignette has septic shock and require volume expansion with normal saline
to improve tissue perfusion. The history of 17 hours of ruptured membranes and temperature
instability suggest neonatal sepsis, with the clinical findings of tachycardia, poor perfusion, and
hypotension supporting evolving shock. The rapid recognition and treatment of septic shock,
including the restoration adequate tissue perfusion, is essential to improving outcome in affected
neonates.

Defining adequate perfusion in a neonate is difficult. Blood pressure continues to be the


hemodynamic indicator of systemic perfusion, though it may not reflect actual tissue perfusion
and oxygen delivery. The goal of maintaining the mean blood pressure greater than the
gestational age in the first day after birth still remains the standard at many centers. Clinical
indicators, including weak distal pulses and a capillary refill time greater than 2 seconds, may be
seen with decreased tissue perfusion even before hypotension develops. Biochemical markers of
the cellular effects of inadequate tissue perfusion include an increased serum lactate level and
metabolic acidosis. Perfusion to the individual organs that falls below the ability of the organ to
maintain blood flow will lead to decreased renal, hepatic, cardiac, and brain function. Increased
oxygen consumption in an infant with septic shock may also lead to hypoglycemia if not
recognized and treated.

The initial step in the treatment of decreased perfusion and hypotension associated with septic
shock in a full-term neonate or older preterm infant remains volume expansion with a crystalloid
fluid bolus (normal saline or lactated Ringer solution). Recommendations for a preterm infant
may differ based on center of care. Packed red blood cells should be reserved for the infant with
evidence of acute volume loss or a hemoglobin value of 10 g/dL (100 g/L) or less.

Hydrocortisone therapy is often a third-line agent for neonates who have septic shock and fail to
respond to volume resuscitation and vasoactive medications. Although not the first step in the
management of septic shock, the clinician should consider correction of hypocalcemia to
optimize cardiac function. The newborn in the vignette has a low-normal ionized calcium level
and should be monitored closely. Attempts to increase perfusion should be the first method used
to correct metabolic acidosis in a neonate with septic shock, such as the newborn in the vignette.
Sodium bicarbonate therapy should be reserved for correction of ongoing metabolic acidosis that
is unresponsive to medical intervention in an intubated patient.

PREP Pearls
• Poor tissue perfusion is associated with decreased tissue oxygenation, which leads to an
increased serum lactate and metabolic acidosis.
• The full-term neonate presenting with septic shock requires volume expansion to improve
tissue perfusion.

American academy of pediatrics 168


American Academy of Pediatrics PREP 2015

American Board of Pediatrics Content Specification(s)


• Understand the metabolic consequences of continued poor perfusion in a newborn infant

Suggested Reading
• Brierley J, Carcillo IA, Choong K, et al. Clinical practice parameters for hemodynamic
support of pediatric and neonatal septic shock: 2007 update from the American College
of Critical Care Medicine. Crit Care Med. 2009;37(21:666-688.
doi:10.1097/CCM.0b013e31819323c6.
• Wynn JL, Wong HR. Pathophysiology and treatment of septic shock in neonates. an
Perinatol. 2010;37(2):439-479. doi:10.1016/j.clp.2010.04.002.

American academy of pediatrics 169


American Academy of Pediatrics PREP 2015

Item 56
A 10-year-old boy presents to his pediatrician with a complaint of frequent headaches over the
past 2 months. The headaches are occipital, dull, and pounding, lasting 1 to 2 hours, and he
sometimes feels dizzy. He has not had nausea, vomiting, or visual changes. The headaches
worsen when he sneezes or laughs. Ibuprofen and acetaminophen have not helped. His physical
examination findings are unremarkable. He has full range of motion of his neck and no
tenderness to palpation of the spinous processes. On neurologic examination, his fundi are
normal, there is no blurring of the optic disc margins, and his extraocular movements are full and
conjugate. Motor examination shows hypotonia of his arms with mildly decreased grip strength
and areflexia bilaterally. His lower extremities have normal tone and strength.

Of the following, the test MOST likely to yield the correct diagnosis is
A. cerebral angiogram
B. electroencephalogram
C. electromyogram and nerve conduction study
D. lumbar puncture
E. magnetic resonance imaging of the neck

American academy of pediatrics 170


American Academy of Pediatrics PREP 2015

Item 56 Preferred Response: E


The boy in the vignette has posterior headaches that worsen with Valsalva maneuver, which
suggests a symptomatic Chiari I malformation. Chiari I malformation is defined as extension of
the cerebellar tonsils greater than 5 mm below the foramen magnum. This is often asymptomatic
and found incidentally on magnetic resonance imaging of the cervical spinal cord. Symptomatic
Chiari malformation can cause increased intracranial pressure, brainstem or cervical myelopathy,
or a spinal cord syrinx. Signs of increased intracranial pressure include abnormal optics discs
with blurred disc margins or cranial nerve VI palsy (inability to fully abduct the eye). Brainstem
myelopathy can cause dysphagia or dysarthria. Cervical myelopathy can cause bilateral upper
extremity weakness with hypotonia and areflexia, and often lower extremity hyperreflexia or
spasticity. A spinal cord syrinx initially causes sensory loss at the level of the syrinx; this can
progress to include weakness of the arms and legs.

In pediatrics, hypotonia is almost always chronic and the majority of cases are caused by central
nervous system abnormalities. Outside the neonatal period, common causes of hypotonia are
neurogenetic syndromes caused by chromosomal abnormalities and brain anomalies such as
malformations or leukodystrophies. Chronic encephalopathy caused by birth-related hypoxic
ischemic encephalopathy is a common cause of hypotonia in young infants, and over time this
usually evolves into increased tone.

Acquired hypotonia is rare and requires prompt evaluation. In infants, botulism should be
considered. In addition to new onset poor head control and hypotonia, botulism can present with
bilateral ptosis that can be mistaken for encephalopathy. Diagnosis is by clinical examination and
confirmatory stool toxin testing. In older children, new onset hypotonia can be the presenting
symptom of weakness, caused by myasthenia gravis, for instance. In myasthenia gravis, there is
usually double vision, dysphagia or choking, and a waxing and waning course, in addition to
mild limb weakness or hypotonia.

In addition to headache, the boy in the vignette has signs of cervical spine dysfunction, with
weakness, hypotonia, and areflexia of his arms, but not his legs, so the best study would be
magnetic resonance imaging of the neck. Cerebral angiography evaluates the blood vessels of the
brain, and would not be helpful here. Electroencephalography would evaluate for seizures, but he
does not have these. Electromyography and nerve conduction study would identify muscle or
nerve disorders. Rare entities such as brachial plexopathy can cause weakness, hypotonia, and
areflexia of the arms, but these are not typically bilateral and would not cause headaches.
Lumbar puncture is the diagnostic test for idiopathic intracranial hypertension, and he does have
headaches that worsen with a Valsalva maneuver, but he does not have other signs of increased
intracranial pressure such as papillitis or cranial nerve VI palsy. His bilateral upper extremity
weakness, hypotonia, and areflexia suggest a cervical spinal cord problem.

PREP Pearls
• Occipital headache that worsens with Valsalva maneuver, with a normal fundus
examination, suggests a symptomatic Chiari I malformation.
• Bilateral arm weakness or hypotonia with preserved leg strength and tone suggests a
lesion in the cervical spine.
American academy of pediatrics 171
American Academy of Pediatrics PREP 2015

American Board of Pediatrics Content Specification(s)


• Plan the appropriate evaluation of hypotonia in patients of various ages

Suggested Reading
• Khoury C. Chiari malformations. UpToDate. Available online only for subscription.
• Peredo DE, Hannibal MC. The floppy infant: evaluation of hypotonia. Pediatr Rev.
2009;30(9):e66-e76. doi:10.1542/pir.30-9-e66.

American academy of pediatrics 172


American Academy of Pediatrics PREP 2015

Item 57
A 5-year-old boy presents for evaluation of tall stature (height >3 standard deviations). Review
of his growth chart reveals he has always been tall (Item Q57). His head circumference has
grown similarly and is significantly above the 97th percentile. He is meeting all of his
developmental milestones. The child's mother is 162.6 cm (5 ft, 4 in) tall. His father is 193 cm (6
ft, 4 in) tall and was also very tall for his age as a child. Physical examination shows no other
abnormal findings.

Of the following, the MOST likely diagnosis is


A. familial tall stature
B. Klinefelter syndrome
C. Marfan syndrome
D. pituitary gigantism
E. Sotos syndrome (cerebral gigantism)

American academy of pediatrics 173


American Academy of Pediatrics PREP 2015

Item 57 Preferred Response: A


The purpose of the diagnostic evaluation of tall stature is to distinguish the commonly occurring
normal variant constitutional (or familial) variety from the rare pathologic causes. When there is
a history of advanced growth in the family, the physical examination shows proportional growth
(height, weight, and head circumference), and there are no other concerning findings (such as
signs of early puberty or syndromic features), as is the case for the child in this vignette, a
diagnosis of familial tall stature can be made. No laboratory tests are indicated.

Klinefelter syndrome (XXY syndrome) occurs in 1:1,000 live male births and is associated with
tall stature, gynecomastia, and decreased upper-to-lower body segment ratio. Cognitive
development is usually normal, although speech and language delays are frequently noted. Mild
mental retardation has been reported in some patients. The testes are invariably small, although
androgen production by Leydig cells in the testes is often at the low-normal range. The
proportional growth of the patient in this vignette makes the diagnosis of Klinefelter syndrome
less likely.

Marfan syndrome is an autosomal dominant connective tissue disorder. Tall stature, increased
arm span, and decreased upper-to-lower body segment ratio are present. Height is typically
above the 95th percentile after 3 years of age. Additional findings include pectus deformities,
arachnodactyly, ocular abnormalities (ectopia lentis), and cardiac abnormalities (aortic root
dilatation and mitral valve anomalies). Other than tall stature, the child in this vignette has none
of these physical findings.

In children with open epiphyses, overproduction of growth hormone results in gigantism. Often,
some acromegalic features are seen. In young children, rapid growth of the head may precede
linear growth. Coarsening of facial features to include broad nose, enlarged tongue, and
excessive mandible growth also can occur secondary to elevated growth hormone levels. Given
that gigantism is extremely rare and the presentation is usually dramatic (unlike the insidious
onset of acromegaly in adults), this child's steady growth suggests that pituitary gigantism is
unlikely.

Sotos syndrome is an autosomal dominant disorder associated with rapid postnatal and early
childhood growth with an estimated incidence of 1 in 15,000 newborns. It is caused by
haploinsufficiency of the NSDI protein. Classic features include height or a head circumference
greater than or equal to 98th percentile, learning disabilities, craniofacial features that include a
triangular shaped face with prominent chin, frontal bossing, apparent hypertelorism, and down
slanting palpebral fissures. Neonatal hypotonia, feeding problems, advanced bone age, scoliosis,
seizures, and cardiac and genitourinary anomalies are also common. Given the child in this
vignette has none of these other concerning findings, Sotos syndrome is unlikely.

PREP Pearls
• Familial tall stature is the most likely diagnosis when there is proportional growth
(height, weight, and head circumference) consistent with family history and with no other
concerning findings.
• Syndromic causes of tall stature are commonly associated with abnormal upper-to-lower
body segment ratios, developmental delay, and characteristic facial findings.
American academy of pediatrics 174
American Academy of Pediatrics PREP 2015

American Board of Pediatrics Content Specification(s)


• Differentiate among the causes of tall stature

Suggested Reading
• Rosenfeld RG, Cohen P. Disorders of growth hormone/insulin-like growth factor
secretion and action. In: Sperling MA, ed. Pediatric Endocrinology. 3rd ed. Philadelphia.
PA: Saunders Elsevier; 2008:254-334.
• Visser R, Kant SG, Wit IM, Breuning MH. Overgrowth syndromes: from classical to
new. Pediatr Endocrinol Rev. 2009;6(3):375-394.

American academy of pediatrics 175


American Academy of Pediatrics PREP 2015

Item 58
You are reviewing the laboratory results of a healthy 2-yearold child seen for a health
supervision visit. His hemoglobin level is 11.2 g/dL (112 g/L), hematocrit is 33.5% (0.34), mean
corpuscular volume is 74 µm3 (74 ft.), and blood lead level is 6.1 µg/dL (0.3 µmol/L).

Of the following, the MOST appropriate statement regarding this child's lead level is that it

A. falls below the current reference value for lead poisoning


B. is too low to cause neurocognitive effects
C. requires chelation therapy to reverse central nervous effects
D. should be treated with a low calcium diet
E. should trigger a thorough dietary and exposure history

American academy of pediatrics 176


American Academy of Pediatrics PREP 2015

Item 58 S Preferred Response: E


In 2012, the Advisory Committee on Childhood Lead Poisoning Prevention (ACCLPP)
recommended replacing the previous blood lead level (BLL) "level of concern" of 10 µg/dL
(0.48 umol/L) with a "reference" level representing the 97.5th percentile of BLLs obtained from
US children aged 1 to 5 years using data from 2 consecutive cycles of the National Health and
Nutrition Examination Survey. Currently, this upper reference value is 5 µg/dL (0.24 umol/L). If
this reference value is exceeded, the clinician must do a thorough history, looking for
environmental and dietary exposures that could contribute to the elevated level. This
recommendation acknowledges the growing body of evidence that there is no threshold level for
lead toxicity. Health effects, most notably neurocognitive and behavioral, can be detected with a
very low BLL. Studies show that with an increase in BLL from 1 µg/dL (0.05 umol/L) to 4
µg/dL (0.19 umol/L), IQ can show a decline of 2 to 5 points. In addition, there are behavioral
consequences of increased BLL, including decreased attention, increased impulsivity, and
reduced academic achievement. Other organ systems affected by lead exposure include the
cardiovascular, endocrine, and immunologic systems.

Unfortunately, the predominant effects of lead on neurocognitive function do not appear to be


reversible. Chelation therapy for children with BLLs less than 45 µg/dL, (2.17 µmol/L) has not
shown any benefit. Enrichment programs such as Head Start may help improve neurocognitive
and academic outcomes, but to what extent is unclear. It is also unclear whether the benefits from
these programs for children with elevated BLLs exceed or merely reflect the benefits for children
without detectable lead levels.

Lead poisoning remains an important public health problem without effective treatment.
Therefore, the ACCLPP stresses the need for primary prevention, as well as appropriate
screening. Primary prevention involves ensuring a safe environment for all children, early
identification, mitigation of risk factors (environmental, occupational, avocational), and
counseling about healthy diet. Calcium, iron, and vitamin C reduce lead absorption, therefore
diets rich in these nutrients should be encouraged for patients at risk for lead toxicity. BLL
screening is currently required for Medicaid recipients at 1 and 2 years of age. The Centers for
Disease Control and Prevention and the American Academy of Pediatrics also recommend lead
screening for all immigrant, refugee, and foreign-adopted children, as well as children whose
parents have occupational or hobby exposure. Universal screening for children should be
considered when local and regional data indicate risk, or when there is a significant proportion of
pre-1980s housing in the community.

PREP Pearls
• The reference standard for blood lead levels is currently 5 µg/ dL (0.24 µmol/L) and
levels at or above this should trigger further investigation and follow-up.
• There is no biological threshold below which lead is without effects.
• The primary effects of lead intoxication are neurocognitive and behavioral, but other
organ systems including the cardiovascular, endocrine, gastrointestinal, and immunologic
may be affected.
• The neurocognitive effects of lead intoxication appear to be irreversible.

American academy of pediatrics 177


American Academy of Pediatrics PREP 2015

American Board of Pediatrics Content Specification(s)


• Understand the importance of a screening examination for lead during early periodic
screening evaluations
• Recognize the multiple sources of exposure to lead
• Understand the outcomes associated with lead poisoning

Suggested Reading
• Advisory Committee on Childhood Lead Poisoning Prevention of the Centers for Disease
Control and Prevention. Low level lead exposure harms children: a renewed call for
primary prevention. Centers for Disease Control and Prevention website.
• American Academy of Pediatrics. Lead exposure in children: prevention, detection, and
management. Pediatrics. 2005;116(4):1036-1046. doi:10.1542/peds.2005-1947.
• Wheeler W, Brown MI. Blood lead levels in children aged 1-5 years - United States,
1999-2010. MMWR Moth Mortal Wkly Rep. 2013;62(13):245248. Accessed January 7,
2014.

American academy of pediatrics 178


American Academy of Pediatrics PREP 2015

Item 59
You are discussing sexuality in your adolescent patients with a medical student. You are asked
for an explanation for the dramatic drop in birth rates for adolescents over the last 2 decades.

Of the following, the MOST accurate reason for this phenomenon is that

A. the abortion rate has increased over the same period


B. abstinence education is commonly available in schools
C. more accurate sex-related information is available on the internet
D. more adolescents are using contraceptive methods effectively
E. oral sex rates have increased, whereas vaginal intercourse rates have decreased

American academy of pediatrics 179


American Academy of Pediatrics PREP 2015

Item 59 Preferred Response: D


One of the goals of adolescence is the development of a sexual identity and the ability to develop
a stable sexual relationship in the future. The hormonal changes at the onset of puberty, along
with the influence of peers, media, and the Internet, increase sexual awareness, interest, and
feelings. Adolescence is a time of exploration, and 7 in 10 teenagers will have had vaginal
intercourse by 19 years of age. However, currently more teenagers are waiting longer to start
(until after 15 years of age) than before, and those who are sexually active are more likely to use
condoms and other forms of contraception more effectively. This improvement is credited to the
availability of comprehensive sex education and has resulted in the drop in both pregnancy and
birth rates, as well as a declining abortion rate. Abstinence education alone, that is not part of a
comprehensive sex education program, has not been proven to be as effective as comprehensive
sex education. Also, once adolescents begin sexual activity and are in a noncoercive relationship,
they are very unlikely to stop all activity going forward. Therefore, they need accurate
information on contraception before they begin.

Early on, most adolescents engage in noncoital sexuality. Often, this experimentation takes place
with peers of the same age and gender during role-playing or games and in a nonromantic
setting. Early onset of coital activity is often a red flag for other issues such as abuse, substance
abuse, and emotional adjustment problems, and is especially a concern when there is a 3 year or
greater age difference between the early adolescent and the partner. Those girls who mature early
appear older than their chronologic age and are more likely to receive advances from older peers
and engage in early sexual experimentation.

Most middle adolescents (~15-17 years of age) have begun to experiment sexually within a
romantic relationship. While the number engaging in oral sexual activity has increased over time,
this has not affected the rate of vaginal intercourse and thus pregnancy rates. Experimentation
between adolescents of the same gender is not uncommon nor is voyeuristic behavior. The
development and acceptance of a homosexual or bisexual identity occurs over time.

Parents need to understand that sexual experimentation is important in the development of their
teenagers' future ability to engage in intimate relationships. Pediatricians need to encourage
parents to engage in discussions around sexuality early and repeatedly, as this has been shown
both to delay the start of sexual activity and prevent risky behavior. Their children will have
questions about their physical changes and later about sexual behavior, relationships, values, and
the consequences of sexual behavior. Some tips for parents are available on the
HealthyChildren.org website listed in the suggested readings. Unfortunately, the myriad sites
with sex-related information available on the internet often do not have accurate information. Of
greater concern is that the media influence girls to see themselves as sexual objects, while boys
are pressured into exhibiting their manhood through sexual conquests.

PREP Pearls
• Early and repetitive discussions around sexuality by parents have been shown to decrease
the early onset of sexual activity and reduce high-risk sexual behaviors.
• School-based comprehensive sexuality education has been successful at getting
adolescents to use contraception more often and effectively, and has resulted in a falling
pregnancy, birth, and abortion rate in the United States.
American academy of pediatrics 180
American Academy of Pediatrics PREP 2015

American Board of Pediatrics Content Specification(s)


• Recognize the common patterns of sexual behavior and experimentation in adolescents of
various ages
• Plan appropriate parental counseling regarding adolescent sexuality
• identify common sources of information sought by adolescents regarding sexuality

Suggested Reading
• American Academy of Pediatrics. How to talk about sex with your teen. Healthy
Children website.
• Brooks-Gunn 1, Pa ikoff R. Sexuality and developmental transitions during adolescence.
In: Schulenberg J, Maggs JL, Hurrelmann K, Chassin L. eds. Health Risks and
Developmental Transitions During Adolescence. Cambridge, UK: Cambridge University
Press; 1997:190-219).
• Tulloch T, Kaufman M. Adolescent sexuality. Pediatr Rev. 2013;34(1):2938.
doi:10.1542/pir.34-1-29.
• Levine SB. Facilitating parent-child communication about sexuality. Pediatr Rev.
2011;32(3):129-130. doi:10.1542/pir.32-3-129.
• Martinez G, Copen CE, Abma IC. Teenagers in the United States: sexual activity,
contraceptive use, and childbearing, 2006-2010 national survey of family growth. Vital
Health Star 23. 2011;(31):1-35.
• Sieving RE, Oliphant JA, Blum RW. Adolescent sexual behavior and sexual health.
Pediatr Rev. 2002;23(12):407-416. doi:10.1542/pir.23-12-407.
• Strausburger VC. Sexuality, contraception, and the media. Pediatrics_ 2010;126(3):576-
582. doi:10.1542/peds.2010-1544.

American academy of pediatrics 181


American Academy of Pediatrics PREP 2015

Item 60
A 2-year-old girl is brought to your office for follow-up of severe atopic dermatitis. The child
has been doing better with regular use of topical triamcinolone, hypoallergenic skin moisturizer,
and bedtime diphenhydramine. She had an episode of wheezing illness during each of the past 2
winters that was triggered by a viral infection. She had received albuterol treatments during the
illness. The parents felt the albuterol helped the wheezing, though inconsistently. In the course of
the visit, you counsel the parents about the possibility of development of other atopic disorders,
such as allergic rhinitis and asthma. The parents ask what factors increase the risk of their child
developing asthma or other atopic disorders.

Of the following, the BEST response to the parents' questions is


A. allergic sensitization to milk, egg, peanut
B. body mass index greater than 95th percentile
C. gastroesophageal reflux
D. neutrophilia during wheezing episodes
E. nonallergic rhinitis with eosinophilia

American academy of pediatrics 182


American Academy of Pediatrics PREP 2015

Item 60 Preferred Response: A


Allergic sensitization to foods increases the risk of the child in the vignette developing, or having
increasing severity of, atopic disorders. The Asthma Predictive Index has predictive factors
derived from observation of children in a Tucson cohort who had wheezed at least once during
the first 3 years after birth. Factors include the presence of clinician-diagnosed eczema, parental
asthma, clinician-diagnosed allergic rhinitis, wheezing apart from colds, and eosinophilia greater
than or equal to 4%. A modified version of the asthma predicted index includes allergic
sensitization to greater than or equal to 1 aeroallergen as a major criterion, allergic sensitization
to milk, egg, or peanuts as a minor criterion, and has been endorsed by the National Asthma
Education and Prevention Program (NAEPP) for the diagnosis of asthma.

Aside from environmental exposures (allergic and nonallergic such as tobacco smoke exposure),
non-environmental factors such as genetic factors, developmental factors such as prematurity,
and host factors such as infections and diet can predispose to and influence the severity of
allergic disorders in infants and children. Gender-related differences in the prevalence of asthma
have been reported. Childhood asthma affects boys more than girls. The incidence in females
begins to rise at puberty, and by early adulthood, the prevalence is about equal. By 40 years of
age, the trend is reversed and more women than men have asthma. Early nutrition, including
prenatal exposure to nutrients, may also be relevant as a risk factor for the development of
asthma and allergies. Conflicting results have been reported, possibly because of inherent
difficulties with assessing diet and controlling for confounders. Maternal intake of the
antioxidant vitamin E, zinc, and vitamin D during pregnancy may modulate the risk for wheezing
and asthma in young children, though the data is conflicting.

Obesity and increased body mass index (BMI) have been statistically associated with asthma in
many studies, more so with nonallergic asthma. Increased BMI may also be associated with
increased asthma severity. However, biologic causality has not been proven. When data from
16,171 participants in the Third National Health and Nutrition Examination Survey (NHANES
III) were analyzed, obesity was an independent risk factor for dyspnea, but not for airflow
obstruction.

Gastroesophageal (GE) reflux is common in patients with asthma and has been identified as a
potential trigger. Gastroesophageal reflux is thought to affect asthma through increased vagal
tone, heightened bronchial hyperreactivity, or microaspiration. However, GE reflux does not
cause asthma.

Nonallergic rhinitis with nasal eosinophilia syndrome (NARES) is the most common type of
inflammatory non-allergic rhinitis. Nonallergic rhinitis with nasal eosinophilia syndrome is
characterized by large numbers (inconsistently defined as > 5% to > 20%) of eosinophils on
nasal smear. Patients with chronic NARES have negative skin and in vitro tests for allergen-
specific immunoglobulin E (IgE), and thus lack evidence of increased systemic production of
IgE. Notably, while IgE production has been demonstrated to occur locally in the nasal tissue of
some patients with NARES, other researchers (using similar techniques) have been unable to
demonstrate consistent responses to nasal allergen challenge, suggesting that nonspecific
hyperreactivity of the nasal mucosa due to inflammation may play a role. Nonallergic rhinitis
with nasal eosinophilia syndrome may be associated with a higher incidence of asthma and more
American academy of pediatrics 183
American Academy of Pediatrics PREP 2015

severe symptoms than when the predominant pattern is characterized by neutrophilia, but is not
considered a risk factor predicting development or severity of allergic disease.

Based on the proportion of eosinophil and neutrophil counts in sputum, 4 inflammatory subtypes
of asthma have been described: eosinophilic, neutrophilic, mixed granulocytic asthma (both
increased eosinophils and neutrophils), and paucigranulocytic asthma (normal levels of both
eosinophils and neutrophils). Studies have noted neutrophilic inflammation in some patients with
severe asthma and in small numbers of patients studied during asthma exacerbations. Intense
neutrophilic inflammation has also been demonstrated in patients ventilated for acute severe
asthma and in those with sudden fatal asthma exacerbations. Children with asthma triggered by
viral infections may have neutrophilia with wheezing episodes. Viral infections, such as those
caused by rhinovirus, respiratory syncytial virus, parainfluenzae, and metapneumovirus, are
important causes of wheezing in children, but the presence of neutrophilia with wheezing
episodes is not a predictor of occurrence or severity of allergic disease.

PREP Pearls
• The presence of allergic sensitization to aeroallergens or certain foods (milk, egg,
peanuts) is considered a risk factor influencing the development and severity of allergic
disorders in children.
• Non environmental factors, such as genetics, diet, and infection, also impact on the
presence and severity of allergic disorders.

American Board of Pediatrics Content Specification(s)


• Recognize the non-environmental factors (eg, genetics, diet, infection) that influence the
incidence and severity of atopy in infants and children

Suggested Reading
• Castro-Rodriguez JA. The Asthma Predictive Index: a very useful tool for predicting
asthma in young children. I Allergy Clin Immunol. 2010;126(2):212-216.
doi:10.1016/j.jaci.2010.06.032.
• Gern JE. Rhinovirus and the initiation of asthma. Curr Opin Allergy an Immunol.
2009;9(1):73-78. doi:10.1097/AC1.0b013e3283118f1b.
• National Asthma Education and Prevention Program. Expert Panel Report 3 (EPR-3):
Guidelines for the Diagnosis and Management of Asthma-Summary Report 2007. I
Allergy Clin Immunol. 2007;120(5 Suppl):S94-S138. doi:10.1016/j.jaci.2007.09.029.

American academy of pediatrics 184


American Academy of Pediatrics PREP 2015

Item 61
You are evaluating a 12-year-old girl who was brought to the emergency department with
complaints of severe headache that started 3 weeks ago and has been worsening. The headaches
are not preceded by any specific symptoms, are described as throbbing in nature, and are in the
back of the head and behind the eyes. They are worse at night and in the morning, and are
associated with vomiting. She also has sensitivity to light and sound.

Vital signs show a temperature of 37°C, heart rate of 60 beats/min, blood pressure of 140/90 mm
Hg, respiratory rate of 15 breaths/min, and Spot is 98% on room air. Physical examination shows
a generally well-developed and well-nourished girl who is awake, oriented, and lying still. She is
complaining of a "10 out of 10" headache. Pupils are 6 mm, equal, and reactive to light. Her right
eye cannot move laterally, but the rest of her extraocular movements and cranial nerve functions
are intact. Funduscopic examination shows bilateral papilledema. She does not have any other
focal neurologic deficits. She is breathing comfortably, and lungs are clear to auscultation
bilaterally. Cardiovascular examination is unremarkable. Abdomen is soft, nontender,
nondistended, and with no organomegaly. Extremities are warm and well perfused.

Of the following, the MOST appropriate next step in management is


A. acetazolamide
B. computed tomography scan of head
C. lumbar puncture with opening and closing pressure
D. mannitol
E. morphine

American academy of pediatrics 185


American Academy of Pediatrics PREP 2015

Item 61 Preferred Response: B


The child in the vignette has elevated intracranial pressure (ICP), as evidenced by symptoms of
headache, vomiting that is worse when supine, and photophobia, as well as physical examination
findings of hypertension, bradycardia, and papilledema. The most important next step is a
computed tomography (CT) scan of the head, to evaluate for the presence or absence of
hydrocephalus, and if present, whether it is obstructive or nonobstructive.

Intracranial contents include blood, brain, and cerebrospinal fluid (CSF). Even in infants with
open fontanelles, the calvarium is rigid and noncompliant. Thus, as indicated by the Monroe-
Kellie doctrine, an increase in the volume of any of those components will cause a compensatory
decrease in the other components, followed by a rapid rise in ICP when that mechanism is
exhausted. Swelling of the brain component, or cerebral edema, can be caused by trauma,
infection, stroke, hypoxic-ischemic encephalopathy, metabolic disturbances, or mass. The CSF
component can be increased because of decreased CSF reabsorption from the arachnoid
granulation system, for example, after a subarachnoid hemorrhage or from lesions obstructing
CSF flow. Blood component can be increased with cerebral hemorrhage. Any of these
mechanisms can cause elevated ICP.

Elevated ICP causes dysfunction of various parts of the brain, which may vary depending on
regional variations in pressure, as well as neuronal damage and ischemia. Common symptoms of
elevated ICP include lethargy, irritability, visual disturbances, vomiting, and headache.
Symptoms are often worse when the patient is supine and without the benefit of gravity-driven
drainage of blood and CSF. Physical findings can include macrocephaly, full or bulging
fontanelle in infants, papilledema, abducens palsy, and dilated, poorly reactive, or asymmetric
pupils. Late findings of elevated ICP include obtundation, decerebrate posturing, apnea, and
dilated and unreactive pupils, which may indicate uncal or tonsillar herniation.

As indicated by the symptoms of headache, photophobia, and vomiting, and physical findings of
elevated ICP, the most likely diagnosis in this patient is pseudotumor cerebri. Lumbar puncture
with opening and closing pressure can aid in the diagnosis, and acetazolamide can be an effective
treatment for pseudotumor cerebri. However, an intracranial obstructive mass lesion cannot be
excluded based on this clinical picture. In that condition, a lumbar puncture can cause a large
pressure gradient between the cranium and atmosphere, causing downward cerebral herniation.
This phenomenon is less likely to occur in diffuse cerebral edema. Thus, a cranial CT should be
performed to exclude obstructive hydrocephalus before performing a lumbar puncture. Mannitol
can be an effective agent to reduce ICP, but it is not routinely used in pseudotumor cerebri, and it
is more important to first establish the nature of the elevated ICP. Morphine and other
medications that could potentially depress respiratory drive should be avoided at all costs. In
elevated ICP, hyperventilation is a natural compensatory mechanism to decrease the blood
component of the cranial vault because hypocapnia decreases cerebral blood flow Even a slight
increase in PCO2 from slowed respirations because of narcotics can lead to herniation and death.

Elevated ICP is a serious condition that must be recognized, diagnosed, and treated promptly.
Depending on age, historical findings can include irritability, photophobia, headache, vomiting,
and altered sensorium. Physical examination findings can include fontanelle fullness,
bradycardia, hypertension, extraocular and pupillary abnormalities, and hyperventilation.
American academy of pediatrics 186
American Academy of Pediatrics PREP 2015

Narcotics and sedatives should be absolutely avoided. Lumbar puncture can aid in diagnosis, but
only after obstructive hydrocephalus has been excluded.

PREP Pearls
• Computed tomography of the head is the first step when evaluating a patient with
suspected increased intracranial pressure (ICP).
• Narcotics and sedatives should be avoided in cases of elevated ICP.
• Nighttime or early morning vomiting can be a sign of elevated ICP.
• Lumbar puncture with measurement of opening and closing pressure can be helpful in
elevated ICP, especially in pseudotumor cerebri or infections, but only after obstructive
hydrocephalus has been excluded.

American Board of Pediatrics Content Specification(s)


• Recognize the clinical findings associated with increased intracranial pressure in patients
of various ages
• Understand the indications and contraindications for examination of the cerebrospinal
fluid in a patient who has increased intracranial pressure

Suggested Reading
• Kinsman SL, Johnston MV. Hydrocephalus. In: Kliegman RM, Stanton BF, St. Geme JW
III, Schor NF, Behrman RE, eds. Nelson's Textbook of Pediatrics. 19th ed. Philadelphia,
PA: Saunders Elsevier; 2011: 2008-2010.
• Larsen GY, Goldstein B. Consultation with the specialist: increased intracranial pressure.
Pediatr Rev. 1999;20(7):234-239. doi:10.1542/ pir.20 -7-234.

American academy of pediatrics 187


American Academy of Pediatrics PREP 2015

Item 62
A 14-year-old adolescent boy presents to your office with a 6-day history of pain over his left
thigh that has been increasing to the point that on the day of evaluation, he refuses to bear weight
on the extremity. The day prior to the onset of symptoms, he was hit with a lacrosse stick in his
left thigh. He initially complained of pain, and ice was applied to the area several times over the
next 24 hours. Over the next 2 days, he described the area as being stiff, but then began to feel
increasing pain in this leg. He felt the area was swollen, but did not notice any redness. He
reports no fever, chills, or other symptoms.

His vital signs show a temperature of 37.4°C, a heart rate of 96 beats/min, a blood pressure of
110/74 mm Hg, and a respiratory rate of 22 breaths/min. His physical examination findings are
unremarkable, except for his refusal to bear weight on his left leg. Additionally, there is swelling
above the patella on the left without erythema. He complains of pain with flexion at the knee and
hip on the left.

A complete blood cell count shows the following:


• White blood cell count, 11,500/µL (11.50 x 109/L), with 74% neutrophils, 24%
lymphocytes, and 2% eosinophils
• Hemoglobin, 12.8 g/dL (128 g/L)
• Platelets, 380 x 103/ µL (380 x 109/L)
• Erythrocyte sedimentation rate, 68 mm/h
• C-reactive protein, 52 mg/L (495.2 mmol/L)
A radiograph of the left femur demonstrates periosteal elevation in the distal femur. Magnetic
resonance imaging demonstrates a lytic lesion in the distal shaft of the left femur.
The orthopedic surgeon performs an aspiration of the lesion that shows abundant neutrophils and
is sent for culture.
Of the following, the MOST appropriate antibiotic agent to treat this infection initially pending
culture results is
A. ampicillin
B. ceftriaxone
C. clindamycin
D. doxycycline
E. trimethoprim-sulfamethoxazole

American academy of pediatrics 188


American Academy of Pediatrics PREP 2015

Item 62 Preferred Response: C


The adolescent boy in this vignette presents with a history of worsening leg pain 6 days after
trauma to the left thigh. The findings on physical examination do not definitively localize the site
of pathology, but the periosteal reaction on the plain radiograph suggests a lesion in the distal
femur that is confirmed by the magnetic resonance imaging examination.

Staphylococcus aureus is the leading cause of acute hematogenous osteomyelitis. Pending


culture results initial empiric coverage should be appropriate for methicillin - sensitive and
methicillin-resistant S aureus. Less commonly acute osteomyelitis is associated with group A
Streptococcus infection and Streptococcus pneumoniae. In children younger than 3 years of age,
Kingella kingae is also a potential pathogen. In neonates, group B Streptococcus and gram-
negative enteric organisms may cause osteomyelitis. Salmonella is an important cause of
osteomyelitis in children with hemoglobinopathies such as sickle cell disease.

In this case, antibiotic coverage should include both S aureus and streptococci while awaiting
culture and sensitivity results. Of the agents listed, clindamycin provides coverage for S aureus
(methicillin-sensitive and methicillin-resistant) and group A streptococcus. The drug is also
concentrated in bone. Ampicillin and ceftriaxone do not provide adequate S aureus coverage.
While doxycycline and trimethoprim/sulfamethoxazole have activity against methicillin resistant
S aureus and methicillin-sensitive S aureus, they are not effective against group A Streptococcus
or pneumococcus. Additionally, there is insufficient information on trimethoprim/
sulfamethoxazole and doxycycline in the treatment of osteomyelitis to recommend them as the
drug of choice for treating such an infection.

PREP Pearls
• Hematogenous osteomyelitis typically occurs in the metaphysis of long bones and may
follow minor traumata the infected area.
• Staphylococcus aureus is the most common cause of hematogenous osteomyelitis with
group A Streptococcus and Streptococcus pneumoniae being other potential etiologies.
Empiric antibiotic Therapy should be directed against these pathogens, including
methicillin-resistant S aureus, unless the incidence of methicillin-resistant S aureus is
known to be very low.
• Group B Streptococcus and enteric gram-negative rods such as Escherichia coli are
possible causes of osteomyelitis in neonates. Kingella kingae is a potential cause of
osteomyelitis in children younger than 3 years of age. In children with sickle cell disease
and other hemoglobinopathies, Salmonella is an important cause of osteomyelitis.

American Board of Pediatrics Content Specification(s)


• Identify the etiology of osteomyelitis in patients of various ages
• Recognize the clinical findings associated with osteomyelitis in various anatomic
locations
• Plan the appropriate management of osteomyelitis in patients of various ages

American academy of pediatrics 189


American Academy of Pediatrics PREP 2015

Suggested Reading
• American Academy of Pediatrics. Staphylococcal infections. In: Pickering LK, Baker C),
Kimberlin DW, Long SS, eds. Red Book: 2012 Report of the Committee on Infectious
Diseases. 29th ed. Elk Grove Village, IL: American Academy of Pediatrics; 2012:653-
668.
• Kaplan SL. Osteomyelitis in children. Infect Dis Clin North Am. 2005;19(4):787-797.
dol:10.1016/1.idc.2005.07.006.
• Liu C. Bayer A, Cosgrove SE, et al. Clinical practice guidelines by the Infectious
Diseases Society of America for the treatment of methicillin-resistant Staphylococcus
aureus infections in adults and children: executive summary. Clin Infect Dis.
2011;52(3):285-292. doi:10.1093/cidicir034.
• Yagupsky P, Porsch E, St Geme I W III. Kingella kingae: an emerging pathogen in young
children. Pediatrics. 2011;127(3);557-565. doi:10.1542/ peds.2010-1867.

American academy of pediatrics 190


American Academy of Pediatrics PREP 2015

Item 63
A 14-year-old adolescent boy presents to the emergency department because he has had flank
pain radiating to the groin and blood in his urine for the last 8 hours. There is no history of fever,
burning on urination, or trauma. He had a similar episode 1 year ago associated with passage of
small brown particles in his urine. On physical examination, you notice the patient is in
discomfort and has mild dehydration, a temperature of 38.3°C, heart rate of 100 beats/min,
respiratory rate of 28 breaths/min, and blood pressure of 134/80 mm Hg. The patient refuses to
let you examine his abdomen. You suspect recurrent urinary tract stones as the underlying cause
of the patient's symptoms.

Of the following, the MOST appropriate time to begin 24-hour urine collection to evaluate this
boy's condition is after
A. adequate pain control and hydration
B. admission to the hospital
C. the follow-up outpatient visit
D. imaging studies show no stones in the kidney
E. the placement of an indwelling urinary catheter

American academy of pediatrics 191


American Academy of Pediatrics PREP 2015

Item 63 Preferred Response: C


Children are increasingly being diagnosed with nephrolithiasis or renal stones. An underlying
risk factor for renal stone formation, such as urinary metabolic abnormality, urinary tract
infection, or a structural abnormality of the genitourinary tract, is identified in nearly 80% of the
children with renal stones. Urinary metabolic abnormality, reported in 40% to 50% of children
with recurrent stones, is the most commonly identified risk factor for renal stones. Therefore, it is
currently recommended to evaluate all children with renal stones for urinary metabolic disorders.

The metabolic evaluation requires a 24-hour urine collection, which is analyzed for the presence
of solutes associated with increased risk for stone formation and for decreased excretion of
inhibitors of renal stone formation. The 24-hour urine collection should be performed after
hospital discharge, when the patient is at home doing normal activities, consuming a regular diet,
free of infection, and without intravenous fluid therapy. Patients and parents should be instructed
on the collection of urinary stories and to bring in any stones passed in the urine so that they can
be sent for chemical analysis.

High urinary concentrations of calcium, cystine, oxalate, and uric acid due to increased renal
excretion or low urine volume is associated with crystal formation, which leads to renal stone
formation. Increased urinary excretion of citrate (most important), magnesium, and
pyrophosphate is associated with decreased risk for renal stone formation. A low level of these
inhibitors is associated with an increased risk for nephrolithiasis in children and adults.

The patient described in the vignette has recurrent renal stones. Pain control and hydration are
important during the acute event, which is not the appropriate time for 24-hour urine collection.
The evaluation for an underlying metabolic abnormality is indicated irrespective of the imaging
results in this case. Occasionally, non-toilet-trained children with recurrent renal stones may
require urinary catheterization to obtain a 24-hour urinary collection. There is no indication for
an indwelling urinary catheter for the patient in the vignette.

PREP Pearls
• Children with renal stones should be evaluated for urinary metabolic disorders by a 24-
hour urine collection, with analysis for solutes associated with increased risk of stone
formation and decreased excretion of inhibitors of stone formation.
• A risk factor for stone formation is identified in nearly 80% of the children with renal
stones.
• A urinary metabolic abnormality is reported in 40% to 50% of children with recurrent
renal stones.
• The 24-hour urine collection should be performed once the patient is discharged and is at
home, doing normal activities, consuming a regular diet, free of infection, and without
intravenous fluid therapy.

American Board of Pediatrics Content Specification(s)


• Recognize factors contributing to the development of urinary tract stones
• Plan the appropriate management of urinary tract stones

American academy of pediatrics 192


American Academy of Pediatrics PREP 2015

Suggested Reading
• McKay CP. Renal stone disease. Pediatr Rev. 2010;31(5):179-188. doi:10.1542/pir.31-5-
179.
• Gearhart JP, Herzberg GZ, Jeffs RD. Childhood urolithiasis: experiences and advances_
Pediatrics. 1991;87(4):445-450. Accessed January 10,2014. Polito C, La Manna A,
Signoriello G. Marte A. Recurrent abdominal pain in childhood urolithiasis. Pediatrics.
2009;124(6):0088-e1094. doi:10.1542/ peds.2009-0825.

American academy of pediatrics 193


American Academy of Pediatrics PREP 2015

Item 64
A 2,000-g, 34-week-gestation female newborn was admitted to the neonatal intensive care unit at
3 days of age because of jaundice, lethargy, and vomiting. The pregnancy was complicated by
illicit drug abuse and poor prenatal care. Delivery was by cesarean birth secondary to maternal
preeclampsia, and the Apgar scores were 6 and 7 at one and five minutes, respectively. She
initially fed well with cow milk-based formula but was noted to develop jaundice at 24 hours
after birth; the baby became increasingly difficult to feed, with multiple episodes of postprandial
emesis. Because of these symptoms, a sepsis workup was performed and the baby was started on
intravenous antibiotics.

Physical examination demonstrates a lethargic, jaundiced infant. She has a temperature of


37.0°C, pulse rate of 130 beats/min, respiratory rate of 20 breaths/min, blood pressure of 88/56
mm Hg, and oxygen saturation of 97% on room air. The liver is palpated 5 cm below the right
costal margin. The remainder of the examination findings are normal. Laboratory studies are
obtained and include the following results:
• Total bilirubin, 15 mg/dL (256.5 mmol/L)
• Direct bilirubin, 5.5 mg/dL (94 mmol/L)
• Alanine aminotransferase, 205 U/L
• Aspartate aminotransferase, 170 U/L
• Prothrombin time, 21 s
• Partial thromboplastin time, 39 s
• International normalized ratio, 1.95
• Urine test strip, positive for reducing substances

Of the following, the MOST appropriate next diagnostic test is


A. cell culture for cytomegalovirus from urine
B. erythrocyte galactose-1-phosphate uridyltransferase
C. polymerase chain reaction for herpes simplex virus in cerebrospinal fluid
D. serum α-1 antitrypsin measurement with protease inhibitor typing
E. urine succinylacetone

American academy of pediatrics 194


American Academy of Pediatrics PREP 2015

Item 64 Preferred Response: B


The infant described in the vignette has direct hyperbilirubinemia, which is defined by a serum
direct bilirubin concentration of more than 1.0 mg/dL (17.1 umol/L) with total bilirubin values of
less than 5.0 mg/dL (85.5 µmol/L) or greater than 20% of the total bilirubin for values greater
than 5.0 mg/dL (85.5 µmol/L). Direct hyperbilirubinemia indicates cholestasis and is an
abnormal finding that requires additional evaluation.

In the child in the vignette, the clinical and laboratory findings do not suggest an obstructive
cause of cholestasis. The infant's presentation with vomiting, lethargy, and significant
hepatomegaly would be unusual in extrahepatic biliary atresia (EBA). Furthermore, this high
level of total bilirubin (total/ direct = 15/5.5 mg/dL) early in the neonatal course is atypical for
EBA. When direct hyperbilirubinemia is diagnosed within the first few days after birth,
particularly in association with other clinical symptoms, infectious and metabolic disorders must
be considered. Thus, sepsis workup is an important first step in this neonate. Hepatic dysfunction
(international normalized ratio of 1.95) with hepatomegaly and cholestasis during the first few
days after birth (particularly, as in this case, within the first 24 hours after commencing feeding)
suggest an inborn metabolic error. Although several metabolic disorders may present during the
neonatal period with evidence of cholestasis and hepatic compromise, the associated finding of
positive reducing substances in the urine indicate a carbohydrate metabolic disorder, the most
prevalent being galactosemia. An erythrocyte cell galactose-1-phosphate uridyltransferase level
will confirm the diagnosis.

Galactosemia is a disorder of galactose metabolism that, untreated, leads to feeding problems,


failure to thrive, life-threatening hepatocellular damage, bleeding, and sepsis. A lactose- or
galactose-restricted diet initiated within the first 10 days after birth will result in a rapid
resolution of symptoms and greatly reduce the risk of complications. Even with early dietary
management, children with galactosemia remain at increased risk for developmental, cognitive,
and motor delay. The diagnosis of galactosemia is established by measurement of erythrocyte
galactose- 1 -phosphate uridyltransferase (GALT) activity, erythrocyte galactose-1-phosphate
concentration, and GALT molecular genetic testing. In classic galactosemia, GALT activity is
less than 5% of control values.

Tyrosinemia type I must also be considered in neonates presenting with this constellation of
clinical findings. Tyrosinemia type I results from a deficiency in fumarylacetoacetate hydrolase
(FAH), the final enzymatic step in the tyrosine catabolic pathway. In affected patients, the
intermediate compounds maleylacetoacetate and fumarylacetoacetate are converted to toxic
metabolites succinylacetone and succinylacetoacetate, which are responsible for hepatorenal
toxicity. The diagnosis is suggested by an elevated urine succinylacetone level. Succinylacetone
can also inhibit the porphyrin synthesis pathway, leading to the accumulation of 5-
aminolevulinate, a potent neurotoxin. Tyrosinemia type I usually presents during the neonatal
period with liver involvement, marked by elevated transaminases, hypocholesterolemia, and
hyperbilirubinemia. In the child in the vignette, the finding of positive urinary reducing
substances suggests a carbohydrate, as opposed to an amino acid metabolic disorder.

Direct hyperbilirubinemia in the neonatal period is most commonly identified during the
evaluation of prolonged "physiological" jaundice, often in otherwise thriving infants. Therefore,
American academy of pediatrics 195
American Academy of Pediatrics PREP 2015

in all infants with clinical jaundice persisting beyond 2 weeks of age direct reacting (conjugated)
bilirubin should be measured. If the direct bilirubin level is elevated, biliary tract integrity should
be assessed. Evaluation of hepatobiliary integrity is of paramount importance to determine if
direct hyperbilirubinemia is the consequence of extrahepatic biliary tract obstruction (either EBA
or choledochal cyst). Recent studies have shown the best surgical outcomes for infants with
biliary atresia when the diagnosis is established by 30 to 45 days of age. A biliary tract
ultrasound should be done initially to rule out choledochal cyst, and further biliary tract
diagnostic evaluation, including percutaneous liver biopsy, may be performed when the infant
reaches 1 month of age.

Cholestasis may also result from intrahepatic bile duct paucity (eg, in α-1-antitrypsin deficiency
and Alagille syndrome). Obtaining a serum α-1-antitrypsin level is part of this evaluation
process.

The "gold standard" for identifying extrahepatic biliary tract obstruction remains the
percutaneous liver biopsy. Recent data indicate that biliary tract ultrasonography, conducted by
an experienced ultrasonographer, may be helpful. The finding of a "triangular cord sign:' which
represents the fibrous remnant of an obliterated extrahepatic biliary tree, has been shown to be
highly specific for biliary atresia. Other diagnostic modalities, including magnetic resonance
cholangiopancreatography and endoscopic retrograde cholangiopancreatography, are currently
being investigated. Abdominal computed tomography may suggest an absent or atretic gall
bladder, but this imaging study has not proven to be of significant value in the evaluation of
biliary atresia. Hepatobiliary scintigraphy, using a radiolabeled derivative of iminodiacetic acid,
has long been touted as an important adjunct to the evaluation of neonatal cholestasis. Although
definitive evidence of radiolabeled excretion into the small bowel may confirm biliary tract
patency, the study has a significant incidence of both false-positive and false-negative results.

PREP Pearls
• Direct hyperbilirubinemia in the first week after birth suggests a metabolic or infectious
etiology.
• Hepatic dysfunction within a few days (often < 24 hours) of introducing breast milk or
cow milk-based formulas suggests a diagnosis of galactosemia.
• Percutaneous liver biopsy remains the most sensitive and specific test for extrahepatic
biliary atresia.

American Board of Pediatrics Content Specification(s)


• Plan the appropriate diagnostic evaluation of conjugated hyperbilirubinemia in a neonate

Suggested Reading
• King LS, Trahms C, Scott CR. Tyrosinemia Type 1. GeneReviews. Louis Li.
Galactosemia. GeneReviews.
• Moyer V, Freese DK, Whitington PF, et al. Guideline for the evaluation of cholestatic
jaundice in infants: recommendations of the North American Society for Pediatric
Gastroenterology, Hepatology and Nutrition. I Pediatr Gastroenterol Nutr.
2004;39(2):115-128.

American academy of pediatrics 196


American Academy of Pediatrics PREP 2015

• Roach JP, Bruny JL. Advances in the understanding and treatment of biliary atresia. Curr
Opin Pediatr. 2008;20(3):315-319. doi:10.1097/ MOP.0b013e3282frdc43.
• Suchy FL Neonatal cholestasis. Pediatr Rev 2004;25(10:388-396. doi:10.1542/pir.25-11-
388.

American academy of pediatrics 197


American Academy of Pediatrics PREP 2015

Item 65
A 2-year-old girl arrives at your outpatient clinic with her mother. She is concerned that her left
leg and arm seem to be larger than the leg and arm on the right. The mother reports that the child
was 4.2 kg at birth and has continued to have rapid growth since that time. She also reports low
blood glucose levels during the neonatal period. The mother states the child has always had a
large tongue.

Her current height is 94 cm (95th percentile) and her current weight is 15.9 kg (> 95th
percentile). Physical examination is remarkable for a large tongue, anterior ear lobe creases,
umbilical hernia, nephromegaly, and hemihypertrophy of the left arm and leg (Item Q65).
Of the following, the MOST important laboratory workup that should be followed closely over
time in this patient is

A. α-fetoprotein
B. insulin level
C. insulin-like growth factor 1
D. liver function tests
E. thyroid function tests

American academy of pediatrics 198


American Academy of Pediatrics PREP 2015

Item 65 Preferred Response: A


The patient in the vignette has classic Beckwith-Wiedemann syndrome (BWS), which is a
growth disorder manifested by macrosomia, macroglossia, neonatal hypoglycemia, ear creases
and pits, hemihypertrophy, and visceromegaly. Patients also can have embryonal tumors (Wilms
tumor, hepatoblastoma, neuroblastoma), umbilical hernia or omphalocele, nephrocalcinosis,
medullary sponge kidney disease, cardiomegaly, and nephromegaly. Traditionally, the
macrosomia, macroglossia, and hypoglycemia are noted in the neonatal period. Hemihyperplasia
is noted in segmental regions of the body or specific organs. Developmental and cognitive
outcomes are typically normal. Routine surveillance is recommended because of the risk for
embryonal tumors that is highest in childhood during the first 8 years of age. Abdominal
ultrasonography is recommended every 3 months until age 8 years, and serum α-fetoprotein
concentration is monitored every 2 to 3 months in the first 4 years after birth for early detection
of hepatoblastoma and Wilms tumor. The estimated tumor risk for children with BWS is 7.5%.
Between age 8 years and mid-adolescence, renal ultrasonography is recommended annually to
assess for renal disease, as well as consideration of measurement of urinary calcium-creatinine
ratio. Most individuals with BWS have this condition in association with imprinting disorders,
such as methylation abnormalities or paternal uniparental disomy, of chromosome 11 p15. Thus,
most cases are not inherited.

Within the differential diagnosis of a child with hemi-hyperplasia, a physician should consider
BWS, Proteus syndrome (PTEN hamartoma tumor syndrome), Klippel-Trenaunay-Weber
syndrome, isolated hemihyperplasia, and neurofibromatosis type 1. A subset of individuals with
isolated hemihyperplasia could have BWS, but with few clinical findings. Children with isolated
hemihyperplasia have a tumor risk of approximately 5.9%. These children should also undergo
frequent tumor surveillance just like the children with BWS.

Other disorders that can present with macrosomia in the neonatal period include Simpson-
Golabi-Behmel syndrome, Costello syndrome, and Sotos syndrome. These disorders typically
also have developmental delay along with the macrosomia, which is not common with BWS.
Children with BWS typically do not present with over-growth in association with growth
hormone abnormalities (insulin-like growth factor 1 and insulin-like growth factor binding
protein 3). The organomegaly commonly noted is not associated with liver dysfunction or
thyroid abnormalities, though it can be associated with renal abnormalities. The
hyperinsulinemic-hypoglycemia noted at birth in 50% of patients with BWS resolves
spontaneously in early infancy and would not be a continued problem for a 2-year-old patient,
such as the girl in this vignette.

PREP Pearls
• Within the differential diagnosis of a child with hemihyperplasia, a physician should
consider Beckwith-Wiedemann syndrome (BWS), Proteus syndrome (PTEN hamartoma
tumor syndrome), Klippel-Trenaunay-Weber syndrome, isolated hemihyperplasia, and
neurofibromatosis type 1.
• Children with isolated hemihyperplasia have a tumor risk of approximately 5.9%.
• The risk for embryonal tumors in children with Beckwith Wiedemann Syndrome is
highest during the first 8 years of age, therefore high-risk surveillance is recommended,
including abdominal ultrasonography every 3 months until age 8 years, and serum α-
American academy of pediatrics 199
American Academy of Pediatrics PREP 2015

fetoprotein concentration every 2 to 3 months in the first 4 years after birth for early
detection of hepatoblastoma and Wilms tumor. The estimated tumor risk for children
with BWS is 7.5%.

American Board of Pediatrics Content Specification(s)


• Recognize the clinical features and risks associated with overgrowth syndrome.

Suggested Reading
 Clericuzio CL, Martin RA. Diagnostic criteria and tumor screening for individuals with
isolated hemiphyperplasia. Genet Med. 2009;11(3):220222.
doi:10.1097/GIM.0b013e31819436ct
 Hoyme HE, Seaver LH, Jones KL, Procopio F, Crooks W, Feingold M. Isolated
hemihyperplasia (hemihypertrophy): report of a prospective multicenter study of the
incidence of neoplasia and review. Am I Med Genet. 1998;79(4):274-278.
doi:10.1002/(SICI)1096-8628(19981002)79:4.
 Zarate YA, Mena R, Martin LJ, Steele P, Tinkle BT, Hopkin RJ. Experience with
hemihyperplasia and Beckwith-Wiedemann syndrome surveillance protocol. Am J Med
Genet A. 2009,149A(18):1691-1697. doi:10.1002/ ajmg.a.32966.

American academy of pediatrics 200


American Academy of Pediatrics PREP 2015

Item 66
A 5-year-old boy presents to your office for his annual routine health supervision visit. He has
been generally healthy, and his growth and development are normal. The mother reports that she
thinks he is ignoring her requests more the past few weeks, but she has no other concerns. On
physical examination, you notice white fibrous material in the right external ear canal overlying
the tympanic membrane. The patient reports mild pain on examination, but denies any preceding
trauma or drainage. Hearing screen reveals a threshold of 35 db at 1,000 Hz and 2,000 Hz on the
right side, compared to 25 db on the left side.

Of the following, the MOST appropriate next step is to


A. attempt removal with cerumen spoon
B. irrigate the external auditory canal
C. prescribe ciprofloxacin otic drops
D. refer to audiology for further assessment
E. refer to otolaryngology immediately

American academy of pediatrics 201


American Academy of Pediatrics PREP 2015

Item 66 Preferred Response: A


The boy in the vignette later admitted to being hit with a paper "spit" ball a few weeks ago. The
foreign object in this case was paper, therefore the first step is to attempt removal via
instrumentation. Patients with foreign bodies of the external auditory canal are often
asymptomatic like this young boy, so frequently these objects are found incidentally during
routine otoscopy. Foreign bodies in the ear and nose are common occurrences in young children,
especially in those younger than 6 years of age. The most common objects include beads,
pebbles, tissue paper, small toys, seeds, and insects.

Most aural foreign bodies can be removed without complications using simple techniques and
without referral to the otolaryngologist. The most appropriate technique depends on the object.
Options for removal include instrumentation, suction, or irrigation. Successful extractions
require immobilization or cooperation of the child, adequate visualization, and proper equipment.

The instrument chosen to remove a foreign body from the ear canal depends on the size, nature
of the object, and the skill of the practitioner. Alligator forceps may be used to remove objects
that can be easily grasped under direct visualization or through the operating head of an
otoscope. An ear curette or cerumen spoon may be used to remove round or smooth objects that
are difficult to grasp with forceps. Insects should be killed before removal by instilling 1% to 2%
lidocaine, isopropyl alcohol, or mineral oil into the external auditory canal. Suction may be used
for objects that are difficult to grasp if the proper equipment is available.

Irrigation may be used to remove small, inorganic, non-absorptive objects. The irrigation
solution should be at body temperature and the flow directed toward the superior margin of the
external ear canal. Irrigation is contraindicated if the tympanic membrane is perforated or if the
foreign body is vegetable matter or an alkaline button battery. Irrigation is not appropriate in this
case because the paper might simply absorb the water, swell, and become further wedged into
the external ear canal.

Sometimes it is difficult to determine if white matter in the ear canal is due to an infection, either
from external otitis or otitis media with tympanic membrane perforation. One should expect to
see erythema or tenderness of the ear canal or pain on manipulation of the pinna if external otitis
is present. This is not the case here. Also, because of a lack of evidence of otitis media or
perforation of the tympanic membrane, there is no indication to treat with topical antibiotic
drops. This option would just delay the necessary treatment.

Mild conductive hearing loss may occur because of the presence of an aural foreign body, yet
formal audiology assessment should not delay the treatment unless there is concern for
penetrating trauma. If hearing loss is the presenting complaint for a foreign body in the external
ear canal, one should perform a hearing screen after the foreign body is successfully removed.
Complications may result from multiple manipulations, so an otolaryngologist should be
consulted once the diagnosis is made and if simple attempts at removal fail. Referral is also
indicated if the proper equipment and environment are not available. When the foreign body is a
battery or sharp object, or anytime there is concern for tympanic membrane perforation or injury
to the external ear canal, middle or inner ear, urgent referral to otolaryngology is indicated.

American academy of pediatrics 202


American Academy of Pediatrics PREP 2015

PREP Pearls
• Most foreign bodies in the external ear canal can be removed by the primary care
provider using simple techniques.
• Irrigation to remove a foreign body is contraindicated if the tympanic membrane is
perforated or if the foreign body is vegetable matter or an alkaline button battery.
• An urgent referral to otolaryngology is indicated when the aural foreign body is a battery
or sharp object, and any time there is concern for tympanic membrane perforation or
injury to the external ear canal or middle or inner ear.

American Board of Pediatrics Content Specification(s)


 Recognize the clinical findings associated with foreign body in the external ear canal

Suggested Reading
 Ansley IF, Cunningham MJ. Treatment of aural foreign bodies in children. Pediatrics.
1998;101(0638-641. Accessed January 14,2014.
 Isaacson G, Ojo A. Diagnosis and management of foreign bodies of the outer ear.
UpToDate. Available online only for subscription.
 Steele D. Foreign body removal from the external auditory ear canal. In: King C,
Henretig FM, eds. Textbook of Pediatric Emergency Procedures. 2nd ed. Philadelphia,
PA: Lippincott Williams & Willkinss 2008:589.

American academy of pediatrics 203


American Academy of Pediatrics PREP 2015

Item 67
A 5-year-old boy is a new patient brought to your office for easy bruisability. His mother noticed
an increasing number of bruises and red dots on his trunk and legs over the past few weeks
without any witnessed trauma. His weight is at the tenth percentile, but his height and head
circumference are below the third percentile. He has low-set ears, 3 café-au-lait spots on his
chest, as well as scattered bruises and petechiae on his trunk and legs. You also notice that his
thumbs are abnormal in appearance (Item Q67). The remainder of the physical examination is
unremarkable. His vital signs are stable.

The following are the results of the child's complete blood cell count:
• White blood cell count, 2,500/µL (2.5 x 109/L), with 25% polymorphonuclear leukocytes,
65% lymphocytes, 8% monocytes, and 2% eosinophils
• Hemoglobin, 11 g/dL (11 g/L)
• Mean corpuscular volume, 100 µm3 (100 fL)
• Platelet count, 25,000 x 103/ µL (25 x 109/L)
• Prothrombin time, normal
• Partial thromboplastin time, normal

You refer the patient to a specialist for further evaluation. The mother would like to know the
possible diagnoses.

Of the following, the MOST appropriate response would be that he likely


A. had a congenital infection that led to the dysmorphic features and cytopenias
B. has acute lymphocytic leukemia
C. has a genetic mutation that can eventually lead to bone marrow failure
D. has a nutritional deficiency affecting his growth and bone marrow production
E. has an X-linked condition associated with immunodeficiency

American academy of pediatrics 204


American Academy of Pediatrics PREP 2015

Item 67 Preferred Response: C


The child in the vignette has features consistent with Fanconi anemia (FA), a rare congenital
disorder associated with bone marrow failure, congenital anomalies, chromosomal instability,
and defective DNA repair. Ninety-eight percent of cases are autosomal recessive, while 2% are
X-linked recessive. Most patients with FA have macrocytosis, cytopenias, and some degree of
bone marrow dysfunction within the first decade after birth. The most common physical features,
found in 20% to 50% of patients with FA, include café-au lait spots, short stature, abnormal
thumbs and radii, and abnormal head, eyes, kidneys, and ears. It is important to detect patients
with FA to recognize the predisposition to malignancies such as leukemia (acute myeloid
leukemia and myelodysplastic syndrome) and squamous cell carcinoma. Laboratory testing to
diagnose FA involves detection of chromosomal breakage in peripheral blood cells after culture
with diepoxybutane or mitomycin C. Multiple genes involving DNA repair have been found to
lead to FA.

The only curative treatment for the bone marrow failure component of FA, at this time, is
hematopoietic stem cell transplantation. Patients with FA require modified conditioning
regimens for bone marrow transplantation because of their defect in DNA repair mechanisms.
Androgens and cytokines (eg, granulocyte-colony-stimulating factor, granulocyte macrophage-
colony-stimulating factor) have also been used in the treatment of cytopenias.

Congenital infections (eg, cytomegalovirus) can cause cytopenias in the neonatal period, but
these usually resolve over time. Leukemia and nutritional deficiencies (eg, vitamin B12 or folate)
can cause macrocytosis and cytopenias, but would not explain the dysmorphic features. Wiskott-
Aldrich syndrome is an X-linked condition characterized by immunodeficiency,
thrombocytopenia, and eczema, but typically does not have any congenital anomalies.

PREP Pearls
• Fanconi anemia is a rare congenital disorder, usually of autosomal recessive inheritance.
Associated with high incidence of bone marrow failure, malignancy, congenital
anomalies, chromosomal instability, and defective DNA repair.
• The most common physical features in patients with Fanconi anemia, include cafe-au-lait
spots, short stature, abnormal thumbs and radii, and abnormal head, eyes, kidneys, and
ears.
• The only curative treatment for the bone marrow failure component of Fanconi anemia is
hematopoietic stem cell transplantation.

American Board of Pediatrics Content Specification(s)


• Recognize clinical and laboratory findings associated with Fanconi anemia

Suggested Reading
• Eiler ME, Frohnmayer D, Frohnmayer L, Larsen K, Owen J, eds. Fanconi Anemia:
Guidelines for Diagnosis and Management. 3rd ed. Eugene, OR: Fanconi Anemia
Research Fund; 2009: 372
• Green AM, Kupfer GM. Fanconi anemia. Hematof Disco! Clin North Am.
2009;23(2):193-214. doi:10.1016/j.hoc.2009.01.008.

American academy of pediatrics 205


American Academy of Pediatrics PREP 2015

• Kee Y, D'Andrea AD. Molecular pathogenesis and clinical management of Fanconi


anemia. J ain Invest. 2012;122(11):3799-3806. doi:10,1172/ JCI58321.

American academy of pediatrics 206


American Academy of Pediatrics PREP 2015

Item 68
An 8-year-old boy has been struggling in school over the past 2 years. He recently had testing
performed by the school psychologist. His mother brought these test results in to discuss with
you. On the Wechsler Intelligence Scale for Children, he had a performance score of 88, a verbal
score of 94, and a full scale score of 92. He had a Wide Range Achievement Test composite
reading score of 89, math computation score of 83, and spelling score of 85.

Of the following, the BEST statement to make to this parent is that

A. additional evaluation is required to determine if the boy has a learning disability


B. the difference between his reading composite score and math computation score indicates
he has a learning disability
C. the difference between his verbal IQ and performance IQ scores indicates he has a
learning disability
D. the math computation score on its own indicates he has a learning disability
E. the overall lower achievement test scores relative to his IQ scores indicate he has a
learning disability

American academy of pediatrics 207


American Academy of Pediatrics PREP 2015

Item 68 Preferred Response: A


A "learning disability" is a disorder in the psychological processing of language-based
information (spoken or written) that can lead to a difficulty in reading, writing, spelling,
speaking, listening, or performing mathematics. Because school-based instruction is highly
reliant on both understanding and using language, impairment in this arena can lead to serious
difficulties in many different areas of school performance quite separate from intelligence-based
abilities. Therefore, a student may have adequate ability as measured by IQ, but poor
performance in a formal learning setting. An example of a learning disability is the case of a very
intelligent child with poor mathematical performance in school who makes rapid gains in
performance once visualization strategies are used for his mathematical instruction.

The boy in the vignette, who has somewhat lower-than-average IQ scores (average score = 100),
could be struggling in school because of his intellectual abilities rather than a learning disability.
Another interpretation is that this child's struggles in school may be unrelated to either IQ or
learning disability, but rather he may have attention-deficit/ hyperactivity disorder (ADHD) or he
may be mentally preoccupied with thoughts about stressors at home. Therefore additional
evaluation is needed to determine if the child has a learning disability.

On the Wechsler IQ test, a verbal IQ 6 points higher than the performance IQ would not raise a
suspicion of a learning disability. It would be slightly more suspicious if the verbal IQ were
significantly lower than the math score because one's language abilities are the primary pathway
through which learning can become impaired. Also if the degree of difference between verbal
and performance IQ was greater, such as greater than 15 points or 1 standard deviation, it would
be more suggestive of a specific learning problem. However, even this would not be sufficient to
diagnose a learning disability. One would still need corroborative information about functional
difficulty processing new linguistic information that is out of proportion to overall intelligence.
An example would be a child who has difficulty decoding written words but can understand the
same information if spoken.

A low math computation score on the academic achievement test (the Wide Range Achievement
Test) indicates that current math knowledge is below average, but does not explain "why." It
could be that he has had poor instruction or has had reasons to not pay attention like being
bullied.

PREP Pearls
• Intelligence tests are assessments of the potential for learning, and cannot on their own
indicate the presence of a learning disability.
• Achievement tests assess level of current knowledge. Although not diagnostic of a
learning disability, achievement test scores below that expected based on intelligence
raises the possibility of a learning disability.
• Learning disabilities are disorders of the psychological processing of language-based
information (spoken or written) that can lead to a difficulty in reading, writing, spelling,
speaking, listening, or performing mathematics.

American academy of pediatrics 208


American Academy of Pediatrics PREP 2015

American Board of Pediatrics Content Specification(s)


• Interpret the results of specialized and standardized achievement tests, with emphasis on
understanding the significance of discrepancies between categories

Suggested Reading
• Braaten EB, Norman D. Intelligence (IQ) testing. Pediatr Rev. 2006;27;403408.
doi:10.1542/pir.27-11-403.
• Shea SE. Intellectual disability (mental retardation). Pediatr Rev. 2012;33;110-121.
doi:10.1542tpir.33-3-110.

American academy of pediatrics 209


American Academy of Pediatrics PREP 2015

Item 69
A 15-year-old adolescent girl comes to the office for a pre-participation examination before
starting her soccer season. During the past year, she has gained 15 kg. Her weight is now 75 kg,
height is 165 cm, and body mass index is 27.5. Her blood pressure is taken with an appropriately
sized large adult cuff. The remainder of her physical examination is unremarkable. Laboratory
studies are obtained.

Row Low-density High-density Fasting Blood


lipoprotein Lipoprotein glucose mg/dL Pressure
mg/dL (mmot/L) mg/dL (mmol/L) (mmol/L) mm Hg
A 110 (2.85) 50 (1.29) 90 (5.0) 135/85
B 110 (2.85) 50 (1.29) 135 (7.5) 150/95
C 145 (3.76) 20 (0.52) 90 (5.0) 135/85
D 145 (3.76) 20 (0.52) 150 (8.3) 150/95
E 145 (3.76) 50 (1.29) 150 (8.3) 150/95

Of the following, the results (Item Q69) MOST likely to place this child at risk for future
coronary artery disease are displayed in
A. Row A
B. Row B
C. Row C
D. Row D
E. Row E

American academy of pediatrics 210


American Academy of Pediatrics PREP 2015

Item 69 Preferred Response: D


Risk factors for development of atherosclerotic disease and coronary artery disease include
personal and family history, as well as hyperlipidemia. It is well known that smoking, obesity,
hypertension, and diabetes are all risk factors. Early coronary artery disease in family members is
also a risk factor (men younger than 55 years of age and women younger than 65 years of age).
A previous history of Kawasaki disease is a risk factor for later onset coronary artery disease,
especially if aneurysms were noted at the time of presentation. Using a body mass index above
the 85th percentile, the prevalence of obesity is as high as 30% of the general US population,
therefore increased attention to this risk factor is warranted.

Cholesterol and lipid screening is recommended twice in all children, once between the ages of 9
and 11 years and again at 17 to 21 years of age. For any patient, if the total cholesterol is greater
than 200 mg/dL (5.18 mmol/L), high-density lipoprotein is less than 45 mg/dL (1.17 mmol/L),
and low-density lipoprotein (LDL) are greater than 130 mg/dL (3.37 mmol/L), then they are at
increased risk for development of adult cardiovascular disease. Thus for the adolescent girl in
this vignette, of the choices listed, LDL cholesterol level of 145 mg/dL (3.76 mmol/L), high-
density lipoprotein cholesterol level of 20 mg/ dL (0.52 mmol/L), fasting glucose level of 150
mg/dL (8.3 mmol/L), and blood pressure of 150/95 mm Hg would place her at the highest risk of
cardiovascular disease.

PREP Pearls
• Hypertension, obesity, diabetes, smoking, and elevated cholesterol are major risk factors
for coronary artery disease, along with a family history of early cardiovascular disease.
• Early screening for elevated cholesterol and lipids is an important part of routine health
maintenance.

American Board of Pediatrics Content Specification(s)


• Know the risk factors associated with coronary artery disease
• Understand the recommendations of the National Cholesterol Education Program for
Children

Suggested Reading
• Bonow RO. Primary prevention of cardiovascular disease: a call to action. Circulation.
2002;106(25):3140-3141. doi:10.1161/01. CIR.0000048067.86569.E1.
• Expert Panel on Integrated Guidelines for Cardiovascular Health and Risk Reduction in
Children and Adolescents. Expert panel on integrated guidelines for cardiovascular health
and risk reduction in children and adolescents: summary report. Pediatrics.
2011;128(suppl 5):S213-S256. doi:10.1542/peds.2009-2107C.
• Gidding SS. New cholesterol guidelines for children? Circulation. 2006;114:989-991.
doi:10.1161/CIRCULATIONAHA.106.647891.
• National Heart Lung and Blood Institute, US Department of Health and Human Services.
Integrated guidelines for cardiovascular health and risk reduction in children and
adolescents: the report of the expert panel. National Heart Lung and Blood Institute
website.

American academy of pediatrics 211


American Academy of Pediatrics PREP 2015

• NCEP Expert Panel on Blood Cholesterol Levels in Children and Adolescents. National
Cholesterol Education Program for Children (NCEP): highlights of the report of the
expert panel on blood cholesterol levels in children and adolescents. Pediatrics.
1992;89(3):495-501.

American academy of pediatrics 212


American Academy of Pediatrics PREP 2015

Item 70
You are seeing a 6-year-old girl in the emergency department for acute onset of abdominal pain
that has been worsening throughout the day. The pain is severe, but comes and goes according to
the parents. The patient has been vomiting for the last 2 hours. She had a stool that the mother
describes as containing blood and mucus. On physical examination, the patient has a tender
abdomen with a palpable mass in the right upper quadrant and several plaques and papules on
her legs (Item Q70).

Of the following, the BEST test to establish the diagnosis for this patient is
A. abdominal computed tomography
B. abdominal radiograph
C. abdominal ultrasonography
D. barium enema
E. barium swallow

American academy of pediatrics 213


American Academy of Pediatrics PREP 2015

Item 70 Preferred Response: C


The rash and constellation of symptoms in the girl in this vignette is consistent with Henoch-
Schonlein purpura (HSP). HSP is a leukocytoclastic vasculitis primarily affecting small blood
vessels. The European League Against Rheumatism and Paediatric Rheumatology Europe
Society criteria for HSP include purpura or petechiae with lower limb predominance and at least
1 of the following: arthritis or arthralgia, abdominal pain, histopathology demonstrating
immuno-globulin A deposition, or renal involvement demonstrated by hematuria or proteinuria.
Occasionally, the purpura are preceded by an urticarial or maculopapular rash that fades. The
vasculitic rash follows, usually within 24 hours, with a purpuric, necrotic, or deep bruising
appearance. The purpuric rash is most often present on dependent areas, but can be seen on the
arms, face, and ears.

Abdominal pain occurs in over half of HSP patients and can precede the pathognomonic rash by
as much as 2 weeks. Intestinal bleeding may occur in up to 33% of cases, and may require
evaluation by a subspecialist. The patient in this vignette has intussusception, which only occurs
in 1% to 5% of children with HSP, but is an important complication to recognize. Patients who
are suspected of having intussusception should be evaluated with abdominal ultrasonography and
a surgical consultation. Computed tomography would expose the child to a significant amount of
radiation unnecessarily. Abdominal radiography would not be diagnostic, though abnormalities
such as air fluid levels or fluid levels may be observed. A barium enema would be unlikely to
show the intussusception, because most cases in this age group are small bowel to small bowel,
and a barium swallow would not be indicated in a case of suspected intussusception.

Arthritis affects about 75% of children with HSP. Most often, the knees and ankles are affected.
The arthritis of HSP is usually oligoarticular, nondeforming, and self-limited. Approximately
15% of patients have arthritis as the presenting feature of HSP.

The most common renal manifestation of HSP is microscopic hematuria. Renal involvement is
relatively common, occurring in 20% to 60% of patients, with chronic renal impairment
occurring in 2% to 15% and end-stage renal disease in only 1% of HSP patients. Most renal
involvement presents within the first 6 weeks of disease, and 97% occur within 6 months.
Rare manifestations seen in HSP include scrotal edema, periorbital edema, hand swelling,
pulmonary hemorrhage, seizures, stroke, and mental status changes. The clinical symptoms of
HSP can last up to a month, and can recur in one-third of patients.

Management of HSP is primarily supportive, including analgesics and nonsteroidal anti-


inflammatory drugs (NSAIDs). Early use of corticosteroids in the hospital setting should be
considered in the case of gastrointestinal manifestations, particularly abdominal pain. Arthritis
should be managed with NSAIDs; however, corticosteroids may be used in severe or
nonresponsive cases. The use of corticosteroids in mild cases of HSP remains controversial. The
rare, life-threatening, or severe manifestations of HSP, including acute renal failure,
intussusception, and severe arthritis, often require management by a subspecialist.

American academy of pediatrics 214


American Academy of Pediatrics PREP 2015

PREP Pearls
• Common manifestations of Henoch-Schonlein purpura (HSP) include a vasculitic rash,
abdominal pain, and arthritis.
• Uncommon manifestations of HSP include acute renal failure, intussusception, scrotal
edema, periorbital edema, hand swelling, pulmonary hemorrhage, seizures, stroke, and
mental status changes.
• Abdominal pain occurs in 50% of patients with HSP, gastrointestinal bleeding in up to
33%, and intussusception in 1% to 5%.
• Corticosteroid therapy should be considered in patients with HSP and abdominal pain.

American Board of Pediatrics Content Specification(s)


• Recognize the typical and atypical clinical findings associated with Henoch-Schonlein
purpura, and manage appropriately

Suggested Reading
• Lanzkowsky S, Lanzkowsky L, Lanzkowsky P. Henoch-Schoenlein purpura. Pediatr Rev.
1992;13(0130-137. doi:10.1542/pir.13-4-130.
• Weiss PF. Pediatric vasculitis. Pediatr an North Am. 201459(2):407-423.
doi:10.1016/j.pc1.2012.03.013.

American academy of pediatrics 215


American Academy of Pediatrics PREP 2015

Item 71
You are volunteering as a team physician for a high school soccer tournament. During a game, a
14-year-old adolescent girl is elbowed in the head by another player. You evaluate the injured
teenager on the sideline; the girl reports dizziness, nausea, and headache, but physical
examination findings are unremarkable. You hold the girl out of the remainder of the game.
After the game, the girl reports that all symptoms resolved within 15 minutes after the injury.
She and her parents would like to know if she could participate in a tournament game later that
day.
Of the following, the MOST appropriate recommendation for this family is that same-day return
to play is
A. appropriate for this girl because physical examination was completely normal
B. appropriate for this girl because she did not lose consciousness
C. appropriate for this girl because symptoms resolved in less than 30 minutes
D. not appropriate for a child with a suspected concussion
E. not appropriate for this girl because headache was one of her symptoms

American academy of pediatrics 216


American Academy of Pediatrics PREP 2015

Item 71 S Preferred Response: D


The soccer player in the vignette has sustained a concussion. Same day return-to-play is never
appropriate for children and adolescent athletes following concussion.

The Consensus Statement on Concussion in Sport delineates the criteria for diagnosis of sports
concussion. An athlete must sustain a direct blow to the head or experience linear or rotational
acceleration of the skull as a result of a transmitted force. The direct or transmitted force must
result in symptom development, generally within several hours following injury. Symptoms fall
into l of 4 groups: somatic (eg, headache, vision changes), cognitive (eg, memory disturbance,
difficulty concentrating), mood changes (eg, irritability, depression), and sleep disturbance.

This girl's injury meets the criteria for concussion diagnosis; she experienced a direct blow to her
head and became symptomatic immediately after her injury.
The treatment of concussion includes physical rest from all exertional activities, including
physical activities with minimal risk of contact. Cognitive rest should also be implemented for a
child with concussion symptoms. Cognitive rest may include time off from school, school
accommodations (eg, limited tests and homework), and curtailing screen time (eg, watching
television, using computers or smart boards, and texting).

Initiating a step-wise gradual return to physical activities is indicated once individuals with
concussion have been asymptomatic for 24 hours, as long as the neurologic examination is
unremarkable. Additional evaluation, such as computerized neuropsychological testing, may be
used as an adjunct to determine when return to play is appropriate. Both the Clinical Report on
Sports-Related Concussion from the American Academy of Pediatrics and the Consensus
Statement on Concussion in Sport detail the appropriate progression for return to physical
activity following sports concussion.

The first 3 responses are incorrect because same day return-to-play following concussion is
contraindicated, even for individuals with short symptom duration and normal physical
examination. Loss-of-consciousness is not a criterion for the diagnosis of concussion. Headache
is the most common concussion symptom, though it is not exhibited in about 25% of individuals
with concussion. The presence or absence of headache should not be used as a marker of
concussion severity.

PREP Pearls
• Same-day return to play is never appropriate for children and adolescent athletes
following a suspected concussion.
• Loss of consciousness is not required for the diagnosis of concussion.
• Up to 25% of children with concussion do not report headache.

American Board of Pediatrics Content Specification(s)


• Understand the criteria for return to play in sports after a head injury

Suggested Reading
 Halstead ME, Walter Ka Clinical report — sport-related concussion in children and
adolescents. Pediatrics. 2010;126(3):597-615. doi:10.1542/ peds.2010-2005.
American academy of pediatrics 217
American Academy of Pediatrics PREP 2015

 McCrory P, Meeuwisse W, Aubry M. et at Consensus statement on concussion in sport:


the 4th International Conference cm Concussion in Sport held in Zurich, November 2012.
Ciin 1 Sport Med. 2013;23(2)10-117. doi:10.1097/15M.0b013e318281367cf.

American academy of pediatrics 218


American Academy of Pediatrics PREP 2015

Item 72
A 5-year-old previously healthy girl presents to your office with a 1-day history of nonbloody
diarrhea. Her mother reports that she developed fever to 38°C, vomited 3 times yesterday, and
then began having watery stools without mucus. The mother estimates that there have been 10
episodes of diarrhea in the last 24 hours. She is unable to determine the child's urine output
because of the child's frequent diarrheal stools. The girl has not been eating, but she has been
drinking water readily.
On physical examination, her temperature is 38°C, heart rate is 130 beats/min, respiratory rate is
28 breaths/min, and blood pressure is 90/60 mm Hg. The child is tired, but nontoxic appearing.
She is irritable, but consoled by her mother. Mucous membranes are mildly dry and capillary
refill is 2 seconds. The remainder of the physical examination is unremarkable.

Of the following, the MOST appropriate next step in the management of this patient's condition
is to prescribe
A. bismuth subsalicylate
B. intravenous fluid bolus
C. kaolin-pectin
D. loperamide
E. oral electrolyte replacement

American academy of pediatrics 219


American Academy of Pediatrics PREP 2015

Item 72 Preferred Response: E


The child described in the vignette has evidence of mild to moderate dehydration, most likely
caused by acute viral gastroenteritis, and should be treated with oral electrolyte replacement.
This is the preferred first-line therapy for fluid and electrolyte losses in children without severe
dehydration, independent of the etiology of dehydration or the patient's age. The desired
properties of an oral replacement solution include:
• Total osmolarity near 245 mmol/L
• Glucose concentration less than 20 g/L
• Sodium concentration 75 mEq/L (75 mmol/L)
• Equivalent millimolar concentrations of glucose and sodium
• Potassium concentration 20 mEq/L (20 mmol/L)
• Base (HCO3) concentration near 30 mEq/L (30 mmol/L)

Properties of commonly used rehydration solutions versus various beverages are shown (Item
C72,).

Children with severe dehydration should be treated with an intravenous fluid (IVF) bolus of
normal saline, followed by IVF therapy. In cases of severe dehydration, careful calculation of
fluid and electrolyte losses to determine the rate of fluid and electrolyte replacement is necessary
to avoid serious complications. Children who fail oral electrolyte replacement should be treated
with IVF therapy until oral electrolyte replacement therapy is tolerated.

Most commonly, acute diarrheal illnesses in children are self-limited, caused by viral pathogens,
and do not require pharmacologic therapy. Data do not support the efficacy of antidiarrheal

American academy of pediatrics 220


American Academy of Pediatrics PREP 2015

agents in the treatment of acute diarrhea or gastroenteritis in children. Bismuth subsalicylate is


an antisecretory agent that can cause toxicity from salicylate absorption. It is rarely used in the
management of diarrheal illness in children.

Loperamide, an antimotility agent, can be used in children 2 years of age and older for chronic
diarrhea associated with inflammatory bowel disease, and for chronic functional diarrhea to
decrease the volume of stool from an ileostomy. The side effects of loperamide can be severe,
including lethargy, paralytic item, toxic megacolon, coma, and even death. Adsorbents such as
kaolin-pectin can be used in children 3 years of age and older. They bind mucus and toxins in the
gastrointestinal tract and increase the bulk of feces; however, kaolin does not decrease fluid or
electrolyte losses or the number of stools passed, and data from high-quality clinical trials fail to
demonstrate its efficacy in the treatment of acute pediatric gastroenteritis.

PREP Pearls
• Oral electrolyte replacement is the preferred first-line therapy for fluid and electrolyte
losses in children with mild to moderate dehydration.
• Acute diarrhea illnesses in children are usually self-limited, caused by viral pathogens,
and do not require pharmacologic therapy.
• Bismuth subsalicylate is an antisecretory agent that can cause toxicity from salicylate
absorption.
• Adverse effects of loperamide can include lethargy, paralytic ileus, toxic megacolon,
coma, and even death.
• Adsorbents such as kaolin-pectin bind mucus and toxins in the gastrointestinal tract and
increase the bulk of feces, but do not decrease fluid or electrolyte losses or the number of
stools passed.

American Board of Pediatrics Content Specification(s)


• Apply age-appropriate guidelines in the use of anti-diarrhea medicines

Suggested Reading
• US Centers for Disease Control and Prevention. Managing acute gastroenteritis among
children: oral rehydration, maintenance, and nutritional therapy from the centers for
disease control and prevention. MMWR Recomm Rep. 2003;52(RR-16):1-16.
• Granado-Villar D, Cunill-De Sautu B, Granados A. Acute gastroenteritis. Pediatr Rev.
2012;33(11):487-495. do i:10.1542/p ir.33 -11 -487.
• Guarino A, Albano F, Ashkenazi S, et al. European Society for Paediatric
Gastroenterology, Hepatology, and Nutrition/European Society for Paediatric Infectious
Diseases evidence-based guidelines for the management of acute gastroenteritis in
children in Europe: executive summary. Pediatr Gastroenterol Nu tr. 2008;46(5):619-621.
doi:10.1097/ MPG.0b013e31816e219e.
• King CK, Glass R, Bresee JS, Duggan C. Managing acute gastroenteritis among children:
oral rehydration, maintenance, and nutritional therapy. MMWR Recomm Rep.
2003152(RR-16):1-16.
• World Health Organization. The treatment of diarrhea: a manual for physicians and other
senior health workers: Geneva, Switzerland: World Health Organization, 2005.

American academy of pediatrics 221


American Academy of Pediatrics PREP 2015

Item 73
A 15-year-old adolescent boy presents to your office for evaluation of a persistent cough,
hoarseness, and chest discomfort. He was diagnosed 2 weeks ago with atypical pneumonia based
on the presence of low-grade fever and scattered wheezes on the patient's lung examination. He
completed a course of azithromycin; however, his cough has increased in frequency over the past
2 weeks. He now reports intermittent shortness of breath, especially when he lies down. He slept
in a recliner last night because of trouble breathing when he attempted to lie flat on his bed, His
mother remarks that his face has seemed more "puffy" than usual over the past few days. He has
no significant past medical history.
His height and weight are at the 75th percentile for his age, His temperature is 37.9°C, heart rate
is 92 beats/min, respiratory rate is 26 breaths/min, blood pressure is 128/74 mm Hg, and pulse
oximetry is 98% on room air.
On physical examination, the patient appears somewhat anxious, but is in no acute distress. His
face appears flushed and slightly swollen. Cardiac examination reveals a regular rhythm without
a murmur, rub, or gallop. Lung examination reveals scattered inspiratory and expiratory wheezes
in the boy's upper lung fields bilaterally. He has shortness of breath worsened by deep breathing.
The adolescent's voice sounds hoarse and he is coughing frequently during the examination. He
seems most comfortable when sitting up on the examination table and leaning forward. His
abdominal examination is benign and without tenderness, masses, or hepatosplenomegaly. His
extremities are well perfused with no peripheral edema. You do note left supraclavicular
lymphadenopathy. The remainder of your physical examination yields no additional remarkable
findings.
As soon as you complete your clinical assessment, his mother asks you to explain what you think
is causing his symptoms.
Of the following, the MOST likely diagnosis is
A. anterior mediastinal mass
B. bronchiectasis
C. cardiomyopathy with congestive heart failure
D. obstructive sleep apnea
E. pericarditis

American academy of pediatrics 222


American Academy of Pediatrics PREP 2015

Item 73 Preferred Response: A


The adolescent boy in the vignette has symptoms and clinical findings that are most consistent
with the diagnosis of an anterior mediastinal mass, complicated by development of superior vena
cava (SVC) syndrome.

The onset of generalized facial swelling in this patient, combined with facial plethora, is
suggestive of SVC syndrome, which is very rare in children, but commonly results from
underlying mediastinal malignancy. In 1 case series of 24 children with SVC syndrome, the most
common associated malignancies were non-Hodgkin lymphoma and acute lymphoblastic
leukemia with mediastinal mass.

Symptoms and findings that have been reported in children with anterior mediastinal masses
associated with SVC syndrome include:
• Swelling of the face, neck, or upper chest
• Facial plethora or cyanosis
• Prominent neck and upper chest veins
• Cough
• Dyspnea
• Orthopnea
• Hoarse voice or stridor
• Chest pain
• Wheezing
• Pleural effusions
• Pericardial effusion

For the adolescent in the vignette, the findings of persistent cough, hoarseness, chest pain, and
wheezing with progressive development of breathing trouble, orthopnea, and facial edema and
"flushing" should prompt consideration of anterior mediastinal mass as his diagnosis. Plain
radiographs of the chest are effective in identifying the presence of the vast majority of
mediastinal masses, and should be obtained in any child in whom this diagnosis is suspected.

The patient in the vignette has unremarkable cardiac examination findings and no hepatomegaly
or pulmonary crackles on examination, making the diagnosis of acquired cardiomyopathy with
congestive heart failure less likely.

Bronchiectasis, characterized by abnormal dilation and distortion of the bronchial tree, resulting
in chronic obstructive lung disease, is unlikely in this boy. He had no significant medical history
before the onset of his current symptoms, and crackles and rhonchi, which are often heard with
bronchiectasis, are absent on lung examination.

Although the majority of patients with pericarditis report chest pain, which is typically improved
by sitting up and leaning forward, the diagnosis of pericarditis would not explain the facial
swelling, plethora, persistent cough, or hoarseness noted on his examination. A pericardial
friction rub, which is a physical finding highly specific for the diagnosis of pericarditis, was not
noted on examination of the adolescent in the vignette.

American academy of pediatrics 223


American Academy of Pediatrics PREP 2015

Obstructive sleep apnea, which is characterized by episodes of partial or complete airway


obstruction during sleep, is not consistent with the persistent and progressive symptoms in the
previously healthy adolescent described in the vignette.

PREP Pearls
• Superior vena cava (SVC) syndrome is very rare in children, but most commonly results
from underlying mediastinal malignancies.
• Onset of generalized facial swelling combined with facial plethora should prompt
clinicians to consider the diagnosis of SVC syndrome.
• Plain radiography of the chest is effective in identifying the vast majority of mediastinal
masses, and should be obtained in any child in whom a mediastinal mass is suspected.

American Board of Pediatrics Content Specification(s)


• Recognize the clinical findings associated with a chest mass

Suggested Reading
• Ingram L, Rivera GK, Shapiro DN. Superior vena cava syndrome associated with
childhood malignancy: analysis of 24 cases. Med Pediatr °mot. 1990;18(0476-481.
• Issa PY, Brihi ER, Janin Y, Slim MS. Superior vena cava syndrome in childhood: report
of ten cases and review of the literature. Pediatrics. 1983;71(3);337-341.
• Lee DA, Margolin J. Emergencies in pediatric cancer patients. UpToDate. Available
online only for subscription.

American academy of pediatrics 224


American Academy of Pediatrics PREP 2015

Item 74
You are asked to assess a newborn who appears dusky 4 hours after birth. The mother underwent
a natural childbirth at term, including immediate skin to skin bonding and delayed clamping of
the umbilical cord. The pregnancy was uncomplicated. Physical examination reveals a ruddy,
appropriate for gestational age newborn with perioral cyanosis, unlabored tachypnea, and mild
jitters. The temperature is 37°C, the heart rate is 160 beats/min, the respiratory rate is 70
breaths/min, and the oxygenation saturation in room air is 95%. Further evaluation reveals a soft
I/VI murmur at the left sternal border, clear breath sounds bilaterally, and no abdominal masses.
The mother reports that her baby has not latched well onto the breast since birth.

Of the following, the MOST appropriate next step management is to obtain a(n)
A. blood glucose level
B. chest radiograph
C. echocardiogram
D. serum calcium
E. venous hematocrit

American academy of pediatrics 225


American Academy of Pediatrics PREP 2015

Item 74 Preferred Response: A


A blood glucose value should be obtained immediately, as the newborn in the vignette is
demonstrating both symptomatic hypoglycemia and polycythemia. Although the hypoglycemia
may be secondary to polycythemia, glucose homeostasis must be achieved without delay. It is
essential to monitor blood glucose values in neonates with polycythemia, because up to 40%
may have hypoglycemia. If required, an intravenous glucose infusion may also assist in the
management of polycythemia through hydration and dilution.

Polycythemia, defined as a venous hematocrit value greater than 65% or a hemoglobin value
greater than 22 g/dL (220 g/L), has been associated with delayed cord clamping. Recent studies
have centered on the timing of cord clamping after birth because of the perceived neonatal
benefits of increased blood volume and iron stores, which reduce the risk of physiologic anemia.
A committee opinion of the American College of Obstetricians and Gynecologists does not
provide the ideal timing for cord clamping because of the lack of evidence in full-term newborns.
Some proponents advocate for delayed clamping up to 2 min after delivery due to the existing
studies, acknowledging the increased risk of polycythemia associated with this practice.

The most common features associated with polycythemia in the newborn include cyanosis, poor
feeding, and hypoglycemia. The increased blood viscosity may lead to decreased
microcirculatory flow. This diminished flow compromises organ function, leading to neurologic,
metabolic, cardiac, renal, and respiratory complications (Item C74, page C-61). A newborn with
polycythemia may look cyanotic because the increased amount of hemoglobin coupled with
normal oxygen extraction allows the deoxyhemoglobin level to exceed 5 g/dL (50 g/L).
Although this level of deoxyhemoglobin gives the clinical appearance of cyanosis, the neonate is
not hypoxic.

Item C74. Findings Associated With Polycythemia


Cardiorespiratory Tachypnea
Congestive heart failure
Apnea
Cyanosis
Central nervous system Lethargy
Jitteriness and tremors
Seizure activity
Stroke
Gastrointestinal Poor feeding
Necrotizing enterocolitis
Genitourinary Renal insufficiency
Renal vein thrombosis
Priapism
Hematologic Plethora
Thrombocytopenia
Disseminated intravascular
coagulation
Hyperbilirubinemia

American academy of pediatrics 226


American Academy of Pediatrics PREP 2015

Metabolic Hypoglycemia
Hypocalcemia

The treatment of polycythemia remains controversial. Asymptomatic newborns with hematocrit


values between 60% and 70% may be hydrated aggressively with enteral feedings or intravenous
fluid, with careful monitoring for 24 to 48 hours. In symptomatic newborns with polycythemia,
partial volume exchange transfusion may be performed to decrease the blood viscosity by
replacing a calculated blood volume with normal saline. Necrotizing enterocolitis is a known
complication of partial volume exchange transfusion.

The newborn in the vignette is manifesting symptomatic hypoglycemia and his blood glucose
should be checked immediately to determine if parenteral glucose is required. Further evaluation
should then be pursued because of the clinical suspicion of polycythemia to guide further
management, including obtaining a venous hematocrit and serum calcium level. Chest
radiography may be performed at this time because of the tachypnea, as other causes should
always be considered. Echocardiography may be considered if the clinical suspicion of
congenital heart disease exists.

PREP Pearls
• Polycythemia is most commonly associated with cyanosis, poor feeding, and
hypoglycemia.
• Delayed cord clamping is associated with polycythemia.

American Board of Pediatrics Content Specification(s)


• Recognize the risks associated with polycythemia in newborn infants, and manage
appropriately

Suggested Reading
• American College of Obstetricians and Gynecologists. Committee opinion No. 543:
timing of umbilical cord clamping after birth. ❑bstet Gyneroi. 2012;120:1522-1526.
doi:10.1097/01.AOG.0000423817.47165.48.
• Garcia-Prats JA. Neonatal polycythemia. UpToDate. Last updated October 29, 2013.
• Hutton EK, Hassan ES. Late vs early clamping of the umbilical cord in full-term infants:
systematic review and meta-analysis of controlled trials. JAMA. 2007;297( 11):1241-
1252. doi:10.1001/jama.297.11.1241.
• Kates EH, Kates JS. Anemia and polycythemia in the newborn. Pecliatr Rev.
2007;28{1):33-34. doi:10.1542Ipir.28-1-33.
• Remon ii, Raghavan A, Maheshwari A. Polycythemia in the newborn. NeoReviews.
2011;12{1):e20-e28. doi:10.1542/ne0.12-1-e20.

American academy of pediatrics 227


American Academy of Pediatrics PREP 2015

Item 75
You are seeing a 2-week-old, full-term male newborn for a routine health supervision visit. He
had a prenatal diagnosis of myelomeningocele, and at birth, was noted to have lower extremity
flaccid paralysis. He underwent surgical repair of the defect on the second day after birth, and he
was discharged home 7 days after birth. The discharge paperwork reports an occipital frontal
head circumference of 36 cm and bilateral lower extremity flaccid paralysis. At home, the infant
has been afebrile, alert, breathing, and eating normally. On physical examination, he has an
occipital frontal head circumference of 37.5 cm, a temperature of 37°C, a blood pressure of
80/54 mm Hg, a heart rate of 110 beats/min, and a respiratory rate of 42 breaths/min. The
lumbosacral wound site is nonerythematous and without exudate. The remainder of his
examination is unchanged from the hospital discharge examination.

Of the following, the BEST next step in management of this infant is to


A. obtain magnetic resonance imaging scans of the head
B. obtain ultrasonograms of the spine
C. recheck occipital frontal head circumference in 1 week
D. refer to the nearest emergency department
E. refer to the pediatric neurosurgery clinic

American academy of pediatrics 228


American Academy of Pediatrics PREP 2015

Item 75 Preferred Response: C


The infant in the vignette is at risk for hydrocephalus because of his underlying diagnosis of
myelomeningocele. His occipital frontal head circumference and his physical examination
should be monitored weekly until his head growth is stable, which may take weeks or months. If
the head circumference is increasing rapidly and there are signs of hydrocephalus, such as
difficulty feeding, difficulty breathing, or encephalopathy, then he would require neurosurgical
intervention. This infant is asymptomatic, so his head circumference can be rechecked in 3 to 10
days.

An ultrasonogram of the spine does not show hydrocephalus, so that study would not be helpful
in this case. The infant's flaccid lower extremities are a result of the myelomeningocele, not of
hydrocephalus; it may be prudent to confirm with the parents that his current lower extremity
movement is the same as it was on discharge from the hospital.

Magnetic resonance imaging of the head would show signs of hydrocephalus, but is a lengthy
and expensive study that would require sedation. For urgent clinical concerns, ultrasonography is
the preferred imaging modality.

This infant has asymptomatic increasing head size, and so does not require referral to either an
emergency department or to the pediatric neurosurgery clinic at this time.

Abnormally increasing occipital frontal head circumference is suggestive of an acute intracranial


process, such as brain tumor or hydrocephalus. Brain tumors often cause additional symptoms
referable to the region of brain involved. For instance, posterior fossa tumors can cause ataxia, in
addition to increasing occipital frontal head circumference and intracranial pressure.
Hydrocephalus can cause increasing occipital frontal head circumference. Risk factors for
hydrocephalus include intraventricular hemorrhage, meningitis, or brainstem malformation such
as aqueductal stenosis. Rare genetic disorders such as X-linked hydrocephalus can also cause
hydrocephalus. Close monitoring of the occipital frontal head circumference and clinical
examination in infants with risk factors is the best screening tool to detect early signs of
hydrocephalus.

PREP Pearls
• Myelomeningocele is a risk factor for hydrocephalus.
• In infants with repaired myelomeningocele, asymptomatic head enlargement can be
monitored clinically.

American Board of Pediatrics Content Specification(s)


• Understand the risk factors for hydrocephalus

Suggested Reading
• Burke R, Liptak GS, the Council on Children With Disabilities. Providing a primary care
medical home for children and youth with spina bifida. Pediatrics. 2011;128(6):e1645-
e1657. doi:10.1542/peds.2011-2219.
• McLone D, Bowman R. Overview of the management of
• myelomeningocele (spina bifida). UpTaDate. Available online only for subscription.
American academy of pediatrics 229
American Academy of Pediatrics PREP 2015

• Purugganan OH. Abnormalities in head size. Pediatr Rev. 2606;27(12): 473-476.


doi:I0.1542/pir,27-12-473.

American academy of pediatrics 230


American Academy of Pediatrics PREP 2015

Item 76
A 14-year-old adolescent boy presents to your office seeking a medication to help him grow and
develop muscles. He has been working out for 1 year without much effect. On physical
examination, his height is 147.3 cm (4 ft, 10 in [adjusted midparental height, 185.4 cm (6 ft, 1
in)]), he weighs 48.1 kg, and his testes are only 3 mL in volume. The child's growth rate is 5 cm
(2 in) per year. Results of laboratory testing are unremarkable, including complete blood cell
count, erythrocyte sedimentation rate, electrolytes, prolactin, insulin like growth factor 1, celiac
panel, and thyroid function testing. Luteinizing hormone, follicle-stimulating hormone, and
testosterone levels are at prepubertal levels. Bone age shows a delay of 2 years from
chronological age.
Although he appears healthy, the boy is distraught about his young appearance compared with
his peers. The boy has been evaluated for this previously, when he was told that he will be "a late
bloomer" and that this is normal. The boy's father was very short as a boy, and was given
medication to make him taller. The family has come to your office seeking a second opinion.

Of the following, the BEST treatment option for this child is to


A. counsel the family that treatment should not begin before 16 years of age
B. obtain a brain magnetic resonance imaging study to evaluate for central causes of poor
growth
C. offer an endocrinology referral for a 1-year to 2-year course of growth hormone therapy
D. offer an endocrinology referral for a 4-month course of testosterone therapy
E. recommend a high-calorie nutrition supplement to enhance growth

American academy of pediatrics 231


American Academy of Pediatrics PREP 2015

Item 76 I-C Preferred Response: D


The boy in the vignette has constitutional delayed puberty (CDP). His family history of CDP
normal prepubertal growth velocity (5 cm/year), and normal screening laboratory test results
suggest he does not have an underlying pathological disorder. Although he had been evaluated
before and was correctly told he was going to be a "late bloomer" with delayed puberty, this
answer does not acknowledge the patient's psychosocial concerns or review the relatively simple,
safe, and effective treatments that could be offered to alleviate them. This patient should be
referred to a pediatric endocrinologist to review treatment options. It is not necessary to refer
boys or girls who clearly are delayed, but are showing true evidence of pubertal progression on
physical examination.

Many late-maturing boys are concerned about how short and underdeveloped they are relative to
their peers. Teasing and low self-esteem are frequently reported. Studies demonstrate that many
feel that their growth delay negatively affected their success at school, work, or socially, and
80% of those affected would have preferred to have had treatment to advance their growth and
pubertal delay. Therefore, for boys approaching 14 years of age and girls who have reached 13
years of age without showing significant physical changes of puberty, endocrinology referral for
evaluation and treatment is appropriate. Typical initial evaluation for delayed puberty includes
luteinizing hormone/follicle-stimulating hormone, testosterone, and prolactin. Even when there is
no underlying medical condition, therapy is effective, safe, and inexpensive. Eighty percent of
boys who are offered treatment accept it, suggesting they are significantly concerned about their
pubertal delay.

For boys with CDP, monthly intramuscular injections of 100 mg testosterone in oil (eg,
testosterone enanthate) for 4 months usually causes a significant increase in linear growth,
weight, penile length, and pubic hair development. One month after injections are complete,
mean linear growth is 3.8 cm from the start of treatment, a significant increase in height for
many of these children. A few months later, testicular size begins to increase, and usually no
further treatments are needed. Low-dose oral androgens are also an option (oxandrolone of 2.5
mg/day) and are sometimes used when a less dramatic effect on growth and maturation is
desired. For boys with permanent hypogonadism, lower doses of testosterone (50 mg
intramuscularly once monthly) are usually started and then gradually increased every 6 months
until a full adult replacement dose is reached.

Counseling the family that the benefits of treatment do not outweigh the risks does not
adequately address the child's impatience to start growing, and for a boy such as the child in this
vignette with significant underlying psychosocial concerns, treatment can easily be offered.
Several studies have shown that despite rapidly advancing growth with androgen treatment, final
height is not affected when treating adolescent boys aged 14 years and older.

A brain magnetic resonance image to evaluate short stature is unlikely to show any significant
findings, given the child has a normal prepubertal growth rate and likely has CDP. Similarly,
treatment with growth hormone would not be the best first choice because this patient is clearly
not growth hormone deficient and would most likely benefit significantly from a short course of
testosterone therapy to advance his pubertal status. A high-calorie diet will likely have minimal
impact on his overall growth and fails to address his main concerns.
American academy of pediatrics 232
American Academy of Pediatrics PREP 2015

PREP Pearls
 For boys with constitutional delayed puberty (CDP) who are impatient to start growing, a
short course of androgen therapy can significantly improve growth and maturation and
alleviate psychosocial concerns.
 After discussing treatment options, most boys with CDP will opt to begin androgen
treatment, suggesting many have unaddressed psychosocial concerns.

American Board of Pediatrics Content Specification(s)


• Recognize the clinical features associated with a delay in sexual maturation of various
causes
• Identify the causes of delayed puberty
• Recognize the psychosocial risks associated with delayed puberty
• Plan the appropriate evaluation of an adolescent boy or girl who has no signs of the onset
of puberty

Suggested Reading
 Allen DB, Cuttler L. Clinical practice: short stature in childhood--challenges and choices.
N EngIJ Med. 2013;368(13)1220-1228. doi:10.1056/ NEJMcp1213178.
 Crowne EC, Shalet SM, Wallace WH, Eminson DM, Price DA. Final height in boys with
untreated constitutional delay in growth and puberty Arch Dis Child. 1990;65(10:1109-
1112. doi:10.1136/adc.65.10.1109.
 Kaplowitz PB. Delayed puberty. Pediatr Rev. 2010;31(5):189-195. doi:10.1542/pir.31-5-
189.

American academy of pediatrics 233


American Academy of Pediatrics PREP 2015

Item 77
You are the primary care physician for newborn twins born at 33 weeks' gestation and
discharged from the neonatal intensive care unit after a 4-week stay. They required oxygen for 3
days after birth, but little other respiratory support. They were diagnosed with gastroesophageal
reflux disease. They have occasional episodes of coughing and gagging without color change.
They are otherwise well.

Of the following, the BEST advice to give the parents is to place the twins in
A. prone sleep position
B. side sleep position
C. sitting devices, such as car seats, during sleep
D. sleep positioners to keep their heads elevated
E. supine sleep position

American academy of pediatrics 234


American Academy of Pediatrics PREP 2015

Item 77 S Preferred Response: E


Since the introduction of the "Back to Sleep" campaign in 1994, the incidence of sudden infant
death syndrome (SIDS) has decreased by half. The American Academy of Pediatrics (AAP)
policy statement on SIDS and other sleep-related infant deaths provides guidance on appropriate
safe sleep recommendations. Evidence strongly supports that the supine sleep position is the
safest, and is recommended in all but a very limited number of cases. In the past, the side sleep
position was advocated to prevent aspiration, but no data support this as protective, but evidence
shows that the SIDS rate is higher for children sleeping in this position. Infants with
gastroesophageal reflux disease should be placed supine to sleep, unless they have an upper
airway disorder that impairs airway protective mechanisms, in particular, certain types of
laryngeal clefts. The association between prone sleep position and SIDS is at least as strong, and
possibly stronger for preterm infants than for term infants. Therefore, preterm infants should
sleep in the supine position as soon as clinical status is stable and at least by 32 weeks
postmenstrual age.

Although SIDS has shown a dramatic decrease, the number of deaths attributed to other sleep-
related causes, including suffocation and strangulation, quadrupled from 1984 to 2004. Many
experts use the term sudden unexpected infant death syndrome (SUIDS) rather than SIDS to
describe deaths of young infants not attributed to a known condition or injury, particularly those
occurring in a sleep environment. In recognition of other risks in the sleep environment, "Back to
Sleep" campaign messages have expanded to include a more comprehensive promotion of safe
sleep. All infants should sleep on a firm, properly fitted mattress in a crib, bassinet, or approved
play yard. The sleep surface should be free of soft objects or loose bedding (including quilts,
blankets, bumpers, sheepskins, toys, etc) that can obstruct the airway. Wedges and positioners
should not be used. Sitting devices such as car seats are not recommended for routine sleep and
can increase the occurrence of gastroesophageal reflux and positional plagiocephaly. Although
room sharing with the parents for at least the first several months of life decreases the risk of
SIDS by as much as 50% and is therefore recommended, bed sharing is more controversial. At
present, the AAP recommends against bed sharing, citing a summary odds ratio for SIDS of 2.88
with bed sharing (95% confidence interval: 1.99-4.18). One of the new recommendations that
pediatricians should relay to parents and to nursery staff is that bed sharing by twins and higher
order multiples leads to several risk factors for SUIDS including overheating, rebreathing, side
positioning, and accidental suffocation, and therefore increases their risk of sleep-related death.
Data that show the benefits of bed sharing by twins are limited; therefore, they should be placed
on separate surfaces for sleep, starting in the newborn nursery. In-bed devices marketed to make
bed sharing safe are not recommended.

Breastfeeding has been shown to be protective against SIDS and should be encouraged. Tobacco,
alcohol, and drug use, both during pregnancy and after birth, increase SIDS risk and are
discouraged. Epidemiologic evidence indicates that pacifier use may be protective against SIDS,
and their use should be considered. Home apnea alarms do not protect against SIDS and are
discouraged. There are limited data to allow for reliable recommendations regarding other
factors that have been suggested as having an impact on sudden death, including ventilation,
swaddling, and environmental toxins.

American academy of pediatrics 235


American Academy of Pediatrics PREP 2015

PREP Pearls
• Except in a very few instances, supine is the safest sleep position for an infant including
those with gastroesophageal reflux.
• The safest place for an infant to sleep is on a firm, well-fitted mattress in an approved
crib, bassinet, or play yard free of other objects and located in the parents' room.
• Current American Academy of Pediatrics recommendations extend beyond "Back to
Sleep" to address other sleep environment factors.
• Bed sharing, while controversial, is not recommended, and even twins should sleep on
separate sleep surfaces.

American Board of Pediatrics Content Specification(s)


• Counsel parents regarding prevention of sudden infant death syndrome

Suggested Reading
• Moon RY, Fu L. Sudden infant death syndrome: an update. Pediatr Rev. 2012;33(7):314-
320. doi:10.1542/pir.33-7-314.
• Task Force on Sudden Infant Death Syndrome. Policy statement: SIDS and other sleep-
related infant deaths: expansion of recommendations for a safe infant sleeping
environment. Pediatrics. 2011;128(5):1030-1039. doi:10.1542/peds.2011-2284.
• Task Force on Sudden Infant Death Syndrome, Moon RY. SIDS and other sleep-related
infant deaths: expansion of recommendations for a safe infant sleeping environment.
Pediatrics, 2011;128(5):e1341-e1367. doi:10.1542/peds.2011-2285.

American academy of pediatrics 236


American Academy of Pediatrics PREP 2015

Item 78
You are seeing a 17-year-old adolescent girl in your office for follow-up of her asthma that now
requires daily medications. She has not been adhering to her treatment regimen. She has
maintained her school grades and has a clear understanding of her illness. She says she is not
depressed, but does admit to being stressed with homework and college applications. You
consider ways to help her comply with the treatment plans.

Of the following, the intervention MOST likely to be successful for this patient is to
A. encourage religiosity and church attendance
B. enroll her in a support group
C. have her mother supervise her treatment plan
D. mark a calendar after each dose taken
E. set her cellphone alarm to medication use times

American academy of pediatrics 237


American Academy of Pediatrics PREP 2015

Item 78 I-C Preferred Response: E


Nonadherence to a therapeutic plan is the commonest reason for failure of therapy. This is an
especially big issue with an adolescent patient. They may misunderstand the instructions, forget,
or choose to ignore advice and the therapeutic plan. Adherence is influenced by a number of
factors including the following: attitude and beliefs about the condition being treated (eg, the
severity of the condition, benefits, and adverse effects of treatment), the complexity of the
regimen, perception of the ability to follow through on the plan, availability of social support, the
cultural context, and emotional status, particularly if the patient is depressed. In addition, the
relationship with the physician is very important, because it enables the development of a
mutually agreed upon therapeutic plan.

Collaborating on a plan will work better than dictating the plan and using aids that the patient is
comfortable with are crucial. Most adolescents are comfortable with technology. The use of the
alarm on her cellphone, for the girl in the vignette, as a cue to take medications would enhance
her self-efficacy and thus her motivation to adhere with the plan. The use of a calendar is less
helpful, as most adolescents do better with an active reminder. Religiosity and church attendance
are a reflection of a conservative outlook and indicate that a teenager is less likely to be
rebellious, but this alone does not indicate she will adhere to a regimen. No single intervention
will work with all patients. Social support can also be helpful. This includes support from a
group or a parent. The involvement of a parent is very important, but an adolescent who will
soon be on her own needs help in achieving her independence, rather than remaining dependent
on a parent as the only means to enhance adherence.

PREP Pearls
• The commonest reason for failure of therapy is lack of adherence
• An understanding of the many factors that influence adherence is crucial.
• Using technology to create reminders has been shown to be effective in improving
adherence.

American Board of Pediatrics Content Specification(s)


• Understand factors that can affect adherence to health maintenance activities by
adolescents
• Understand how to improve adherence to medical regimens by adolescent patients,
including those with chronic illness, and the barriers to such adherence

Suggested Reading
• Arrington-Sanders R. Adherence. Pediatr Rev. 2009;30(2):e9-e10. doi:10.1542/pir.30-2-
e9).
• de Jongh T, Gurol-Urganci I, Vodopivec-Jamsek V, Car J, Atun R. Mobile phone
messaging for facilitating self-management of long-term illnesses. Cochrane Database
Syst Rev. 2012;12:CD007459. doi:10.1002/14651858. CD007459.pub2.
• Hieftje K, Edelman EJ, Camenga DR, Fiellin LE. Electronic media-based health
interventions promoting behavior change in youth: a systematic review. JAMA Pediatr.
2013;167(6):574-580. doi:10.1001/ jamapediatrics.2013.1095.

American academy of pediatrics 238


American Academy of Pediatrics PREP 2015

• Horvath T, Azman H, Kennedy GE, Rutherford GW. Mobile phone text messaging for
promoting adherence to antiretroviral therapy in patients with HIV infection. Cochrane
Database Syst Rev. 2012;3:CD009756. doi:10.1002/14651858.CD009756.
• Perrin JM, Gnanasekaran S, Delahaye J. Psychosocial aspects of chronic health
conditions. Pediatr Rev. 2012;33(3):99-109. doi:10.1542/pir.33-3-99.
• Salema NE, Elliott RA, Glazebrook C. A systematic review of adherence-enhancing
interventions in adolescents taking long-term medicines. 1 Adolesc Health.
2011;49(5):455-466. doi:10.1016/j.jadohealth.2011.02.010.
• Taddeo D, Egedy M, Frappier J-Y. Adherence to treatment in adolescents. Paediatr Child
Health. 2008;13(1):19-24. Accessed February 21, 2014.

American academy of pediatrics 239


American Academy of Pediatrics PREP 2015

Item 79
An 18-year-old female patient comes to your office for evaluation of a 6-month history of daily
nasal congestion, cough, rhinorrhea, sneezing, and postnasal drainage. Her symptoms are year-
round without any seasonal worsening. She has not been able to identify any triggers. She has
tried different medications such as cetirizine, intranasal fluticasone spray, and intranasal
azelastine with some improvement of symptoms. She denies use of over-the-counter nasal
decongestants. She uses aspirin for an occasional migraine headache, perhaps once or twice a
month. Apart from taking twice-daily enalapril for the past 9 months for control of essential
hypertension, she denies use of any other medication. She denies being sexually active and has
had regular menstrual cycles. She has not had any recent changes in weight, appetite, energy, or
constitutional symptoms.
On physical examination, her weight is at the 50th percentile and height at 60th percentile for
age. Her blood pressure is 110/80 mm Hg. She has bilateral edematous turbinates, with mildly
erythematous mucosa and dear secretions in her nares. The rest of her physical examination
findings are unremarkable.

Of the following, the MOST likely cause of this patient's symptoms is

A. angiotensin converting enzyme inhibitor use


B. aspirin sensitivity
C. hormonal
D. nonallergic rhinitis with eosinophilia
E. rhinitis medicamentosa

American academy of pediatrics 240


American Academy of Pediatrics PREP 2015

Item 79 Preferred Response: A


Chronic rhinitis can be due to many causes (Item C79), and include those caused by allergic,
nonallergic, mixed, hormonal, occupational, rhinitis medicamentosa, systemic medication-
induced, atrophic, systemic-disease related, and those associated with structural anomalies such
as tumors, foreign bodies, and deviated nasal septum. The patient in the vignette has chronic
rhinitis, most likely triggered by her use of enalapril, a long-acting angiotensin converting
enzyme (ACE) inhibitor. Her symptoms of daily nasal congestion, rhinorrhea, sneezing, and
postnasal drainage suggest a diagnosis of rhinitis. Her symptoms are of recent onset and do not
have a seasonal pattern or identifiable triggers, making seasonal allergic rhinitis less likely. The
timing of the start of her symptoms after the institution of the enalapril for control of the
essential hypertension strongly implicates it as the most likely culprit. Antihypertensives, such as
ACE inhibitors and angiotensin receptor blockers, can cause chronic rhinitis and cough as side
effects. Other causes of systemic drug-induced rhinitis in general include cardiovascular agents
such as α-1 receptor adrenergic antagonists, phosphodiesterase-5 selective inhibitors, gabapentin,
and nonsteroidal anti-inflammatory drugs (NSAIDs).

Item C79. Types of Rhinitis


• Allergic rhinitis
o Seasonal
o Perennial
o Episodic
• Nonallergic rhinitis
o Vasomotor rhinitis
 Irritant triggered leg, chlorine)
 Cold air
 Exercise (eg, running)
 Undetermined or poorly defined triggers
o Gustatory rhinitis
o Infectious
 Acute
 Chronic
o NARES (nonallergic rhinitis with eosinophilia)
• Occupational rhinitis
o Caused by protein and chemical allergens, immunoglobulin E-mediated
o Caused by chemical respiratory sensitizers, immune mechanism uncertain
o Work-aggravated rhinitis
• Other rhinitis syndromes
o Hormonally induced
 Pregnancy rhinitis
 Menstrual cycle related
o Drug-induced
 Rhinitis medicamentosa
 Oral contraceptives
 Antihypertensives and cardiovascular agents
 Aspirin or nonsteroidal anti-inflammatory drugs
o Other drugs
American academy of pediatrics 241
American Academy of Pediatrics PREP 2015

• Atrophic rhinitis
• Rhinitis associated with inflammatory-immunologic disorders
o Granulomatous infections
o Wegener granulomatosis
o Sarcoidosis
o Midline granuloma
o Churg-Strauss
o Relapsing polychondritis
o Amyloidosis
Reprinted with permission from Wallace DV, et al. The diagnosis and management of rhinitis An
updated practice parameter.J Allergy Clin immunology 2008;122:144

Aspirin and NSAIDs may produce rhinorrhea as an isolated symptom or as part of a symptom
complex of rhinosinusitis, nasal polyposis, and asthma known as "Samter's triad." The recent
onset of the child's symptoms, absence of nasal polyps, and minimal use of aspirin make this a
less likely etiology. In patients with nasal polyposis and aspirin associated respiratory disease
(AERD), aspirin desensitization followed by long-term daily aspirin treatment may be
considered. Longterm studies of patients with AERD suggest that maintenance aspirin therapy
may reduce nasal symptoms, frequency of sinus infections, requirement for nasal polypectomies,
and need for systemic corticosteroids.

Causes of hormonal rhinitis include pregnancy and menstrual cycle-related rhinitis. Pregnancy
rhinitis, when present, is associated with significant nasal congestion, starts after the second
month of pregnancy, and usually disappears within 2 weeks after delivery. During the menstrual
cycle, nasal congestion has been shown in I study to concur with ovulation and rise in serum
estrogens, although additional evidence is needed. There is no direct evidence that the currently
available combined oral contraceptive pills cause nasal symptoms.

Nonallergic rhinitis with eosinophilia (NARES) is characterized by nasal eosinophils in patients


who have perennial symptoms of sneezing paroxysms, profuse watery rhinorrhea, nasal pruritus,
and occasionally reduced sense of smell. Nonallergic rhinitis with eosinophilia is characterized
by large numbers (inconsistently defined as > 5% to > 20%) of eosinophils on nasal smear.
These patients often lack evidence of allergic disease as demonstrated by absence of positive
skin tests and specific immunoglobulin E antibodies in the serum. Patients with NARES are
typically middle-aged and have a characteristic perennial course, but with paroxysmal episodes.
Nonallergic rhinitis with eosinophilia occurs extremely infrequently in childhood and probably
accounts for less than 2% of children with nasal eosinophilia. Nonallergic rhinitis with
eosinophilia is therefore less likely in the patient in this vignette.

Rhinitis medicamentosa is a condition of rebound nasal congestion that follows the overuse of
intranasal a-adrenergic decongestants (such as oxymetazoline and phenylephrine) or recreational
use of cocaine. Benzalkonium chloride in vasoconstrictor spray products, when used for 30 days
or more, may augment local pathologic effects. Patients may develop rebound congestion,
tachyphylaxis, reduced mucociliary clearance because of loss of ciliated epithelial cells, and on
rare occasions, nasal septal perforation. Treatment of rhinitis medicamentosa consists of
discontinuing the use of topical decongestants and administering intranasal corticosteroids to
American academy of pediatrics 242
American Academy of Pediatrics PREP 2015

control symptoms. At times, a short course of oral corticosteroids may be needed to control the
patients' symptoms while the effects of the nasal decongestant spray dissipate. Once the rhinitis
medicamentosa is treated, the patient should be evaluated for underlying conditions such as
allergic rhinitis.

PREP Pearls
• Chronic rhinitis can be caused by both allergic and nonallergic causes.
• Use of antihypertensives, such as angiotensin converting enzyme inhibitors, α-1 receptor
adrenergic blockers, and phosphodiesterase inhibitors, can cause chronic rhinitis.
• Topical use of intranasal α-adrenergic decongestants (such as oxymetazoline and
phenylephrine) or recreational use of cocaine can cause rhinitis medicamentosa and
should be discouraged.

American Board of Pediatrics Content Specification(s)


• Formulate a differential diagnosis of chronic rhinitis

Suggested Reading
• Berges-Gimeno MP, Simon RA, Stevenson DD. Long-term treatment with aspirin
desensitization in asthmatic patients with aspirin-exacerbated respiratory disease. Allergy
Clin Immunol, 2003;111(1):180-186. 6:A710.1067/ mai.2003.7.
• Philpott CM, El-Alami M, Murty GE. The effect of the steroid sex hormones on the nasal
airway during the normal menstrual cycle. C1in Otolaryngol Allied Sci. 2004;29(2):138-
142. doi:10.1111/j.1365-2273.2004.00801.x.
• Wallace DV, Dykewicz MS. Bernstein DI, et al. The diagnosis and management of
rhinitis: an updated practice parameter. I Allergy Clin Im mum)" 2008;122(2 SuppI):S1-
584. doi:10.1016/j.jaci.2008.06.003.

American academy of pediatrics 243


American Academy of Pediatrics PREP 2015

Item 80
A 20-month-old boy is brought to the emergency department because of difficulty breathing. He
was in his state of usual health until 3 days ago, when he developed a fever (up to 39.4°C). Since
then, other symptoms have developed, including decreased oral intake, irritability, and vomiting.
He has had 1 wet diaper in the last 24 hours. His mother brought him in because of progressive
lethargy, cough, and difficulty breathing.
Vital signs show a temperature of 38.5°C, heart rate of 190 beats/min, respiratory rate of 50
breaths/min, blood pressure of 85/65 mm Hg, and SpO2 is 90% on room air. Physical
examination reveals a generally lethargic child in moderate respiratory distress. Conjunctivae
and oral mucous membranes are erythematous and dry. There is no cervical lymphadenopathy.
He is tachypneic with use of accessory muscles and grunting. There is wheezing and scattered
rales on auscultation. Cardiovascular examination reveals tachycardia, normal S1 and S2 with no
rubs, gallops, or murmurs. His extremities are pale and cool, with capillary refill time
approximately 4 seconds. Abdomen is soft with palpable spleen and liver edge 2 cm below the
costal margin. He receives a 20 mL/kg intravenous fluid bolus of 0.9% saline. Subsequently, his
respiratory distress worsens and his pallor increases. Repeat physical examination reveals
development of jugular venous distention, and a palpable liver edge of now 4 cm below the
costal margin.

Of the following, the MOST likely diagnosis is


A. bronchiolitis
B. cardiogenic shock
C. hypovolemic shock
D. pneumonia
E. status asthmaticus

American academy of pediatrics 244


American Academy of Pediatrics PREP 2015

Item 80 Preferred Response:B


The child in this vignette is in shock, as evidenced by lethargy, respiratory distress, tachycardia,
and decreased urine output, all of which indicate decreased organ perfusion. The type of shock is
likely cardiogenic, as demonstrated by tachycardia, poor skin perfusion, hepatosplenomegaly,
rates, jugular venous distention, and worsening condition upon fluid challenge.

Shock is a condition in which oxygen delivery is insufficient to meet the metabolic demands of
vital organs. It is a life-threatening state that must be distinguished from other less serious
conditions so that treatment can be initiated without delay. Oxygen delivery is the product of
cardiac output and oxygen content. It can also be represented by the following formula:

Oxygen delivery = Stroke volume x Heart rate x Oxygen content

As suggested by this equation, an early compensatory mechanism to increase oxygen delivery in


shock, especially in children, is tachycardia. Elevated circulating catecholamines serve to
increase stroke volume, heart rate, and blood pressure, as well as preferential diversion of cardiac
output away from skin, renal, and splanchnic vascular beds toward the heart and brain. The
resultant increased systemic vascular resistance leads to maintained, or even increased, blood
pressure, resulting in compensated shock. The clinician should not be reassured by a normal or
elevated blood pressure when considering shock. Despite representing a compensatory
mechanism, catecholamines and increased systemic vascular resistance can be especially
deleterious in cardiogenic shock because of increased afterload, diastolic dysfunction, increased
myocardial oxygen demands, and impaired coronary artery filling.

Cardiogenic shock further characterizes the insufficiency of oxygen delivery as a failure of the
heart as a pump. This can be caused by impaired contractility, arrhythmias, outflow tract
obstruction, or impaired filling as seen in diastolic dysfunction, restrictive cardiomyopathy,
lesions of reduced cavity size, or tamponade physiology. Impaired contractility is most
commonly seen in myocarditis, primary neuromuscular disorders, or metabolic derangements.
Single ventricle congenital heart disease with ductal-dependent systemic blood flow, such as
hypoplastic left heart syndrome, critical aortic stenosis, and interrupted aortic arch can cause
shock if right-to-left shunting across the ductus arteriosus is impaired. Ductal-dependent
pulmonary blood flow lesions, such as tricuspid atresia or pulmonary atresia can cause severe
cyanosis if left-to-right shunting across the ductus arteriosus is impaired.

American academy of pediatrics 245


American Academy of Pediatrics PREP 2015

Heart failure can masquerade in children as pneumonia, asthma, or bronchiolitis because clinical
signs also include wheezing, grunting, and difficulty breathing. In heart failure, increased left
atrial pressure transmits upstream to the pulmonary circulation, and the resulting increased
pulmonary capillary hydrostatic pressure in turn leads to pulmonary edema. Increased lung water
and decreased pulmonary compliance can cause tachypnea and grunting, and compression of
neighboring bronchioles can lead to so-called "cardiac wheezing!' It is therefore imperative to
recognize shock in this patient. This is especially true because treatment with bronchodilators
can worsen heart failure by causing tachycardia, diastolic hypotension, lactic acidosis, and
increased myocardial oxygen demand. In this patient, hypovolemic shock is less likely in light of
the rales, hepatomegaly, and worsened condition after receiving fluids.

Patients with cardiogenic shock may also be dehydrated because of general illness and increased
insensible losses from tachypnea. Therefore, very cautious use of fluid boluses can help such a
preload-dependent patient experiencing heart failure. However, boluses should be limited to 5

American academy of pediatrics 246


American Academy of Pediatrics PREP 2015

mL/kg at a time, followed by careful reassessment. If preload is adequate and cardiac output is
still impaired, a cardiology consultation should be sought, and inotropes should be started.

One of the most important initial steps in caring for the acutely ill child is determining whether
the patient is in shock. Poor perfusion, altered mental status, and tachycardia distinguish
hypovolemic shock from less serious conditions. Shock can further be classified as cardiogenic
shock if signs or symptoms of congestive heart failure are present, such as rales, hepatomegaly,
or jugular venous distention. Heart failure is one of many nonpulmonary causes of respiratory
distress, and one of the most serious.

PREP Pearls
• The clinician should not be reassured by a normal or elevated blood pressure when
considering shock.
• Cardiogenic shock may be distinguished from other causes of shock by the presence of
rales, hepatomegaly, and jugular venous distention.
• Respiratory distress and wheezing are common signs of cardiac disease in children.
• Cardiogenic shock can be worsened by aggressive fluid resuscitation.

American Board of Pediatrics Content Specification(s)


• Recognize that the blood pressure may not be adversely affected by shock

Suggested Reading
• McKiernan CA, Lieberman SA. Circulatory shock in children. Pediatr Rev.
2005;26(12):451-460. doi:10.1542/pir.26-12-451.
• Turner DA, Cheifetz IM. Shock. In: Kliegman RM. Stanton BF, St. Geme JW III, Schor
NF, Behrman RE, eds. Nelson's Textbook of Pediatrics. 19th ed. Philadelphia, PA:
Saunders Elsevier; 2011:305.
• Yager P, Noviski N. Shock. Pediatr Rev. 2010;31(8):311-319. doi:10.1542/ pir.31-8-311.

American academy of pediatrics 247


American Academy of Pediatrics PREP 2015

Item 81
A 4-year-old girl presents to your office for evaluation of a skin lesion over her right knee that
has a dark central area and 7 cm of surrounding redness. The redness has increased over the past
2 days. She had a temperature of 39.4°C on the day of evaluation. The parents thought she had
been bitten by a spider in the area just before the redness began. She is said to be a bit fussy and
complaining of pain at the site. She has no significant past illnesses and has no recent antibiotic
usage.
Her vital signs show a temperature of 39.2°C, a heart rate of 96 beats/min, a respiratory rate of
16 breaths/min, and a blood pressure of 98/66 mm Hg.
Physical examination shows a 2-cm, tender right inguinal lymph node. The lesion is shown in
Item Q81. She cries but is able to bend the knee. The remainder of the examination findings are
unremarkable. You take a culture by aspirating the lesion.

Of the following, pending culture results, the BEST initial antibiotic choice for this girl is
A. ampicillin
B. ceftriaxone
C. clindamycin
D. metronidazole
E. piperacillin-tazobactam

American academy of pediatrics 248


American Academy of Pediatrics PREP 2015

Item 81 Preferred Response: C


The pustular lesion with a suggestion of surrounding erythema seen in the girl in the vignette
suggests a skin and soft tissue infection caused by Staphylococcus aureus. The central necrotic
component has been especially associated with community-acquired methicillin-resistant S
aureus (CA-MRSA) infection. Early descriptions of such CA-MRSA infections indicated that
they appeared like a spider bite. Skin and soft tissue infections related to group A streptococcal
infection are associated with cellulitis, but typically without a pustular component.

Drainage of purulent cellulitis is recommended with culture performed on the material obtained.
In lesions less than 5 cm in a well-appearing child, drainage alone without antibiotic therapy is
likely sufficient. In the face of significant surrounding erythema and fever to 39.2°C, as in the
girl in this vignette, antibiotic therapy should be initiated while awaiting culture results. Of the
agents listed, clindamycin would be the most appropriate to cover S aureus, including CA-
MRSA. Trimethoprim/sulfamethoxazole and doxycycline would also provide appropriate
coverage for CA-MRSA in this type of infection, although doxycycline would not be routinely
recommended in a child younger than 7 years of age.

Ampicillin covers group A streptococcus, but not S aureus. Ceftriaxone may treat some
methicillin-sensitive S aureus (MSSA), but not methicillin-resistant S aureus (MRSA) infections.
Metronidazole is used for anaerobic bacterial infections. Piperacillin-tazobactam would cover
MSSA but not MRSA infections, and is a broader spectrum agent than needed for a skin and soft
tissue infection in a normal host. Item C81, shows the algorithm for initial management of skin
and soft tissue infections caused by community-acquired S aureus.

If the child was more ill with concern for invasive disease, empiric therapy with vancomycin,
possibly in combination with a semisynthetic penicillin such as nafcillin, would be indicated
until culture results are available.

American academy of pediatrics 249


American Academy of Pediatrics PREP 2015

American academy of pediatrics 250


American Academy of Pediatrics PREP 2015

PREP Pearls
• In normal hosts, skin and soft tissue infections with a pustular component are most often
associated with Staphylococcus aureus infections, while cellulitis alone is most often
caused by group A streptococcal infection.
• Drainage of pustular skin infections is recommended with fluid sent for culture and
sensitivity testing. Lesions smaller than 5 cm can be treated with drainage alone. In larger
lesions or with the presence of systemic signs or fever, antibiotic therapy, including
coverage for methicillin-resistant Staphylococcus aureus (MRSA) is advisable.
• Clindamycin, trimethoprim/sulfamethoxazole, or doxycycline provides adequate empiric
coverage for MRSA.
• In the severely ill child with skin infection, vancomycin, possibly in combination with
nafcillin, would be recommended while awaiting culture results.

American Board of Pediatrics Content Specification(s)


• Plan the appropriate management of cellulitis of the skin of various etiologies

Suggested Reading
• American Academy of Pediatrics. Staphylococcal infections. In: Pickering LK, Baker CI,
Kimberlin OW, Long SS eds. Red Book: 2012 Report of the Committee on Infectious
Diseases. 29th ed. Elk Grove Village, IL: American Academy of Pediatrics; 2012:653-
668.
• Jackson MA, Newland JG. Staphylococcal infections in the era of MRSA. Pediatr Rev.
2011;32(14:522-532. doi:10.1542/pir.32-I2-522.
• Liu C, Bayer A, Cosgrove SE, et al. Clinical practice guidelines by the Infectious
Diseases Society of America for the treatment of methicillinresistant Staphylococcus
aureus infections in adults and children. Clin Infect Dis. 2011;52(3):e18-e55.
doi:10.1093kidiciq146.

American academy of pediatrics 251


American Academy of Pediatrics PREP 2015

Item 82
A 10-day-old neonate presents to the emergency department with severe dehydration. His blood
test results are as follows:
• Sodium, 150 mEq/L (150 mmol/L)
• Potassium, 4.9 mEq/L (4.9 mmol/L)
• Chloride, 115 mEq/L (115 mmol/L)
• Blood urea nitrogen, 44 mg/dL (15.7 mmol/L)
• Creatinine, 0.7 mg/dL (61.9 µmol/L)
The patient was seen in the emergency department 7 days ago with similar concerns and serum
chemistry panels showing elevated serum sodium and creatinine concentrations. He was born at
full term to a 30-year-old gravida 1, para 1 mother via normal vaginal delivery. The neonate was
appropriate for gestational age with no prenatal or postnatal complications. You suspect an
inherited disorder because there is a history of similar episodes in males on the maternal side of
the family.

Of the following, the finding MOST likely to support your diagnosis is


A. glucosuria
B. hematuria
C. hydronephrosis
D. polyuria
E. vesicoureteral reflux

American academy of pediatrics 252


American Academy of Pediatrics PREP 2015

Item 82 Preferred Response: D


Recurrent episodes of hypernatremic dehydration are indicative of increased free water loss.
Such patients may have polyuria characterized by an increased total urine volume that results
from an underlying defect in water balance. This presents with the excretion of large volumes of
dilute urine, as seen in diabetes mellitus (osmotic diuresis), diabetes insipidus (antidiuretic
hormone disorders), and psychogenic polydipsia. Both nocturia and an increased frequency of
micturition may or may not be associated with polyuria.

Episodes of recurrent hypernatremic dehydration, positive family history, and the patient's age
are indicative of diabetes insipidus (DI), as seen in the patient in the vignette. Patients with DI
may have a positive family history of polyuria indicative of increased free water losses.

Disorders with positive family history for glucosuria, hematuria, renal failure, or vesicoureteral
reflux have a different clinical presentation. Glucosuria may be seen in neonatal diabetes;
however, serum chemistry in such patients reveals hyperglycemia with or without ketoacidosis
and ketonuria. Familial hematuria is seen in patients with Alport syndrome (X-linked
inheritance) and benign familial hematuria. Patients who have Alport syndrome present with
asymptomatic microscopic hematuria and a history of renal failure in the male members of the
family. Vesicoureteral reflux presents with recurrent urinary tract infections.

Diabetes insipidus occurs secondary to either decreased secretion of antidiuretic hormone


(ADH), known as central DI, or renal resistance to ADH effects, known as nephrogenic DI.
Patients with DI present with polyuria, polydipsia, and increased thirst. While they may present
with varying degrees of dehydration, laboratory evaluation is consistent with hypernatremia in
association with a dilute urine (urine osmolality < plasma osmolality).

Nephrogenic DI in children is secondary to a mutation in either the ADH-receptor (AVPR 2) or


the aquaporin 2 channels. AVPR 2 mutations have an X-linked inheritance and account for 90%
of cases; males are more severely affected than females. Neonates and infants with nephrogenic
DI present with irritability, failure to thrive, and a preference for water, along with clinical
findings of dehydration. Central DI is characterized by decreased ADH secretion. It can be
idiopathic (most common) or secondary to central nervous system tumors, infiltrative lesions
(histiocytosis), or trauma (surgical or nonsurgical). Water restriction, used to differentiate
nephrogenic DI from central DI, is not indicated in neonates and infants. This differentiation is
made through evaluation of the response to desmopressin (1 µg subcutaneously or intravenously
infused over 20 minutes; maximum dose, 0.4 µg/kg of body weight) by measuring urine
osmolality at baseline and at 30-minute intervals over 2 hours. If the urine osmolality remains at
less than 100 mosmol/kg over baseline, the diagnosis of nephrogenic DI is confirmed.

Psychogenic polydipsia presents with hyponatremia associated with a low urine osmolality that
is consistent with water overload. Maximal urine concentration is usually impaired (500 to 600
mosmol/kg) compared to that in normal patients (800 mosmol/kg or more). Water deprivation in
these patients will increase the urine osmolality (> 500 mosmol/kg), and there will be no
response to desmopressin since endogenous production and release of ADH is normal.

American academy of pediatrics 253


American Academy of Pediatrics PREP 2015

PREP Pearls
• Polyuria is characterized by an increased total urine volume that results from an
underlying defect in water balance.
• Polyuria can be seen in diabetes mellitus (osmotic diuresis), diabetes insipidus
(antidiuretic hormone disorders), and psychogenic polydipsia.
• Patients with diabetes insipidus present with polyuria, polydipsia, increased thirst,
varying degrees of dehydration, and laboratory evaluation consistent with hypernatremic
dehydration associated with dilute urine (urine osmolality < plasma osmolality).
• Neonates and infants with nephrogenic DI present with dehydration, irritability, failure to
thrive, and a preference for water.

American Board of Pediatrics Content Specification(s)


• Recognize the signs and symptoms of diabetes insipidus in patients of various ages
• Interpret the laboratory findings associated with diabetes insipidus

Suggested Reading
• Linshaw MA. Back to basics: congenital nephrogenic diabetes insipidus. Pediatr Rev.
2007;28(1):372-380. doi:10.1542/pir.28-10-372.
• Saborio P, Tipton GA, Chan KM. Diabetes insipidus. Pedlar. Rev. 2000;21(4):122-129.
doi:10.1542/pir.21-4-122.

American academy of pediatrics 254


American Academy of Pediatrics PREP 2015

Item 83
A previously healthy, 4-year-old boy presents to your office following 2 days of diffuse, crampy
abdominal pain, tactile fevers, and watery diarrhea. For the past 24 hours, the diarrhea has been
bloody. Symptoms began shortly after his family visited a local petting zoo. He recently
completed a 10-day course of amoxicillin for otitis media. Physical examination demonstrates a
well-developed, well-nourished child who is in moderate distress because of poorly localized
direct abdominal tenderness without rebound. His pulse rate is 140 beats/min, blood pressure is
96/54 mm Hg, and other vital signs are normal. His mucous membranes are dry and capillary
refill time is 2 seconds. Initial laboratory study results are as follows:
• Hemoglobin, 10.5 g/dL (105 g/L)
• White blood cells, 13,500/µL. (13.5 x 109/L)
• Platelet count, 70 x 103/ µL (70 x 109/L)
• Prothrombin time, 12 s (reference range, up to 12 s)
• Partial thromboplastin time, 23 s (reference range, up to 26 s)
• Urinalysis, 3+ positive for protein and blood, negative for glucose and ketones

Of the following, the MOST likely cause of this child's illness is


A. idiopathic ulcerative colitis
B. ileocolonic intussusception
C. infection with Campylobacter jejuni
D. infection with Clostridium difficile
E. infection with Escherichia coli O157:H7

American academy of pediatrics 255


American Academy of Pediatrics PREP 2015

Item 83 Preferred Response: E


Abdominal pain is one of the most common complaints in childhood and one that frequently
requires urgent evaluation in the office or emergency department. Although the cause is typically
a self-limited illness (constipation, gastroenteritis), the clinician must identify those conditions
that pose a potentially serious or life-threatening situation (most of which will require surgical
intervention) such as appendicitis, intestinal volvulus, intussusception, or obstruction secondary
to congenital (Ladd) bands or postsurgical adhesions. In this clinical setting, the presence of
associated signs, including bilious vomiting (small intestinal obstruction), localized pain, (eg, in
acute appendicitis), and colicky pain in an infant or toddler (intussusception, with or without
evidence of lower gastrointestinal bleeding) will often signal the need for imaging studies and
urgent surgical consultation. Item C83 lists the most common causes of abdominal pain
according to age at presentation. Many of these diagnoses are seen throughout the pediatric age
range. However, because children younger than 5 years of age are often incapable of localizing
the source of pain, reaching a definitive diagnosis is especially challenging in this age group. In
addition to problems requiring surgical or radiologic intervention, specific infectious disorders
must also be considered. The clinical history and available laboratory data described for the boy
in the vignette indicate anemia, thrombocytopenia, and urinary abnormalities in association with
the acute onset of abdominal pain and hematochezia. These findings strongly suggest a diagnosis
of hemolytic-uremic syndrome (HUS) as a consequence of infection with the Shigatoxin-
producing bacterium Escherichia coli O157:H7 (STEC). Microangiopathic hemolytic anemia,
renal dysfunction, and thrombocytopenia define the HUS triad. In the United States, HUS occurs
in approximately 8% of children who have E coli O157:H7 diarrhea, an incidence that is
significantly greater than observed with other Shigatoxin-producing E coli serotypes.

Item C83. Common Age-related Causes of Acute Abdominal Pain in Childhood


Neonate 2mo-2y 2-5y >5y
• Adhesions or Ladd • Adhesions or Ladd • Acute pancreatitis • Acute pancreatitis
bands bands • Adhesions or Ladd • Adhesions or Ladd
• Colic • Dietary protein bands bands
• Dietary protein allergy • Appendicitis • Appendicitis
allergy • Foreign body • Constipation • Cholelithiasis
• Necrotizing ingestion • Foreign body • Cholecystitis
enterocolitis • Gastroenteritis ingestion • Constipation
• Testicular torsion • Hemolytic-uremic • Gastroenteritis • Diabetic ketoacidosis
• Volvulus syndrome • Hemolytic-uremic • Gastroenteritis
• Incarcerated hernia syndrome • Helicobacter pylori
• Intussusception • Henoch-Schonlein gastritis
• Meckel diverticulum purpura • Henoch-Schonlein
• Sickle cell crisis • Intussusception purpura
• Toxin • Lead poisoning • Mesenteric adenitis
• Trauma • Mesenteric adenitis • Ovarian torsion
• Tumor • Pneumonia • Pelvic inflammatory
• Urinary tract infection • Sickle cell crisis disease
• Viral illness • Trauma • Pneumonia
• Urinary tract infection • Sickle cell crisis
• Viral illness • Testicular torsion
• Trauma

American academy of pediatrics 256


American Academy of Pediatrics PREP 2015

• Urinary tract infection


• Viral illness
• Volvulus

Pathologic E coli species are transmitted most commonly via contaminated food products.
Particular reservoirs of STEC include raw or undercooked ground beef, unpasteurized milk, and
any food or beverage (eg, unpasteurized apple cider) that may be contaminated with bovine
feces. Because all Shigatoxin-producing E coli are excreted in the feces of cattle, sheep, deer,
and other ruminants, direct contact with these animals is another significant source of
contamination. Several human outbreaks of E coli O157:117 enteritis have been linked to
animals in petting zoos.

Shigatoxin-producing Escherichia coli O157:H7 is an important cause of acute renal injury and
accounts for 70% to 90% of all HUS cases in countries where these bacteria are endemic. HUS
typically develops 5 to 9 days (mean, 7 days) after the onset of gastrointestinal symptoms and 2
to 6 days (mean, 4 days) after the appearance of bloody stools. Accordingly, when acute enteritis
is associated with the passage of blood, clinical assessment must include both a hematologic
profile (including examination of a blood smear for fragmented cells) and an evaluation of renal
function.

The previous treatment with amoxicillin for otitis media reported for the boy in the vignette
raises the question of a possible role for antibiotics in the development of HUS. Although earlier
studies suggested that antibiotic usage in hemorrhagic colitis increased the risk of developing
HUS, a metaanalysis failed to confirm these findings. Available evidence clearly indicates that
antimicrobial agents do not alter the duration or outcome of illness, and the use of antibiotics for
E coil O157:H7 infection is not recommended. Because most clinical microbiology laboratories
can culture and identify this organism, as well as the other common bacterial pathogens that
cause acute enterocolitis (Salmonella, Shigella dysenteriae, Campylobacter jejuni, Clostridium
difficile), decisions about specific therapy generally should await bacteriologic identification.

In the boy in this vignette, infection with any of the other organisms listed in the answer choices
is unlikely because of the strong association between E coil O157:H7 enterocolitis and renal
compromise associated with HUS. Intussusception would be less likely given the child's age
(unless a specific lead point was present, eg, polyp in a hamartomatous polyp syndrome, Meckel
diverticulum) and the positive urinary findings. Although idiopathic ulcerative colitis may occur
in this age group (more common in the second decade), the acute onset of illness coupled with
the findings of thrombocytopenia, and proteinuria are not consistent with this diagnosis. When
considering a diagnosis of ulcerative colitis, infectious causes of hematochezia must be ruled out.

PREP Pearls
• In the child with abdominal pain, bilious vomiting suggests small intestinal obstruction.
• In the evaluation of acute abdominal pain and hematochezia, hemolytic-uremic syndrome
must be ruled out.
• Antibiotic therapy is not indicated for the treatment of most gastrointestinal infections,
including infection with Escherichia coli O157:H7.

American academy of pediatrics 257


American Academy of Pediatrics PREP 2015

American Board of Pediatrics Content Specification(s)


• Plan the appropriate evaluation of acute abdominal pain
• Formulate an age-appropriate differential diagnosis of acute abdominal pain

Suggested Reading
• D'Agostino J. Common abdominal emergencies in children. Emerg Med Clin North Am.
2002;20(1):139-153. doi:10.1016/50733- 8627%2803%2900055-5.
• Goode B, O'Reilly C, Dunn J, et al. Outbreak of Escherichia coli 0] 57:H7 infections after
petting zoo visits, North Carolina State Fair, October-November 2004. Arch Pediatr
Adolesc Med. 2009;163(1):42-48. doi:10.1001/archpediatrics.2008.525.
• Grant HW, Parker MC, Wilson MS, et al. Adhesions after abdominal surgery in children.
I Pediatr Surg. 2008;43(1):152-156. doi:10.1016/j. jpedsurg.2007.09.038.
• Holtz LR, Neill MA, Tarr Pl. Acute bloody diarrhea: a medical emergency for patients of
all ages. Gastroenterology. 2009;136(6):1887-1898. doi:10.1053/j.gastro.2009.02.059.
• Loening-Baucke V, Swidsinski A. Constipation as cause of acute abdominal pain in
children. I Pediatr. 2007;151(6):666-669. doi:10.1016/j. jpeds.2007.05.006.
• Scholer SJ, Pituch K, Orr DP, Dittus RS. Clinical outcomes of children with acute
abdominal pain. Pediatrics. 1996;98(4 Pt 1):680-685.

American academy of pediatrics 258


American Academy of Pediatrics PREP 2015

Item 84
A 7-year-old boy presents to your office with concerns of activity-induced muscular cramping
and muscular weakness that have been progressively worsening and interfering with his ability to
compete in sports. His family history is notable for a maternal male first cousin with similar
symptoms who is currently 20 years old and wheelchair dependent.
On physical examination, you note muscle weakness that is most prominent proximally with
moderate calf hypertrophy. Neck flexor muscle strength is preserved. Neurologic examination
findings are otherwise normal. The boy's serum creatine phosphokinase concentration is 5 times
the normal range. Myoglobinuria is noted during a period of muscle cramping.
Genetic testing is performed, and his mother and older sister are confirmed to be carriers for this
disorder.

Of the following studies, the MOST appropriate study for this patient's sister is
A. echocardiogram
B. electromyography
C. nerve conduction velocity testing
D. renal ultrasonography
E. skeletal muscle biopsy

American academy of pediatrics 259


American Academy of Pediatrics PREP 2015

Item 84 TE Preferred Response: A


The boy in the vignette presents with progressive weakness and hypertrophy of the proximal
muscles beginning in the pelvis and lower extremities, then moving to the shoulders and neck,
and eventually involving the respiratory muscles. His age in the elementary school range
indicates that he most likely has classic Becker muscular dystrophy (BMD). Patients with BMD
typically have elevated creatine kinase concentrations (5 times that of normal), as also seen in
Duchenne muscular dystrophy (DMD). However, patients with DMD have creatine kinase
concentrations typically more than ten times those of normal. Patients with BMD present with
weakness of the quadriceps femoris initially, proceeding to more diffuse proximal muscular
weakness, activity-induced cramping, flexion contractures of the elbows, scoliosis, and
wheelchair dependency after age 16 years. Patients with DMD become wheelchair dependent
before age 13 years, with symptoms presenting between 3 and 5 years of age. These patients
often have delayed motor milestones. Patients with both BMD and DMD will develop a Gower
maneuver. This involves the patient's use of the arms to assist the weak pelvic girdle muscles
when rising to an upright position. The neck flexor muscle strength is preserved in BMD,
whereas in DMD the neck muscles are eventually affected. Dilated cardiomyopathy is the most
common cause of morbidity and mortality in BMD, usually presenting in the second decade of
life. The average age at death is in the mid-40s age range.

The dystrophinopathies are caused by DMD gene deletions, duplications, or mutations affecting
the dystrophin protein. In DMD, the protein function is absent. In BMD, the disease has a later
onset because the protein is partially functional. The dystrophinopathies are X-linked recessive
disorders with two-thirds of patients possessing a positive family history. One-third of cases are
de novo or spontaneous new mutations. These are X-linked disorders, therefore boys are
typically most severely affected and female heterozygous carriers have an increased risk for
dilated cardiomyopathy as they age. In this vignette, the patient's sister should be screened with
serial echocardiography over time. Muscle biopsy findings are abnormal in affected boys.

Treatment involves aggressive management of the dilated cardiomyopathy with medications and
sometimes heart transplantation. Corticosteroids improve motor function and strength in patients
with dystrophinopathies, thus enabling patients to prolong the ability to ambulate. Patients are
closely followed by cardiology beginning around age 10 years (at least biannually) for the onset
of the dilated cardiomyopathy. They also should be followed by neurology, orthopedics, and
pulmonology for the progressive muscle weakness and eventual respiratory compromise.
Patients should have regular pulmonary function tests and be evaluated for the need for chest
physiotherapy, cough-assist devices, and nocturnal positive-airway pressure devices to help
prevent heart failure. Orthopedic physicians follow the contractures and risk for scoliosis closely.

Patients with dystrophinopathies should receive the pneumococcal vaccine, the annual influenza
vaccination, and receive physical therapy to optimize their mobility and minimize contractures.
Nutritional status should be followed closely. Heterozygous female carriers should have cardiac
surveillance at least every 5 years, beginning in the second to third decade of life secondary to
the elevated risk of dilated cardiomyopathy. This is often overlooked by primary care providers
who do not realize that female carriers are at risk for cardiac involvement as well.

American academy of pediatrics 260


American Academy of Pediatrics PREP 2015

Another well-known disorder of muscular weakness in childhood is spinal muscular atrophy


(SMA). Spinal muscular atrophy is predominantly an autosomal recessive disorder caused by
degeneration and loss of the anterior horn cells in the brain stem and the spinal cord leading to
progressive muscle weakness and respiratory failure. Age at onset ranges from birth to young
adulthood with 5 different subtypes: SMA type 0 (prenatal onset with joint contractures), SMA
type I (onset < 6 months of age), SMA type II (onset 6-12 months of age), SMA type III (onset
after age 12 months), and SMA type IV (adult onset). Patients with types III and N can live
normal lifespans but with evident disability. Common symptoms, especially in SMA I, SMA II,
and SMA 0, include joint contractures, sleep problems, feeding difficulties, failure-to-thrive,
scoliosis, respiratory difficulties progressing to failure, and eventual wheelchair dependency.
One form of SMA is X-linked, whereas most forms are autosomal recessive.

For the sister of a patient with BMD, we would not expect abnormal findings on
electromyography or nerve conduction velocity testing because heterozygous females typically
only manifest the dilated cardiomyopathy as they age, unless significant skewed X-inactivation
(Lyonization), an Xp2 I.2 deletion, or an X-chromosome rearrangement is present, yielding a
more global dystrophic picture. A skeletal muscle biopsy would be invasive and not necessary.
Children affected with DMD or BMD and heterozygous female carriers do not manifest renal
dysfunction or anatomic abnormalities, so renal ultrasonography would be unwarranted.

PREP Pearls
• Dilated cardiomyopathy, which usually presents in the second decade of rife, is the most
common cause of morbidity and mortality In Becker muscular dystrophy (BMD) and
Duchenne muscular dystrophy (DMD). Heterozygous female carriers are at risk for
dilated cardiomyopathy as well.
• Duchenne muscular dystrophy and BMD are X-linked recessive disorders presenting with
progressive proximal muscular weakness and calf hypertrophy in males. Duchenne
muscular dystrophy presents in early childhood, typically between 3 and 5 years of age.
Becker muscular dystrophy typically presents in later childhood or adolescence. They
differ in age at presentation, rate of degeneration, and severity.
• Laboratory abnormalities in DMD and BMD include elevated creatine kinase levels: five
times normal in BMD and ten times normal in DMD.
• Spinal muscular atrophy is predominantly an autosomal recessive disorder caused by
degeneration and loss of the anterior horn cells in the brain stem and the spinal cord,
leading to progressive muscle weakness and eventually respiratory failure in the more
severe subtypes. The various subtypes vary in their age at presentation, severity, and
clinical progression.

American Board of Pediatrics Content Specification(s)


• Understand the inheritance pattern in a patient who has a neuromuscular disorder (eg,
muscular dystrophy, spinal muscular atrophy

Suggested Reading
 American Academy of Pediatrics, Section on Cardiology and Cardiac Surgery. Clinical
report: cardiovascular health supervision for individuals affected by Duchenne or Becker
muscular dystrophy. Pediatrics. 2005;116(6):1569-1573. doi:10.1542/peds.2005-2448.
American academy of pediatrics 261
American Academy of Pediatrics PREP 2015

 Bushby K, Finkel R, Birnkrant DJ, et al; DMD Care Considerations Working Group.
Diagnosis and management of Duchenne muscular dystrophy, part 2: implementation of
multidisciplinary care. Lancet Neurol. 2010;9(2):177-189. doi:10.1016/51474-
4422(09)70272-8.
 Darras BT, Miller DT, Urion DK. Dystrophinopathies. GeneReviews.
http://www.ncbi.nlm.nih.govibooks/NBK1119/.
 Kang PB, Kunkel LM. The muscular dystrophies. In: Striver CR, Beaudet AL, Sly WS,
Valle D, Vogelstein B, eds. The Online Metabolic and Molecular Bases of Inherited
Disease (OMMBID). 8th ed. New York, NY: McGraw-Hill; 2006: chapter 216.

American academy of pediatrics 262


American Academy of Pediatrics PREP 2015

Item 85
A 5-year-old boy presents to your office for his health supervision visit. Although he is clinically
well today, his mother reports that he was treated with amoxicillin 2 months ago for acute otitis
media. She is concerned because the school nurse sent a letter home stating that he recently
failed his hearing screen. His growth and development are normal. He is up to date on his
immunizations. He takes an antihistamine episodically for seasonal allergies. On physical
examination, you note that both tympanic membranes are cloudy, grayish white, and have
limited mobility on insufflation. His nasal mucosa is pale and boggy. A tympanometry
examination reveals a type B curve.

Of the following, the BEST initial plan of care is to


A. prescribe amoxicillin-clavulanate
B. prescribe a nasal decongestant
C. refer to an allergist for allergy testing
D. refer to otolaryngology for tympanostomy tubes
E. request preferential seating in the classroom

American academy of pediatrics 263


American Academy of Pediatrics PREP 2015

Item 85 Preferred Response: E


The boy in the vignette exhibits the characteristic clinical findings of persistent middle ear
effusion (MEE) following acute otitis media (AOM). Given his failed hearing screen and flat
(type B) tympanogram, he should have preferential seating in the classroom until hearing returns
to normal.

On physical examination, the normal tympanic membrane (TM) should be translucent gray with
important landmarks visible and the TM should be in its natural position and mobile. The
diagnosis of AOM or otitis media with effusion (OME) is established by documenting the
presence of fluid behind the TM. The presence of inflammation differentiates AOM from OME.
With OME, the TM may appear opaque or cloudy, an air-fluid level or air bubbles may be
evident on examination, and mobility on pneumatoscopy is limited.

Pneumatic otoscopy remains the preferred method to diagnose MEE. When pneumatic otoscopy
is difficult to perform or when a confirmatory test is desired, tympanometry or acoustic
reflectometry are diagnostic adjuncts that may be used. Tympanometry measures compliance of
the TM across a pressure range (Item C85). Normal tympanic membrane compliance leads to a
type A curve. A type B curve, as in this vignette, appears flat because of decreased or absent
mobility of the TM. Acute otitis media or OME are common causes of limited mobility of the
TM. Other causes include tympanosclerosis or TM perforation. The type C curve is shifted to the
left, indicating greater negative pressure in the middle ear space, and the curve is often somewhat
blunted because of diminished compliance. Type C curves are seen in patients with eustachian
tube dysfunction from various causes, including those with resolving or evolving AOM or OME.
Acoustic reflectometry is the reflection of sound off the TM. The normal TM absorbs sound, so
more sound is reflected when OME or AOM exists.

Item C85: Examples of


common tympanometry
curves. Type A curves
represent normal tympanic
membrane compliance.
Type B curves reflect poor
compliance of the
tympanic membrane (likely
due to a middle ear
effusion). Type C curves
are shifted to the left, or
negative, side of the graph,
indicating progressive
negative pressure in the
middle ear space.

American academy of pediatrics 264


American Academy of Pediatrics PREP 2015

Otitis media with effusion or persistent MEE after an episode of AOM is common and resolves
within 3 months in more than 75% of cases. The American Academy of Pediatrics policy on
OME recommends watchful waiting for 3 months after diagnosis. Recurrent AOM is defined as
3 or more distinctly documented episodes of AOM within 6 months or 4 or more episodes in a
12-month period.

The boy in the vignette shows no signs or symptoms of acute infection nor does he meet criteria
for recurrent OM, so treating with amoxicillin-clavulanate is not indicated. Most AOM treatment
failures manifest within 48 to 72 hours after initiation of therapy, and if this were the case, then
changing the antibiotic coverage would be appropriate. Medical therapies for persistent MEE,
such as the use of antihistamines, decongestants, or intranasal corticosteroids, have not proven
effective. Patients may experience conductive temporary hearing loss because of the effusion, so
providers should make parents aware and ongoing developmental surveillance is important.
When OME persists beyond 3 months, or if language delay is a concern, then hearing should be
evaluated. Children who are found to have normal hearing despite persistent MEE should be re-
evaluated every 3 months. Those who have hearing Loss, other symptoms attributable to the
effusion (otalgia or vestibular disturbance), or abnormal findings on physical examination
beyond simple effusion should be referred to otolaryngology. Referral for allergy testing in a
patient who has mild seasonal allergies would not be cost effective as the initial step in care.
Pediatric health care providers should advocate for preferential seating in the classroom for
patients with temporary or permanent hearing impairment that might affect their school
performance. This can be easily achieved immediately and will affect the child's learning by
helping him adapt to the environment while he experiences temporary conductive hearing loss
because of middle ear disease.

Risk factors for developing acute or chronic OME and recurrent AOM include the following: age
younger than 2 years, atopy, prolonged bottle use with feeding in the supine position, exposure to
smoke, cleft palate or craniofacial anomalies, Down syndrome, child care attendance,
compromised immune system, incomplete vaccination, ciliary dysfunction, and history of
allergies or chronic sinusitis. These risk factors should be addressed in the case of recurrent
AOM or chronic OME. The decision to consider tympanostomy tube placement should be
individualized and may be offered for recurrent AOM or if sequelae occur.

Sequelae of recurrent AOM or chronic OME include delayed language development, decreased
attention, and poor academic performance because of hearing loss-either conductive or
sensorineural; TM damage-atrophy, perforation, retraction, atelectasis, scarring; cholesteatoma;
and rarely, mastoiditis.

PREP Pearls
• Primary care providers should advocate for preferential sealing in the classroom to
optimize learning, until hearing returns to normal, because of the common occurrence of
decreased hearing in patients with middle ear effusion.
• Tympanic membrane mobility is diminished because of acute or chronic middle ear
disease, including infection, effusion, scarring, and perforation.
• Risk factors associated with the development of recurrent or chronic middle ear disease
should be addressed to reduce the prevalence and severity.
American academy of pediatrics 265
American Academy of Pediatrics PREP 2015

• Common sequelae of recurrent or chronic middle ear disease include hearing loss,
damage to the tympanic membrane, and rarely, mastoiditis.

American Board of Pediatrics Content Specification(s)


• Recognize the various etiologies of diminished tympanic membrane mobility and risk
factors associated with the development of recurrent or chronic middle ear disease
• Recognize the clinical findings and complications associated with middle ear disease
other than otitis media

Suggested Reading
• Daly KA, Hunter LL, Giebink GS. Chronic otitis media with effusion. Pediatr Rev.
1999;20(3):85-93. doi:10.1542/pir.20-3-85.
• Gould IM, Matz PS. Otitis media. Pediatr Rev. 2010;31(3)302-116. doi:10.1542/pir.31-3-
102.
• Kerschner IE. Otitis media. In: Kliegman RM, Stanton BF, St. Geme JW III, Schor NF,
Behrman RE, eds. Nelson Textbook of Pediatrics. 19th ed. Philadelphia, PA: Saunders
Elsevier; 2011:2209-2210.
• Klein JO, Pelton S. Acute otitis media in children: prevention of recurrence. UpToDate.
Available online only for subscription.
• Lieberthal AS, Carroll AE, Chonmaitree T, et al. The diagnosis and management of acute
otitis media. Pediatrics. 2013;131(3):e964-99. doi:10.1542/peds.2012-3488.
• Rosenfeld RM, Schwartz SR, Pynonen MA, et al. Clinical practice guideline:
tympanostomy tubes in children. Otolaryngol Head Neck Surg. 2013;149(1 Suppl):Sl-
S35. doi:10.1177/0194599813487302.

American academy of pediatrics 266


American Academy of Pediatrics PREP 2015

Item 86
A 3-year-old girl presents to your clinic for leg pain. The mother states that 3 weeks ago, her
daughter had a fever to touch that resolved spontaneously after a few days; however, the child
has been more tired since then. Over the past week, the child has been refusing to walk and
asking to be carried. In the office, her oral temperature is 38.3°C, pulse rate is 90 beats/min,
respiratory rate is 24 breaths/min, and blood pressure is 100/60 mm Hg. On examination, the
child is alert, playful, and in no apparent distress. The edge of her spleen is palpable 4 cm below
the left costal margin. She has anicteric sclerae and shotty lymphadenopathy. There is no
deformity, swelling, or tenderness of either leg. The remainder of the physical examination is
unremarkable. The following are the results of the child's complete blood cell count:

• White blood cell count, 3,500/µL (3.5 x 109/L), with 15% polymorphonuclear leukocytes,
75% lymphocytes, 8% monocytes, and 2% eosinophils
• Hemoglobin, 9.0 g/dL (90 g/L)
• Mean corpuscular volume, 85 µm3 (85 fL)
• Platelet count, 45,000 x 103/µL (45 x 109/L)
• Reticulocyte count, 0.5%
• Erythrocyte sedimentation rate, 25 mm/h (0 mm/h-20mm/h)

Of the following, the MOST likely diagnosis in this child is


A. acute lymphoblastic leukemia
B. aplastic anemia
C. juvenile idiopathic arthritis
D. osteomyelitis
E. osteosarcoma

American academy of pediatrics 267


American Academy of Pediatrics PREP 2015

Item 86 Preferred Response: A


The child in the vignette who presents with pancytopenia, bone pain, fever, and organomegaly is
most likely to have acute lymphocytic leukemia (ALL). The differential diagnoses for new-onset
pancytopenia are numerous, including malignancy, bone marrow failure, autoimmune disorders,
infection, or peripheral destruction of hematopoietic cells. When more than 1 blood cell type is
affected, an examination of the bone marrow by aspiration and biopsy may be warranted.

The most common malignancy in children is leukemia; ALL accounts for 80% of all childhood
leukemias. There are approximately 3,000 new cases of childhood ALL per year in the United
States. The peak age at diagnosis is between 2 and 5 years of age. The presentation can be
variable; from an incidental finding on routine laboratory testing in an asymptomatic child to a
life-threatening hemorrhage or respiratory distress (because of a mediastinal mass). The most
common presenting clinical features are fever, pallor, bruising, petechiae, bone pain,
hepatosplenomegaly, and lymphadenopathy. Anemia, thrombocytopenia, and an abnormal
leukocyte and differential count are common, but may not be present at the time of diagnosis
despite the leukemic infiltration of the bone marrow. Approximately 20% of cases of childhood
ALL present with leukocytosis with a white cell count greater than 50,000/µL (50 x 109/L In
patients with leukopenia, however, blasts may not be seen on routine peripheral blood smears.

Aplastic anemia is much less common in children, with peaks at 10 to 25 years of age and after
60 years of age. The diagnosis of acquired aplastic anemia (AA) requires the presence of 2 of the
following: hemoglobin less than 10 g/dL (100 g/L), platelet count less than 50 x 103/µL (50
x109/L), and absolute neutrophil count less than 1,500/ µL (1.5 x 109/L). Classification of the
severity of AA is based on bone marrow cellularity. Most cases of acquired AA are idiopathic.
The anemia is usually macrocytic with reticulocytopenia. A detailed history should be obtained
and a careful physical examination performed to distinguish between acquired AA and inherited
bone marrow failure syndromes, such as Fanconi anemia, dyskeratosis congenita, Shwachman-
Diamond syndrome, and congenital amegakaryocytic thrombocytopenia. In contrast to ALL,
hepatosplenomegaly and bone pain are not usually seen in patients with AA.

Pancytopenia and pain can be seen in juvenile idiopathic arthritis, but hepatosplenomegaly
would not be typical. One might also expect to see a higher erythrocyte sedimentation rate in this
type of inflammatory disease. Bone diseases such as osteomyelitis and osteosarcoma could
present with bone pain and refusal to walk, and possibly pancytopenia. Neither of these
conditions would explain the hepatosplenomegaly. Furthermore, the fever would most likely be
persistent in untreated osteomyelitis. Osteosarcoma is the most common pediatric bone cancer,
with 400 new cases per year in children in the United States. The mean age at diagnosis of
osteosarcoma is in adolescence during the second decade of life, which is thought to correlate
with the period of most rapid bone growth; therefore, osteosarcoma would be a less likely
diagnosis in a 3-year-old child than leukemia.

PREP Pearls
• The most common presenting clinical features of acute lymphocytic leukemia in children
are fever, pallor, bruising, petechiae, bone pain, hepatosplenomegaly, and
lymphadenopathy.

American academy of pediatrics 268


American Academy of Pediatrics PREP 2015

• Anemia, thrombocytopenia, and an abnormal white blood cell (WBC) and differential
count are common, but may not be present at the time of diagnosis despite the leukemic
infiltration of the bone marrow.
• Approximately 20% of cases of childhood acute lymphocytic leukemia (ALL) present
with leukocytosis with WBC counts greater than 50,000/µL (50 x 109/L); patients with
ALL who present with leukopenia may not have blast cells on routine peripheral blood
smears.
• The differential diagnoses for new-onset pancytopenia include malignancy, bone marrow
failure, autoimmune disorders, infection, or peripheral destruction of hematopoietic cells.

American Board of Pediatrics Content Specification(s)


• Plan the appropriate diagnostic evaluation of multiple pancytopenias
• Distinguish acquired aplastic anemia from childhood leukemia

Suggested Reading
• Inaba H, Greaves M. Mullighan CG. Acute lymphoblastic leukaemia.
• Lancet. 2013;381(9881)3943-1955. doi:10.1016/S0140- 6736(12)62187-4.
• Hunger SR Loh ML, Whitlock JA, et al; COG Acute Lymphoblastic Leukemia
Committee. Children's Oncology Group's 2013 blueprint for research: acute
lymphoblastic leukemia. Pediatr Blood Cancer. 2013;60(6):957-63.
doi:10.1002/pbc.24420.
• Samarasinghe S, Webb DK. How I manage aplastic anemia in children. Br J HaernatoL
2012;157(1):26-40. doi:10.1111/1.1365-214L2012.09058.x. Weinzierl EP, Arber DA.
The differential diagnosis and bone marrow evaluation of new-onset pancytopenia. Am J
Clin Pathol. 2013;139(1):9-29. doi:10.1309/AJCP5OAEEYGREWUZ.

American academy of pediatrics 269


American Academy of Pediatrics PREP 2015

Item 87
During a health supervision visit, the mother of a 5-yearold boy states that she has been having
difficulty with his behavior. She says he is much more oppositional than her daughter was at this
same age and that his behavior is getting worse. He has several tantrums each day when he does
not get his way, and these tantrums even occur at public places like a grocery store. He has a
generally intense mood, rather than being an "easy-going" childlike his sister. At night, his
parents let him fall asleep wherever he wants to and then carry him up to his bed, so as to avoid
having conflicts about enforcing a bedtime. His mother also states that he is an extremely picky
eater. For this reason, she always makes one of his preferred foods at each meal, despite what the
rest of the family is eating. The boy has no history of developmental difficulties.

Of the following, the MOST appropriate advice to offer would be to


A. coach mom to make a single meal at night for the whole family
B. coach mom to stop bringing him to public places like the grocery store
C. recommend a trial of a gluten-elimination diet for 2 weeks to 4 weeks
D. refer the boy for play therapy with a one-on-one counselor
E. refer the family to a behavior management specialist

American academy of pediatrics 270


American Academy of Pediatrics PREP 2015

Item 87 I-C Preferred Response: E


The origin of oppositional difficulties in this child is most likely a mismatch between parental
expectations (expecting that their child would be easygoing and compliant), and their child's
inherent temperament (strong willed, with irregular daily patterns and negative initial reactions
to novel stimuli like a new food). Challenging-to-parent temperamental traits have been found to
be lifelong characteristics of individuals. Research on behavior management techniques shows us
that functional outcomes for children with a challenging-to-parent temperament are highly
influenced by caregiver responses to their child. When strong-willed children are raised by
compassionate and adaptable parents, there is every reason to expect they will become highly
functional adults and can become future leaders in their environments.

When interacting with a parent who is dealing with this challenge, it is helpful to acknowledge
that some children are simply more difficult to parent than other more "easygoing" children.
Doing this avoids blaming parents for their child's difficulties, while still making it clear that the
solution will depend on them learning approaches to manage their child. A parent who feels
blamed for the child's misbehavior is less likely to follow through with a referral to a behavior
therapist, or to try implementing new parenting strategies that could be really helpful. After
empathizing with their experiences, a referral to a behavior management specialist would be the
most appropriate intervention in this case.

Research-proven approaches for managing oppositional behaviors depend on changing how


caregivers (ie, parents and teachers) interact with children, not on asking the child to
independently reflect on his or her actions and come to an internal decision to make different
behavioral choices in the future. When one-on-one counseling is the only care delivery model
available, one should refer the parents to a counselor to learn behavior management methods
themselves. Individual play therapy sessions with a counselor are unlikely to lead to behavior
change for this child.

It is also appropriate to coach the mother to make a single meal for the whole family at night,
which the child is free to choose to eat or not eat, and to have a consistent plan for what will
happen if the child refuses to eat (ie, no other options offered, or a single acceptable option, such
as a fruit, would be offered). This dinnertime advice is a single example of a behavior
management technique that a specialist might advise. However, correcting a mealtime problem
would address only one of the many difficulties this child and parent are having.
Although it is reasonable to offer the behavior management advice that the parent should avoid
bringing a tantrum-prone child on any unnecessary shopping trips, it would not be appropriate to
tell a parent to avoid bringing the child out in public at all.

A gluten-free diet might be advisable for a child with symptoms suggestive of a gastrointestinal
disorder like celiac disease; however, strong-willed behavior and tantrums are not
pathognomonic of celiac disease. Elimination diets are commonly advised on talk shows and on
the internet for all kinds of child behavior problems. Because a gluten-free diet is going to be
experienced by the child as aversive, and it is difficult to maintain healthy nutrition in a child on
a gluten-free diet, the principle of first do no harm means this intervention should be avoided
unless clinically necessary.

American academy of pediatrics 271


American Academy of Pediatrics PREP 2015

PREP Pearls
• Effective discipline involves setting limits that are delivered in a calm, consistent, and
caring fashion.
• The caregivers of the child with recurrent or worsening discipline problems should be
referred to a therapist who can provide behavior management advice.
• One-on-one counseling with a young child is unlikely to resolve discipline problems.

American Board of Pediatrics Content Specification(s)


• Advise parents regarding appropriate discipline and limit-setting for children of various
ages

Suggested Reading
• French WP, Kisicki MD. Management of disruptive behavior disorders. Pediatr Ann.
2011;40(11):563-568. doe 10.3928/00904481-20111007-07.
• Greene RW. The Explosive Child. New York, NY: Harper Collins Publishers; 2001.
• Turecki S. The Difficult Child. New York, NY: Bantam Books; 2000.

American academy of pediatrics 272


American Academy of Pediatrics PREP 2015

Item 88
A 4-month-old female infant is seen in the emergency department (ED) with respiratory distress.
You are in the midst of a respiratory syncytial virus epidemic. She has been ill for 2 weeks, with
a 50% decrease in her feedings and history of rapid breathing. The family was so worried about
her today that they called an ambulance. She was given an albuterol nebulizer treatment by
emergency medical services 1 hour ago, but they did not see any change in her respirations. She
does not take any other medicines; she has never been hospitalized or had surgery. Her family
moved from out of state when she was 1 month old, and they do not have a pediatrician.

On arrival in the ED, she is breathing rapidly at a rate of 70 breaths/min without any cough or
stridor. She is alert, but not vigorous, and has typical trisomy 21 facies. Her oxygen saturation is
88% on room air. Her heart rate is 180 beats/ min. Her blood pressure is 90/60 mm Hg. On
physical examination, her chest is clear with equal breath sounds. Her cardiac examination
shows a hyperdynamic precordium, a 2/6 holosystolic murmur at the left midsternal border, and
there is also a continuous murmur in the middle of the left scapula that radiates across the back.
Her liver is 4 cm below the right costal margin. Her femoral pulses are equal and bounding. Her
capillary refill time is 2 seconds.

Of the following, the BEST next step in management of this patient is to administer
A. ceftriaxone
B. continuous albuterol
C. furosemide
D. high-flow oxygen
E. methylprednisolone

American academy of pediatrics 273


American Academy of Pediatrics PREP 2015

Item 88 Preferred Response: C


The infant in this vignette most likely has congestive heart failure (CHF) and the best next step
in management is to administer a diuretic such as furosemide. The presentation of CHF varies
with age.

Newborns who present with CHF may be very ill, as they frequently will have a systemic
circulation that is ductal dependent. An example of this would be a newborn with hypoplastic left
heart syndrome. This patient needs a patent ductus arteriosus for systemic circulation. If the
pulmonary vascular resistance (PVR) drops for any reason (such as the patient receiving oxy-
gen), then increased flow will flood the infant's lungs and cause respiratory symptoms and
worsening CHF.

Infants at age 4 to 8 weeks may present with symptoms of CHF when the PVR drops naturally.
This may be difficult to distinguish from primary respiratory problems, especially during the
bronchiolitis season. Respiratory distress is the typical presentation of an infant with a
ventriculoseptal defect (VSD). If the VSD is small, a loud holosystolic murmur will be heard,
and if it is very large, only a murmur of increased flow through the pulmonary valve may be
appreciated. Infants may also present with poor weight gain and failure to thrive. Of patients
with trisomy 21, 40% to 50% will have congenital heart disease. Of those infants, 40% to 50%
will have an atrioventricular canal-type defect. Patients with this lesion have a distinctive
electrocardiogram, as well as cardiomegaly on chest radiograph. The physical examination in
this patient will also show increased work of breathing.

Children older than 1 year of age who present with heart failure are less likely to have
undiagnosed congenital heart disease if they have had access to regular medical care. In this age
group, it is more likely that the patient has acquired heart disease or cardiomyopathy.
Myocarditis, often viral, can have an acute presentation with a shock-like syndrome. If a gradual
decrease in function is seen, as with inheritable types of dilated cardiomyopathy, children will
adapt and may not complain of decreased exercise tolerance. Patients with known congenital
heart disease who have had previous surgical procedures and have developed ventricular
dysfunction may also present with decreased exercise tolerance at any age.

For the infant in the vignette, furosemide is the treatment of choice. It will alleviate the
pulmonary congestion caused by circulatory overload. The infant in the vignette has physical
examination findings consistent with an atrioventricular canal defect commonly seen with
trisomy 21. Her murmur also suggests a patent ductus arteriosus, which would contribute to her
symptoms of heart failure. High flow oxygen would make the left-to-right shunt worse in both
lesions. Continuous albuterol would increase the heart rate and myocardial oxygen demand,
ceftriaxone would not treat the heart failure symptoms, and methylprednisolone would be used to
treat reactive airway disease, which is unlikely in this situation. As this infant has CHF,
furosemide would be the first intervention.

PREP Pearls
• Furosemide is the treatment of choice for congestive heart failure (CHF) and will
alleviate the pulmonary edema.

American academy of pediatrics 274


American Academy of Pediatrics PREP 2015

• Congestive heart failure may be difficult to differentiate from respiratory problems in


infancy.
• Trisomy 21 is often associated with congenital heart disease (CHD) and may present with
CHF.
• Echocardiography is crucial in making the diagnosis of CHD or cardiomyopathy.

American Board of Pediatrics Content Specification(s)


• Recognize the clinical findings associated with congestive heart failure in children of
various ages
• Plan the appropriate initial diagnostic evaluation of congestive heart failure in children of
various ages

Suggested Reading
• Bernstein D. Heart failure. In: Kliegman RM, Behrman RE, Jenson HB, Stanton BF, eds.
Nelson Textbook of Pediatrics. 18th ed. Philadelphia, PA: WE Saunders Co: 2007:442.
• Rosenthal D, Chrisant MR, Edens E, et al. International Society for Heart and Lung
Transplantation: practice guidelines for management of heart failure in children. I Heart
Lung Transplant. 2004;23(12):1313-1333. doi:10.1016/j.healun.2004.03.018.

American academy of pediatrics 275


American Academy of Pediatrics PREP 2015

Item 89
A 3-year-old girl presents to your practice with knee swelling. She was referred to rheumatology
after the initial workup ruled out septic arthritis. The pediatric rheumatologist informs you that
the child has juvenile idiopathic arthritis. The patient is antinuclear antibody positive and
rheumatoid factor negative.

Of the following, the patient is at the MOST risk for


A. bone erosions
B. cataracts
C. iridocyclitis
D. leg length discrepancy
E. osteoporosis

American academy of pediatrics 276


American Academy of Pediatrics PREP 2015

Item 89 Preferred Response: C


Uveitis, also known as iridocyclitis, is a serious complication of juvenile idiopathic arthritis
(JIA) resulting from chronic nongranulomatous inflammation of the anterior eye chamber that
affects the iris and ciliary body of the eye. The uveitis associated with JIA is usually clinically
silent with an insidious onset. Risk factors for developing uveitis in JIA patients include JIA
subtype, age at onset of disease, and antinuclear antibody (ANA) status. The highest risk group is
female patients with pauciarticular JIA (≤4 joints in the first 6 months of diagnosis) that are
ANA positive and diagnosed before 4 years of age. Screening guidelines have been developed
for uveitis based on risk (Item C89). The severity of uveitis does not correlate well with arthritis
activity; therefore, the status of joint disease should not affect the frequency of screening.
Although ANA is often positive in patients with JIA who have uveitis, it is important to
remember that ANA is positive about 57% of the time in all patients with JIA.
Item C89. Uveitis Screening Guidelines for Patients with Juvenile
Idiopathic Arthritis
JIA Onset Type Antinuclear Disease Onset Disease Onset
Antibody Status <7 y of Age >7 y of Age
Pauciarticular or Positive Every 3-4 months Every 6 mo for
polyarticular for 4 y, then every 4 y,then yearly
6mo for 3y, then
yearly
Negative Every 6 mo for Yearly
4y then yearly
Systemic Either Yearly Yearly

No laboratory studies are diagnostic of JIA. Some laboratory studies can support the diagnosis of
JIA such as anemia, leukocytosis, elevated acute phase reactants, and
hypergammaglobulinemias. However, these laboratory values are not consistently abnormal.

Cataracts are a late complication of chronic uveitis or chronic ophthalmic steroid use and are not
associated with specific laboratory findings. A young patient recently diagnosed with JIA would
be unlikely to have this complication. A positive rheumatoid factor (RF) is uncommon in
children younger than 7 years, and is only positive in 10% to 15% of patients with JIA. Patients
with JIA who have bone erosions are more likely to be RF positive. Osteoporosis is a
complication seen with chronic steroid use in patients with difficult-to-control JIA. Osteoporosis
is not associated with any antibody study abnormalities. Vitamin D and calcium should be
supplemented in patients with chronic steroid use to help prevent osteoporosis. Leg length
discrepancy, resulting from increased inflammation in affected joints that increases blood flow
and growth factors, is more common in patients with JIA who have prolonged poor disease
control. Some of these patients may have increased acute phase reactants, such as erythrocyte
sedimentation rate, C-reactive protein, and thrombocytosis; however, there is no association of
leg length discrepancy with antibody studies.

American academy of pediatrics 277


American Academy of Pediatrics PREP 2015

PREP Pearls
• No laboratory studies are diagnostic of juvenile idiopathic arthritis (AA). Laboratory
results suggestive of JIA include anemia, leukocytosis, thrombocytosis, and elevated
inflammatory markers (erythrocyte sedimentation rate and C-reactive protein).
• Antinuclear antibody (ANA) titers are present in just over half of patients with JIA.
• A positive ANA test is associated with a higher risk of uveitis in young, female patients.
• Rheumatoid factor is positive in a small number of patients with JIA and is associated
with bone erosions.

American Board of Pediatrics Content Specification(s)


• Recognize the laboratory findings associated with juvenile rheumatoid (idiopathic)
arthritis and its complications

Suggested Reading
• Berard R. Approach to the child with joint inflammation. Pediatr Clin North Ain.
2012;59(2):245-262. dok10.1016/j.pcl.2012.03.003.
• Espinosa M, Gottlieb BS. Juvenile idiopathic arthritis. Pediatr Rev. 2012;33(7):303-313.
doi:10.1542/pir.33-7-303.

American academy of pediatrics 278


American Academy of Pediatrics PREP 2015

Item 90
A 10-year-old boy presents to your clinic for evaluation of right heel pain. He reports pain on
both sides of the heel that generally occurs after basketball practice. He denies swelling, redness,
bruising, or limping. On physical examination, the boy reports pain when you squeeze the right
heel; the remainder of the examination is unremarkable.

Of the following, the BEST next step is to


A. have the boy rest from sports for 1 month, then return to activity
B. have the boy use padded heel cups and allow return to activity as tolerated
C. have the boy use a walker boot for 4 to 6 weeks before returning to activity
D. refer to orthopedics for possible surgical management
E. refer to rheumatology for evaluation of possible juvenile idiopathic arthritis

American academy of pediatrics 279


American Academy of Pediatrics PREP 2015

Item 90 Preferred Response: B


The child in the vignette likely has Sever disease, an irritation of the calcaneal apophysis. The
use of padded heel cups in the shoes appears to help the symptoms of Sever disease by
decreasing impact to the heel. Ice, over-the-counter analgesics, and stretching exercises may also
help alleviate symptoms. Children with Sever disease tend to have a waxing and waning course
while the calcaneal apophysis is active, generally for 1 to 2 years. Sever disease does not appear
to lead to bone or joint damage, therefore these children should be allowed to participate in
physical activities provided they do not exhibit severe pain, limp, or alterations in gait or
physical activity mechanics that may increase susceptibility to other injuries.

Physes are growth areas of the long bones that contribute to bone lengthening. Apophyses are
growth centers that are present at locations where tendons attach to bone. The physes and
apophyses are cartilaginous, therefore these areas are particularly vulnerable to overuse injuries
as a result of traction and direct pressure. Osgood-Schlatter disease, an apophysitis involving the
tibial tubercle, and little league shoulder, a separation at the proximal humerus growth plate, are
other common examples of overuse injuries involving the growth centers that can occur in
skeletally immature children and teens. Adults, on the other hand, are more likely to experience
overuse injuries involving the tendons and ligaments, as these structures are the weakest points
of the adult skeleton.
The Achilles tendon attaches to the superior aspect of the calcaneal apophysis, therefore
contraction of the calf muscles applies tension there. In addition, direct force applied to the
plantar surface of the calcaneus occurs with high-impact activities and can irritate the apophysis.
Children with Sever disease typically report activity-related pain over the medial, lateral, or
posterior aspect of the heel. Affected individuals are generally between 7 and 14 years of age;
girls with Sever disease tend to be younger than boys with the condition. Some children exhibit
intermittent antalgic gait. A history of swelling, mechanical symptoms involving the ankle (eg,
catching or locking), or constitutional symptoms is not consistent with Sever disease and should
prompt investigation into other causes of heel pain.

The hallmark physical examination sign of Sever disease is pain when the examiner applies
compressive force to the medial and lateral sides of the heel. Flexible flatfoot and decreased calf
muscle flexibility are common associated findings. Radiographs may show sclerosis and
fragmentation of the calcaneal epiphysis; however, Sever disease is a clinical diagnosis and
radiologic studies are not needed in the absence of signs or symptoms suggesting another
diagnosis.

This patient has pain after activity and does not have any limp. He should be allowed to
participate in sports and recreation activities as tolerated. Immobilization may be indicated for
the rare patient with persistent pain, despite rest from activities and symptomatic treatment;
however, immobilization can lead to stiffness and weakness and should not be the first step in
management. Surgical management is not indicated for Sever disease. This child does not have
joint swelling, morning stiffness, pain in other joints, or constitutional symptoms to suggest a
rheumatologic cause of his pain, therefore he does not require referral to rheumatology.

American academy of pediatrics 280


American Academy of Pediatrics PREP 2015

PREP Pearls
• Sever disease (calcaneal apophysitis) is a common cause of heel pain in young athletes
between 7 and 14 years of age.
• Children with Sever disease can participate in physical activities as long as they are not
experiencing gait alterations or severe pain.
• The cartilage growth centers of the immature skeleton are particularly vulnerable to
overuse injuries.

American Board of Pediatrics Content Specification(s)


• Plan the appropriate management of an athlete with an overuse injury
• Identify the common overuse injuries in athletes

Suggested Reading
• Perhamre S. Lundin F, Klassbo M, Norlin R. A heel cup improves the function of the
heel pad in Sever's injury: effects on heel pad thickness. peak pressure and pain. Scand J
Med Sci Sports. 2012;22(0:516-522. doi:1O.11ll/j.l600-0838.2010.01266. x.
• Sarwark IF, LaBella C. Pediatric Orthopaedics and Sports Arjuries: A Quick Reference
Guide. Eik Grove Village. IL: American Academy of Pediatrics; 2010:650 pp.

American academy of pediatrics 281


American Academy of Pediatrics PREP 2015

Item 91
A 2-year-old girl presents with a 6-week history of slowly progressive right neck swelling with
redness. This started as a small "bump" under her right jaw with pink overlying skin. The area of
involvement slowly enlarged and the overlying skin became a deeper red. The parents report that
she was prescribed a 10-day course of oral clindamycin 3 weeks ago, but they have noticed no
improvement in the redness or swelling. She has not had fever or change in appetite or activity.
Her vital signs and growth parameters are normal for her age. The only remarkable finding on
physical examination is a 1- to 2-cm, firm, red, nontender area of swelling under the right jaw

(Item Q91).
Of the following, the MOST appropriate next step in the management of this patient's condition
is
A. a complete blood cell count with peripheral smear
B. a 10-day course of trimethoprim-sulfamethoxazole
C. excisional biopsy of the lesion
D. incision and drainage of the lesion
E. ultrasonography of the neck

American academy of pediatrics 282


American Academy of Pediatrics PREP 2015

Item 91 Preferred Response: C


The child described in the vignette has subacute or chronic cervical lymphadenitis, defined as
lymph node enlargement and inflammation that has persisted for several weeks. Cervical
lymphadenopathy is used to describe enlarged lymph nodes in the neck. However, the 2 terms
(lymphadenitis and lymphadenopathy) are often used interchangeably. In children younger than
5 years of age who have subacute or chronic, nontender, unilateral lymphadenitis with a
characteristic appearance of the overlying skin (violaceous and thin) and no constitutional
symptoms, the diagnosis of nontuberculous mycobacterial (NTM) infection is nearly certain and
excisional biopsy of the lymph node is appropriate. Incision and drainage of the lymph node is
contraindicated because of the risk of creating a chronically draining fistula. Although
ultrasonography of the neck can demonstrate lymphadenopathy, it is not a necessary step in
making the diagnosis. If radiologic imaging is desired before surgical intervention, computed
tomography would better reveal the anatomy of the neck and the extent of lymph node
involvement.

The differential diagnosis of subacute/chronic lymphadenitis in children includes infectious and


noninfectious causes. The first step in the evaluation of a child with chronic cervical
lymphadenopathy is to perform a careful history including the duration and location of lymph
node swelling, associated constitutional symptoms, ill contacts, animal exposures, trauma, dental
problems, immunization status, medications, travel history, and whether or not the lymph node
swelling is recurrent or persistent. A thorough physical examination is essential to assess the
number, size, shape, consistency, mobility, and tenderness of the lymph nodes. Evidence of
oropharyngeal abnormalities (periodontal disease, pharyngitis, aphthous ulcers, etc),
conjunctivitis, hepatosplenomegaly, bone or joint abnormalities (arthritis, bony tenderness or
palpable lesions), or skin rashes can provide clues to the diagnosis. A tuberculin skin test ,should
be performed, especially in children older than 5 years of age with risk factors for tuberculosis.
The tuberculin skin test in children with NTM cervical lymphadenitis can minimally react or
respond with up to 20 mm of induration, but these children are not ill-appearing. In children with
severe, rapidly progressive lymphadenopathy, or those who are ill-appearing with symptoms
lasting more than 6 to 8 weeks, laboratory and radiology investigations should include a blood
culture, complete blood cell count with peripheral smear, appropriate serologies (ie,
cytomegalovirus, Epstein-Barr virus, toxoplasmosis) and appropriate imaging to evaluate for
evidence of malignancy, collagen vascular disease, or persistent infection.

In children with subacute or chronic cervical lymphadenitis caused by NTM in whom complete
excision of the lymph node is not possible, or who are not candidates for surgical excision
because of the risk of facial nerve damage or other concerns, antimicrobial therapy with a
macrolide (clarithromycin or azithromycin) in combination with a rifamycin (rifampin or
rifabutin) or ethambutol for 12 weeks to up to 6 months is recommended. This therapy has been
shown to result in a cure in two-thirds of patients, but one-third developed chronic draining
fistulae and nearly 80% had antibiotic-associated adverse effects. Although trimethoprim -
sulfamethoxazole can be useful for the treatment of acute cervical lymphadenitis caused by
Staphylococcus aureus (less effective for group A Streptococcus), it is not a first-choice therapy
for the treatment of NTM cervical lymphadenitis.

American academy of pediatrics 283


American Academy of Pediatrics PREP 2015

PREP Pearls
• Excisional biopsy is the optimal therapy for the treatment of subacute or chronic cervical
lymphadenitis caused by nontuberculous mycobacteria (NTM).
• Incision and drainage of subacute or chronic cervical lymphadenitis caused by NTM risks
the development of a chronic draining fistula.
• Subacute or chronic lymphadenitis caused by NTM occurs most often in children
younger than 5 years of age and manifests as nontender, unilateral lymphadenitis with a
characteristic appearance of the overlying skin (violaceous and thin) and no constitutional
symptoms.
• In children with NTM subacute or chronic cervical lymphadenitis in whom surgical
excision of the node is not possible, the recommended treatment is antimicrobial therapy
with a macrolide (clarithromycin or azithromycin) in combination with a rifamycin
(rifampin or rifabutin) or ethambutol for 12 weeks to up to 6 months.

American Board of Pediatrics Content Specification(s)


• Identify the age-related etiology of chronic cervical lymphadenopathy
• Plan the appropriate management of chronic cervical lymphadenopathy

Suggested Reading
• American Academy of Pediatrics. Cat-scratch disease. In: Pickering LK, Baker CJ,
Kimberlin DW, Long SS, eds. Red Book: 2012 Report of the Committee on infectious
Diseases. 29th ed. Elk Grove Village, IL: American Academy of Pediatrics; 2012:269-
271.
• American Academy of Pediatrics. Diseases caused by nontuberculous mycobacteria. In:
Pickering LK, Baker CJ, Kimberlin DW, Long SS, eds. Red Book: 2012 Report of the
Committee on Infectious Diseases. 29th ed. Elk Grove Village, IL: American Academy
of Pediatrics; 2012:759-767. Lindeboom JA, Kuijper EJ, Bruijnesteijn van Coppenraet
ES, Lindeboom R, Prins JM. Surgical excision versus antibiotic treatment for
nontuberculous mycobacterial cervicofacial lymphadenitis in children: a multicenter,
randomized, controlled trial. Clin Infect pis. 2007;44(8):1057- 1064. doi:10.1086/512675.
• Swanson DS. Cervical lymphadenitis in children. Etiology and clinical manifestations.
UpToDate. Available online only for subscription.

American academy of pediatrics 284


American Academy of Pediatrics PREP 2015

Item 92
A 15-year-old, previously healthy adolescent boy presents to the emergency department for
evaluation of an injury to his nose that he sustained approximately 2 hours ago. He was playing
basketball when he was inadvertently struck directly over his nasal bridge by another player's
elbow. He experienced no loss of consciousness. Immediately following the injury, he had an
episode of bleeding from his nose that lasted approximately 5 min, but resolved with application
of direct pressure. His parents decided to bring him to the emergency department because of
severe swelling and pain across his nasal bridge.

On physical examination, the patient appears well. He is alert and fully oriented, and his vital
signs are normal for his age. There is marked swelling, ecchymosis, and tenderness to palpation
across his nasal bridge with flattening of the nasal dorsum. Inspection and speculum examination
of the intranasal cavity reveals no findings suggestive of a septal hematoma and no epistaxis or
rhinorrhea. There is no dental malocclusion or palatal instability on examination. The remainder
of physical examination findings, which include a complete neurologic evaluation, are
unremarkable. He is having no respiratory difficulty.

Computed tomography of the facial bones reveals a minimally displaced fracture of his nasal
bridge with no associated septal hematoma.

Of the following, the BEST next step in the management of this patient's condition is
A. emergent otolaryngology consultation in the emergency department
B. outpatient otolaryngology follow-up in 3 to 5 days
C. outpatient otolaryngology follow-up in 10 to 14 days
D. outpatient otolaryngology follow-up in 4 to 6 weeks
E. reassurance with no need for subspecialty follow-up

American academy of pediatrics 285


American Academy of Pediatrics PREP 2015

Item 92 TE Preferred Response:B


The adolescent boy in the vignette presents with a minimally displaced fracture of his nasal
bones, with no associated septal hematoma, persistent bleeding, signs of nasal obstruction, or
findings of associated orbital or ethmoid fractures. The best next step in the management of his
condition is reassessment by an otolaryngologist at an outpatient follow-up visit within 3 to 5
days.

Nasal bone fractures are the most common facial fractures that occur in children. Although
relatively uncommon in young children, the incidence of these fractures increases in the
adolescent and young adult years. Childhood nasal injuries typically result from falls, contact
sports, weight lifting, and automobile crashes (including those involving bicyclists or
pedestrians). Nasal bone fractures most commonly result from a direct blow to the face in the
anterior-posterior direction, though they can also occur with a lateral blow.

In children, the nose has a more prominent soft cartilaginous portion, which will bend easily,
allowing the injury-causing force to dissipate across the midface. This may result in significant
edema and ecchymosis of the nasal structures in children, which can make examination of the
facial bones and nasal structure challenging. Direct forces to the nose can lead to fractures of the
nasal skeleton and result in deviation or depression of the nasal bones and the nasal septum.
Also, greenstick-type fractures of the nose are more common in children. Although nasal bone
deformities may be detected during the initial clinical evaluation of affected patients, injury-
related swelling may prevent their detection in many cases.

Management of pediatric nasal bone fractures depends on a number of factors, including the age
of the affected child, degree of nasal obstruction, the presence of nasal septal hematomas (which
should be incised and drained emergently), and the presence of associated ocular, facial,
intracranial, and cervical spine injuries. An optimal cosmetic result is one of the primary long-
term concerns for nasal bone fractures.

Children with nasal bone fractures who do not have extreme degrees of deviation, associated
septal hematomas, airway obstruction, persistent epistaxis, or a concern for other significant
associated injuries should be referred for outpatient re-evaluation by an otolaryngologist within 3
to 5 days. This brief period between the initial evaluation and a follow-up examination allows
post-injury edema to resolve, which enables improved visualization of the nasal structures. Any
deviation of the nasal septum can be reduced by the otolaryngologist at this short-term follow-up
visit without a significant impact on the cosmetic outcome. Parents should be advised to bring a
recent photograph of their child to the otolaryngology follow-up visit 3 to 5 days after the initial
injury, so that the specialist may compare the appearance of the child's nasal contours before and
after the injury.

If more than 7 to 10 days elapse between the time of a child's nasal bone injury and reassessment
by an otolaryngologist, reduction of the nasal fractures may be much more difficult, and may
result in permanent displacement. This is because the active growth centers in children's nasal
bones promote rapid healing, and fracture fragments begin to form a strong fibrous union in their

American academy of pediatrics 286


American Academy of Pediatrics PREP 2015

deviated positions by this time. Therefore, follow-up in either 10 to 14 days or 4 to 6 weeks after
injury would be too late for easy mobilization of displaced fracture elements.

Emergent otolaryngology consultation in the emergency department is not necessary for the
adolescent boy in the vignette, given that the fracture of his nasal bones is minimally displaced,
and there is no associated septal hematoma, persistent epistaxis, concern for airway obstruction,
or clinical suspicion for serious associated injuries.
Finally, because the patient in the vignette has a displaced fracture of his nasal bridge (even
though the degree of displacement is minimal), which may require reduction by an
otolaryngologist, reassurance only would not be appropriate and could result in a cosmetic
deformity.

PREP Pearls
• Children with known or suspected nasal bone fractures who do not have extreme degrees
of deviation, associated septal hematomas, airway obstruction, persistent epistaxis. or a
concern for other significant associated injuries should be referred for outpatient re-
evaluation by an otolaryngologist within 3 to 5 days.
• Although nasal bone deformities may be detected during the initial clinical evaluation of
affected patients, injury-related swelling may prevent their detection in many cases.
• If greater than 7 to 10 days elapse between the time of a child's nasal bone injury and
reassessment by an otolaryngologist, reduction of fractures by the specialist may be much
more difficult, and may result in permanent displacement.

American Board of Pediatrics Content Specification(s)


• Understand the importance of early referral for surgical correction of displacement of the
nasal bones

Suggested Reading
 Kazahaya K. Otolaryngologic trauma. In: Fleisher GR, Ludwig S. eds. Textbook of
Pediatric Emergency Medicine. bth ed. Philadelphia, PA: Lippincott Williams & Wilkins;
2010:1298-1315.
 Mendez DR, Lapointe A. Nasal trauma and fractures in children. UpToDate. Available
online only for subscription.

American academy of pediatrics 287


American Academy of Pediatrics PREP 2015

Item 93
A 5-day-old term newborn is brought to the emergency department for the evaluation of jerking
motions of his arms and legs. The newborn was delivered vaginally weighing 3,500 g and
discharged to home 36 hours after delivery. He has been exclusively breastfed, with an
increasingly poor suck, irritability, and decreased activity noted by the mother over the past 24
hours. Upon admission to the emergency department, his weight is 3,150 g and his vital signs are
normal. A physical examination reveals an inconsolable, hypotonic newborn with marked
jaundice that involves his legs and the palms of his hands. You note a brief jerking of his right
arm, followed by bicycling movements of his upper and lower extremities.

Of the following, the MOST likely cause of the jerking is

A. acute bilirubin encephalopathy


B. benign neonatal myoclonus
C. herpes simplex encephalitis
D. hypocalcemia
E. hypoglycemia

American academy of pediatrics 288


American Academy of Pediatrics PREP 2015

Item 93 Preferred Response: A


The jerking motions and bicycling movements of the newborn in the vignette represent findings
associated with acute bilirubin encephalopathy (ABE). The newborn has marked jaundice of his
legs and his palms, which suggests his serum bilirubin level is greater than 15 mg/dl, (256.5
µmol/L). His clinical findings of lethargy, poor tone, irritability, and poor feeding are typically
seen with ABE.

The clinical signs of ABE in the neonatal period represent a spectrum. The earliest findings are
often subtle and nonspecific. Moderate ABE manifests with arching of the neck and trunk,
increasing lethargy, decreased feeding, and irritability with a shrill cry. Intervention at these
early phases may prevent the sequelae of kernicterus. Worsening signs suggestive of severe ABE
include bicycling movements of the arms and legs, inconsolable crying, inability to eat, fever,
seizures, and coma. Active intervention at this point is unlikely to affect the risk of the
development of kernicterus.

Identification of infants at risk of developing ABE and subsequent kernicterus sequelae is


essential. The American Academy of Pediatrics clinical practice guideline for the management of
hyperbilirubinemia in the newborn infant of 35 weeks or longer gestation provides a framework
that assists in the early identification of at-risk infants. These guidelines recommend that infants
showing signs of moderate to severe ABE be given an immediate exchange transfusion, even if
the total serum bilirubin is falling, acknowledging that the sequelae of kernicterus may be
prevented if severe ABE does not develop. If suspected in the outpatient setting, infants
exhibiting symptoms of ABE should be admitted directly to the hospital and not be referred to
the emergency department where treatment may be delayed.

The newborn in the vignette is manifesting the clinical findings of severe ABE and requires
emergent admission for evaluation and treatment. An evaluation for the underlying cause of his
hyperbilirubinemia should include a total and direct bilirubin, blood type, direct antiglobulin test
(DAT), complete blood cell count, and glucose-6-phosphate dehydrogenase deficiency testing. It
is appropriate to check blood glucose and serum calcium levels because of his history of poor
feeding, but these are unlikely to be the cause of his jerking motions and bicycling movements.
A sepsis evaluation may be performed because of his severe hyperbilirubinemia of unclear
origin. Although not likely to be the cause of his clinical findings, herpes simplex should always
be considered. The jerking movements of benign neonatal dorms are present only in sleep, thus
excluding this as the cause of his abnormal movements.

PREP Pearls
• The earliest findings of acute bilirubin encephalopathy (ABE) are often subtle and
nonspecific. Moderate ABE manifests with arching of the neck and trunk, increasing
lethargy, decreased feeding, and irritability with a shrill cry. Worsening signs suggestive
of severe ABE include bicycling movements of the arms and legs, inconsolable crying,
inability to eat, fever, seizures, and coma.
• Infants showing signs of moderate to severe ABE require an immediate exchange
transfusion, even if the total serum bilirubin is falling, acknowledging that the sequelae of
kernicterus may be prevented if severe ABE does not develop.

American academy of pediatrics 289


American Academy of Pediatrics PREP 2015

American Board of Pediatrics Content Specification(s)


• Recognize the clinical features and sequelae of acute bilirubin encephalopathy in
newborn infants, and manage appropriately

Suggested Reading
 American Academy of Pediatrics Subcommittee on Hyperbilirubinemia. Management of
hyperbilirubinemia in the newborn infant 35 or more weeks of gestation. Pediatrics.
2004;114(1):297-316. doi:10.1542/ peds.114.1.297.
 Johnson L, Bhutani VK, Karp K, Sivieri EM, Shapiro SM. Clinical review from the pilot
USA Kernicterus Registry (1992 to 2004).1 Perinatal. 2009; 9:525-S45.
doi:10.1038/jp.2008.211.
 Lauer BJ, Spector ND. Hyperbilirubinemia in the newborn. Pediatr Rev. 2011;32(8):341-
349. doi:10.1542/pir.32-8-341.
 Shapiro SM. Chronic bilirubin encephalopathy: diagnosis and outcome. Selnin Fetal
Neonatal Med. 2010;15(31:157-163. doi:10.1016/). siny.2009.12.004.
 Critique(s) 93-95

American academy of pediatrics 290


American Academy of Pediatrics PREP 2015

Item 94
You saw a 15-month-old girl for a health supervision visit last week. At that time, she had no
medical problems and was developing typically. After leaving your office, she developed a
temperature of 39°C and had 1 generalized tonic-clonic seizure about 2 minutes long. She was
evaluated in the emergency department, diagnosed with acute otitis media and discharged home.
Today, the girl and her mother return for follow-up. The mother expresses concern about the
chance of another febrile seizure. You tell her that her risk of recurrent febrile seizure is about
30%.

Of the following, the MOST correct statement is that this girl's risk of recurrent febrile seizure
would be higher if

A. a bacterial pathogen had been identified


B. her temperature had been 40°C or higher
C. her seizure had lasted 30 minutes or longer
D. she were 24 months old or older
E. there were a family history of febrile seizures

American academy of pediatrics 291


American Academy of Pediatrics PREP 2015

Item 94 Preferred Response: E


The girl in the vignette had a simple febrile seizure. Febrile seizures are generalized tonic clonic
seizures that occur in children, typically between 6 months and 6 years of age, associated with a
fever. They are divided into 2 types: simple and complex. Simple febrile seizures last less than
15 minutes and occur only once in 24 hours. Complex febrile seizures last longer than 15
minutes, have focal features, or present with more than 1 seizure in 24 hours. Complex febrile
seizures suggest an underlying disorder causing seizures, such as a brain lesion or an epileptic
syndrome, and may require further diagnostic evaluation. Simple febrile seizures often have no
underlying cause and resolve over time.

After a first simple febrile seizure, approximately 30% of children will have a recurrent febrile
seizure. The factors that increase the risk of recurrent simple febrile seizure are younger age at
the first febrile seizure, family history of febrile seizures, lower degree of fever at the time of
seizure, and shorter time interval between the onset of fever and the first seizure. The features of
complex febrile seizures, such as duration of the first seizure or increased numbers of seizures in
the first episode, do not increase recurrence risk. The finding of a bacterial pathogen does not
increase the risk of recurrent febrile seizures. For estimating recurrence risk, all the seizures that
occur in one 24-hour period count as just 1 seizure episode. Even though recurrence risk may be
low, children who have had a flurry of seizures or prolonged seizures may benefit from a
"rescue" benzodiazepine medication at home, in case they do recur.

PREP Pearls
• Simple febrile seizure recurrence risk is approximately 30%.
• Risk factors for recurrent febrile seizure include younger age at the first febrile seizure,
family history of febrile seizures, lower degree of fever at the time of seizure, and shorter
time interval between the onset of fever and the first seizure.

American Board of Pediatrics Content Specification(s)


• Understand the risk factors associated with febrile seizures
• Understand the difference between simple and complex febrile seizures

Suggested Reading
• Fishman M. Febrile seizures. UpToDate. Available online only for subscription.
• H irtz DG. Febrile seizures. Pecliatr Rev. 1997;18(1):5-9. doi:10.1542/pir.18-1-5.

American academy of pediatrics 292


American Academy of Pediatrics PREP 2015

Item 95
A 7-year-old girl presents to your office for concern for precocious puberty with progressive
breast development. She is overweight but otherwise healthy. Her mother had her first period at
age 15 years and is concerned that if her daughter is starting puberty now, she may be at risk for
early menstrual periods.

On physical examination, you are able to palpate what feels like a small amount of breast tissue
but also an appreciable amount of adipose tissue around her breast area. She has no pubic hair.
You are concerned for precocious puberty but also consider that she may simply be overweight,
and not have true breast tissue present. You need to determine if this patient needs further
workup for precocious puberty by an endocrinologist.

Of the following, the BEST test to help make this decision is


A. bone age
B. brain magnetic resonance imaging
C. estradiol level
D. 17-hydroxyprogesterone level
E. luteinizing hormone level

American academy of pediatrics 293


American Academy of Pediatrics PREP 2015

Item 95 Preferred Response: A


Precocious puberty is defined as the development of pubertal signs before 8 years of age in girls
and before 9 years of age in boys. In 1999, the Pediatric Endocrine Society recommended lower
age limits for girls: before 6 years of age for blacks and before 7 years of age for whites.
However, considerable controversy still exists concerning how best to define precocious puberty.
A significant risk associated with precocious puberty is loss of final adult height caused by early
fusion of epiphyseal growth plates from sex steroid exposure.

The best way to determine if the child in this vignette will have impaired final height is to make
a height prediction based on bone age. A bone age radiograph (radiograph of the left hand and
wrist) can be compared to that expected for the patient's current chronological age, and along
with the patient's current height, can be used to make a height prediction. Numerous bone age
atlases and standards exist to make this height prediction, which is usually done by a pediatric
endocrinologist. When the bone age is delayed in an otherwise normal child, often there will be
period of catch-up growth, and final height will be predicted to be higher than the current growth
percentile. Similarly, when bone age is advanced, there is potentially less time for catch-up
growth, and final height may be compromised.

The bone age radiograph (Item C95) shows a bone age of 10 years when the child was only 7
years old, significantly advanced from the child's chronological age. This led to a height
prediction that she would be much shorter than her midparental height and serves as the main
impetus to consider further evaluation. It is important to note that an advanced or delayed bone
age does not necessarily indicate an underlying pathological problem. For example, taller
children with constitutionally advanced growth and short children with constitutionally delayed
growth will have advanced and delayed bone ages, respectively, but will both ultimately grow to
normal heights. The bone age study can be used by the consulting endocrinologist to help decide
American academy of pediatrics 294
American Academy of Pediatrics PREP 2015

whether this patient simply has constitutionally advanced growth or may need additional
workup.

A brain magnetic resonance imaging scan is important to consider, especially if there are any
neurologic symptoms or if the family ultimately opts to treat the child for precocious puberty.
However, only 2% of girls with central precocious puberty starting between 6 and 8 years of age
have abnormal brain imaging studies. The incidence of relevant findings is much higher for girls
with central precocious puberty starting before 6 years of age. A better first test would be a bone
age study.

Laboratory results are also important in the evaluation of this child. A baseline luteinizing
hormone (LH) level of greater than 0.3 IU/L, a gonadotropin-releasing hormone-stimulated LH
level above 4 to 8 IU/L, and a random estradiol level in the pubertal range (> 20 pg/mL or > 73
pmol/L) may provide supportive evidence for progressive central precocious puberty. However,
this child has evidence of puberty on physical examination and concern that breast development
is progressing, so the question is not whether she has pubertal findings, but rather whether she
would benefit from treatment. This is where the bone age study is most helpful. The
compromised final height prediction should then prompt these additional laboratory tests, which
can be sent by the consulting endocrinologist.

A 17-hydroxyprogesterone level test is used to screen for 21-hydroxylase deficiency, a form of


congenital adrenal hyperplasia. In this condition, signs of adrenarche (androgen exposure) such
as pubic hair would be expected. As early breast development is a separate issue, it would not be
appropriate to include a 17-hydroxyprogesterone level in the evaluation of a patient for central
precocious puberty.

PREP Pearls
• Bone age can he used to predict final height, using one of many standard atlases and the
patient's current age and height.
• A compromised final height prediction from an advanced hone age should lead to
consideration of additional workup for many conditions.

American Board of Pediatrics Content Specification(s)


• Understand the relationship between bone age and chronologic age

Suggested Reading
• Chalumeau M, Chemaitilly W, Trivin C. Adan L, Brian G. Brauner R. Central precocious
puberty in girls: an evidence-based diagnosis tree to predict central nervous system
abnormalities. Pediatrics. 2002;109(1):6167. doi:10.1542/peds.109.1.61.
• Greulich W, Pyle S. Radiographic Atlas of Skeletal Development of the Hand and Wrist.
2nd Edition. Redwood City, CA: Stanford University Press; 1999.
• KapIowitz P. Treatment of central precocious puberty. Cu r r Op in Endocrinal Diabetes
Obes. 2009;16(1):31-36. doi:10.1097/ MED.0b013e328320a650.
• Kaplowitz PB, Oberfield SE. Reexamination of the age limit for defining when puberty is
precocious in girls in the United States: implications for evaluation and treatment; Drug

American academy of pediatrics 295


American Academy of Pediatrics PREP 2015

and Therapeutics and Executive Committees of the Lawson Wilkins Pediatric Endocrine
Society, Pediatrics. 1999;104(4 Pt 1):936-941.

American academy of pediatrics 296


American Academy of Pediatrics PREP 2015

Item 96
An otherwise healthy 14-year-old boy complains of occasional discomfort and heaviness in the
left side of his scrotum. He denies sexual activity and any history of trauma and has had no
dysuria or discharge. He is afebrile. On physical examination, he is at sexual maturity rating 4.
Upon inspection, there is no sign of inflammation and no discoloration.

Of the following, the BEST next step in evaluation of this patient is to

A. examine him in the upright position with and without Valsalva maneuver
B. order scrotal ultrasonography with Doppler flow studies
C. order a urinalysis and urine culture
D. order urine nucleic acid amplification testing for gonorrhea and chlamydia
E. refer him for urgent evaluation by a urologist

American academy of pediatrics 297


American Academy of Pediatrics PREP 2015

Item 96 Preferred Response: A


While most are asymptomatic, 10% of boys with a varicocele will present with heaviness or
discomfort in the scrotum as described by the boy in the vignette. A varicocele is an abnormal
dilation of the pampiniform plexus in the scrotum as a result of valvular incompetence of the
internal spermatic vein. It can decompress when the patient is in the supine position, therefore it
is important to examine the patient standing, especially noting whether the mass enlarges with
the Valsalva maneuver. A grade 1 varicocele is palpable only with Valsalva maneuver; a grade 2
is not visible but is palpable without Valsalva; and a grade 3 is found on visual inspection. The
paratesticular mass is classically described as a "bag of worms?' Typically, varicoceles occur on
the left side, with bilateral involvement occurring in only 2% of cases and unilateral right side
involvement even less frequent. In advanced cases the involved testicle may be smaller than the
contralateral side, although this is not usually apparent on physical examination. The higher the
varicocele grade, the greater the risk for poor outcome, notably decreased fertility.

The diagnosis of varicocele is made clinically, and additional studies are rarely needed for
confirmation. However, testing may be indicated to evaluate complications or to rule out other
diagnoses. Ultrasonography with Doppler flow is used most often to evaluate for testicular
torsion. The physical findings of that condition (acute pain, scrotal swelling, marked testicular
tenderness, absent cremasteric reflex) readily differentiate it from a varicocele. However, a
urologist may obtain nonemergent scrotal ultrasonography for a patient with varicocele to assess
testicular growth. A urinalysis and urine culture should be obtained if there are complaints of
dysuria or if there is evidence of epididymitis, but again the physical findings (acute scrotal pain,
erythema, and swelling) are distinct from those of varicocele. Nucleic acid amplification studies
for gonorrhea and chlamydia are appropriate for sexually active adolescents, particularly if they
have had multiple partners, but this infection is not related to the presence of a varicocele. A
nonurgent urology consult is indicated for symptomatic patients such as the boy in the vignette,
those with higher-grade varicoceles, or those with significant testicular size discrepancy.

The treatment goal for varicoceles is to preserve fertility, although how best to do this is
controversial. To date, it is unknown if aggressive surgical treatment during adolescence
preserves fertility. Among men with primary infertility, 40% are noted to have a varicocele, but
only 20% of men with varicoceles are infertile. Identifying teens with varicoceles who may be at
increased risk for infertility is important. One possible marker is the presence of testicular
hypotrophy on the involved side; however, a few studies show that even with a 20% difference
in testicular size, some adolescents will undergo "catch-up" growth of the smaller testicle.
Researchers are exploring the role of gonadotropin monitoring to predict outcome. Long-term
follow-up studies are needed to best define which adolescents will benefit from varicocelectomy
to preserve long-term fertility.

PREP Pearls
• Physical examination to detect varicoceles should be done with the patient standing, with
and without the Valsalva maneuver.
• Varicoceles typically involve the left side and are described as having a "bag of worms"
appearance.

American academy of pediatrics 298


American Academy of Pediatrics PREP 2015

• Referral to a urologist for a varicocele is indicated for patients who have a higher grade
varicocele, testicular size discrepancy, or are symptomatic.
• The goal of varicocele treatment is to preserve fertility.

American Board of Pediatrics Content Specification(s)


• Recognize the clinical findings associated with a varicocele. and manage appropriately

Suggested Reading
• Elder IS. Disorders and anomalies of the scrotal contents. In: Kliegman RM, Stanton
BMD, St. Geme JW III, Schor NF, Behrman RE, eds. Nelson Textbook of Pediatrics.
19th ed. Philadelphia, PA: Saunders Elsevier; 2011:1858-1864.
• Fine RG, Poppas DP. Varicocele: standard and alternative indications for repair. Curr
Opin Urol. 2012;22(6):513-516. doi:10.1097/ MOU.0b013e328358e1a4.
• Merriman LS, Kirsch Al. Varicocele in adolescence: where are we now? Curr tiro! Rep.
2012;13(4):311-317. doi:10.1007/s11934-012-0256-y.

American academy of pediatrics 299


American Academy of Pediatrics PREP 2015

Item 97
A 16-year-old girl is brought to your office by her mother for concerns about falling school
grades. The girl seems to have lost interest in a number of activities she previously enjoyed and
seems irritable. Her mother states that her daughter has been drunk on a few occasions in the last
few months and has developed gastrointestinal symptoms with abdominal pain, loose bowel
movements, and a weight loss of about 10 pounds. She has had difficulty falling asleep. She was
recently seen in an emergency department for back pain; she was advised to get further
evaluation for fibromyalgia. In looking through your records, you note that she was treated for
chlamydia 4 months ago, and today she admits that this was the result of a sexual assault.

Of the following, the MOST likely explanation for her symptoms is

A. binge drinking
B. irritable bowel syndrome
C. new-onset depression
D. post-traumatic stress disorder
E. somatoform disorder

American academy of pediatrics 300


American Academy of Pediatrics PREP 2015

Item 97 TE Preferred Response: D


Adolescents are at high risk for sexual violence. During the high school years, approximately 1
in 5 male and female adolescents may be victims of dating violence. Rape is a legal term,
therefore prevalence data are dependent on how information is collected and measured.
Unfortunately, sexual violence is an under-reported crime and the unsupported victims often go
on to have a range of early symptoms and future negative outcomes. These symptoms are often
referred to as the "rape-trauma-syndrome?' In the initial phase, victims experience fear, disbelief,
anxiety, guilt, and emotional lability, followed days to weeks later by a longer lasting phase of
reorganization and recovery. Up to 80% of victims will experience post-traumatic stress disorder,
as did the girl in the vignette. These victims are also at risk for substance abuse, eating disorders,
self-cutting, truancy, academic problems, risky sexual behavior, and revictimization. Victims
who acknowledge and report an assault, and seek professional services often have a better
outcome than those who do not. This makes it important for all adolescents to be screened for
sexual violence and to get them help early. Even more important is to have children participate in
prevention programs. Pediatricians should be knowledgeable about services available in the
communities in which they work and be available to provide initial psychological support.

The girl in the vignette is already indulging in binge drinking, displaying mood disturbances, and
may be developing fibromyalgia, all of which are seen with increased frequency in sexual assault
victims. Binge drinking, irritable bowel syndrome, and depression alone would not account for
the full range of symptoms she is experiencing. She does not meet criteria for the diagnosis of
somatization disorder, which includes the need for 2 pain symptoms, 2 gastrointestinal
symptoms, and 1 sexual and 1 pseudoneurologic symptom.

PREP Pearls
• Adolescents experience trauma, including sexual violence, at higher rates than either
children or adults.
• Screening all adolescents for a history of violence is important. Victims who
acknowledge and report having been assaulted, and seek services often do better.
• Victims of sexual violence may experience post-traumatic stress disorder and are at risk
for substance abuse, eating disorders, self-cutting, truancy, academic problems. Risky
sexual behavior and revictimization.

American Board of Pediatrics Content Specification(s)


• Recognize and address the emotional needs of an adolescent victim of sexual assault and
manage appropriately.

Suggested Reading
• Gillies D, Taylor F, Gray C, ❑'Brien L, D'Abrew N. Psychological therapies for the
treatment of post-traumatic stress disorder in children and adolescents. Cochrane
Database Syst Rev. 2012;12:CD006726. doi:10,1002/14651838.CD006726.pub2.
• Kaufman M, the Committee on Adolescence. Care of the adolescent sexual assault
victim. Pediatrics. 2008;122(2):462-470, doi:10.1542/peds,2008- 1531

American academy of pediatrics 301


American Academy of Pediatrics PREP 2015

• Nooner KB, Linares LO, Batinjane I, Kramer RA, Silva R. Cioitre M. Factors related to
posttraumatic stress disorder in adolescence. Trauma Violence Abuse. 2012;13(3):153-
166. doi:10.1177/1524833012447698.
• Walsh K, Danielson CK, McCauley 1, et al, Longitudinal trajectories of posttraumatic
stress disorder symptoms and binge drinking among adolescent girls: the role of sexual
victimization. I Adolesc Health. 2012;50M:54-59. doi;10.1016/j.jadohealth.2013.05,017.

American academy of pediatrics 302


American Academy of Pediatrics PREP 2015

Item 98
You are seeing a 10-year-old girl for follow-up of her asthma. The girl and her mother complete
the Childhood Asthma Control Test handed to them by the nurse. You notice a discrepancy in
the scoring between the child and the mother. As you go over the questions with them, they start
arguing. The child reports more symptoms than the mother is aware of. The mother feels the
child is prone to exaggeration. The physical examination is unremarkable.

Of the following, the BEST next step to assess control in this child is

A. the Asthma Quality of Life Questionnaire


B. the Asthma Therapy Assessment Questionnaire
C. peak flow measurement
D. Review of school nurse records
E. perform a spirometry

American academy of pediatrics 303


American Academy of Pediatrics PREP 2015

Item 98 I-C Preferred Response: E


The next step to assess asthma control in the child in this vignette is to perform a spirometry in
order to objectively assess lung function.

The 2007 National Heart, Lung, and Blood Institute guidelines (Expert Panel Report 3, "EPR3")
recommend periodic assessment of asthma control in all individuals with asthma. The parameters
of control assessment in the EPR3 include the use of validated tools such as the Asthma Control
Test (ACT) (> 12 years of age), Childhood Asthma Control Test (C-ACT) (4-11 years of age),
Asthma Therapy Assessment Questionnaires, and Asthma Control Questionnaires. These tools,
while validated screening measures, are based on self-report, with the ACT and C-ACT
measuring recall of asthma control over the past 4 weeks. A score of greater than 20 indicates
good control. A score of less than 19 indicates lack of control, and a score of less than 14
indicates very poor control. The aspects of asthma control assessed by the ACT and C-ACT
include nocturnal symptoms, ability to perform regular activities, and subjective assessment of
control. Part of the C-ACT is filled out by the parent and part by the child (using pictorial cues if
needed). The additional advantages of these measures are that they promote dialogue between
the caretaker and child, and between the provider, the caretaker, and child. Individual parameters
in these questionnaires can also be used to set goals and strategies to achieve asthma control. For
example, if the child is doing well in the daytime, but wakes up often because of asthma, then
emphasizing the importance of the pre-bedtime dose and trying to control comorbid conditions,
such as postnasal drainage or reflux, may help alleviate the limitations in achieving optimal
asthma control.

Other validated tools to measure asthma control such as the Asthma Therapy Assessment
Questionnaire (ATAQ) have been used both in research studies and in clinical care. These
instruments are easily interpretable and have evidence to support their use in clinical decision
making. The ATAQ can be self-administered by adults or used in interviewer-administered
versions for younger patients. The instrument consists of questions relating to the patient's past
month of asthma control and is scored on a scale of 0 to 4, with a higher score meaning more
control issues. While these instruments cover most of the guideline characteristics of control,
practical differences, such as in ease of administration, may determine preferential adoption in
clinical practice. Since the ATAQ measures domains of asthma control similar to those assessed
in C-ACT, administration of the ATAQ will not provide additional discriminatory information in
this case.

The Asthma Quality of Life Questionnaire (AQLQ) is a disease-specific, health-related quality


of life instrument developed by Elizabeth Juniper and colleagues that measures the functional
problems (physical, emotional, social and occupational) that are most troublesome to adults (17-
70 years) with asthma. There are 32 questions in the AQLQ and they are in 4 domains
(symptoms, activity limitation, emotional function, and environmental stimuli). A shorter version
for clinical use with 15 questions has been developed (called MiniAQLQ). The AQLQ does not
assess domains of asthma control and would not contribute to the clinical assessment of asthma
control.

Review of school nurse records is helpful to assess asthma control, or explore reasons for failure
to achieve control in the school setting. It may also help evaluate for associated conditions such
American academy of pediatrics 304
American Academy of Pediatrics PREP 2015

as vocal cord dysfunction, exercise-induced bronchospasm, or malingering. While this would


provide useful adjunct historical information, further determination of her current asthma control
at this clinic visit will require the use of an objective measure of lung function such as
spirometry.

Peak expiratory flow rate can be an objective method to evaluate airway obstruction. However,
the EPR3 states that "for diagnostic purposes, spirometry is generally recommended over
measurements by a peak flow meter in the clinician's office because there is wide variability
even in the published predicted peak expiratory flow reference values." Reference values may
also vary with the brand of meter used. Interpretation of peak flow values would depend on
back-ground knowledge of the child's personal best, which in turn would be dependent on the
child performing peak flows twice a day for a period of time. Peak flow meters therefore are
better utilized as home-based monitoring tools and not as diagnostic tools in the office.
The EPR3 recommends spirometry as one of the measures to assess asthma control (apart from
diagnosing asthma). The spirometry is objective and can be used in children 5 years of age and
older. The spirometric parameters that are helpful include the forced expiratory volume at 1
second (FEV1) and FEV1/FVC (forced vital capacity) ratio greater than 80%, indicating well-
controlled asthma (Item C98, page 85). The other parameters in conjunction with the spirometric
indices can help the provider gauge the degree of asthma control.

PREP Pearls
• Assessment of lung function indices using spirometry, on an annual basis after the age of
5 years, is recommended in the Expert Panel guidelines to obtain objective measures of
asthma control.
• Asthma control assessment tools, such as the Asthma Control Test, Childhood Asthma
Control Test, Asthma Control Questionnaires, or Asthma Therapy Assessment
Questionnaires, should be routinely used in the ambulatory management of children with
asthma.
• Peak flow measurements are helpful for home-based assessment of asthma control, but
are not helpful in diagnosing asthma in the office because reference values are not well-
established and are device-dependent.

American Board of Pediatrics Content Specification(s)


• Plan appropriate outpatient management of a patient with asthma (eg, self-assessment,
education, pulmonary function testing, drug therapy, asthma action plans)

Suggested Reading
• American Academy of Pediatrics. EQIPP. Asthma: Diagnosing and managing in
pediatrics. 2013. Expires 1/25/2016.
• National Heart. Lung, and Blood Institute. National asthma education and prevention
program expert panel report III: guidelines for the diagnosis and management of asthma.
National Institutes of Health, US Department of Health and Human Services website.
• Reddel HR, Taylor DR, Bateman ED, et al. An official American Thoracic
Society/European Respiratory Society Statement: asthma control and exacerbations:
standardizing endpoints for clinical asthma trials and clinical practice. Ant J Respir Crit
Care Med. 2009;180(1):59-99. doi:10.1164/rccm.200801-060ST.
American academy of pediatrics 305
American Academy of Pediatrics PREP 2015

Item 99
You are taking care of a 10-year-old girl who is in the intensive care unit because of refractory
status epilepticus. Her seizures have been difficult to control on traditional anti-epileptic
therapies, so she has been placed on a pentobarbital infusion for the past week. She is intubated
and on mechanical ventilation because of her coma. Over the past 48 hours, she has developed a
fever as high as 38.5°C, appears to be working harder to breathe, and requires increased
ventilator support to maintain her oxygen saturation and blood gases.
Vital signs show a temperature of 38.5°C, heart rate of 120 beats/min, respiratory rate of 20
breaths/min, and blood pressure of 100/60 mm Hg. Physical examination shows a generally
sedated and intubated patient. Pupils are 4 mm equal and sluggishly reactive. Gag reflex is not
present. She withdraws minimally to painful stimuli. Cardiovascular examination shows
tachycardia with regular rhythm and warm, well-perfused extremities. Lung examination reveals
tachypnea, bronchial breath sounds, and crackles in the right lung fields. The left lung is clear to
auscultation, with good air movement and no wheezing. Abdomen is soft, nontender, and
nondistended with no organomegaly. A chest radiograph is shown in Item Q99.

Of the following, the MOST likely cause of her worsened respiratory status is
A. acute respiratory distress syndrome
B. atelectasis
C. pulmonary edema
D. pulmonary embolism
E. ventilator associated pneumonia

American academy of pediatrics 306


American Academy of Pediatrics PREP 2015

Item 99 S Preferred Response: E


The child in this vignette received endotracheal intubation and was placed on mechanical
ventilation because of refractory status epilepticus. Over the course of a week, she developed
fever, tachypnea, and increased ventilator support, and chest radiography showed right middle
lobe consolidation, all of which suggest ventilator-associated pneumonia (VAP).

Respiratory failure is defined as inadequate function of the respiratory system to provide


oxygenation or carbon dioxide removal. Ventilatory assistance should be planned to support the
respiratory system to achieve both of those functions. In some conditions, this could be achieved
with noninvasive treatments such as supplemental oxygen, helium-oxygen mixture, high-flow
nasal cannula, or noninvasive positive pressure ventilation. If those measures are inadequate or
not feasible, endotracheal intubation and mechanical ventilation may be necessary. The child in
the vignette has decreased respiratory drive or apnea primarily from status epilepticus or
secondary to anti-epileptic medication. Patients given pentobarbital infusions universally require
intubation and mechanical ventilation. In the absence of a primary pulmonary cause of
respiratory failure, ventilator settings can include a respiratory rate normal for the child's age,
and a fraction of inspired oxygen (FiO2) of 0.21. If volume control ventilation is the chosen
mode, a tidal volume of 8 mL/ kg can be prescribed, and in pressure control ventilation, enough
pressure to achieve that tidal volume is adequate. Settings can be titrated according to arterial,
venous, or capillary blood gas analysis and pulse oximetry.

Ventilator-associated pneumonia can be diagnosed by new onset fever, leukocytosis, positive


respiratory culture, or new infiltrate on chest radiography. It can cause worsened gas exchange,
increased ventilator requirement, increased hospital and intensive care unit (ICU) stay, and even
death. Other potential complications of endotracheal intubation and mechanical ventilation
include ventilator-induced lung injury, neuromuscular weakness from sedatives required to
maintain tube placement, post-extubation upper air-way obstruction, subglottic stenosis, and
VAP. Measures to attenuate some of this risk include avoidance of high inflation pressures, high
FiO2, oversedation, and agitation. Weaning and early extubation should be attempted as soon as
the patient is able. Also, appropriate selection of endotracheal tube size is important, and should
follow the general rule that diameter (mm) = 4 + age/4. Protocols also exist to prevent VAP,
including elevation of the head of the bed to 30 degrees and measures for oral decontamination,
including chlorhexidine oral care. Initial treatment usually includes empiric broad-spectrum
antimicrobials to cover for gram-positive and gram-negative nosocomial infections, with
subsequent tailoring according to culture results.

Atelectasis can occur during mechanical ventilation because of generally impaired airway
clearance and mucous plugging. However, it is not usually associated with fever, and the chest
radiograph shown is more indicative of an infiltrative process rather than a discrete segment of
atelectasis. Similarly, pulmonary edema and pulmonary embolism do not cause fever. Acute
respiratory distress syndrome can occur as a consequence of pneumonia, shock, or systemic
inflammation, but 4 criteria must be satisfied, including acute onset, noncardiogenic pulmonary
edema, bilateral pulmonary infiltrates, and PaO2/FiO2 ratio of less than 200.

Ventilator-associated pneumonia is a potential complication of endotracheal intubation and


mechanical ventilation, and is usually marked by fever, impaired gas exchange, increased
American academy of pediatrics 307
American Academy of Pediatrics PREP 2015

requirement of ventilator settings, and new infiltrate on chest radiograph. Contemporary ICU
practices include protocols to prevent VAP. Daily assessment of extubation readiness to limit
ventilator days is the most effective strategy.

American academy of pediatrics 308


American Academy of Pediatrics PREP 2015

PREP Pearls
• Ventilator-associated pneumonia is a potential complication of intubation and mechanical
ventilation, and can be prevented by minimizing ventilator days.
• In respiratory failure without primary lung pathology, initial ventilator settings can be
chosen to mimic a normal breathing pattern and gas exchange.
• Post-extubation upper airway obstruction is a potential complication of endotracheal
intubation and mechanical ventilation, and can be prevented by appropriate sizing of
endotracheal tube and minimizing agitation when intubated.

American Board of Pediatrics Content Specification(s)


• Plan the appropriate ventilatory support for patients with various conditions
• Understand the potential complications associated with endotracheal intubation

Suggested Reading
• Carlo WA, Ambalavanan N. Conventional ventilation: traditional and new strategies.
Pediatr Rev. 1999;20(12):e117-e126. doi:10.1542/pir.20-12-e117.
• Nitu ME, Eigen H. Respiratory failure. Pediatr Rev. 2009;30(12):470-478.
doi:10.1542/pir.32-6-240.
• Sarnaik AP, Mastropietro CM. Mechanical ventilation. In: Kliegman RM, Stanton BF, St.
Geme JW III, Schor NF, Behrman RE, eds. Nelson's Textbook of Pediatrics. 19th ed.
Philadelphia, PA: Saunders Elsevier; 2011:329,

American academy of pediatrics 309


American Academy of Pediatrics PREP 2015

Item 100
A 16-year-old adolescent boy presents to the emergency department for evaluation of a painful
skin lesion on his left lower extremity that has been progressing over the last 3 days. Three days
ago, he scratched his left lower calf while climbing over a fence in the playground. The next day,
he noticed some redness surrounding the scratch site, but he did not seek medical attention. Over
the next 24 hours, he noted a marked increase in pain (9 out of 10) at the site. He also began to
feel warm, had a decrease in appetite, and complained of diffuse achiness and malaise. He has
been previously well and is not taking any medication.

His vital signs show a temperature of 39.2°C, a heart rate of 110 beats/min, a blood pressure of
120/76 mm Hg, and a respiratory rate of 22 breaths/min.

Physical examination in the emergency department shows an ill-appearing adolescent. The left
lower extremity has a 4 x 5 cm erythematous area surrounding a scratch mark with bluish
discoloration. The area is exquisitely sensitive to touch. There is no swelling or discharge at the
site. The remainder of the examination findings are unremarkable.

Of the following, the MOST likely cause of this lesion is infection with
A. Clostridium perfringens
B. group A Streptococcus
C. methicillin-resistant Staphylococcus aureus
D. mixed bacterial infection
E. Pseudomonas aeruginosa

American academy of pediatrics 310


American Academy of Pediatrics PREP 2015

Item 100 Preferred Response: B


The findings of severe pain, exquisite sensitivity, and bluish discoloration at the site of skin
injury plus the degree of systemic illness described for the boy in this vignette are highly
concerning for a diagnosis of necrotizing fasciitis caused by infection with group A β-hemolytic
Streptococcus (GABHS). This infection can occur in otherwise healthy individuals. In addition
to antimicrobial therapy, necrotizing fasciitis requires prompt surgical debridement to remove
necrotic tissue and decrease toxin production. Co-infection of Staphylococcus aureus with
GABHS may also occur in this setting. Prior to widespread varicella immunization, necrotizing
fasciitis with GABHS was reported as a complication of chicken pox.

Clostridium perfringens has been associated with gas gangrene following trauma. In this entity,
subcutaneous emphysema with skin blisters and serosanguinous discharge are seen in association
with the muscle necrosis at the site of injury. Necrotizing fasciitis in association with anaerobes
such as C perfringens, as part of a mixed infection of anaerobic and aerobic bacteria, is seen in
persons with underlying medical conditions such as diabetes, recent surgery, immune
compromise, or peripheral vascular disease. Methicillin resistant Staphylococcus aureus (MRSA)
is a leading cause of skin and soft tissue infection, but the lesions typically present with a
purulent component. Necrotizing fasciitis with MRSA alone has only been rarely reported.
Mixed bacterial infections leading to the degree of local and systemic illness described in this
vignette are not commonly described in normal hosts. Pseudomonas aeruginosa infection has
been associated with gangrenous lesions and severe disease in immunocompromised hosts or
after severe crush injury, neither of which applies in this case.

Initial antibiotic therapy in suspected necrotizing fasciitis typically includes clindamycin in


addition to penicillin. Clindamycin has been shown to suppress toxin production and modulate
cytokine release. Additionally, it would provide coverage against community-acquired MRSA
pending culture results.

PREP Pearls
• Necrotizing fasciitis is clinically characterized by an exquisitely painful, tender lesion
with bluish discoloration at the site of infection.
• Surgical debridement is a critical component of the management of necrotizing fasciitis.
• Group A Streptococcus is the leading cause of necrotizing fasciitis in the previously
normal host.

American Board of Pediatrics Content Specification(s)


• Recognize the clinical findings associated with necrotizing fasciitis

Suggested Reading
• Laupland KB, Davies HD, Low DE, Schwartz B, Green K, McGeer A. Invasive group A
streptococcal disease in children and association with varicella-zoster infection.
Pediatrics. 2000;105(5):e60. doi:10.1542/ peds.105.5.e60.
• Stevens DL, Baddour LM. Necrotizing soft tissue infections. UpToDate. Available online
only for subscription.

American academy of pediatrics 311


American Academy of Pediatrics PREP 2015

Item 101
A 15-year-old adolescent girl returns to your office for evaluation of proteinuria noted on two
health supervision visits a year apart. Today, her temperature is 37.9°C, heart rate is 76
beats/min, respiratory rate is 17 breaths/min, and blood pressure is 110/60 mm Hg. She has
normal growth parameters and normal physical examination findings. She is currently
menstruating.

Her urinalysis 1 year ago was only significant for 1+ proteinuria. Her urinalysis today
demonstrates a specific gravity of 1.035, a pH of 6.0, 3+ protein, 4+ blood, and no leukocyte
esterase or nitrites. Her urine microscopy shows greater than 100 red blood cells per high-power
field, less than 5 white blood cells per high-power field, and no crystals or bacteria.

Of the following, the MOST appropriate next step in evaluation of this patient is
A. first morning urinalysis, urine microalbumin, and urine creatinine the next day
B. first morning urinalysis, urine protein, and urine creatinine the next day
C. first morning urinalysis, urine protein, and urine creatinine when not menstruating
D. 24-hour urine collection for microalbumin when not menstruating
E. 24-hour urine collection for protein excretion when not menstruating

American academy of pediatrics 312


American Academy of Pediatrics PREP 2015

Item 101 Preferred Response: C


Asymptomatic adolescent female patients, such as the girl in the vignette, should not have a
urinalysis performed while menstruating. Urinalysis at the time of menstruation will usually be
positive for blood, protein, and, occasionally, leukocytes. Therefore, in such patients, urine
should be checked several days after menstruation is completed.

According to the Kidney Disease Outcome Quality Initiative guidelines from the National
Kidney Foundation, 24-hour urine collection is not routinely indicated to evaluate proteinuria in
children. Studies demonstrate that the urine protein-creatinine ratio in a first morning urine
sample is as sensitive as 24-hour urine collection for detecting pathologic proteinuria.
A urine dipstick will become positive for protein when the urine albumin secretion is more than
300 mg/day. Normal urine albumin secretion is less than 30 mg/day. Testing for high
albuminuria (formerly called microalbuminuria) detects urine albumin secretion in the range of
30 to 330 mg/day. High albuminuria is the earliest manifestation of diabetic nephropathy and is
also noted to be an early marker in cardiovascular disease-associated renal injury.
Microalbuminuria testing should be limited to early detection of renal injury secondary to
chronic conditions associated with renal failure.

A persistent dipstick-positive proteinuria, or a urine protein-creatinine ratio of greater than 0.2, is


considered abnormal. Therefore, a first morning urine specimen when the patient is not
menstruating should be tested. A urine protein-creatinine ratio of greater than 0.2 in a first
morning sample (necessary to rule out orthostatic proteinuria) is abnormal and indicative of renal
pathology that requires evaluation by a pediatric nephrologist. In the absence of risk factors for
chronic renal disease in this patient, there is no indication for testing for microalbuminuria.

PREP Pearls
• Asymptomatic adolescent female patients should not have a urinalysis performed while
menstruating.
• Twenty-four-hour urine collection is not routinely indicated for evaluating proteinuria.
• High albuminuria (microalbuminuria) testing should be limited to early detection of renal
injury secondary to chronic conditions.
• A urine protein-creatinine ratio of greater than 0.2 in a first morning sample Is abnormal
and requires evaluation by a pediatric nephrologist

American Board of Pediatrics Content Specification(s)


• Plan the appropriate clinical and laboratory evaluation of proteinuria

Suggested Reading
• Feld LG, Schoeneinan Mj, Kaskel FJ. Evaluation of a child with asymptomatic
proteinuria. Pecliair Rep 1984;5(0248-254. cloi:10.1542/ pir.5-8-248.
• Hogg Ft), Portman RI, Milliner D, Lemley KV, Eddy A, I ngelfinger J. Evaluation and
management of proteinuria and nephrotic syndrome in children: recommendations from a
pediatric nephrology panel established at the National Kidney Foundation conference on
proteinuria, albuminuria, risk, assessment, detection, and elimination (PARADE).
Pediatrics. 2000;105(6):1242-1249. doi:10.1542/peds.105.6.1242.

American academy of pediatrics 313


American Academy of Pediatrics PREP 2015

• National Kidney Foundation. Evaluation of laboratory measurements for clinical


assessment of kidney disease, guideline 5: assessment of proteinuria. NKF web site,
• Sebestyen IF, Alon US. The teenager with asymptomatic proteinuria: think orthostatic
first. au Pediatr. 2011;50(3):179-182. doii10.1177/0009922810380904.

American academy of pediatrics 314


American Academy of Pediatrics PREP 2015

Item 102
A 3½-week-old neonate is seen in your office for vomiting. He was born at term and his birth
weight was 3,250 g. He is breastfed. For the past 2 to 3 days, he has been exhibiting nonbilious
vomiting after each feeding. After vomiting, he seems eager to resume nursing. Over the past 24
hours, his mother has noted fewer wet diapers and less stool than usual. The baby has
experienced no fever, diarrhea, or upper respiratory tract symptoms. Physical examination
demonstrates an alert infant sucking vigorously on a pacifier. His weight is 3,550 g, temperature
is 37°C, pulse rate is 160 beats/ min, respiratory rate is 40 breaths/min, and blood pressure is
70/50 mm Hg. His skin turgor is normal. The remainder of the examination findings are
unremarkable except for slight abdominal distention. You obtain the following laboratory
studies:
• Sodium, 130 mEq/L (130 mmol/L)
• Potassium, 3.2 mEq/L (3.2 mmol/L)
• Chloride, 95 mEq/L (95 mmol/L)
• Bicarbonate, 30 mEq/L (30 mmol/L)
• Blood urea nitrogen, 15 mg/dL (5.4 mmol/L)
• Creatinine, 0.2 mg/dL (18 umol/L)

Of the following, the MOST appropriate initial treatment for this infant is
A. continued nursing, pending results of further diagnostic tests
B. intravenous 5% dextrose and 0.3% sodium chloride at 15 mL/h
C. intravenous 5% dextrose and 0.45% sodium chloride at 25 mL/h
D. intravenous 0.9% sodium chloride, 70 mL over 1 hour, followed by infusion with 5%
dextrose and 0.3% sodium chloride at 15 mL/h
E. oral rehydration with a glucose-electrolyte solution containing 70 mEq to 90 mEq (70-90
mmol/L) of sodium chloride per liter

American academy of pediatrics 315


American Academy of Pediatrics PREP 2015

Item 102 Preferred Response: C


The progressive nonbilious vomiting and hypochloremic metabolic alkalosis described for the
infant in the vignette suggest the presence of a gastric outlet obstruction. Infantile hypertrophic
pyloric stenosis (HPS) is the most likely cause of such clinical findings. The initial goals of
therapy are assessment of hydration status and institution of appropriate intravenous rehydration,
with gradual correction of any electrolyte disturbances. Only when hydration status has
normalized and electrolyte abnormalities have been corrected should surgical pyloromyotomy be
undertaken. Given this infant's birth weight of 3,250 g and an expected mean weight gain of 20
g/day, the current weight should be approximately 3,750 g. The infant's actual weight of 3,550 g
likely indicates mild dehydration (~5%). Based on the serum electrolyte data, fluid therapy
should be initiated with 5% dextrose and 0.45% sodium chloride at approximately 1.5 times the
calculated maintenance rate (~600 mL per 24 hours, based on the infant's hydrated weight) or 25
ml/h. Oral rehydration therapy, including breast milk and glucose electrolyte solutions, is not
recommended in this infant whose serum electrolyte values indicate metabolic derangements
consistent with HPS.

The infant in the vignette is estimated to be only 5% dehydrated, therefore vigorous fluid
management with a bolus infusion is not required. In most cases of HPS, appropriate initial
intravenous fluid therapy involves a sodium chloride concentration between 0.45% and 0.9%,
depending on the magnitude of the electrolyte deficits. Intravenous therapy with concentrations
of less than 0.45% sodium chloride should be avoided, unless the condition is diagnosed early in
the disease course and all electrolyte and acid-base values are normal. Potassium should be
added to the infusate as soon as urinary output is established to prevent progressive hypokalemia.
However, because the serum potassium concentration rises as the serum pH normalizes,
potassium levels must be carefully monitored. Bolus infusions with 10 to 20 mL/kg 0.9% sodium
chloride are reserved for infants who demonstrate clinically moderate-to-severe intravascular
fluid depletion (ie, >7% dehydration).

Hypertrophic pyloric stenosis is the most frequent cause of metabolic alkalosis in the first few
weeks after birth and the most common indication for abdominal surgery, with an estimated
incidence of 1 in 250 live births. The condition occurs more commonly in white males, but
greater risk of occurrence in first-born male children has not been proven. Familial clustering has
been reported. Other, less common causes of gastric outlet obstruction include duodenal stenosis,
gastric duplication, antral web, and annular pancreas.

Although hypochloremic, hypokalemic metabolic alkalosis represents the classic electrolyte and
acid-base imbalances of HPS, many infants have normal serum electrolyte values, therefore
increasing the importance of having a "high index of suspicion" for this diagnosis. The
magnitude of any acid-base and electrolyte disturbance is related directly to the duration of
vomiting. Persistent emesis results in a progressive loss of gastric fluid and hydrochloric acid.
Hypokalemia ensues as the kidneys retain hydrogen ions in favor of potassium. As extracellular
fluid volume and hydrogen ions continue to be lost because of continued postprandial emesis and
intravascular volume contracts, increasing the relative serum bicarbonate concentration leads to a
contraction alkalosis. Serum potassium values fall further as alkalosis stimulates potassium-
hydrogen exchange across cell membranes, with potassium driven intracellularly and hydrogen
ions driven extracellularly.
American academy of pediatrics 316
American Academy of Pediatrics PREP 2015

PREP Pearls
• Hypochloremic, hypokalemic metabolic alkalosis represents the classic electrolyte and
acid-base imbalances of hypertrophic pyloric stenosis (HPS).
• In the fluid management of HPS, infused sodium concentrations of less than 0.45%
should be avoided.
• Serum potassium levels will rise in patients with alkalosis as the blood pH returns to
normal.
• Bolus intravenous fluid infusions should not be utilized in patients presenting with
dehydration levels less than or equal to 5%.

American Board of Pediatrics Content Specification(s)


• Recognize the acid-base changes associated with pyloric stenosis, and manage
appropriately

Suggested Reading
• Dinkevich E, Ozuah PO. Pyloric stenosis. Pediatr Rev. 2000;21(7):249-250.
doi:10.1542/pir.21-7-249.
• Garcia VF, Randolph IG. Pyloric stenosis: diagnosis and management. Pediatr Rev.
1990;11(10):292-296. doi:10.1542/pir.11-10-292.
• Rice HE, Caty MG, Glick PL. Fluid therapy for the pediatric surgical patient. Pediatr Clin
North Am. 1998;45(4):719-727.
• Roberts KB. Fluid and electrolytes: parenteral fluid therapy. Pediatr Rev.
2001;22(10:380-387. doi:10.1542/pir.22-11-380.

American academy of pediatrics 317


American Academy of Pediatrics PREP 2015

Item 103
A 5-day-old, breastfed, male term neonate presents to your office with a temperature up to
38.9°C, poor feeding, lethargy, vomiting, and diarrhea that have progressively worsened over the
last 2 days. The pregnancy and delivery were unremarkable. The birth weight was 2,950 g. The
baby did well the first 2 days after birth and was discharged with the mother. Since discharge, his
feeding has become more difficult, with vomiting and fussiness. He becomes sleepier after
feeding and develops watery diarrhea.

His current temperature is 38.6°C, pulse is 140 beats/min, respiratory rate is 40 breaths/min, and
blood pressure is 88/54 mm Hg. His weight is 2,700 g. On physical examination, you note an ill-
appearing infant with decreased tone, Jaundice, scleral icterus, distended abdomen with
hepatomegaly, bruising, and mild respiratory distress.

His red reflex is abnormal and consistent with cataracts. The child is admitted to the hospital for
evaluation for possible sepsis, and intravenous antibiotics are begun. The following are the
results of the neonate's laboratory tests:

• White blood cell count, 18,000/µL. (18 x 109/L)


• Aspartate aminotransferase, 138 U/L
• Alanine aminotransferase, 76 U/L
• Total bilirubin, 12 mg/dL (205.3 µmol/L)
• Direct bilirubin, 2 mg/dL (34.2 µmol/L)
• Prothrombin time, 36.9 s
• International normalized ratio, 4.8
• Partial thromboplastin time, 50 s
• Urine reducing substances, 3 +
• Blood culture, positive for Escherichia coil
• Urine and cerebrospinal fluid cultures, negative
• TORCH titers, negative

Of the following, the MOST likely diagnosis is


A. galactosemia
B. hereditary fructose intolerance
C. hereditary hemochromatosis
D. tyrosinemia type 1
E. urea cycle defect

American academy of pediatrics 318


American Academy of Pediatrics PREP 2015

Item 103 Preferred Response: A


The male neonate in the vignette has classic galactosemia, which is a disorder of galactose
metabolism that results in life-threatening issues such as lethargy, poor growth, vomiting with
poor feeding, hepatocellular damage with elevated liver function tests, and clinical jaundice,
bleeding diathesis, and sepsis in undiagnosed infants. Urine-reducing substances will be positive
in addition to the coagulation and liver function abnormalities.

The neonate will be well initially then quickly decompensate after just a few days of age. The
neonatal symptoms will resolve immediately if a lactose- or galactose-restricted diet is given in
the first 10 days after birth to avert cataracts, imminent liver failure, sepsis, intellectual disability,
and potential newborn death. Most patients, even with early treatment, will still be at risk for
some developmental delays, including verbal dyspraxia and motor delays. Some patients will
have fine motor tremors and abnormalities in coordination and gait. Girls are at high risk for
premature ovarian failure. In cases of delayed treatment, the patient will commonly have a long-
term impact of intellectual disability and poor growth.

Most neonates with galactosemia are diagnosed through newborn screening via a heel prick,
which is mandatory in all state newborn screening panels, because it is an easily treated disease
that prevents serious sequelae if diagnosed and treated early. Following a positive newborn
screen, a measurement of erythrocyte galactose- 1-phosphate uridyltransferase (GALT) enzyme
activity, erythrocyte galactose1-phosphate (gal- 1-P) concentration, and GALT molecular
sequencing can establish the diagnosis.

Once the newborn screening finds abnormalities pointing to this disorder, immediate changes
should be made to the neonate's diet until the diagnosis is confirmed to avoid potentially serious
consequences. A neonate's formula should be switched to a soy-based or elemental formula
containing sucrose, fructose, and nongalactose carbohydrates. All milk and formula products
containing galactose or casein hydrolysates should be discontinued. Medications containing
lactulose also should be avoided.

Even after dietary avoidance of galactose, patients should continue to have periodic
ophthalmologic examinations for cataracts and screening assessments for developmental delays.
If any verbal dyspraxia or motor delays are noted, appropriate interventions and therapies should
be put into place.

Hereditary hemochromatosis is unlikely in the neonate in the vignette because it would not
present in early infancy with such rapid decompensation. Hemochromatosis is a disorder caused
by the abnormal storage of iron in joints, liver, heart, pancreas, and lungs. Most people will
develop symptoms between the ages of 40 and 60 years.

Neonates with hereditary fructose intolerance develop symptoms in infancy after ingesting fruits,
juices, or other foods containing fructose, not typically in the neonatal period, but after infant
foods and juices begin to be introduced into their diet. Symptoms typically include nausea,
diarrhea, vomiting, hypoglycemia, elevated uric acid, and abdominal pain associated with failure
to thrive. Continued ingestion can result in kidney and liver damage. Treatment is strict
avoidance of fructose ingestion.
American academy of pediatrics 319
American Academy of Pediatrics PREP 2015

Tyrosinemia type 1 presents in infancy with significant liver involvement and eventually renal
tubular dysfunction, growth failure, and rickets. Untreated children may present with repeated
neurologic crises involving a change in mental status, peripheral neuropathy, abdominal pain,
and occasionally respiratory failure. If untreated, many die before the age of 10 years.
Laboratory abnormalities include increased succinylacetone concentration in the blood and urine;
elevated tyrosine, methionine, and phenylalanine on serum amino acids; and elevated tyrosine
metabolites on urine organic acids.

Urea cycle defects present with a progressive obtundation, severe hyperammonemia, and
respiratory alkalosis in the first week after birth, which progresses to death very quickly with
irreversible brain damage if not immediately recognized and treated.

PREP Pearls
• Immediate recognition of galactosemia is important because of the life-threatening
consequences that will rapidly unfold if the galactose-containing diet is not discontinued
and changed to a soy-based or elemental formula.
• Galactosemia is mandatory on newborn screening. Treatment should not be delayed
while a definitive diagnosis is in process.
• Classic laboratory abnormalities of galactosemia include positive-urine reducing
substances, abnormal liver function studies, coagulation abnormalities suggestive of a
progressive bleeding diathesis, and a positive blood culture. especially due to Escherichia
• Physical findings of galactosemia in the neonatal period could include jaundice,
hypotonia, scleral icterus, bruising, bleeding, and cataracts in the face of rapidly
progressive liver failure.

American Board of Pediatrics Content Specification(s)


• Recognize the clinical features associated with galactosemia
• Recognize the laboratory features associated with galactosemia
• Plan the appropriate immediate and long term management of galactosemia, while
considering the long-term prognosis

Suggested Reading
• Elsas LI II. Galactosemia. GeneReviews. http://www.ncbi.nlm.nih.gov/
books/NBK1518/.
• King LS, Trahms C. Scott CR. Tyrosinemia type 1. GeneReviews.

American academy of pediatrics 320


American Academy of Pediatrics PREP 2015

Item 104
A 7-year-old girl with Down syndrome presents to your office for evaluation of nonbloody, non
painful purulent ear drainage for the past 3 weeks. She had tympanostomy tubes placed 2 years
ago, but the tubes fell out a few months ago. The mother has been using the topical antibiotic-
steroid drops the otolaryngologist gave them previously to treat ear drainage, but there has been
no improvement. The patient is afebrile and looks well. On physical examination, you find a
deep retraction pocket of the tympanic membrane (Item Q104).

ITEM Q104: Ear examination far the girl described in the vignette. Note, the tympanic
membrane is adherent to the stapes (S).

Of the following, the MOST appropriate next step is to


A. begin evaluation for immunodeficiency
B. give a new prescription of the topical recommended by otolaryngology
C. obtain culture and sensitivity of the drainage
D. prescribe a broad spectrum oral antibiotic
E. refer immediately to otolaryngology

American academy of pediatrics 321


American Academy of Pediatrics PREP 2015

Item 104 Preferred Response: E


This case in the vignette is characteristic of an acquired cholesteatoma and the patient should be
referred immediately to otolaryngology. A cholesteatoma is an abnormal accumulation of
squamous epithelium in the middle ear and mastoid. It may be congenital or acquired. Fewer
than 5% of all cholesteatomas are felt to be congenital. Acquired cholesteatomas are an
uncommon result of chronic middle ear disease. The incidence in children ranges from 5 to 15
per 100,000. It is believed that the persistent presence of inflammatory mediators and chronic
eustachian tube dysfunction may lead to atrophy of the tympanic membrane (TM) and formation
of retraction pockets over time. As a retraction pocket is progressively drawn into the middle ear
space because of longstanding negative middle ear pressure, it forms a pouch that traps
desquamating cells, thus creating a cholesteatoma. Old tympanostomy tube sites, the
posterosuperior portion of the TM, and the pars flaccida are the most common areas of
occurrence.

Warning signs for cholesteatoma include the following: (1) white round mass behind an intact
TM, (2) deep retraction pocket with or without granulation tissue, (3) focal granulation on the
surface of the TM, (4) persistently draining ear for more than 2 weeks despite treatment, and (5)
new-onset hearing loss in a previously operated ear. As a cholesteatoma grows, it invades and
destroys the local structures, including the ossicles and the temporal bone. Complications include
sensorineural hearing loss, cranial nerve palsies (CN 6, 7), vertigo, venous thrombosis, and
central nervous system infections (brain abscess, meningitis).

Because of the invasive nature of cholesteatomas, whenever a cholesteatoma is suspected by the


physical examination, prompt otolaryngology consultation is mandatory. Surgical excision is the
mainstay of treatment. The risk of recurrence is significant, especially with larger, more
infiltrative lesions, so close postoperative follow-up is important.

Children with Down syndrome, cleft palate, or other craniofacial anomalies are at risk for
recurrent middle ear disease, possibly because of mechanical or anatomic features that
predispose to eustachian tube dysfunction. Indeed, children with Down syndrome may have an
increased risk of immunodeficiency, so this evaluation should be considered; however, it is not
the most important next step in evaluating the child in the vignette who is otherwise well
appearing. The combination of persistent ear drainage despite appropriate topical therapy and the
findings on physical examination should raise concern for cholesteatoma. Therefore,
otolaryngology referral is more urgent. If the patient had more serious bacterial infections, then
immune function should be evaluated. Simply continuing the same topical medication or adding
a broad-spectrum oral antibiotic is not appropriate therapy, given the duration of drainage and
the abnormality of the TM. Performing a culture and sensitivity of the drainage would be helpful
to direct treatment of persistent purulent drainage, but this step alone is not sufficient given the
concern for cholesteatoma on physical examination.

PREP Pearls
• Prompt otolaryngology consultation is mandatory whenever a cholesteatoma is suspected
on physical examination.
• Acquired cholesteatomas occur most commonly in areas of chronic retraction pockets,
old perforations, or previous surgical sites of the tympanic membrane.
American academy of pediatrics 322
American Academy of Pediatrics PREP 2015

• If left untreated, cholesteatomas will invade and destroy the local structures (ossicles and
temporal bone).
• The complications of untreated cholesteatomas are serious and include sensorineural
hearing loss, cranial nerve palsies, vertigo, venous thrombosis, and central nervous
system infections.

American Board of Pediatrics Content Specification(s)


• Recognize the clinical findings associated with a cholesteatoma and the consequences if
left untreated

Suggested Reading
• Isaacson G. Diagnosis of pediatric cholesteatoma. Pediatrics. 2007;120(3):603-608.
doi:10.1542/peds.2007-0120.
• Isaacson GC. Cholesteatoma in children. UpToDate. Available online only for
subscription.
• Gould JM, Matz PS. Otitis media. Pediatr Rey. 2010:31(3):102-116. doi:10.1542ipir.31-
3-102.
• Sie KC. Cholesteatoma in children. Pediatr Clin North Arm 1996;43(6):1245-1252.

American academy of pediatrics 323


American Academy of Pediatrics PREP 2015

Item 105
A 7-year-old boy is brought to your office for a preoperative consultation prior to tonsillectomy.
The father is concerned because he himself had bleeding 1 week after a tonsillectomy as a child
and required cauterization to control the bleeding. His son has a history of recurrent nosebleeds,
2 to 3 times per week for the past month, with the most recent episode lasting 20 min. The
nosebleeds occur from either nostril and do not seem to coincide with upper respiratory
infections or allergic rhinitis symptoms. He does not have any other bleeding symptoms. The
child had a circumcision as a neonate without any bleeding complications, but has not had any
other invasive procedures since then. Family history is also significant for a paternal grandfather
who was known to be a "free bleeder." There are no bleeding symptoms on the maternal side of
the family. On examination, the child has a few bruises of varying ages on both legs.
Examination of the nares reveals injected nasal mucosa with no sites of active bleeding. The
remainder of the physical examination is unremarkable. His vital signs are stable. The following
are the results of the child's complete blood cell count:

• White blood cell count, 7,500/µl (7.5 x 109/L), with normal differential
• Hemoglobin, 9.5 g/dL (95 g/L)
• Mean corpuscular volume, 72 µm3 (72 fl)
• Platelet count, 245,000 x 103/ µl (245 x 109/L)
• Prothrombin time, normal
• Partial thromboplastin time, normal

Of the following, the MOST appropriate next step in management would be to


A. check a bleeding time
B. check a factor VIII activity level
C. check a factor XII activity level
D. check a von Willebrand panel
E. reassure based on the normal coagulation test results

American academy of pediatrics 324


American Academy of Pediatrics PREP 2015

Item 105 Preferred Response: D


The child in the vignette with nosebleeds, bruising, and a family history suggestive of autosomal
dominant inheritance has a presentation most consistent with von Willebrand disease (vWD),
therefore the most appropriate next step in management would be to order a von Willebrand
panel.

Von Willebrand factor (vWF) promotes platelet adhesion and aggregation, and acts as a carrier
protein for factor VIII. A deficiency or defect in vWF impairs platelet function and leads to a
more rapid degradation of factor VIII. This can cause mucocutaneous bleeding symptoms, such
as menorrhagia, epistaxis, or hemorrhage after tonsillectomy. Von Willebrand disease is the most
common inherited bleeding disorder, and is caused by a quantitative or qualitative defect in the
vWF protein. Type 1 vWD is an autosomal dominant partial deficiency of vWF. It accounts for
75% of all diagnoses of vWD, with an estimated prevalence of up to 1% of the population. It is
associated with a mild to moderate risk of bleeding. In many patients with type I vWD, the vWF
activity may not be low enough to lower the factor VIII activity below the normal range. In that
case, the partial thromboplastin time (PTT) will remain normal, as seen in the boy in the
vignette. Because vWF does not have a direct effect on prothrombin time (PT)-based factors
(factor 7 and tissue factor), the PT will also be normal in vWD.

Type 3 vWD is an autosomal recessive homozygous deficiency (ie, absence) of vWF, and leads
to severe disease that can mimic hemophilia. Type 2 vWD has several variants, all of which
result from qualitative defects (abnormal functioning) in the vWF protein. Making the diagnosis
of vWD can be challenging because many factors can transiently increase vWF levels, such as
stress (eg, surgery, anxiety, exercise), inflammation, and estrogen. Samples must be handled
properly and processed in a timely manner to avoid proteolysis and platelet contamination. For
this reason, the National Heart Lung and Blood Institute (NHLBI) states that “repeated testing
for vWD is sometimes needed to identify low levels of vWF."

According to the NHLBI vWD expert panel clinical practice guidelines published in 2007, the
criteria for diagnosis of vWD are the following:
• Clinical criteria: Personal or family history of mucocutaneous bleeding or physical
evidence of mucocutaneous bleeding.
• Laboratory criteria: vWF activity assay (eg, as measured by ristocetin cofactor assay) less
than 30 IU/dL.
These NHLBI criteria do not preclude the diagnosis of vWD in patients with only mildly
decreased vWF activity levels of 30 to 50 IU/dL, if there is a strong personal or family history of
bleeding symptoms.

The treatment options for patients with vWD include therapy to increase vWF levels and
antifibrinolytics. Desmopressin 1- desamino-8-D- arginine vasopressin (DDAVP) administered
intravenously or intranasally (only Stimate has the appropriate concentration of DDAVP for use
in bleeding disorders) can cause release of vWF from the endothelial cells. Von Willebrand
factor levels should be checked before and after administration of this medication, in either form,
to document an adequate rise in vWF levels in response to this medication. In patients who do
not have a sufficient response to DDAVP, or in cases of acute life-threatening hemorrhage, vWF
can be immediately replaced using plasma-derived concentrates containing vWF and factor VIII.
American academy of pediatrics 325
American Academy of Pediatrics PREP 2015

The different plasma-derived concentrates vary in their ratios of factor VIII to vWF, and in their
levels of high-molecular weight vWF multimers. Antifibrinolytics such as aminocaproic acid and
tranexamic acid are given orally or intravenously and can be useful in controlling
mucocutaneous bleeding. These agents inhibit fibrinolysis and thereby help stabilize clots once
they are formed. Topical agents such as bovine thrombin and fibrin sealant may be used
intraoperatively or in the setting of traumatic wounds. 'Whenever possible, patients should be
referred to a pediatric hematologist or the regional Hemophilia Treatment Center for
comprehensive care.

The patient in the vignette who presents with recurrent, prolonged nosebleeds, normal PT and
PTT, and a family history of paternal relatives with bleeding symptoms should raise concern for
possible vWD. If the bleeding symptoms were present on the maternal side of the family, factor
VIII deficiency (an X-linked recessive disorder) should be a consideration. Factor VIII
deficiency would also cause prolongation of the PTT. Factor XII deficiency is a cause of
prolonged PTT, but is not associated with any clinically significant bleeding. Bleeding time was
used as a screening test for vWD and platelet disorders in the past, but is no longer recommended
because of its poor reliability and low sensitivity.

PREP Pearls
• Von Willebrand disease (vWD) is the most common inherited bleeding disorder, and is
caused by a quantitative or qualitative defect in the von Willebrand factor (vWF) protein.
• Making the diagnosis of vWD can be challenging because many factors can transiently
increase vWF levels. such as stress (eg. surgery, anxiety, exercise), inflammation, and
estrogen. Samples must be handled properly and processed in a timely manner to avoid
proteolysis and platelet contamination. For this reason, the National Heart Lung and
Blood Institute states that "repeated testing for vWD is sometimes needed to identify low
levels of vWF.
• A deficiency or defect in vWF can cause mucocutaneous bleeding symptoms, such as
menorrhagia, epistaxis, or hemorrhage after tonsillectomy.

American Board of Pediatrics Content Specification(s)


• Recognize the clinical findings associated with von Willebrand disease
• Plan the appropriate management of von Willebrand disease

Suggested Reading
• Hyatt SA, Wang W, Kerlin BA, O'Brien 5H. Applying diagnostic criteria for type 1 von
Willebra rid disease to a pediatric population. Pediatr Blood Cancer. 2009;52(0:102-107.
doi;10.1002/pbc.21755.
• US Department of Health and Human Services, National Heart Lung and Blood Institute;
NHLBI von Willebrand disease expert panel. Clinical Practice Guidelines: Me Diagnosis.
Evaluation, and Maliageinew of von Willebrand Disease.N1H publication 08-5832.
Bethesda, MD: National Institutes of Health; December 2097.

American academy of pediatrics 326


American Academy of Pediatrics PREP 2015

Item 106
The parents of a 10-year-old boy report being very concerned with his persistent lying both to
them and to other adults. They report that he started lying to them when he was 3 years of age.
Examples of lies he has told recently include denying he had broken a lamp in their house,
saying that he had finished his homework when he had not, and denying having taken a toy from
his sister when he had. His parents report that they regard lying as sinful and that his older sister
almost never lied. The parents and the boy often get into significant arguments about a suspected
lie. They report that he is frequently oppositional at other times as well.

Of the following, the BEST next step for his parents to take is to
A. ask him for more details that would falsely embellish the suspected lie, then confront him
about the inconsistencies
B. bring him to a mental health counselor since his lying began at 3 years of age
C. implement a negative consequence, such as no television for 2 weeks, for every lie told
D. provide discipline based on the child's behavior without reference to the specific lie
E. tell their son that he will earn screen time after a set number of days without lying

American academy of pediatrics 327


American Academy of Pediatrics PREP 2015

Item 106 Preferred Response: D


Lying behavior can cause a great amount of anxiety and concern for parents. Telling lies is a
normal human behavior that appears long before children could reasonably be expected to adopt
a personal moral code of truthfulness or honesty. As soon as children realize that their parents
are not always aware of their actions, lying to obtain benefits or to avoid negative consequences
becomes a possibility.

Parents often engage in dysfunctional scenarios regarding lying interventions. For example, a
parent who already knows that a child has done something that is not allowed, such as taking a
sibling's toy, may initiate a behavior intervention with a question like, "Did you take her toy?"
This approach essentially asks the child to practice lying, because the child knows a negative
consequence will result if they say yes and a negative consequence will result if they say no but
are still found out. The only desirable option left for the child would be to tell a lie convincingly
enough to succeed in avoiding a negative consequence. In other words, working to catch a child
lying and then punishing those lies can have the opposite effect, ultimately, of training the child
to get better and better at telling lies over time. Behavior therapists see "intermittent
reinforcement" in this sequence, in that a behavior that is occasionally rewarded is often
repeated.

The preferred approach with a child who is repeatedly lying to a parent to avoid punishment is to
provide discipline/consequences based on what the parent actually knows transpired. In a sibling
conflict where each child makes a case about the other being at "fault” a disciplinary choice is
required which affects both individuals for involvement in the transgression, rather than applying
discipline based on the most convincing story. Avoiding giving the child opportunities for and
encouragement to lie demonstrates the parenting value that "actions speak louder than words"
This is not to say that lying behavior should never be punished on its own. However, when lying
is a recurrent issue, switching the parents' focus to delivering consequences for the specific
misbehaviors, rather than the lies they attempt to cover up, helps to halt a worsening lying habit.
Lying at 3 years of age is not unusual or pathological for a normally developing child. A typical
lie a 3-year-old child might tell could involve eating unobserved a cookie that they were told not
to eat, and then when asked if they ate the cookie the child answers "no." The fact that the child
in the vignette was first observed to lie at age 3 years would not indicate a need for mental health
treatment.

Child mental health counseling, in general, is not an appropriate or effective intervention for
isolated lying behavior. However, referral to a mental health professional who can provide
parenting support and advice might be appropriate, if the lying and/or other areas of parenting
difficulty are causing significant dysfunction in the household.

Temporarily removing access to something the child enjoys as a consequence for a behavioral
transgression is an appropriate behavior management strategy throughout most of childhood.
However, the nature, timing, and duration of this consequence all greatly influence the
effectiveness. For instance, a 5-year-old child is unlikely to remember the reason for receiving a
punishment, such as not being allowed to watch television for 2 full weeks. Another risk with
long-lasting consequences at young ages is that other undesired behaviors are quite likely to
occur during that time frame, which leads the parent to enact additional restrictions that can no
American academy of pediatrics 328
American Academy of Pediatrics PREP 2015

longer be differentiated by the child, such as having their no-television ban extended for multiple
months. Once everything rewarding or fun has been removed as transgressions stack up, parents
may find they have little leverage left with their child.

Stealing can be a more problematic behavior than lying, and the parents should intervene by
having the child undo the theft. The child should either return stolen articles or render their
equivalent in earned money or reparation services. Children with recurrent stealing behaviors
should be referred for professional assistance.

PREP Pearls
• Lying is not an abnormal behavior for children.
• For children who lie, parents should try to deliver discipline based on their child's known
actions rather than in response to the child's story.
• Trying to actively catch children telling lies is counterproductive, providing a reward for
successful lying behavior.
• For children who steal, part of the discipline response should be for parents to help the
child make amends by having them return the stolen goods.

American Board of Pediatrics Content Specification(s)


• Plan the appropriate management of lying behavior in school-age children
• Plan the appropriate management of stealing behavior in school-age children

Suggested Reading
• Ahern EC, Lyon TD. Young children's emerging ability to make false statements. Dev
Psychol. 2011;47(1):61-66. doi: 10.1037/a0021272.
• Phelan TW. 1-2-3 Magic: Effective Discipline for Children 2-12. 3rd ed. Glen Ellyn, IL:
ParentMagic Inc; 2003.

American academy of pediatrics 329


American Academy of Pediatrics PREP 2015

Item 107
A 4-month-old infant with Tetralogy of Fallot is brought to your office for a routine health
supervision visit. He has been doing well, growing and thriving, and has not had any episodes of
cyanosis. He is scheduled for a cardiology appointment in 1 week, with the plan for surgical
repair scheduled for the week after. On arrival at your office, his oxygen saturation is 88% on
room air. He is well appearing and interactive. A physical examination reveals that his weight is
6.1 kg, his heart rate is 120 beats/min, and his respiratory rate is 28 breaths/min. His chest is
clear. His cardiac examination is significant for a regular rate and rhythm. There is a long 4/6
systolic murmur heard at the left, mid to upper sternal border.

Immediately after the administration of vaccines, he becomes visibly more cyanotic. Repeat
oximetry is now 60% on room air, and the baby is breathing 60 breaths/min. There is a-soft 1/6
systolic murmur at the left upper sternal border.

Of the following, the BEST initial intervention for this patient is to


A. administer diphenhydramine
B. administer epinephrine
C. administer oxygen at 2 L by nasal cannula
D. apply ice bag to face
E. place the infant in prone knee chest position

American academy of pediatrics 330


American Academy of Pediatrics PREP 2015

Item 107 Preferred Response: E


The patient in this vignette has Tetralogy of Fallot. The best initial intervention for the worsening
cyanosis would be to place him in a prone knee chest position.

Cyanotic congenital heart disease is a group of lesions that allow desaturated blood to enter the
systemic circulation. The lesions include those that have too little pulmonary blood flow or right-
to-left shunting. Conditions with decreased pulmonary blood flow include Tetralogy of Fallot,
critical pulmonic stenosis, and congenital heart disease that has been palliated with a modified
Blalock-Taussig (BT) shunt. Complications can occur from the newborn to early adult period
and differ by lesion and type of surgical repair.

Patients with each of these conditions will have distinctive physical examination findings that
may help differentiate their underlying diagnosis and treatment if they were to present in distress
to a new facility or physician.

The patient with the BT shunt will have either a median sternotomy or a thoracotomy scar on
either side of their chest. He or she will have a loud, continuous murmur on the side of the
thoracotomy scar that can be best heard in the midclavicular region. If the murmur is very soft or
becomes inaudible in a patient with acute cyanosis, suspect shunt occlusion. This can occur in
the setting of dehydration or fever and is life threatening. An infant with an occluded BT shunt
will have decreased oxygen saturation from their usual baseline of 75% to 85% that will not
change with oxygen delivery. The initial management in this setting is to give fluids to attempt to
increase flow through the shunt.

Patients with Tetralogy of Fallot are susceptible to hyper-cyanotic spells or "Tet Spells" when
they are volume depleted or dehydrated. They also have intracardiac and specific anatomy and
physiology that makes their pathophysiology different from the patients with occluded BT shunt.
The components of Tetralogy of Fallot are pulmonic stenosis, aortic override, ventricular septal
defect (VSD), and right ventricular hypertrophy. The flow through the VSD is usually left to
right because the right ventricular pressure is less than that of the left, and the pulmonary
vascular resistance is less than the systemic vascular resistance. The specifics of the right
ventricular outflow tract obstruction will determine the likelihood of an infant having a Tet spell,
but the initiation will begin with a sudden drop in systemic vascular resistance, such as might be
seen with crying or fever. This will cause the intracardiac shunt across the VSD to become right
to left (from the right ventricle to the left ventricle). A loud pulmonary stenosis murmur that
becomes soft or absent in this patient is a dangerous finding because it means that the flow out of
the right ventricle has decreased. As the desaturated blood goes out to the periphery, it will cause
acidosis and hyperventilation. This in turn will cause increased rate of blood return to the right
heart and exponentially worsen the desaturation as more blood is sent across the VSD. The
pulmonic stenosis does not allow much of this returning blood to go to the lungs and get
oxygenated. Such infants' saturations will continuously decrease as the spell worsens. The Tet
spell physiology can be treated in several ways, but the initial step is to increase the systemic
vascular resistance quickly. The knee chest maneuver, as done for the infant in the vignette, will
instantly increase the systemic vascular resistance and can be done before any equipment is
available.

American academy of pediatrics 331


American Academy of Pediatrics PREP 2015

The patient with critical pulmonic stenosis who has no VSD, on the other hand, will have a loud
murmur that will be proportional to the obstruction at the pulmonary valve. This patient's
murmur will get louder as the obstruction gets worse. This infant's saturations will decrease but
not become critically low.

Oxygen would not be dangerous for any of the infants described above. They all have decreased
pulmonary blood flow and increasing the dissolved oxygen and saturating all the hemoglobin in
the blood is desirable. However, 2 L of nasal cannula would not provide enough oxygen to treat
a Tet spell. In such situations, 100% oxygen by a non-rebreather mask would be needed.
Although there is decreased pulmonary blood flow in the midst of a Tet spell, providing
supplemental oxygen helps to increase the dissolved oxygen concentration in the systemic
circulation.

Increasing the systemic vascular resistance is the first intervention for a Tet spell. It would not be
helpful for the infant with a BT shunt obstruction or the patient with critical pulmonic stenosis.
Diphenhydramine would sedate the infant slightly and possibly lower the systemic blood
pressure if given intravenously. This could worsen the Tet spell, even though sedation would
help the hyperventilation. Morphine is used as one of the interventions in a Tet spell to both
sedate and decrease hyperventilation.

Epinephrine would increase the systemic vascular resistance and the blood pressure. It might
help decrease the right to left shunt, but would not be the first intervention.
Ice to the face would cause a vagal response and lower the heart rate. It could also increase the
infant's agitation and crying, thereby lowering the systemic vascular resistance. This could make
the Tet spell worse.

The knee chest maneuver is easy to do and can be done at the first sign of a Tet spell. This
maneuver can often stop the spell immediately and is the first intervention while oxygen,
intravenous fluids, and possibly morphine are being assembled.

PREP Pearls
• The first intervention in a hypercyanotic spell in patients with Tetralogy of Fallot (TOF)
is to place the child in the prone knee chest position.
• Loud murmurs that disappear in patients with a Blalock-Taussig Shunt or with TOF are
red flags for dangerous, potentially life-threatening physiology.

American Board of Pediatrics Content Specification(s)


• Recognize complications associated with cyanotic congenital heart disease

Suggested Reading
• Bernstein D. Tetralogy of Fallot. In: Kliegman RM, Behrman RE, Jenson HB, Stanton
BF, eds. Nelson Textbook of Pediatrics. 18th ed. Philadelphia, PA: Saunders Elsevier;
2007:430.1.
• Park MP, Salamat M, eds. Tetralogy of Fallot. The Pediatric Cardiology Handbook.
Philadelphia, PA: Mosby Elsevier; 2010:130-135.

American academy of pediatrics 332


American Academy of Pediatrics PREP 2015

Item 108
You are supervising an intern in the outpatient pediatric clinic. She is scheduled to see a 6-year-
old girl with a chief complaint of leg pain. You review the patient's chief complaint and the signs
and symptoms that will guide your differential diagnosis. You discuss the causes of joint pain
and the more and less concerning symptoms. The resident asks you what symptoms would
require an immediate workup.

Of the following, the MOST accurate response to the resident's question would be
A. decreased range of motion and morning stiffness of the knee
B. knee pain and grinding sensation when walking upstairs
C. knee swelling and pain with activity
D. knee swelling associated with fever
E. nighttime knee and leg pain associated with activity

American academy of pediatrics 333


American Academy of Pediatrics PREP 2015

Item 108 Preferred Response: D


For the child in this vignette, knee swelling associated with fever is the most concerning
presentation. Any patient with swelling in a single joint associated with fever should be referred
immediately for orthopedic evaluation, because septic arthritis is a medical emergency.

The specific symptoms and patterns of pain associated with any case of joint pain can lead the
clinician to the appropriate disease process and referral. The history should address several
specific domains that can help guide the diagnosis. The location of the pain is important. The
clinician should assess whether the pain is localized to the joint or exists in other areas such as
the muscle, tendon, or bone, as well as the time of day and pattern of the pain. Morning pain that
lasts 1 hour or more, improving throughout the day, points toward an inflammatory cause. If the
pain is constant, an orthopedic cause should be considered. Pain that is poorly localized suggests
a pain syndrome. Pain that is worsened by a specific activity or is worse on more active days
points to an overuse syndrome. Nighttime pain can be caused by benign nocturnal limb pain of
childhood ("growing pains") or benign joint hypermobility; however, severe nighttime pain
should always make one consider leukemia and lymphoma. If swelling is reported, the clinician
should discern whether the swelling is persistent or transient. Swelling caused by arthritis will
persist for days at a time, whereas overuse syndromes can have transient swelling that comes and
goes, lasting a few hours to a day. Limping associated with inflammation is usually more
pronounced in the morning and improves with activity, whereas limping associated with injury
or overuse is usually worsened with activity. The clinician should also inquire about systemic
symptoms such as fever, rash, weight loss, history of recent illness, and infectious exposures.
Fever and weight loss associated with bone or joint pain and swelling should raise concern for
leukemia and lymphoma. Several infections can be associated with arthritis such as Lyme
disease, parvovirus B19, and mycoplasma. These patients will typically have other symptoms
and may have a history of exposure. Patients with transient synovitis will have a history of recent
illness.

Joint pain in the absence of arthritis is a common com-plaint in the pediatric clinic. The most
common causes of joint pain in children include benign nocturnal limb pain of childhood, benign
joint hypermobility, overuse syndromes, and pain amplification syndromes (Item C108).

Decreased range of motion and morning stiffness of the knee is consistent with an inflammatory
arthritis, but the differential diagnosis of arthritis in a single joint should also include septic
arthritis.

Knee pain and a grinding sensation when walking upstairs is the presenting pattern for
patellofemoral syndrome, an overuse syndrome that is common in teenagers.

Knee pain with activity is likely caused by trauma or overuse. Overuse syndromes may cause
mild swelling and should be considered in a patient who performs repetitive movements that
cause well-localized pain. The pain occurs during the repetitive activity, but can also occur at
other times when the specific joint is used. These syndromes result from increased stress over
areas of growth during skeletal maturation. Overuse pain should be managed with modification
of the repetitive stress-causing activity and nonsteroidal anti-inflammatory drugs. If bone

American academy of pediatrics 334


American Academy of Pediatrics PREP 2015

fragments or skeletal defects are found on imaging studies, the patient should be referred to
orthopedics. Most overuse syndromes respond well to physical therapy.

Item C108. Causes of Joint Pain in Children


Infection or infection- Lyme disease
related Osteomyelitis
Post streptococcal arthritis
Reactive arthritis
Rheumatic fever
Septic arthritis
Toxic synovitis
Inflammatory Juvenile idiopathic arthritis
Systemic lupus erythematosus
Other autoimmune conditions
Hematology, oncology Leukemia
Lymphoma
Hemophilia
Sickle cell disease
Orthopedic, trauma Fracture
Sprain and strains
Slipped Capital femoral epiphysis
Legg-Calve-Perthes disease
Panner disease Meyer dysplasia
Osteochondritis dissecans
Kohler disease
Freiberg disease
Overuse syndromes Little league shoulder
Little league elbow (golfer's elbow)
Tennis elbow
Gymnast's wrist
Iliac crest apophysitis
Anterior superior iliac spin
Sinding-Larsen-Johansson
Osgood-Schlatter disease
Sever disease
Other musculoskeletal Benign nocturnal pains of childhood
pain Fibromyalgia
Complex regional pain syndrome

Amplified pain syndromes should be considered when musculoskeletal pain is not well localized,
is of fluctuating intensity without remission, and systemic symptoms are absent. These patients
may also have physical limitations, mood changes, poor sleep, and social dysfunction. Findings
on laboratory tests and imaging studies are unremarkable. Complex regional pain syndrome

American academy of pediatrics 335


American Academy of Pediatrics PREP 2015

should be considered when pain is localized to a specific region and is associated with
neuropathic symptoms (burning, dysesthesia, paresthesia, allodynia, cold hyperalgesia) and
autonomic dysfunction (cyanosis, mottling, hyperhidrosis, coolness, and edema).

Nighttime knee and leg pain associated with activity is commonly seen with benign joint
hypermobility. These patients have joint pain that occurs or worsens later in the day, is worsened
by activity, and is relieved by rest. The pain comes and goes and is localized to the joints.
Patients can have short-lived morning complaints lasting a few minutes. These patients can
hyperextend their knees and elbows past 190°, touch thumb to wrist, extend the pinky to 90°, and
place their hands flat on the floor without bending at the knee. If there are associated skin
abnormalities, recurrent joint dislocations, or lens abnormalities, the patient should receive
further evaluation for the genetic syndromes associated with hypermobility. Physical therapy
including isometric exercises, proprioception training, and minimizing stretching is a good first
step in treatment. Joint protection using soft bracing can be beneficial during sports.

Nighttime limb pain can also be caused by benign nocturnal limb pain of childhood, which is a
characterized by chronic intermittent bilateral shin pain without daytime symptoms or limitation.
Benign nocturnal limb pain of childhood commonly occurs between 3 and 5 years of age and 8
and 12 years of age (outside of the growth spurt periods). Criteria for diagnosis include 3 months
of pain; intermittent pain with symptom-free intervals of days, weeks, or months; pain late in the
day or awakening the child at night; pain not related specifically to joints; pain severity
significant enough to interrupt normal activity such as sleep; and normal physical examination,
laboratory, and radiographs.

Patients with generalized pain of high intensity may have juvenile fibromyalgia. Although there
are no specific diagnostic criteria in children, these patients usually present with poorly localized
diffuse pain, allodynia, disturbed sleep, fatigue, and functional impairment. These patients
commonly have comorbid conditions such as migraine, irritable bowel syndrome, chronic
abdominal pain, and chronic fatigue. Laboratory studies are normal in these patients. Several
clues should suggest this diagnosis: incongruent affect (La belle indifference), high pain
tolerance reported by the family, enmeshment, sleep disturbance, missed school days, and
allodynia. Treatment takes a multifocal approach including addressing the pain, encouraging
return to school and other activities, exercise, physical therapy, desensitization, and counseling.

PREP Pearls
• Benign nocturnal limb pain of childhood ("growing pains") is characterized by nighttime
chronic intermittent bilateral shin pain without daytime symptoms or limitation.
• Benign joint hypermobility is characterized by joint pain occurring at the end of the day,
worsened with activity, and improved with rest in a child with joint hyperextension.
• Overuse syndromes should be considered in a patient performing repetitive movements
that cause well-localized pain.

American Board of Pediatrics Content Specification(s)


 Recognize the typical presentation of a patient with functional joint complaints

American academy of pediatrics 336


American Academy of Pediatrics PREP 2015

Suggested Reading
• Ansel) BM. Rheumatic disease mimics in childhood. Curl- Opin Rheinnatol.
2000;12(5):445.
• Kimura Y, Southwood, TR. Evaluation of the child with Joint pain or swelling.
UpToDate. Available online only by subscription.

American academy of pediatrics 337


American Academy of Pediatrics PREP 2015

Item 109
A 15-year-old adolescent boy presents to your office for evaluation 1 day after experiencing an
injury to the ear during rugby practice. He reports that during a scrum, the pinna of the right ear
was pulled forward, causing his ear to "fold over!' He now has pain and swelling of the auricle.
On physical examination, you note a large nodular swelling over the superior aspect of the
auricle.
Of the following, the MOST appropriate management of this injury is
A. aspiration of the swollen area with compression dressing
B. observation only
C. surgical removal of the nodule
D. topical antibiotics to prevent secondary infection
E. topical steroid medication to decrease swelling

American academy of pediatrics 338


American Academy of Pediatrics PREP 2015

Item 109 S Preferred Response: A


The boy in the vignette has sustained a large auricular hematoma as a result of trauma to the
pinna. Acute drainage of the hematoma and application of a compression dressing will decrease
his risk of permanent deformity. Auricular hematomas generally result from blunt trauma or
shearing injury to the pinna, most often as a result of participation in rugby or wrestling.
Auricular hematoma can lead to the formation of excess fibrocartilage beneath the
perichondrium, resulting in a deformity known as "cauliflower ear!' Treatment consists of
aspirating the hematoma using sterile technique with an 18- or 20-gauge needle, followed by
application of a compression dressing to decrease the risk of hematoma reaccumulation. To help
prevent auricular hematomas, primary care physicians should counsel young rugby players and
wrestlers that use of protective headgear can decrease the risk of ear injury.

Observation is not appropriate for this patient, because drainage is needed to minimize the risk of
deformity. Referral to otolaryngology for surgical incision and drainage is indicated for recurrent
auricular hematoma. Oral antibiotic prophylaxis may be indicated in patients who require
surgical drainage, but is generally not needed after needle aspiration. Steroid medication will not
alleviate the hematoma.

PREP Pearls
• Auricular hematomas occur because of direct or shearing forces applied to the pinna of
the ear, most often resulting from participation in rugby or wrestling.
• Prompt drainage of an auricular hematoma with application of a compressive dressing
can decrease the risk of permanent deformity.
• Use of protective headgear during wrestling and rugby may decrease the risk of auricular
hematoma.

American Board of Pediatrics Content Specification(s)


• Plan the appropriate management of a sports-related ear injury

Suggested Reading
 Jones SE, Mahendran 5. Interventions for acute auricular haematoma. Cochrane Database
Syst Rev. 2004;2:CD004166. dal:10.1002/14651858. C DO 04166 .pub2.

American academy of pediatrics 339


American Academy of Pediatrics PREP 2015

Item 110
A 7-year-old boy with a history of urinary withholding presents to your office with left scrotal
pain and swelling that developed gradually over the last 3 to 4 days. He now complains of pain
with urination, as well as subjective fever. He denies nausea, vomiting, or abdominal pain. On
physical examination, his temperature is 38.5°C, heart rate is 70 beats/min, respiratory rate is 20
breaths/min, and blood pressure is 100/60 mm Hg. The left scrotal area is swollen and tender to
palpation. Pain is relieved with testicular elevation. There is a normal cremasteric reflex.
Urinalysis results are unremarkable.

Of the following, the MOST likely diagnosis is


A. acute cystitis
B. epididymitis
C. scrotal hernia
D. testicular torsion
E. urethritis

American academy of pediatrics 340


American Academy of Pediatrics PREP 2015

Item 110 Preferred Response: B


The boy described in the vignette presents with epididymitis characterized by left scrotal pain
and swelling associated with dysuria, fever, a normal cremasteric reflex, and pain relief with
elevation of the testis. Epididymitis is more common in older adolescents, especially those who
are sexually active, but can occur in prepubertal males and those without a history of sexual
activity. The typical presentation is acute or subacute scrotal pain and swelling with urinary
frequency, dysuria, and urethral discharge. Fever may be present. On physical examination, the
affected testis is in the appropriate position (vertical lie) in the scrotum, and there may be some
erythematous overlying skin. Occasionally, an inflammatory nodule can be palpated along the
epididymis. The cremasteric reflex should be present, and this finding (in addition to the position
of the testis in the scrotum) can distinguish epididymitis from testicular torsion. Relief with
elevation of the testis (Prehn sign) may or may not be present.

In epididymitis, urinalysis can be unremarkable or pyuria may be present, but the urine culture
often is negative. In sexually active boys, the most common pathogen is Chlamydia followed by
Neisseria gonorrhea, Escherichia coli, and then viruses. In younger children, Mycoplasma and
viruses are most common. Bacterial urinary tract pathogens, such as E coli, causing epididymitis
in prepubertal boys and adolescents who are not sexually active, suggest structural abnormalities
of the urinary tract.

Treatment of epididymitis in children should be directed at the underlying etiology. In


prepubertal and adolescent boys who are not sexually active and lack evidence of urinary tract
infection, treatment is supportive and includes rest and nonsteroidal anti-inflammatory agents.
Whether or not to give antimicrobial therapy is controversial and should be based on the
individual patient's presentation and the clinician's practice.

Patients with acute cystitis may have lower abdominal complaints, dysuria, pyuria, a positive
urine culture (if caused by a bacterial pathogen), and lack testicular com-plaints. Urethritis is
usually caused by a sexually transmitted infection, and is characterized by urethral discharge,
dysuria, itching or burning, and heaviness in the testes without pain. However, patients may be
asymptomatic when Chlamydia trachomatis is the cause. Urethritis can accompany epididymitis.
A scrotal hernia is an inguinal hernia extending into the scrotum. There typically is an obvious
bulge within the scrotum that may or may not be associated with pain. Unlike the swelling
caused by epididymitis, the bulge from a hernia can be intermittent and increased with Valsalva
maneuvers. An incarcerated hernia would be associated with increasing pain, a tender firm mass
in the scrotum, and the overlying skin could be erythematous or discolored. Elevation of the
testis would not relieve the pain.

PREP Pearls
• Epididymitis is characterized by scrotal pain and swelling associated with dysuria, a
normal cremasteric reflex, and pain relief with elevation of the testis.
• Epididymitis is more common in older adolescents, especially those who are sexually
active, but can occur in prepubertal males and those without a history of sexual activity.
• In sexually active males, the most common etiologies of epididymitis are Chlamydia and
Neisseria gonorrhea, followed by Escherichia coli and viruses.

American academy of pediatrics 341


American Academy of Pediatrics PREP 2015

• In prepubertal children and those who are not sexually active, Mycoplasma and viruses
are the most common causes of epididymitis.
• Routine bacterial urinary tract pathogens, such as E coli, causing epididymitis In
prepubertal boys and adolescents who are not sexually active, suggest structural
abnormalities of the urinary tract

American Board of Pediatrics Content Specification(s)


• Recognize the clinical findings associated with epididymitis

Suggested Reading
 Breland I)), Rubinstein ML. Scrotal masses. In: Rudolph C, Rudolph A, Lister G, First L,
Gershon A, eds. Rudolph's Pediatrics. 22nd ed. New York, NY: McGraw-Hill; 2011:294-
295.
 Hwang LY, Moscicki AB, Shafer MA. Sexually transmitted infections. In: Rudolph C.
Rudolph A, Lister G. First L. Gershon A, eds. Rudalph's Pediatrics. 22nd ed. New York,
NY: McGraw-Hill; 2011:932-933.
 Somekh E, Gorenstein A, Serour F. Acute epididymitis in boys: evidence of a post-
infectious etiology.] Urol. 2004;171(11:391-394. doi:10.1097/01.
ju.0000102160.55494.1f.
 Tracy CR, Steers WD, Costabile R. Diagnosis and management of epididymitis_ Urol
Clin North Ant. 2008;35(1):101-108. doi:10.1016/j. uc1.2007.09.013.

American academy of pediatrics 342


American Academy of Pediatrics PREP 2015

Item 111
An 18-month-old girl who is new to your practice presents for evaluation of right wrist swelling,
which her mother initially noticed approximately 1 week ago. There is no reported history of
trauma. The girl has had no recent fevers, and she displays no other symptoms of illness. The
mother tells you that the girl has been healthy, takes no medications, and that she is adopted and
came to live with her new family only 3 months ago. Few details were provided to the adoptive
family regarding the child's birth history, past medical history, and family medical history. The
girl reportedly began walking around 14 months of age, although the mother notes that “she
always seems bow-legged when she walks”
The girl's vital signs are within normal limits; her height and weight are both at the fifth
percentiles for her age. She is playful and appears well. On physical examination, you note that
the girl has a prominent forehead, but no facial dysmorphisms. Her right wrist seems slightly
swollen when compared to the left wrist, but there is no overlying bruising, warmth, or erythema,
and the girl is able to flex and extend both wrists without limitation and they are nontender and
have no bruising. The remainder of a full physical examination is unremarkable.

You order plain radiographs of the girl's right wrist, which reveal widening of the distal radius
and ulna with cupping of the right radial metaphysis, in addition to generalized osteopenia (Item
Q111).

Of the following, the MOST likely cause of this child's wrist swelling is
A. Caffey disease
B. child abuse
C. osteogenesis imperfecta
D. rickets
E. scurvy

American academy of pediatrics 343


American Academy of Pediatrics PREP 2015

Item 111 I-C Preferred Response: D


The child in the vignette presents with clinical and radiologic findings consistent with rickets, an
uncommon disorder that can occur in exclusively breastfed infants who are not receiving vitamin
D supplementation or in dark-skinned children who are not exposed to adequate sunlight because
of lifestyle or geographic location. Rickets can be diagnosed by characteristic changes on plain
radiography, such as cupping and fraying of the costochondral junctions and epiphyses,
demineralization, widened epiphyses, and cortical thinning. Children affected by rickets will
have decreased serum concentrations of vitamin D metabolites, and elevated serum alkaline
phosphatase concentration.

The details in the vignette are supportive of a diagnosis of rickets, but differentiating the findings
associated with fractures related to other musculoskeletal disorders from those associated with
inflicted fractures can be challenging in many cases. Children with unexplained fractures can be
victims of inflicted injury. A number of underlying musculoskeletal disorders are associated with
increased bone fragility that can be subtle and difficult to diagnose. Pediatric health care
providers should be aware of certain clinical features and fracture patterns that are associated
with child abuse, which has major implications for the safety and wellbeing of the children
affected, as well as those that are associated with underlying musculoskeletal disorders.

No fracture, when considered in isolation, can distinguish an abusive from a nonabusive cause.
During the evaluation of individual bony abnormalities, the lesion site and type, age and
developmental stage of the child, and associated reported history can help clinicians to determine
the likelihood of abuse. A number of studies, including a recent systematic review of the child
abuse literature, have found that abusive fractures are detected most commonly in infants
younger than 1 year of age. When any infant or child presents with a fracture in the absence of a
confirmed cause, inflicted injury should be considered as a potential cause. Fracture patterns that
should further raise suspicion for child abuse include posterior rib fractures, metaphyseal
fractures (also known as "bucket-handle" or corner fractures), humeral and femur fractures
(especially those occurring in a nonambulatory child), and skull fractures (especially complex
skull fractures). Multiple fractures are also associated with abusive trauma, though they can
result from both accidental trauma or underlying musculoskeletal disorders. Repeat fractures
occurring at an unusual location for repeat injury, fractures in radiographic stages of healing that
do not correspond to the reported clinical history, or multiple fractures in various stages of
healing should heighten a clinician's suspicion for abuse.

When abuse is suspected, a radiographic skeletal survey should be obtained to evaluate the entire
skeletal system for potential injuries. Whenever possible, clinicians should consult with
providers possessing specific expertise and experience in the management of suspected child
abuse cases. Physicians are also required to report suspected child abuse to the appropriate child
protective service and/or law enforcement agency in their respective areas.

Osteogenesis imperfecta (OI) is a musculoskeletal disorder resulting in fractures that are most
frequently mistaken for abusive fractures. Osteogenesis imperfecta is a heterogeneous group of
disorders, usually caused by mutations in the gene involved in the production of type I collagen
(COL1A 1 and COL1A2), which forms the structural framework of bone. Although OI is an
inherited disorder, the presentation of the disease even within the same family can be quite
American academy of pediatrics 344
American Academy of Pediatrics PREP 2015

variable. Osteogenesis imperfecta can usually be diagnosed by obtaining a thorough medical and
family history, performing a physical examination, and interpreting the results of appropriate
laboratory and radiographic studies. Many children-though not all children-with OI will have
diagnostic signs such as osteopenia, bony deformities, and "wormian bones" of the skull on
radiography. Findings such as blue sclera, discolored teeth, limb deformities, and a triangular-
shaped face may be apparent on the physical examination of some children with OI, but these
features may be subtle or absent in a number of patients with OI. The child described in the
vignette has no reported family history of OI, no reported history of prior fractures, and no
physical examination findings that would suggest the diagnosis of OI.

Caffey disease, also called infantile cortical hyperostosis, is a rare musculoskeletal disorder that
most often occurs in infants. A characteristic feature of Caffey disease is excessive new bone
formation (hyperostosis), which mainly affects the mandibles, scapulae, clavicles, and long bone
diaphyses. Infants affected by Gaffey disease often develop swelling of the joints and soft
tissues, with pain and erythema in the affected areas. Signs and symptoms of Caffey disease are
generally apparent within the first few months after birth and often resolve by early childhood.
The clinical presentation and radiographic findings for the child in the vignette are not consistent
with that of Gaffey disease.

Scurvy, which arises from a deficiency of ascorbic acid (vitamin C), generally presents with
hemorrhagic symptoms (petechiae, ecchymoses, and gum bleeding), hyperkeratosis,
hypochondriasis, and hematologic abnormalities, in addition to nonspecific generalized
symptoms such as malaise, fatigue, anorexia, and failure to thrive. Although tenderness and
swelling over the long bones can occur in children with scurvy, fractures do not generally result
from this disease process. Scurvy would be very unlikely in the girl in the vignette given her lack
of the characteristic signs and symptoms.

PREP Pearls
• No fracture, when considered in isolation, can distinguish an abusive from a nonabusive
cause. The fracture site and type, age and developmental stage of the child. And
associated reported history can help clinicians to determine the likelihood of abuse.
• When any infant presents with a fracture in the absence of a confirmed cause. inflicted
injury should be considered as a potential cause.
• Fracture patterns that are particularly suspicious for an abusive mechanism include
posterior rib fractures, metaphyseal fractures, humeral and femur fractures (especially
those occurring in nonambulatory children), and skull fractures (especially complex skull
fractures).

American Board of Pediatrics Content Specification(s)


• Differentiate the findings associated with inflicted fractures from those of fractures
related to other musculoskeletal disorders

Suggested Reading
• Flaherty EG, Perez-Rosello JM, Levine MA, et al. Evaluating children with fractures for
child physical abuse. Pediatrics. 2014;133(2):e477-e489. doi:10.1542/peds.2013-3793.

American academy of pediatrics 345


American Academy of Pediatrics PREP 2015

• Jenny C. Evaluating infants and young children with multiple fractures. Pediatrics.
2006;118(3):1299-1303. doi:10.1542/peds.2006-1795.
• Kemp AM, Dustan F, Harrison S, et al. Patterns of skeletal fractures in child abuse:
systematic review. BM1. 2008;337:a1518. doi:10.1136/bmj.a1518.
• Pandya NK, Baldwin K, Kamath AF, et al. Unexplained fractures: child abuse or bone
disease: a systematic review. Clin Orthop Relat Res.2011:469(3):805-812.
doi:10.1007/s11999-010-1578-z.

American academy of pediatrics 346


American Academy of Pediatrics PREP 2015

Item 112
The normal nursery staff calls you to assess a term newborn who has turned dusky twice after
breastfeeding. The newborn was delivered 4 hours ago by scheduled elective cesarean delivery at
term following an uneventful pregnancy. Artificial rupture of the membranes occurred at the
time of delivery and revealed copious clear amniotic fluid. The newborn emerged vigorous, and
no resuscitation was needed. The nurses report that the newborn has spit clear mucus several
times since delivery, requiring bulb suctioning. An examination reveals a pink, well-perfused
newborn with a heart rate of 140 beats/min, a respiratory rate of 70 breaths/ min, and an oxygen
saturation of 95% on room air. The newborn has clear lung fields with mild subcostal retractions
and no murmur.

Of the following, the MOST appropriate next step in management is to


A. initiate nasal cannula oxygen supplementation
B. insert an orogastric tube
C. obtain a chest radiograph
D. order an echocardiogram
E. perform chest physiotherapy

American academy of pediatrics 347


American Academy of Pediatrics PREP 2015

Item 112 Preferred Response: B


The newborn in the vignette should have an orogastric tube inserted to rule out esophageal
atresia (EA). A fetus with EA can present in utero with polyhydramnios because of the inability
to swallow amniotic fluid. When the membranes are ruptured before delivery, copious amniotic
fluid may be seen. If not identified in the delivery room, newborns with EA will often have
spitting, drooling, choking, and respiratory distress because of their continued inability to handle
secretions or feedings. These findings should prompt a clinician to attempt to pass an orogastric
tube. The inability to pass an orogastric tube into the stomach suggests an underlying diagnosis
of EA. The diagnosis of EA may be confirmed with chest radiography, which demonstrates
coiling of the catheter tube in the esophagus (Item C112A).

Five types of congenital tracheoesophageal fistula (TEF) and EA have been described, with type
C (EA with distal pouch fistula) occurring in approximately 85% of cases. Four types are
associated with EA, which present in the newborn period . Occurring in only 4% of cases, type E
(H-type fistula with a patent esophagus) may present later in infancy with refractory
bronchospasm and recurrent pneumonias. More than 50% of infants with EA have other
associated anomalies, with VACTERL (vertebral, anorectal, cardiac, tracheal, esophageal, renal,
limb) syndrome most common. Prompt identification of EA in full-term newborns leads to a
survival rate greater than 95% if no cardiac disease is present. The short- and long-term
complications of TEF/EA are related both to the underlying structural abnormalities and the
consequences of the surgical correction .

A newborn with the diagnosis of EA requires transfer to a tertiary center with a pediatric surgical
team. Stabilization at the referring center is focused on the maintenance of a patent airway and
prevention of aspiration pneumonia. After inserting an orogastric tube in the newborn in the
vignette, chest radiography should then be performed to demonstrate the location of the tube in
the esophagus. Supplemental oxygen may be required if an infant with EA is hypoxic, but the
newborn in the vignette has an acceptable oxygen saturation value of 95% in room air.

American academy of pediatrics 348


American Academy of Pediatrics PREP 2015

Endotracheal intubation should be avoided, as severe gastric distention may arise if a distal
pouch fistula is present. Echocardiography should be performed at the tertiary center because up
to 35% of infants with EA are found to have structural heart disease. Chest physiotherapy has no
role in the care of the newborn with EA.

Complication Occurrence Rate


Gastroesophageal reflux 40%
Strictures 6-41%
Anastomatic leak 15-20%
Feeding intolerance nearly 100%
Lower respiratory tract infections, bronchitis, or 85%
aspiration pneumonia
Tracheomalacia 10-20%
Scoliosis and chest wall deformities 16-19%

PREP Pearls
• Esophageal atresia should be considered in newborns with spitting, drooling, choking,
and respiratory distress because of their continued inability to handle secretions or
feedings.
• The inability to pass an orogastric tube into the stomach suggests an underlying diagnosis
of esophageal atresia.

American Board of Pediatrics Content Specification(s)


• Recognize the clinical and laboratory features associated with tracheoesophageal fistula
in a newborn infant

Suggested Reading
• Burge DM, Shah K, Spark P, et al. Contemporary management and outcomes for infants
born with oesophageal atresia. Br/ Surg. 2013;100(4):515-521. do1:10.1002/bjs.9019.

American academy of pediatrics 349


American Academy of Pediatrics PREP 2015

• Khan S, Orenstein SR. Congenital anomalies: esophageal atresia and tracheoesophageal


fistula. In: Kleigman RM, Stanton B, St Geme JW III, Schor NF, Behrman RE, eds.
Nelson Textbook of Pediatrics. 19th ed. Philadelphia, PA: Saunders Elsevier; 2011:1262-
120.
• Oermann CM. Congenital anomalies of the intrathoracic airways and tracheoesophageal
fistula. UpToDate. Available online only for subscription.

American academy of pediatrics 350


American Academy of Pediatrics PREP 2015

Item 113
You are seeing a 25-month-old girl for a health supervision visit. She missed several previous
appointments. She has no chronic medical problems. She has had several hospitalizations for
pneumonia, but never required intubation. At her 12-month visit, she had 2 words and was
pulling to stand and cruising. In the office today, her parents report that she has more than 50
words, puts 2 words together, and can walk independently and run. They are concerned because
she is not able to go up or down stairs and that she is clumsy, falling often. The girl's general
physical examination is unremarkable. Her neurologic examination shows normal eye
movements, normal facial strength, and no tongue fasciculations. She is weak and hypotonic in
all 4 extremities. Her legs are weaker than her arms, she has finger trembling without-stretched
fingers, and you are unable to elicit any reflexes.

Of the following, the test MOST likely to yield the correct diagnosis is
A. cerebrospinal fluid analysis
B. electroencephalogram
C. magnetic resonance imaging of the brain
D. nerve conduction study with repetitive stimulation
E. survival motor neuron gene (SMN1) testing

American academy of pediatrics 351


American Academy of Pediatrics PREP 2015

Item 113 Preferred Response: E


The girl in the vignette has spinal muscular atrophy (SMA). SMA is inherited in an autosomal
recessive pattern, caused by mutations in both copies of the survival motor neuron 1 (SMN1)
gene, located on chromosome 5. SMN1 gene testing establishes the diagnosis. The severity of
the clinical manifestations, however, depends at least somewhat on another gene, survival motor
neuron 2 (SMN2), also located on chromosome 5. The SMN2 gene codes small amounts of the
same protein as the SMN1 gene. The more copies of the SMN2 gene a patient with SMA has, the
milder their symptoms. SMA can present from the neonatal period to adulthood.

Symptoms of SMA in all ages include limb weakness, hypotonia, and minimal or absent
reflexes. Many, but not all patients, have tongue fasciculations. Finger trembling is almost
always present after infancy. Other findings can include arthrogryposis in neonates, bell-shaped
chest with paradoxical breathing in infants, and pronounced lower extremity weakness in older
children. Cognition and facial strength is preserved at all ages.

The girl described in the vignette has gross motor delay and normal language. Her physical
examination results show limb weakness, hypotonia, and areflexia. These findings suggest a
problem of the nerves or muscles. Magnetic resonance imaging of the brain,
electroencephalogram, or cerebrospinal fluid analysis would not help make the correct diagnosis.
Nerve conduction studies with repetitive stimulation evaluate the neuromuscular junction. This
girl does not have symptoms of a neuromuscular junction disorder, such as myasthenia gravis or
botulism, therefore this test would not be the most helpful test.

PREP Pearls
• Spinal muscular atrophy (SMA) can present at all ages, from the neonatal period to
adulthood.
• Symptoms of SMA in all ages include limb weakness, hypotonia, and minimal or absent
reflexes.
• In children with SMA, finger trembling is almost always present, while tongue
fasciculations are often but not always seen.

American Board of Pediatrics Content Specification(s)


• Recognize the clinical findings associated with different forms of spinal muscular
atrophy

Suggested Reading
• Bodamer OA. Spinal muscular atrophy. UpToDate. Available online only for
subscription.
• Prior TW, Russman BS. Spinal muscular atrophy. GeneReviews. Updated January 27,
2011. Accessed April 22, 2013.

American academy of pediatrics 352


American Academy of Pediatrics PREP 2015

Item 114
An 8-year-old girl is referred for evaluation of poor linear growth. Her height was at the third
percentile at 6 years of age, but her linear growth velocity then declined over the next 2 years.
Her weight continues to track at the third percentile. Her history is significant for a mild speech
impairment thought to be related to her high arched palate. On physical examination, you also
appreciate a short fifth metacarpal bone in both hands (Item Q114). The remainder of her
physical examination is unremarkable.

Of the following, the BEST test to confirm this patient's underlying diagnosis is
A. AM growth hormone level
B. insulin-like growth factor 1
C. karyotype
D. PTPN11 testing
E. thyroid-stimulating hormone

American academy of pediatrics 353


American Academy of Pediatrics PREP 2015

Item 114 Preferred Response: C


Girls with Turner syndrome, such as the girl in the vignette, have characteristic physical features
coupled with complete or partial absence of the second sex chromosome. Guidelines from the
Turner Syndrome Study Group recommend considering the diagnosis of Turner syndrome in any
girl with unexplained growth failure, pubertal delay, or any constellation of the following clinical
findings (Item C114).
Item C114. Clinical Findings in Girls With Turner Syndrome
System Clinical Findings
Unexplained growth Short stature with growth velocity less than the 10th percentile for age
failure
Pubertal delay Markedly elevated levels of follicle-stimulating hormone
Cardiac anomalies Most commonly coarctation of the aorta or hypoplastic left heart
Characteristic fades Low hairline, low-set ears, small mandible
Nails Hypoplasia, hyperconvex, and uplifted
Skeletal Congenital hip dislocation, cubitus valgus, short metacarpal bones,
scoliosis
Skin Widespread nipples, shield chest, edema of the hands or feet, nuchal
folds, multiple pigmented nevi
Other Learning disabilities, social awkwardness, type II diabetes mellitus,
inflammatory bowel disease, high arched palate, chronic otitis media

The girl in this vignette has a high arched palate, short metacarpal bones, and poor linear growth.
The most common clinical feature seen in Turner syndrome is poor linear growth (short stature).
A karyotype should be considered in a girl with abnormal linear growth and otherwise normal
screening test results, even with unremarkable physical examination findings and no other
phenotypic findings of Turner syndrome.

Given the child's poor linear growth, growth hormone deficiency is also a possible diagnosis. A
random or AM growth hormone level, however, is not useful, as growth hormone levels peak
overnight during deep sleep. During the day, they are often low. That is why growth hormone
stimulation testing is sometimes performed by a pediatric endocrinologist. A better test to screen
the growth hormone axis is insulin-like growth factor 1 or insulin like growth factor binding
protein 3. Concentrations of these hormones are much more stable in the circulation and can be
measured during clinic visits. However, the child in this vignette has classic physical findings
associated with Turner syndrome, and a karyotype (chromosomal analysis) is the best test to
make this diagnosis.

PTPN11 testing for Noonan syndrome would not be a first-line test without specific findings of
this disorder present on examination. Noonan syndrome is characterized by proportionate
postnatal short stature, dysmorphic facial features, chest deformities, and congenital heart disease
(most commonly pulmonary valve stenosis and hypertrophic cardiomyopathy). Developmental

American academy of pediatrics 354


American Academy of Pediatrics PREP 2015

delay, cryptorchidism, and clotting disorders are also common. This condition is autosomal
dominant. Both males and females can have Noonan syndrome.

Hypothyroidism is a very common cause of poor growth, and all children who are growing
poorly deserve consideration of thyroid function testing. However, the child in the vignette very
clearly has physical examination signs consistent with a syndromic cause of her poor growth,
and a karyotype will confirm the diagnosis of Turner syndrome in this child.

PREP Pearls
• The most common clinical feature seen in Turner syndrome is poor linear growth (short
stature).
• Turner syndrome can present with normal physical examination findings other than poor
growth or delayed puberty.
• Girls with unexplained short stature should have karyotyping performed.

American Board of Pediatrics Content Specification(s)


• Recognize the clinical features associated with primary gonadal dysgenesis (Turner
syndrome) in patients of various ages

Suggested Reading
• Bondy CA, Turner Syndrome Study Group. Care of girls and women with Turner
syndrome: a guideline of the Turner Syndrome Study Group. J Clin Endocrinal Metab.
2007;92(1)710-25. doi:10.1210/jc.2006-1374.
• Pinsker JE. Clinical review: Turner syndrome: updating the paradigm of clinical care. J
Clin Endocrinal Metab. 2012;97(6):E994-E1003. doi:10.1210/ jc.2012-1245.

American academy of pediatrics 355


American Academy of Pediatrics PREP 2015

Item 115
An 8-week-old infant who was the product of an uneventful pregnancy and delivery is taken into
your office for evaluation of a rash. Her mother reports that she had no infections during the
pregnancy. The infant spent 5 days in the nursery for phototherapy. Since discharge, the infant
has done well, but over the past day she has developed blisters in her diaper area. She has had no
fever and continues to eat well. Physical examination shows a well-appearing infant who has
several 1-cm bullae over the lower abdomen, buttocks, and round crusted erosions with a rim of
scale. A photo of the lesions is shown (Item Q115).

Of the following, the MOST appropriate management for this infant is to


A. admit her to the hospital for acyclovir therapy after lumbar puncture
B. initiate oral clindamycin after obtaining a culture of the fluid
C. prescribe topical nystatin to be applied to the bullae
D. reassure the parents that this will resolve without treatment
E. refer her to a dermatologist for a skin biopsy

American academy of pediatrics 356


American Academy of Pediatrics PREP 2015

Item 115 Preferred Response: B


The infant in the vignette has characteristic findings of bulbous impetigo and therefore should be
treated with an anti-staphylococcal agent such as clindamycin. Bullous impetigo is the result of a
superficial infection by exfoliative toxin-producing Staphylococcus aureus and is not usually
associated with systemic symptoms; therefore, oral antibiotics are sufficient in most cases. In
very young patients (younger than 1 month of age), some experts would recommend parenteral
clindamycin because of the concern for the immunocompetency of that age group. The usual
organism is from phage group II and is most often type 71, which produces exfoliative toxin A or
exfoliative toxin B. These toxins are responsible for the characteristic findings. Initially, bullous
impetigo appears as vesicles, which rapidly progress to flaccid bullae that may contain clear
yellow fluid. The fluid may progress to a cloudy appearance. The Nikolsky sign (top layer of
skin coming off when lightly rubbed) is not present. The bullae rupture easily, revealing
superficial, moist, red erosions surrounded by a collarette of scale. Eventually a light brown crust
may cover the lesions, which ultimately heal without scars. The lesions are often present in the
diaper area and over the buttocks, as is seen with the patient in the vignette.

Bullous impetigo most commonly occurs in children younger than 5 years of age and very
frequently affects newborns and young infants. It may appear as soon as a few days after birth,
and nursery outbreaks have been documented. Treatment of bullous impetigo depends on the
prevalence of methicillin-resistant Staphylococcus aureus (MRSA) in the community. Fluid from
a bullous lesion should be cultured before treatment is started. Some experts recommend
treatment with topical anti-staphylococcal agents such as mupirocin when there are few or
localized lesions; topical bacitracin would not be considered sufficient treatment. For non-MRSA
disease, amoxicillin-clavulanate or dicloxacillin may be used with erythromycin or clindamycin
for penicillin-allergic patients. For MRSA-positive organisms, clindamycin or trimethoprim-
sulfamethoxazole may be used; trimethoprim-sulfamethoxazole is contraindicated for infants
younger than 2 months of age. Hospitalization and intravenous vancomycin should be reserved
for patients with systemic disease, such as that which occurs with severe staphylococcal scalded
skin syndrome. Severe staphylococcal scalded skin syndrome is differentiated from bullous
impetigo by the presence of systemic symptoms, widespread erythema, perioral lesions, and a
positive Nikolsky sign.

Other vesicular conditions that may occur during the neonatal period include transient neonatal
pustular melanosis, epidermolysis bullosa, mastocytomas, and herpetic infections. Transient
neonatal pustular melanosis is often present at birth, and appears as small (2-3 mm) pustules that
evolve into hyperpigmented macules that resolve in 3 to 4 months. For patients with this
condition, reassurance to the family and observation are appropriate. Epidermolysis bullosa is a
group of genetic disorders, some of which can present at birth or soon after. These are
differentiated from bulbous impetigo by a more widespread appearance, mucosal and nail
involvement, and recurrence of lesions. Skin biopsy is often indicated to make the diagnosis. A
mastocytoma is distinguished clinically from bullous impetigo by the presence of the
characteristic peau d'orange skin appearance, Darier sign (blistering or urticating at the site of
trauma), and recurrence. Neonatal mucocutaneous herpes simplex infection can present with
vesicles, but they differ from impetigo with smaller size, tendency to cluster, associated mucous
membrane involvement and, when present, systemic symptoms. Neonates for whom herpes
simplex infection is considered should be admitted to the hospital for a sepsis evaluation and
American academy of pediatrics 357
American Academy of Pediatrics PREP 2015

parenteral antiviral therapy. The presence of bullae is not characteristic of candida diaper
dermatitis and therefore antifungal agents are not indicated.

PREP Pearls
• Bullous impetigo is caused by exfoliative toxin-producing Staphylococcus aureus and
occurs commonly during the neonatal period and early infancy.
• Bullous impetigo is characterized by bullae that rupture leaving a red, moist erosion with
a collarette of scale; lesions heal without scarring.
• Bullous impetigo is generally not associated with systemic symptoms and usually can be
treated with oral or topical antistaphylococcal antibiotics.
• Nursery outbreaks of bullous impetigo require contact investigation and isolation to
prevent spread to other newborns.

American Board of Pediatrics Content Specification(s)


• Recognize the clinical findings associated with neonatal impetigo

Suggested Reading
• Hoath SB, Narendrau V. The skin. In: Martin RI, Fanaroff AA, Walsh MC, eds. Fanaroff
and Martin's Neonatal-Perinatal Medicine. 9th ed. St Louis, MO: Mosby Elsevier;
2011:1711-1716.
• Lawrence HS, Nopper A). Superficial bacterial skin infections and cellulitis. In: Long 55,
Pickering 1.-K, Prober CG, eds. Principles and Practice of Pediatric Infectious Disease.
4th ed. Edinburgh, Scotland: Saunders Elsevier; 2012:427-429.
• Pasternack MS, Swartz MN. Cellulitis, necrotizine fasciitis, and subcutaneous tissue
infections. In: Mandell GL, Bennett JE, Dolin R, eds. Mandell, Douglas, and Bennett's
Principles and Practice of Infectious Diseases. 7th ed. Philadelphia, PA; Churchill
Livingstone Elsevier; 2009:1289-1312.
• Piechowicz L, Garbacz K, Budzyriska A, Dabrowska-Szponar M_ Outbreak of buitous
impetigo caused by Staphylococcus aureus strains of phage type 3C/71 in a maternity
ward linked to nasal carriage of a healthcare worker. Eur Derrnatol. 2012;22(2): 252-255.
doi:10.1684iejd.2012.1671.

American academy of pediatrics 358


American Academy of Pediatrics PREP 2015

Item 116
A 16-year-old, sexually active adolescent girl presents to your office with lower abdominal pain
that has lasted for 2 days. She has had some dysuria and loose bowel movements over the last
few days. Her current menstrual period came after an interval of 5 weeks, and she finished her
flow 3 days ago. She notes that her menstrual flow was heavier and more painful than usual. She
is afebrile and has normal vital signs. On physical examination, she exhibits diffuse lower
abdominal tenderness with no rebound or guarding. Bimanual examination elicits pain on
movement of her cervix and palpation of her adnexa, with no palpable masses. A pregnancy test
result is negative and complete blood cell count is normal.

Of the following, the MOST likely cause of her pain is


A. endometriosis
B. pelvic inflammatory disease
C. primary dysmenorrhea
D. ruptured ovarian cyst
E. urinary tract infection

American academy of pediatrics 359


American Academy of Pediatrics PREP 2015

Item 116 Preferred Response: B


Lower abdominal or pelvic pain in an adolescent girl may be acute or chronic and have a number
of possible causes. The history and physical examination should help narrow the differential
diagnosis. Intermenstrual intervals can range from 3 to 6 weeks and therefore the 5-week interval
reported by the teenager in the vignette is within normal limits. Any pelvic symptoms should
prompt a pregnancy test and the results are reported as negative for the adolescent girl in the
vignette. She has had acute pain over 2 days and notes that her last menstrual period was heavier
and more painful than usual. This should raise a concern that this is not just primary
dysmenorrhea (painful menstruation). For a sexually active girl with a negative pregnancy test,
the most important consideration is a sexually transmitted infection. Her bimanual examination
results confirm the diagnosis of pelvic inflammatory disease (PIP), meeting the US Centers for
Disease Control and Prevention minimal criteria (abdominal tenderness with cervical motion and
adnexal tenderness). PID often presents with a number of less specific symptoms including
urinary and gastrointestinal systems.

A urinary tract infection would not present with menstrual changes. Ovarian cyst rupture or
torsion is usually abrupt in onset, and sharp and severe in intensity. Endometriosis is part of the
differential diagnosis for chronic pelvic pain and a definitive diagnosis requires a laparoscopic
evaluation. It is an important diagnosis to consider in adolescents with severe dysmenorrhea that
does not respond to over-the-counter pain medications.

PREP Pearls
• Understanding variations in a normal menstrual cycle is helpful when educating and
evaluating symptoms in an adolescent.
• Documenting a last menstrual period on each encounter and performing a pregnancy test
if late with menses or in the presence of any pelvic symptoms is very important.
• A change in menstrual flow and increased degree of pain may indicate the presence of a
sexually transmitted infection.

American Board of Pediatrics Content Specification(s)


• Recognize the gynecologic etiologies of acute and chronic abdominal pain
• Recognize normal variations in the menstrual cycle in adolescent girls

Suggested Reading
• Kola B, Swati-Dave 5, Derlet M, Ejac 5, Rosenberg II, Tetrokalashvili M. Index of
suspicion case 3: abdominal pain, vomiting, and weakness in an obese adolescent, case 3:
cyclical abdominal pain and urinary symptoms in a teenage girl. Pediatr Rev.
2010;31(10):433-438. don10.1542/pir.31-10-433.
• Laufer MR, Goldstein DP. Gynecologic pain: dysmenorrhea, acute and chronic pelvic
pain, endometriosis and premenstrual syndrome. In: Em ens SI, Laufer MR, Goldstein
DP, eds. Pediatric and Adolescent Gynecology. Philadelphia, PA: Lippincott Williams &
Wilkins; 2005:417-476.
• Trent M. Pelvic inflammatory disease. Pediatr Rev. 2013:34(4)163-172_
doi:10.1542/pir.34-4-163.

American academy of pediatrics 360


American Academy of Pediatrics PREP 2015

Item 117
The parents of a 16-year-old male adolescent seek your help in dealing with the coach of the
varsity soccer team. The adolescent has been having shortness of breath and noisy breathing that
begins about 15 min into his game and requires him to be pulled out. The coach has berated him
for being unmotivated, not conditioned enough, and has been less inclined to put him in to play.
The parents are hoping for a sports scholarship in college and state that he practices intensely and
is very committed. He is also an excellent swimmer and typically does not have these symptoms
while swimming. His physical examination reveals a muscular and well-toned young man with
clear lungs on auscultation, and an otherwise unremarkable examination. You make a tentative
diagnosis of exercise-induced bronchoconstriction.

Of the following, your diagnosis would be MOST strongly supported by the history that
A. breathing exercises do not prevent episodes
B. he is undergoing counseling for anxiety disorder
C. prior warming up for 15 min prevents episodes
D. symptoms are accompanied by inspiratory stridor
E. symptoms occur at onset of strenuous exercise

American academy of pediatrics 361


American Academy of Pediatrics PREP 2015

Item 117 Preferred Response: C


The adolescent in the vignette has exercise-induced bronchospasm (EIB) that is most strongly
supported by the historical observation that prior warming up for 15 min prevents episodes.
Exercise-induced bronchoconstriction is defined as the transient narrowing of the lower airways
that occurs after strenuous exercise. The symptoms of EIB typically occur 5 to 10 min after onset
of strenuous physical activity and include symptoms of cough, wheeze, chest tightness, shortness
of breath, or excessive mucous production. These symptoms occur even in patients who are in
good physical condition and occur in response to heating and humidifying large volumes of
inspired air during a short period. Symptom severity is linked to the water content of the inspired
air or the level of ventilation sustained during exercise, which explains why the patient is able to
swim without difficulty. Exercise-induced bronchospasm may be observed in patients who have
or do not have chronic asthma. The EIB practice parameter recommends that the term Exercise-
induced Asthma (EIA) no longer be used "because exercise does not induce asthma but rather is
a trigger of bronchoconstriction." The diagnosis of EIB usually requires demonstration of a
decrease in Forced Expiratory Volume in 1 second (FEV1) of 10% to 15% of the pre-exercise
value after exercise. Exercise-induced bronchospasm is a manifestation of bronchial hyper-
responsiveness and is often the first sign of asthma, although not all individuals with EIB
develop asthma. β2-agonist by inhalation is the most widely used treatment. It is recommended
immediately before exercise to prevent EIB. Pre-exercise warm-up done at 60% to 80%
maximum heart rate provides partial attenuation of EIB secondary to the occurrence of a
refractory period lasting from 1 to 3 hours.

Symptoms accompanied by inspiratory stridor, occurring at the onset or anytime during the
duration of physical activity, and responding to breathing (relaxation) techniques, suggest the
possibility of exercise-induced laryngeal dysfunction (EILD). Exercise-induced laryngeal
dysfunction is a condition that primarily includes vocal cord dysfunction (VCD) and other glottic
abnormalities that can mimic EIB. Inspiratory stridor is a differentiating hallmark sign with
EILD. The subjective perception of difficulty taking in air (rather than breathing out air) also
suggests upper airway tightness. Lack of response to bronchodilators such as albuterol is another
differentiating feature suggesting EILD. Inspiratory flattening or truncation of the flow-volume
loop on spirometry and a decrease in FEV1 with no alteration in the FEV1/FVC ratio (when
symptomatic) are consistent with, but not diagnostic of, vocal cord dysfunction. The diagnosis
can be confirmed by demonstrating paradoxical vocal cord movement while performing direct
laryngoscopic visualization during an acute attack

Vocal cord dysfunction has been described more commonly in individuals who are high
achievers and can be triggered by stress and emotional conditions. The literature also reveals a
high incidence of VCD in persons with psychiatric conditions; therefore, the history of
undergoing counseling for an anxiety disorder might raise suspicion of these symptoms being
attributable to VCD rather than EIB. Concomitant VCD has been reported in up to 40% of
individuals with asthma and EIB, rendering the diagnosis and management a little more
challenging. There is no pharmacological treatment for VCD (aside from treating co-existing
asthma or EIB). Breathing exercises designed to promote abdominal or diaphragmatic breathing
and to eliminate abnormal vocal movement are helpful in the management of this condition.

American academy of pediatrics 362


American Academy of Pediatrics PREP 2015

PREP Pearls
• Exercise-induced bronchospasm (EIS) should be suspected in any child who exhibits
symptoms of shortness of breath, chest tightness and wheeze within 5 to 15 min of onset
of strenuous physical activity.
• The symptoms of EIB can be mitigated by pre-exercise warming-up at maximal heart rate
of 60% to 80% or pretreatment with inhaled short acting β2-agonist medication.
• Clinical features such as history of "difficulty taking in air," inspiratory stridor, and lack
of response to short acting β2-agonist inhaler should raise the suspicion of exercise-
induced laryngeal dysfunction or vocal cord dysfunction.

American Board of Pediatrics Content Specification(s)


• Recognize the clinical features associated with exercise-induced asthma

Suggested Reading
• Maturo S, Hill C, Bunting G, et al. Pediatric paradoxical vocal-fold motion: presentation
and natural history. Pediatrics. 2012;128(6):e1443- 1449. doi:10.1542/peds.2011-1003.
• Weiler JM, Anderson SD, Randolph C, et al. Pathogenesis, prevalence, diagnosis, and
management of exercise-induced bronchoconstriction: a practice parameter. Ann Allergy
Asthma Immunal. 2010;105(6 Suppl):S1-S47. doi:10.1016/j.anai.2010.09.021.

American academy of pediatrics 363


American Academy of Pediatrics PREP 2015

Item 118
A 5-year-old girl was admitted from the emergency department because of fever and lethargy.
She was recently diagnosed with acute lymphoblastic leukemia and is in the middle of her
induction cycle of chemotherapy. She was tachycardic and hypotensive while in the emergency
department and received a total of 100 mL/kg of 0.9% saline in boluses. She also was started on
a dopamine drip of 5 µg/kg Per min. Blood cultures were drawn, and she was started on empiric
antimicrobial therapy, including vancomycin and cefepime. Due to respiratory distress and
hypoxemia, she required intubation and mechanical ventilation. Within 24 hours, her blood
cultures from both peripheral venipuncture and her indwelling central venous catheter have
grown gram-negative bacilli. She is now on relatively high ventilator settings, and her arterial
blood gas has a pH of 7.30, PCO2 of 50 mm Hg, and PO2 of 80 mm Hg on FiO2 of 0.60.

Vital signs show a temperature of 38.5°C, heart rate of 130 beats/min, respiratory rate of 30
breaths/min, blood pressure of 90/30 mm Hg, and SpO2 of 94%. Physical examination shows an
intubated and sedated girl that awakens to stimuli, with no focal neurologic findings.
Cardiovascular examination shows tachycardia with regular rhythm. She has warm extremities,
with capillary refill less than 1 second. Lungs have scattered crackles and decreased air
movement bilaterally. Abdomen is soft, nontender, and nondistended with no palpable spleen or
liver. A chest radiograph is shown in Item Q118.

Of the following, the MOST likely cause of her respiratory failure is


A. acute respiratory distress syndrome
B. congestive heart failure
C. pulmonary embolism
D. pulmonary hemorrhage
E. pulmonary hypertension

American academy of pediatrics 364


American Academy of Pediatrics PREP 2015

Item 118 Preferred Response: A


The child in this vignette, who has bilateral infiltrates on chest radiography, a PaO2/FiO2 ratio of
125, and a noncardiogenic cause of respiratory failure in the setting of sepsis, most likely has
acute respiratory distress syndrome (ARDS). The pathogenesis of ARDS in this patient is
increased pulmonary capillary permeability because of sepsis, septic shock, or systemic
inflammatory response syndrome.

Acute respiratory distress and failure caused by ARDS occurs in both adults and children, and is
a serious cause of morbidity and mortality. According to the 1994 American-European
Consensus Conference on ARDS, diagnostic criteria include respiratory failure, a ratio of arterial
oxygen tension to fraction of inspired oxygen (PaO2/FiO2) of less than 200, bilateral pulmonary
infiltrates on chest radiographs, and a noncardiac cause of pulmonary edema. The term acute
lung injury (ALI) indicates patients with a PaO2/FiO2 ratio between 200 and 300.

An important event in the pathogenesis of ARDS is the breakdown of the alveolar-pulmonary


capillary barrier, which leads to the leakage of proteinaceous fluid into the airspaces. This causes
decreased lung compliance, alveolar collapse, and disordered gas exchange marked by
ventilation-perfusion mismatch. Patients often breathe rapidly and shallowly and become
hypoxic because of decreased compliance and intrapulmonary shunting. Surfactant production
and function is also impaired by alveolar type II cell injury, which leads to further alveolar
collapse. The repetitive opening and closing of lung units in the face of increased surface tension
exacerbates the inflammatory cascade, leading to the secretion of proinflammatory cytokines,
such as tumor necrosis factor a and interleukins, which exacerbates the cycle by increasing
capillary permeability.

Acute respiratory distress syndrome can be caused by direct lung injury from pneumonia,
ventilator-induced lung injury, chest trauma, aspiration pneumonitis, acute chest syndrome in
sickle cell disease, or indirectly from systemic causes such as sepsis, transfusion-related lung
injury, pancreatitis, trauma, or systemic inflammation from other numerous causes.
Landmark investigations performed by the ARDS Research Network have led to the widespread
acceptance of the "open lung" strategy of ventilator management in ARDS. The aim is to protect
the lungs from further injury by maintaining alveolar recruitment with high positive end-
expiratory pressure and by limiting inflation pressures and FiO2. The clinician targets lower tidal
volumes of 4 to 6 mL/kg and accepts higher level of partial pressure of carbon dioxide (PCO2),
which is a strategy known as "permissive hypercapnia."

Although congestive heart failure is common in children with leukemia, it is unlikely in this
patient because she has received 100 mL/kg in fluid resuscitation and has no palpable spleen or
liver. Pulmonary embolism is rare in this age group and is not associated with sepsis, which is
the acute condition of this patient. Pulmonary hemorrhage can occur in patients who have
insufficient clotting from bone marrow suppression, but bleeding severe enough to cause
respiratory failure usually coincides with hemoptysis or the presence of blood in the endotracheal
tube. Pulmonary hypertension can cause hypoxia, but is not associated with sepsis, nor is it a
common cause of respiratory failure in children with leukemia.

American academy of pediatrics 365


American Academy of Pediatrics PREP 2015

Acute respiratory distress syndrome is a condition of respiratory failure in adults and children
marked by decreased lung compliance and hypoxia from ventilation-perfusion mismatch and
intrapulmonary shunting. It should be treated with the evidence-based guidelines of lung
recruitment with positive end-expiratory pressure, and limitation of tidal volume, inflation
pressures, and FiO2.

PREP Pearls
• Acute respiratory distress syndrome can be caused by direct lung injury or indirectly
from nonpulmonary inflammatory conditions.
• Acute respiratory distress syndrome is associated with pulmonary alveolar-capillary
damage, decreased surfactant production and function, and proinflammatory cytokines.
• Acute respiratory distress syndrome should be treated to protect the lungs, minimizing
inflation pressures and oxygen toxicity, and accepting higher PCO2.

American Board of Pediatrics Content Specification(s)


• Recognize the clinical features of acute respiratory distress syndrome, including
associated sequelae

Suggested Reading
• Carlo WA, Ambalavanan N. Conventional ventilation: traditional and new strategies.
Pediatr Rev. 1999;20(12):e117-e126. doi:10.1542/pir.20-12-e117. Dauger S, Durand P,
Javouey E, Mercier JC. Acute respiratory distress syndrome in children. In: Fuhrman BP,
Zimmerman JJ. eds. Pediatric Critical Care. 4th ed. Philadelphia, PA: Saunders Elsevier:
2011:212-218.
• Fiser DH. Adult respiratory distress syndrome. Pediatr Rev. 1993;14(5):163166.
doi:10.1542/pir.14-5-163.
• Nitu ME, Eisen H. Respiratory failure. Pediatr Rev. 2009;30(12):470-478.
doi:10.1542/pir.32-6-240.

American academy of pediatrics 366


American Academy of Pediatrics PREP 2015

Item 119
A 7-year-old boy is admitted for evaluation and treatment of swelling and redness around his left
eye. His illness began with a 9-day history of fever to 38.9°C, cough, congestion, and runny
nose. On the evening prior to admission, he complained of mild pain in his left eye, but was able
to watch television without difficulty seeing the screen. On the morning of admission, he woke
up with marked swelling, redness, and increased pain in his left eye. He is now unable to open
the eye.

The boy's temperature is 38.7°C, pulse rate is 110 beats/ min, blood pressure is 107/75 mm Hg,
and respiratory rate is 22 breaths/min. Examination shows swelling and erythema surrounding
the left eye. He is unable to open the eye and complains of pain on attempts to move the eye.
You are unable to assess extraocular movements because of the swelling. His nose is congested,
and his pharynx is unremarkable. His neck is supple and the remainder of the examination
findings are unremarkable.

Of the following, the BEST initial treatment for this patient is


A. cephalexin and clindamycin
B. clindamycin and gentamicin
C. vancomycin and ceftriaxone
D. vancomycin and gentamicin
E. vancomycin and rifampin

American academy of pediatrics 367


American Academy of Pediatrics PREP 2015

Item 119 Preferred Response: C


The redness, swelling, and pain in this boy's eye, including pain on movement as described in
this vignette, raise concern for orbital cellulitis. Additional findings in orbital cellulitis may
include proptosis, chemosis, ophthalmoplegia, and blurred vision. In view of the inability to open
the boy's eye, computed tomography examination with contrast would be indicated to assess the
extent of the infection.

Orbital cellulitis, especially in this setting with a 9-day prodrome of upper respiratory tract
infection, arises as an extension of ethmoidal sinusitis though the lamina papyracea into the
muscle and fat of the orbit. Thus, antibiotic management should include agents active against the
common pathogens of sinusitis (eg, Streptococcus pneumonia, Haemophilus influenza,
Moraxella catarrhalis), Streptococcus pyogenes, and Staphylococcus aureus (including
methicillin-resistant S aureus [MRSA]). Given the deep-seated nature of the infection, hospital
admission with parenteral antibiotic therapy is generally indicated. Of the regimens listed,
vancomycin and ceftriaxone provide the best coverage for this array of bacteria. The other
regimens do not provide adequate coverage for H influenza or M catarrhalis, and cephalexin and
gentamicin may not penetrate well into the orbital space. Clindamycin and ceftriaxone would be
another appropriate regimen in this setting. If MRSA is not a concern in a given area, ampicillin-
sulbactam would be another appropriate therapeutic regimen.

Potential complications of orbital cellulitis include local abscess formation, optic nerve
compression with visual loss, and intracranial extension that can lead to cavernous sinus venous
thrombosis, subdural empyema, intracranial abscess, and meningitis. Given the potential severity
of this infection, management may include consultations with specialists in pediatric
ophthalmology, pediatric otolaryngology, and infectious disease. In addition to antibiotic
therapy, surgery may be indicated in the face of these complications or failure to respond to
antibiotic therapy over 24 to 48 hours.

Orbital cellulitis must be differentiated from periorbital (or preseptal cellulitis). Clinically,
periorbital cellulitis, which is an infection limited to the soft tissues anterior to the orbital
septum, may also present with fever, unilateral eye swelling with erythema, and warmth and
tenderness over the affected eye. However, with the infection limited to the preseptal region
proptosis, pain with eye movement and vision impairment are not observed and intracranial
complications do not occur. If there is uncertainty, computed tomography with contrast can
differentiate the 2 entities. Outpatient management with oral antibiotic therapy and close follow-
up may be considered in a nontoxic child with periorbital cellulitis.

PREP Pearls
• Periorbital cellulitis is a localized infection anterior to the orbital septum that may present
with fever, swelling with erythema and tenderness over the affected eye, but without the
proptosis and pain with eye movements seen in orbital infection.
• Orbital cellulitis is characterized by redness, swelling and pain over the infected eye with
proptosis, chemosis and ophthalmoplegia. Potential complications of orbital cellulitis
include local abscess formation, visual impairment secondary to optic nerve compression,
and extension of the infection into the central nervous system.

American academy of pediatrics 368


American Academy of Pediatrics PREP 2015

• Orbital cellulitis in children typically arises as a complication of ethmoid sinusitis


extending through the lamina papyracea of the lateral ethmoid wall into the orbital space.
• Computed tomography with contrast defines the extent of orbital cellulitis and can
differentiate orbital from periorbital (preseptal) disease.
• Antibiotic management should include agents active against the common pathogens of
sinusitis.

American Board of Pediatrics Content Specification(s)


• Understand the natural history of orbital and periorbital (preseptal) cellulitis
• Recognize and differentiate the clinical and radiologic findings associated with orbital
cellulitis and periorbital (preseptal) cellulitis
• Recognize pathogens commonly associated with orbital cellulitis
• Plan the appropriate diagnostic evaluation of orbital cellulitis
• Plan the appropriate management of orbital cellulitis, including associated complications

Suggested Reading
• Hauser A, Fogarasi S. Periorbital and orbital cellulitis. Pediatr Rev. 2010; 31(6):242-249.
doi:10.1542/pir.31 -6 -242.
• Wald ER, Applegate KE, Bordley C, et al. Clinical guideline for the diagnosis and
management of acute bacterial sinusitis in children aged 1 to 18 years. Pediatrics.
2013;132(1):e262-e280. doi;10.1542/peds.2013 -1071.

American academy of pediatrics 369


American Academy of Pediatrics PREP 2015

Item 120
In your office, you are evaluating a 12-year-old boy who has had blood in his urine for the past 1
day. He describes the urine as bright red. He has no complaints of fever, frequency, painful
urination, or flank pain. His vital signs are normal for his age. His physical examination findings
are normal. His urinalysis demonstrates a specific gravity of 1.015, pH of 6.0, 4+ blood, 4+
leukocyte esterase, and no protein or nitrites. His urine microscopy shows more than 100 red
blood cells per high-power field, 10 to 50 white blood cells per high-power field, and no crystals
or bacteria. You discuss the evaluation of gross hematuria with the medical student assigned to
your practice.

Of the following, the MOST accurate statement regarding the evaluation of this child is
A. cystoscopy is usually indicated in making the diagnosis
B. depth of color in gross hematuria is indicative of the degree of blood loss
C. his gross hematuria is associated with an identifiable underlying etiology
D. if the urine sediment is clear and supernatant red, the etiology of hematuria is blood
E. if the urine sediment is red and supernatant clear, the etiology of hematuria is not blood

American academy of pediatrics 370


American Academy of Pediatrics PREP 2015

Item 120 Preferred Response: C


The differential diagnosis for red urine is quite extensive (Item C120, page 104), but the patient's
history and urinalysis are helpful in differentiating the possible causes. Hematuria described as
bright red, as for the child in this vignette is usually indicative of lower urinary tract bleeding,
whereas glomerular hematuria (as in nephritis) is usually described as cola, tea or brown colored.
Presence of blood clots, with or without dysuria, is indicative of urinary tract bleeding. An
underlying cause is more frequently identified in patients with gross hematuria compared with
those presenting with asymptomatic microscopic hematuria.

A detailed history, physical examination, and urinalysis usually provide clues to the underlying
cause of gross hematuria. In patients with symptomatic hematuria, the presenting history can
guide the choice of investigations for identifying the underlying cause. In patients with history of
trauma, a computed tomography (CT) scan of the abdomen and pelvis is indicated. Patients with
nephrolithiasis present with varying degrees of flank pain/discomfort with the severe colicky
pain radiating to the flank more commonly seen with large and/or obstructing calculi. Such
patients may have dysuria with associated infection or hypercalciuria. Renal ultrasonography is
the preferred initial imaging modality for patients with suspected nephrolithiasis. Spiral CT is the
most sensitive test for renal stones, but the risk of radiation exposure in young
children/adolescents should be weighed against the benefit of detecting small stones missed on
renal ultrasonography. Patients with a history of perineal and meatal irritation may present with
gross hematuria and can be given supportive care and reassurance. Patients with
glomerulonephritis may present with edema or history suggestive of an underlying autoimmune
disorder (joint pain and swelling, fatigue, malaise, oral ulcers, or skin rash).

Children presenting with asymptomatic gross hematuria would usually require a detailed
evaluation for the underlying cause. The workup in such cases includes urinalysis (to
differentiate between red blood cells (RBCs] and heme pigments), urine calcium-creatinine ratio
(hypercalciuria), serum creatinine and C3 (nephritis), renal ultrasonography (congenital
anomalies or bladder mass), and hemoglobin electrophoresis (sickle cell disease). The patient in
the vignette has asymptomatic bright red hematuria of 1 day's duration, which suggests lower
urinary tract bleeding. The likely cause for the patient's symptoms could be infection (cystitis),
hypercalciuria, congenital renal anomalies, or bladder mass.

Pigmenturia (hemoglobinuria and myoglobinuria) can result in hematuria with a positive result
on dipstick test for blood, but both conditions lack the presence of RBCs on microscopy.
Therefore, the urine in pigmenturias has clear sediment (lack of RBC) whereas the supernatant is
red because of heme or myoglobin pigment. In patients with glomerular or lower urinary tract
bleeding, the sediment is red (presence of RBC in urine) and the supernatant is clear.
Cystoscopy is rarely indicated for children presenting with hematuria (gross or microscopic). It is
indicated in the case of a bladder mass or urethral trauma.

American academy of pediatrics 371


American Academy of Pediatrics PREP 2015

Item 0120. Differential Diagnoses of Gross Hematuria


Appearance Source of Clinical Findings Differential Urine analysis
Bleeding Diagnoses /findings
Cola, iced tea, Glomerular or Usually painless hematuria, Glomerulonephritis Positive hematuria
or fruit punch upper urinary Symptoms related to finding on urine
colored tract underlying etiology dipstick test; RBCs
and RBC Casts on
microscopy
1. Hemoglobinuria: Positive hematuria
RBC hemolysis; finding on urine
2. Myoglobinuria: dipstick test, no RBCs
Rhabdomyolysis on microscopy
Bright Red Lower urinary All have dipstick positive hematuria and RBC on microscopy
tract Flank pain radiating to groin Nephrolithiasis Gravel/ dust/small
stone in urine
Dysuria; pain on micturition Urinary tract Clots in urine, Urine
Fever may or may not be infection dipstick also positive
present for nitrites, WBC's,
and urine microscopy
for WBC's, and
bacteria
History of renal trauma Urinary tract trauma
No significant history Hypercalciuria* Elevated urine
Renal cyst or mass Calcium/creatinine
Bladder cyst or mass ratio
Sickle cell disease
*Hypercalciuria may present with pain or dysuria; BBC, red blood cell

PREP Pearls
• An underlying cause is more frequently identified in patients with gross hematuria
compared with those presenting with asymptomatic microscopic hematuria.
• A detailed history, physical examination, and urinalysis usually provide clues to the
underlying cause of gross hematuria.
• Hematuria described as bright red is usually indicative of lower urinary tract bleeding,
whereas glomerular hematuria (as in nephritis) is usually described as cola, tea or brown
colored.
• Pigmenturia (hemoglobinuria and myoglobinuria) results in hematuria with a positive
finding on dipstick test for blood and absence of ABCs on microscopy.
• Cystoscopy is rarely indicated for children presenting with hematuria.

American Board of Pediatrics Content Specification(s)


• Plan the appropriate clinical evaluation of gross hematuria

Suggested Reading
• Massengill SF. Hematuria. Pediatr Rev. 2008;29:342-348. doi:10.1542/ pir.29-10-342.
• Patel HP, Bissler JJ. Hematuria in children. Pediatr Clin North Am.
• 2001;48:1519. doi: 10.1016/SO031-3955%2805%2970389-8.
• Reidy KJ, Rio MD. In: Hematuria. American Academy of Pediatrics Textbook of
Pediatric Care. 19th ed. Mclnerny TK, Adam HM, Campbell DE, Kamat DP, Kelleher

American academy of pediatrics 372


American Academy of Pediatrics PREP 2015

KJ, eds. Elk Grove Village, IL: American Academy of Pediatrics; 2009;chap 188:1566-
1570.

American academy of pediatrics 373


American Academy of Pediatrics PREP 2015

Item 121
A 4 year-old girl has a 12-month history of intermittent, poorly localized abdominal pain. She
now presents with a recurrence of crampy abdominal pain. She has vomited after each meal for
the past 24 hours. She has no history of fever or diarrhea. The child woke up frequently during
the previous night because of the pain. Her parents report that she seems more comfortable lying
on her side in a knee-chest position. The family history is negative for gastrointestinal disease.
The child's grandfather died of a heart attack at 55 years of age, and her father, age 40,
underwent a coronary artery bypass procedure last year. Her height is 97 cm (25th percentile),
weight is 14 Kg (10th percentile), temperature is 37.2°C, pulse rate is 130 beats/min, respiratory
rate is 25 breaths/min, and blood pressure is 90/60 mm Hg. The girl is difficult to examine
because of her abdominal pain. Her lungs are clear to auscultation and her cardiac examination is
normal. She complains of severe pain wherever her abdomen is palpated. Initial laboratory data
include the following:

• White blood cell count, 10.4 x 103/µL (10.4 x 109/L)


• Hemoglobin, 12.5 g/dL (125 g/L)
• Sodium, 135 mEq/L (135 mmol/L)
• Chloride, 100 mEq/L (100 mmol/L)
• Potassium, 4.5 mEq/L (4.5 mmol/L)
• Amylase, 240 U/L (normal <80 U/L)
• Lipase, 700 U/L (normal <70 U/L)
• Aspartate aminotransferase, 60 U/L (reference range, 20-50 U/L)
• Alanine aminotransferase, 70 U/L (reference range, 20-60 U/L)

Of the following, the test MOST likely to demonstrate the underlying cause of this girl's illness is
A. abdominal ultrasonography
B. magnetic resonance cholangiopancreatography
C. serum calcium
D. serum lipids
E. sweat chloride

American academy of pediatrics 374


American Academy of Pediatrics PREP 2015

Item 121 Preferred Response: D


The presentation of a child with symptoms of severe abdominal pain, vomiting, and antalgic
knee-chest position (assumed to relieve pressure on the pancreas) in association with marked
pancreatic enzyme elevations are pathognomonic signs of acute pancreatitis. Her 12-month
history of apparently similar, yet less severe episodes, suggests a chronic, relapsing problem.
Recurrent pancreatitis is the second most common pancreatic disorder, after cystic fibrosis, in the
pediatric age group. The strong family history of premature atherosclerotic coronary artery
disease indicates that a familial dyslipidemia may be a potential cause for her illness.
Accordingly, the clinician should obtain a serum lipid profile.

Symptoms in the child with acute pancreatitis include abdominal pain (90% of patients),
vomiting (> 60%), abdominal tenderness (80%), and abdominal distention (20%). These
symptoms often increase in severity with meals. Fever, tachycardia, hypotension, jaundice,
abdominal guarding, and rebound tenderness occur less frequently and herald a more severe
presentation. When a diagnosis of pancreatitis is suspected, serum amylase and lipase levels
should be measured and are always increased. However, the magnitude of the enzyme elevation
does not always correlate with the severity of pancreatic injury. Abdominal ultrasonography is
not required to establish a diagnosis of pancreatitis, though this test may be helpful in assessing
the degree of pancreatic edema as well as the presence of peripancreatic fluid. Magnetic
resonance cholangiopancreatography and the emerging technique of endoscopic ultrasonography
are useful studies in evaluating possible gallstone pancreatitis, but the absence of other signs of
cholestasis (eg, jaundice) makes this diagnosis unlikely. Hypercalcemia, particularly in the
setting of hyperparathyroidism, is a known cause of acute and recurrent pancreatitis, but the
family history and presentation of the girl in the vignette strongly suggest a lipid metabolic
disorder.

Acute, sporadic pancreatitis is more common than recurrent, chronic pancreatitis during
childhood and adolescence. Most cases of acute, sporadic pancreatitis are caused by blunt
abdominal trauma or are classified as "idiopathic:' Causes of chronic, relapsing pancreatitis
include infections, autoimmune disorders, genetic conditions, and drugs.
Pancreatitis is a common complication of lipoprotein metabolic disorders that are associated with
hypertriglyceridemia. Of the heritable hyperlipidemias, familial lipoprotein lipase (LPL)
deficiency and type V hyperlipidemia are the disorders most frequently associated with chronic
relapsing pancreatitis. The LPL enzyme is responsible for intravascular hydrolysis of dietary
triglycerides carried in chylomicrons. Affected patients have functionally absent LPL and
manifest grossly lipemic plasma in the fasting state, with plasma triglyceride measurements in
excess of 1,500 mg/dL (17.0 mmol/L) (and as high as 25,000 mg/dL [282.5 mmo1/L]).
Hypercholesterolemia is another characteristic finding, with total plasma cholesterol
concentrations approximating 10% to 20% of the triglyceride value. Familial type V
hyperlipidemia also is characterized by extremely high triglyceride concentrations, typically
ranging from 500 to 10,000 mg/dL (5.7-113.0 mmol/L), and presents with elevations in both
chylomicron and low-density lipoprotein concentrations.

The most common cause of recurrent pancreatitis in the United States is hereditary pancreatitis
of which 2 types have been described. The molecular defects for both types I and II involve
mutations in the gene on chromosome 7q35 that codes for cationic trypsinogen, and both types
American academy of pediatrics 375
American Academy of Pediatrics PREP 2015

are transmitted as autosomal dominant disorders with 80% penetrance. Gene mutations allow for
autoactivation of trypsinogen in the pancreas instead of in the duodenum, leading to
intrapancreatic activation of other enzymes and subsequent pancreatic autodigestion and
inflammation.

A sweat chloride test may be helpful in the diagnosis of cystic fibrosis (CF). In CF, mutation in
the cystic fibrosis transmembrane conductance regulator (CFTR), which serves as a chloride
channel, results in reduced chloride and water secretion. The consequence is production of
abnormally thick, viscid mucus by all exocrine organs, including the pancreas, where the CFTR
is located on the apical membrane of pancreatic ductal epithelium. In addition to causing
exocrine pancreatic fibrosis and insufficiency, ductal plugging leads to 1 or more episodes of
pancreatitis in a small percentage (< 5%) of those who have CF.

PREP Pearls
• In the child with recurrent pancreatitis, a careful family history will often suggest a likely
cause.
• Lipoprotein lipase deficiency and type V hyperlipidemia are commonly associated with
recurrent pancreatitis.
• Recurrent pancreatitis may occur in the setting of cystic fibrosis, with or without a history
of pancreatic insufficiency.
• Although increased serum amylase and lipase levels are the most characteristic laboratory
findings in pancreatitis, the magnitude of these enzyme elevations do not correlate with
the degree of pancreatic injury.

American Board of Pediatrics Content Specification(s)


• Recognize the clinical and laboratory features associated with pancreatitis
• Formulate a differential diagnosis for a patient who has chronic or recurrent pancreatitis

Suggested Reading
• Belamarich PF. Lipoprotein disorders. Pediatr Rev. 1996;17(4):144. doi:10.1542/pir.17-
4-144.
• De Boeck K, Weren M, Proesmans M, Kerem E. Pancreatitis among patients with cystic
fibrosis: correlation with pancreatic status and genotype. Pediatrics. 2005;I15(4):e463-
e469. doi: 10.1542/peds.2004-1764.
• Kandula L, Lowe ME. Etiology and outcome of acute pancreatitis in infants and toddlers.
I Pediatr. 2008;152(1):106-110. doi:10.1016/j. jpeds.2007.05.050.
• Pietzak MM, Thomas DW. Pancreatitis in childhood. Pediatr Rev. 2000; 21(12):406-412.
doi:10.1542/pir.21-12-406.
• Whitcomb DC, Lowe ME. Acute and chronic pancreatitis. In: Walker WA, Goulet 0,
Kleinman RE, Sherman PM, Shneider BL, Sanderson IR, eds. Pediatric Gastrointestinal
Disease: Pathophysiology, Diagnosis, Management. 3rd ed. Hamilton, ON, Canada: BC
Decker Inc; 2004:1584-1597.
• Wittrup I-1H, Tybjaerg-Hansen A, Abildgaard S, Steffensen R, Schnohr P, Nordestgaard
BG. A common substitution (Asn29lSer) in lipoprotein lipase is associated with increased
risk of ischemic heart disease, f Gin Invest. 1997;99(7)1606-1613.
doi:10.1172/JC1119323.
American academy of pediatrics 376
American Academy of Pediatrics PREP 2015

• Yadav D, Pitchumoni CS. Issues in hyperlipidemic pancreatitis. I Clin Gastroenterol.


2003;36(1):54-62.

American academy of pediatrics 377


American Academy of Pediatrics PREP 2015

Item 122
You are charged with helping your hospital's pharmacy and therapeutic committee to formulate a
policy for recommending drug choices to treat pediatric pneumonia. In preparing for the
meeting, you review the literature on the topic.

Of the following, the MOST reliable of available analyses would be a


A. case-control study
B. double-blind, placebo-controlled trial
C. meta-analysis of published studies
D. published editorial on the subject
E. review of the product insert of relevant drugs

American academy of pediatrics 378


American Academy of Pediatrics PREP 2015

Item 122 Preferred Response: C


A systematic review of a clinical question entails a comprehensive review of the existing
literature on the topic. In performing a systematic review of the literature on a treatment regimen,
a meta-analysis offers the advantage of combining the results of multiple studies meeting
specified criteria on the subject using statistical methods that then provides a quantified estimate
of the benefits or adverse effects of the therapeutic intervention. A high quality meta-analysis
requires a comprehensive search of the literature and a critical evaluation of studies to be
included. A meta-analysis may be limited by the quality and number of studies analyzed.

Case control studies are evaluations of patients with a given condition (in this case pneumonia)
who received a specific treatment and comparing their outcomes to those receiving another
regimen. Such studies are limited by being retrospective in nature and generally involve only a
single specific population.

A double-blind, placebo-controlled trial, if randomized, is a study design in which a group of


patients are randomly assigned to receive the drug being studied or a control drug (placebo or
present standard treatment) and then assessed for prespecified outcomes. Limitations may
include the nature of the study population and the inability to compare multiple treatment
regimens.

A published editorial likely gives expert opinion based on existing evidence, but is limited by the
review and opinions of the authors.

The product insert of a drug describes the US Food and Drug Administration (FDA) approved
features of a given agent and includes studies submitted to the FDA, but may not provide a
comparison of alternative agents.

PREP Pearls
• A systematic review includes a comprehensive review of the literature on a clinical
question.
• A meta-analysis uses statistical methods to combine results of multiple studies on a given
topic.

American Board of Pediatrics Content Specification(s)


• Understand the uses and limitations of systematic review and meta-analysis

Suggested Reading
• Bair-Merritt MH. Systematic reviews and meta-analysis. Pediatr Rev. 2009;30(10):409-
410. doi:10.1542/pir.30-10-409.
• Cochrane Collaboration. What is meta-analysis? The Cochrane Collaboration website.
Accessed December 12, 2013.
• Stroup DF. Berlin JA, Morton SC, et al. Meta-analysis of observational studies in
epidemiology: a proposal for reporting: meta-analysis of observational studies in
epidemiology (MOOSE) group. JAMA. 2000;283(15):2008-2012.
doi:10.1001/jama.283.15.2008.

American academy of pediatrics 379


American Academy of Pediatrics PREP 2015

Item 123
A 3-year-old girl comes into your office for evaluation of fever and ear pain for 2 days. When
she awoke this morning, her parents noted a "bump" behind her right ear. The patient has a
history of recurrent ear infections and she had tympanostomy tubes placed when she was 18
months of age. She has been receiving a "pink antibiotic" for the last several weeks to prevent
infections because the tubes have fallen out. On physical examination, her temperature is 39°C,
pulse rate is 100 beats/min, respiratory rate is 16 breaths/min, and blood pressure is 94/65 mm
Hg. Growth parameters are at the 25th percentile for age. She is ill-appearing, but alert and
interactive during the examination. The right tympanic membrane is erythematous and dull
appearing. The right postauricular area is swollen, erythematous, and tender to touch. Her right
pinna is displaced outwardly. The remainder of the physical examination findings, including
neurologic assessment, are unremarkable.

Of the following, the MOST appropriate next step in management is to


A. admit and consult otolaryngology for tympanocentesis with culture
B. apply warm compresses to the postauricular area
C. give ceftriaxone and see the patient back in 3 days
D. obtain plain skull radiographs
E. prescribe levofloxacin orally and see the patient back in 5 days

American academy of pediatrics 380


American Academy of Pediatrics PREP 2015

Item 123 Preferred Response: A


The physical examination findings of erythema, swelling, and pain over the mastoid bone with
displacement of the pinna outwardly should always raise concern for the possibility of acute
mastoiditis with or without acute or chronic otitis media. The diagnosis of mastoiditis warrants
prompt clinical and surgical evaluation and the initiation of broad-spectrum antimicrobial
therapy; therefore, the girl in the vignette should be admitted to the hospital and otolaryngology
consulted. Antimicrobial therapy should be guided by the results of culture and sensitivity
obtained via tympanocentesis.

A history of recurrent otitis media is a risk factor for mastoiditis. Mastoiditis occurs most often
in children younger than 2 years of age. The incidence of mastoiditis has decreased dramatically
with the widespread treatment of otitis media with antibiotics. Mastoiditis has several serious
potential complications related to direct extension and rarely hematogenous spread of infection,
so prompt diagnosis and treatment are imperative. Extracranial complications of mastoiditis (and
otitis media) include cholesteatoma, subperiosteal abscess, facial nerve palsy, hearing loss,
labyrinthitis, osteomyelitis, and Bezold abscess of the sternocleidomastoid muscle. Intracranial
complications of mastoiditis (and otitis media) are uncommon and include meningitis, brain
abscess, epidural or subdural empyema, and carotid artery and venous sinus thrombosis.
The treatment of acute mastoiditis depends on the extent of disease and presence or absence of
complications. The cornerstones of therapy for acute mastoiditis are antimicrobial therapy and
drainage of the middle ear and mastoid. Obtaining material for culture and sensitivity from the
middle ear is paramount. Applying warm compresses to the postauricular area, treating with
ceftriaxone with follow-up in 3 days, and treating with oral levofloxacin are not aggressive
enough treatment in the case of probable mastoiditis in an ill-appearing young child. When
children present with the characteristic clinical findings of acute mastoiditis, imaging is not
always necessary. When imaging is needed, computed tomography, rather than plain skull films,
is the study of choice for diagnosis and determining extension of disease. If intracranial
extension is a concern, then magnetic resonance imaging should be performed.

The bacteria most likely to cause mastoiditis in children include Streptococcus pneumoniae
(including multidrug-resistant S pneumoniae), Streptococcus pyogenes, and Staphylococcus
aureus (including methicillin-resistant S aureus).

Chronic mastoiditis is often polymicrobial, including anaerobes and gram-negative bacilli,


particularly Pseudomonas, Escherichia coli, and Klebsiella. Pseudomonas aeruginosa should be
considered as a possible pathogen in children with a history of recurrent otitis media and recent
antibiotic use, such as the patient described in the vignette. P aeruginosa most commonly causes
infection in the setting of tympanic membrane perforation and chronic middle ear infection.

Patients with acute mastoiditis should be admitted and parenteral empiric broad-spectrum
antibiotics directed against these common pathogens should be initiated. Myringotomy or
tympanocentesis with aspiration and drainage of the middle ear for diagnostic and therapeutic
purposes is needed. If patients do not respond promptly (within 24 to 48 hours) to this
conservative approach, then further surgical intervention is warranted. The culture and sensitivity
results of material obtained via tympanocentesis or surgical drainage of the mastoid will further

American academy of pediatrics 381


American Academy of Pediatrics PREP 2015

direct antibiotic management. For patients with central nervous system (CNS) complications, it
is imperative that bactericidal agents, with adequate CNS penetration, are selected.

PREP Pearls
• The cornerstones of therapy for acute mastoiditis are directed antimicrobial therapy and
drainage of the middle ear and mastoid.
• The bacteria most likely to cause acute mastoiditis in children include Streptococcus
pneumoniae. Streptococcus pyogenes, and Staphylococcus aureus. Pseudomonas
aeruginosa should be considered as a possible pathogen in children with a history of
recurrent otitis media and recent antibiotic use.
• Several serious potential complications related to direct extension, or rarely,
hematogenous spread of infection from mastoiditis can occur. These include extracranial
complications such as cholesteatoma, subperiosteal abscess, facial nerve palsy, hearing
loss, labyrinthitis, osteomyelitis, and Bezold abscess (involving the sternocleidomastoid
muscle) and intracranial complications such as meningitis, brain abscess, epidural or
subdural empyema, and carotid artery and venous sinus thromboses.

American Board of Pediatrics Content Specification(s)


• Identify the etiology of mastoiditis
• Recognize the clinical findings associated with mastoiditis
• Plan the appropriate management of mastoiditis
• Plan the appropriate diagnostic evaluation of mastoiditis

Suggested Reading
• Anderson KJ. Mastoiditis. Pediatr Rev. 2009;30(6):233-234. doi:10.1542/ pir.30-6-233.
• Lewis K, Shapiro NL, Cherry JD. Mastoiditis. In: Feigin RD, Cherry JD, Demmler-
Harrison GI, Kaplan SL, eds. Feigin & Cherry's Textbook of Pediatric Infectious
Diseases. 6th ed. Philadelphia, PA: Saunders Elsevier; 2009:238-243.
• Lustig LR, Limb CJ, Baden R, LaSalvia MT. Chronic otitis media, cholesteatoma, and
mastoiditis in adults. UpToDate. Available online only far subscription.
• Thompson PL, Gilbert RE, Long PF, Saxena S, Sharland M, Wong IC. Effect of
antibiotics for otitis media on mastoiditis in children: a retrospective cohort study using
the United Kingdom general practice research database. Pediatrics. 2009;123(2):424-430.
doi:10.1542/peds.2007-3349.
• Wald ER. Acute mastoiditis in children: clinical features and diagnosis. UpToDate.
Available online only for subscription.
• Wald ER. Acute mastoiditis in children: treatment and prevention. UpToDate. Available
online only for subscription.

American academy of pediatrics 382


American Academy of Pediatrics PREP 2015

Item 124
A 10-year-old boy is brought to your office for pallor and easy bruising over the past month. He
was diagnosed with acute lymphoblastic leukemia 3 years ago and has been in remission after
receiving standard-dose chemotherapy. The mother is very concerned that his leukemia has
recurred and asks you about this.

Of the following, the MOST appropriate statement would be that

A. allogeneic stem cell transplantation for relapsed acute lymphoblastic leukemia requires a
human leukocyte antigen-matched sibling donor
B. patients who are older in age at the time of relapse have a more favorable prognosis
C. patients with a longer initial remission have a more favorable prognosis than those who
relapse within 24 months of diagnosis
D. patients with relapsed acute lymphoblastic leukemia achieve second remission less than
20% of the time
E. relapsed acute lymphoblastic leukemia occurs more frequently in extramedullary sites,
such as the central nervous system and testicles, than in the bone marrow

American academy of pediatrics 383


American Academy of Pediatrics PREP 2015

Item 124 I-C Preferred Response: C


Children with acute lymphoblastic leukemia (ALL) who experience remission for a longer period
before a relapse have a more favorable prognosis after relapse than those who experience a
relapse during or soon after therapy.

Multidrug chemotherapy regimens and standardized care through large multicenter cancer
cooperative groups have drastically increased the rates of initial remission and long-term survival
for children with leukemia over the past 5 decades. Furthermore, with risk-stratified
chemotherapy and improved supportive care, the cure rate for newly diagnosed pediatric ALL is
approximately 85% in the United States. Despite this progress, the Children's Oncology Group
data show no significant improvement in outcomes for children with a relapse of ALL from the
1988-1994 era to the 1995-2002 era. Increased drug resistance of leukemic blasts at the time of
relapse compared with blasts at initial diagnosis is thought to contribute to the difficulty in cure
of a relapse in ALL. Genomic studies show that in most cases, the cells responsible for relapse
were present as a dormant subpopulation at the time of diagnosis and then expand during therapy
and eventually lead to relapse. Less frequently, in some cases of ALL relapse, the blast cells that
have undergone a relapse have genetic mutations that are different from those of the primary
leukemia cells, thus suggesting a secondary malignancy.

The first relapse of ALL has several risk stratification schemes that are based on time of relapse,
immunophenotype (eg, B-lineage, T-lineage), site of relapse (bone marrow, extra medullary
[central nervous system and testes] or combined), and minimal residual disease after induction
therapy. The Berlin-Frankfurt-Munster Group (Germany), Children's Oncology Group (United
States), and St. Jude Children's Research Hospital (United States) have different definitions of
early relapse: less than 18 months from diagnosis, less than 36 months from diagnosis, and less
than 6 months from completion of therapy, respectively. Various studies have shown that early
relapse is associated with aggressive disease and a poor prognosis, with less than a third of these
patients surviving, compared with a 50% survival rate for patients with late relapse. Older age10
years of age) is considered a poor prognostic indicator for patients with ALL that is newly
diagnosed or in relapse. The bone marrow is the most common site of relapse. Isolated bone
marrow relapse has a worse prognosis than a combined or isolated extramedullary relapse.
Allogeneic stem cell transplantation (SCT) can be performed with a matched sibling or unrelated
matched donor, though the former is still the preferred donor type and may have a higher
likelihood of success. Emerging data suggest that in some patients experiencing a second
remission with late relapse or isolated extramedullary relapse, continued chemotherapy alone
may be just as effective as SCT.

PREP Pearls
• Various studies have shown that early relapse of acute lymphocyte leukemia (ALL) is
associated with aggressive disease and a poor prognosis, with less than a third of these
patients surviving, compared with a 50% survival rate for patients with late relapse.
• Older age (≥ 10 years of age) is considered a poor prognostic indicator for patients with
ALL that is newly diagnosed or in relapse.
• The bone marrow is the most common site of relapse in pediatric ALL. Isolated bone
marrow relapse has a worse prognosis than a combined or isolated extramedullary
relapse.
American academy of pediatrics 384
American Academy of Pediatrics PREP 2015

• Sites of extramedullary relapse are the central nervous system and testicles.
• Allogeneic stern cell transplantation can be performed with a matched sibling donor or
unrelated matched donor, though the former is still the preferred donor type.

American Board of Pediatrics Content Specification(s)


• Recognize clinical findings associated with leukemia, including sites of relapse

Suggested Reading
• Bhojwani D, Pui CH. Relapsed childhood acute lymphoblastic leukaemia. Lancet Oncol.
2013;14(6);e205-e217. doi:10.1016/S)470-2045(12)70580-6.
• Inaba H, Greaves M, Mulligan CG. Acute lymphoblastic leukaemia. Lancet.
2013;381(9881):1943-1955. doi:10.1016/S0140-6736(12)62187-4.
• Mulligan CG, Phillips LA, Su X, et al. Genomic analysis of the clonal origins of acute
lymphoblastic leukemia. Science. 2008;322(5906):13771380.
doi:10.1126/science.1164266.

American academy of pediatrics 385


American Academy of Pediatrics PREP 2015

Item 125
You are seeing a 5-year-old boy born to a drug-addicted mother. He was placed in a foster home
at 3 years of age because of neglect and suspected abuse and was adopted into his current home 1
year ago. The adoptive parents report that at least twice a day, when he does not get his way, he
will have a significant tantrum lasting at least 15 to 30 minutes. He is usually aggressive during
these tantrums, throwing things at his parents, kicking walls, and destroying his toys and books.
You have previously diagnosed him with oppositional defiant disorder and have not seen
evidence of attention-deficit/ hyperactivity disorder. After you provided the adoptive mom with
tantrum management techniques and a referral to mental health services, she privately asks you,
given his early life experiences, if there is any hope for her son's problem behaviors to improve.

Of the following, the MOST appropriate statement to make to this parent is that

A. a calm, caring home environment with consistent discipline is an effective treatment for
children with early neglect and maltreatment
B. medications for severe aggressive behavior will usually create lasting improvements in
aggression even after medication discontinuation
C. most children with aggressive oppositional defiant disorder at this age will develop
antisocial personality disorder by adulthood
D. regardless of what we do now, most children in this situation will grow out of their
aggression problems within a few years
E. research has shown that intrauterine drug exposure is not linked to aggressive or conduct
disordered behaviors in adulthood

American academy of pediatrics 386


American Academy of Pediatrics PREP 2015

Item 125 I-C Preferred Response: A


There are many risk factors for aggressive behavior in children:
• Birth complications
• Nonresponsive parenting in the first year after birth
• Coercive, escalating discipline in the toddler years
• Lack of supervision or monitoring in the adolescent years
• Lack of parental warmth
• Low IQ
• Parent-modeled use of aggression
• Impulsivity
• Living in a disadvantaged neighborhood
• A mental health disorder (eg, attention-deficit/hyperactivity disorder, post-traumatic
stress disorder, oppositional defiant disorder)
• Maltreatment by parent(s)
• Poor communication skills
• Male gender (after the preschool years)

The boy in the vignette suffered neglect and abuse for the first 3 years of his life. As a result he
has several risk factors for aggression. The fact that his mother was addicted to drugs may have
been a risk factor for his current behavior problems, but not all substances have the same risks.

Many randomized, controlled trials have provided evidence that various methods of behavior
management training are clinically effective for oppositional defiant disorder, conduct disorder,
and aggression. This approach was previously labeled as "parent management training." But the
term was changed because it implies that parents are at fault for their child's behavior, which
does not help build a therapeutic alliance. The core element of all behavior management training
programs is instruction about the creation of a calm, caring home environment with consistent
discipline. There is significant reason to hope for behavioral improvement for the boy in the
vignette, but it may take years of corrective parenting and "proving" to him that someone really
is there for him and has his best interests in mind.

Medications like alpha-agonists and antipsychotics are sometimes given to children with
significant oppositionality and aggression and can have short-term benefits, but they have not
been demonstrated to yield any long-term improvements in functional outcome.

Approximately 30% of children with oppositional defiant disorder (which may be an appropriate
diagnosis for the child in the vignette) will progress to having conduct disorder, and
approximately 50% of children with early-onset conduct disorder will progress to having
antisocial personality disorder as adults. Therefore, the chance of this child becoming an adult
with antisocial personality disorder is at most 15%.

This is a high-risk period for the boy in the vignette. Continued unresponsive or punitive
parenting at this age will make recovery from his early childhood difficulties much less likely.
As such, one cannot reasonably state that regardless of what is done now most children will
outgrow this problem within a few years.

American academy of pediatrics 387


American Academy of Pediatrics PREP 2015

Intrauterine drug exposure is clearly linked to neurobehavioral outcomes in children. However,


the outcome depends on many factors including the specific substance abused by the mother, the
amount and frequency of use, and whether multiple drugs were abused.

PREP Pearls
• Children who have been abused, neglected, and/or prenatally exposed to drugs of abuse
are at significant risk for aggression and acting out problems.
• Young children with aggression problems of multifactorial origin are best treated with
behavior management training, and with such care less than 15% will exhibit antisocial
behaviors as adults.

American Board of Pediatrics Content Specification(s)


• Recognize the various environmental and biological contributors to the development and
maintenance of aggressive behaviors

Suggested Reading
• Bennett D, Bendersky M, Lewis M. Preadolescent health risk behavior as a function of
prenatal cocaine exposure and gender. J Dev Behav Pediarr. 2007;28:467-472.
• jellinek M. Conduct disorders. In: Kliegman RM, Stanton BF, St Geme IW Schor NF,
eds. Nelson Textbook of Pediatrics. 19th ed. 2011.
• Walter I-1J, DeMaso DR. Disruptive behavioral disorders. In: Kliegman RM, Stanton
BF, St Geme JW, Schar NF, eds. Nelson Textbook of Pediatrics. 19th ed. 2011.

American academy of pediatrics 388


American Academy of Pediatrics PREP 2015

Item 126
A 9-year-old girl, recently adopted internationally, is seen for the first time in your office. She is
currently taking no medications and has not had any surgeries. Her weight is at the 50th
percentile for age, and her height is at the 75th percentile. She does not appear ill, but is
somewhat anxious. Her oxygen saturation is 100% on room air, heart rate is 90 beats/ min, and
respiratory rate is 20 breaths/min. Her blood pressure in her right arm is 130/90 mm Hg. The
blood pressure in her right leg is 90/50 mm Hg. Her neck does not show any jugular venous
distention. The chest is clear with equal breath sounds, and her abdominal examination does not
show any organomegaly. There is a 2/6 holosystolic murmur at the left midsternal border that
obscures S1. Her femoral pulses are 1+ when compared to her radial pulses of 3+, and there is
radial-femoral delay. You clinically suspect coarctation of the aorta.

Of the following, the other cardiac lesion this child's history and physical findings BEST suggest
is
A. atrioventricular canal with a large atrial septal defect component
B. left ventricular hypertrophy and outflow tract obstruction
C. severe pulmonic stenosis
D. total anomalous pulmonary venous return
E. ventricular septal defect

American academy of pediatrics 389


American Academy of Pediatrics PREP 2015

Itern126 Preferred Response: E


The patient in the vignette has significant findings on physical examination that suggest
structural heart disease. The murmur is holosystolic and obscures S1. That indicates that the
murmur begins in a short period when all the valves of the heart are momentarily closed and no
blood is moving; this is called isovolumetric contraction (Item C126). S1 is the heart sound made
when the mitral and tricuspid valves close, so this type of murmur is often called regurgitant; it is
also the classic murmur of a ventricular septal defect (VSD). This patient has a VSD. The other
important findings on this patient's physical examination are the differences in blood pressure in
the upper and lower extremities, the radial-femoral delay, and the decreased femoral pulses. All
of these findings suggest a coarctation of the aorta, which is often seen with a VSD.

S2 is the sound made when the pulmonary and aortic valves close. Usually a split in S2 is
detectable, as the pulmonic valve closes after the aortic valve. A respiratory variation is seen in
this timing difference. When the right ventricular volume increases (as in inspiration) a more
obvious split of S2 into A2 and P2 is seen. If the right ventricular volume is already very large,
as can be seen with a large atrioseptal defect (ASD), there will not be as much opportunity for
decrease in the timing difference, which creates the "fixed split" of S2. A large ASD will also
cause increased flow in the right ventricular outflow tract in systole and may cause a murmur at
the left upper sternal border. It is important to listen for a "fixed split" in S2 when a murmur in
this location is noted; this will help differentiate it from an innocent flow murmur. If there is a
systolic click in this location (left upper sternal border) and a murmur, then pulmonary valve
disease would be suspected. In the case of severe pulmonic stenosis, there should be a loud,
harsh murmur, grade 3 or more, which will extend far into systole but will not be holosystolic.
The murmur expected from left ventricular hypertrophy and outflow tract obstruction would also
be systolic, but not holosystolic: it would not obscure S1. It would begin along the left mid-
sternal border and then radiate up to the right upper sternal border following the left ventricular
outflow tract. Physical findings associated with total anomalous pulmonary venous drainage may
vary depending on whether there is an associated ASD or venous obstruction. It would not cause
a holosystolic murmur as described with a VSD.

The physical findings of the patient in the vignette also suggest a coarctation of the aorta. Blood
pressure is lower in the legs and the femoral pulses are decreased. Normally, the blood pressure
is higher in the legs than the arms. Ideally, the right arm and leg are compared to lessen the
chance of an incorrect measurement because of an aberrant right subclavian artery.

Blood pressures are not routinely measured in pediatric patients until age 3 years and even then,
not in both upper and lower extremities. If a murmur suggesting a VSD is heard, it is important
to document femoral pulse quality and strength, and measure the blood pressure in the right arm
and leg.

American academy of pediatrics 390


American Academy of Pediatrics PREP 2015

American academy of pediatrics 391


American Academy of Pediatrics PREP 2015

PREP Pearls
• The lower extremity blood pressures should be higher than the upper extremity blood
pressures.
• If a ventricular septal defect is suspected on physical examination, co-existing coarctation
of the aorta should also be suspected. The femoral pulses and upper and lower extremity
blood pressures should be documented.

American Board of Pediatrics Content Specification(s)


• Recognize the major clinical findings associated with the various types of acyanotic
congenital heart disease
• Plan the initial management of hypertension in a patient with coarctation of the aorta
• Know how to evaluate a child with femoral anteversion

Suggested Reading
• Bernstein D. Coarctation with ventricular septal defect. In: Kliegman RM, Stanton BF, St
Geme JW III, Schor NF, eds. Nelson Textbook of Pediatrics. 19th ed. Philadelphia, PA:
Saunders Elsevier; 2011:1570.
• Bernstein D. Coarctation of the aorta. In: Kliegman RM, Stanton BF, St Geme JW III,
Schor NF, eds. Nelson Textbook of Pediatrics. 19th ed. Philadelphia, PA: Elsevier
Saunders; 2011:1567-1569.
• Oski FA. Acyanotic congenital heart disease. In: McMillan JA, Feigin RD, DeAngelis
CD, Jones Jr MD, eds. Oski's Solution: Oski's Pediatrics: Principles and Practice. 4th ed.
Philadelphia, PA: Lippincott Williams & Wilkins; 2008:1553.
• Geggel RL, Fyler D. History, growth, nutrition physical exam and routine laboratory
tests. In: Keane JF, Fyler DC, Lock JE, eds. Nadas' Pediatric Cardiology. 2nd ed.
Philadelphia, PA: Elsevier; 2006:11.
• Lennox EG. Cardiology. In: Tschudy MM, Arcara KM, eds. The Harriet Lane Handbook.
19th ed. St Louis, MO: Mosby Elsevier; 2011:154-200. Park MK. Basic tools in routine
evaluation of cardiac patients. In: Pediatric Cardiology for Practitioners. 5th ed. St Louis,
MO: Mosby Elsevier; 2007: 17-33.

American academy of pediatrics 392


American Academy of Pediatrics PREP 2015

Item 127
A 3-year-old boy is brought to the emergency department for evaluation of a limp. The family
states that the patient has been crying each of the last 3 evenings. This morning, the mother
noticed the patient was limping and has been very irritable. On physical examination, the child's
temperature is 38°C. He is fussy and appears ill. The patient withdraws when you palpate along
the spine. The child walks with a nonspecific limp, and he flexes his knees, keeping his spine
straight, to retrieve a toy he has dropped. His lower extremities are not tender to palpation and
appear normal. Plain radiographs of the spine are normal. Laboratory evaluation demonstrates a
white blood cell count of 18,000/µL (18 x 109/L), erythrocyte sedimentation rate of 30 mm/h,
and a C-reactive protein level of 3.0 mg/L (28.6 nmol/L). Results of a basic metabolic panel and
urinalysis are unremarkable. Blood and urine cultures are pending.

Of the following, the BEST next test to diagnose this patient's condition is
A. computed tomography of the abdomen
B. magnetic resonance imaging of the spine
C. plain radiography of the pelvis and hips
D. technetium bone scan
E. ultrasonography of the abdomen and kidneys

American academy of pediatrics 393


American Academy of Pediatrics PREP 2015

Item 127 Preferred Response: B


The child in the vignette has signs and symptoms consistent with discitis. Magnetic resonance
imaging of the spine is the most sensitive and specific test to confirm the diagnosis. Early in the
disease, plain radiographs are typically normal, whereas later in the disease course the disc space
may have some narrowing. A technetium bone scan would show signs of inflammation, but
would not be specific for this diagnosis. Computed tomography of the spine will show discitis
earlier than plain radiography and can augment magnetic resonance imaging by helping to
distinguish soft tissue from bone.

Discitis, or infection of the intervertebral disc, is thought to be caused by hematogenous spread


of infection; however, no consistent primary infection source has been identified in children.
Discitis is rare and usually presents in children younger than 5 years. Most children with discitis
will be afebrile or have a low-grade fever. They present with a limp or refusal to walk, back pain,
and irritability. The lumbar region is most often affected, and most patients will have localized
tenderness. Inflammatory markers and white blood cell counts may be increased, however these
test results are not specific. In the majority of cases, blood cultures are negative. Other cultures,
such as urine culture, should be considered to help identify possible sources of infection. The
most common causative organism is Staphylococcus aureus. Treatment consists of
immobilization and 4 to 6 weeks of antibiotics for children with positive cultures, elevated
erythrocyte sedimentation rates, or elevated white blood cell counts. For patients with normal
laboratory studies, the length and route of treatment is more controversial. Surgery is rarely
indicated.

Computed tomography of the abdomen would not be appropriate in this case, because the boy in
the vignette has findings that localize to the spine. Plain radiography of the pelvis and hips would
not be appropriate in this patient with a normal lower extremity examination and no pain with
movement or palpation of the lower extremities. Ultrasonography of the abdomen and kidneys is
not indicated based on history or physical examination. He has no urinary tract symptoms, a
normal urinalysis, and basic metabolic panel.

PREP Pearls
• Children with discitis present with limping or refusal to walk, back pain, and are often
afebrile.
• Discitis occurs most often in children younger than 5 years.
• Magnetic resonance imaging is the most sensitive and specific test for diagnosing discitis.
• Staphylococcus aureus is the most common causative organism in discitis.

American Board of Pediatrics Content Specification(s)


• Formulate a differential diagnosis of back pain in children of various ages

Suggested Reading
• Ferrnandez M, Carrot CL, Baker Cl. Discitis and vertebral oseomyelitis in children: an
18-year review. Pediatrics. 2000:105(6):1299-1304. doi:10.1542/peds.105.6.1299.
• Haidar R, Saad S, Khoury N. Musharrafieh U. Practical approach to the child presenting
with back pain. Eur I Pediatr. 2011;170(2):149-156. doi:10.1007/500431-010-1220-9.

American academy of pediatrics 394


American Academy of Pediatrics PREP 2015

Item 128
A 15-year-old adolescent boy presents to your office for follow-up 3 days after a right shoulder
injury. He was playing basketball when another player pulled his arm posteriorly while he
reached up to rebound the ball. He had the feeling that his shoulder popped out. The boy was
seen in the emergency department after the injury. Radiography confirmed that he had dislocated
his shoulder and he underwent reduction under sedation. You counsel the family about this
injury.

Of the following, the MOST accurate statement is that


A. adolescent boys have lower rates of recurrent dislocation than do adolescent girls
B. adolescents are at lower risk for recurrent dislocations compared with adults
C. in adolescents with shoulder dislocation, the humeral head is generally posteriorly
displaced
D. rotator cuff tears are often associated with shoulder dislocation in adolescents
E. without surgery, he is likely to experience recurrent dislocation over the next 12 months

American academy of pediatrics 395


American Academy of Pediatrics PREP 2015

Item 128 Preferred Response: E


The boy in the vignette has suffered an acute traumatic shoulder dislocation. Given his age and
gender, he is likely to suffer an additional dislocation in the subsequent year. Adolescents are
particularly vulnerable to recurrent dislocations, therefore many orthopedic providers
recommend surgical stabilization after an initial traumatic shoulder dislocation.

Shoulder dislocations can be acute, resulting from trauma, as is the case for the adolescent in the
vignette, or can be a result of multidirectional instability, when lax ligaments allow the humeral
head to move off the glenoid. Ligamentous laxity of the shoulder can be associated with
generalized hypermobility of the joints. In athletes who engage in sports with repetitive overhead
shoulder movements, such as swimming or volleyball, repetitive microtrauma can result in
localized ligamentous laxity.

Acute traumatic dislocations generally occur when a force is aimed at the arm while the shoulder
is held in abduction and external rotation. The boy in the vignette likely had his shoulder in this
position when rebounding the basketball. Most young patients have an associated Bankart lesion,
a tear of the labrum, which is the soft cartilage lining of the glenoid.

Individuals with shoulder dislocations nearly always experience immediate pain and describe a
sensation that the shoulder is "out of place On physical examination, the acromion will appear to
be very prominent and the proximal humeral head will not be palpable in its normal position.
In the acute care setting, radiography should be performed before reduction to document bony
injuries. Radiography allows a clinician to evaluate for the presence of a bony Ban-kart lesion
(labrum injury with a fracture of the glenoid) or a Hill-Sachs lesion (an impaction fracture
involving the humeral head).

Athletes with a shoulder dislocation should be treated acutely using a reduction maneuver to
restore the normal position of the humeral head. Experienced clinicians may be able to perform a
shoulder reduction maneuver in the field before muscle spasm makes sedation and analgesia
necessary. However, in most cases, a health care provider is not present at the time of injury, and
reductions are generally performed in the emergency department using procedural sedation.

Following reduction, the shoulder should be immobilized for 2 to 3 weeks. Although slings are
typically used to immobilize the shoulder, there is some evidence that use of a brace that
positions the shoulder in external rotation for the first 24 hours after an injury reduces the risk of
recurrence. After a few weeks, patients should undergo physical therapy to improve range-of-
motion and strength in the shoulder. For some patients, elective surgical repair of torn ligaments
and the labrum may be indicated to stabilize the shoulder.

Younger patients and males have high rates of recurrent dislocation. Male adolescents have
approximately a 75% chance of redislocation in the first year after initial dislocation. Some
providers feel that individuals in this demographic group should be considered surgical
candidates after an initial dislocation.

American academy of pediatrics 396


American Academy of Pediatrics PREP 2015

With most shoulder dislocations, the humeral head is displaced anteriorly and inferiorly.
Posterior dislocations make up only 5% of dislocations. Rotator cuff tears are uncommon in
adolescents.

PREP Pearls
• Shoulder dislocations can be caused by acute trauma or can result from laxity of the
shoulder ligaments.
• Adolescents and boys have high rates of redislocation after initial traumatic dislocation.

American Board of Pediatrics Content Specification(s)


• Understand the natural history of shoulder dislocation
• Plan the appropriate initial management of a sports-related shoulder injury

Suggested Reading
• Hovelius L, Olofsson A, Sandstrom B, et al. Nonoperative treatment of primary anterior
shoulder dislocation in patients forty years of age and younger. a prospective twenty-five-
year follow-up. Bone Joint Surg Am. 2008;90(5):945-952. doi:10.21D6IJBIS.G'00070.
• Mather RC 3rd, Orlando LA, Henderson RA, Lawrence IT, Taylor DC. A predictive
model of shoulder instability after a first-time anterior shoulder dislocation. J Shoulder
Elbow Surg. 2011;20(2):259-266. doi:10.1016/j. jse.2010.10.037.
• Sarwark IF, La Bella C. Pediatric Orthopaedics and Sports Injuries: A Quick Reference
Guide. Elk Grove Village, IL: American Academy of Pediatrics; 2010:650 pp.

American academy of pediatrics 397


American Academy of Pediatrics PREP 2015

Item 129
You are caring for a neonate with congenital syphilis in a busy intensive care unit and prescribe
penicillin 50,000 U/kg per day divided every 8 hours.

Of the following, the MOST likely medication error to occur in this scenario is
A. a "look-alike" drug is administered to the patient
B. a "sound-alike" drug is administered to the patient
C. penicillin is administered to the patient at the wrong dose
D. penicillin is administered to the patient at the wrong time
E. penicillin is administered to the wrong patient

American academy of pediatrics 398


American Academy of Pediatrics PREP 2015

Item 129 S SBP Preferred Response: C


For the patient described in the vignette, the most likely medication error to occur is that
penicillin is administered to the patient at the wrong dose. The abbreviation "U" for "units" is on
the official "Do Not Use List" published by The Joint Commission (Item C129, page C-112)
because the "U" can easily be mistaken for a zero and result in a ten-fold overdose of medication.
In order to avoid this error, "units" should be spelled out completely.

Strategies to prevent medication errors include:


• ensuring an adequate number of pharmacy and nursing staff to prepare and administer
medications
• providing appropriate equipment and systems for medication administration
• standardizing measurement systems (ie, kilograms instead of pounds)
• standardizing order sheets or utilizing computerized physician order entry with alerts
• employing evidence-based clinical pathways with order sets
• developing error tracking systems
• providing educational programs
• adopting family-centered care with discussion of medication regimens
• fostering a nonpunitive culture for reporting errors
• championing quality improvement and patient safety initiatives

American academy of pediatrics 399


American Academy of Pediatrics PREP 2015

In particular, the discussion of medication regimens with families is an additional step in the care
process that helps ensure the correct drug and doses are given. Nurses and family members are
usually the first individuals to notice a change in the appearance, volume, or schedule of the
patient's medication.

For the patient described in the vignette, common hospital measures to prevent medication
errors, such as using contrasting labeling for look-alike drugs and confirming a patient's identity
before administering a medication, make administering the wrong medication and giving a
medication to the wrong patient less likely than giving the wrong dose. In addition, giving
penicillin at the wrong time would not be an expected error with the medication prescription
provided.

PREP Pearls
• The avoidance of "Do Not Use" Abbreviations published by The Joint Commission can
prevent medication errors.
• Strategies to prevent medication errors include:
o ensuring an adequate number of pharmacy and nursing staff to prepare and
administer medications
o providing appropriate equipment and systems for medication administration
o standardizing measurement systems (ie. kilograms instead of pounds)
o standardizing order sheets or utilizing computerized physician order entry with
alerts
o employing evidence-based clinical pathways with order sets
o developing error tracking systems, providing educational programs
o adopting family-centered care with discussion of medication regimens
o fostering a nonpunitive culture for reporting errors championing quality
improvement and patient safety initiatives

American Board of Pediatrics Content Specification(s)


• Understand and apply methodologies to prevent medication errors

Suggested Reading
• American Academy of Pediatrics. Prevention of medication errors in the pediatric
inpatient setting. Pediatrics. 2003;112(2):431-436.
• For Fescue ES, Kaushal R, La ndr igan CP, et al. Prioritizing strategies for preventing
medication errors and adverse drug events in pediatric inpatients. Pediatrics. 2003;111(4
Pt 1):722-729,
• Institute for Safe Medication Practices. institute for Safe Medication Practices' list of
error-prone abbreviations, symbols, and dose designations. Institute for Safe Medication
Practices website.
• The Joint Commission. Facts about the Official "Do Not Use' List. The Joint Commission
website.

American academy of pediatrics 400


American Academy of Pediatrics PREP 2015

Item 130
A 14-month-old boy is transported to the emergency department by emergency medical services
because of high fever and increasing lethargy. His mother reports that he was at his baseline state
of good health until he began having fever 2 days ago. This morning, his mother noted that he
seemed much more lethargic and looked pale and that his hands and feet seemed "blue and cold."
On physical examination, the boy is toxic appearing and responsive only to painful stimuli. He
has thready peripheral pulses and cool extremities. His temperature is 40.2°C, heart rate is 190
beats/min, respiratory rate is 60 breaths/min, and blood pressure is 40/18 mm Hg. His airway is
intact, and his oxygen saturation by pulse oximetry is 94% on room air. Multiple attempts by
paramedics to establish peripheral intravenous access were unsuccessful, and 3 attempts by an
experienced emergency department nurse have also been unsuccessful. You order administration
of 100% oxygen by a non-rebreather face mask for the boy.

Of the following, the BEST next step in the management of this patient is
A. emergent surgical consultation for venous cutdown
B. placement of an intraosseous needle
C. placement of a femoral venous catheter
D. placement of a nasogastric tube for enteral hydration
E. reattempt peripheral intravenous catheter placement using ultrasonographic guidance

American academy of pediatrics 401


American Academy of Pediatrics PREP 2015

Item 130 S Preferred Response: B


The young boy in the vignette presents with clinical findings consistent with the diagnosis of
septic shock. In addition to support of his airway and breathing, he requires emergent fluid
resuscitation. Multiple attempts to establish peripheral intravenous access have been
unsuccessful; therefore establishing intraosseous access is the best next step in managing his
condition.

Rapid volume resuscitation is imperative to restoring adequate organ perfusion in children


presenting with various forms of circulatory collapse, including septic shock. Obtaining rapid
vascular access with at least 2 large bore peripheral intravenous catheters is the ideal first-line
measure to restore circulation in such patients. This access, however, can be challenging because
most children with shock have cool, poorly perfused extremities. If sufficient intravenous access
cannot be obtained quickly, the American Heart Association's Pediatric Advanced Life Saving
(PALS) guidelines recommend placing an intraosseous needle as the best next step in
management for children and infants past the neonatal period. Intraosseous needle placement is
an acceptable alternative as a first attempt at vascular access in children with full
cardiopulmonary arrest or severe shock. Intraosseous needle placement may also be necessary in
critical situations when venous assess cannot be quickly achieved (such as for children with
shock, status epilepticus, extensive burns, and multiple trauma).

When placement of an intraosseous needle is necessary for resuscitating a critically ill child, an
18- or 20-gauge manual intraosseous infusion needle, commercial battery-powered driver device
(EZ-IO) or bone injection gun (ie, BIG or FAST1) can be used. The site most often frequently
used to place an intraosseous needle in children is the flat part of the proximal tibia, just below
the growth plate (1-3 cm below and medial to the tibial tuberosity). Other potential placement
sites include the distal femur, distal tibia, proximal humerus, and the anterior-superior iliac spine.
The site selected should be prepared using an aseptic technique before needle placement; the
clinical provider should use universal precautions throughout the procedure. In conscious
children, the skin, subcutaneous tissue, and periosteum at the site of needle insertion should be
anesthetized with 1% lidocaine solution. When establishing intraosseous access in the proximal
tibia, the provider should position the child's leg with slight external rotation and stabilize it on a
firm surface. If using a manual intraosseous needle, the needle with stylet in place should be
inserted through the skin perpendicular to the tibia, and slightly angled away from the growth
plate once the correct insertion site is identified. The needle should be advanced through the
subcutaneous tissue and then through the periosteum, using a twisting motion with gentle but
firm pressure. The provider should continue inserting the needle through the cortical bone until a
sudden decrease in resistance is felt as the needle enters the marrow space.

When placed correctly, needles should stand easily without support. Once an intraosseous needle
is placed, the stylet should be removed and a syringe can be attached. Aspiration of bone marrow
or blood into the needle hub confirms correct placement, though these fluids may not always be
aspirated. A small volume of saline should infuse easily when flushed through the needle without
infiltration. Once proper placement is confirmed, the intraosseous needle should be stabilized
and secured, and the site should be checked frequently for signs of swelling and displacement.
Technique for intraosseous needle placement using a commercial driver or injection device will
not be reviewed here.
American academy of pediatrics 402
American Academy of Pediatrics PREP 2015

Any resuscitation fluid or intravenous drug can be safely administered by the intraosseous route,
including crystalloid and colloid fluid solutions, epinephrine, antiepileptics, antibiotics, glucose,
and blood products. Intraosseous fluid and medication dosing is equivalent to that used for
intravenous administration. Samples for basic laboratory studies such as hemoglobin, serum
glucose, electrolytes, and bacterial cultures may also be obtained from an intraosseous line.
Although there are few absolute contraindications to intraosseous needle placement, intraosseous
needles should not be placed in fractured or previously cannulated bones or in extremities with a
vascular interruption. Intraosseous needle placement should also be avoided in children with
underlying bone diseases such as osteogenesis imperfecta and osteopetrosis, in those with burns
or infections near the access site, and in children with right-to-left congenital cardiac shunts.

Obtaining surgical consultation for assistance with establishing reliable vascular access in this
child with decompensated shock would not be unreasonable, but immediate fluid resuscitation is
required to prevent further clinical deterioration. An intraosseous needle should be placed right
away and goal-directed fluid resuscitation should be initiated before surgical consultation.
Eventual placement of a femoral venous catheter may also be needed in this critically ill child,
but intraosseous needle placement is the preferred next step for rapid intravascular access.
Central venous catheter placement requires considerably more time than intraosseous needle
placement, even when performed by individuals experienced in central line placement.
Administering fluids enterally by a nasogastric tube would not be an adequate method for
restoring intravascular volume in any child with decompensated (or even compensated) shock.
Although ultrasonography is a useful tool that may aid in the placement of peripheral as well as
central intravenous lines, additional attempts to place a peripheral intravenous catheter are
unlikely to be successful and would not be the best next step for this critically ill, poorly perfused
boy. Intraosseous needle placement is much more likely to be successful than ultrasound-guided
intravenous cannulation in this scenario.

PREP Pearls
• If adequate intravenous access cannot be obtained quickly, placement of an intraosseous
needle is the best next step in management for critically ill children and infants past the
neonatal period.
• Any resuscitation fluid or intravenous drug can be safely administered by the
intraosseous route, at doses equivalent to those used for intravenous administration.
• Intraosseous needles should not be placed in fractured or previously cannulated bones or
in extremities with a vascular interruption.

American Board of Pediatrics Content Specification(s)


• Plan the appropriate use of intraosseous therapy

Suggested Reading
• American Heart Association. In: Chameides L, Samson RA, Schexnayder SM, Hazinski
MF, eds. Pediatric Advanced Life Support Provider Manual. Dallas, TX: American
Hearth Association; 2011:109-110.
• Luck RP, Hanes C, Mull CC. Intraosseous access. I Emerg Med. 2010;39(4):468-475.
doi:10.I016/j.jemermed.2009.04.054.
American academy of pediatrics 403
American Academy of Pediatrics PREP 2015

• Vogt I, Waltzman M, Lottenberg L. Intraosseous vascular access for in hospital use.


Pediatr Emerg Care. 2012;28(2):185-199. doi:10.1097/ PEC.0b013e3182449edc.
• Yager P. Noviski N. Shock. Pediatr Rev. 2010;31(8):311-319. doi:10.1542/ pir.31-8-311.

American academy of pediatrics 404


American Academy of Pediatrics PREP 2015

Item 131
You are assessing a newborn with hypoglycemia 8 hours after birth. She was born at 37 weeks'
gestation to a 40-year old primigravida woman whose pregnancy was notable only for the use of
assisted reproductive technology. The newborn required admission to the special care nursery
shortly after birth for intravenous glucose therapy to maintain her blood sugar values over 45
mg/dL (2.5 mmol/L). Your physical examination reveals a large for gestational age newborn
with an abdominal wall defect (Item Q131, page Q-37).

Of the following, the additional clinical finding this patient is MOST likely to have is
A. anorectal malformation
B. hypospadias
C. large tongue
D. patent urachus
E. single umbilical artery

American academy of pediatrics 405


American Academy of Pediatrics PREP 2015

Item 131 Preferred Response: C


The newborn in the vignette has the clinical findings of Beckwith-Wiedemann syndrome (BWS),
which is associated with macroglossia (large tongue). Beckwith-Wiedemann syndrome is related
to alterations on chromosome 11p15.5, with a familial transmission rate of 15%. There is an
increased incidence seen in pregnancies requiring assisted reproductive technology. Beckwith-
Wiedemann syndrome is an overgrowth syndrome with affected infants often being large for
gestational age at birth. Other manifestations of somatic overgrowth include macroglossia,
visceromegaly, and hemihypertrophy. Islet cell hyperplasia leads to the hypoglycemia seen in up
to 50% of affected infants. Abdominal-intestinal wall defects including omphalocele, umbilical
hernia, and diastasis recti are commonly seen with BWS.

An omphalocele is an abdominal-intestinal wall defect formed by the protrusion of bowel into


the base of the umbilical cord, whereas a gastroschisis is formed by loops of bowel extending
through a defect in the abdominal wall to the right of the umbilical cord. Both require immediate
surgical evaluation and management in the neonatal period. The initial management in the
delivery room includes covering the exposed bowel with saline-soaked sterile dressings,
inserting a gastric tube to decompress the bowel, and placing the newborn in a bowel bag to the
level of the axilla to minimize fluid and heat losses. Parenteral fluid resuscitation is often needed
because of increased insensible losses associated with the defects.

The type of abdominal-intestinal wall defect guides further evaluation. Gastroschisis is an


isolated defect seen in infants with intrauterine growth restriction: approximately 10% will have
coexisting intestinal atresia or stenosis. Omphalocele is associated with other anomalies in up to
70% of cases. Beckwith--Wiedemann syndrome may be seen in 10% of affected infants, whereas
karyotype anomalies, including trisomy 13, 18, and 21, may be found in up to 30% of cases.
Echocardiography should be performed in any infant with an omphalocele because of the 50%
risk of congenital heart disease.

Anorectal malformations, hypospadias, patent urachus, and single umbilical artery are not
associated with BWS. They are unlikely to be found in the newborn in the vignette

PREP Pearls
• An omphalocele is an abdominal wall defect formed by the protrusion of bowel into the
base of the umbilical cord, whereas a gastroschisis is formed by loops of bowel extending
through a defect in the abdominal wall to the right of the umbilical cord.
• The initial management of an abdominal-intestinal wall defect in the delivery room
includes covering the exposed bowel with saline-soaked sterile dressings, inserting a
gastric tube to decompress the bowel, and placing the newborn in a bowel bag to the level
of the axilla to minimize fluid and heat losses.

American Board of Pediatrics Content Specification(s)


• Plan the appropriate evaluation and management of a newborn infant who has abdominal-
intestinal wall defect

Suggested Reading

American academy of pediatrics 406


American Academy of Pediatrics PREP 2015

• Chabra S. Management of gastroschisis: prenatal, perinatal, and neonatal. NeoReviews.


2006;7(8):e419-e427. doi:10.1542/neo.7-8-e419.
• Hwang P, Kousseff BG. Omphalocele and gastroschisis: an 18-year review study. Genet
Med. 2004;6(4):232-236. do1:10.109701. G1M.0000133919.68912.A3.
• Kastenberg Z1, Dutta S. Ventral abdominal wall defects. NeoReviews. 2013;14(8):e402-
e411. doi:10.1542/nen.14-8-e402.
• Weksberg 12, Shuman C. Beckwith M. Beckwith-Wiedemann syndrome. Eu r J Hum
Genet. 2010;18:8-14. doi:10.1038/ejhg.2009.106.

American academy of pediatrics 407


American Academy of Pediatrics PREP 2015

Item 132
The parents of a 6-year-old boy bring him to your office for evaluation because he is struggling
in first grade. The results of learning testing done at the school are pending, but the parents were
told he most likely has a learning disorder, and the school wants you to evaluate him for a
medical cause of his problems. The boy's parents report he pays attention well at school and at
home and that he is trying hard to learn. He has no chronic medical problems. His vision and
hearing are normal. His physical examination results show an occipital frontal head
circumference at the 90th percentile, with a height and weight at the 50th percentile. His blood
pressure is 96/56 mm Hg, heart rate is 92 beats/min, and respiratory rate is 32 breaths/min. On
his back and legs, he has 8 flat, brown macules with regular borders, 6 mm to 10 mm wide.
Of the following, the additional finding this boy is MOST likely to develop is

A. cutaneous hemangioma
B. iris coloboma
C. leptomeningeal angioma
D. optic glioma
E. renal angiomyolipoma

American academy of pediatrics 408


American Academy of Pediatrics PREP 2015

Item 132 Preferred Response: D


The boy in the vignette most likely has neurofibromatosis type 1 (NF1). This diagnosis could be
definitively made if there is an optic glioma.

Neurofibromatosis type 1 is an autosomal, recessively inherited neurocutaneous disorder


characterized by café au lait macules, neurofibromas, and bony lesions. Often, children with NF1
have macrocephaly and learning disorders, as does the boy in the vignette. Diagnosis is based on
the presence of 2 or more of the following clinical criteria:
• Six or more café au lait macules, greater than 5 mm in diameter in prepubertal children or
15 mm in diameter in postpubertal children
• Two or more neurofibromas
• One or more plexiform neurofibroma
• Axillary or inguinal freckling
• Optic glioma
• Two or more iris hamartomas (Lisch nodules)
• Osseous lesion such as sphenoid dysplasia or tibial pseudarthrosis
• First-degree relative with NF I

The clinical presentation of macrocephaly, learning disorder, and more than 6 cafe au lait
macules that are greater than 5 mm in diameter in a prepubertal boy suggests a diagnosis of NF
1. Obtaining a family history and performing dermatologic and ophthalmologic examinations in
the office can expedite a diagnosis for the boy and his family.

In the absence of family history, the clinical diagnosis of NF1 can be difficult, because the
findings are often absent in younger patients. Neurofibromas and optic gliomas are often slow
growing, and the patient may be asymptomatic until late in childhood, adolescence, or even
adulthood. Referral to an ophthalmologist may be necessary to diagnose and establish follow-up
care for an optic glioma. Medical genetics referral would be helpful to establish a genetic
diagnosis and estimate risk for future children.

Cutaneous hemangioma and iris coloboma are not particularly associated with NF1 and are not
part of the diagnostic criteria. Leptomeningeal angioma is associated with Sturge Weber
syndrome, which this boy does not have. Renal angiomyolipomas are seen in individuals with
tuberous sclerosis.

PREP Pearls
• Diagnosis of neurofibromatosis type 1 (NF1) is based on clinical criteria.
• Typical physical findings of NF1 may not be present until late childhood or adolescence.

American Board of Pediatrics Content Specification(s)


• Recognize the clinical findings associated with neurofibromatosis in patients of various
ages

Suggested Reading
• Friedman JM. Neurofibromatosis 1. GeneReviews. Updated May 3, 2012. Accessed
April 26, 2013.
American academy of pediatrics 409
American Academy of Pediatrics PREP 2015

Item 133
A 13-year-old adolescent girl who was treated successfully for leukemia with chemotherapy and
cranial irradiation at 2 years of age now presents for evaluation of a "lump" on the side of her
neck. She also reports an increase in fatigue over the last few months, as well as a 3.2-kg weight
gain. She denies any skin changes, temperature intolerance, or changes in school performance.
On physical examination, you note a firm, hard, fixed mass over the left thyroid. Her thyroid
gland also appears to be enlarged.

Of the following, the BEST test to determine the underlying cause of this girl's abnormal
physical finding is
A. fine-needle aspiration thyroid biopsy
B. iodine uptake scan
C. thyroid peroxidase antibody
D. thyroid ultrasonography
E. thyrotropin

American academy of pediatrics 410


American Academy of Pediatrics PREP 2015

Item 133 Preferred Response: A


The child in the vignette has a history of radiation exposure and a thyroid nodule. The best test to
determine if the nodule is benign or malignant is fine-needle aspiration (FNA) biopsy.
Sometimes FNA results are inconclusive, and surgical removal of part or all of the thyroid
becomes necessary. Fine-needle aspiration is most often performed under ultrasonographic
guidance.

All patients with a thyroid nodule require ultrasonography to assess for features of thyroid
malignancy to determine if there are nonpalpable lesions that may also warrant FNA and to
assess the lymph nodes in the central and lateral neck_ Given this patient's history of radiation
exposure to the neck and the presence of a palpable nodule, FNA is still needed and is the only
test offered that can give a definitive diagnosis.

A '"iodine uptake scan can be used to determine if the nodule is hot (hyperactive) or cold
(inactive). In this case, '"iodine uptake should only be performed if the child's thyrotropin (TSH)
level is at the bottom of the normal range or actually suppressed. Given the patient had no signs
of hyperthyroidism, this is unlikely.

Radiation exposure can also lead to autoimmune disease (Hashimoto disease), and given the
child's thyroid enlargement, weight gain, and fatigue, checking thyroid peroxidase antibodies to
look for autoimmune disease is reasonable. In addition, measurement of thyrotropin levels is
recommended. It is important to verify that thyrotropin levels are not suppressed. Thyroid
nodules and thyroid cancer are also more common in patients with high-normal or elevated
thyrotropin, especially in those with a history of prior radiation exposure. However, an abnormal
thyrotropin level would not diagnose the underlying cause of this patient's thyroid nodule. For
that diagnosis, FNA is needed.

For low-risk patients (those with no history of radiation therapy to the neck and no family history
of thyroid cancer) in whom a thyroid nodule is found incidentally, ultrasonography should be
done first and FNA would be warranted for nodules greater than or equal to 1 cm in size or for
any concerning characteristics seen in the nodule on imaging studies.

PREP Pearls
• A history of radiation exposure increases the risk of thyroid malignancy later in life.
• The best test to assess for malignancy in the evaluation of a discrete thyroid nodule in a
high-risk patient is fine-needle aspiration.

American Board of Pediatrics Content Specification(s)


• Recognize the clinical features associated with a thyroid cyst/tumor
• Plan the appropriate evaluation and management of a thyroid mass/ nodule

Suggested Reading
• American Thyroid Association (ATA) Guidelines Taskforce on Thyroid Nodules and
Differentiated Thyroid Cancer, Cooper DS, Doherty GM, et al. Revised American
Thyroid Association management guidelines for patients with thyroid nodules and

American academy of pediatrics 411


American Academy of Pediatrics PREP 2015

differentiated thyroid cancer. Thyroid. 2009;19(11):1167-1214.


doi:10.1089/thy.2009.0110.
• Rosenfeld RG, Cohen P. Disorders of growth hormone/insulin-like growth factor
secretion and action. In: Sperling MA, ed. Pediatric Endocrinology. 3rd ed. Philadelphia.
PA: Saunders Elsevier; 2008:254-334.
• Waguespack SG, Francis G. Initial management and follow-up of differentiated thyroid
cancer in children. J Natl Compr Canc Netw. 2010;(8)11:1289-1300.

American academy of pediatrics 412


American Academy of Pediatrics PREP 2015

Item 134
A 5-year-old boy comes to your office for a mild upper respiratory infection. While there, the
parent also mentions concerns about hair loss. The child was hospitalized 2 months ago for a
femur fracture due to a motor vehicle collision and the child received heparin prophylaxis. His
past medical history is otherwise unremarkable, and he is appropriately grown. The parent denies
any skin changes, temperature intolerance, or unusual hair product use. He is not currently taking
any medications. On physical examination, there is mild nasal congestion and moderate diffuse
hair loss without scaly patches (Item Q134). He has a positive result on hair pull test. The
remainder of his examination is unremarkable.

Of the following, the MOST likely etiology of this child's hair loss is
A. heparin given during his hospitalization
B. stress from the upper respiratory infection
C. thyroid disease
D. Trichophyton tonsurans infection
E. trichotillomania

American academy of pediatrics 413


American Academy of Pediatrics PREP 2015

Item 134 Preferred Response: A


The child in the vignette shows the characteristic diffuse, nonscaly hair thinning of telogen
effluvium. The cause of this phenomenon is likely multifactorial, but medications have been
implicated as triggers in many cases. As many as 50% of patients who take anticoagulants,
particularly low molecular weight heparins, experience hair thinning a few weeks after the drug
exposure. Thus, for the patient in the vignette, exposure to heparin during a hospitalization 2 to 3
months ago is a likely trigger for this child's hair loss. Other medications that may be associated
with telogen effluvium include oral contraceptives (particularly when interrupted after long-term
use), isotretinoin, anticonvulsants, psychotropics (lithium, sodium valproate, fluoxetine), β-
adrenergic blockers (metoprolol and propranolol), angiotensin-converting enzyme inhibitors,
isoniazid, and antiretroviral agents (indinavir).

Telogen effluvium represents a disruption in the normal hair life cycle. Normally, more than
85% of scalp hairs are in the anagen or growth phase, which lasts approximately 3 years. As the
hair follicle begins to involute, the hair enters the 2-week-long catagen or resting phase. The hair
then moves to the telogen phase lasting approximately 3 months, during which the hair fiber is
shed. Approximately 13% of scalp hairs are in the telogen phase normally and about 50 to 150
hairs are shed daily. During telogen effluvium, the anagen phase is terminated early for many
more follicles than usual, pushing them to telogen and leading ultimately to increased daily hair
loss.

Factors other than medications can also lead to telogen effluvium. Commonly associated triggers
include acute, severe illness (particularly with high fever), childbirth, major surgery, and thyroid
disease. However, because of the duration of the hair growth cycles, the inciting event for an
episode of telogen effluvium usually occurs 2 to 3 months before hair loss is detected. Therefore,
an intercurrent upper respiratory infection is an unlikely trigger for this child's hair loss. If hair
loss persists for 6 weeks, the child would be considered to have chronic telogen effluvium.
Causes for chronic telogen effluvium include thyroid disease, iron-deficiency anemia,
malnutrition, chronic illness (eg, lupus), zinc deficiency, or syphilis. The short duration of this
child's condition and the absence of other symptoms of thyroid disease make this a less likely
trigger than the recent heparin exposure. The primary management of telogen effluvium is to
remove or treat the underlying cause. Most patients experience complete hair regrowth within 6
months.

Trichotillomania typically presents with irregular frontotemporal patches of hair loss with broken
and variable length hairs within the affected region. Therefore, this child's widespread hair loss
and absence of broken hairs makes this a less likely diagnosis. Likewise, tinea capitis caused by
Trichophyton tonsurans infection presents with patches of scaly hair loss and can be ruled out
based on physical appearance.

PREP Pearls
• Telogen effluvium occurs 2 to 3 months after an inciting event.
• The physical findings of telogen effluvium include increased daily hair loss, positive hair
pull test, widespread hair loss without scale, and generalized hair thinning.
• Triggers for telogen effluvium include stressors (severe illness, high fever, childbirth,
trauma), various medications (including heparin, oral contraceptives, isotretinoin,
American academy of pediatrics 414
American Academy of Pediatrics PREP 2015

anticonvulsants, psychotropics, β-adrenergic blockers, angiotensin-converting enzyme


inhibitors, isoniazid, and antiretroviral agents), thyroid disease, iron deficiency,
malnutrition, and chronic illness.
• Treatment for telogen effluvium is to eliminate or address the underlying condition.

American Board of Pediatrics Content Specification(s)


• Recognize the clinical findings associated with telogen effluvium, and manage
appropriately

Suggested Reading
• Bedocs LA, Bruckner AL. Adolescent hair loss. Curr Opin Pediatr. 2008; 20(4):431-435.
doi:10.1097/M0P.0b013e328305e285.
• Krowchuk DP, Mancini AL Telogen effluvium. In: Krowchuk DP, Mancini Al, eds.
Pediatric Dermatology: A Quick Reference Guide. 2nd ed. Elk Grove Village IL:
American Academy of Pediatrics; 2011:483-488.
• Patel M, Harrison S, Sinclair R. Drugs and hair loss. Dermatol Clin. 2013; 31(1):67-73.
doi:10.1016/j.det.2012.08.002.
• Tosti A, Pazzaglia M. Drug reactions affecting hair: diagnosis. Dermatol Clin.
2007;25(2):223.231. doi:10.1016/j.det.2007.01.005.

American academy of pediatrics 415


American Academy of Pediatrics PREP 2015

Item 135
A previously healthy 17-year-old adolescent boy well known to your practice presents with the
complaint of intermittent burning during urination for 1 week. He has no urgency, hematuria, or
systemic complaints. He admits to intermittent use of condoms when sexually active. On
physical examination, he is at sexual maturity rating 5. He is uncircumcised, with moistness and
a clear discharge from the urethral meatus. There are no other genital lesions or abnormalities.
Of the following, the MOST appropriate next step in diagnosis and management would be to
order an

A. urethral swab for chlamydia (CT) culture and await results


B. urethral swab for gonorrhea (GC) culture and treat before results available
C. urethral swab for GC-CT nucleic acid amplification testing and await results
D. urine culture and treat him for a urinary tract infection
E. urine specimen for GC-CT nucleic acid amplification testing and await results

American academy of pediatrics 416


American Academy of Pediatrics PREP 2015

Item 135 Preferred Response: E


The symptom combination of dysuria and urethral dis-charge, as seen in the adolescent boy in
the vignette, is diagnostic for urethritis. Occasionally, a male adolescent may have penile itching
and tingling without a discharge; urinary frequency, urgency, or rarely hematuria may also be
present. The organisms responsible for urethritis include those that cause urinary tract infections
and those that may be sexually transmitted. Almost half of all cases are caused by sexual
transmission of Chlamydia trachomatis, often referred to as nongonococcal urethritis (NGU).
Usually, C trachomatis urethritis presents with a scant clear discharge; however, infection may
vary from being asymptomatic to causing a profuse purulent discharge, as is seen with Neisseria
gonorrhoeae infections. The frequency of C trachomatis urethritis reported cases is almost 3
times that of N gonorrhoeae urethritis cases. Both chlamydia and gonorrhea are reportable
diseases, and the rate of partner notification and treatment maybe improved with a specific
diagnosis, therefore it is recommended that testing for these 2 organisms be performed in all
symptomatic patients. This is best accomplished with a nucleic acid amplification test performed
on a urine specimen, for both C trachomatis and N gonorrhoeae. This test has a high sensitivity
and specificity, therefore there is no longer a need for urethral swabs. Other organisms
implicated in urethritis include Ureaplasma urealyticum (20%-30% of NGU cases), Mycoplasma
genitalium, Gardnerella vaginalis, herpes simplex virus, Trichomonas vaginalis, Staphylococcus
saprophyticus, and Escherichia coli. Urinary tract infections are less common in the adolescent
years and are not associated with a urethral discharge in either gender.

Treatment goals for urethritis include symptom relief, preventing transmission to sexual partners
and reducing complications. Untreated C trachomatis may lead to epididymitis, and rarely,
prostatitis or Reiter syndrome. Unless the patient is not likely to return, treatment should await
test results. Empiric treatment is with azithromycin 1 g orally as a single dose for C trachomatis
and ceftriaxone 250 mg intramuscularly for N gonorrhoeae. Patients should be instructed to
abstain from sexual intercourse for 1 week after single dose therapy (to prevent sexual
transmission) and until all partners are treated (to prevent reinfection). The patient should be
tested for other infections, including syphilis and HIV. Counseling should emphasize the proper
use of condoms and the need for their consistent use.

PREP Pearls
• Dysuria and urethral discharge are the most common symptoms of urethritis in male
adolescents.
• Chlamydia trachomatis is the most common organism causing urethritis in male
adolescents.
• It is important to treat the sexual partners of patients with C trachomatis or Neisseria
gonorrhoeae urethritis.

American Board of Pediatrics Content Specification(s)


• Recognize the clinical findings associated with urethritis in male adolescents
• Plan the appropriate evaluation and management of urethritis in male adolescents

Suggested Reading
• Brill JR. Diagnosis and treatment of urethritis in men. Am Pam Physician.
2010;81(7):873-878. Accessed February 21, 2014.
American academy of pediatrics 417
American Academy of Pediatrics PREP 2015

• Chandran L, Boykan R. Chlamydial infections in children and adolescents. Pediatr Rev.


2009;30(7):243-250. doi:10.1542/pir.30-7-243.
• Fortenberry JD, Neinstein LS. Overview of sexually transmitted diseases. In: Neinstein
LS, Gordon CM, Katzman DK, Rosen DS. Woods ER, eds. Adolescent Health Care: A
Practical Guide. 5th ed. Philadelphia, PA: Lippincott Williams & Wilkins; 2008:767-770.
• Marcell AV, Wibbelsman C, Seigel WM, the Committee on Adolescence. Male
adolescent sexual and reproductive health care. Pediatrics. 2011;128(6):e1658-e1676.
doi:10.1542/peds.2011-2384.
• National Center for HIV/AIDS, Viral Hepatitis, STD, and TB Prevention, Division of
STD Prevention. Diseases characterized by urethritis and cervicitis. US Centers for
Disease Control and Prevention website. Updated January 28, 2011. Accessed February
21, 2014.
• Workowski KA, Berman S. Sexually transmitted diseases treatment guidelines 2010.
MMWR. 2010;59(RR-12):40-43. Accessed February 21, 2014.

American academy of pediatrics 418


American Academy of Pediatrics PREP 2015

Item 136
During a wellness visit, the parents of a 1-year-old girl with eczema report that their daughter
develops a facial rash after eating different foods. She developed hives around her mouth and
worsening of her eczema after eating scrambled eggs at 10 months of age. She was noted to
develop a blotchy rash around her mouth when she ate fresh strawberries, but tolerated
strawberry-flavored yogurt. She also gets a similar blotchy rash when she eats French fries and
ketchup, but not with mashed potatoes. Her physical examination is unremarkable, except for dry
skin around her mouth and on her cheeks, and a pea-sized, dry patch on her chin.

Of the following, the MOST appropriate recommendation to the parent is to have the child
A. be evaluated for mastocytosis
B. begin a trial of diphenhydramine before suspected triggers
C. begin low potency corticosteroid cream on the face
D. stop all egg and egg products
E. stop egg, tomato, strawberry, and potato

American academy of pediatrics 419


American Academy of Pediatrics PREP 2015

Item 136 Preferred Response: D


The child in the vignette has atopic dermatitis and possible food allergy to egg. She has a history
of developing allergic urticaria and worsening of her eczema with ingestion of egg. She should
therefore avoid exposure to egg and egg products, and undergo allergy testing to further evaluate
the suspicion of food allergy to egg. Recently published National Institute of Allergy and
Infectious Diseases Food Allergy guidelines suggest that children younger than 5 years of age
with moderate to severe atopic dermatitis (AD) be considered for evaluation for a food allergy
trigger if at least 1 of the following conditions is met: persistent AD, despite optimized
management and topical therapy, or reliable history of an immediate reaction after ingestion of a
specific food. Based on the history, it is possible that this child has an egg allergy, exacerbating
the eczema. Triggers of itching in patients with atopic dermatitis are shown (C136, page C-118).

Many children with AD have sensitive skin that reacts on contact with foods that can cause
irritation. These foods include fresh fruit juices such as strawberry, citrus fruits, ketchup, pasta
sauce, soy sauce, and similarly strong flavored and acidic substances. These contact reactions
can be distinguished from true food allergy in that the suspected food is tolerated if a barrier
moisturizer is applied to prevent direct contact, or if the food can be tolerated in other
preparations. The child in the vignette can tolerate strawberry-flavored yogurt, but has a contact
reaction when strawberry touches the skin around the mouth. She does not have an allergic
reaction to potatoes because she can eat mashed potatoes without any reaction. The ketchup,
along with the French fries, is causing her irritant contact reaction around her mouth. Many
children with a similar history of having local reactions to strongly flavored tomato sauces are
able to tolerate tomato in cooked dishes, thereby distinguishing it from a true immunoglobulin E-
mediated food allergy. As the AD improves and the child's skin gets less sensitive with age, they
are able to tolerate these sauces. While diphenhydramine is helpful in mitigating itching in AD,
its use prior to ingestion of suspected food allergy trigger is not advisable because it could
potentially mask the symptoms of allergy. Use of low potency steroid cream on the face would
lessen the amount of inflammation around the mouth, and might decrease the likelihood and
severity of contact urticaria or dermatitis, but will not alter the course of allergic sensitization to
the food.

Mastocytosis refers to a group of rare disorders of excessive mast cell proliferation and
accumulation limited to the skin (cutaneous mastocytosis), or involving bone marrow and other
extracutaneous tissues (systemic mastocytosis). Children are usually affected by the cutaneous
form of mastocytosis, with most improving or resolving by adolescence. An evaluation for a
mast cell disorder is appropriate in a patient with features of mast cell activation, such as
flushing, tachycardia, diarrhea, fatigue or musculoskeletal pain, hypotensive syncope, or near
syncope affecting at least 2 organ systems. Patients may also present with recurrent and
unexplained episodes of anaphylaxis. The physical examination must include a careful skin
inspection to look for lesions of urticaria pigmentosa (small yellow-tan to reddish-brown
macules, slightly raised papules or nodules, or plaque-like lesions that urticate or flush when
rubbed, Darier sign), or mastocytomas (nodules and bullae). The presence of Darier sign is
pathognomonic for the presence of mast cells within the lesion. However, lesions consistent with
mastocytomas (localized accumulations of mast cells) should not be rubbed or scratched, as this
may lead to generalized flushing and hives. Instead, historical verification of the occurrence of

American academy of pediatrics 420


American Academy of Pediatrics PREP 2015

these symptoms, if the lesion is disturbed, should be attempted. This child does not have
symptoms or signs (absence of skin findings) to suggest further evaluation of mastocytosis.

PREP Pearls
• Children younger than 5 years of age with moderate to severe atopic dermatitis (AD)
should be considered for evaluation of food allergy if there is a history of an immediate

American academy of pediatrics 421


American Academy of Pediatrics PREP 2015

reaction after ingestion of a specific food, or if the AD is hard to control despite


optimized management.
• Triggers of itching in patients with AD can include foods acting as contact irritants,
vasodilators, or allergens.
• Mastocytosis should be suspected in children with yellow-brown macules that urticate on
stroking, or if there are unexplained, recurrent episodes of spontaneous urticarial or
allergic symptoms associated with symptoms involving multiple organ systems.

American Board of Pediatrics Content Specification(s)


• Recognize the relationship between eczema and food allergies, and how to evaluate a
patient for both

Suggested Reading
• Boyce JA, Assa'ad A, Burks AW, et al. Guidelines for the diagnosis and management of
food allergy in the United States: report of the NIAID-sponsored expert panel. )Allergy
Clin Immunal. 2010;126(6 Suppl):S1-S58. doi:10.1016/j.jaci.2010.10.007.
• Brockow K, Akin C, Huber M, Metcalfe DD. Assessment of the extent of cutaneous
involvement in children and adults with mastocytosis: relationship to synsptomatology,
tryptase levels, and bone marrow pathology. I Am Acad Dermatol. 2003;48(4):508-516.
doi:10.1067/ mjd.2003.98.
• Schneider L, Tilles S, Lio P, et al. Atopic dermatitis: a practice parameter update 2012. I
Allergy Clm Immunol. 2013:131(2);295-299.e1-e27. doi:10.1016/j.jaci.2012.12.672.

American academy of pediatrics 422


American Academy of Pediatrics PREP 2015

Item 137
A 3-month-old male infant is brought to the emergency department by his mother because of
vomiting, difficulty breathing, and fussiness. He was in his usual state of good health in the
morning when she left for work. When she returned, the baby was not interested in taking the
bottle, and he vomited what little he took. She was also concerned with his breathing pattern,
which was faster than usual.
Vital signs show a temperature of 37°C, heart rate of 120 beats/min, respiratory rate of 50
breaths/min, and blood pressure of 80/40 mm Hg. Physical examination reveals a fussy infant
with a high-pitched cry. He is moving all extremities equally. Pupils are 4 mm, equal, and
sluggishly reactive. Anterior fontanelle is full. Cardiovascular examination reveals regular rate
and rhythm, and warm and well-perfused extremities, with capillary refill of 2 s. Lung
examination shows a breathing pattern that is fast and deep, is clear to auscultation, and has good
air movement bilaterally. Abdomen is soft, nontender, nondistended, and with no organomegaly.
Skin examination is unremarkable.

Of the following, the procedure MOST likely to reveal the diagnosis is


A. abdominal radiograph
B. chest radiograph
C. computed tomography scan of head
D. electrocardiogram
E. lumbar puncture

American academy of pediatrics 423


American Academy of Pediatrics PREP 2015

Item 137 I-C Preferred Response: C


The infant in this vignette has vomiting, fussiness, a high-pitched cry, a full fontanelle, and
hyperventilation. These findings are most likely caused by an inflicted traumatic brain injury
(iTBI). The cause is most likely inflicted because other accidental mechanisms for TBI are
extremely uncommon in this age group, especially in the absence of a plausible history. The best
diagnostic test to reveal the diagnosis is a computed tomography (CT) scan of the head.
The most common cause of death from child abuse is iTBI, also known as abusive head trauma,
nonaccidental head trauma, or shaken baby-impact syndrome. In the first year after birth, head
injuries that do not have a witnessed plausible history are almost universally from abuse because
infants in that age group have not attained the developmental level to sustain an accidental
injury. Common signs and symptoms of iTBI include lethargy, irritability, vomiting, apnea, and
hyperventilation. The hyperventilation is a compensatory mechanism to reduce intracranial
pressure. The presentation may include other signs of abuse such as bruises, grab marks, or rib
fractures, but isolated head injury is very common. Mechanisms of iTBI in infancy include
tearing of the subdural bridging veins from rapid acceleration-deceleration caused by shaking
and direct impact to the head.

Most of the signs and symptoms of the presentation of a child with iTBI are nonspecific, such as
irritability, disordered breathing, vomiting, and disinterest in feeding. The differential diagnosis
for fussiness in this age group includes sepsis, meningitis, hydrocephalus, congenital heart
disease, corneal abrasion, and hair tourniquet. Colic can also be a cause of fussiness, but
generally occurs around 4 to 6 weeks of age, and is a diagnosis of exclusion. A high index of
suspicion for iTBI must be maintained because of a nonspecific presentation and a large
differential diagnosis. Not only is it a serious condition, but missing the diagnosis can also render
the patient and other children in the house vulnerable to subsequent abuse.

The radiographic test of choice for acute iTBI is CT (Item C137A, Item C137B). Subdural,
epidural, subarachnoid, and intraparenchymal hemorrhages can all occur in iTBI and can be
detected on CT. Injuries of various ages can be seen if the abuse has occurred over time. Head
ultrasonography can be performed in a child with an open fontanelle to detect bleeding and
hydrocephalus, but it is not as sensitive as CT. Magnetic resonance imaging is very sensitive and
specific in detecting intracranial pathology. Magnetic resonance imaging requires the patient to
be motionless for a long duration and thus requires deep sedation and usually endotracheal
intubation, and it is not as practical as CT in the acute setting in this age group. Neurosurgery
and critical care consultations should be obtained for management of head injury. Evaluation for
other signs of trauma is also necessary. An ophthalmology consultation is warranted to evaluate
for retinal hemorrhages (Item CI37C). Although retinal hemorrhages can be seen in other
conditions, injury to more than 1 layer of the retina and traumatic retinoschisis are
pathognomonic of abuse, especially in the context of other signs of abuse. A skeletal survey
should be obtained to evaluate for recent and old fractures. Rib fractures are commonly
associated with shaking injuries. A trauma and surgery consultation should also be considered if
there is suspicion of non-neurologic injury.

American academy of pediatrics 424


American Academy of Pediatrics PREP 2015

American academy of pediatrics 425


American Academy of Pediatrics PREP 2015

Abdominal radiography can be part of the workup for vomiting and irritability, but is not likely
to contribute to a diagnosis of trauma. Chest radiography can reveal rib fractures, but are
sometimes not seen in iTBI. Electrocardiography can be helpful in detecting myocardial
infarction in anomalous left coronary artery from pulmonary artery, which is a condition that can
cause irritability in this age group, but occurs much less frequently than iTBI. Lumbar puncture
can be part of the workup of intracranial pressure, but only after obstructive hydrocephalus has
been ruled out with CT.

Inflicted traumatic brain


injury is a condition that can
manifest with nonspecific
signs and symptoms in
infancy, including fussiness,
altered mental status,
vomiting, and breathing
difficulties. Computed
tomography of the head is
the initial diagnostic study of
choice. Retinal, chest,
skeletal, and abdominal
trauma should also be
evaluated. Police, social
work, and child protection
medical teams should also be
involved to investigate the
crime and protect others.

PREP Pearls
• Inflicted traumatic brain injury should be considered in the differential diagnosis of an
infant with irritability or altered mental status. Computed tomography of the head is the
diagnostic study of choice.
• Inflicted traumatic brain injury can be isolated without any other obvious signs of trauma.
• Infants younger than 1 year of age rarely have an accidental cause of traumatic brain
injury without a plausible history.

American Board of Pediatrics Content Specification(s)


• Recognize the clinical findings associated with non-accidental traumatic brain injury

Suggested Reading
• Dubowitz H, Lane WG. Abused and neglected children. In: Kliegman RM, Stanton BF,
St. Game JW III, Schor NE. Behrman RE eds. Nelson's Textbook of Pediatrics. 19th ed.
Philadelphia, PA: Saunders Elsevier; 201/:329.
• Sirotnak AP, Grigsby T, Krugman RD. Physical abuse of children. Pediatr Rev.
2004125(8)1264-277. doi:10.1542/pir.25-5-264.

American academy of pediatrics 426


American Academy of Pediatrics PREP 2015

Item 138
A previously well, 56-day-old female infant arrives at your office with a 1-day history of fever
(up to 38.9°C). The parents deny cough, congestion, nausea, rash, skin lesions, vomiting, and
diarrhea. The baby lives with her parents and 2 siblings who are 2 years of age and 5 years of
age. No one else at home is ill. The baby is being fed formula 1.5 oz to 2 oz every 2 to 3 hours
(down from her usual 2.5 oz to 3 oz per feed).

You review the baby's birth history and see that her birth weight was 3.3 kg; her birth was a full-
term, uncomplicated pregnancy and delivery; her nursery stay was without complications; and
she went home in 2 days. She has no prior illnesses. She received hepatitis B vaccine in nursery.
The family has been in your practice since the birth of the oldest child and has kept appointments
reliably.

The baby's vital signs show a temperature of 38.8°C, a heart rate of 130 beats/min, and a
respiratory rate of 24 breaths/ min. Physical examination shows a strong cry, but she is
consolable by her mother. She is comfortable in her mother's arms and well appearing. The
remainder of the physical examination findings are unremarkable.

Laboratory results show the following:


• White blood cell count, 12,800/µL (12.8 x 109/L), with 45% neutrophils, 3% bands; 49%
lymphocytes; 3% monocytes
• Hemoglobin, 13.4 g/dL (134 g/L)
• Platelet count, 227 x 103/ µL (227x 109/L)

A urinalysis shows the following:


• Specific gravity, 1.017
• pH, 7.0
• Dipstick, negative
The blood and urine cultures are pending.

Of the following, the BEST next step in the management of this infant's condition would be to
A. admit and observe pending cultures
B. admit and treat with ampicillin and cefotaxime pending culture results
C. discharge home with follow-up in 24 hours
D. perform a lumbar puncture and begin ampicillin and ceftriaxone pending culture results
E. prescribe amoxicillin and follow-up in 24 hours

American academy of pediatrics 427


American Academy of Pediatrics PREP 2015

Item 138 Preferred Response: C


The risk of serious bacterial infection (SBI) in a well-appearing infant between 30 and 90 days of
age with an unremarkable examination, normal complete blood cell count, and negative
urinalysis, as described for the baby in this vignette, is low. When a baby meets these criteria and
there is an expectation as noted in the vignette of good follow-up, the patient can be safely sent
home with follow-up in 24 hours.

In the absence of concerns regarding the baby's returning for serial evaluations, persistent fever,
or the baby getting sicker, there is potential risk and no clear advantage in hospitalizing a baby
for observation in this setting. Initiation of antibiotic therapy (intravenous or oral) without
identifying a focus of infection, given the unremarkable examination and laboratory parameters
described for the baby in this vignette, is not indicated. Although some studies suggest
performing a lumbar puncture (LP) as part of the evaluation of all young infants younger than 90
days of age with fever, this is presently controversial. Regardless, just performing a LP in this
setting would not be an indication to begin treatment with antibiotics in a well-appearing infant.
In fully vaccinated, well-appearing children between 90 days to 24 months of age, clinical
examination is more reliable than in younger infants. In these patients, the risk of invasive
bacterial infection is low enough that observation alone or screening for infection with a
complete blood cell count and urinalysis are appropriate options.

These recommendations only apply to a well-appearing infant with fever. Lethargy, poor
feeding, excessive irritability, or toxic appearance should prompt assessment and treatment for
sepsis or other invasive bacterial infections.

PREP Pearls
• The risk of serious bacterial infection is low in febrile infants 30 to 90 days of age who
are well appearing, with a normal physical examination, complete blood cell count, and
urinalysis. Observation with close follow-up would be appropriate for these patients.
• Lethargy, poor feeding. excessive irritability, or toxic appearance should prompt
assessment and treatment for sepsis or other invasive bacterial infection.

American Board of Pediatrics Content Specification


• Plan the appropriate evaluation and management of fever without source in patients of
various ages

Suggested Reading
• Baraff Li. Management of infants and young children with fever without source. Pedro&
Ann. 2008;37(10):673-679.
• Iduppler AR, Eickhoff JC, Wald ER. Performance of low-risk criteria in the evaluation of
young infants with fever: review of the literature. Pediatrics. 2010;125(2):228-233.
doi:10.1542/peds.2009-1070.
• Jhaveri R, Byington CL, Klein JO, Shapiro ED. Management of the nontoxic-appearing
acutely febrile child: a 21st century approach. I Pecliatr. 2011;159(2):181-185.
doi:10.1016/jjpeds.2011.03.047.
• Trainor JL, Stamos JK, Fever without a localizing source. Pediatr Ann.
• 2011;40(1):21-25. doi:10.3928/00904481-20101214-06.
American academy of pediatrics 428
American Academy of Pediatrics PREP 2015

• Waddle E, Jhaveri R. Outcomes of febrile children without localizing signs after


pneurnococcal conjugate vaccine. Arch Dis Child. 2009;9412):144-147.
doi:10.1136/adc.2007.130583.

American academy of pediatrics 429


American Academy of Pediatrics PREP 2015

Item 139
A 7-year-old boy presents to your office for a health supervision visit. He is in second grade and
doing well at school. His family history is significant for several male members who have
deafness and renal failure occurring between the ages of 16 and 28 years (Item Q139). His vital
signs show a temperature of 37.7°C, heart rate of 80 beats/min, respiratory rate of 16
breaths/min, and blood pressure of 112/60 mm Hg. He has unremarkable findings on physical
examination. His mother is worried about potential kidney problems in her son.

Of the following, the MOST common early finding in this familial condition is
A. anemia
B. elevated creatinine level
C. hematuria
D. hydronephrosis
E. hypertension

ITEM Q139: Pedigree for the family described in the vignette

American academy of pediatrics 430


American Academy of Pediatrics PREP 2015

Item 139 Preferred Response: C


Alport syndrome (AS) is an inherited disorder of basement membrane collagen characterized by
involvement of the kidneys (always), ears (often), and eyes (occasionally). Collagen IV, the
major structural component of all basement membranes, is composed of heterotrimers
(protomers) of a chains. Genes encoding for the chains have been identified on the chromosomes
13q34 (COL4AI, COL4A2), 2q35 (COL4A3, COL4A4), and Xq22 (COL4A5, COL4A6). Gene
defects of the chain lead to abnormal protein assembly, leading to the characteristic
abnormalities of AS. Mutations in COL4A5 result in X-linked dominant AS, the most common
mode of inheritance characterized by affected male family members and asymptomatic female
carriers (Item C139, page C-I 21). COL4A3 and COL4A4 mutations lead to autosomal recessive
and dominant AS, respectively, with the disease having similar severity in male and female
patients.

Hematuria is the earliest manifestation and the universal symptom in patients with AS. It is
usually detected in childhood or even at birth. Seventy-four percent of patients have hematuria
by the age of 6 years. Patients with AS may present with gross/macroscopic hematuria or
asymptomatic microscopic hematuria. Male patients fare poorly, with progression to renal failure
either by age 20 years (juvenile type) or 40 years or later (nonprogressive or adult type).
Progressive renal failure associated with AS is characterized by later onset of increasing
proteinuria, elevated serum creatinine, and the development of hypertension and anemia.
Important extrarenal manifestations of AS include hearing loss and lenticonus. Bilateral
sensorineural hearing loss (initially high frequency and progressively involving lower
frequencies) develops over the first decade, with boys being more severely affected.
Patients in early stages of AS usually have a normal renal ultrasound with no evidence of
hydronephrosis, obstruction, or renal parenchymal abnormality. As renal injury progresses, the
renal parenchyma may show increased echogenicity consistent with medicorenal disease
(nonspecific changes in acute or chronic renal failure).

PREP Pearls
• Hematuria is the earliest manifestation and universal symptom in patients with Alport
Syndrome (AS).
• X-linked dominant is the most common mode of inheritance of AS. Male patients have a
worse prognosis with invariable progression to renal failure, whereas female patients are
usually asymptomatic carriers and rarely develop renal failure.
• Bilateral sensorineural hearing loss in AS develops over the first decade.
• Renal ultrasonography is usually normal in patients in the early stages of AS.

American Board of Pediatrics Content Specification(s)


• Recognize the renal findings associated with Alport syndrome

Suggested Reading
• Fouser L. Consultation with the specialist: familial nephritis/Alport syndrome. Pediatr
Rev. 1998;19:265-267. doi:10.1542/pir.19-8-265.
• Varade WS. Nephritis. In: American Academy of Pediatrics Textbook of Pediatric Care.
19th ed.; McInerny TK, Adam HM, Campbell DE, Kamat DP, Kelleher KJ, eds., Elk
Grove Village, IL:, American Academy of Pediatrics; 2009; chap 296:2291-2292.
American academy of pediatrics 431
American Academy of Pediatrics PREP 2015

Item 140
A 15-year-old adolescent girl presents to your office with the complaint of periumbilical
abdominal pain. This symptom has persisted for the past year, occurring several times per week
at any time of day. The patient denies nocturnal symptoms, weight loss, diarrhea, constipation,
and vomiting. Over the past several weeks, she feels "bloated" after meals and often experiences
postprandial nausea. During the week, she skips breakfast, has an energy bar for lunch, and eats a
large dinner at 8:30 in the evening after returning from swimming practice. She has tried her
father's antacids, and these have provided minimal relief. Height and weight are at the 40th
percentile for age. She is afebrile with other vital signs within the reference range. Physical
examination demonstrates a well-developed, well-nourished adolescent with no abnormal
physical findings.

Of the following, you are MOST likely to recommend


A. changing her eating behavior
B. a Helicobacter pylori antibody titer
C. referral for an upper gastrointestinal tract endoscopy
D. a trial with a proton-pump inhibitor
E. an upper gastrointestinal tract series

American academy of pediatrics 432


American Academy of Pediatrics PREP 2015

Item 140 Preferred Response: A


Although the exact prevalence of chronic abdominal pain in children and adolescents is not
known, a review of available data suggests that this complaint accounts for up to 4% of office
visits to the primary care pediatrician. One study in adolescents reported that 8% of high school
students sought evaluation for abdominal pain during a 1-year period. In the vignette, a 15-year-
old girl presents with dyspeptic symptoms of upper abdominal discomfort, bloating, and
intermittent nausea. She does not manifest any alarming signs or symptoms that would suggest
an organic cause for her complaints (Item C140). Considering her irregular and "unbalanced"
dietary habits, a reasonable initial therapeutic approach would be to encourage a change in her
eating behavior.

Item C140. Warning Signs Indicating a Higher Probability of Organic Disease in Children with
Chronic Abdominal Pain
 Abnormal or unexplained physical findings
 Chronic diarrhea
 Deceleration in growth velocity
 Family history of inflammatory bowel disease
 Gastrointestinal blood loss (gross or occult)
 Symptoms that awaken the patient
 Persistent right upper or right lower quadrant abdominal pain
 Unexplained fevers
 Vomiting
 Weight loss

Children with functional (ie, nonorganic) abdominal pain report symptoms that include isolated
abdominal pain without other physical complaints, dyspepsia, irritable bowel syndrome, or
abdominal migraine. In other patients, symptom clusters include clinical features of more than 1
type of functional pain disorder (eg, Patients 12 years of age or older with symptoms for at least
12 weeks (not necessarily consecutive) during the previous 12 months.
1. Persistent pain or discomfort in the upper abdomen.
2. No evidence that organic disease is likely explaining symptoms.
3. Dyspepsia is not exclusively relieved by defecation or associated with the onset of a
change in stool frequency or character.

For the adolescent in the vignette, with her absence of warning signs, no evidence suggests that
additional laboratory studies are of value in her evaluation. The predictive value of blood tests in
functional abdominal pain (with or without warning signs) has not been adequately studied.
Infection with Helicobacter pylori should be considered for patients with a positive family
history or for those who have emigrated from an endemic area. The H pylori antibody titer is an
unreliable and nonspecific test to evaluate H pylori-associated gastroduodenal disease, and
endoscopy with biopsy remains the gold standard for diagnosis in pediatric patients. In the
absence of a history (ie, family or immigration) suggesting H pylori infection or the presence of
warning signs (Item C140), little evidence suggests that a radiographic study or an endoscopy
with biopsy has a significant yield of mucosal abnormalities. Empiric acid suppression treatment

American academy of pediatrics 433


American Academy of Pediatrics PREP 2015

with a proton pump inhibitor has not been adequately studied in children with a diagnosis of
functional dyspepsia and no evidence-based guidelines for treatment have been established.

PREP Pearls
• Dyspepsia is defined as upper abdominal discomfort with or without symptoms of
nausea, bloating, or borborygmi (belching). It is usually not associated with a significant
underlying acid-peptic disorder.
• The Helicobacter pylori antibody titer is not recommended as a screening test for children
with dyspepsia.
• Blood tests have not been shown to be of value in assessing the child with functional
abdominal pain.
• Patients with functional abdominal pain may present with complaints isolated to the
gastrointestinal tract, or they may describe other pain symptoms.

American Board of Pediatrics Content Specification(s)


• Recognize the significance of dyspepsia in a child with recurrent abdominal pain

Suggested Reading
• American Academy of Pediatrics Subcommittee on Chronic Abdominal Pain. Chronic
abdominal pain in children. Pediatrics. 2005;115(3):812-815. doi:10.1542/peds.2004-
2497.
• Di Lorenzo C, Colletti RB, Lehmann HP, et al. Chronic abdominal pain in children: a
technical report of the American Academy of Pediatrics and the North American Society
for Pediatric Gastroenterology, Hepatology and Nutrition. J Pediatr Gastroenterol Nutr.
2005;40(3):249-261. Huertas-Ceballos AA, Logan 5, Bennett C, Macarthur C. Dietary
interventions for recurrent abdominal pain (RAP) and irritable bowel syndrome (IBS) in
childhood. Cochrane Database Syst Rev. 2009;(1):CD003019.
doi:10.1002/14651858.CD003019.pub3. Hyams JS, Davis P, Sylvester FA, at al.
Dyspepsia in children and adolescents: a prospective study. J Pediatr Gastroenterol Nutr
2000; 30:413418.
• Rasquin A, Di Lorenzo C, Forbes D, et al. Childhood functional gastrointestinal
disorders: child/adolescent. Gastroenterology. 2006; 130(5):1527-1537.
doi:10.1053/j.gastro.08.063.

American academy of pediatrics 434


American Academy of Pediatrics PREP 2015

Item 141
A 2-day-old neonate is transferred to your neonatal intensive care unit (NICU) with altered
mental status and seizures. He was delivered at 39 weeks' gestation via spontaneous vaginal
delivery without complications to a 37-year-old mother-gravida 4, now para 2-with
unremarkable prenatal laboratory test results. His growth parameters are appropriate for
gestational age. At 28 hours of age, the patient began to develop deep, rapid breathing that
progressed within 8 hours to respiratory distress with central nervous system excitation (patient
couldn't fall asleep). A sepsis workup was initiated at the outside hospital with a hazy chest
radiograph, normal complete blood cell count, liver function tests, and C-reactive protein. Blood,
urine, cerebrospinal fluid cultures, and viral studies were sent, and the child has been started on a
regimen of ampicillin, gentamicin, and acyclovir. Arterial blood gases show a respiratory
alkalosis.

One hour later at the outside hospital, he developed seizure-like activity with tonic–clonic
movement of all 4 extremities and desaturations as low as 80%. A loading phenobarbital dose
was given. An ammonia level was obtained and was 1,613 µg/dL (1,152 µmol/L), with
worsening of his neurologic status. He was intubated due to obtundation and is now hypotonic
and hyporeflexic. Transfer to a tertiary NICU was initiated.

Upon arrival at the NICU, the initial laboratory results show the following:
Electrolytes:
• Sodium,145 mEq/L (145 mmol/L)
• Potassium, 4.2 mEq/L (4.2 mmol/L)
• Chloride, 112 mEq/L (112 mmol/L)
• Bicarbonate, 20 mEq/L (20 mmol/L)
• Blood urea nitrogen, 6 mg/dL (2.1 mmol/L)
• Creatinine, 0.9 mg/dl. (80 µmol/L)
• Glucose, 112 mg/dL (6.2 mmol/L)
• Anion gap, 13 mEq/L (13 mmol/L)
• Total bilirubin, 6.0 mg/dL (102.6 umol/L) Serum amino acids:
• Glutamine, 49.3 mg/dL (3,376.0 µmol/L); reference range, 474.0 to 737.0 µmol/L
• Alanine, 4.6 mg/dL (516 µmol/L); reference range, 182.0 to 398.0 µmol/L
• Citrulline, undetectable; reference range, 14.0 to 32.0 umol/L
• Ornithine, 0.3 mg/dL (23 umol/L); reference range, 25.0 to 103.0 umol/L
• Proline, 4.0 mg/dL (347 umol/L); reference range, 93.0-264.0 µmol/L
• Urine orotic acid, 24 mmol/mol creatinine; reference range, 1.4 to 5.3 mmol/mol
creatinine

Upon arrival at the NICU, the mother reveals that there is a positive family history for a maternal
uncle with early neonate death at 2 weeks of age from respiratory distress. The family's ethnicity
is European Caucasian. Consanguinity is denied. The mother also has avoided meat and milk
products for the majority of her life.

American academy of pediatrics 435


American Academy of Pediatrics PREP 2015

Of the following, the MOST likely diagnosis is


A. carbarnoyl phosphate synthetase I deficiency
B. galactosemia
C. methylmalonic acidemia
D. ornithine transcarbamylase deficiency
E. tyrosinemia type 1

American academy of pediatrics 436


American Academy of Pediatrics PREP 2015

Item 141 Preferred Response: D


The male neonate in the vignette has a classic presentation of a urea cycle disorder of which
ornithine transcarbamylase (OTC) deficiency is one type. These disorders are caused by
mutations resulting in the absence or partial functioning of 1 of the first 4 enzymes in the urea
cycle biochemical pathway responsible for the breakdown of nitrogen, a waste byproduct of
protein catabolism in the body. In a normal individual, nitrogen is broken down into urea and
excreted through the urine. If nitrogen levels build up in the body because of inefficient
breakdown, it accumulates quickly in the form of ammonia. Hyperammonemia is quite toxic to
the brain and can cause irreversible damage without immediate intervention. Neonates will
appear normal at birth, but within 48 to 72 hours they will develop cerebral edema. This typically
manifests as poor feeding, obtundation, hypothermia, seizures, hyperventilation, hyporeflexia,
unusual posturing, and ultimately, coma. Classic laboratory findings include elevated ammonia
levels (> 210 µg/dL [150 µmol/L]) with a normal anion gap and normal glucose level in the
presence of respiratory alkalosis on blood gas measurement. To distinguish between the specific
types of urea cycle defects, one must order a plasma amino acid analysis and a urine orotic acid.
Ornithine transcarbamylase deficiency is associated with extremely high urinary orotic acid
levels. Carbamoyl phosphate synthetase I (CPS I) deficiency is associated with low to
undetectable urine orotic acid levels, thus excluding CPS I deficiency. Definitive diagnosis is
dependent on either enzymatic analysis or molecular genetic testing of the genes involved. All of
the urea cycle disorders are autosomal recessive in inheritance except OTC, which is X-linked
recessive. The patient in the vignette has classic OTC deficiency inherited from his mother. The
mothers often complain of protein or meat intolerance. Of carrier females, 15% can develop
hyperammonemia in times of stress, such as pregnancy and delivery, and could potentially
require chronic lifetime management for periods of hyperammonemia. A family history of early
infant death, presumably because of hyperammonemia, may also be seen.

Treatment of urea cycle disorders is a metabolic emergency necessitating immediate recognition


to avoid irreversible brain damage. Severe hyperammonemia is treated with dialysis and
hemofiltration to rapidly reduce the plasma ammonia concentration, along with the intravenous
administration of arginine hydrochloride and nitrogen scavenger drugs to promote the excretion
of excess nitrogen through alternative pathways. Restriction of protein for 12 to 24 hours is
essential, with calories provided through carbohydrates and fat. Care must be taken to stabilize
the patient with intravenous fluids and inotropic drugs if necessary.

Long-term management mandates the use of specialized formulas, oral nitrogen-scavenging


drugs, dietary restriction of protein, and avoidance of hyperammonemic episodes. Patients are at
high risk of decompensation, necessitating hospitalization for close observation of clinical status
and ammonia levels if they have gastrointestinal and respiratory illnesses. Most patients are
routinely treated on a long-term basis by biochemical or metabolic geneticists, in addition to
their primary care provider. They should have emergency protocols in place at home, at the
primary care provider's office, and at the local hospital.

Methylmalonic acidemia is an organic acid disorder that commonly presents with elevated
ammonia levels, high ketone levels, and a high anion gap metabolic acidosis (> 20). Elevated
plasma and urine concentrations of methylmalonic acid in the absence of hyperhomocysteinemia
or homocystinuria are diagnostic. Other common laboratory findings are neutropenia and
American academy of pediatrics 437
American Academy of Pediatrics PREP 2015

thrombocytopenia. During infancy or in the neonatal period, patients can present with lethargy,
low tone, hypothermia, respiratory impairment, vomiting, dehydration, and obtundation. This is
another metabolic emergency.

Tyrosinemia type I presents in infancy with significant liver involvement and eventually renal
tubular dysfunction, growth failure, and rickets. Untreated children may present with repeated
neurologic crises involving a change in mental status, peripheral neuropathy, abdominal pain,
and occasionally respiratory failure. If untreated, many die before the age of 10 years.
Laboratory abnormalities include increased succinylacetone concentration in the blood and urine;
elevated tyrosine, methionine, and phenylalanine on serum amino acids; and elevated tyrosine
metabolites on urine organic acids.

Galactosemia presents in the neonatal period with jaundice, hypotonia, sclera' icterus, bruising,
bleeding, and cataracts in the face of rapidly progressive liver failure. Classic laboratory
abnormalities of galactosemia include positive urine-reducing substances, abnormal liver
function studies, coagulation abnormalities suggestive of a progressive bleeding diathesis, and a
culture workup for sepsis, especially due to Escherichia tail.

Specific urea cycle disorders include N-acetyl glutamate synthetase deficiency, CPS I deficiency,
OTC deficiency, citrullinemia type I, argininosuccinic aciduria, and arginase deficiency.

PREP Pearls
• Urea cycle defects are caused by the body's inability to metabolize waste nitrogen
byproducts created by the breakdown of protein and other nitrogen-containing molecules,
resulting in severe hyperammonemia that is quickly toxic to the brain and necessitating
immediate recognition of this metabolic emergency.
• Infants with urea cycle defects will appear normal at birth, but will develop cerebral
edema within 48 to 72 hours. This typically manifests as poor feeding, obtundation,
hypothermia, seizures, hyperventilation, hyporeflexia, unusual posturing, and ultimately,
coma.
• Classic laboratory findings of urea cycle defects include elevated ammonia levels (> 210
µg/dl [150 µmol/L]) with a normal anion gap and normal glucose level in the presence of
respiratory alkalosis on blood gas measurement.
• Severe hyperammonemia is treated with dialysis and hemofiltration to rapidly reduce the
plasma ammonia concentration, intravenous administration of arginine hydrochloride and
nitrogen scavenger drugs to promote the excretion of excess nitrogen through alternative
pathways, and protein restriction with cardiovascular stabilization.

American Board of Pediatrics Content Specification(s)


• Recognize the clinical features associated with urea cycle defects
• Plan the appropriate immediate and long-term management of urea cycle defects, while
considering the long-term prognosis

Suggested Reading
• Lanpher BC, Gropman A, Chapman KA, Lichter-Konecki U. Urea Cycle Disorders
Consortium, Summar ML. Urea cycle disorders overview. GerieReviews.
American academy of pediatrics 438
American Academy of Pediatrics PREP 2015

• Summar M. Current strategies for the management of neonatal urea cycle disorders. J
Pediatr. 2001;138(1 SupplhS30-539.
• Summar M, Tuchman M. Proceedings ofa consensus conference for the management of
patients with urea cycle disorders. J Perham 2001;138(1 suppl):S6-S10.
doi:10.1067/mpd.2001.111831.

American academy of pediatrics 439


American Academy of Pediatrics PREP 2015

Item 142
A 12-year-old girl comes to your clinic for evaluation of bilateral pink eyes, low grade fever, and
ear pain for 2 days. She reports that she had a lot of crusting in the eyelashes when she awoke
this morning. On physical examination, she has minimal clear to mucoid eye drainage and
conjunctival injection, but no photophobia or visual changes. The patient has diffuse pharyngeal
erythema, but no exudate or tonsillar enlargement. Her tympanic membranes are gray with
normal landmarks. You notice some tenderness to movement of the tragus and find pea-sized
tender nodules in the preauricular area bilaterally. The patient is very concerned about these
bumps that she had not previously noticed.

Of the following, the BEST recommendation at this time is to


A. obtain a complete blood cell count
B. obtain Epstein-Barr virus titers
C. prescribe amoxicillin orally
D. reassure and treat symptomatically
E. refer to surgery for biopsy

American academy of pediatrics 440


American Academy of Pediatrics PREP 2015

Item 142 Preferred Response: D


This case shows that not all ear pain stems from the ear itself as the girl in the vignette suffers
from ear pain related to her preauricular lymphadenopathy. Ear pain (otalgia) is one of the most
common complaints of childhood. The differential diagnosis of ear pain is categorized by the
anatomic location from which the pain originates. Primary otalgia is caused by an abnormality of
the ear itself: inflammation, infection, or trauma to the auricle, auditory canal, tympanic
membrane, middle ear, or inner ear. When the ear appears normal on physical examination, as in
the vignette, secondary otalgia should be considered. Secondary pain is referred from another
location and the causes include temporomandibular joint CDC dysfunction, parotitis, pre- or
postauricular lymphadenopathy, pharyngitis, tonsillitis, dental infection, maxillary sinusitis,
facial nerve palsy, or cervical spine injury. A thorough examination would reveal these potential
causes of secondary otalgia. Treatment of the underlying cause will lead to resolution of the ear
pain.

It is not unusual for benign reactive preauricular lymphadenopathy to occur with viral
conjunctivitis, especially that caused by adenovirus, which often presents with conjunctivitis,
fever, pharyngitis, headache, upper respiratory symptoms, and sometimes, gastrointestinal
complaints. This is such a common constellation of symptoms that one should feel comfortable
treating symptomatically and giving reassurance. Further workup, including obtaining a
complete blood cell count or Epstein-Barr virus titers, is not warranted. Occasionally, adenoviral
infections can cause otitis media or pneumonia, but antibiotics should not be prescribed when
viral disease is suspected. Enlarged lymph nodes caused by an acute bacterial infection would be
more tender, feel warm to touch, have overlying erythema, and may be fluctuant on examination.
The preauricular lymphadenopathy in this case is not of unusual size, location, consistency, or
duration, so biopsy is not indicated.

PREP Pearls
• Primary otalgia is caused by an abnormality of the ear itself: inflammation, infection, or
trauma to the auricle, external auditory canal, tympanic membrane, middle ear, or inner
ear.
• When the ear examination findings are normal, consider secondary causes of otalgia or
referred pain: temporomandibular joint dysfunction, parotitis, pre- or postauricular
lymphadenopathy, pharyngitis, tonsillitis, dental infection, maxillary sinusitis, facial
nerve palsy, or cervical spine injury.

American Board of Pediatrics Content Specification(s)


• Understand the possible etiologies of ear pain, including referred pain from other
anatomic sites

Suggested Reading
• Ely JW, Hansen MR, Clark EC. Diagnosis of ear pain. Am Fain Physician.
2008;77(5):621-628. Accessed January 14, 2014.
• Greenes D. Evaluation of earache in children. UpToDate. Available online only for
subscription.

American academy of pediatrics 441


American Academy of Pediatrics PREP 2015

Item 143
A 4-year-old boy is referred to the hospital for the following abnormal complete blood cell
count:
• White blood cell count, 95,000/µL (95 x 109/L), with 10% polymorphonuclear
leukocytes, 25% lymphocytes, 8% monocytes, and 2% eosinophils, 55% blasts
• Hemoglobin, 7.0 g/dL (70 g/L)
• Mean corpuscular volume, 90 µm3 (90 fL)
• Platelet count, 20,000 x 103/ µL (20 x 109/L)

Bone marrow studies confirm the diagnosis of acute lymphoblastic leukemia. His parents give
informed consent to start chemotherapy. On the second day of receiving induction
chemotherapy, the child is noted to have markedly decreased urine output with an elevated
serum creatinine level.

Of the following, the laboratory results that would be MOST likely to be seen in this patient are
A. decrease in lactate dehydrogenase, potassium, and uric acid
B. decrease in lactate dehydrogenase, uric acid, and white blood cell count
C. increase in calcium, phosphorus, and uric acid
D. increase in calcium, potassium, and uric acid
E. increase in phosphorus, potassium, and uric acid

American academy of pediatrics 442


American Academy of Pediatrics PREP 2015

Item 143 Preferred Response: E


The boy in this vignette has newly diagnosed acute leukemia and is at risk for tumor lysis
syndrome (TLS), especially after he starts treatment, as a result of his high tumor burden (ie,
high peripheral leukocyte count with high percentage of blasts). Tumor lysis syndrome occurs
when malignant cells are rapidly lysed, typically after chemotherapy or radiation therapy has
been initiated, resulting in the release of cellular components and metabolic abnormalities.
Tumor lysis syndrome is more likely to occur in malignancies with a high rate of proliferation
such as acute leukemia and lymphoma, or those with a high sensitivity to treatment. Tumor lysis
syndrome may lead to renal insufficiency with decreased urine output and elevated serum
creatinine. Brisk lysis of tumor cells can cause an increase in serum uric acid, potassium,
phosphorus, and lactate dehydrogenase, with a decrease in serum calcium. Unrecognized and
untreated TLS can progress to renal failure, arrhythmias, seizures, and death. Various criteria are
used to diagnose clinical and laboratory TLS, all of which include elevations in uric acid,
potassium, phosphate, and creatinine with hypocalcemia.

Empiric management with aggressive hydration promotes the excretion of uric acid and
phosphate. The addition of sodium bicarbonate to alkalinize the urine can increase the solubility
of uric acid and was historically included in TLS management strategies. However, it does not
significantly increase the solubility of xanthine and hypoxanthine, the precursors of uric acid,
which can lead to xanthine obstruction in the renal tubules. Therefore, alkalization of the urine is
no longer universally recommended. Allopurinol, a xanthine analog, is a competitive inhibitor of
xanthine oxidase that blocks the conversion of xanthine and hypoxanthine to uric acid and has
been shown to decrease the incidence of obstructive uropathy caused by uric acid precipitation.
Rasburicase is a recombinant urate oxidase that rapidly decreases uric acid levels by degrading
uric acid to allantoin, a highly soluble substance with no known adverse effects in humans.

Of note, rasburicase is contraindicated in patients with glucose-6-phosphate deficiency because


of the increased risk of hemolysis.

PREP Pearls
• Tumor lysis syndrome (TLS) occurs when malignant cells are rapidly lysed, typically
after chemotherapy or radiation therapy has been initiated, resulting in the release of
cellular components and metabolic abnormalities. Brisk lysis of tumor cells can cause an
increase in serum uric acid, potassium, phosphorus, and lactate dehydrogenase, with a
decrease in serum calcium.
• Tumor lysis syndrome is more likely to occur in malignancies with a high rate of
proliferation such as acute leukemia and lymphoma, or those with a high sensitivity to
treatment.
• Tumor lysis syndrome may lead to renal insufficiency with decreased urine output and
elevated serum creatinine. Unrecognized and untreated TLS can progress to renal failure,
arrhythmias, seizures, and death.

American Board of Pediatrics Content Specification(s)


• Plan an appropriate diagnostic evaluation to exclude tumor lysis syndrome in patients
suspected of having leukemia

American academy of pediatrics 443


American Academy of Pediatrics PREP 2015

Suggested Reading
• Cairo MS, Coither B, Reiter A, Younes A. Recommendations for the evaluation of risk
and prophylaxis of tumor lysis syndrome (TLS) in adults and children with malignant
diseases: an expert TLS panel consensus. Br J Haematol. 2010;149(4):578-586.
doi:10.11110365-2141.2010.08143.x.
• Coiflier B, Altman A, Pui CH, Younes A, Cairo MS. Guidelines for the management of
pediatric and adult tumor lysis syndrome: an evidence-based review. ] Gin Oncol.
2008;26(16):2767-2778. doi:10.1200/ JCO.2007.15.0177.
• McBride A, Westervelt P. Recognizing and managing the expanded risk of tumor lysis
syndrome in hematologic and solid malignancies. J Hematol Oncol. 2012;5:75.
doi:10.1186/1756-8722-5-75.

American academy of pediatrics 444


American Academy of Pediatrics PREP 2015

Item 144
You are seeing a 7-year-old boy who is having school difficulties. His teacher informed the
mother that he is not learning as well or as easily as the other children in his class. At home,
when she is trying to help him with his schoolwork, he is often off task and does not seem to
grasp things very quickly. His schoolwork is sloppy, and he often fails to turn his assignments in
to his teacher. You decide to evaluate him for attention-deficit/hyperactivity disorder.

Of the following, the statement that would MOST strongly support your suspected diagnosis is
A. his early cognitive milestones were in the normal range
B. his father and paternal uncle carried this diagnosis as children
C. his mother gave him 3 doses of a sibling's methylphenidate, which improved his attention
D. last year's teacher found him to be consistently inattentive and impulsive
E. this child has a history of significant alcohol exposure in utero

American academy of pediatrics 445


American Academy of Pediatrics PREP 2015

Item 144 Preferred Response: D


The boy in the vignette is exhibiting symptoms of attention deficit/hyperactivity disorder
(ADHD). The steps required to make the diagnosis include confirming that symptoms have been
persistent since an early age, reviewing the current edition of the Diagnostic and Statistical
Manual of Mental Disorders diagnostic criteria to ensure an adequate number of inattentive
and/or hyperactive symptoms are present, and considering whether other diagnoses (eg, anxiety
or learning disability) might be responsible for the inattentive or hyperactive symptoms. If
symptoms of ADHD are present in only 1 setting (only at home or only at school) the diagnosis
of ADHD should not be made. Children may be hyperactive and inattentive through learned
behavior patterns that only appear in one setting, such as in a home that does not provide any
consistent limits. True ADHD creates pervasive dysfunctions across settings.
Among the response choices listed, the fact that last year's school teacher also found him to be
inattentive and impulsive is most supportive of an ADHD diagnosis because this implies both the
presence of symptoms in different settings and symptom persistence over time.

The presence of early cognitive milestones in the normal range decreases the likelihood that the
boy's behaviors are the result of a developmental impairment, but does not otherwise support the
ADHD diagnosis. Most children who have ADHD have intact early cognitive milestones, as is
the case for most mental health conditions.

Although multiple members of the same family may have ADHD, the presence of a parent's past
diagnosis of ADHD is a less potent predictor of their child having ADHD than the actual
symptoms their child demonstrates.

Behavioral and attentional response to methylphenidate does not confirm a diagnosis of ADHD.
This is why treatment guidelines do not recommend a therapeutic trial of stimulant to help
confirm the presence of ADHD. Stimulants have temporary attention-focusing effects on all
people who take them, an outcome used in the past to help non-ADHD military pilots on long
duration missions.

Alcohol exposure in utero puts a child at risk of developing fetal alcohol syndrome (FAS), which
does include inattention and hyperactivity among the possible clinical manifestations. The
"comorbid" diagnosis of ADHD in children with FAS has been reported in 50% to 94% of those
with heavy alcohol exposure. However, ADHD symptoms in a child with FAS are often of a
different pattern than typical ADHD symptoms, including very early onset inattention. Also
children with ADHD have difficulty with focusing and sustaining attention, whereas children
with FAS have trouble with encoding and attentional shift (an expected transition of attention).
Children with FAS tend not to respond as well to treatment with stimulant medication as children
with pure ADHD; it has been suggested that a unique differential response unlike typical ADHD,
would favor dextroamphetamine over methylphenidate for FAS treatment.

PREP Pearls
• The evaluation for ADHD should include:
1. a full symptom history
2. assessment of functioning in multiple settings (through a rating scale often
completed by the school and parent)
American academy of pediatrics 446
American Academy of Pediatrics PREP 2015

3. review of the inattention/hyperactivity symptom checklist from the DSM


4. consideration of other disorders that might also present with inattention and
hyperactivity.
• Response to a stimulant medication trial is not an appropriate method for diagnosing
ADHD.

American Board of Pediatrics Content Specifications


• Plan the appropriate diagnostic evaluation of ADHD
• Plan the appropriate management of ADHD

Suggested Reading
• American Academy of Pediatrics, Subcommittee on Attention-Deficit/ Hyperactivity
Disorder. ADHD: Clinical practice guideline for the diagnosis, evaluation, and treatment
of attention-deficit/hyperactivity disorder in children and adolescents. Pediatrics.
2011;128(5):1007-1022. doi:10.1542/peds.2011-2654.
• Borin E. The US military needs its speed. Wired. August 10, 2013.
• Peadon E, Elliott EJ. Distinguishing between attention-deficit hyperactivity and fetal
alcohol spectrum disorders in children: clinical guidelines. Neuropsychiatr Dis Treat.
2010;6:509-515. doi:10.2147/NDT.S7256.
• Peadon E. Rhys-Jones B, Bower C, Elliott EJ. Systematic review of interventions for
children with fetal alcohol spectrum disorders. BMC Pediatr. 2009;9:35. dot:
10.1186/1471-2431-9-35.

American academy of pediatrics 447


American Academy of Pediatrics PREP 2015

Item 145
A 15-year-old adolescent girl comes for evaluation with complaints of intermittent shoulder pain
that has waxed and waned for the past 3 to 4 months. She has been on multiple courses of
nonsteroidal anti-inflammatory medications, such as ibuprofen, has had improvement with each
course for 1 to 2 weeks, and then the shoulder pain recurs. Over the last few days, she has had
worsening shoulder pain and shortness of breath. She was unable to sleep last night. She is able
to speak, but is most comfortable sitting upright on the stretcher. Her heart rate is 130 beats/min,
respiratory rate is 32 breaths/ min, and blood pressure is 90/60 mm Hg. She is anxious and has
increased work of breathing. On physical examination, her neck veins are visible. Chest is clear
with equal breath sounds. Her cardiac examination is difficult because of her body habitus, but
her heart sounds are quite soft. There is no murmur, rub, or gallop. Her abdominal examination is
not reliable because she cannot lie flat.

You obtain a chest radiograph (Item Q145). She is admitted to the hospital and an
echocardiogram shows a large pericardial effusion.

Of the following, the intervention MOST likely to worsen this patient's clinical status is
A. ceftriaxone
B. dopamine
C. furosemide
D. normal saline bolus
E. supplemental oxygen

American academy of pediatrics 448


American Academy of Pediatrics PREP 2015

Item 145 Preferred Response:C


The patient described in the vignette has symptoms that began several months ago, but have
acutely worsened in the last few days. With the additional information that her chest radiograph
and her echocardiogram show a large pericardial effusion, it is important to decide if she has
tamponade physiology. She has several signs that she is now experiencing tamponade: her neck
veins are distended, her blood pressure is low for her level of anxiety and distress, and her heart
sounds are soft. These findings suggest, in order, that she is unable to fill the right side of her
heart easily, that her cardiac output is decreased, and that her heart is further away from her chest
wall than would be expected. In this situation, administration of furosemide would be dangerous.
It would lower her circulating blood volume, make compression of her right ventricle more
severe, and acutely decrease her systolic output. Of the choices given, this is most likely to
worsen the patient's status.

Administration of ceftriaxone would be helpful if this was a bacterial pericarditis and the bacteria
were sensitive to that antibiotic. If this were bacterial pericarditis, a much more fulminant course
would be expected. Antibiotics would not likely cause an adverse effect. Dopamine may be
needed if the patient became hypotensive and would not make the effusion worse. A normal
saline bolus would be the best intervention while more definitive treatment is being arranged,
including draining the effusion. Supplemental oxygen would not hurt the patient in this setting of
decreased cardiac output; it would be helpful to keep tissue and myocardial oxygenation as high
as possible.

The causes of pericardial effusions are many. The major categories are metabolic (uremia,
hypothyroid states), infectious (viral, bacterial, fungal) with or without myocarditis, and
autoimmune (lupus, juvenile rheumatoid arthritis, acute rheumatic fever). Presentation can be
acute or chronic or a combination, as in this case. The most likely cause in this patient is an
autoimmune process of several months' duration, which responded to anti-inflammatory
medications. Her shoulder pain is referred from diaphragmatic irritation. She now exhibits
tamponade physiology because her pericardium cannot stretch any further. As her effusion

American academy of pediatrics 449


American Academy of Pediatrics PREP 2015

accumulates, her right atrium and right ventricle are collapsing. Her heart sounds are soft
because of fluid encasing the heart. The flow to her lungs, and therefore to her left ventricle and
her systemic circulation, is not adequate, thus making her blood pressure low.

PREP Pearls
• Pericardial effusions can present with positional chest pain, abdominal pain, or shoulder
pain.
• A slowly accumulating pericardial effusion can suddenly become dangerous when
tamponade physiology develops.

American Board of Pediatrics Content Specification(s)


• Understand the natural history of pericarditis
• Recognize the clinical findings associated with pericarditis and plan appropriate initial
management
• Recognize pathogens commonly associated with pericarditis

Suggested Reading
• LeWinter MM, Tischler MD. Pericardial diseases. In: Bonow RO, Mann DL, Zipes DP,
Libby P, eds. Braunwald's Heart Disease: A Textbook of Cardiovascular Medicine. 9th
ed. Philadelphia, PA: Saunders Elsevier; 2012:1651-1671.
• Maisch B, Ristic AC. Practical aspects of management of pericardial disease. Heart.
2003;89(9):1096-1103. doi:10.1136/heart.89.9.1096.
• Spicer R, Ware S. Diseases of the pericardium. In: Kliegman RM, Stanton BF, St Geme J
W Ill, Schor NF, eds. Nelson Textbook of Pediatrics. 19th ed. Philadelphia, PA: Saunders
Elsevier; 2011:1635-1637.

American academy of pediatrics 450


American Academy of Pediatrics PREP 2015

Item 146
You are seeing a 14-year-old female gymnast who complains of back pain. The pain has been
increasing over the last several months. She has no history of injury, but has been complaining of
pain more frequently with bending backwards. She denies numbness, tingling, or urinary or stool
incontinence. She states that the pain is worse with activity and is relieved with rest.
Occasionally, ibuprofen decreases the pain. On physical examination, she is a well-appearing girl
with pain with extension of her spine. She has point tenderness over the left lower lumbar
paraspinal muscles.

Of the following, the next BEST step in evaluation of this patient is


A. bone scintigraphy
B. computed tomography of the lumbar spine
C. dual-energy X-ray absorptiometry scan
D. magnetic resonance imaging
E. radiography of the lumbar spine

American academy of pediatrics 451


American Academy of Pediatrics PREP 2015

Item 146 C Preferred Response: E


The girl in the vignette is a gymnast with chronic back pain exacerbated by activity and relieved
with rest. She has no neurologic signs or symptoms. On physical examination, she has point
tenderness and pain with spinal extension. This clinical picture should raise concern for
spondylolysis, a bony defect in the pars interarticularis. Radiography would be the next best step
to evaluate for this diagnosis. Although computed tomography and magnetic resonance imaging
of the lumbar spin would give the diagnosis of spondylolysis, plain radiography may yield the
diagnosis and are less invasive and more cost effective. Bone scintigraphy would identify an area
of metabolic activity at the site of spondylolysis but it is nonspecific and further imaging is
required for diagnosis. The girl in the vignette is a gymnast, but shows no signs or symptoms of
the classic "female athlete triad" (disordered eating, amenorrhea, and osteoporosis), therefore a
dual-energy X-ray absorptiometry scan is not indicated.

Although uncommon in young children, spondylolysis is a common cause of back pain in


athletes older than 10 years during periods of rapid growth. Spondylolysis usually occurs with
repetitive trauma and is frequently seen in athletes participating in ballet and other forms of
dancing, football, gymnastics, high jump activities, and weight lifting. Most often presenting
with mild-to-moderate lumbar pain on physical examination, children typically have localized
spinal tenderness with muscle spasm and loss of the normal lordosis. Lumbar flexion and
extension may be limited, and the pain is usually worsened with hyperextension of the spine.
Treatment is conservative, including rest and avoidance of aggravating activities.

The approach to a patient with back pain should consist of a thorough history and physical
examination that includes assessment of the timing and location of the pain. The clinician should
ask whether the pain is associated with activity, whether the patient participates in any sports,
whether there is any history of trauma, and whether there was acute versus insidious onset. If
nothing in the history is suggestive and the physical examination is normal, then a diagnosis of
nonspecific back pain can be considered and treated conservatively with physical therapy and
nonsteroidal anti-inflammatory medication. If no improvement is noted after 1 month of
conservative management, the patient should be re-evaluated. If the child's history raises concern
for a disease process or injury, then anteroposterior and lateral plain radiographs should be
obtained, with or without oblique views depend-ing on the suspected diagnosis. Historically
oblique views have been recommended for diagnosis of spondylolysis, but anteropoterior and
lateral views may be sufficient for this diagnosis. Laboratory tests should be considered to rule
out infection, inflammation, or an oncologic process. If the laboratory tests and radiography do
not yield a diagnosis, then further imaging should be considered. Magnetic resonance imaging
(MRI) is recommended for patients in whom infection, a herniated disc, or a tumor are
suspected. Computed tomography can be helpful for diagnostic confirmation if spondylolysis is
suspected, but not identified on plain radiography. A bone scan is an acceptable alternative to
MRI in young children in cases in which infection or tumor is suspected, but the results can be
nonspecific, and require further testing. Bone scan can also be used to localize metabolic bone
activity in the spine to identify areas for further imaging in cases of suspected stress fracture or
spondylolysis. If all test results are negative and there is no response to conservative treatment,
then a referral to a pain management service should be considered. A differential diagnosis of
back pain is shown in Item C146, page C-127.

American academy of pediatrics 452


American Academy of Pediatrics PREP 2015

PREP Pearls
• Initial assessment of back pain in children with an abnormal physical examination or
concerning history should begin with plain radiography and laboratory studies to rule out
infection, inflammation, and tumor.
• An oblique view radiograph is the best first imaging method for diagnosing
spondylolysis.
• Magnetic resonance imaging is the best choice for evaluating infection, herniated disc, or
tumor in children with back pain.
• Computed tomography is preferred in cases of suspected spondylolysis with normal plain
radiographs.

American academy of pediatrics 453


American Academy of Pediatrics PREP 2015

American Board of Pediatrics Content Specification(s)


• Plan the appropriate evaluation of back pain
• Plan the appropriate management of back pain

Suggested Reading
• Haidar R, Saad S, Khoury N, Musharrafieh U. Practical approach to the child presenting
with back pain. Eur J Pediatr. 2011;170(2):149-156. do1:10.1007/s00,131- 010-1220-9.
• Haus BM, Micheli L. Back pain in the pediatric and adolescent athlete. Chu Sports Med.
2012;31(3):423-440. doi:10.1016/j.csm.2012.03.011.

American academy of pediatrics 454


American Academy of Pediatrics PREP 2015

Item 147
A 15-year-old adolescent girl presents to your office for evaluation of a right knee injury that
occurred 4 days ago. She was playing lacrosse, and twisted her body with her foot planted. She
experienced immediate onset of pain and continues to have pain with weight bearing. She reports
a sensation of "something catching" inside her knee. On physical examination, there is a
moderate effusion of the right knee. She is unable to flex the knee beyond 90°, nor straighten the
knee to within 20° of full extension. She has tenderness over the medial joint line. Radiographs
of the knee are unremarkable.

Of the following, the MOST appropriate next step in management is


A. gradual return to activities as tolerated
B. physical therapy to improve range of motion
C. referral to an orthopedic specialist
D. use of a knee immobilizer for 4 weeks
E. use of a patella-stabilizing brace for 6 weeks

American academy of pediatrics 455


American Academy of Pediatrics PREP 2015

Item 147 Preferred Response: C


The clinical history and physical examination for the girl in the vignette suggest that she has
sustained a meniscal tear. She has mechanical symptoms, clicking and locking, and a very
limited range of motion in the knee. The next appropriate steps in management would be to
recommend the patient be non-weight-bearing, with the use of crutches, and referral to an
orthopaedic specialist for additional evaluation and treatment. Individuals with mechanical
symptoms after a knee injury often require surgical intervention.

The menisci are kidney bean-shaped, shock-absorbing cartilage cushions in the knee. Meniscal
tears are uncommon in children, with rates of tear increasing during midadolescence. Tears tend
to occur with a twisting or pivoting motion, particularly if the foot is planted. In teenagers,
meniscal tears are often associated with ligament injuries.

Following a meniscal tear, patients typically recall acute onset of pain and may report feeling a
"pop" at the time of injury. Following the injury, they often experience pain with extreme flexion
of the affected knee. On physical examination, clinicians may detect effusion and joint line
tenderness. The examiner should perform the McMurray test, holding the patient's knee in
flexion, then rotating the tibia and extending the knee; pain or a palpable click may indicate the
presence of a meniscal tear.

Radiographs should be considered to look for bony injuries, particularly in a skeletally immature
patient. Magnetic resonance imaging is the preferred test to confirm the diagnosis of meniscal
injury, evaluate morphology of any tears, and to assess for other injuries (eg, osteochondral
fractures).

Return to physical activity would be inappropriate for this adolescent as she has mechanical
symptoms and limited range of motion. Physical therapy is unlikely to improve range of motion
because the patient likely has a meniscal tear that obstructs movement of the knee. Because this
girl has mechanical symptoms, she should be made nonweight-bearing pending additional
evaluation. The use of a knee immobilizer will make use of crutches more difficult. A patellar
stabilizing brace would be indicated for a patient with patellofemoral symptoms, but would not
be of benefit for this patient.

PREP Pearls
• Following an acute knee injury, mechanical symptoms, such as catching or locking,
suggest intra-articular pathology and a likely need for surgical intervention.
• Meniscal tears tend to occur with a twisting or pivoting motion, particularly if the foot is
planted.

American Board of Pediatrics Content Specification(s)


• Understand when orthopedic consultation is required for a sports-related knee injury

Suggested Reading
• Kramer DE, Micheli LJ. Meniscal tears and discoid meniscus in children: diagnosis and
treatment. I Am Acad Orthop Stag. 2009;17(11):698-707.

American academy of pediatrics 456


American Academy of Pediatrics PREP 2015

• Sarwark JF, LaBella C. Pediatric Orthopaedics and Sports Injuries: A Quick Reference
Guide. Elk Grove Village, IL: American Academy of Pediatrics; 2010.

American academy of pediatrics 457


American Academy of Pediatrics PREP 2015

Item 148
An 18-month-old girl presented to the emergency department 2 days ago with nasal congestion
and was diagnosed with a febrile urinary tract infection. A bag urine specimen was obtained;
urinalysis was positive only for leukocyte esterase, and culture grew 50,000 colony forming
units/mL of Escherichia coli. She was discharged to home with oral trimethoprim-
sulfamethoxazole treatment.
In your office, the patient is playful, afebrile, and has normal vital signs for age. Her physical
examination findings are normal. You tell the mother to stop the trimethoprim sulfamethoxazole.

Of the following, the principle that best explains why this action is appropriate is its
A. generalizability
B. negative predictive value
C. reliability
D. sensitivity
E. validity

American academy of pediatrics 458


American Academy of Pediatrics PREP 2015

Item 148 S Preferred Response: E


The validity of a test refers to how well the test measures what it purports to measure. For the
patient described in the vignette who has nasal congestion and an unremarkable urinalysis, the
result of a urine culture obtained using a urine collection bag is likely not valid (accurate)
because the test detects bacteria colonizing the genitourinary area, in addition to bacteria
infecting the bladder. The appropriate test for a child who is not toilet-trained is a urine specimen
for culture obtained by urethral catheterization.

The internal validity of a test refers to its accuracy, while the external validity of a test relates to
its generalizability. There cannot be external validity without internal validity; if a test is not
accurate, it does not matter if it is generalizable. A bag urine specimen for culture can be
obtained in any patient, so the test is generalizable, but the poor accuracy of the test in this case
makes it not valid. The reliability of a test refers to the stability of its performance; if the test is
repeatedly performed on the same individual, will it have the same result? For the patient
described in the vignette, the culture result from a bag urine specimen may be reliable, but the
result may not be accurate. The sensitivity of the test (bag urine culture) is its ability to correctly
determine that a patient is infected, while the specificity is its ability to correctly classify an
uninfected individual. The patient in the vignette is not infected but the urine culture was
positive, so the test is sensitive, but not specific. The sensitivity and specificity of a test are not
affected by the prevalence of disease. The negative predictive value of a test is the probability
that a patient with a negative urine culture is not infected, while the positive predictive value is
the probability that a patient with a positive urine culture actually is infected. For a urine culture
obtained by bag collection, the negative predictive value is high, but the positive predictive value
is low. Prevalence of disease affects the positive and negative predictive values. A higher disease
prevalence results in a higher positive predictive value (fewer false positives) and lower negative
predictive value (more false negatives).

PREP Pearls
• The validity of a test refers to how well the test measures what it purports to measure.
• The reliability of a test refers to the stability of its performance over time.
• The sensitivity of a test is the proportion of patients with a positive test result who have
the disease.
• The specificity of a test is the proportion of patients with a negative test result who do not
have the disease.

American Board of Pediatrics Content Specification(s)


• Understand validity and how it might be compromised
• Understand reliability and how it might be compromised
• Understand generalizability and how it relates to validity

Suggested Reading
• Bewick V. Cheek L, Ball J. Statistics review 13: receiver operating characteristic curves.
Crit Care. 2004;8(6):508-512.
• Godwin M, Ruhland L, Casson 1, et al. Pragmatic controlled clinical trials in primary
care: the struggle between external and internal validity. BMC Med Res Methodol.
2003;3:28. doi:10.1186/1471-2288-3-28.
American academy of pediatrics 459
American Academy of Pediatrics PREP 2015

Item 149
A 14-year-old adolescent boy presents to your office for evaluation 4 hours after he sustained a
puncture wound to his left heel. He stepped on a rusty nail while walking without shoes across a
parking lot. The nail initially lodged in his left heel, but he pulled it out at that time. His heel
bled slightly, but the bleeding resolved after only a few minutes. The patient's mother brought
him to your office for evaluation because he has been complaining of constant pain in his left
heel since his injury. The teenager has a history of obesity, type 2 diabetes mellitus, and asthma.
His medications include an inhaled corticosteroid and metformin. The patient's immunizations
are up to date; a review of his chart confirms that he received a tetanus booster 2 years ago.
The adolescent appears very well and is in no distress. He is afebrile, and his vital signs are
normal for his age. His weight is at the 97th percentile for his age. Examination of the boy's left
foot reveals a 4-mm puncture wound on his left heel that is surrounded by some jagged skin
tissue. The wound is not bleeding, and there is no surrounding erythema or warmth. There is no
drainage from the wound. On palpation of the wound, you do not detect any retained foreign
body.
While you are irrigating and debriding the puncture wound, the teenager's mother asks you about
the likelihood that the wound will become infected.

Of the following, the factor MOST likely to increase the patient's risk of developing a wound
infection is
A. daily inhaled corticosteroid use
B. history of type 2 diabetes mellitus
C. lack of shoes at the time of the injury
D. relative location of the wound on the heel rather than forefoot
E. setting of the injury (paved surface)

American academy of pediatrics 460


American Academy of Pediatrics PREP 2015

Item 149 Preferred Response: B


The teenager in the vignette has sustained a plantar puncture wound from a rusty nail. Of the
factors listed, his medical history of type 2 diabetes mellitus is most likely to increase his risk of
developing a wound infection.

Puncture wounds are a common type of minor injury sustained by children, accounting for
approximately 3% to 5% of all traumatic injuries presenting to pediatric emergency departments.
Although most children sustaining puncture wounds have uncomplicated courses, serious
complications can arise from puncture wounds, especially those involving the plantar surface of
the foot. All pediatric clinicians should understand the principles underlying appropriate
evaluation and management of various puncture wounds.

Puncture wounds may arise from various circumstances. Although more than half of all puncture
wounds involve the plantar surface of the foot, other affected sites include the legs, arms, hands,
and, less commonly, the trunk and head. Most plantar puncture wounds are caused by nails.
However, glass, wood, plastic, and other metal objects can be involved as well.

Infection is the most common complication that arises from puncture wounds. Wound infection
is more likely when puncture wounds are deep, when there is more devitalized tissue, and in
cases involving retained foreign bodies. Other factors that have been identified as increasing the
risk for puncture wound infection include wounds involving the forefoot, punctures occurring
through shoes, and an underlying history of diabetes mellitus.

Bacterial agents most commonly implicated in puncture wound infections include


Staphylococcus aureus, β-hemolytic streptococci, and anaerobic bacteria. Infections from
Pasteurella multocida are often seen in puncture wounds arising from animal bites. Pseudomonas
aeruginosa has been commonly isolated in patients sustaining plantar puncture wounds while
wearing tennis shoes at the time of injury. Aside from infection, complications that may arise
(though less commonly) from puncture wounds include retained foreign bodies, injury to
neurovascular structures, and tattooing of skin from debris, which can lead to permanent
cosmetic deformity.

For all children presenting with puncture wounds, a careful history is essential to guide
appropriate management and to identify risk factors for complications. Physical examination
should include a complete evaluation of the injured area, including assessment of circulation and
motor function distal to the wound. The wound must be inspected for retained foreign material
and signs of infection. If there is any suspicion about the possibility of a retained foreign body,
diagnostic imaging should be performed. Most glass and metal objects will appear on plain
radiography. In addition, tetanus immunization status should be reviewed for all children
presenting with puncture wounds; tetanus immunization and tetanus toxoid should be
administered when indicated.

Uninfected puncture wounds should be irrigated with copious amounts of sterile saline, cleansed
with an antiseptic solution, and debrided whenever jagged edges are present. Skin should be
cleansed of foreign material to avoid permanent scarring. Foreign bodies should be removed to

American academy of pediatrics 461


American Academy of Pediatrics PREP 2015

reduce the risk of wound infection, reduce pain, and avoid subsequent damage to underlying
neurovascular structures.

Children with simple uninfected plantar puncture wounds can generally be treated with rest,
elevation of the affected foot, and intermittent warm water soaks. Families should be educated
about the signs and symptoms of wound infection and asked to return if these signs and
symptoms occur.

There is no evidence to support the routine administration of prophylactic antibiotics to prevent


infection after puncture wounds. Antibiotics may be prescribed in cases when a patient's risk for
infection is particularly high, at the discretion of the treating clinician, such as in punctures
resulting from animal or human bites, for highly contaminated wounds, or in
immunocompromised patients who have an elevated risk of infection.

Inhaled corticosteroid use has not been associated with increased risk of wound infections.
The lack of shoes by the patient in the vignette at the time of the injury does not increase his risk
for wound infection. In fact, infection with P aeruginosa is associated with plantar puncture
wounds that are sustained while wearing sneakers.

Sustaining a plantar puncture wound while walking on a paved surface has not been associated
with an increased risk of infection or other complications.
Puncture wounds affecting the forefoot, as opposed to the heel, have been found to carry a higher
risk of wound infection.

PREP Pearls
• Infection is the most common complication arising from puncture wounds.
• Risk factors for the development of puncture wound infections include wounds that are
especially deep, have a large amount of devitalized tissue, involve retained foreign
bodies, involve the forefoot, occur through shoes, and affect patients with underlying
conditions impairing immunity, including diabetes mellitus.
• Children with simple uninfected plantar puncture wounds can generally be treated with
rest, elevation of the affected foot, and intermittent warm water soaks. Families should be
educated about the signs and symptoms of wound infection and asked to return if the
signs and symptoms occur.
• There is no evidence to support the routine administration of prophylactic antibiotics to
prevent infection after puncture wounds.

American Board of Pediatrics Content Specification(s)


• Plan the appropriate evaluation of various puncture wounds, including a puncture wound
through a sneaker, and manage appropriately

Suggested Reading
• Baldwin G, Colbourne M. Puncture wounds. Pediatr Rev. 1999;20(1):21-23,
doi:10.1542/pir.20-1-21.
• Spiro DM, Zonfrillo MR, Meckler GD. Wounds. Pediatr Rev. 20113;31(8J:326-334.
doi:10.1542/pir.31-8-326.
American academy of pediatrics 462
American Academy of Pediatrics PREP 2015

Item 150
You are called by the nursery staff to inform you of a newborn admitted to your service. The
mother presented in labor at 36 3/7 weeks of gestation after receiving good prenatal care that was
complicated only by unknown group B Streptococcus screening status. She received 1 dose of
cefazolin 4 hours prior to delivery. Spontaneous rupture of the membranes occurred 8 hours prior
to delivery and the mother remained afebrile. The newborn appears clinically well 1 hour after
delivery, with a glucose value of 51 mg/dL (2.8 mmol/L). The mother is requesting discharge 24
hours after delivery.

Of the following, the MOST appropriate initial management is to


A. observe for 24 hours with no additional evaluation
B. observe for 48 hours with no additional evaluation
C. obtain a blood culture and complete blood cell count with differential and observe for 24
hours
D. obtain a blood culture and complete blood cell count with differential and observe for 48
hours
E. obtain a blood culture and complete blood cell count with differential and treat with
antibiotics

American academy of pediatrics 463


American Academy of Pediatrics PREP 2015

Item 150 Preferred Response: B


The newborn in the vignette should be observed in the hospital for 48 hours with no additional
evaluation unless clinical signs of sepsis develop. Mothers who are admitted in labor before 37
weeks and 0 days with unknown group B streptococcal (GBS) status should receive antibiotic
prophylaxis. Late preterm newborns who appear well at birth may simply be observed for 48
hours or more in the hospital if their mother received adequate antibiotic therapy starting at 4 or
more hours before delivery, had ruptured membranes less than 18 hours, and did not develop
chorioamnionitis.

Indications for diagnostic evaluation and antibiotic treatment of a newborn at risk for early-onset
GBS, regardless of gestational age, include any clinical signs of neonatal sepsis or maternal
chorioamnionitis. Chorioamnionitis has commonly been defined in the literature as the
development of a maternal fever associated with 2 of the following clinical findings: uterine
tenderness, maternal or fetal tachycardia, and foul smelling, purulent amniotic fluid. Maternal
fever is seen in 95% to 100% of cases of chorioamnionitis. The US Centers for Disease Control
and Prevention cite a maternal temperature of more than 100.4°F (38°C) as an intrapartum sepsis
risk factor that warrants maternal evaluation and potential management for presumed
chorioamnionitis as determined by the obstetric providers. Well-appearing newborns exposed to
maternal chorioamnionitis should undergo a limited evaluation (blood culture and complete
blood cell count with differential and platelets) and antibiotics should be initiated. Any newborn
with signs of sepsis regardless of intrapartum risk factors should undergo a full diagnostic
evaluation (blood culture, complete blood count with differential and platelets, chest radiography
if respiratory symptoms are present, and lumbar puncture if cardiorespiratory status is stable) and
immediately be given antibiotic therapy.

The revised guidelines for the prevention of perinatal GBS disease were published in 2010,
including an algorithm for neonatal management (Item C150). This algorithm defines
management and treatment strategies based on clinical presentation, maternal chorioamnionitis,
gestational age, adequacy of antibiotic treatment, and duration of membrane rupture. Review of
this algorithm supports observation of the late preterm newborn in the vignette for 48 hours.

American academy of pediatrics 464


American Academy of Pediatrics PREP 2015

American academy of pediatrics 465


American Academy of Pediatrics PREP 2015

PREP Pearls
• Indications for diagnostic evaluation and antibiotic treatment of a newborn at risk for
early-onset group B streptococcal disease include any clinical signs of neonatal sepsis or
maternal chorioamnionitis.
• Late preterm newborns who appear well at birth may simply be observed for 48 hours or
more in the hospital if their mother received adequate antibiotic therapy for 4 hours or
more hours before delivery, had ruptured membranes less than 18 hours, and did not
develop chorioamnionitis.

American Board of Pediatrics Content Specification(s)


• Plan the management of a neonate whose mother is febrile at the time of delivery

Suggested Reading
• American Academy of Pediatrics, Committee on Fetus and Newborn. Management of
neonates with suspected or proven early-onset bacterial sepsis. Pediatrics.
2012;129(5):1006-1015.
• American Academy of Pediatrics, Committee on Infectious Diseases and Committee on
Fetus and Newborn. Recommendations for the prevention of perinatal group B
streptococcal (GBS) disease. Pediatrics. 2011; 128(3):611-616. doi:10.1542/peds.2011-
1466.
• Tita ATN, Andrews WW. Diagnosis and management of clinical chorioamnionitis. Clin
Perinatol. 2010;37(2):339-354. doi: 10.1016/j. clp.2010.02.003.
• Verani JR, McGee L, Schrag SJ; Division of Bacterial Diseases, National Center for
Immunization and Respiratory Diseases, Center for Disease Control and Prevention.
Prevention of perinatal group B streptococcal disease, revised guidelines from CDC,
2010. MMWR Recomm Rep. 2010;59(R-10):1-32. Accessed February 25, 2014.

American academy of pediatrics 466


American Academy of Pediatrics PREP 2015

Item 151
A 7-year-old girl with a seizure disorder has been taking valproate for 1 year. She has not had
any seizures during that time, nor any adverse effects from the medication. She is growing and
developing well. Her mother brought the girl to your office because she has been unusually
sleepy. There was no trauma and no known ingestions, and the parents have been supervising her
medications closely. She has had no recent illness. You recommended checking a valproate level
and liver enzymes, and now have the following results:
• Valproate level, 61.00 ug/mL (423 µmol/L); reference range, 50 ug/mL to 120 ug/mL
(347-832 umol/L)
• Alanine aminotransferase, 34 U/L
• Aspartate aminotransferase, 41 U/L
Of the following, the test MOST likely to reveal the cause of her sleepiness is
A. blood urea nitrogen
B. complete blood cell count
C. free and total carnitine
D. repeat valproate level
E. serum ammonia

American academy of pediatrics 467


American Academy of Pediatrics PREP 2015

Item 151 Preferred Response: E


Valproate can cause hyperammonemic encephalopathy without transaminitis. Of the choices
listed, ammonia is the most likely to be abnormal and causing the sleepiness in the girl in the
vignette. It is also possible that she is having sub-clinical seizures, also called nonconvulsive
status epilepticus. If the parents have a medication to stop seizures at home, such as rectal
diazepam, they should try this. If this does not cause her mental status to improve, she should be
evaluated in the emergency department for other causes of encephalopathy such as infection or
electrolyte abnormality. For hyperammonemia caused by valproate, the dose of valproate will
need to be reduced or the medication may need to be discontinued altogether. The girl's
neurologist should be consulted to help manage her seizures in this setting.

Valproate can cause neutropenia and thrombocytopenia, but these do not typically present with
sleepiness, so a complete blood count with differential is not the best choice. Renal dysfunction
is not likely to occur in this patient with a normal valproate level. Checking levels of free and
total carnitine would not be helpful to find a cause of sleepiness. Although valproate has been
associated with low levels of free and total carnitine, it is unclear whether this causes any
symptoms in children and whether giving carnitine is indicated. The valproate level was within
the normal range, so it is unlikely that this is causing the girl's sleepiness; rechecking the level is
unlikely to aid in diagnosis.

PREP Pearls
• Valproate can cause hyperammonemic encephalopathy without transaminitis.
• Valproate can cause neutropenia and thrombocytopenia.

American Board of Pediatrics Content Specification(s)


 Recognize side effects and toxicities associated with anticonvulsant drugs
 Recognize laboratory abnormalities associated with anticonvulsant drug therapy

Suggested Reading
• Lexicomp. Valproate: pediatric drug information. UpToDate. Available online only for
subscription.

American academy of pediatrics 468


American Academy of Pediatrics PREP 2015

Item 152
A 10-year-old girl presents for poor linear growth and slow weight gain. In the last year, her
linear growth has stopped completely, and she continues to struggle to gain weight. Review of
systems reveals frequent fatigue, anorexia, and a decreased energy level. She has also had
occasional episodes of emesis in the morning that are becoming more frequent.
Physical examination shows some darkening of her skin and mucous membranes. The remainder
of her examination is unremarkable. Initial laboratory testing shows a serum sodium
concentration of 131 mEq/L (131 mmol/L), potassium concentration of 5.8 mEq/L (5.8 mmol/L),
and a glucose level of 60 mg/dL (3.3 mmol/L). The rest of her chemistry results are
unremarkable. Complete blood cell count shows eosinophilia.

Of the following, the BEST next test to determine her underlying diagnosis is
A. insulin like growth factor 1
B. karyotype
C. serum cortisol
D. thyrotropin
E. tissue transglutaminase antibody

American academy of pediatrics 469


American Academy of Pediatrics PREP 2015

Item 152 Preferred Response: C


The child in the vignette has Addison disease, the most common cause of adrenal insufficiency
in the industrialized world. Classic chronic symptoms include weakness, fatigue, anorexia,
nausea, abdominal pain, and diarrhea. Poor growth can also occur, as seen in this child.
Hyperpigmentation of the skin and mucous membranes can develop over time. Acute symptoms
can include muscle, joint and abdominal pain, and hypotension.

Laboratory evaluation may reveal hyponatremia, hyperkalemia, and hypoglycemia if the child
becomes ill and develops an adrenal crisis. Sometimes the complete blood cell count (CBC) will
reveal anemia and eosinophilia as well. Serum cortisol will be very low, revealing the diagnosis
of adrenal insufficiency. Other supportive testing includes elevated adrenocorticotropic hormone
(ACTH) levels (frequently > 100 pg/mL [22 pmol/L]), and ACTH stimulation testing can reveal
a low cortisol response (< 18 µg/dL [497 nmol/L]). Mineralocorticoid deficiency is confirmed
with a relatively low aldosterone value in the face of hyperreninemia. Adrenal antibodies can
also be measured to confirm the diagnosis of Addison disease as the underlying cause of adrenal
insufficiency.

Other causes of poor growth include growth hormone deficiency, hypothyroidism, and Turner
syndrome. Although insulin like growth factor 1, thyrotropin, and karyotype testing are
reasonable in the evaluation of a child with poor growth, the additional symptoms shown by the
child in this vignette, along with her hyperpigmentation and laboratory findings, make these
diagnoses much less likely.

Similarly, tissue transglutaminase testing for celiac disease is warranted in the routine evaluation
of a child with poor growth, especially with gastrointestinal symptoms. However, for girl in the
vignette, her severe symptoms and classic laboratory findings fit much better with adrenal
insufficiency as the most likely diagnosis.

PREP Pearls
• Most children who have Addison disease experience a constellation of ill-defined
symptoms to include fatigue, generalized muscular weakness, loss of appetite, and poor
weight gain. A high index of suspicion is needed to diagnose Addison disease.
• A low serum cortisol measurement at the time of acute illness will confirm the diagnosis
of adrenal insufficiency.

American Board of Pediatrics Content Specification(s)


• Recognize the clinical features associated with Addison disease
• Recognize the clinical and laboratory manifestations of adrenal insufficiency

Suggested Reading
• Antal Z, Zhou P. Addison disease. Pediatr Rev. 2009;30(12):491-493.
doi:10.1542/pir.30-12-491.
• Loraux DL. Adrenocortical insufficiency. In: Becker KL, ed. Principles and Practice of
Endocrinology and Metabolism. 3rd ed. Hagerstown, MD: Lippincott Williams &
Wilkins; 2001:739-742.

American academy of pediatrics 470


American Academy of Pediatrics PREP 2015

Item 153
During a 9-year-old boy's health supervision visit, his mother tells you that she recently heard a
news report about melanoma in children. She asks you if her child is at risk for this disease.

Of the following, the factor MOST likely to increase his risk is


A. a 7-mm congenital pigmented nevus
B. a family history of basal cell skin cancer
C. a history of a blistering sunburn
D. a history of having a renal transplant
E. male sex

American academy of pediatrics 471


American Academy of Pediatrics PREP 2015

Item 153 Preferred Response: D


Melanoma is a rare cancer in children and youth despite a 2% per year increase in incidence over
the last 30 years. Risk factors for melanoma include white race, female gender, adolescent age
group, family history of melanoma (but not other skin cancers), and immunodeficiency. Prior
organ transplant confers a 4-fold increased risk of melanoma compared with the general pediatric
population, and survivors of childhood cancer, especially leukemia and retinoblastoma, are at
even higher risk. Other aspects of the history and physical examination that are found more
frequently in children and youth with melanoma include xeroderma pigmentosum, presence of
dysplastic nevi, and a large number of melanocytic nevi. Few, small congenital nevi (less than a
few centimeters in diameter) are not associated with melanoma development among children.
Dysplastic nevi may transform into melanoma, but they also indicate an increased risk of
developing melanoma elsewhere on the body. Although blistering sunburns during childhood are
risk factors for onset of melanoma during adulthood and sometimes adolescence, they are not
often associated with childhood-onset melanoma. Recent evidence however suggests UV-A
exposure from tanning beds is a risk factor in the increasing rate of melanoma in both
adolescents and adults.

Clinical features of melanoma are well defined for adults, but are less consistent for children. For
adults, the "ABCD" (Asymmetry, Border irregularity, uneven Color and Diameter > 6 mm)
approach has increased awareness and recognition of melanoma. In pediatric patients, melanoma
often is recognized only because of a change in a pre-existing nevus. These changes include
rapid growth, bleeding, color change, or itch. Additional findings can include unexplained lymph
node enlargement, subcutaneous mass, and pain from distant metastases. Unfortunately in
children, melanoma may be diagnosed late because the lesions are often asymptomatic or even
amelanotic. Vigilance for changes in pre-existing nevi, especially in the presence of the above
risk factors, is important for early diagnosis of melanoma in children.

PREP Pearls
• The incidence of melanoma among adults and children has been increasing over the last
30 years, but it still remains uncommon in children.
• Risk factors for melanoma development in pediatric patients include white race; female
gender; adolescent age group; immunodeficiency; solid organ transplant; childhood
malignancies; the presence of xeroderma pigmentosum, dysplastic nevi, or many
melanocytic nevi; and a family history of melanoma.
• Melanoma in children is most often detected because of a change in the appearance of a
pre-existing nevus, although lesions are often asymptomatic.
• The "ABCD" approach to diagnosing melanoma is less reliable in children than in adults.

American Board of Pediatrics Content Specification(s)


• Recognize the clinical findings associated with melanoma

Suggested Reading
• Herzog C. Melanoma. In: Kliegman RM, Stanton BMD, St. Geme JW III, Schor NF,
Behrman RE, eds. Nelson Textbook of Pediatrics. 19th ed. Philadelphia, PA: Saunders
Elsevier; 2011:1772.

American academy of pediatrics 472


American Academy of Pediatrics PREP 2015

• Paradela S, Fonseca E, Prieto VG. Melanoma in children. Arch Pathol Lab Med.
2011;135(3):307-316. doi:10.1043/2009-0503-RA.1.
• Wong JR, Harris JK, Rodriguez-Galindo C, Johnson KJ. Incidence of childhood and
adolescent melanoma in the United States: 1973-2009. Pediatrics. 2013;131(5):846 -854.
doi:10.1542/peds.2012-2520.

American academy of pediatrics 473


American Academy of Pediatrics PREP 2015

Item 154
A 15-year-old adolescent girl presents to your office for a routine physical examination. She is
now in high school and maintaining her grades. You speak with her alone as part of the visit, and
after discussing the limits of confidentiality, you screen her for high-risk behaviors. She reveals a
number of behaviors that put her health at risk but asks that you keep the information
confidential.

Of the following, you are MOST likely to deny her request if she is
A. depressed and has some fresh cut marks on her arms
B. experiencing frequent recurrences of herpes genital ulcers
C. requesting an intrauterine device for contraception
D. sexually active with inconsistent protection against pregnancy
E. using marijuana frequently

American academy of pediatrics 474


American Academy of Pediatrics PREP 2015

Item 154 I-C P S Preferred Response: A


If the girl in the vignette is depressed and exhibiting self-harm behavior, it may be necessary to
intervene and notify her family. In seeking autonomy from parents and exploring the world
around them, adolescents may engage in risky behaviors. Every visit with an adolescent is an
opportunity to screen for these behaviors and intervene if necessary. It is important to remember
that if you don't ask, they won't tell. Ensuring time alone and confidentiality with the adolescent
becomes crucial in eliciting sensitive information.

Confidentiality refers to an agreement between the patient and provider that information
discussed during or after the encounter will not be shared with other parties without the
adolescent's explicit permission. Laws regarding notification of parents vary by state; if none
exist, then the mature minor doctrine is usually used. A mature minor is a minor who is
emotionally and intellectually mature enough to provide informed consent. The minor may live
under the supervision of a parent or guardian. Most states allow minors to consent to treatment
for sexually transmitted infections, contraception, alcohol and substance abuse, and mental
health care services.

It is important to inform an adolescent at the first encounter that confidentiality will need to be
breached if they are at risk of harming themselves (suicidal), or others (homicidal), or if they are
being abused. If the parents need to be involved, that decision should be made with the
knowledge of the adolescent and a plan should be made together as to how best to let the parents
know.

Guidance on dealing with an individual patient is influenced by the relationship with their
parents, legal constraints, nature of the behavior, and severity of the consequences. In addition,
the history may be helpful (le, have they exhibited poor decision making with serious
consequences to life and limb? Exhibited impulsiveness? Shown an ability to learn from past
mistakes?). This information might lead to the decision to override the patient's preference, ie,
justified paternalism. However, it is very important that this decision be documented in the
medical record.

PREP Pearls
• Ensuring time alone and confidentiality with an adolescent is crucial in eliciting sensitive
information.
• Confidentiality must be breached it the adolescent is at risk for harming themselves
(suicidal), others (homicidal), or if they are being abused.
• The plan for how confidential information will be shared with the parent should be made
with the adolescent, before disclosure.

American Board of Pediatrics Content Specification(s)


• Understand when a parent must be notified about an adolescent's medical condition

Suggested Reading
• American Academy of Pediatrics. Adolescent consent and confidentiality. Pediatr Rev.
2009;30(10:457-459. doi:10.1542/pir.30 -11-457.

American academy of pediatrics 475


American Academy of Pediatrics PREP 2015

• Cummings CL, Mercurio MR. Ethics for the pediatrician: autonomy, beneficence, and
rights. Pediatr Rev. 2010;31(6):252-255. doi:10.1542/pir.31- 6-252.
• Prager LM. Depression and suicide in children and adolescents. Pediatr Rev.
2009;30(6):199-206. doi:10.1542/pir.30-6-199.

American academy of pediatrics 476


American Academy of Pediatrics PREP 2015

Item 155
The father of a 6-year-old boy calls your office to report that the boy had an allergic reaction
while undergoing a dental filling for carious teeth. The boy had been very anxious about the
procedure. Within 2 min of receiving the injection of local anesthetic lidocaine, he complained of
feeling dizzy. His face turned pale and he was nauseated, according to what the father had been
told. No hives or wheezing was reported. The procedure was immediately stopped and the child
was given diphenhydramine and monitored. His symptoms improved and he was sent home after
spending an hour under observation. He had been on day 9 of a 10-day course of amoxicillin-
clavulanic acid for the tooth infection, and had also been taking ibuprofen for pain as needed for
the prior 3 days. He has had dental procedures done before without problems and has never had
any reactions with balloons or rubber gloves.

Of the following, the MOST likely cause of the child's reaction is


A. amoxicillin-clavulanic acid
B. ibuprofen
C. latex
D. lidocaine
E. vasovagal reaction

American academy of pediatrics 477


American Academy of Pediatrics PREP 2015

Item 155 Preferred Response: E


The most likely cause of the reaction for the child in the vignette is a vasovagal reaction.
Vasovagal syncope refers to syncope triggered by emotional or orthostatic stress such as
venipuncture, painful or noxious stimuli, fear of bodily injury, prolonged standing, heat
exposure, or exertion. This "classical" presentation commonly occurs in young and otherwise
healthy individuals. There is often a prodrome of nausea, pallor, and diaphoresis consistent with
increased vagal tone. Syncope is typically of short duration and occurs in the standing position.
The supine position restores adequate blood flow to the brain and reverses the temporary loss of
consciousness. This course may help distinguish vasovagal syncope from syncope associated
with arrhythmia, which is typically of abrupt onset and of short duration. Anaphylaxis is
typically associated with circulatory changes, such as tachycardia (initially) and hypotension,
respiratory findings, such as wheeze, and cutaneous symptoms, such as urticarial rashes.

The history, physical examination, and objective clinical and laboratory tests are important
components in the evaluation and diagnosis of drug hypersensitivity. The history should focus on
previous and current drug use, the toxicity and allergenicity of previously and currently used
drugs, and the temporal sequence of events between initiation of therapy and onset of symptoms.
Physical examination should include all systems that could possibly account for the clinical
presentation.

Immediate hypersensitivity type I reactions are immunoglobulin E (IgE)-mediated and result in


immediate reactions, such as anaphylaxis. These are exemplified by symptoms of urticaria,
laryngeal edema, wheezing, and cardiorespiratory collapse, which typically occur within min of
exposure to the drug. Cutaneous manifestations are the most common presentation for drug
allergic reactions. Although drug allergic reactions may present with noncutaneous physical
findings, these are generally nonspecific and are not nearly as helpful in diagnosis and
management decisions. A retrospective diagnosis of anaphylaxis may be determined by detecting
an increase in serum total tryptase levels above baseline.

The history of the boy in this vignette having tolerated the amoxicillin-clavulanic acid for 9 days
rules out immediate hypersensitivity to this agent as a cause of this child's symptoms.
Additionally, there were no cutaneous findings of urticarial rash, which is often seen in IgE-
mediated reactions. The rate of penicillin-induced anaphylaxis after parenteral administration is
about 1 to 2 per 10,000 treated patients. Approximately 10% of patients report a history of
reacting to a penicillin-Class antibiotic. However, when evaluated for penicillin allergy, up to
90% of these individuals are able to tolerate penicillins and therefore carry an erroneous label of
being "penicillin allergic."

The child is not latex allergic because he can tolerate exposure to other rubber products including
balloons and rubber gloves. The history of tolerating ibuprofen for 3 days or more and the
absence of symptoms of IgE-mediated allergy make allergy to ibuprofen and nonsteroidal anti-
inflammatory drugs less likely.

Possible systemic allergy to local anesthetics is often of concern to patients and their dentists or
physicians. Documentation of IgE-mediated reactions is extremely rare. Most adverse reactions
to local anesthetics are caused by nonallergic factors that include vasovagal responses, toxic or
American academy of pediatrics 478
American Academy of Pediatrics PREP 2015

idiosyncratic reactions because of inadvertent intravenous epinephrine, or anxiety. Local


anesthetics are either group 1 benzoic acid esters (eg, procaine, benzocaine) or group 2 amides
(eg, lidocaine, mepivacaine). On the basis of patch testing, the benzoic acid esters cross-react
with each other, but they do not cross-react with the group 2 amide drugs. It is not known what,
if any, relevance this has on immediate-type reactions to local anesthetics, though it has been
speculated that a person allergic to a benzoic acid ester may be able to tolerate an amide, and
vice versa.

PREP Pearls
• Immediate hypersensitivity to a drug would typically manifest with symptoms within
minutes to hours of being exposed to the medication with cutaneous symptoms of
urticarial rash. Systemic reactions such as laryngeal edema, wheezing, and
cardiorespiratory collapse would signal the onset of a severe systemic allergic reaction or
anaphylaxis.
• While anaphylactic reactions would present as tachycardia and hypotension, vasovagal
reactions would have bradycardia and hypotension.

American Board of Pediatrics Content Specification(s)


• Recognize the clinical features associated with a drug allergy or hypersensitivity, and
manage appropriately

Suggested Reading
• Joint Task Force on Practice Parameters; American Academy of Allergy, Asthma and
Immunology; American College of Allergy, Asthma and Immunology; Joint Council of
Allergy, Asthma and Immunology. Drug allergy: an updated practice parameter. Ann
Allergy Asthma Immunol. 2010;105(4):259-273. doi:10.1016/j.anai.2010.08.002.
• Task Force for the Diagnosis and Management of Syncope, European Society of
Cardiology (ESC), European Heart Rhythm Association (EHRA), et al. Guidelines for
the diagnosis and management of syncope (version 2009). Eur Heart J.
2009;30(21):2631-2671. doi:10.1093/eurheartj/ehp298.

American academy of pediatrics 479


American Academy of Pediatrics PREP 2015

Item 156
A previously healthy 4-week-old, full-term male patient is brought to the emergency department
because he stopped breathing for approximately 20 seconds and turned blue around the lips. He
has also had slight nasal congestion and poor oral intake today. His mother did not report any
complications during pregnancy. He was born at term and discharged home in 2 days. He usually
drinks four 6-oz bottles of formula per day, and today he is only finishing 2 oz of each bottle. His
2 older siblings have been ill with cough and rhinorrhea.
Vital signs show a temperature of 38.0°C, pulse rate of 150 beats/min, respiratory rate of 60
breaths/min, blood pressure of 80/40 mm Hg, and oxygen saturation of 93% on room air.
Physical examination shows a tired-appearing but well-developed, well-nourished baby. Anterior
fontanelle is soft, open, and flat. Eyes and oral mucous membranes are erythematous. Pupils are
2 mm, equal, and reactive. His heart has a regular rate and rhythm, and extremities are warm and
well-perfused, with capillary refill time of 2 s. Respiratory examination shows tachypnea with
mild intercostal retractions, nasal flaring, and coarse breath sounds bilaterally. Abdomen is soft,
nontender, nondistended, and with no organomegaly. Supplemental oxygen by nasal cannula is
initiated with deep suctioning of the nasopharynx and results in improvement of oxygen
saturation to 100%. The infant is observed to have 2 separate periods of apnea that last 15
seconds and are associated with a drop in oxygen saturation to 80% that resolves with gentle
stimulation.

Of the following, the MOST appropriate diagnostic test at this time is


A. computed tomography of brain
B. electroencephalogram
C. lumbar puncture
D. nasal secretions for respiratory syncytial virus
E. sleep study

American academy of pediatrics 480


American Academy of Pediatrics PREP 2015

Item 156 Preferred Response: D


The infant in this vignette most likely has apnea from a respiratory tract infection, based on the
history of being full term, previously healthy, with upper respiratory tract symptoms, decreased
oral intake, and the presence of sick contacts. The duration of the respiratory pause and perioral
cyanosis indicates a serious event of apnea. Although several different respiratory tract infections
can cause apnea, respiratory syncytial virus (RSV) is one of the most common. Evaluation of
nasal secretions for respiratory viruses is the most appropriate diagnostic test.

Apnea is the clinical condition of the absence of respiratory air flow. The 3 general categories of
apnea include central apnea from insufficient respiratory drive originating from the brainstem,
obstructive apnea from upper airway obstruction, and a mixed etiology including both central
and obstructive causes (Item C156). Central apnea in neonates can be caused by a relatively
immature brainstem respiratory center and is a very common problem in preterm infants. It can
also be caused by other illnesses that can affect the respiratory center in both newborns and older
children, such as brain hemorrhage, drugs, seizures, hypoxic injury, or increased intracranial
pressure. A mixed etiology of central and obstructive apnea is also common in premature
neonates, from the combination of an immature brainstem as well as an underdeveloped upper
airway. This problem usually resolves by 37 to 40 weeks of gestational age. Obstructive apnea,
which can be caused by airway obstruction at any point from the pharynx to the distal trachea,
can occur both in infants and older children. In infants, common causes include macroglossia,
Pierre-Robin sequence, subglottic stenosis, and laryngotracheomalacia, leading to intermittent
collapse of the airway. In older children, common causes of obstructive sleep apnea include
enlarged tonsils or adenoids, nasal polyps, obesity, and insufficient upper airway tone from
trisomy 21 or neuromuscular conditions. Central apnea can be distinguished from periodic
breathing, which can reflect short pauses in breathing, by the presence of apnea for periods of
longer than 15 to 20 seconds, cyanosis, hypotonia, bradycardia, or pallor.

Item Cl 56: Differential Diagnosis of Infantile Apnea


• Normal respiratory pauses
o Mild apnea associated with choking
o Periodic breathing during sleep
o Occasional 5- to 15-second apneas during sleep
• Acute illnesses associated with apnea
o Sepsis
o Acute neurologic syndromes
o Infantile botulism
o Respiratory syncytial virus infection
o Pertussis
o Other
• Chronic conditions associated with apnea
o Convulsions
o Gastroesophageal reflux, sensitive laryngeal chemoreceptors
o Cardiac dysrhythmias, "prolonged Q-T" syndrome
• Abnormalities of respiratory control
o Immature respiratory center
 Apnea of prematurity
American academy of pediatrics 481
American Academy of Pediatrics PREP 2015

 Excessive periodic breathing


o Respiratory center dysfunction
 Obstructive sleep apnea
 Idiopathic ventilation,
o "Ondine's curse"
 Drug-induced or posttraumatic hypoventilation/apnea
 Arnold-Chiari-associated apnea
 Leigh's syndrome-associated apnea
• Idiopathic apnea of infancy
Reprinted with permission. Mandell'. Infantile apnea. PethatrRev.19845;275

Diagnostic workup and treatment depends on possible causes and the age group. In infants,
neuroimaging may be warranted if the condition is severe, or if hemorrhage, hydrocephalus, or
other central nervous system anatomic abnormalities that could lead to central apnea are
suspected. However, central apnea in most premature and full-term infants generally improves
over time as the respiratory center of the brain-stem matures. Rigid or flexible endoscopy can be
helpful to diagnose anatomic conditions of upper airway obstruction, such as subglottic stenosis
or tracheo bronchomalacia. Apnea of prematurity can be treated with theophylline or caffeine.
Continuous positive airway pressure (CPAP) or high-flow nasal cannula can be used for mixed
or obstructive conditions. For older children, it should be stressed that obstructive sleep apnea is
not a benign condition. It can lead to impaired development from sleep disturbances, and if
severe, pulmonary hypertension and death. More than 75% of children who have obstructive
sleep apnea snore. Referral to pediatric pulmonology or otolaryngology may be warranted.
Possible interventions include nighttime CPAP, tonsillectomy, adenoidectomy, and weight loss,
where applicable.

Respiratory syncytial virus is a single-stranded RNA virus that commonly causes bronchiolitis in
children younger than 1 year of age, usually in yearly epidemics. Bronchiolitis generally causes
small airway obstruction from necrosis of the bronchiolar epithelium, edema, and increased
mucus production. Apnea is a common manifestation of RSV infection, especially in infants who
are younger than 6 weeks of age, formerly premature, or have other pre-existing chronic
illnesses. The cause of apnea in RSV is not completely understood, but could be at least partially
obstructive in nature because of nasopharyngeal and upper airway obstruction from increased
mucous production.

Although neuroimaging, electroencephalography, and lumbar puncture can be important in the


workup of central apnea in infants to rule out hemorrhage, seizure, or meningitis, these causes
are unlikely in the setting of an acute upper respiratory infection. A sleep study would be more
useful in older children or in cases of suspected obstructive sleep apnea.
The evaluation of apnea depends on the age group and acuity of presentation. Apnea caused by
respiratory infections in children generally resolves when the infection improves. Treatment of
apnea is generally supportive, but could include medications in apnea of prematurity, acute or
chronic positive pressure ventilatory support, or surgical correction of airway lesions.

PREP Pearls
• Causes of apnea in children can include central, obstructive, or mixed.
American academy of pediatrics 482
American Academy of Pediatrics PREP 2015

• Obstructive sleep apnea is not a benign condition. It can lead to impaired development,
pulmonary hypertension, cor pulmonale, and death.
• The pathogenesis of apnea from respiratory infections is not well understood, but
generally improves along with the infection. Evaluation of nasal secretions for respiratory
viruses is the diagnostic study of choice.

American Board of Pediatrics Content Specification(s)


• Plan the appropriate clinical and diagnostic evaluation of apnea of various etiologies
• Plan appropriate management for apnea of various etiologies

Suggested Reading
• Carlo WA. Apnea. In: Kliegman RM, Stanton BF, St. Geme 1W III,Schor NF, Behrman
RE, eds. Nelson's Textbook of Pediatrics. 19th ed.Philadelphia, PA: Saunders Elsevier;
2011:580-581.
• Darville T, Yamauchi T. Respiratory syncyt iaI virus. Perlin& Rev.1998;19(21:55-61.
doi:10.1542/pir.19-2-55.
• Fu LY, Moon RY. Apparent life-threatening events: an update. Pediatr
Rev.2012;33(8):361-369. doi:10.1542/pir.33-8-361-369.
• Matiz A, Roman EA. Apnea. Pediatt Rev. 2003;24(032-34. doi:10.1542/ pir.24-1-32.
• McBride JT. Infantile apnea. Pediarr Rev. 1984;5(9):275-284. doi:10.1542/

American academy of pediatrics 483


American Academy of Pediatrics PREP 2015

Item 157
The parents of a 17-year-old college student in your practice call to ask your advice. The student
awoke this morning to find a bat flying around his dormitory room. He and his roommate opened
the window and the bat flew out. He feels well and on self-examination noticed no skin lesions
or bite marks. His immunizations are up to date, including receipt of diphtheria-tetanus-pertussis
vaccine at 11 years of age.

Of the following, the BEST course of action is to


A. begin a 5-day course of amoxicillin-clavulanic acid
B. begin a rabies vaccine series
C. get a tetanus vaccine booster
D. monitor his skin for signs of new skin lesions over the next 2 weeks
E. provide reassurance

American academy of pediatrics 484


American Academy of Pediatrics PREP 2015

Item 157 S Preferred Response: B


Although rabies virus infection is rare in the United States, the vast majority of indigenously
acquired rabies in the United States is from bat exposure. A scratch or bite from a bat may not be
readily apparent. In settings in which the contact is unknown, such as an unattended child in a
room with a bat or, as in this vignette, sleeping in a room in which a bat is found, potential rabies
exposure must be considered. If the animal is captured, it can be euthanized and its brain
examined for rabies virus. If not (or if the brain examination is positive), institution of rabies
prophylaxis is recommended. Present guidelines call for a combination of human rabies immune
globulin (HRIG) and vaccine given on day 0 and subsequent doses of rabies vaccine on days 3,
7, and 14. If apparent, infiltration with as much of the HRIG as possible around the bite site is
recommended. The remainder of the dose is given intramuscularly (IM). The vaccine is
administered IM in the deltoid or anterior aspect of the thigh. Once signs or symptoms of rabies
appear, HRIG and vaccine are not of benefit.

All mammals are potentially at risk for acquiring rabies with bats, raccoons, skunks, foxes,
coyotes, and bobcats being the primary potential sources for human and animal exposure in the
United States. Lagamorphs (eg, rabbits, hares) and rodents (eg, squirrels, hamsters, guinea pigs,
gerbils, rats, mice) rarely transmit rabies.

Secondary bacterial infection maybe a concern with larger animal bites, but is not a significant
risk from a bat exposure, as reported in this vignette, making administration of antibiotics
unnecessary. Similarly, assessing the need for tetanus prophylaxis may be indicated after a dirty
wound from a larger animal, but not with a bat exposure with no visible lesion. Local reactions
are not a hallmark of bat bites such that monitoring for skin lesions over time is of no clinical
utility. Reassurance would not be appropriate here because, as noted previously, finding a bat in
a room with someone who has been sleeping there is considered adequate exposure to rabies and
prophylaxis as outlined above is indicated.

PREP Pearls
• Bat exposure is the leading cause of potential transmission of rabies in the United States.
Bat bites are small and may not be felt in the setting of an unattended child or someone
asleep in a room in which a bat is found.
• Rabies prophylaxis consists of a dose of human rabies immune globulin and rabies
vaccine on day 0 and additional doses of vaccine on days 3, 7, and 14.
• All mammals may become infected with rabies with bats, raccoons, skunks, foxes,
coyotes, and bobcats as the leading sources in the United States. Lagamorphs (eg, rabbits,
hares) and rodents rarely transmit rabies.

Suggested Reading
• American Academy of Pediatrics. Rabies. In: Pickering LK, Baker CJ, Kimberlin DW,
Long SS, eds. Red Book: 2012 Report of the Committee on Infectious Diseases. 29th ed.
Elk Grove Village. IL: American Academy of Pediatrics; 2012:600-607.
• Rupprecht CE, Briggs D, Brown CM, et al. Use of a reduced (4-dose) vaccine schedule
for postexposure prophylaxis to prevent human rabies. Recommendations of the

American academy of pediatrics 485


American Academy of Pediatrics PREP 2015

Advisory Committee on Immunization Practices. Mortal Morbid Weekly Rep.


2010;59(RR-2):1-12. Updated March 19, 2010. Accessed February 24, 2014.

American academy of pediatrics 486


American Academy of Pediatrics PREP 2015

Item 158
You are evaluating a 4-month-old during a health supervision visit. She was born at 29 weeks'
gestation with a weight of 1,250 g. She has chronic lung disease of prematurity. Her mother says
that she is feeding every 3 to 4 hours without breathing problems and has 8 to 10 wet diapers a
day. Her current medications include furosemide and thiazide. Her evaluation reveals an afebrile
infant, heart rate of 110 beats/ min, respiratory rate of 38 breaths/min, blood pressure of 80/40
mm Hg, with normal physical examination findings and growth parameters for her gestational
age. The mother is concerned because she read on the Internet that prolonged diuretic use will
increase her child's risk of kidney stones.

Of the following, the BEST medication choice to address the mother's concern is
A. acetazolamide
B. bumetanide
C. furosemide
D. thiazide
E. triamterene

American academy of pediatrics 487


American Academy of Pediatrics PREP 2015

Item 158 Preferred Response: D


Thiazide diuretics decrease calcium excretion in the urine. Thiazides decrease sodium/chloride
absorption in the distal tubule. The ensuing diuresis leads to increased calcium absorption in the
proximal tubule resulting in a significant fall in calcium excretion in the urine. Therefore, for the
child in the vignette, thiazide diuretics would decrease the risk of kidney stones.

Although multiple factors have been associated with nephrocalcinosis in neonates, prescription
of loop diuretics (furosemide, bumetanide) for managing bronchopulmonary dysplasia is a major
risk factor. Loop diuretics increase the excretion of calcium in the urine, leading to an increased
risk for calcium deposits in the renal parenchyma. Other factors contributing to neonatal
nephrocalcinosis include: (1) increased urinary phosphate excretion, (2) hypercalcemia between
32 and 40 weeks of gestation, (3) immature renal tubular function in preterm neonates associated
with alkaline urine pH and low urinary citrate concentration (risk factors for calcium phosphate
precipitation), and (4) renal injury secondary to ischemia or nephrotoxic medications leading to
tubular injury, which promotes calcium deposition.

Acetazolamide is a proximal tubule carbonic anhydrase inhibitor. Its use leads to an elevation in
urinary pH, which is a risk factor for calcium stones. Increased risk for hypercalciuria and
nephrocalcinosis has been reported in neonates receiving loop diuretics and acetazolamide
concomitantly.

Triamterene is a potassium-sparing diuretic used for treating hypertension in combination with


thiazide diuretics. Potential nephrotoxicity has been reported with triamterene including an
increased risk for crystalluria, kidney stones, cast formation, or reversible acute kidney injury.

PREP Pearls
• Thiazide diuretics decrease urinary calcium excretion by increasing absorption of calcium
in the proximal renal tubule.
• The use of loop diuretics (furosemide, bumetanide) is a risk factor for nephrocalcinosis.
• Loop diuretics increase the excretion of calcium in urine, leading to increased risk for
calcium deposits in the renal parenchyma.
• Acetazolamide, a proximal tubule carbonic anhydrase inhibitor, elevates urinary pH,
which is a risk factor for calcium stones.
• Triamterene increases the risk of crystalluria, kidney stones, cast formation, or reversible
acute kidney injury.

Suggested Reading
• Hufnagle KG, Khan SN, Penn D, Cacciarelli A, Williams P. Renal calcifications: a
complication of long-term furosemide therapy in preterm infants. Pediatrics.
1982;70(3):360. Accessed February 27, 2014.
• Itami N, Kawaguti M, Akutsu Y, Mizutani K, Tochimaru H, Takekoshi Y. Renal
calcifications in preterm infants. 1 Pediatr, 1989;I14(6):1068.
• Kair LR, Leonard DT, Anderson JM. Bronchopulmonary dysplasia. Pediatr Rev.
2012;33:255-264. doi:10.1542/pir.33-6-255.

American academy of pediatrics 488


American Academy of Pediatrics PREP 2015

• Lemann J Jr, Gray RW, Maierhofer WJ, Cheung HS. Hydrochlorothiazide inhibits bone
resorption in men despite experimentally elevated serum 1,25-dihyclroxyv ita min D
concentrations. Kidney Int. 1985;28(6):951-958.
• Sica DA, Gehr TW. Triamterene and the kidney. Nephron. 1989;51(4):454-46L

American academy of pediatrics 489


American Academy of Pediatrics PREP 2015

Item 159
You are asked to see a 3-week-old neonate who has just been hospitalized because of an apparent
life-threatening event. He was delivered at term following an uncomplicated pregnancy. He
weighed 2,800 g. He was started on a soy protein-based formula at birth because of a family
history of allergy to cow-milk protein. At about 2 weeks of age, he developed postprandial
emesis and was switched to a hydrolyzed protein-based formula. However, the vomiting
episodes have persisted, and the baby spits up once or twice after each feeding. He currently
consumes 6 oz of formula every 3 hours to 4 hours. Several hours ago, immediately after
feeding, the baby appeared to be choking, stopped breathing, and developed perioral cyanosis.
His mother slapped him on his back and formula was expelled from his nose and mouth. The
parents called 911, and the infant was brought to the emergency department, where he appears
active and alert. His weight is 3,700 g, temperature is 37.0°C, respiratory rate is 30 breaths/min,
pulse rate is 130 beats/min, and oxygen saturation is 99% on room air. Physical examination
findings are unremarkable. In the emergency department, blood, urine, and spinal fluid cultures
are obtained, and he is started on intravenous antibiotics and admitted for observation and
continued care.

Of the following, the MOST appropriate next step is


A. a barium upper gastrointestinal tract series
B. intraesophageal pH monitoring
C. reduced feeding volumes
D. treatment with an H2-receptor antagonist
E. treatment with a proton-pump inhibitor

American academy of pediatrics 490


American Academy of Pediatrics PREP 2015

Item 159 Preferred Response: C


In the vignette, a 3-week-old newborn presents with an apparent life-threatening event. This is an
observer-dependent phenomenon, characterized by a combination of apnea, abnormal muscle
tone (limpness, rigidity), choking, and color change (pallor, cyanosis, plethora). Apparent life-
threatening events typically are reported in infants at 1 to 2 months of age, and are rarely
described after 8 months of age. Although specific etiologies are poorly understood,
gastroesophageal reflux (GER) has been causally implicated. However, supportive data are
conflicting, and the relationship between apnea and GER remains controversial. Accordingly, in
this infant whose clinical history otherwise suggests a diagnosis of uncomplicated GER, which
may be exacerbated by overfeeding, current best evidence indicates that the most appropriate
approach to management would include a reduction in feeding volumes.

Gastroesophageal reflux is defined by the passage of gastric contents into the esophagus, with or
without regurgitation or vomiting. It is a normal physiologic process occurring several times per
day in healthy infants, children, and adults. Most episodes are transient, lasting less than 3
minutes, occurring in the initial 2 hours postprandially, and followed by rapid esophageal
clearance of the refluxate. These brief, recurring GER events are generally unassociated with
clinical signs or symptoms. During the first 3 months after birth, regurgitation (defined as
effortless passage of gastric contents into the mouth) or vomiting events (which may be
projectile) are noted daily in 50% of infants. These GER episodes arise from several potential
anatomic and physiologic mechanisms, the most prominent being repeated, transient relaxation
of the lower esophageal sphincter (LES). Available reports show that 60% to 70% of infants
experience more than 1 episode of emesis per day by 3 to 4 months of age. Resolution of GER
symptoms occurs in most infants by 1 year of age. The emesis prevalence rate decreases to 10%
by 7 to 9 months of age and to 5% by 10 to 12 months of age. As in the vignette, a diagnosis of
GER in infants is based solely on a history of spitting-up gastric contents. Formal
intraesophageal pH monitoring is of no value in routine evaluation. This study should be
reserved for assessing equivocal reflux cases, or in an attempt to correlate symptoms with reflux
episodes. A barium esophagram cannot quantify the extent or severity of GER, and is of use
solely to document anatomic integrity.

Symptoms and signs that have been associated with prolonged or increased GER include both
respiratory and nonrespiratory events, as listed in Item C159. Although regurgitation or vomiting
are not consistent findings in older patients with GER-related complications, spitting-up, with or
without expulsion of gastric contents from the mouth, is considered a necessary condition for
reaching a clinical diagnosis of reflux during infancy.

Early studies suggest that a potential mechanism for reflux induced apnea involved acid
stimulation of pharyngeal and esophageal chemoreceptors, leading to laryngospasm. However,
more recently, large case series have failed to demonstrate a consistent GER-apnea link. Where
respiratory status was monitored along with both esophageal pH and bioelectrical impedance in
infants presenting with a history of apnea, only 15% of apneic episodes were correlated with
GER. Furthermore, these episodes were as likely to occur with nonacid as with acid GER.
Additional data have failed to demonstrate any clinical efficacy of acid reduction therapy in
preventing or ameliorating apnea events. Thus, acid blockade with either an H2-receptor
antagonist or a proton pump inhibitor is not indicated in this clinical setting.
American academy of pediatrics 491
American Academy of Pediatrics PREP 2015

Gastroesophageal reflux has also been implicated as a causative or exacerbating factor for other
respiratory disorders. When studied using esophageal pH monitoring, 60% to 70% of children
with reactive airway disease demonstrate pathological reflux. However, it is unclear whether this
degree of GER is a primary problem or a secondary phenomenon caused by lung hyperinflation
and downward movement of the diaphragm leading to upward displacement of the LES into the
chest, accompanied by a reduction in LES pressure. Several studies have attempted to assess the
role of acid reduction therapy in ameliorating asthma symptoms in both children and adults, and
most have failed to demonstrate consistent treatment efficacy. However, a few reports have
shown improvement in asthma symptoms following acid blockade for patients with poorly
controlled asthma, particularly those subjects manifesting predominantly nocturnal symptoms.
Gastroesophageal reflux has been implicated in the etiology of recurrent pneumonia and
interstitial lung disease. Although these complications have been reported in otherwise healthy
infants and children, they are common problems in children with significant neurological
impairment. In these cases, lung disease is presumably the consequence of failure of normal
airway mechanisms to protect the lungs from aspirated gastric contents. Several case series have
presented conflicting results regarding the efficacy of either medical or surgical GER therapy in
improving lung function and reducing the risk of pneumonia in affected patients.
Gastroesophageal and respiratory function are closely linked, and therefore the finding of
pathological GER in patients with chronic pulmonary disease is not surprising. In 1 report, 27%
patients with cystic fibrosis (CF) reported symptoms of heartburn. However, when studied by
esophageal pH monitoring, the prevalence of pathological GER in CF was even higher.
Accordingly, GER may be considered an exacerbating factor for children and adults with a wide
range of chronic respiratory disorders. Again, the effectiveness of acid blockade on respiratory
symptoms in these patients has not been clearly demonstrated.

American academy of pediatrics 492


American Academy of Pediatrics PREP 2015

PREP Pearls
• During infancy, the presence of emesis or regurgitation is required to make a diagnosis of
gastroesophageal reflux.
• A clear relationship between gastroesophageal reflux and apnea has not been established,
and acid suppression has not been shown to alter the clinical course related to apnea
during infancy.

American academy of pediatrics 493


American Academy of Pediatrics PREP 2015

• In patients with poorly controlled asthma, acid suppression therapy may be useful in the
setting of predominantly nocturnal asthmatic symptoms.

American Board of Pediatrics Content Specification(s)


• Recognize the clinical features associated with gastroesophageal reflux, including
symptoms associated with other organ systems (eg, respiratory)
• Understand the prognosis for patients who have gastroesophageal reflux

Suggested Reading
• Akinola E, Rosenkrantz TS, Pappagallo M, McKay K, Hussain N. Gastroesophagea/
reflux in infants c 32 weeks gestational age at birth: lack of relationship to chronic lung
disease. Am f Perinatol. 2004;21(2):57-62. doi:10.1055/s-2004-820512.
• American Academy of Pediatrics. EQIPP. GER or GERD? Diagnosis and management.
2014. Expires 5/1/2017.
• Condino AA, Soridheimer 1, Pan 2, Gralla 1, Perry 0, O'Connor IA. Evaluation of
gastroesophageal reflux in pediatric patients with asthma using impedance-pH
monitoring. J Pediarr. 2006;149(2):216-219. doi:10.1016/j4peds.2006.03.022.
• Mousa H, Woodley FW, Metheney M, Hayes I. Testing the association between
gastroesophageal reflux and apnea in infants. f Pediarr Gastroenterol Nutt
2005;41(2):169-177.
• Stordal K, Johannesdottir GB, Bentsen BS, et at. Acid suppression does not change
respiratory symptoms in children with asthma and gastro -oesophageal reflux disease.
Arch Dis Child. 2005;9G(9):956-960. doi:10.1136/adc.2004.068890.
• Vandenplas 1/, Hauser B. Gastro-oesophageal reflux, sleep pattern, apparent life
threatening event and sudden infant death: the point of view of a gastro-enterologist. Eur
f Pediarr. 2000;159(10726-729. doi:10.1007/ s004310000544.
• Vandenplas Y, Rudolph CD, Di Lorenzo C, et al. Pediatric gastroesophageal reflux
clinical practice guidelines: joint recommendations of the North American Society for
Pediatric Gastroenterology, Hepatology, and Nutrition (NASPGHAN) and the European
Society for Pediatric Gastroenterology, Hepatology, and Nutrition (ESPGHAN). J Pediatr
Gastroenteral IVutr. 2009;49(0498-547. doi:10.1097/MPG.0b013e3181b7f563.

American academy of pediatrics 494


American Academy of Pediatrics PREP 2015

Item 160
A 6-month-old infant is brought into the emergency department in the morning by his mother
due to progressive lethargy, vomiting, and diarrhea. She reports he developed a low-grade fever
about 18 hours to 24 hours prior to arrival. He has been unable to take his normal feeds during
this time, so she has been supplementing with oral electrolyte solution with limited success.

When she put him to bed last night, he was tired but interactive. This morning, he is difficult to
arouse and not making good eye contact. You are immediately concerned about dehydration.
You order laboratory tests to be drawn as the intravenous line is placed.

Vital signs show a temperature 37.6°C, heart rate of 145 beats/min, respiratory rate of 32
breaths/min, and blood pressure of 88/53 mm Hg. On physical examination, you note an ill-
appearing, lethargic infant with a sunken fontanelle, decreased tearing, and dry skin with tenting.
Liver margin is palpable 2 cm below the costal margin. Mild hypotonia is present. He is also
difficult to arouse. Blood glucose level at time of intravenous placement is 30 mg/dL (1.7
mmol/L). Urinalysis shows no ketones or signs of infection. Complete blood cell count and C-
reactive protein tests are unremarkable. Serum ketones are 0.2 mg/dL (0.02 mmol/L, reference
range, 0.5-3.0 mg/dL [0.05-0.30 mmol/L]). Serum bilirubin is 1.0 mg/dL (17.1 mmol/L).
Aspartate aminotransferase and alanine aminotransferase levels are 180 U/L and 192 U/L,
respectively. Ammonia level is 175 ug/dL (125 µmol/L). Blood, urine, and cerebrospinal fluid
cultures are pending. Family history is remarkable for a prior sibling that died of sudden infant
death syndrome at 5 months of age after a viral illness. Past medical history is unremarkable.
The infant is begun on broad spectrum antibiotics. Based on the examination and studies
obtained thus far, you highly suspect an inborn error of metabolism. You immediately begin a
10% dextrose solution at an infusion rate of 10 mg/ kg per min, with close monitoring of the
blood glucose level. Metabolic studies are pending.

Of the following, the MOST likely diagnosis is


A. hereditary tyrosinemia type 1
B. maple syrup urine disease
C. medium-chain acyl-CoA dehydrogenase deficiency
D. peroxisomal biogenesis disorder
E. phenylalanine hydroxylase deficiency

American academy of pediatrics 495


American Academy of Pediatrics PREP 2015

Item 160 Preferred Response: C


The patient in the vignette, has medium-chain acyl-CoA dehydrogenase deficiency (also known
as MCAD), which is the most common defect in the fatty acid oxidation pathway. This enzyme's
role is to fuel hepatic ketogenesis. It is an autosomal recessive inborn error of metabolism that
primarily affects white patients of northern European origin. Medium-chain acyl-CoA
dehydrogenase deficiency is a hepatic fatty acid oxidation disease that frequently presents with
episodic hypoketotic hypoglycemia with hyperammonemia and elevated liver function tests. It is
frequently noted with fasting or illness and usually manifests in the first 2 years after birth. A
child may present with vomiting and lethargy after a period of fasting, or a history of a viral
infection accompanied by poor oral intake. Often, the patient has hepatomegaly or even seizures
during the acute episode. It has been found to be a cause of a small percentage of sudden infant
death syndrome. Roughly 20% of previously unrecognized patients with MCAD die with the
first episode if not treated promptly, However, since its inclusion in newborn screening, most
patients are diagnosed in the neonatal period and treated appropriately, thus decreasing mortality
and eliminating many sudden death events. Medium-chain acyl-CoA dehydrogenase deficiency
can also be present with a Reye like syndrome or a comatose presentation, thus illustrating its
varying presentation in families. Therapy is aimed at avoidance of fasting, which is crucial in
infancy, and immediate treatment of acute illness episodes with intravenous infusion of 10%
glucose and oral L-carnitine (100-200 mg/kg per day). In conjunction with the avoidance of
fasting, most infants and toddlers are treated with daily carnitine supplementation and a mild
reduction of dietary fat to about 20% of total calories. Most acute episodes occur in early
childhood, with fewer events noted after age 4 years. It is imperative that siblings of affected
children be screened as well. Heterozygous carriers are not symptomatic.

Although MCAD is part of the newborn screening evaluation, a biochemical diagnosis requires
urine acylglycines, urine organic acids, and an acylcarnitine profile that typically shows
elevation of the C6:0, C8:0, and C10:0 acylcarnitine species. The acylcarnitine profile is
diagnostic whether the child is sick or well. Definitive diagnosis is via MCAD enzymatic activity
measurement in fibroblasts, fatty acid β-oxidation determination in fibroblasts, or molecular
sequencing of the ACADM gene.

Mitochondrial β-oxidation has a major role in the body's energy production, especially during
fasting. This pathway is quite complicated, with more than 20 individual steps and up to 11
inherited defects in proteins affecting this process in humans. Disorders include carnitine
transporter deficiency; carnitine palmitoyltransferase (CPT) I and II deficiency;
carnitine/acylcarnitine translocase deficiency; very long-chain, medium-chain, and short-chain
acyl-CoA dehydrogenase deficiencies; and long-chain 3-hydroxyacyl-CoA dehydrogenase
(LCHAD) and short-chain 3-hydroxyacyl-CoA dehydrogenase deficiencies. Other disorders in
the fatty acid oxidation pathway are manifest by skeletal or cardiac muscle weakness, such as
very long-chain acyl-CoA dehydrogenase deficiency, LCHAD, CPT II deficiency, short-chain
acyl-CoA dehydrogenase deficiency, carnitine transporter defects, and carnitine/acylcarnitine
translocase deficiency.
Very long-chain acyl-CoA dehydrogenase deficiency has 2 phenotypic clinical presentations:
hypertrophic cardiomyopathy and a less severe form with recurrent hypoglycemia. Carnitine
deficiency is the primary feature of the carnitine transporter defect, but is a secondary
manifestation of all the other fatty acid oxidation disorders, with the exception of CPT I
American academy of pediatrics 496
American Academy of Pediatrics PREP 2015

deficiency, which is associated with elevated carnitine levels. Long-chain 3-hydroxyacyl-CoA


dehydrogenase in a fetus has been shown to be strongly associated with HELLP (hemolysis;
elevated liver enzymes; low platelet count) syndrome and acute fatty liver in the mother during
pregnancy, at times necessitating emergency delivery in the final trimester.

Classic maple syrup urine disease (MSUD) in untreated infants presents with an unusual maple
syrup smell in the cerumen, with progressive evolution of ketonuria, irritability, decreased
feeding, and encephalopathy, quickly evolving to coma and respiratory deterioration if it is not
immediately recognized. By the end of the first day after birth, elevated branched-chain amino
acids (leucine, isoleucine, and valine) and allo-isoleucine are noted in the plasma, as well as
elevated branched-chain hydroxyacids and ketoacids in the urine. This is identified on newborn
screening via tandem mass spectrometry.

Hereditary tyrosinemia type 1, if untreated, presents in the first year with progressive liver
involvement and renal tubular acidosis in association with poor growth, rickets, and episodic
neurologic crises. It is treated with nitisinone and a low-tyrosine diet that greatly affects overall
survival. If untreated, many patients die by 10 years of age from liver failure, liver cancer, or
neurologic crises.

Peroxisomal biogenesis disorders are a continuum of disorders that include Zellweger syndrome,
neonatal adrenoleukodystrophy, and infantile Refsum disease. Patients with Zellweger syndrome
present in the neonatal period with low tone, poor feeding, distinctive facies, liver dysfunction,
and seizures, progressing to death by the end of the first year. Neonatal adrenoleukodystrophy
and infantile Refsum disease present with hearing loss, retinal dystrophy, developmental delay,
low tone, and liver dysfunction at later ages with slow progression. Elevated plasma very-long-
chain fatty acids are classic on laboratory analysis.

Phenylalanine hydroxylase deficiency is caused by the inability to metabolize dietary


phenylalanine, which can lead to irreversible brain damage over time. It is asymptomatic in the
neonatal period, but as time progresses, the patient develops microcephaly, eczema, seizures, a
musty body odor, reduced skin and hair pigmentation, behavioral issues, and severe intellectual
deficits. It is identified on newborn screening and treated with strict adherence to a low protein
diet and specialized medical formulas that are phenylalanine-free.

PREP Pearls
• Medium-chain acyl-CoA dehydrogenase deficiency (MCAD) is a hepatic fatty acid
oxidation disorder presenting with episodic, hypoketotic (low ketones in blood and urine)
hypoglycemia along with hyperammonemia, and elevated liver function tests in the face
of a prolonged fasting episode or intercurrent
• Medium-chain acyl-CoA dehydrogenase deficiency is mandatorily screened for on state
newborn screening profiles because it is a treatable disorder that can be fatal if not
immediately treated and proactively recognized.
• Treatment involves intravenous infusion of 10% dextrose and carnitine supplementation.
Do NOT delay treatment while awaiting diagnostic confirmation if MCAD is suspected
because it could have fatal consequences for the patient.

American academy of pediatrics 497


American Academy of Pediatrics PREP 2015

• Fatty acid oxidation disorders can affect heart, liver, or skeletal muscle, depending on the
specific type of fatty acid defect.

American Board of Pediatrics Content Specification(s)


• Recognize the clinical features associated with disorders of fatty acid and carnitine
metabolism
• Recognize the laboratory features associated with disorders of fatty acid and carnitine
metabolism

Suggested Reading
• Matern D, Rinaldo P. Medium-chain acyl-coenzyme A dehydrogenase deficiency.
GeneReviews.
• Morris AAM, Spiekerkoetter U. Disorders of mitochondrial fatty acid oxidation and
related metabolic pathways. In: Fernandes J, Saudubray J-M, van den Berghe G. Walter
JH, eds. Inborn Metabolic Diseases: Diagnosis and Treatment. 5th ed. Heidelberg,
Germany: Springer-Verlag; 2012:201-214.
• Strauss AW, Andresen BS, Bennett Ml. Mitochondrial fatty acid oxidation defects. In:
Sarafoglou K, Hoffmann GF, Roth K, eds. Pediatric Endocrinology and Inborn Errors of
Metabolism. New York, NY: McGraw-Hill; 2008:17-32.

American academy of pediatrics 498


American Academy of Pediatrics PREP 2015

Item 161
The parents of a 7-year-old boy come to your office for a referral to otolaryngology to discuss
tonsillectomy and adenoidectomy. The patient has a history of recurrent otitis media and
tympanostomy tubes were placed at 3 years of age. He has had fewer ear infections since, but his
mother attributes his difficulty with speaking clearly to problems with hearing. On physical
examination, you notice that the patient has hypernasal speech that is difficult to understand and
notching of the posterior border of the hard palate. The patient is comfortable breathing through
his nose and the remainder of his physical examination is unremarkable. A hearing screen done
in your office today is within normal limits.

Of the following, your BEST recommendation for the patient at this time is that
A. adenoidectomy is contraindicated
B. antibiotic prophylaxis is recommended until surgery
C. recurrent otitis media necessitates tonsillectomy and adenoidectomy
D. referral to a new speech therapist is needed
E. a trial of a nasal decongestant spray is warranted

American academy of pediatrics 499


American Academy of Pediatrics PREP 2015

Item 161 Preferred Response: A


Recurrent acute otitis media and chronic otitis media with effusion and hearing loss are
indications to consider referral for tonsillectomy and adenoidectomy (T&A). It is important that
pediatricians are aware of the contraindications and complications of these procedures. The boy
in the vignette exhibits the characteristics of submucous cleft palate (SMCP), so adenoidectomy
is contraindicated.

The oropharynx should be examined for SMCP in all children for whom adenoidectomy is being
considered. Submucous cleft palate should be suspected in patients with notching or palpation of
a V shape along the junction of the hard and soft palates. The classic characteristics of SMCP
includes the triad of bifid uvula, diastasis of the muscles in the midline of the soft palate with
intact mucosa, and notching of the posterior border of the hard palate. Only a small percentage of
cases of SMCP are symptomatic, but one of the most common consequences of SMCP is
recurrent acute otitis media or chronic serous otitis media. Adenoidectomy is contra-indicated in
these patients because of the risk of acquired velopharyngeal insufficiency (VPI) after the
procedure and worsening of an existing speech problem. Velopharyngeal insufficiency
sometimes develops after adenoidectomy in children without predisposing factors.
Additional contraindications to T&A include other velopharyngeal conditions that predispose to
VPI (cleft palate, impaired palatal function because of neuromuscular causes), anemia
(hemoglobin < 10 g/dL [100 g/L), disorders of hemostasis, and acute local nonobstructive
infection of the upper airway.

Fortunately, the operative complications of T&A are rare, but they may be life threatening.
These include anesthesia-related problems, such as difficulties with airway management, local
damage caused by endotracheal intubation, aspiration, arrhythmias, adverse reactions to the
anesthetic agent including malignant hyperthermia, and hemorrhage. The most common
postoperative complications are throat pain, otalgia, and general anxiety or sleep disturbance.
The incidence of postoperative hemorrhage is low, but it can be serious. Dehydration is a risk in
children with persistent vomiting or refusal to swallow because of pain. Infection, upper airway
obstruction, and VPI are less common complications. Children younger than 3 years of age,
those with severe obstructive sleep apnea, coagulopathy, or other comorbid conditions
(craniofacial or neuromuscular disorders, sickle cell disease, prematurity, morbid obesity) benefit
from postoperative observation in the hospital.

Pain relief is an important part of postoperative care, and acetaminophen or a nonsteroidal anti-
inflammatory drug such as ibuprofen should be offered. Aspirin and ketorolac are
contraindicated because of the increased risk of hemorrhage. In 2013, the US Food and Drug
Administration added a black box warning that codeine should not be used for pain in children
after tonsillectomy or adenoidectomy because its use may lead to rare, but life-threatening
adverse events or death in certain children.

The vignette highlights the importance of awareness of all these contraindications and
complications. Although T&A may be considered in patients with recurrent otitis media, it is not
necessary and is contraindicated in the boy in the vignette. Antibiotic prophylaxis or the use of
nasal decongestant sprays is not clearly indicated from the existing evidence, especially in this
asymptomatic boy with normal hearing. Hypernasal speech may be associated with VPI, but
American academy of pediatrics 500
American Academy of Pediatrics PREP 2015

other articulation problems are not common and ongoing speech therapy may be helpful. A
change in therapist is not warranted.

PREP Pearls
• Contraindications to tonsillectomy or adenoidectomy include velopharyngeal conditions
that predispose to velopharyngeal insufficiency, anemia (hemoglobin < 10 g/dL [100
g/l]), disorders of hemostasis, and acute nonobstructive upper airway infection.
• Life-threatening complications associated with tonsillectomy or adenoidectomy are rare.
• Throat pain, otalgia, anxiety, or sleep disturbance are the most common postoperative
complications.
• Codeine should not be used for pain management in children after tonsillectomy or
adenoidectomy.

American Board of Pediatrics Content Specification(s)


• Recognize complications associated with tonsillectomy and/or adenoidectomy, including
those associated with velopharyngeal insufficiency

Suggested Reading
• Isaacson G. Tonsillectomy care for the pediatrician. Pediatrics. 2012;130(2):324-334.
doi:10.1542/peds.2011-3857.
• Paradise JL. Tonsillectomy and adenoidectomy in children. UpToDate. Available online
only for subscription.
• US Food and Drug Administration. FDA drug safety communication: safety review
update of codeine use in children: new boxed warning and contraindication on use after
tonsillectomy and/or adenoidectomy. US Food and Drug Administration website.
http://www.fda.gov/Drugs/ DrugSafety/ucm339112.htm.
• US Food and Drug Administration. FDA drug safety communication: codeine use in
certain children after tonsillectomy and/or adenoidectomy may lead to rare, but life-
threatening adverse events or death. US Food and Drug Administration website.
• Wetmore RF. Tonsils and adenoids. In: Kliegman RM, Stanton BF, St. Geme JW III,
Schor NF, Behrman RE, eds. Nelson Textbook of Pediatrics. 19th ed. Philadelphia, PA:
Saunders Elsevier; 2011:1442-1445.

American academy of pediatrics 501


American Academy of Pediatrics PREP 2015

Item 162
A 17-year-old adolescent boy presents to your clinic for left leg pain and swelling. One month
ago, he began to have intermittent pain. The pain has been continuous for the past 3 days and has
limited his ability to walk. He plays on the varsity football team, but does not recall any direct
trauma to that area of the leg. In the office, he is afebrile and has normal vital signs. He is alert
and in no apparent distress. There is marked swelling of the left distal thigh, but no bruising or
discoloration of the skin. His dorsalis pedis pulses are strong and symmetric. His
cardiopulmonary examination is unremarkable. You order a radiograph of his leg (Item Q162).
Of the following, the MOST likely diagnosis is
A. healing fracture
B. osteomyelitis
C. osteoid osteoma
D. osteosarcoma
E. vascular malformation

American academy of pediatrics 502


American Academy of Pediatrics PREP 2015

Item 162 Preferred Response: D


The adolescent in this vignette most likely has a malignant bone tumor, such as osteosarcoma
(OS), which is the most common malignant bone tumor in children. The incidence is 4.4 per
million per year with the peak incidence seen in the second decade of life during the adolescent
growth spurt. Some familial cancer syndromes such as hereditary retinoblastoma, Rothmund-
Thomson syndrome, and Li-Fraumeni syndrome may predispose affected individuals to OS, but
most cases are not associated with known germline abnormalities. Osteosarcoma typically
involves the long bones, most commonly around the knee joint, with the distal femur and
proximal tibia being the most frequent sites of involvement. In decreasing frequency, the other
common sites of OS are the distal humerus, middle and proximal femur, pelvis, and jaw.
Approximately 15% to 20% of patients with OS present with observable metastatic disease on
initial staging evaluation, most frequently in the lungs and other areas in the bone. However, all
patients with OS are presumed to have micrometastatic disease at diagnosis. This is based on the
finding that with amputation alone, 80% of patients will have disease recurrence. Therefore, all
patients with OS require systemic chemotherapy, in addition to surgical excision of the tumor.
Extent of disease at diagnosis is the most important prognostic factor. The primary site of the
tumor is also an important variable, because it can affect the resectability of the tumor. Most
patients present with pain and swelling in the affected bone. Symptoms can vary in duration
before presentation. Respiratory symptoms, fever, and weight loss are rare, but can occur with
extensive metastases. Multi-agent chemotherapy and complete surgical resection (most
frequently with a limb salvage procedure) are the mainstays of treatment.

In adolescents with a suspected bone tumor, plain radiographs of the affected area should be the
initial step in evaluation. Malignant bone tumors usually reveal a mixed lytic and sclerotic lesion,
a soft tissue mass with ossification, and periosteal new bone formation with lifting of the cortex,
known as the Codman triangle (Item C162).

The adolescent in the vignette presents with worsening leg pain that started 1 month ago with
radiographic evidence of a large mixed lytic and sclerotic lesion, known as a Codman triangle.
Therefore, OS would be the most likely diagnosis. Although pathologic fractures can occur with
bone tumors, no old or recent fracture is visible on radiography. The absence of fever makes the
diagnosis of osteomyelitis unlikely. A vascular malformation that was present since birth would
be unlikely to suddenly cause pain and swelling, and does not typically invade the bone. With
venous mal-formations, discoloration of the overlying skin may be seen.

American academy of pediatrics 503


American Academy of Pediatrics PREP 2015

Osteoid osteoma (OO) is the third most common benign bone tumor, after osteochondroma and
nonossifying fibroma, and usually occurs in boys between 7 and 25 years of age. It usually
presents with pain that is worse at night and relieved with nonsteroidal anti-inflammatory drugs
(NSAIDs). The lesion may be tender to palpation and cause swelling. Swelling is common in 00
and may be associated with high levels of prostaglandin production in the tumor, which is also
the rationale for pain relief with NSAIDs. The tumor rarely exceeds 1.5 cm, but it can be
associated with a significant amount of inflammation. The lesions are usually seen in the
diaphysis or metaphysis of long bones such as the femur or tibia. A large mixed Lytic and
sclerotic lesion with a Codman triangle would not be typical of this benign tumor.

PREP Pearls
• Osteosarcoma (OS) is the most common malignant bone tumor in children. The peak
incidence is in the second decade of life during the adolescent growth spurt.
• Osteosarcoma typically involves the long bones, most commonly around the knee joint,
with the distal femur and proximal tibia being the most frequent sites of involvement. In
decreasing frequency, the other common sites of OS are the distal humerus, middle and
proximal femur, pelvis, and jaw.

American academy of pediatrics 504


American Academy of Pediatrics PREP 2015

• Although all patients with OS are presumed to have micrometastatic disease (thus the
rationale for systemic chemotherapy in addition to wide surgical excision of the tumor),
approximately 15% to 20% of patients with OS present with macrometastatic disease on
initial staging evaluation, most frequently in the lungs and other bony sites.
• Osteoid osteoma is the third most common benign bone tumor, after osteochondroma and
nonossifying fibroma, and usually occurs in boys between 7 and 25 years of age. It
commonly presents with pain that is worse at night and relieved with non-steroidal anti-
inflammatory drugs.

American Board of Pediatrics Content Specification(s)


• Recognize the clinical findings associated with osteosarcoma
• Recognize the clinical and laboratory findings associated with osteoid osteoma
• Understand site(s) of metastasis of malignant bone tumors in children of various ages

Suggested Reading
• Arndt CA, Rose PS, Folpe AL, Laack NN. Common musculoskeletal tumors of
childhood and adolescence. Mayo Clin Proc. 2012;87(5):475-487.
doi:10.1016/j.mayocp.2012,01.015.
• Gereige R, Kumar M. Bone lesions: benign and malignant. Pediatr Rev. 2010;31(9):355-
363. doi:10.1542/pir.31-9-355.
• Laurence N, Epelman M, Markowitz RI, James C, Jaramillo D, Chauvin NA. Osteoid
osteomas: a pain in the night diagnosis. Pediatr RadioL.2012:42(12):1490-1501.
doi:10.1007/s00247-012-2495-y.

American academy of pediatrics 505


American Academy of Pediatrics PREP 2015

Item 163
You are seeing a 9-year-old girl in your office who was diagnosed with type 1 diabetes mellitus
1 month ago. She is currently doing well both physically and emotionally. At the end of the
appointment, her mother tells you that she herself is feeling overwhelmed with the stress of
managing this new medical problem. The parents are divorced, with the girl's father living across
the country. The mother has few others to help her with parenting. She is employed, but the
family is struggling financially. The mother is wondering how household and life stress will
affect her daughter's diabetes. You want to respond to the mother's concerns.

Of the following, the MOST accurate statement would be that


A. caregiver stress specific to the management of diabetes mellitus is associated with better
diabetes outcomes
B. general family stress and caregiver stress specific to diabetes mellitus are unlikely to
influence her child's clinical outcomes
C. nonspecific family stress is associated with improved diabetes outcomes
D. over time, parents of children who have diabetes mellitus report experiencing less stress
than do parents of children with asthma
E. parent stress from managing a child who has diabetes mellitus usually does not affect
parental mental health

American academy of pediatrics 506


American Academy of Pediatrics PREP 2015

Item 163 Preferred Response: A


There are different types of life stresses that vary by type and degree. Some kinds of stress, such
as multiple catastrophic events happening over a short period, can overwhelm a person's
emotional well-being and ability to function (at least over the short term). Other kinds of stress,
such as managing a single high-risk chronic medical illness, may actually generate increased
functioning and focus on solving or managing that particular problem. Research on chronic
medical conditions like type I diabetes, as seen in the child in the vignette, demonstrates that
parents who experience the management of the diabetes as an isolated life stressor are likely to
have children with better clinical outcomes. This is because they have greater involvement in the
management of the condition, as opposed to parents who are so stressed overall in their lives that
they can spare no additional focus and attention on their child's condition. Children whose
parents do not find their child's illness to be stressful may have poorer clinical outcomes,
presumably through less parental engagement in the day-to-day management that is often
required.

This phenomenon can also be understood in terms of what is well known about anxiety and
learning-that a little anxiety increases one's focus and ability to perform, but too much anxiety
becomes overwhelming and decreases functioning. Thus, a little daily stress about a child's
illness can improve the child's outcomes, but too much stress risks a breakdown in parental
functioning.

A parent experiencing stress about a child's type I diabetes commonly has poorer mental health,
including depression and decreased life satisfaction. All of that daily focus on keeping a child
healthy does in fact take a toll.
Parents of children with chronic illnesses like cancer, asthma, cystic fibrosis, and diabetes report
having greater overall parenting stress than their peers. Parents of children with asthma and type
1 diabetes typically experience equal levels of generalized stress.

PREP Pearls
• Disease-specific parental stress about their child's chronic medical condition is likely to
improve management of the disorder, as long as that stress is not overwhelming.
• Excessive parental stress about their child's chronic medical condition can impair their
ability to manage the disorder.
• Chronic medical conditions of children often have an overall negative impact on parental
psychological functioning.

American Board of Pediatrics Content Specification(s)


• Understand the effect of socioeconomic stressors on family dynamics

Suggested Reading
• Helgeson VS, Becker D, Escobar 0, Siminerio L. Families with children with diabetes:
implications of parent stress for parent and child health. Pediatr Psycho!. 2012;37(4):467-
478. doi: 10.1093/jpepsy/jsr110.
• Hamann SE, Wolfe-Christensen C, Ryan JL, et al. Parental overprotection, perceived
child vulnerability, and parenting stress: a cross-illness comparison. I Clin Psycho' Med
Settings. 2010;17(4):357-365. doh 10.1007/510880-010-9213-4.
American academy of pediatrics 507
American Academy of Pediatrics PREP 2015

• Perrin 1M, Gnanasekaran S, Delahaye J. Psychological aspects of chronic health


conditions. Pediatr Rev. 2012;33:99-109. doi:10.1542/pir.33-3-99.

American academy of pediatrics 508


American Academy of Pediatrics PREP 2015

Item 164
You are called to see a 9-year-old boy brought in by ambulance from the community swimming
pool. He dove into the pool but did not come up; his 3 friends jumped in and tried to pull him
out. The lifeguards arrived and performed cardiopulmonary resuscitation with use of an
automated external defibrillator. When emergency medical services arrived 5 min later, he had
regained consciousness and had a heart rate of 120 beats/min. When you see him 30 min later in
the emergency department, the patient is awake and talking. His mother is with him.
The mother tells you that there are distant relatives with seizure disorder on her mother's side,
one of whom wore hearing aids and one great grandfather who died at age of 23 years during
basic training in the army. You admit the patient overnight for observation. While on telemetry,
he has rare premature ventricular contractions.

Of the following, the electrocardiogram pattern you would MOST likely see in this patient is
A. first-degree atrioventricular block
B. generalized low voltages
C. increased left ventricular voltages
D. prolonged QTc
E. Wolff-Parkinson-White syndrome

American academy of pediatrics 509


American Academy of Pediatrics PREP 2015

Item 164 Preferred Response: D


The most common type of tong QT syndrome (LQTS), type I, classically presents as drowning
or near-drowning in cold water. Any child being evaluated after a drowning or near-drowning
episode, especially if the event begins with diving into water, should be suspected of having
LQTS type I. The patient in the vignette is alert and conscious after the automated external
defibrillator (AED) is used on him, which suggests that he had a ventricular arrhythmia that
responded to defibrillation. He also has a family history that includes early death, seizures, and
hearing loss. Seizures can be secondary to poor brain perfusion in the context of hypotension.
Congenital hearing loss is seen with the more severe form of LQTS type I. Stress and exercise
can also be triggers for ventricular arrhythmias in LQTS; this would be consistent with the great
grandfather who died during basic training. Given this combination of factors, a prolonged QTc
would be the most likely finding to expect on the electrocardiogram (ECG).

First-degree atrioventricular block would not respond to defibrillation, and by itself, unassociated
with higher degree of atrioventricular block, would not be a cause of syncope. Generalized low
voltages would be seen if myocarditis was suspected, but there is no history to suggest this
diagnosis (viral illness or fever). Increased left ventricular voltages may occur as a normal
variant or may be seen in cases of hypertrophic cardiomyopathy. Hypertrophic cardiomyopathy
can cause sudden death or syncope with exertion, but it is not associated with hearing loss.
Wolff-Parkinson-White (WPW) syndrome is also a rare cause of sudden death, especially if the
patient has had syncope with atrial fibrillation and subsequent ventricular fibrillation. This
rhythm would also respond to defibrillation. Wolff-Parkinson-White syndrome is not associated
with hearing loss in the family, and the initiation of atrial fibrillation is not typically associated
with diving into cold water.

Further evaluation and treatment of a patient with suspected LQTS will depend on the
presentation. In the case of cardiac arrest requiring defibrillation, the patient will need to be
treated with β-blockers and an implantable cardiodefibrillator. In a less urgent situation, such as
when LQTS is discovered on an ECG performed for other reasons such as chest pain, a more
step-wise evaluation is possible. The ECG morphology can suggest the LQTS type, and although
genetic testing can often confirm this, it is still not 100% sensitive.

In patients with type I LQTS (without a history of syncope or cardiac arrest), treatment with a
nonselective beta-blocker such as nadolol or propranolol is very effective. In patients with type II
LQTS, the classic trigger is being awakened from sleep by a loud noise. (β-blockers are used, but
may not be as effective. In type III LQTS, the ventricular arrhythmias typically occur at night
when the patients have bradycardia, and are not provoked by a trigger.

Type I and II LQTS are defects in potassium channel activity with a decrease in function or too
little potassium current. Type III LQTS is a defect in sodium channel activity with an increase in
function or too much current; patients with these conditions are also often treated with a second
medication, such as mexiletine, to block the sodium current.

All patients with LQTS are cautioned against diving into cold water, and taking any medication
that may prolong the QT interval. The current Bethesda guidelines for exercise restrictions in
patients with LQTS limit older patients, especially high school children, fairly severely (please
American academy of pediatrics 510
American Academy of Pediatrics PREP 2015

see Maron BJ, Zipes DP. Eligibility recommendations for competitive athletes with
cardiovascular abnormalities. J Am Coll Cardiol. 2005;45(suppl). doi: 10.1016 /j.
jacc.2005.02.002.).

PREP Pearls
• Long QT type I, the most common type, has a classic trigger of diving into cold water as
a cause of ventricular arrhythmia such as Torsade de Pointe.
• Triggers for events vary for the different types of long QT syndrome.
• Family history is key in making the diagnosis. Children of individuals with long QT
syndrome have a 50% chance of inheriting the disease.

American Board of Pediatrics Content Specification(s)


• Understand the clinical significance of a prolonged corrected QT interval

Suggested Reading
• Basheshirt A, Brerryo, Moss AJ, et al. Genetics of SCD. Curr Cardiol Rev.
2011;13(5):364-376.
• Levine E, Roserio SZ, Budzikowski AS, et al. Congenital long QT syndrome:
considerations for primary care physicians. Cleve Clin J Med. 2008;75(8) 591-600.
doi:10.3949/ccjm.75.8.
• Maron BI, Zipes DP. Eligibility recommendations for competitive athletes with
cardiovascular abnormalities. I Arn Call Cordial. 2005;45(suppl).
doi:10.1016/j.jacc.2005.02.002.

American academy of pediatrics 511


American Academy of Pediatrics PREP 2015

Item 165
A 2-year-old girl is brought to your office by her parents. The mother states that her daughter
trips much more often than her other children, and she has noticed that the girl's feet are both
turned inward—her left foot more than her right. Her daughter often sits on her feet. On physical
examination, the child allows you to move all extremities and no pain or swelling are noted. In
the prone position, you note that her feet are rotated toward the midline. You observe the child's
gait and note the patella faces forward while the toes turn inward.

Of the following, the BEST next step for this patient is


A. measurement of the thigh-foot angle every 6 to 12 months
B. observation and reassurance
C. plain radiographs of the lower extremities
D. referral to an orthopedic surgeon for serial casting
E. referral to an orthotist for nighttime splints

American academy of pediatrics 512


American Academy of Pediatrics PREP 2015

Item 165 Preferred Response: B


The 2-year-old girl in the vignette has tibial torsion. The current treatment recommendation for
tibial torsion is observation and reassurance.
In-toeing in children is usually caused by benign conditions. The most common cause in children
aged 1 to 3 years is tibial torsion. In cases of tibial torsion, there is rotation of the tibia so that the
foot and knee are misaligned. Internal rotation is more common than external rotation. Tibial
torsion can be unilateral or bilateral; when unilateral, the left side is more likely to be affected.
Children are commonly diagnosed after parents report clumsiness and frequent tripping.
Measurement of the thigh-foot angle can confirm the presence of tibial torsion; serial
measurements are not indicated for this condition. Previously used treatments such as splinting,
shoe modifications, braces, and exercises have been shown to be ineffective and are no longer
recommended. The natural history of tibial torsion is resolution by 4 years of age, so observation
and reassurance is recommended. Long-term consequences such as disability are rare, and tibial
torsion is not a risk factor for degenerative joint disease. Surgical treatment is rarely
recommended. External tibial torsion, seen in out-toeing, can be associated with neuromuscular
conditions. Children with gait disturbance or pain associated with external tibial torsion should
be referred for orthopedic evaluation.

Femoral anteversion is the most common cause of in-toeing after 3 years of age. Femoral
anteversion is an increase in the angle of the femoral neck greater than 20 degrees relative to the
shaft of the femur. It is bilateral, more common in girls, and is associated with the W sitting
position. The in-toeing usually increases in severity until 5 years of age, then spontaneously
resolves by 10 to 12 years of age. Usually no treatment is required for femoral anteversion. Plain
radiography would not be recommended for the routine evaluation of in-toeing.
Metatarsus adductus is the medial deviation of the forefoot relative to the hindfoot and is the
most common cause of in-toeing between birth and 1 year of age. Most cases of flexible
metatarsus adductus resolve spontaneously and do not require braces, shoes, or splinting. In
patients with inflexible metatarsus adductus (metatarsus varus), serial casting has been shown to
be effective especially if performed early.

Referral to an orthopedic surgeon for serial casting would be appropriate for fixed metatarsus
adductus. Referral to an orthotist for nighttime splints is not appropriate because this has been
shown not to be effective for tibial torsion. Rarely, in-toeing is associated with pathologic
conditions such as neuromuscular disorders, clubfoot, skewfoot, and hip disorders.

PREP Pearls
• In infants, the most common cause of in-toeing is metatarsus adductus.
• In toddlers, the most common cause of in-toeing is tibial torsion, which usually resolves
spontaneously by 4 years of age without treatment.
• In children older than 3 years, femoral anteversion is the most common cause of in-toeing
and usually resolves spontaneously by 12 years of age.

American Board of Pediatrics Content Specification(s)


• Recognize the clinical findings associated with tibial torsion

Suggested Reading
American academy of pediatrics 513
American Academy of Pediatrics PREP 2015

• Scherl SA. Common lower extremity problems in children. Pediatr Rev. 2004;25(2):52-
62. doi:10.1542/pir.25-2-52.
• Smith BC. Lower extremity disorders in children and adolescence. Pediatr Rev.
2009;30(8):287-293. doi:10.1542/pir.30-8-287.
• Suen PW, Lincoln TL. Common rotational variations in children. J Ant Acad Orthop
Surg. 2003;11(5):312-320.

American academy of pediatrics 514


American Academy of Pediatrics PREP 2015

Item 166
The director of a local community organization is planning a 5-km (3.1-mile) race that is the
culmination of a summer running program for children aged 9 years to 12 years. She asks for
your advice about the best way to plan for adequate hydration of race participants. She estimates
that most runners will finish the race in 40 minutes or less.

Of the following, the MOST appropriate recommendation for hydration of runners in this race is
that participants should be provided with
A. drinks supplemented with electrolytes at every mile on the course
B. drinks supplemented with electrolytes at the finish of the race
C. water at the finish of the race
D. water at the start, midway, and finish of the race
E. water at the start of the race

American academy of pediatrics 515


American Academy of Pediatrics PREP 2015

Item 166 S Preferred Response: D


In the policy statement Climatic Heat Stress and The Exercising Child and Adolescent, the
American Academy of Pediatrics recommends that children 9 to 12 years of age drink 3 to 8 oz
of fluids every 20 minutes during sports participation. If most participants will finish in 40
minutes, race organizers should provide water at the start, halfway point, and end of the race.
Sports drinks generally contain sugar and electrolytes. These sport drinks have a sweet taste,
therefore children may hydrate more when given access to them. However, for exercise lasting 1
hour or less, water is sufficient for hydration.

The 5k race described in the vignette will take place during the summer. Exercise during hot
weather can put athletes at risk for heat illness. Children and teenagers can regulate body
temperature as effectively as adults during exercise as long they hydrate appropriately. To reduce
the risk of heat illness, children should be encouraged to incrementally increase activity when
starting a new exercise program. Parents and coaches should make sure that training schedules
for activities during hot weather months take into account the need for gradual heat
acclimatization. Community physicians can help provide education to organizers of youth
athletic events regarding the need to modify or cancel activities with extreme heat and humidity.
Mass participation event organizers should understand the importance of staffing the event with
personnel familiar with the signs, symptoms, and treatment of heat illness.

If only 1 station with water or sports drinks were provided at the start or finish of the race,
athletes would not have the opportunity to hydrate every 20 minutes as recommended for
children and adolescents. While electrolyte-containing sports drinks may encourage hydration,
hydration stations with water at the beginning, middle, and end of the race is sufficient for this 5
km race.

PREP Pearls
• Children and teenagers can regulate body temperature as effectively as adults during
exercise, as long they hydrate appropriately.
• Children should be provided with opportunities for hydration every 20 minutes during
physical activity.
• For exercise lasting 1 hour or less. water is sufficient for hydration.

American Board of Pediatrics Content Specification(s)


• Plan optimal age-appropriate replacement for fluid losses associated with athletic activity

Suggested Reading
• Council on Sports Medicine and Fitness and Council on School Health, Bergeron MF,
Devore C, Rice SG. Policy statement-Climatic heat stress and the exercising child and
adolescent. Pediatrics. 2011:128(3):e741-e747. doi:10.15421peds.2011-1664.
• Rivera-Brown AM, Rowland TW, Ramirez-Marrero FA, Santacana G, Vann A. Exercise
tolerance in a hot and humid climate in heat-acclimatized girls and women. bit J Sports
Med. 2006;27(12):943-950. doi:10.1055/s-2006-923863.

American academy of pediatrics 516


American Academy of Pediatrics PREP 2015

Item 167
You are seeing the last patient in your clinic and realize that you are 15 minutes late for an
important meeting. You need to give the patient 2 prescriptions and referral information for a
subspecialty clinic visit.

Of the following, the intervention MOST likely to prevent a medical error in the scenario
described in the vignette is
A. increasing the nurse-to-patient ratio
B. increasing the time allotted for each patient encounter
C. providing a patient handout about subspecialty referrals
D. reducing physician fatigue
E. using computerized physician order entry

American academy of pediatrics 517


American Academy of Pediatrics PREP 2015

Item 167 S SBP Preferred Response: E


The physician described in the vignette is behind schedule and late for an appointment, while
trying to provide care to the last clinic patient of the day who needs prescriptions and a
subspecialty referral. In this rushed situation, a medication prescribing error is a significant risk.
Among the response choices, computerized physician order entry (CPOE) would be most likely
to prevent an error. CPOE reduces error by:
• the elimination of handwriting errors
• avoidance of errors caused by sound-alike drugs
• avoidance of errors caused by trailing or leading zeros
• decision support regarding drug formulation and dosing
• links to clinical pathways for correct drug prescribing decision support
• links to drug-drug interaction warnings

Computerized physician order entry is one strategy to prevent medical errors in the setting of
stress and distraction. Ensuring appropriate staffing (nurse to patient ratio), standardized
operating systems, reasonable workload (ie, time to see patients), family-centered care with
education (ie, patient handouts), and reducing physician fatigue have all been proven to reduce
medical errors. However, for the situation described in the vignette, we cannot know if staffing is
inadequate, the workload is imbalanced, patient education is lacking, or the physician is fatigued.
We can determine that the physician is rushed and that CPOE may reduce medication
prescription errors.

PREP Pearls
• Stress, distraction, and fatigue increase the risk of medical errors.
• Computerized physician order entry may reduce medical errors.

American Board of Pediatrics Content Specification(s)


• Recognize what interventions can reduce error in situations (eg, stress, fatigue,
distraction) at high risk for medical error

Suggested Reading
• Abramson EL, Kaushal R. Computerized provider order entry and patient safety. Pediatr
Clin North Am. 2012;59(6):1247-1255. doi:10.1016/j. pc1.2012.08.001.
• van Rosse F, Murat B, P.,.demaker CM, ran Vught AJ, Egberts AC, Bollen CleV. The
effect of computerized physician order entry on medication prescription errors and
clinical outcome in pediatric and intensive care: a systematic review. Pediatrics.
2009;123(4):1184-1190. doi:10,1542/ peds.2008-1494
• Varkey P, Aponte P. Swanton C, Fischer D, Johnson SF, Brennan MD. The effect of
computerized physician-order entry on outpatient prescription errors. Mating Care
interface. 2007;20(3):53-57.
• Woodward HI, Mytton OT, Lemer C, et al. What have we learned about interventions to
reduce medical errors? Arm Rev Public Health. 2010;31:479-497.
doi:10.1146/annurev.publhealth.012809.103544.

American academy of pediatrics 518


American Academy of Pediatrics PREP 2015

Item 168
A 14-year-old adolescent girl presents to an Arizona urgent care, where you practice, after being
stung by a scorpion. Approximately 3 hours before arrival, the teenager-who is on a camping trip
with her Girl Scout troop-felt a sharp pain in her left calf just after she crawled into her sleeping
bag to rest for the night. Upon feeling the pain, she got out of the sleeping bag, shook the bag,
and saw a scorpion (Centruroides exilicauda) crawl out.

Her temperature is 36.8°C, heart rate is 100 beats/min, blood pressure is 90/60 mm Hg,
respiratory rate is 18 breaths/ min, and pulse oximetry is 99% on room air. A medical history
form provided to you by one of the trip chaperones indicates that the adolescent has up-to-date
immunizations and no significant past medical history or allergies.

The patient appears anxious, but is in no distress. She complains of a sharp pain and occasional
tingling in her left calf, but she has no other symptoms. A complete physical examination is
significant only for a 2-mm puncture wound on the girl's left calf with no surrounding swelling
or erythema. A full neurologic examination, including close assessment of the cranial nerves,
reveals no abnormalities. You tap gently on the puncture wound on her left calf, which causes
her to cry out sharply with pain. You apply a cool compress to the site of the wound.

Of the following, the BEST next step in management for this patient's symptoms is
administration of
A. Centruroides exilicauda antivenin therapy
B. intramuscular midazolam
C. intravenous calcium gluconate
D. oral acetaminophen
E. oral diphenhydramine

American academy of pediatrics 519


American Academy of Pediatrics PREP 2015

Item 168 Preferred Response: D


The adolescent girl in the vignette presents with pain and tingling in her left calf, a few hours
after being stung by a bark scorpion (Centruroides sculpturatus). Administration of oral
acetaminophen is the next best step in her care at this time.

Scorpion envenomation is a significant clinical problem, with approximately 5,000 deaths


reported worldwide each year from scorpion stings. The American Association of Poison Control
Centers received more than 17,000 calls related to scorpion stings in 2009. In the United States,
most significant systemic reactions arise from stings of C scuipturatus, also known as
Centruroides exilicauda, or the bark scorpion. Bark scorpion envenomations are potentially fatal
to young children; thus, pediatric providers must be familiar with their clinical manifestations
and management.

Scorpions are arthropods that envenomate prey by stinging. They generally avoid humans, and
sting only when threatened, as a means of self-defense. Scorpions live in dry, warm, desert
regions as well as in tropical regions. In the United States, C sculpturatus is responsible for the
most clinically significant envenomations. This species is found in the Southwestern states
including Arizona, New Mexico, Texas, southern California, and Nevada, as well as in northern
Mexico.

C sculpturatus is a yellow-brown scorpion that may range from just over 1 cm to more than 7 cm
in length. Members of this species can be differentiated from other scorpion species by several
characteristics, including the presence of a small protruding tubercle at the base of their stingers,
known as subaculear tooths.

Envenomation by C sculpturatus can cause severe toxicity, especially in small children, and has
resulted in fatalities. In the 1930s, C sculpturatus stings caused 40 reported deaths, which
primarily involved infants and children. Advances in supportive care have dramatically reduced
deaths related to scorpion stings in recent decades, with only 1 reported death in the United
States since 1970.

Scorpion venoms are complex mixtures of bioactive substances. The primary toxic substances in
C sculpturatus venom are neurotoxins, which act to cause increased and repetitive firing of
neuronal axons.

Clinical symptoms of scorpion envenomation can vary widely, from localized pain, to skeletal
neuromuscular hyperactivity with or without cranial nerve dysfunction, to multisystem organ
failure and even death. Severity of clinical symptoms tends to correlate inversely with age, with
infants and young children at the highest risk for severe toxicity. Localized pain and paresthesias
at the site of the sting are typically the first clinical symptoms, manifesting almost immediately
after envenomation. Affected patients may have a positive "tap test:' in which tapping over the
site of the sting results in severe pain. Lack of inflammatory changes at the sting site is another
characteristic of C sculpturatus envenomation. Dysautonomias (most commonly tachycardia and
hypertension), hyperthermia, and rhabdomyolysis from muscular hyperactivity, gastrointestinal
disturbances, pancreatitis, and eventual multisystem organ failure may arise from C scuipturatus

American academy of pediatrics 520


American Academy of Pediatrics PREP 2015

envenomation. The most common presenting symptoms in younger patients include restlessness,
writhing, flailing, and roving eye movements.

A clinical grading system to describe the severity of scorpion envenomation was developed by
Curry and colleagues. Based on this system, children with grade I envenomation are those
experiencing pain or paresthesia at the envenomation site only; those with grade II envenomation
have pain or paresthesia both local to and remote from the envenomation site; those with grade
III envenomation have either cranial nerve or somatic skeletal neuromuscular dysfunction (but
not both); and those with the most severe envenomations (grade IV) display both cranial nerve
and somatic skeletal neuromuscular dysfunction.

Diagnosis of C sculpturatus envenomation is a clinical one in most cases. Children from endemic
areas presenting with typical neurotoxic symptoms should be assumed to have been stung. Older
children, such as the adolescent girl in the vignette, may be able to give a clear history of being
stung by a scorpion, but younger children (who typically experience more severe symptoms)
may not be able to do so. Often, a caregiver may note that a scorpion was seen on or near the
child around the time of symptom onset.

Differential diagnoses for scorpion envenomation with associated neurotoxicity include seizure,
black widow spider envenomation, organophosphate toxicity, sympathomimetic toxicity,
strychnine poisoning, or intoxication with phenothiazine. In infants presenting with unexplained
crying and restlessness, the differential diagnosis is even broader, but will not be discussed in
detail here.

The mainstay of therapy for scorpion envenomation is good supportive care, especially support
of the airway, breathing, and circulation. All patients presenting after scorpion stings should be
monitored for progression to higher-grade symptoms. Children with lower-grade (grades I and
II) symptoms can be treated conservatively with application of ice, analgesics, and tetanus
prophylaxis as indicated. Most of these children can be cared for at home following observation
for progression of symptoms for at least 4 to 5 hours from the time of envenomation.

For patients with higher-grade (grades III and IV) symptoms arising from scorpion
envenomation, opioid analgesics, as well as sedation with short-acting benzodiazepines, such as
midazolam, are recommended. The use of C sculpturatus antivenin is controversial but may have
benefit in decreasing the duration of severe symptoms. Centruroides antivenin was only recently
approved by the US Food and Drug Administration and is not available in all centers. A medical
toxicologist or other physicians with expertise in treating children with higher-grade scorpion
envenomation should be consulted when the administration of antivenin is being considered.
Centruroides antivenin and intramuscular midazolam would not be indicated for the adolescent
girl in the vignette given that she has only low-grade (grade I) symptoms. Intravenous calcium
gluconate does not have an established role in the treatment of patients with scorpion
envenomation, nor does oral diphenhydramine.

American academy of pediatrics 521


American Academy of Pediatrics PREP 2015

PREP Pearls
• Clinical signs and symptoms of scorpion envenomation may vary widely, from localized
pain and paresthesias near the site of the sting, to skeletal neuromuscular hyperactivity
with or without cranial nerve dysfunction, to multisystem organ failure.
• Infants and young children are at the highest risk for severe toxicity from scorpion
envenomation.
• The mainstay of therapy for scorpion envenomation is good supportive care. All children
presenting after scorpion stings should be closely monitored for at least 4 to 5 hours for
progression of symptoms.
• Children with lower-grade symptoms can be treated conservatively with application of
ice, analgesics, and tetanus prophylaxis as indicated.

American Board of Pediatrics Content Specification(s)


• Plan the appropriate management of a scorpion sting

Suggested Reading
• Bond GR. Snake, spider, and scorpion envenomation in North America. Pediatr Rev.
1999;20(5)147-150. doi:10.1542/pir.20-5-147.
• Holve S. Treatment of snake, insect, scorpion, and spider bites in the pediatric emergency
department. Czar Opin Pediatr. 1996;8(3):256-260.
• Skolnik AB, Ewald MB. Pediatric scorpion envenomation in the United States:
morbidity, mortality, and therapeutic innovations. Pediatr Emerg Care. 2013;29(1):98-
103. doi:10.1097/PEC.0b013e31827b5733.

American academy of pediatrics 522


American Academy of Pediatrics PREP 2015

Item 169
While in the normal nursery, the nurse asks you to assess a newborn who has nasal stuffiness.
The newborn was delivered 18 hours earlier by spontaneous vaginal delivery complicated by
meconium-stained amniotic fluid. The newborn emerged vigorous and required only bulb
suctioning in the delivery room. The newborn breastfed poorly overnight and had nasal stuffiness
that briefly improved with bulb suctioning. Physical examination reveals a pink, well-perfused
newborn with audible nasal congestion, mild supraclavicular retractions, and minimally
decreased aeration of the lungs bilaterally.

Of the following, the BEST next management step for this newborn is to
A. administer intranasal normal saline drops
B. continue to closely monitor
C. obtain a radiograph of the chest
D. order a computed tomography image of the neck
E. pass a nasal suction catheter bilaterally

American academy of pediatrics 523


American Academy of Pediatrics PREP 2015

Item 169 Preferred Response: E


Newborns who present with nasal stuffiness should have a suction catheter passed through both
nares to verify patency. In the newborn period, nasal obstruction presents with a spectrum of
clinical findings ranging from nasal stuffiness to respiratory failure. Severe nasal obstruction can
be life threatening because newborns are obligate nasal breathers. Rapid identification and
stabilization are essential.

Nasal obstruction is most commonly caused by mucosal inflammation. In the immediate


neonatal period, suctioning is often the underlying cause and the resultant swelling responds to
supportive measures including humidity and saline drops. The initial evaluation will often
demonstrate transient improvement with suctioning and passage of a 5-French catheter. If
evidence of nasal obstruction persists, further evaluation with a fiberoptic scope, computed
tomography (CT) scan, or magnetic resonance imaging (MRI) may be needed to rule out less
common causes such as encephalocele and glioma.

Choanal atresia or stenosis should be suspected if a 5-French catheter is unable to pass through
one or both nares. Bilateral choanal atresia often presents in the delivery room in a newborn who
is pink only with crying and turns blue when the crying stops. Attempts at ventilation using a bag
mask are often unsuccessful, with prompt placement of an endotracheal tube improving
oxygenation and ventilation. Neonates with unilateral choanal atresia or mixed choanal
atresia/stenosis may present in the nursery with mild to moderate respiratory distress including
nasal stuffiness, increased work of breathing with retractions, decreased aeration, and cyanosis.
Affected neonates may worsen over time, as continued attempts to decrease their congestion by
bulb suctioning only cause further mucosal swelling and increased obstruction. Newborns
suspected to have choanal atresia or stenosis should be immediately transferred to a tertiary
center for further evaluation, including flexible nasendoscopy and CT imaging of the paranasal
sinuses and skull base. Several surgical techniques including transpalatal and endoscopic
resection have been developed to establish patency of the nares. Restenosis may occur in up to
20% of cases.

Newborns with choanal atresia should undergo a complete multisystem evaluation because up to
75% of cases have other anomalies. CHARGE (coloboma, heart defects, choanal atresia,
retardation of growth and development, genitourinary anomalies, ear anomalies and hearing loss)
association is the most common condition seen with choanal atresia.

The newborn in the vignette has findings suggestive of unilateral choanal atresia or stenosis and
should have a nasal suction catheter passed bilaterally. If the catheter passes through both nares,
intranasal normal saline drops may be considered to decrease potential underlying mucosal
swelling related to bulb suctioning while the newborn is closely monitored. Continued increased
work of breathing, in spite of the demonstration of bilateral patent nares, may prompt the
clinician to obtain a radiograph of the chest. Further imaging with CT or MRI may be done in
consultation with a subspecialist.

American academy of pediatrics 524


American Academy of Pediatrics PREP 2015

PREP Pearls
• Newborns who present with nasal stuffiness should have a suction catheter passed
through both nares to verify patency.
• Choanal atresia or stenosis should be suspected if a 5-French catheter is unable to pass
through one or both nares.
• CHARGE (coloboma, heart defects, choanal atresia, retardation of growth and
development, genitourinary anomalies, ear anomalies and hearing loss) association is the
most common condition seen with choanal atresia.

American Board of Pediatrics Content Specification(s)


• Recognize the clinical findings associated with both unilateral and bilateral choanal
atresia
• Plan the appropriate evaluation of choanal atresia
• Understand the association of choanal atresia with other congenital anomalies

Suggested Reading
• Hengerer AS, Brickman TM, Jeyakumar A. Choanal atresia: embryologic analysis and
evolution of treatment, a 30-year experience. Laryngoscope. 2008;118(51,862-866.
doi:10.1097/MLG.0b013e3181639b91.
• Krakovitz PR, Koltai PJ. Neonatal nasal obstruction. NeoReyiews. 2007:8(5):e199-e205.
doi:10.1542/neo.8-5-e199.
• Ramsden JD, Campisi P, Forte V. Choanal atresia and choanal
• stenosis. Otolaryngol Clin North Am. 2009;42(2):339-352. doi:10.1016/j.
otc.2009.01.001.

American academy of pediatrics 525


American Academy of Pediatrics PREP 2015

Item 170
You are evaluating a 15-year-old girl in your clinic for new-onset right facial weakness. She first
noticed it this morning when she was putting on makeup and could not pucker her lips or close
her right eye all the way. She denies limb weakness, facial pain, sensory loss, and changes in
hearing or taste. Her only medication is oral contraceptives. She occasionally smokes cigarettes.
The girl has not had any recent immunizations. Her general physical examination is
unremarkable. Neurologic examination shows normal pupillary reflexes, eye movements, and
facial sensation. When raising her eyebrows, the right side of her forehead is weak and has fewer
wrinkles than the left side of her forehead. She has right ptosis and weak right eye closure, and
the right side of her mouth droops. Her tongue is midline and her extremities have normal
strength and tone. It is now afternoon and her symptoms have not improved.

Of the following, the MOST appropriate treatment for her condition is


A. acyclovir
B. aspirin
C. heparin
D. prednisone
E. tissue plasminogen activator

American academy of pediatrics 526


American Academy of Pediatrics PREP 2015

Item 170 Preferred Response: D


The girl in the vignette has Bell palsy, or idiopathic unilateral facial nerve palsy. If started within
3 days of symptom onset, a course of prednisone improves the chances of complete recovery.
Most cases of Bell palsy do not have an identifiable underlying cause, and are presumed to be a
postviral inflammation of cranial nerve VII, the facial nerve. History and physical examination
should assess for secondary causes of facial nerve palsy, such as viral reactivation of herpes
simplex or varicella zoster virus, acute otitis media, meningitis (including Lyme disease),
leukemia, or lymphoma. In the absence of signs and symptoms of secondary causes, the
diagnosis of Bell palsy can be made clinically; imaging is not needed. Most cases of Bell palsy
have a good recovery, especially if the weakness is mild or the symptoms improve rapidly.
In Bell palsy, oral steroids, typically prednisone, at a dosage of 2 mg/kg per day for 5 days,
followed by a 5-day taper, can increase the chance of complete recovery. Until eye closure
strength returns, the cornea needs to be protected, which can be done with artificial tears during
the day and topical lubricant at night or taping the eyelid shut. Ophthalmology referral to monitor
the cornea can be helpful. Studies have not shown additional benefit of antiviral agents in the
absence of clinical signs of viral infection such as vesicles, and even with clinical signs,
acyclovir is not recommended as the sole treatment for Bell palsy.

It is critical to distinguish Bell palsy from a stroke on clinical examination. The key finding in
Bell palsy is forehead weakness, which manifests as decreased forehead wrinkling on the
affected side, as is present in the girl in the vignette. A pediatric patient who presents with
unilateral facial weakness, with sparing of the forehead, should be referred to the nearest
emergency department immediately for evaluation for stroke. The optimal treatment for stroke in
pediatric patients is currently under investigation. Aspirin, intravenous tissue plasminogen
activator, and mechanical thrombolysis have all been used. Intravenous heparin is not used as an
initial treatment for stroke, unless a thrombus or critical arterial stenosis is identified.

PREP Pearls
• Unilateral facial weakness involving the forehead is suggestive of facial nerve palsy (not
of stroke).
• Oral prednisone, started within 3 days of onset of symptoms, can improve the chance of
complete recovery in Bell palsy.

American Board of Pediatrics Content Specification(s)


• Recognize the clinical findings associated with Bell palsy, and manage appropriately

Suggested Reading
• Clark GD. Facial nerve palsy in children. UptoDate. 2012. Available online only for
subscription.
• Sullivan FM, Swan IR, Donnan PT, et al. Early treatment with prednisolone or acyclovir
in Bell's palsy. N Engl I Med. 2007;357(16):15981607. doi:10.1056/NEJMoa072006.

American academy of pediatrics 527


American Academy of Pediatrics PREP 2015

Item 171
An 11-year-old girl presents for concerns about early puberty and irregular menses. She had
breast development beginning 6 months ago and then had her first menstrual period 1 month ago.
Family history reveals that her mother had menarche at 12 years of age, and her midparental
height is 160 cm (5 ft, 3 in). On physical examination, the girl is 155 cm (5ft, 1 in) tall, has
sexual maturity rating 4 breasts with a secondary mound, and sexual maturity rating 3 pubic hair.
Her family is concerned because her menstrual periods are irregular and heavy.

Laboratory value results include the following:


 Luteinizing hormone, less than 0.2 mIU/mL (0.2 IU/L)
 Follicle-stimulating hormone, less than 0.7 mIU/mL (0.7 IU/L)
 Estradiol, 144 pg/mL (529 pmol/L); reference range for a sexual maturity rating 4 female,
21 pg/mL to 85 pg/mL (77 pmol/L-312 pmol/L)

Of the following, you are MOST likely to tell the family that
A. her height of 155 cm (5 ft, 1 in) is within 5 cm (2 in) of her midparental height and
consistent with her genetic potential
B. her laboratory results show a very low luteinizing hormone and follicle-stimulating
hormone, suggesting that she is very early in pubertal development
C. her progression through puberty is normal because the average age of menarche in North
America is 12 ± 1 years of age
D. irregular menses are common for the first 2 years after menarche, and follow-up should
be scheduled in 6 months
E. the rapid pace of pubertal development and abnormal laboratory values suggest that she
needs further workup for her symptoms

American academy of pediatrics 528


American Academy of Pediatrics PREP 2015

Item 171 Preferred Response: E


The child in this vignette rapidly advanced through puberty, progressing from thelarche (breast
development) to menarche (onset of menses) in only 5 months. In the United States, thelarche
occurs at an average of 10.7 years of age ± 1 year, with menarche occurring at 12.7 years of age
± 1.0 year. More recently, it has been found that thelarche is occurring earlier, with 5% of girls
showing breast development by 7 years of age and 9% by 8 years of age. This earlier
development has been associated with obesity. Even so, the span of time between the onset of
breast development and the age of menarche is still 2.3 ± 1.0 year.

This child has very low (suppressed) luteinizing hormone (LH) and follicle-stimulating hormone
(FSH) despite having rapidly progressed through puberty, and very high estradiol levels,
suggesting an exogenous source or autonomous ovarian production of estrogen is suppressing
LH and FSH levels. These laboratory findings, along with her rapid pace of pubertal
development, suggest additional workup is warranted to find the source of estrogen secretion.

This child was ultimately found to have an ovarian granulosa cell tumor. Although her height is
within 2 inches of her midparental height, the concerning laboratory findings and rapid pace of
pubertal development still warrant evaluation. Because of her physical examination findings,
laboratory results do not suggest she is early in pubertal development, but rather that the LH and
FSH are suppressed from very high estradiol levels. In addition, although it is true that irregular
menses are common for the first 2 years after menarche, this child may be experiencing irregular
menses from changes in estrogen levels secreted by an exogenous source or from autonomous
ovarian production, and thus further evaluation is warranted.

PREP Pearls
• The span of time between the onset of breast development and the age of menarche
averages 2.3 ± 1.0 year.
• Early puberty or rapid pubertal progression warrant consideration for an underlying
pathological diagnosis.

American Board of Pediatrics Content Specification(s)


• Recognize the stages of sexual development and the range of age of onset of each
• Understand factors that influence the timing of puberty

Suggested Reading
• Pescovitz OH, Eugster EA. Precocious puberty. In: Pescovitz OH, Eugster EA. Pediatric
Endocrinology: Mechanisms, Manifestations, and Management. Hagerstown, MD:
Lippincott, Williams & Wilkins; 2004:316-333.
• Rosenfeld RL. Puberty in the female and its disorders. In: Sperling MA, ed, Pediatric
Endocrinology. 2nd ed. Philadelphia, PA: Saunders Elsevier; 2002:455-518.
• (-146 Critique(s) 171-172

American academy of pediatrics 529


American Academy of Pediatrics PREP 2015

Item 172
An otherwise healthy child visits your office for evaluation of hypopigmented areas on both
knees that are nonpruritic, sharply demarcated, and nonscaly (Item Q172). The mother reports
that his father has similar lesions. The family requests a skin biopsy to confirm the diagnosis.

Of the following, a skin biopsy on this child would MOST likely demonstrate
A. absence of melanocytes
B. follicular disruption
C. fungal hyphae
D. immunoglobulin E deposition
E. neutrophilic infiltration

American academy of pediatrics 530


American Academy of Pediatrics PREP 2015

Item 172 Preferred Response: A


The child in the vignette demonstrates the typical appearance of vitiligo: lesions with chalk white
skin color and sharply demarcated borders. In this condition, melanocytes undergo destruction,
seen microscopically as loss or absence of melanocytes, and decreased melanin in the few
remaining melanocytes. Vitiligo is a fairly common cause of depigmentation with peak onset
between 10 and 30 years of age; more than 50% of patients have onset before 20 years of age.
Initial lesions commonly involve the hands, forearms, feet, and face, although eventually the
condition can affect any body part, including mucous membranes. Lesions are usually
asymptomatic. Affected areas are more easily seen in people of darker complexion, although
detection of subtle lesions may be enhanced by using an ultraviolet light at 365 nm wavelength.
In nearly half of patients, hair within the lesion is affected as well-producing leukotrichia
(poliosis, loss of pigment); however, follicular disruption is not noted. Lesions occurring in a
dermatomal distribution are called segmental vitiligo and may have a worse prognosis for
repigmentation. Lesions are extremely sensitive to sun injury.

The cause of vitiligo is unclear, although it is likely multifactorial and many cases involve a
genetic predisposition. Twelve percent to 35% of affected pediatric patients have a family history
of vitiligo. There is an observed association between the condition and autoimmune disorders,
most notably autoimmune thyroiditis, though the association is better defined for adult patients
than for children. Because vitiligo may precede the onset of autoimmune thyroiditis, some
experts recommend thyroid antibody screening as part of the initial evaluation for vitiligo,
though treatment of thyroid disease will not affect the vitiligo.

Lesions often develop at sites of previous minor trauma, and new injury can lead to extension or
development of additional disease areas in affected patients (Koebner phenomenon). Patients
with vitiligo are observed to have greater rates of anxiety and depression than the general
population, and stress (physical, psychological) has been proposed as an inciting event for the
condition. Vitiligo is not associated with fungal infection, atopic disease, or infectious etiologies;
therefore, skin biopsy is unlikely to show fungal hyphae, immunoglobulin E deposition, or
neutrophilic infiltration.

Treatment of vitiligo is usually observation because up to 25% of patients undergo spontaneous


repigmentation. However, the psychological stress of these very noticeable lesions should be
considered, and various treatments have been tried to ameliorate them. Camouflage cosmetic
products can be helpful for many patients. High-potency topical steroids and calcineurin
antagonists have shown some benefit, particularly for children. Phototherapy, sometimes
combined with topical steroids, is commonly used for patients with widespread disease. Vitamin-
D derivatives such as calcipotriol have been effective in some cases. Treatment with a 308-nm
excimer laser has recently shown promise for localized lesions.

There are many other causes of depigmentation in children including postinflammatory


hypopigmentation, tinea versicolor, hypopigmented nevi, lichen sclerosus, and morphea. Among
the most common is pityriasis alba, an asymptomatic condition primarily affecting children
between 3 and 16 years of age. Pityriasis alba occurs most commonly on the face and appears as
indistinct, irregular, slightly scaly, faintly hypopigmented areas. Lesions are more easily seen in
children with darker skin tone and after sun exposure. The cause of the condition is unknown,
American academy of pediatrics 531
American Academy of Pediatrics PREP 2015

but many experts feel it is a low-grade eczematous dermatitis. An infectious cause has been
proposed, but there is no confirmed relationship between pityriasis alba and a specific organism.
Skin biopsy is rarely indicated, but if done, it may show a variety of nonspecific histopathologic
findings. However, there is usually a decreased amount of melanin in a normal or increased
number of epidermal melanocytes. Treatment is primarily aimed at careful sun protection and
adequate lubrication; low potency topical corticosteroids or calcineurin inhibitors may be used
when there is concern about the cosmetic appearance. The condition usually resolves by late
adolescence.

PREP Pearls
• Vitiligo, characterized by the absence or destruction of melanocytes, is a common cause
of skin hypopigmentation in children. characterized by absence or destruction of
melanocytes
• Vitiligo presents as sharply demarcated, markedly hypopigmented areas, often involving
hands, forearms, feet, and face; eventually anybody area may be involved, including hair
and mucous membranes.
• Vitiligo is sometimes associated with autoimmune diseases, particularly autoimmune
thyroiditis.
• Pityriasis alba is a form of low-grade eczema leading to indistinct areas of
hypopigmentation, particularly involving the face.
• Pityriasis alba, is treated with careful sun protection and lubrication, typically resolves by
late adolescence.

American Board of Pediatrics Content Specification(s)


• Recognize the clinical manifestations of vitiligo
• Recognize the clinical features of pityriasis alba

Suggested Reading
• In SI, Yi SW, Kang HY, Lee ES, Sohn S, Kim YC. Clinical and histopathological
characteristics of pityriasis alba. Clin Exp Dermatol.2009;34(5):591-597.
doi:10.1111/j.1365-2230.2008.03038.x
• Lio PA. Little white spots: an approach to hypopigmented macules. Arch
• Dis Child Educ Pract Ed, 2008;93(3):98-103, doi:10.1136/adc.2007.135194.
• Yaghoobi R, Omidian M, Bagherani N. Vitiligo: a review of the published work. J
Dermatol. 2011;38(5):419-431. doi:10.11110346-8138.2010.01139.x.

American academy of pediatrics 532


American Academy of Pediatrics PREP 2015

Item 173
An 18-year-old young woman is brought to your office by her mother for follow-up after an
emergency department visit for loss of consciousness while partying with her friends. She admits
to binge drinking alcohol on weekends but denies the use of drugs. She states that no violence or
trauma occurred. There is no known family history of epilepsy, syncope, or cardiac problems.
She currently is well with an unremarkable physical examination. You review with the girl the
potential consequences of binge drinking alcohol on her body.

Of the following, the MOST likely acute medical complication of her behavior is
A. atrial fibrillation
B. hepatic encephalopathy
C. hepatic steatosis
D. pancreatitis
E. seizure

American academy of pediatrics 533


American Academy of Pediatrics PREP 2015

Item 173 S Preferred Response: E


Adolescence is a time of exploration and most teenagers will try alcohol and put themselves at
risk for the consequences of acute use. The young woman in the vignette probably had a seizure
as a consequence of binge drinking. Although not common, acute alcohol ingestion by
adolescents and young adults can result in a seizure that is usually generalized and tonic-clonic
in nature, but occasionally can be a partial seizure. This may occur in someone with no history of
epilepsy. Seizures are more likely to occur after acute withdrawal in chronic alcohol users.

More common, and of greater concern, are the behavioral issues that occur after heavy alcohol
use. In addition to central nervous system depression, adolescents may experience sleep
disturbances, mood swings, agitation, anxiety, depression, memory loss, and academic failure. In
addition, there may be aggressive behavior and violence. Accidents are the leading cause of
death in adolescents and approximately half of the adolescents involved in an automobile crash
are intoxicated; burns, falls, and drowning are also more likely. Dating violence is another major
concern. Often, the victim is too intoxicated to resist and may suffer memory loss for the event.
In addition to physical harm, other potential consequences include unwanted pregnancies and
sexually transmitted infections.

Chronic use of alcohol, which is less likely to be seen in adolescents, can affect every organ in
the body. Liver effects start with steatosis, proceed to fibrosis, cirrhosis, and eventually liver
failure with hepatic encephalopathy. Arrhythmias, hypertension, cardiac myopathy, and strokes
may result from the effects of alcohol on the cardiovascular system. Other complications include
pancreatitis, an increased incidence of a number of cancers, and suppression of the immune
system.

PREP Pearls
• Alcohol ingestion can affect every organ system in the body.
• In the pediatric population, the effects of acute alcohol ingestion are more commonly
encountered than the effects of chronic alcohol ingestion.
• Seizures may occur after acute alcohol ingestion even without a history of epilepsy.

American Board of Pediatrics Content Specification(s)


• Identify the major physiologic consequences associated with alcohol use/abuse
• Recognize the major behavioral consequences of alcohol use/abuse

Suggested Reading
• Adger H Jr, Saha S. Alcohol use disorders in adolescents. Pediatr Rev. 2013;34(3):103-
114. doi:10.1542/pir.34-3-103.
• Karceski S. How do alcohol and drugs influence seizure activity? Practical Neurology.
2006;Aug:44-46. Accessed February 21, 2014.
• National Institute on Alcohol Abuse and Alcoholism. Beyond hangovers: understanding
alcohol's impact on your health. National Institutes of Health website.

American academy of pediatrics 534


American Academy of Pediatrics PREP 2015

Item 174
The mother of a 2-year-old child with severe atopic dermatitis emails you a photograph of the
child's left hand. The mother reports that the child has developed new bumps that look different
from the typical red, dry, itchy eczematous rash. She describes the rash as a mixture of little
blisters and sores. The child has been running a fever of 38.6°C for the past 2 days and has been
tired and irritable. You view the photograph (Item Q174) and request the mother to bring the
child in to see you that day for an urgent visit.

Of the following, your MOST likely diagnosis is


A. eczema herpeticum
B. hand-foot-mouth disease
C. impetigo
D. molluscum contagiosum
E. verruca vulgaris

American academy of pediatrics 535


American Academy of Pediatrics PREP 2015

Item 174 Preferred Response: A


The child in the vignette most likely has eczema herpeticum as a complication of atopic
dermatitis (AD).

Atopic dermatitis can be complicated by viral skin infections, such as herpes simplex, warts, and
Molluscum contagiosum, implicating local defects in T-cell function. The presence of punched-
out erosions, vesicles, or infected skin lesions, as seen in this child, that do not respond to oral
antibiotics should initiate a search for a potentially serious form of herpes simplex infection
termed Kaposi varicelliform eruption or eczema herpeticum. Herpes simplex infection can be
diagnosed by a Giemsa-stained Tzanck smear of cells scraped from the vesicle base, commercial
immunofluorescence assays, or viral culture. Prompt institution of antiviral treatment,
administered intravenously, is of critical importance in the patient with widespread AD because
life-threatening dissemination has been reported. Notably, in patients with AD, smallpox
vaccination or even exposure to recently vaccinated subjects can cause a severe, widespread,
life-threatening dermatitis called eczema vaccinatum, which is similar in appearance to eczema
herpeticum.

Molluscum contagiosum commonly presents as single or multiple small, flesh-colored papules


with central umbilication that typically resolves spontaneously in most immunocompetent
individuals. Verrucae, or warts, can occur singly or in coalescing lesions, and can be flat, mosaic,
or filiform in appearance. The diagnosis is usually made based on clinical observation.
Spontaneous regression occurs in as many as two-thirds of warts within 2 years. The absence of
fever, preceding illness, and oral vesicles on the buccal mucosa and tongue make hand, foot, and
mouth disease a less likely diagnosis in this child.

Skin infections, particularly with Staphylococcus aureus, can be a common and recurrent
problem in patients with AD. Impetigo is a superficial bacterial infection most commonly caused
by S aureus or β-hemolytic streptococci. It may be classified as primary impetigo (direct
bacterial invasion of previously normal skin) or secondary impetigo (infection at sites of minor
skin trauma, such as abrasions, or underlying conditions such as eczema). Nonbullous impetigo
is the most common clinical form of impetigo with papular lesions that progress to vesicles and
pustules that enlarge and break down to form thick, adherent crusts with a characteristic golden
appearance. Patients with moderate-to-severe AD have been found to make immunoglobulin E
antibodies against staphylococcal toxins present on their skin. Recent studies have demonstrated
that TH2 immune responses increase binding of S aureus to inflamed skin lesions, contributing
to the underlying inflammation present in patients with AD. This is supported by the observation
that treatment with topical glucocorticoids or tacrolimus reduces S aureus counts on atopic skin.
Hand washing to minimize the spread of staphylococci and preventive measures to decrease the
amount of skin colonization may be helpful. Topical therapy is recommended in the management
of impetigo with a small number of nonbullous lesions. Systemic antibiotics are indicated only in
patients with extensive infection.

American academy of pediatrics 536


American Academy of Pediatrics PREP 2015

PREP Pearls
• Children with atopic dermatitis (AD) can have skin infections, such as herpes simplex,
warts, impetigo, and Molluscum contagiosum, complicating their course and
management.
• Herpes simplex infection of the skin manifesting as eczema herpeticum or Kaposi
varicelliform eruption is a complication of AD that mandates urgent diagnosis and
prompt institution of appropriate antiviral therapy.
• The presence of AD is a contraindication to receiving the small pox vaccine because
patients can potentially get eczema vaccinatum, a severe and life-threatening dermatitis.

American Board of Pediatrics Content Specification


• Recognize the clinical findings associated with atopic dermatitis
• Recognize complications associated with atopic dermatitis
• Plan the appropriate management of atopic dermatitis

Suggested Reading
• Kuo Yoshida 7, De Benedetto A, Beck LA, et al. The cutaneous innate immune
response in patients with atopic dermatitis. I Allergy din Immunol. 2013;131(2):266-278.
doi.110.1016/j.jaci.2012.12.1563.
• Pongdee T. Bleach bath recipe for skin conditions. American Academy of Allergy
Asthma & Immunology website.
• Schneider I., Tilles 5, Lio P, et al. Atopic dermatitis: a practice parameter update 2012.
JAliergy Clip Immunol. 2013:131(2);295-299.el-e27. doi;10.1016/j.jaci.2012.12.672.

American academy of pediatrics 537


American Academy of Pediatrics PREP 2015

Item 175
You are taking care of a 3-day-old newborn with a long, narrow thorax with short ribs and short
limbs. She is diagnosed with asphyxiating thoracic dystrophy (Jeune syndrome) by a genetics
consultant. Vital signs show a temperature of 37°C, heart rate of 160 beats/min, respiratory rate
of 70 breaths/ min, blood pressure of 80/50 mm Hg, and oxygen saturation of 95% on room air.
Her breathing is fast and shallow. During inspiration, there is exaggerated paradoxical breathing
in which her abdomen protrudes out while her chest moves in, There is good air exchange
bilaterally with no wheezing or crackles.

Of the following, the BEST characterization of her respiratory pathophysiology is


A. decreased forced expiratory volume in 1 s
B. decreased lung elastance
C. decreased total lung capacity
D. increased dynamic lung compliance
E. increased functional residual capacity

American academy of pediatrics 538


American Academy of Pediatrics PREP 2015

Item 175 Preferred Response: C


The newborn in this vignette has Jeune syndrome, which is a form of asphyxiating thoracic
dystrophy. As with many forms of thoracic deformity, this syndrome causes restrictive lung
disease and decreased total lung capacity. Item C175A lists causes of restrictive pulmonary
disease in children.

Jeune syndrome is a rare autosomal recessive genetic disorder characterized by skeletal dysplasia
that is evident at birth, manifesting with deformities and respiratory distress. Skeletal deformities
include shortened ribs, small thoracic cage, reduction in limb bone length, and a characteristic
trident-shaped appearance of the pelvis on radiographic evaluation. Most affected children die of
respiratory failure because of pulmonary hypoplasia in infancy or early childhood. Micro-cystic
renal disease can also occur, leading to chronic renal insufficiency.

Restrictive lung disease is the failure of lung expansion from parenchymal, pleural, or chest wall
excursion disorders. Parenchymal causes can be further subdivided into alveolar disorders, such
as atelectasis, hemorrhage, hypoplasia, and infection, and extra-alveolar causes such as fibrosis
and interstitial edema. Pleural causes include infection, pleural effusion, and pneumothorax.
Chest wall excursion causes include diaphragmatic hernia, skeletal malformations, abdominal
distention, thoracic dystrophy, flail chest, and kyphoscoliosis.

In older children, the impact of chest wall deformities on lung function can be assessed by
pulmonary function tests (PFTs). Pulmonary function tests can be helpful in differentiating
obstructive causes of respiratory insufficiency (ie, asthma, cystic fibrosis, and other airway
abnormalities) from restrictive causes. Severity of disease and response to treatment modalities
can also be assessed. The most useful measurements in PFTs are forced vital capacity (FVC),
forced expiratory volume in 1 second (FEV1, and forced expiratory flow (FEF) between 25% and
75% FVC (FEF25%-75%). In obstructive diseases, FVC, FEV1, and FEF are all decreased. In
restrictive diseases, FVC and FEV1, are either normal or decreased. The ratio of FEV, to FVC is
increased because FVC is decreased to such a large extent. Item C175B and Item C175C show
the PFT findings in obstructive and restrictive pulmonary disease.

American academy of pediatrics 539


American Academy of Pediatrics PREP 2015

American academy of pediatrics 540


American Academy of Pediatrics PREP 2015

In the newborn in this vignette, assessing total lung capacity using PFTs would not be feasible
because of her age. Pulmonary function tests require effort and the ability to follow commands,
and generally cannot be done until the patient can cooperate. Total lung capacity in this newborn
would be decreased because of decreased thoracic volume. Lung compliance, which is the
reciprocal of elastance, is the change in volume for a given change in pressure. Restrictive lung
diseases, such as pneumonia, pulmonary edema, and atelectasis, as well as chest wall
abnormalities, manifest as decreased compliance and increased elastance. Functional residual
capacity is increased in obstructive lung disease, but decreased in restrictive lung disease. Item
C175D depicts a classic spirogram showing tidal breathing and the divisions of total lung
capacity.

American academy of pediatrics 541


American Academy of Pediatrics PREP 2015

Two broad classifications of respiratory failure include obstructive and restrictive diseases.
Obstructive diseases generally cause resistance to airflow at any point from the pharynx to the
terminal bronchioles, and can lead to air trapping and increased functional residual capacity.
Restrictive diseases are marked by insufficient lung expansion and therefore decreased lung
volume.

PREP Pearls
• Two broad classifications of respiratory disease include obstructive and restrictive.
• Pulmonary function testing can differentiate restrictive lung disease from obstructive lung
disease and assess disease severity, but requires patient effort
• Skeletal deformities involving the thoracic cage can cause restrictive lung disease.

American Board of Pediatrics Content Specification(s)


• Recognize the association of thoracic deformities with restrictive pulmonary disease

Suggested Reading
• Eigen H. Pulmonary function testing a practical guide to its use in pediatric practice.
Pediatr Rev. 1986;7(8):235-245. doi:10.1542/pir.7-8-235.
• Ellis DG. Chest wall deformities. Pediatr Rev. 1989;11(5)147-151. doi:10.1542 /pi
1%11-5-147.
• Pagtakhan RD, Chernick V. Respiratory failure in the pediatric patient. Pediatr Rev.
1982;3(8):247-256. doi:10.1542/pir.3-8-247.

American academy of pediatrics 542


American Academy of Pediatrics PREP 2015

Item 176
An 8-year-old African-American boy in your practice is on home intravenous antibiotic therapy
with nafcillin for treatment of acute osteomyelitis of the distal humerus of his left arm. An
aspirate in the operating room at the time of diagnosis grew methicillin-sensitive Staphylococcus
aureus (MSSA). He has responded well clinically with resolution of fever and decrease in pain at
the initial site of infection. He has no complaints today and is now being seen for follow-up 2.5
weeks into therapy. His temperature is 37.1°C. His physical examination reveals a well-healing
incision site over the distal humerus in his left arm. The examination findings are unremarkable.

A complete blood cell count obtained today shows the following:


• White blood cell count, 6,200/µL (6.2 x 109/L), with 6% neutrophils, 76% lymphocytes,
14% monocytes, 3% eosinophils, and 1% basophils
• Hemoglobin, 13.4 g/dL (134 g/L)
• Platelet count, 387 x 103/µL (387 x 109/L)
• Erythrocyte sedimentation rate, 22 mm/h (reference ranger 25.00 mm/h)
• C-reactive protein, 0.60 mg/L (5.7 nmol/L) (reference range, < 0.80 mg/L [< 7.6
nmol/L])
The complete blood cell count at the time of his hospital admission showed the following:
• White blood cell count, 12,000/µL (12 x 109/L), with 45% neutrophils, 5% bands, 40%
lymphocytes, and 10% monocytes
• Hemoglobin, 13.8 g/dL (138 g/L)
• Platelet count, 450 x 103/µL (450 x 109/L)

Of the following, the MOST likely cause of this patient's neutropenia is


A. cyclical neutropenia
B. drug-induced neutropenia
C. ethnic neutropenia
D. intercurrent viral illness
E. laboratory error

American academy of pediatrics 543


American Academy of Pediatrics PREP 2015

Item 176 S Preferred Response: B


The child in this vignette, although well appearing and afebrile, presents with marked
neutropenia (absolute neutrophil count [ANC) = 372/mm3). He has been on intravenous therapy
with nafcillin for osteomyelitis caused by infection with methicillin-susceptible Staphylococcus
aureus for 2.5 weeks. He had a normal ANC at the time of diagnosis. In this setting, drug-
induced neutropenia secondary to nafcillin is the most likely diagnosis and stopping the drug
(and continuing treatment with an alternative agent such as cefazolin) should lead to
improvement in the ANC over the next 1 to 2 weeks. This is a relatively rare idiosyncratic
reaction seen with nafcillin and penicillin G.

Cyclic neutropenia is an inherited disorder that would be expected to present before 8 years of
age and typically is characterized by cycles of neutropenia occurring approximately every 3
weeks. During the nadirs, fever, oral ulcerations, gingivitis, pharyngitis, skin infections, and
invasive bacterial infections may be seen.

People of African descent may normally have a mild neutropenia with ANC between 1,000 and
1,500/mm`. This type of ethnic neutropenia is not associated with an increased risk of infection.
The normal ANC at time of presentation and the severity of the observed neutropenia in this case
argue against the diagnosis of ethnic neutropenia for the boy in this vignette.

Acute viral infections such as Epstein-Barr virus and influenza may be associated with
neutropenia early in the course of infection. The child in this case has no stigmata of acute viral
infection to suggest this as a consideration here, Sepsis or severe bacterial infection may also
lead to neutropenia, but the child would have to be clinically ill for this to be a consideration.
Given the timing of the neutropenia in relation to the nafcillin therapy and the other components
of the complete blood cell count being normal, there is no reason to suspect a laboratory error as
the explanation for the child's neutropenia. A repeat white blood cell count and differential
would be expected to demonstrate the same finding.

This adverse reaction to nafcillin is a relatively rare example of the types of drug reactions to
antibiotics seen in children. Allergic or hypersensitivity reactions are the most common adverse
events associated with exposure to antibiotics. Such reactions may be immediate, beginning
within 1 hour of the first dose of the drug, or delayed, occurring after days of treatment.
Immediate reactions are typically manifest by urticarial rash, pruritus, and potentially
angioedema. These reactions are immunoglobulin E-mediated and carry the risk of anaphylaxis
with re-exposure to the offending agent.

Delayed drug reactions typically begin 7 to 10 days after onset of therapy with a given agent and
are characterized by maculopapular eruptions. Amoxicillin is the most commonly implicated
agent associated with these types of reactions. A severe form of delayed antibiotic reaction is
drug rash with eosinophilia and systemic symptoms (DRESS) syndrome that can occur 1 to 12
weeks into a course of treatment. The β-lactam drugs (penicillins and cephalosporins) are the
antibiotics most often associated with allergic reactions.

Other examples of drug reactions unrelated to allergy may include gastrointestinal upset or
diarrhea, nephrotoxicity (eg, aminoglycosides, vancomycin), or "red man" syndrome (too rapid
American academy of pediatrics 544
American Academy of Pediatrics PREP 2015

infusion of vancomycin) may lead to excessive release of histamine, causing the "red man"
appearance. This is not an allergic reaction and can be managed, when it occurs, by slowing the
rate of infusion from 1 to 2 hours per dose.

PREP Pearls
• Marked neutropenia can occur in the course of treatment with nafcillin or penicillin G.
• Drug allergies are the most common manifestation of adverse drug reactions.
• Immediate drug reactions, beginning within 1 hour of drug intake, are associated with
urticaria, angioedema, and potentially anaphylaxis, and are immunoglobulin E-mediated.
• Delayed drug allergies typically occur 7 to 10 days after beginning a course of therapy
and are associated with maculopapular eruptions, but anaphylaxis is not a risk in this
setting.

American Board of Pediatrics Content Specification(s)


• Recognize the adverse effects associated with the use of various antibiotic drugs

Suggested Reading
• Greene GR, Cohen E. Nafcillin-induced neutropenia in children. Pediatrics.
1978;61(1):94-97.
• Pichler W. Drug allergy: classification and clinical features. UpToDate. Available online
only for subscription.
• Segel GB, Halterman JS. Neutropenia in pediatric practice. Pediatr Rev. 2008;29(1):12-
24. doi:10.1542/pir.29-1-12.

American academy of pediatrics 545


American Academy of Pediatrics PREP 2015

Item 177
You are evaluating a 2-day-old neonate in the hospital. The mother mentions that the newborn is
feeding every 3 to 4 hours and has had 7 wet diapers in the last 24 hours. She is concerned about
the funny urinary stream she noticed in the morning. His physical examination is significant only
for an uncircumcised male infant with a ventral opening of the penile urethra (Item Q177).

Of the following, the MOST appropriate next step in the management of this patient's condition
is
A. circumcision
B. reassurance
C. renal ultrasonography
D. urology referral
E. urine culture

American academy of pediatrics 546


American Academy of Pediatrics PREP 2015

Item 177 Preferred Response: D


The patient in the vignette has distal hypospadias. In patients with hypospadias, the urethral
meatus opens on the ventral surface of the penis proximal to the normal glanular location.
Hypospadias is the most common malformation of the male genitalia, occurring in 1 in 300
births. Hypospadias is classified by the anatomic location of the meatus and the presence or
absence of chordee (ventral curvature of the penis). Hypospadias can be classified as first degree
(on the glans), second degree (on the penile shaft), and third degree (penoscrotal to perineal).
Another classification approach is distal (glandular, coronal, subcoronal), middle (distal shaft,
midshaft, posterior penile), or proximal (penoscrotal, scrotal, perineal). The most common form
of hypospadias is distal (subcoronal)/first degree.

Proximal hypospadias is more frequently associated with other malformations than is distal
hypospadias. Genital malformations (such as cryptorchidism and inguinal hernia) are more
common (8%-15%) than urinary tract abnormalities (such as vesicoureteral reflux, ureteropelvic
junction, ectopic kidneys, and renal agenesis) in patients with hypospadias. The association of
distal forms of hypospadias with renal anomalies (1% in patients with distal and 5% in patients
with proximal hypospadias) is no greater than that of the general population, making it
unnecessary to perform renal ultrasonography in patients such as the child in this vignette.
The penile foreskin is used in hypospadias repair; therefore, circumcision is contraindicated in
these patients. Patients with hypospadias should be referred to a urologist within the first few
weeks after birth. This initial evaluation addresses parental concerns and provides them with
information about the required surgical intervention. The optimal time for hypospadias repair is
at 6 months of age. A referral to genetics should be considered in patients with third-degree or
proximal hypospadias.
Urine culture and reassurance are not the next indicated steps in evaluating the patient in the
vignette.

PREP Pearls
• In hypospadias, the urethral meatus opens on the ventral surface of the penis, proximal to
the normal glanular location.
• Proximal hypospadias is more frequently associated with other malformations than is
distal hypospadias.
• The penile foreskin is used in hypospadias repair; therefore, circumcision is
contraindicated in these patients.
• Patients with hypospadias should be referred to a urologist within the first few weeks
after birth.

American Board of Pediatrics Content Specification(s)


• Recognize disorders associated with hypospadias
• Plan the appropriate management of hypospadias

Suggested Reading
• Madhok N, Scharbach K, Shahid-Saless S. Hypospadias. Pediatr Rev. 2009;30:235-237.
doi:10.1542/pir.30-6-235.
• Snodgrass WT. Consultation with the specialist: hypospadias. Pediatr Rev. 2004;25:63-
67. doi:10.1542/pir.25-2-63.
American academy of pediatrics 547
American Academy of Pediatrics PREP 2015

Item 178
A 9-year-old girl presents to the emergency department because of an episode of hematemesis.
Her past medical history is unremarkable except for her birth at 30 weeks' gestational age,
followed by a 4-week stay in the neonatal intensive care unit. During her neonatal
hospitalization, she was on a ventilator for 10 days because of respiratory distress syndrome and
was also treated for suspected sepsis. Her hospital records indicate that both umbilical artery and
vein catheterizations were employed for management. On physical examination, she is afebrile,
pale, and diaphoretic. Vital signs include a resting heart rate of 100 beats/min and a blood
pressure of 90/60 mm Hg. The lungs are clear to auscultation, and a grade 2/6 holosystolic
murmur is heard throughout the precordium. Abdominal examination demonstrates no
hepatomegaly. The spleen tip is palpated 4 cm below the left costal margin. Rectal examination
shows dark, guaiac test positive stool. Initial laboratory data demonstrate the following:

• Hematocrit, 24% (0.24)


• White blood cell, 2,800/µL. (2.8 x 109/L)
• Platelets, 80 x 103/ µL (80 x 109/L)
• Blood urea nitrogen, 32 mg/dL (11.4 mmol/L)
• Creatinine, 1.0 mg/dL (88.4 umol/L)
• Prothrombin time, 12 s
• Partial thromboplastin time, 33 s
• International normalized ratio, 0.9
• Aspartate aminotransferase, 60 U/L (reference range, 20-50 U/L)
• Alanine aminotransferase 30 U/L (reference range, 20-60 U/L)
• Albumin, 4.0 g/dL (40.0 g/L)
After the intravenous administration of 1,000 mL of normal saline, her vital signs are improved
and she demonstrates no orthostatic changes. A nasogastric tube is passed, and it returns fluid
that contains coffee-ground-appearing particles that clears after lavage with room temperature
saline.

Of the following, the MOST appropriate test to demonstrate the etiology of this child's bleeding
is
A. abdominal computed tomography
B. abdominal ultrasonography
C. hepatobiliary scintigraphy
D. percutaneous liver biopsy
E. tagged red blood cell scan

American academy of pediatrics 548


American Academy of Pediatrics PREP 2015

Item 178 Preferred Response: B


The child in the vignette presents with hematemesis and splenomegaly, with a medical history
that includes premature delivery and instrumentation of the umbilical vein during the neonatal
period. These findings strongly suggest a diagnosis of portal hypertension (PH) and bleeding
either from esophageal or gastric varices. After initial hemodynamic stabilization, a nasogastric
aspirate returned coffee-ground material, indicating cessation of active bleeding. In this setting,
esophagogastroduodenoscopy (EGD) is the preferred modality for identifying the source of
upper gastrointestinal bleeding. This procedure should be conducted within the first 24 hours of
presentation and after the patient is stabilized. Before endoscopy, and especially in the patient
who presents with signs of PH and who has not experienced a bleeding episode, imaging studies
will help define the cause of portal venous obstruction. The most appropriate of these tests is
abdominal ultrasonography with Doppler assessment of blood flow.

Portal hypertension is defined either as a portal venous pressure of more than 5 mm Hg, or a
portal to hepatic vein pressure gradient of more than 10 mm Hg. In most cases, PH is suspected
on the basis of combined clinical and laboratory data. As the result of portal venous congestion,
collateral circulation develops at the junction of the high pressure and the low pressure venous
system. The result is the formation of varices that, in children, develop predominantly in the
esophagus and stomach.

The child's medical history is of paramount importance in raising an index of suspicion for PH.
Any patient with a history of chronic liver disease is at risk. Intrahepatic causes include disorders
leading to presinusoidal, sinusoidal, or postsinusoidal venous obstruction. In children, liver
disorders leading to cirrhosis comprise the most prevalent causes of PH, with biliary atresia
being the most common. Extra-hepatic PH may develop as the result of posthepatic and
prehepatic causes of portal venous obstruction. A common prehepatic etiology is portal vein
thrombosis, arising from instrumentation of the umbilical vein during the neonatal period. Other
etiologies of portal vein thrombosis include neonatal omphalitis (and other intra-abdominal
infections), severe dehydration, and blunt abdominal trauma. Posthepatic etiologies of portal
venous obstruction include Budd Chiari syndrome and veno-occlusive and cardiac diseases. A
family history of metabolic disorders or a hypercoagulable state are additional risk factors.
Physical examination in the majority of children with PH will demonstrate splenomegaly. A
prominent exception is the child with asplenia or polysplenia, as in the fetal form of biliary
atresia. Splenomegaly is the most sensitive among clinical and laboratory indicators (excluding
imaging or endoscopic studies) for the presence of esophageal varices. Depending on the
etiology of PH, the liver size may be enlarged (metabolic disease, Budd-Chiari syndrome),
normal, or small (cirrhosis). The presence of ascites indicates underlying liver disease. Aside
from splenomegaly, other physical evidence of varices (caput medusae, rectal hemorrhoids) is
uncommon in children.

Laboratory studies will suggest a diagnosis of PH if the portal venous pressure gradient is
sufficient enough to cause splenomegaly and consequent hypersplenism. Leukopenia and
thrombocytopenia are common. Splenic sequestration of red blood cells may also occur. Anemia
in the patient with PH may be the result of hypersplenism, variceal bleeding, or portal
gastropathy. In patients with PH secondary to intrinsic liver disease, particularly hepatitis B and
C, hypoalbuminemia has been reported to be a nonspecific factor associated with the presence of
American academy of pediatrics 549
American Academy of Pediatrics PREP 2015

varices. Elevations in aminotransferases or coagulation abnormalities indicate hepatocellular


injury and dysfunction, but these tests do not specifically suggest PH.

Following ultrasonographic documentation of portal venous obstruction and hepatofugal


(reverse) portal venous flow, endoscopic intervention will allow for visualization and appropriate
management of esophageal and gastric varices (eg, endoscopic ligation of esophageal varices).
Other diagnostic studies are not indicated. Computed tomography may demonstrate varices, but
is associated with significant radiation exposure. Furthermore, the advantage of ultrasonography
includes the ability to use Doppler technology to assess blood flow. Tagged red blood cell scans
may be useful in evaluating gastrointestinal blood loss in cases in which the site of bleeding is
unknown, but is only valuable in the actively bleeding patient. Hepatobiliary scintigraphy may
be used in the evaluation of cholestasis and percutaneous liver biopsy remains an essential part
of the workup for patients with chronic liver disease. However, in the child in the vignette, the
clinical and laboratory findings indicate normal hepatobiliary function and suggest PH secondary
to a prehepatic cause.

PREP Pearls
• Abdominal ultrasonography with Doppler is the best noninvasive test for assessing portal
vein status.
• Splenomegaly is the most sensitive indicator of portal hypertension and varices.
• In the patient with upper gastrointestinal bleeding, a coffee-ground nasogastric aspirate
indicates cessation of active bleeding.

American Board of Pediatrics Content Specification(s)


• Identify the physical and laboratory features associated with portal hypertension

Suggested Reading
• Fagundes ED, Ferreira AR, Roquete ML et aI. Clinical and laboratory predictors of
esophageal varices in children and adolescents with portal hypertension syndrome. I
Pediatr Gastroenterol Nutr. 2008;46(2):178-183. doi:10.1097/MPG.0b013e31815611-07.
• Gana JC, Turner D, Mieli-Vergani G, et al. A clinical prediction rule and platelet count
predict esophageal varices in children. Gastroenterology, 2011:141(6):2009-2016.
doi:10.1053/j.gastro.2011.08.049.
• Gugig R, Rosenthal P. Management of portal hypertension in children. World J
Gastroenterol. 2012;18(10:1176-1184. doi:10.3748/wjg.v18.i11.117(
• Shneider BL, Bosch J, de Franchis R, et al. Portal hypertension in children: expert
pediatric opinion on the report of the Baveno V Consensus Workshop on Methodology of
Diagnosis and Therapy in Porta Hypertension. Pediatr Transplant. 2012;16(5):426-437.
doi:10.1111/j.1399- 3046.2012.01652.x.

American academy of pediatrics 550


American Academy of Pediatrics PREP 2015

Item 179
A 5-year-old boy is brought to your office as a new patient with a complicated history of
weakness in his thighs and upper arms with exercise intolerance, recurrent migraine headaches,
recurrent vomiting, loss of appetite, and seizures. The symptoms started at the age of 2.5 years.
The seizures are described as altered consciousness, with paralysis of one side of his body that
progresses to a generalized rhythmic jerking of all 4 extremities. The mother has noted that
following his seizures, he experiences some loss of skills without re-attainment of the skills. The
seizures had initially occurred every 3 to 4 months, but are now monthly. She has also noted
some gradual hearing loss. She reports some polydipsia and polyuria.

His early development was normal, but his development slowed with some periods of regression
following the onset of his seizures. In fact, he now seems delayed (at about the level of a 3-year-
old) when compared to his peers. He also has problems with attention. His mother has a history
of migraines, an unexplained cardiomyopathy, and exercise intolerance. His height and weight
are at less than the fifth percentile. Other than myopathic facies, he is nondysmorphic in
appearance. No hepatosplenomegaly is noted. His musculature is thin, with noted hypotonia and
weakness. His skin is pale.

Laboratory investigations show lactic acidosis both in blood and in the cerebrospinal fluid (> 2.5
mmol) and an elevated lactate-to-pyruvate ratio (> 20 mmol). Cerebrospinal fluid protein was
elevated at 80 mg/dL. Brain magnetic resonance imaging shows white matter change in the
frontal and parietal white matter, with decreased T1 and increased T2 signal. There is abnormal
diffusion within the globus pallidus bilaterally. There is a mild degree of ventriculomegaly, but
no obstructive hydrocephalus. There is no intracranial mass lesion (Item Q179).

American academy of pediatrics 551


American Academy of Pediatrics PREP 2015

Of the following, the MOST likely diagnosis is


A. biotinidase deficiency
B. congenital disorder of glycosylation
C. homocystinuria
D. lysosomal storage disease
E. mitochondrial disease

American academy of pediatrics 552


American Academy of Pediatrics PREP 2015

Item 179 Preferred Response: E


The patient in the vignette has a mitochondrial disorder most likely MELAS (mitochondrial
encephalomyopathy with lactic acidosis and stroke-like episodes). He present with early normal
development followed by the develop ment of childhood-onset myopathy, seizures, migraines
early signs of diabetes mellitus, stroke-like episodes associated with periods of regression,
laboratory evidence of lactic acidosis in both serum and cerebrospinal fluid (CSF), one elevated
CSF protein. In addition, neuroimaging is suggestive of bilateral globus pallidus involvement in
the context 01 progressive white matter changes. This is a classic presentation for MELAS. This
patient's family history of symptoms in his mother also suggests a potential maternal inheritance
pattern that is common among mitochondrial disorders arising from mutations in the
mitochondrial DNA.

Mitochondrial disorders are a heterogeneous group of diseases arising from functional


abnormalities in oxidative phosphorylation, also known as the mitochondria] respiratory chain,
which is the final common biochemical pathway for aerobic metabolism. Tissues and organ
systems that are extremely dependent on aerobic metabolism are typically involved to the
greatest degree, including the heart, muscles, and the nervous system. Mitochondrial disorders
can result from gene mutations or deletions in the nuclear DNA or the mitochondrial DNA
(mtDNA), thus they can be inherited in an autosomal-recessive, autosomal-dominant, X-linked,
or maternal inheritance pattern. Mitochondrial DNA is transmitted exclusively by maternal
inheritance, as women give their mitochondrial DNA to all of their offspring and men do not
transmit mtDNA to any of their offspring. A woman can transmit a variable amount of mutated
mtDNA to each of her offspring, thus yielding significant clinical variability among her children.
If a defect is in the nuclear DNA that encodes components of the respiratory chain, it can be
transmitted by the mother or father to their offspring in a dominant or recessive pattern.
Mitochondria' disorders can present at any age and may initially affect only a single organ or
multiple organ systems; however, the predominant features tend to be neurologic and exhibit
myopathic symptoms in most individuals. One of the cardinal features is progressive organ
system involvement over the course of the disease. Significant clinical variability is common and
presenting symptoms may include short stature, cardiomyopathy, diabetes mellitus, proximal
myopathy, intolerance to exercise, pancytopenia, sensorineural deafness, optic atrophy,
pigmentary retinopathy, ptosis, and external ophthalmoplegia. Neurologic manifestations are
typical and include developmental delay, seizures, dementia, migraine headaches, spasticity,
hypotonia, neuropathy, ataxia, encephalopathy, and stroke-like episodes. Many affected patients
will manifest a particular cluster of features that will delineate a specific mitochondrial
syndrome. Some of the more common syndromes include myoclonic epilepsy with ragged-red
fibers, Leigh syndrome, Keams-Sayre syndrome, MELAS, and chronic progressive external
ophthalmoplegia.

Diagnosis is sometimes made by recognizing a characteristic clinical presentation of a specific


disorder involving the mitochondria; however, it is frequently necessary to take a good family
history, check blood or cerebrospinal fluid (CSF) lactate or pyruvate levels, obtain
neuroimaging, order cardiac evaluation, order molecular genetic testing, and obtain a muscle
biopsy specimen for histologic evidence, such as ragged red fibers and respiratory chain enzyme
analysis, supporting a mitochondrial diagnosis. Neuroimaging findings are diagnostic and may
illustrate basal ganglia calcification, diffuse atrophy, focal atrophy of the cortex or cerebellum,
American academy of pediatrics 553
American Academy of Pediatrics PREP 2015

generalized leukoencephalopathy (white matter changes), or cerebellar atrophy. Laboratory


findings commonly feature lactic acidosis in the blood or CSF.

Treatment of mitochondrial disorders is mainly supportive from a systemic standpoint because


there is no defined cure at this time. Certain mitochondrial disorders can benefit from
supplementation of vitamins and cofactors, such as coenzyme Q10, riboflavin, idebenone, or
carnitine depending on the disorder, but a systematic review of the Cochrane database finds no
evidence of the global benefit of these supplements for mitochondrial disorders.
Biotinidase deficiency, if untreated, presents with seizures, hypotonia, developmental delay,
vision abnormalities, hearing loss, ataxia, hair loss, and skin rashes in young children. As the
children age, they develop spastic paresis and motor weakness. It is commonly diagnosed on
newborn screening because it is a highly treatable disorder that is amenable to oral
supplementation of biotin with excellent outcomes. Once many of the manifestations described
have occurred, they are typically irreversible; thus the importance of early recognition and
treatment.

Congenital disorders of glycosylation are a group of disorders caused by altered glycosylation of


N-linked oligosaccharides. These disorders present in infancy with clinical variability, ranging
from severe developmental delay, hypotonia, and systemic involvement to coagulopathy with
stroke-like episodes, to normal development with associated recurrent hypoglycemia and failure
to thrive. Patients often have inverted nipples, abnormal subcutaneous fat distribution,
strabismus, gastroesophageal reflux, hypoproteinemia because of protein-losing enteropathy, and
a hypoplastic cerebellum with ataxia. It is diagnosed with a carbohydrate-deficient transferrin
isoform analysis, which is a biochemical screening test.

Homocystinuria patients have a "marfanoid" appearance. This disorder is caused by


cystathionine p-synthase deficiency resulting in intellectual delays, ectopia lentis, severe myopia,
tall stature, and thromboembolism that can lead to an early death. Biochemical features include
significantly elevated concentrations of plasma homocystine, total homocysteine, and
methionine. Treatment is focused on correction of the biochemical abnormalities involving
plasma homocystine and homocysteine concentrations that can help prevent thrombosis. Patients
are typically placed on protein- and methionine-restricted diets, as well as betaine, folate, and
vitamin supplementation.

Lysosomal storage disorders are a heterogeneous group of disorders that present with
accumulation of undigested or partially digested macromolecules in varying organs, causing
cellular dysfunction and systemic pathology. They are classified by the type of the accumulated
substrate: mucopolysaccharidoses, mucolipidosis, oligosaccharidosis, etc. Patients can present
with coarse facies, macroglossia, dysostosis multiplex, cardiomegaly, hepatosplenomegaly,
unusual ophthalmologic findings (corneal clouding, cherry-red spot in the macula),
developmental delay, regression, hypotonia, seizures, and intellectual disability.

PREP Pearls
• MELAS (mitochondrial encephalomyopathy with lactic acidosis and stroke-like
episodes) is a mitochondrial disorder that presents with childhood onset of myopathy,
seizures, migraines, early signs of diabetes mellitus, stroke-like episodes associated with
American academy of pediatrics 554
American Academy of Pediatrics PREP 2015

periods of regression, and laboratory evidence of lactic acidosis in both serum and
cerebrospinal fluid.
• Mitochondrial disorders can present at any age, occur with any inheritance pattern, and
may affect only a single organ or multiple organ systems; however, the predominant
features tend to be neurologic and myopathic in most individuals with progressive organ
system involvement over time.
• Tissues and organ systems that are extremely dependent on aerobic metabolism are
typically involved to the greatest degree, including the heart, muscles, and the nervous
system.

American Board of Pediatrics Content Specification(s)


• Recognize the clinical features associated with mitochondrial disorders

Suggested Reading
• Chinnery PF. Mitochondria! disorders overview. GeneReviews. Chinnery P, Majamaa K,
Turnbull D, Thorburn D. Treatment for mitochondria! disorders. Cochrane Database Syst
Rev. 2006;1:CD004426. DiMauro 5, Hirano M. MELAS. GeneReviews.
• Munnich A, Rustin P. Clinical spectrum and diagnosis of mitochondrial disorders. Am
/Med Genet. 2001;106:4-17. doi:10.1002/ajmg.1391.

American academy of pediatrics 555


American Academy of Pediatrics PREP 2015

Item 180
A 10-year-old girl presents for her routine health maintenance visit. Upon further questioning,
the mother admits that the patient always snores at night and that the snoring has gotten worse
this last year. She has been treated twice for throat infections this year and the mother has been
told her daughter's tonsils are large. They have tried allergy medication episodically without
much improvement. On physical examination, you notice darkening under the eyes, pale and
large nasal turbinates, plus adenoidal facies. The tonsils are hypertrophied and the left tonsil is
significantly larger than the right.

Of the following, the BEST next step in management is


A. prescribe a nasal steroid for daily use
B. prescribe an oral antihistamine for daily use
C. reassure that the tonsils will get smaller with age
D. refer for polysomnography
E. referral to otolaryngology

American academy of pediatrics 556


American Academy of Pediatrics PREP 2015

Item 180 Preferred Response: E


Although excision of the tonsils and adenoids of children and adolescents has been a
controversial subject for many years, there are some absolute indications for surgery. The girl in
the vignette is an example of one of those indications. Significant asymmetry without signs of
acute infection of the tonsils on examination should raise suspicion for malignancy, especially if
the asymmetry develops rapidly. An immediate referral to otolaryngology is needed for further
evaluation. Other absolute indications for surgical excision include extreme obstruction of the
airway, interference with swallowing, and uncontrollable hemorrhage from tonsillar blood
vessels.

Other conditions that lead to the consideration of tonsillectomy or adenoidectomy include


recurrent acute or chronic tonsillitis or sinusitis, recurrent acute otitis media or chronic otitis
media with effusion, alteration of voice quality because of adenotonsillar hypertrophy, refractory
halitosis, peritonsillar abscess, or syndrome of periodic fever, aphthous stomatitis, pharyngitis,
and cervical adenitis (PFAPA syndrome). The decision to perform elective tonsillectomy or
adenoidectomy should be individualized according to potential risks and benefits for each
patient.

The use of daily nasal corticosteroids is considered first-line therapy for several mucosal
etiologies of nasal obstruction (eg, rhinitis, nasal polyposis) and may be effective in cases of
adenoidal hypertrophy. Those who respond inadequately to pharmacologic therapy should be
referred to otolaryngology for evaluation and possible surgical management. Oral antihistamines
can also be helpful in patients with persistent allergic rhinitis as the cause of their nasal
obstruction.

Adenotonsillar hypertrophy is most pronounced between 3 and 8 years of age and is usually
associated with acute or chronic infections. These structures tend to involute in most children
after about 10 years of age because the immunologic activity of this lymphoid tissue decreases
after puberty. Therefore, snoring and sleep disordered breathing should resolve with time, rather
than worsen as in this patient. Plus, one should not simply reassure the family, given the
asymmetry of the tonsillar tissue in the girl in this vignette.

Adenotonsillectomy is suggested for healthy children who have adenotonsillar hypertrophy and
obstructive sleep apnea. It is not recommended for patients who have hypertrophy without
symptoms. Although uncommon, those with chronic airway obstruction from enlarged tonsils
and adenoids may develop cor pulmonale or adenoidal facies (elongation and flattening of the
midface and retrognathic mandible), so evaluation is important. Tonsil and adenoid size is not
predictive of the presence or severity of obstructive sleep apnea, so polysomnography should be
performed. Polysomnography is considered the gold standard for assessing suspected obstructive
sleep apnea, and referral for this examination before otolaryngology consultation would be
appropriate if the concern for malignancy (asymmetric tonsils) was not present.

PREP Pearls
• Significant asymmetry of the tonsils without signs of acute infection should raise
suspicion for malignancy and warrants prompt referral to otolaryngology.

American academy of pediatrics 557


American Academy of Pediatrics PREP 2015

• Adenotonsillar hypertrophy is most pronounced between 3 and 8 years of age and is


usually associated with acute or chronic infections.
• Involution of tonsillar and adenoidal lymphoid tissue occurs in most children around the
time of puberty.
• Polysomnography is the gold standard for assessing suspected obstructive sleep apnea.

American Board of Pediatrics Content Specification(s)


• Recognize conditions associated with tonsillar and/or adenoidal hyper trophy

Suggested Reading
• Bhattacharyya N. Clinical presentation, diagnosis, and treatment of nasal obstruction.
UpToDate. Available online only for subscription.
• Isaacson G. Tonsillectomy care for the pediatrician. Pediatrics. 2012;130(2)324-334.
doi:10.1542/peds.2011-3857.
• Marcus CL, Brooks LI, Draper KA, et al. Diagnosis and management of childhood
obstructive sleep apnea syndrome. Pediatrics. 2012;130(3):576584.
doi:10.1542/peds.2012-1671.
• Paradise IL. Tonsillectomy and adenoidectomy in children. UpToDate. Available online
only for subscription.
• Paruthi S. Management of obstructive sleep apnea in children. UpToDate. Available
online only for subscription.
• Wetmore RF. Tonsils and adenoids. In: Kliegman RM, Stanton BE, St. Geme IW III,
Schor NF, Behrman RE, eds. Nelson Textbook of Pediatrics. 19th ed. Philadelphia, PA:
Saunders Elsevier; 2011:1442-1445.

American academy of pediatrics 558


American Academy of Pediatrics PREP 2015

Item 181
An 18-month-old boy presents to your clinic for a routine well-child visit. There is no history of
serious illnesses or infections. His weight and height are at the 50th percentile for age. In the
office, he is afebrile with normal vital signs.

He is alert and in no apparent distress. On physical examination, you palpate a large nontender,
right upper quadrant abdominal mass. The remainder of the physical examination is
unremarkable. Ultrasonography is performed and shows a large mass suggestive of a malignancy
in the liver.

Of the following, the laboratory findings MOST likely to be seen in this patient would be
A. anemia, elevated titers for hepatitis C, and elevated carcinoembryonic antigen
B. anemia, thrombocytosis, and elevated α-fetoprotein
C. elevated d-dimers, thrombocytopenia, and decreased fibrinogen
D. elevated ferritin, elevated catecholamines, elevated lactate dehydrogenase
E. leukocytosis, thrombocytopenia, and elevated β-human choriogonadotropin

American academy of pediatrics 559


American Academy of Pediatrics PREP 2015

Item 181 Preferred Response: B


The child in this vignette has a presentation concerning for hepatoblastoma, which is associated
with anemia, thrombocytosis, and elevated α-fetoprotein (AFP) concentrations.
The incidence of all pediatric liver tumors is 1.5 per million annually. Two-thirds of hepatic
masses in children are malignant, and of these, two-thirds are hepatoblastoma (HB).
Hepatoblastoma usually occurs in infants and very young children, whereas the next most
common pediatric liver tumor, hepatocellular carcinoma usually occurs in patients older than 10
years of age. Both of these liver tumors have a slight male predominance. Hepatoblastoma may
be associated with very low birth weight infants, hemihypertrophy, and several cancer genetic
syndromes such as Beckwith-Wiedemann, familial adenomatous polyposis, Li-Fraumeni, and
trisomy 18.

Most childhood liver tumors are found incidentally on palpation by a caretaker or pediatrician,
but in cases of advanced disease, pain, weight loss, nausea, vomiting, or anorexia may also be
seen. Evaluation of a suspected liver tumor includes ultrasonography to look for increased
echogenicity of the right upper quadrant that would suggest malignancy. Doppler flow studies
can help assess tumor vascularity. To visualize tumor margins and determine resectability,
computed tomography (CT) or magnetic resonance imaging is necessary. The lungs are the most
common sites of metastasis, therefore CT of the chest should also be performed. Mild
normocytic anemia and thrombocytosis are often seen. The main tumor marker for HE is AFP,
which is elevated in more than 90% of cases. The most important prognostic factor in HB is
complete tumor resection. In many cases, when the tumor cannot be entirely resected,
chemotherapy is given. Reduction in AFP levels can be a good indicator of response to
chemotherapy.

Carcinoembryonic antigen may be elevated in carcinomas of the gastrointestinal tract, lung, and
breast, but are rarely elevated in pediatric malignancies. Elevated d-dimers, thrombocytopenia,
and decreased fibrinogen would be concerning for disseminated intravascular coagulopathy,
secondary to a lesion such as hemangioendothelioma. Increased ferritin, catecholamines, and
lactate dehydrogenase may be seen in neuroblastoma. Elevated β-human choriogonadotropin is
associated with germ cell tumors or pregnancy, but is not characteristic of HB.

PREP Pearls
• The incidence of all pediatric liver tumors is 1.5 per million annually. Two-thirds of
hepatic masses in children are malignant, and of these, two-thirds are hepatoblastoma
(HB). Hepatoblastoma usually occurs in infants and very young children.
• Hepatoblastoma may be associated with very low birthweight infants, hemihypertrophy,
and several cancer genetic syndromes such as Beckwith-Wiedemann, familial
adenomatous polyposis, Li-Fraumeni, and trisomy 18.
• Most childhood liver tumors are found incidentally on palpation by a caretaker or
pediatrician, but in cases of advanced disease, pain, weight loss, nausea, vomiting, or
anorexia may also be seen.
• Initial radiographic evaluation for a suspected liver mass should be ultrasonography of
the abdomen.
• Mild normocytic anemia and thrombocytosis are often seen in HB. The main tumor
marker for HB is α-fetoprotein, which is elevated in more than 90% of cases.
American academy of pediatrics 560
American Academy of Pediatrics PREP 2015

• The most important prognostic factor inn is complete tumor resection.

American Board of Pediatrics Content Specification(s)


• Recognize the laboratory findings associated with hepatoblastoma

Suggested Reading
• Agarwala S. Primary malignant liver tumors in children. Indian Pediatr.
• 2012;79(6):793-800. doi:10.1007/s12098-012-0704-1.
• Czauderna P, Lopez-Terrada D, Hiyama E, Haberie B, Malogolowkin MH, Meyers RL.
Hepatoblastoma state of the art: pathology, genetics, risk stratification, and
chemotherapy. Curr Opin Pediatr. 2013;26(1):19-28.
doi:10.1097/MOP.0000000000000046.

American academy of pediatrics 561


American Academy of Pediatrics PREP 2015

Item 182
On your office schedule today is a health supervision visit with a 4-year-old boy who was
recently placed into a foster home. The medical student working with you asks why children are
placed in foster care.

Of the following, the MOST likely statement to answer this student's question is that
A. 10% of children are in foster care through voluntary placement by their parents
B. due to greater disruptive behaviors, more boys than girls are placed in foster care
C. juvenile delinquents comprise the majority of children in foster care
D. neglect is the most common reason for placement of children in foster care
E. physical abuse is the most common reason for foster care placements

American academy of pediatrics 562


American Academy of Pediatrics PREP 2015

Item 182 SBP Preferred Response: D


Neglect by parents is by far the most common reason why children are placed into foster care
custody. Reasons for foster care placement include neglect (78%), physical abuse (18%), sexual
abuse (9%), emotional abuse (8%), medical neglect (2%), other reasons such as abandonment,
threats of harm, or congenital drug addiction (10%). (Note these frequencies total more than
100% because some children have more than one reason for placement.)

Fewer than 1% of foster children are placed there by parents voluntarily. Even parents who are
obviously unable to function as a parent tend not to want to terminate their parental rights,
requiring state service organizations to perform the more difficult process of terminating parental
rights through court proceedings (when reunification clearly does not make sense). For financial
and system capacity reasons, states may resist accepting a child into foster care placement on a
voluntary basis unless there is clear neglect or abuse. Placing a child with complex medical or
psychiatric needs into foster care/state custody is sometimes considered by parents once their
health insurance benefits have been exhausted and home care needs have become overwhelming.

There is no significant difference in the number of boys and girls in foster care. Although boys
may exhibit more physically disruptive behavior, this is not the reason for their foster care
placement.
Juvenile delinquents constitute a minority of the children in foster care. Approximately 40% of
children in foster care are adolescents, and nearly one third are younger than 5 years of age.
Fewer than 1 in 5 children in foster care have been victims of physical abuse. Most children are
in foster care because of parental neglect.

PREP Pearls
• Parental neglect is the main reason children are placed into foster care.
• Approximately 1 in 5 children in foster care have been victims of physical abuse.

American Board of Pediatrics Content Specification(s)


• Understand the psychosocial issues surrounding children in foster care

Suggested Reading
• Szilagyi M. The pediatric role in the care of children in foster and kinship care. Pediatr
Rev. 2012;33(11):496-507. doi:10.1542/pir.33-11-496.
• Szilagyi MA, lee SH. Epidemiology of foster care placement and overview of the foster
care system in the United States. UpToDate. 2013. Available online only for subscription.
• US Department of Health and Human Services. Child Maltreatment 2010. Accessed
February 20, 2014.

American academy of pediatrics 563


American Academy of Pediatrics PREP 2015

Item 183
A 14-year-old adolescent boy is brought to your office for evaluation of chest pain. He was
playing soccer yesterday and had to come out of the game because of the discomfort. He
describes the pain as dull and points to the middle of his chest. It was worse when he was
running downfield. The pain was as bad as when he broke his arm, and he grades it as a 9 out of
10. When he had the pain, he felt dizzy but did not pass out. The pain resolved with rest and
fluids. He does not remember any sensation of rapid heart rate. He recalls that he has had similar,
but less intense, episodes of pain with exercise for the last year. It usually happens when he is
pushing himself and running. He has a past history of reactive airway disease as an infant, but is
not taking any medications for that now He has never been hospitalized or had surgery. His
family history is significant for asthma and hypertension, but no individuals have died suddenly
or under the age of 30 years.
On physical examination, his heart rate is 90 beats/min, blood pressure is 95/60 mm Hg, and
respiratory rate is 18 breaths/min. Findings on neck examination are unremarkable, and his chest
is clear with equal breath sounds bilaterally. His cardiac examination shows a normal S1 and S2,
and no murmur, rub, thrill, or gallop. His liver is not enlarged, and his pulses are equal. In your
office, he begins to experience chest pain again. You obtain an electrocardiogram

(Item Q183).
Of the following, the MOST likely diagnosis in this child is
A. anomalous coronary artery
B. exercise-induced asthma
C. hypertrophic cardiomyopathy
D. pleuritis secondary to mycoplasma pneumonia
E. spontaneous pneumothorax

American academy of pediatrics 564


American Academy of Pediatrics PREP 2015

Item 183 S Preferred Response: A


Exertional chest pain is a warning sign for coronary artery disease. Although the cause may not
be cardiac, it needs to be ruled out with a careful history and physical examination, and when
there is any doubt, electrocardiogram (ECG). The patient in the vignette had chest pain that was
severe and episodic, but was not associated with shortness of breath or respiratory symptoms.
His symptoms improved with rest and worsened with peak exercise. He was very dizzy with the
chest pain as well. Until proven otherwise, the suspicion for a coronary artery anomaly would be
highest for this patient even before ECG is done.

Hypertrophic cardiomyopathy could cause exercise-induced chest pain from left ventricular
outflow tract obstruction and decreased coronary perfusion. If that were the case, one would
expect a loud outflow murmur. If the child had pleuritis, a rub would be expected on
examination, and there would be pain with changes in position. A spontaneous pneumothorax
would be associated with decreased breath sounds on the affected side and possibly mild
tachypnea. It would not likely be recurrent in nature.

The ECG in this patient shows the rare, but alarming findings of ST segment elevation in the
anterior and lateral leads (V1 through V6) with ST segment depression in the inferior leads
associated with cardiac ischemia. Ischemia occurs in pediatric patients with anomalous coronary
arteries in 2 age groups. The patient in the vignette is an adolescent athlete and has an anomalus
left coronary artery coming from his right coronary artery. In this condition, the left coronary
artery must go between the aorta and the pulmonary artery to get to the left side of the heart and
will be compressed when the patient is exercising, and the great vessels dilate with increase
cardiac output and exertion. Even though this is a rare condition, it accounts for 17% to 25% of
sudden death in young athletes. Only hypertrophic cardiomyopathy is more common in the
United States as a cause of sudden death in young athletes.

A different type of coronary artery anomaly presents in infancy: the anomalous left coronary
artery from the pulmonary artery. In this condition, the flow into the left coronary artery is from
the pulmonary artery. At birth, when the pulmonary vascular resistance is high, the flow is
toward the heart. When the pulmonary vascular resistance drops at approximately 1 month of
age, the flow reverses into the lungs, making the heart ischemic. Infants with this rare anomaly
will present with irritability with feedings, poor perfusion, or shock.

PREP Pearls
• Exertional chest pain is a warning sign for coronary artery disease.
• Chest pain with exercise raises suspicion for a cardiac etiology.
• One of the most dangerous types of chest pain with exertion is caused by an anomalous
coronary artery.
• Anomalous coronary artery accounts for 17% to 25% of sudden death in US young
athletes.

American Board of Pediatrics Content Specification(s)


• Recognize the cardiovascular and non-cardiovascular causes of chest pain in children of
various ages

American academy of pediatrics 565


American Academy of Pediatrics PREP 2015

Suggested Reading
• Bashore T, 0' Laughlin M. Coronary angiography in children with heart disease. In:
Garson A Jr, Bricker T, Fisher DJ, Neish SR, eds, The Science and Practice of Pediatric
Cardiology. 2nd ed. Philadelphia, PA: Lippincott Williams & Wilkins; 1998:1025-1026,
• Campbell RM, Berger S. Preventing pediatric sudden death: where do we start?
Pediatrics. 2006;118(2):802-804. doi:10.1542/peds.2006-0564.
• Ellison A, RiehIe TJ, Berger S, Campbell RM. Preventing—with the goal of
eradicating—sudden cardiac death in children. Contemp Pediatr. 2005;22(1037.
• Abnormal vascular connections and structures. In: Snider AR, Ritter SB, Serwer GA, eds.
Echocardiography in Pediatric Heart Disease. 2nd ed. St Louis, MO: Mosby; 1997:488-
490.

American academy of pediatrics 566


American Academy of Pediatrics PREP 2015

Item 184
You are meeting with the parents of a child who was diagnosed with leukemia. The parents have
researched the drugs the oncologist recommended to treat their child's leukemia and are
concerned about the potential adverse effects and how "toxic" they are to the body. They have
also researched alternative treatments for leukemia. You believe in incorporating the principles
of autonomy and beneficence into your patient care.

Of the following, the statement that BEST represents those principles is


A. alternative therapies to treat leukemia do not have the research to support their use
B. alternative treatments could be considered for use in conjunction with the standard
medical therapy
C. the cost of unproven treatments can take away resources from other patients
D. declining standard treatment for leukemia is a family decision that physicians should
respect
E. the risk of treatment with potentially harmful medications should be compared only with
the risk of no treatment

American academy of pediatrics 567


American Academy of Pediatrics PREP 2015

Item 184 I-C P S Preferred Response: B


The response that best represents autonomy and beneficence is that alternative treatments could
be considered for use in conjunction with the standard medical therapy. This approach
incorporates the principles of autonomy by allowing the family to make decisions about their
child's medical care and beneficence, ensuring that no harm will be done to the patient.
Leukemia is a life-threatening condition and conventional medicine has a high rate of cure. The
family in the vignette is exploring complementary and alternative medicine (CAM) treatment for
their child. It is important to discuss with the family the reasons why they are exploring these
options and to make an effort to understand the family's needs.

It is not uncommon for families with chronic or life-threatening conditions to explore alternative
therapies and to try unconventional therapies. Clinicians should be willing to discuss these
treatments with patients and their families. It is possible that some natural or alternative
treatments could be used along with conventional treatment. It is important that the clinician
open up a dialogue with the family to learn what types of treatments the family wants to include.
The patient should be monitored for side effects of CAM treatment.

Complementary and alternative medicine is defined as a group of diverse medical and health care
systems and practices that are not generally considered part of conventional medicine. CAM can
include acupuncture, homeopathy, naturopathy, meditation, prayer, yoga, biofeedback, hypnosis,
guided imagery, art therapy, music therapy, herbs, vitamins, nutritional therapy, massage,
chiropractic care, and biofield and bioelectromagnetic therapies. Several of these modalities are
now accepted as beneficial in conjunction with conventional medicine. It has been reported that
up to 40% of children use dietary supplements. In regional surveys of families that use CAM,
45% report using an herbal product. The widespread use of CAM makes it an important subject
for clinicians to address in patient interactions.

The clinician should always be guided by the ethical principles of autonomy, beneficence,
nonmaleficence, and justice. Autonomy is the right of the patient to make decisions about
personal medical care consistent with personal values. It is important that the families using
CAM be able to discuss this use with the physician and make informed decisions. Beneficence is
the duty of the physician to act in the best interest of the patient. The physician should listen to
each family's concerns and desire to use CAM, and recommend or not recommend the best
options for the patient. Nonmaleficence is the duty to do no harm. The physician should advise
the family against any CAM that can do harm or interfere with other treatments. Justice is
fairness, and the clinician should treat any conflicting interest of individuals in a fair manner.
The physician should work to ensure fair access to care for their patient.

Compared with conventional medicine, alternative treatments to treat leukemia do not have
evidence-based research data to support their use, but ignoring interest in CAM does not promote
autonomy for the patient and their family. Although the cost of unproven treatments may take
away resources from others and this could violate the principle of justice, the family's resources
and access to care should not be limited by this alone. In the case described in the vignette, the
physician should not accept the family's decision to decline treatment. This would go against the
principle of beneficence, because leukemia is a life-threatening condition with a standard of care

American academy of pediatrics 568


American Academy of Pediatrics PREP 2015

that has good outcomes. The ethical principle of nonmaleficence would guide against comparing
the risk of potentially harmful medications only with the risk of no treatment at all.

PREP Pearls
• Complementary and alternative medicine (CAM) can include acupuncture, homeopathy,
naturopathy, meditation, prayer, yoga, biofeedback, hypnosis, guided imagery, art
therapy, music therapy, herbs, vitamins, nutritional therapy, massage, chiropractic care,
and biofield and bioelectromagnetic therapies.
• Clinicians should provide an environment where families can openly discuss the use of
CAM.
• Respecting patient autonomy requires the clinician to consider the use of CAM.

American Board of Pediatrics Content Specification(s)


• Recognize and apply ethical principles regarding the use of complementary and
alternative medicine

Suggested Reading
• Gardiner P, Riley D. Herbs to homeopathy-medicinal products for children. Pediatr Chin
North Am. 2007;54(6):859-874. doi:10.1016/j. pc1.2007.10.005.
• Mears BJ. Ethics for the pediatrician: the ethics of complementary and alternative
medicine. Pediatr Rev. 2010;31(7):e49-e51. doi:10.1542/pir.31-7-e49.

American academy of pediatrics 569


American Academy of Pediatrics PREP 2015

Item 185
A 16-year-old adolescent girl presents to your office for a preseason physical examination. Her
mother reports that she is a competitive swimmer who participated in the state finals for the
butterfly stroke last year. The girl has no significant past medical history. In the office, her blood
pressure is 128/95 mm Hg. On physical examination, you note the girl is well muscled in her
arms and chest, and has small patches of dark hair on her upper chest and back.

Of the following, the substance MOST likely to cause the girl's physical examination findings is
A. creatine
B. ephedrine
C. human growth hormone
D. nandrolone
E. recombinant erythropoietin

American academy of pediatrics 570


American Academy of Pediatrics PREP 2015

Item 185 Preferred Response: D


While the girl in the vignette's muscle development could be the result of her training regimen,
her increased muscle mass could also be caused by the use of nandrolone, an anabolic steroid.
Hirsutism and hypertension are known adverse affects of anabolic steroids.

As increasing numbers of children and teenagers are participating in sports, the pressure to
succeed in athletics has increased. Young athletes may learn about the ergogenic benefits of
performance-enhancing drugs from media stories about their use by high-profile athletes in
professional sports, such as baseball and cycling. Teenagers are more prone to risk taking and
may overlook the potential adverse effects of performance-enhancing drugs. Athletes that
participate in sports that emphasize strength and a muscular physique are more likely to use
performance-enhancing drugs. Research studies demonstrate that a substantial number of young
athletes use ergogenic aids; for example, in a recent survey of male and female middle and high
school students, 6% of respondents admitted to anabolic steroid use.

Creatine is a widely available nutritional supplement that improves performance of short bursts
of high-intensity exercise. Phosphorylated creatine drives the production of adenosine
triphosphate from adenosine diphosphate, leading to increased energy stores for brief, explosive
physical activities. Muscle cramping and diarrhea are the most common adverse affects of
creatine use.

Ephedrine is a stimulant medication that was banned because of an increased risk of lethal
arrhythmias, seizures, and strokes. Ephedrine can cause hypertension, but would not cause
increased muscle definition or hirsutism.

Human growth hormone (HGH) supplementation does not improve sports performance, but can
increase lean body mass. Excessive HGH use leads to acromegaly; individuals with acromegaly
have an increased risk of diabetes and hypertension. Excess HGH does not result in hirsutism.
Recombinant erythropoietin increases red blood cell production; ergogenic benefits result from
increased oxygen-carrying capacity. Erythropoietin use can lead to polycythemia and subsequent
increased risk of hypercoagulability.

PREP Pearls
• Performance-enhancing drug use is not uncommon in middle school and high school
aged athletes.
• Athletes that participate in sports that emphasize strength and a muscular physique are
more likely to use performance-enhancing drugs.

American Board of Pediatrics Content Specification(s)


• Recognize the clinical findings associated with the use of performance-enhancing drugs
or nutritional supplements

Suggested Reading
• Dandoy C, Gereige RS. Performance-enhancing drugs. Pediatr Rev. 2012;33(6):265-271.
doi:10.1542/pir.33-6-265.

American academy of pediatrics 571


American Academy of Pediatrics PREP 2015

• Eisenberg ME, Wall M, Neumark-Sztainer D. Muscle-enhancing behaviors among


adolescent girls and boys. Pediatrics. 2012;130(6):1019-1026 doi:10.1542/peds.2012-
0095.

American academy of pediatrics 572


American Academy of Pediatrics PREP 2015

Item 186
A 15-year-old ranch worker from west Texas presents with a 6-month history of a chronic cough,
intermittent fevers, and progressive dyspnea on exertion. He initially had a flulike illness, but the
cough has persisted. He has been seen multiple times by his family doctor who prescribed
several courses of antibiotics without improvement. He presents to the emergency department
with chest pain and severe dyspnea.
On physical examination, he has a nontoxic appearance but is visibly dyspneic. His temperature
is 37°C, heart rate is 85 beats/min, respiratory rate is 24 breaths/min, and blood pressure is
115/70 mm Hg. Auscultation of his lungs shows decreased breath sounds on the right side. A
chest radiograph reveals complete collapse of the right lung, a large tension pneumothorax, and a
fluid level in the pleural space. The scout film from his chest computed tomography scan is
available (Item Q186).

Of the following, the MOST likely etiology of this patient's illness is


A. Coccidioides posadasii
B. Mycobacterium tuberculosis
C. Nocardia asteroides
D. Staphylococcus aureus
E. Streptococcus pneumoniae

American academy of pediatrics 573


American Academy of Pediatrics PREP 2015

Item 186 Preferred Response: A


The boy described in the vignette presents with a chronic, indolent pulmonary infection, most
likely caused by Coccidioides posadasii, found in West Texas. Coccidioides is a dimorphic
fungus found in the dry soil of areas with low rainfall in the southwest United States, northern
Mexico, and areas in Central and South America. Pulmonary disease occurs when the spore form
of the fungus is inhaled. Most children (60%) are asymptomatic or have a self-limited infection.
The most common symptoms are malaise, fever, cough, chest pain, headache, and myalgias
(Valley fever), which develop 1 to 3 weeks after inhalation of spores. Most cases resolve within
2 to 3 weeks, but fatigue and weight loss can persist for months. The most common chest
radiographic findings are a unilateral pulmonary infiltrate, hilar adenopathy, or a pleural
effusion. The chest radiograph obtained for the patient described in the vignette shows complete
collapse of the right lung, a large tension pneumothorax, and a fluid level in the pleural space,
consistent with chronic Coccidioides infection and sequelae. Pulmonary sequelae such as
nodules, cavities, or chronic fibrocavitary pneumonia are uncommon in children (< 5%).

Acute pulmonary infection caused by Coccidioides can present with only dermatologic
manifestations such as erythema nodosum, erythema multiforme, or an erythematous
maculopapular rash. Nonpulmonary acute infection is uncommon and usually associated with
trauma with direct inoculation of spores. Disseminated infection caused by Coccidioides is rare
(0.5% to 1%), usually occurring in immunocompromised individuals, and can involve the lungs,
central nervous system, skin, bones, and joints.

Pulmonary infection caused by Nocardia species also can be chronic in presentation, most
commonly occurring in immunocompromised children, such as those with chronic
granulomatous disease, organ transplantation, or human immunodeficiency virus infection.
Pulmonary disease caused by Mycobacterium tuberculosis can result in chronic infection, but
would likely result in progressive cough, fever, and weight loss. Chest radiography findings in
children with tuberculosis may demonstrate lymphadenopathy (hilar, mediastinal, subcarinal,
paratracheal), pulmonary infiltrates, atelectasis, pleural effusion, cavitary lesions, or miliary
disease, but chronic fibrocavitary pneumonia resulting in tension pneumothorax would be
unlikely. Pneumonia caused by both Streptococcus pneumoniae and Staphylococcus aureus
manifests as acute, not chronic disease.

PREP Pearls
• Coccidioides is a dimorphic fungus found in the dry soil of areas with low rainfall in the
southwest United States, northern Mexico, and areas in Central and South America.
• Most children (60%) with Coccidioides infection are asymptomatic or have a self-limited
infection.
• Acute pulmonary infection caused by Coccidioides can be associated with only
dermatologic manifestations such as erythema nodosum, erythema multiforme, or an
erythematous maculopapular rash.
• Disseminated infection caused by Coccidioides is rare (0.5% to 1%), usually occurring in
immunocompromised individuals, and can involve the lungs, central nervous system,
skin, bones, and joints.

American academy of pediatrics 574


American Academy of Pediatrics PREP 2015

American Board of Pediatrics Content Specification(s)


• Understand the epidemiology of Coccidioides
• Recognize the clinical features associated with Coccidioides infection

Suggested Reading
• American Academy of Pediatrics. Coccidioidomycosis. In: Pickering LK, Baker CI,
Kimberlin DW, Long SS, eds. Red Book: 2012 Report of the Committee on Infectious
Diseases. 29th ed. Elk Grove Village, IL: American Academy of Pediatrics; 2012:289-
291.
• American Academy of Pediatrics. Nocardiosis. In: Pickering LK, Baker CI, Kimberlin
DW, Long SS, eds. Red Book: 2012 Report of the Committee on Infectious Diseases.
29th ed. Elk Grove Village, IL: American Academy of Pediatrics; 2012:521-522.
• American Academy of Pediatrics. Tuberculosis. In: Pickering LK, Baker CJ, Kimberlin
DW, Long SS. eds. Red Book: 2012 Report of the Committee on Infectious Diseases.
29th ed. Elk Grove Village, IL: American Academy of Pediatrics; 2012:736-759.
• Montenegro BL, Arnold IC. North American dimorphic fungal infections in children.
Pediatr Rev. 2010;31(6):e40-e48. doi:10.1542/pir.31-6-e40.
• US Centers for Disease Control and Prevention. Coccidioidomycosis (Valley fever). US
Centers for Disease Control and Prevention website. Accessed February 25, 2014.

American academy of pediatrics 575


American Academy of Pediatrics PREP 2015

Item 187
A 16-year-old male football player presents to the emergency department with pain in his left
upper abdomen and left shoulder 30 min after a player on the opposing team forcefully
"rammed" his helmet into his left upper abdomen during a tackle. He did not lose consciousness
and sustained no trauma to his head. He denies any episodes of vomiting.
His temperature is 36.7°C, heart rate is 120 beats/min, respiratory rate is 22 breaths/min, and
blood pressure is 128/76 mm Hg. In general, he is alert but appears very uncomfortable. He is
lying on an examination table in a "fetal position”. On physical examination, the teenager's
abdomen is soft, but you note that he has marked tenderness to palpation with voluntary guarding
over the left upper quadrant. His bowel sounds are normal. He states that his left shoulder also
hurts, but there is no tenderness to palpation of his shoulder, which has full range of motion. The
remainder of the physical examination is unremarkable.
The results of a complete blood cell count are within reference range. Additional laboratory
studies are pending at this time.

Of the following, the diagnostic modality that would be MOST likely to reveal the diagnosis is
A. computed tomography of the abdomen
B. diagnostic abdominal laparoscopy
C. diagnostic peritoneal lavage
D. focused assessment with ultrasonography for trauma (FAST)
E. plain radiography of the abdomen

American academy of pediatrics 576


American Academy of Pediatrics PREP 2015

Item 187 TE Preferred Response: A


The adolescent boy in the vignette presents with focal tenderness and guarding over the left
upper quadrant of his abdomen, with referred pain in his left shoulder, after sustaining blunt
trauma to this region of his abdomen. Though he is tachycardic, his blood pressure is stable, and
his initial hemoglobin is within normal limits. Computed tomography (CT) of the abdomen
would be the most useful diagnostic modality in evaluating this patient, who, based on his
clinical findings, likely has a splenic injury.

Patients sustaining nonpenetrating (ie, blunt) trauma to the abdomen who have no hemodynamic
abnormalities or peritoneal signs should be evaluated with contrast-enhanced CT, with the
decision for operative management based on the specific organ(s) involved and the magnitude of
injury. Abdominal CT allows for rapid and precise identification of specific organ injuries and
their extent following trauma, and can also diagnose retroperitoneal and pelvic organ injuries that
are difficult to assess with a physical examination, focused assessment sonography in trauma
(FAST), and peritoneal lavage. Computed tomography is recommended for children with
suspected injuries to the liver, spleen, or kidney following an abdominal trauma, provided that
they are not hypotensive or displaying signs of peritonitis. In many cases, identification of intra-
abdominal injuries in hemodynamically stable children with CT can allow for nonoperative
management.

Relative contraindications to the use of CT include delay until the scanner is available, an
uncooperative child who cannot be safety sedated, and allergy to the contrast agent when
nonionic contrast is unavailable. When it is determined that an injured child will need to be
transferred to another facility, such as a pediatric trauma center, for definitive care, CT and other
time-consuming tests should generally not be performed at the transferring facility and left to the
receiving facility.

Diagnostic abdominal laparoscopy would not be the most useful diagnostic modality in this boy,
given that he is hemodynamically stable and most likely has a splenic injury, which can
potentially be managed nonoperatively. Operative management is indicated for those children
with frank peritoneal signs or hemodynamic instability following abdominal trauma that cannot
be normalized by fluid resuscitation. It is also indicated in most cases of penetrating abdominal
trauma. In these situations, however, abdominal laparotomy is the indicated procedure rather
than laparoscopy.

Although diagnostic peritoneal lavage (DPL) continues to be used by some surgeons to detect
intra-abdominal bleeding in children following trauma, CT is now considered the preferred
diagnostic study in most injured children, because of its higher sensitivity for detecting intra-
abdominal injuries as well as its noninvasiveness. Diagnostic peritoneal lavage may be useful in
detecting intra-abdominal bleeding in children with hemodynamic instability who are unstable
for transport to the CT scanner, or when CT and FAST are not available. In general, only the
pediatric surgeon who will care for the injured children should perform DPL, because this test
may interfere with future abdominal assessment, which may affect the decision on whether to
proceed with operative management.

American academy of pediatrics 577


American Academy of Pediatrics PREP 2015

Focused assessment sonography in trauma uses ultrasonography technology to determine the


presence of intraabdominal hemorrhage. Focused assessment sonography in trauma can be
performed rapidly at a patient's bedside while other evaluations and/or procedures are taking
place. It is noninvasive and can be repeated frequently. When performed by an experienced
technician, FAST can detect hemoperitoneum with good sensitivity and specificity; however, it
is operator-dependent and not consistently able to identify isolated intraparenchymal injuries,
which account for up to one-third of pediatric solid organ injuries. Although the use of FAST in
the evaluation of pediatric trauma victims is evolving, its usefulness in children remains
uncertain based on the available evidence. Focused assessment sonography in trauma could
certainly be used as an adjunct in evaluating patients such as the adolescent boy in this vignette,
but CT would still be the most useful diagnostic modality at this time.

Plain radiography of the abdomen might be useful in excluding the presence of free intra-
abdominal air in this patient, but it would not be useful in detecting or evaluating the extent of
solid organ injury. Computed tomography would be much more useful in evaluating this patient
with suspected splenic injury.

PREP Pearls
• Abdominal computed tomography (CT) allows for rapid and precise identification of
specific organ injuries and their extent following trauma.
• Computed tomography is recommended for children with suspected injuries to the liver,
spleen, or kidney following abdominal trauma, provided that they are not hypotensive or
displaying signs of peritonitis.
• When it is determined that an injured child will need to be transferred to another facility,
such as a pediatric trauma center, for definitive care, CT and other time-consuming tests
should generally not be performed at the initial facility.

American Board of Pediatrics Content Specification(s)


• Plan the appropriate evaluation of abdominal trauma

Suggested Reading
• American College of Surgeons Committee on Trauma. Abdominal and pelvic trauma.
Advanced Trauma Life Support for Doctors. 8th ed. Chicago, IL: American College of
Surgeons; 2008:111-124.
• American College of Surgeons Committee on Trauma. Pediatric trauma. Advanced
Trauma Life Support for Doctors. 8th ed, Chicago, IL: American College of Surgeons;
2008:225-240.
• Guralnick S, Serwint JR. Blunt abdominal trauma. Pediatr Rev. 2008;29(81:294-295.
doi:10.1542/pir.29-8-294.

American academy of pediatrics 578


American Academy of Pediatrics PREP 2015

Item 188
While rounding in the nursery, you are asked to assess a term newborn with intermittent jerking
of all 4 extremities. The newborn was born 48 hours ago at term by urgent cesarean delivery for
cord prolapse and fetal bradycardia. She required resuscitation at birth with bag-mask ventilation
for 30 seconds. Apgar scores assigned include 1 at one min (1 for heart rate) and 7 at five min (2
for heart rate, 2 for respiratory rate, 1 for tone, 1 for reflex irritability, and 1 for color). The cord
pH at the time of delivery was 7.18. She was initially monitored in the transitional care nursery
for 24 hours. She is now in the room with her mother and has been breast-feeding well. While
sound asleep, you note 1 to 5 rapid jerks of the right arm lasting several seconds, followed by
similar jerking in the right leg. It is not suppressed with gentle pressure or when the limbs are
repositioned. These movements resolve when the newborn awakens. The rest of the examination
is unremarkable.

Of the following, the jerking motions in this newborn are MOST likely
A. benign sleep myoclonus
B. focal clonic seizures
C. focal tonic seizures
D. jittery movements
E. myoclonic seizures

American academy of pediatrics 579


American Academy of Pediatrics PREP 2015

Item 188 Preferred Response: A


The intermittent jerking motions of the newborn in the vignette likely represent benign neonatal
sleep myoclonus. Benign neonatal sleep myoclonus presents in the first weeks after birth, with
myoclonic jerking of the extremities that occurs only during sleep and resolves upon awakening.
It is often confused with seizure activity. Electroencephalographic (EEG) monitoring will
demonstrate no epileptiform activity. No treatment is required and generally the condition
resolves spontaneously during infancy.

Normal repetitive movements such as benign neonatal sleep myoclonus, jittery movements, and
limb dorms (Item C188A) must be distinguished from seizure activity in the neonatal period.
Seizure activity is a clinical manifestation of neurologic compromise that requires immediate
diagnosis and treatment of the underlying cause.

Neonatal seizures are often categorized into 5 types: myoclonic, focal or multifocal clonic, focal
tonic, generalized, and subtle. Differing from benign neonatal sleep myoclonus, myoclonic
seizures are quick, single jerks of an extremity or trunk that persist when the neonate is awake
and are often associated with metabolic or genetic conditions (Item C188B). Normal repetitive
movements may also be confused with focal or multifocal clonic seizures, which present as slow
rhythmic jerking of an extremity or twitching of the face (Item C188C). Focal tonic seizures
manifest with slowly evolving stiffening and posturing of an extremity often accompanied by
eye deviation and head turning (Item C188D). After a significant brain injury, generalized
seizures with trunk stiffening accompanied by extremity extension or flexion may be seen.
Subtle seizure requires a high level of clinical suspicion, because the presentation is varied and
includes eye deviation, chewing motions, tongue thrusting, and bicycling.

Although the newborn in the vignette is at risk for the development of seizures in the first 24
hours after birth related to her birth depression secondary to the umbilical cord prolapse, the
jerking motions observed more than 48 hours after birth are most likely benign neonatal sleep
myoclonus. Jittery movements involve more than 1 extremity at a time, are present when the
neonate is awake, and resolve with sleep. The jerking motions are not consistent with focal
clonic seizures, focal tonic seizures, or myoclonic seizures. If the clinician is not confident in the
diagnosis of benign neonatal sleep myoclonus, an EEG may be performed to rule out
epileptiform activity.

PREP Pearls
• Benign neonatal sleep myoclonus presents in the first weeks after birth with myoclonic
jerking of the extremities that occurs only during sleep and resolves upon awakening.
• Normal repetitive movements such as benign neonatal sleep myoclonus, jittery
movements, and limb clonus must be distinguished from seizure activity in the neonatal
period.
• If the clinician is not confident of the diagnosis of benign neonatal sleep myoclonus, an
electroencephalogram may be performed to rule out epileptiform activity.

American Board of Pediatrics Content Specification(s)


• Differentiate between normal and abnormal repetitive movements during infancy

American academy of pediatrics 580


American Academy of Pediatrics PREP 2015

Suggested Reading
• Egger J, Grossmann G, Auchterlonie IA. Benign sleep myoclonus in infancy mistaken for
epilepsy. BM]. 2003;326:975-476. doi:10.1136/ brM.326.7396.975.
• Gillam-Krakauer M, Carter BS. Neonatal hypoxia and seizures. Pediatr Rev.
2012;33:387-397. doi:10.1542/pir.33-9-387.
• Jensen FE. Neonatal seizures: an update on mechanisms and management. Gin Perinatal.
2009;36(0881-900. doi:10.1016/j.clp.2009.08.001.
• Maurer VO, Rizzi M, Bianchetti MG, Ramelli GP. Benign neonatal sleep myoclonus: a
review of the literature. Pediatrics. 2010;125(4):e919-e924. doi:10.1542/peds.2009-1839.
• Olsen DM. Neonatal seizures. NeoReviews. 2012;13(4):e213-e223. doi:10.1542/neo.13-
4-e213.

American academy of pediatrics 581


American Academy of Pediatrics PREP 2015

Item 189
A 4-month-old male infant comes to your office for a scheduled health supervision visit. He was
born at term, has been growing and developing appropriately, and has had no medical problems.
His father has noticed that the infant sometimes looks "cross-eyed." On ophthalmological
examination, a red reflex is present bilaterally and the corneal light reflex is central bilaterally.
The infant fixes on your face and tracks in all directions without ocular misalignment. The
remainder of his physical examination is unremarkable.

Of the following, the BEST recommendation for this infant is


A. follow-up at the 6-month health maintenance visit
B. magnetic resonance imaging of the brain
C. pediatric neurology referral
D. pediatric ophthalmology referral
E. pediatric vision therapy referral

American academy of pediatrics 582


American Academy of Pediatrics PREP 2015

Item 189 Preferred Response: D


The infant in the vignette has intermittent strabismus. This can be normal in early infancy, but if
it has not resolved by 4 months of age, the infant should be referred to a pediatric
ophthalmologist. Strabismus refers to any kind of ocular malalignment. Strabismus can be
caused by an outward deviation of one eye, "exotropia” an inward deviation of 1 eye,
"esotropia," or an upward deviation of 1 eye, "hypertropia:' When the malalignment is constant,
it is called a "tropia" as in "exotropia; and when it is only seen with breakdown of binocular
fusion, it is called a "phone as in "exophoria." In the clinic, this can be diagnosed using the
alternate cover test. In this test, the child focuses on an object a few feet away. The examiner
covers first 1 eye and then the other eye, without letting binocular fixation occur in between. If 1
of the eyes moves, then the child has either an intermittent tropia or a phoria.

Pseudostrabismus is the appearance of ocular malalignment caused by various facial features,


such as a broad nasal bridge or eye folds appearing to cover parts of the eye, making the eyes
appear to be crossed while they are appropriately aligned (Item 189A). If there is any uncertainty
or concern about the physical examination findings, referral to a pediatric ophthalmologist is
appropriate.

Vision screening is recommended at all ages. Early diagnosis and treatment of strabismus and
other ocular disorders can prevent amblyopia, which is permanent vision impairment or vision
loss. Constant strabismus should be referred to a pediatric ophthalmologist at any age.
Intermittent strabismus, noted on physical examination or by parental report, in infants 4 months
and older should also be referred to the ophthalmologist. Other screening and referral
recommendations are listed in Item C189B, page C-161. Evaluation of eye alignment and
assessment of the red reflex is essential at all health supervision visits.

The infant in the vignette is 4 months old and has intermittent strabismus, therefore he should be
referred to a pediatric ophthalmologist now and not wait until reevaluation at the 6-month health
supervision visit. Magnetic resonance imaging of the brain and pediatric neurology referral are
not needed for a normally developing child, unless neurologic abnormalities are noted by the
ophthalmologist. Vision therapy leg, patching) is not the first step in the evaluation of
strabismus, though it may be recommended later by the ophthalmologist.

American academy of pediatrics 583


American Academy of Pediatrics PREP 2015

American academy of pediatrics 584


American Academy of Pediatrics PREP 2015

PREP Pearls
• Constant strabismus should be referred to a pediatric ophthalmologist at any age.
• Intermittent strabismus, noted on physical examination or by parental report, should be
referred to a pediatric ophthalmologist if present in infants 4 months and older.

American Board of Pediatrics Content Specification(s)


• Evaluate a patient for ocular tropias and phorias
• Plan the appropriate evaluation of strabismus, including timing of evaluation to prevent
complications
• Differentiate the clinical findings associated with strabismus from those of
pseudostrabismus

Suggested Reading
 Rogers GL, Jordan CO. Pediatric vision screening. Pediatr Rev. 2013;34(3):126-133.
doi:10.1542/pir.34-3-126.

American academy of pediatrics 585


American Academy of Pediatrics PREP 2015

Item 190
A 14-year-old adolescent girl presents for evaluation of short stature. Her height is at the fifth
percentile. Her parents noticed that her growth rate slowed significantly over the last year, and
they are concerned that she will be much shorter than her siblings, all of whom reached the 25th
percentile in height. The girl reports that she had her first menstrual period 1 month ago. You
inform the family about the average height gain after menarche.

Of the following, the MOST accurate prediction for height gain after reaching this stage of
puberty is
A. 1 cm
B. 4.5 cm
C. 7.5 cm
D. 10 cm
E. 13 cm

American academy of pediatrics 586


American Academy of Pediatrics PREP 2015

Item 190 Preferred Response: C


Just before puberty, linear growth rates (growth velocity) gradually decelerate. Then, at the start
of puberty, at approximately 9 years of age, growth begins to accelerate as serum gonadotropins
(luteinizing hormone, follicle-stimulating hormone) gradually increase, but the exact timing of
this can be variable. Luteinizing hormone levels increase 23-fold from prepuberty to late
puberty. In girls, sex hormone levels increase as a consequence of ovarian and adrenal
maturation, and physiologic leukorrhea (vaginal discharge) may occur secondary to rising
estrogen stimulation. Production of sex hormones at puberty (both estrogen and testosterone)
stimulates growth rates, with the pubertal growth spurt for girls beginning at sexual maturity
rating 2. Peak height velocity of approximately 8 to 9 cm/year is achieved for girls at sexual
maturity rating 3, about 1 year before menarche. After menarche, only about 7.5 cm of growth
remains, and growth is 99% complete at a bone age of 15. It is expected that the child in this
vignette, who is now postmenarchal, would only grow a small amount after menarche.
The pubertal growth spurt in boys begins an average of 2 years later than in girls. A peak height
velocity of 9.5 to 10 cm/year is achieved by sexual maturity rating 4. Growth is 99% complete at
a bone age of 17. The combination of a longer period of prepubertal growth and a greater
pubertal height velocity results in the average adult height difference of 13 cm between men and
women.

Additional physiological changes occur during puberty. Estrogens promote lipogenesis and
redistribution of body fat toward the adult female body habitus. In contrast, androgens are
lipolytic and promote muscular development. Thus, the increase in body mass index that occurs
in both boys and girls during pubertal development is because of differences in body
composition, with a higher percentage being body fat in girls and lean body mass in boys.

PREP Pearls
• Girls achieve an average peak growth velocity of 8 to 9 cm/year by sexual maturity rating
3. Boys' growth velocity peaks at an average of 10 cm/year by sexual maturity rating 4.
• After menarche, growth slows down rapidly, with only about 7.5 cm of linear growth
potential remaining.

American Board of Pediatrics Content Specification(s)


• Recognize the physiologic changes that commonly precede menarche

Suggested Reading
• Bordini B, Rosenfield RL. Normal pubertal development: Part I: The endocrine basis of
puberty. Pediarr Rev_ 2011;32(6):223-229. doi:10.1542/ pir.32-6-223.
• Bordini B, Rosenfield RL. Normal pubertal development: Part II: clinical aspects of
puberty. Pediatr Rev. 2011;32(7):281-292. doi:10.1542/pir.32-7-281.
• Roche AF. The final phase of growth in stature. Growth, Genetics, and Hormones.
1989;5(4):4-6.
• Rogol AD, Roemmich JN, Clark PA. Growth at puberty. JAdolesc Health. 2002;31(6
suppl):192-200. doi:10.1016/51054-139X(02)00485-8.

American academy of pediatrics 587


American Academy of Pediatrics PREP 2015

Item 191
You are seeing an infant in your office for her 2-month health supervision visit. The parents are
concerned about a red bump that first appeared on her face at 2 weeks of age. The lesion has
since grown to cover a large portion of her cheek, chin, and infraorbital area (Item Q191). The
infant is growing and developing normally, and her physical examination findings are otherwise
unremarkable.

Of the following, the BEST treatment option for this lesion is


A. intralesional corticosteroid injection
B. observation
C. oral propranolol
D. parenteral α-interferon
E. surgical excision

American academy of pediatrics 588


American Academy of Pediatrics PREP 2015

Item 191 Preferred Response: C


The infant in the vignette has a segmental infantile hemangioma (IH) in a location that could
impair function by obstructing visual development. Therefore, early treatment is advised. Recent
studies indicate that oral propranolol is an effective treatment for IH, causing softening and
regression of the lesion. Although no controlled trials are available, a multicenter retrospective
chart review found that propranolol was more effective than oral steroids at treating this type of
lesion. A recent consensus conference urged a conservative approach to initiating propranolol for
IH, with particular caution regarding the potential cardiovascular, pulmonary, and hypoglycemic
effects of the medication. Like all other modalities used for IH treatment, propranolol does not
have US Food and Drug Administration (FDA) approval for use for this indication.

Item C191A. Table 1: High-risk Hemangiomas


Clinical Scenario Potential Sequelae
Potentially disfiguring sites Permanent distortion of anatomic landmarks,
 Nasal tip including fibrofatty residua, anetoderma, textural
 Perioral changes, and hyperpigmentation
 Glabella
 Ear Residual telangiectasias
 Eyebrow
Central facial location, particularly those medial to High risk of ulceration, especially perioral and neck
the outer canthi and >0.5 cm in size fold hemangiomas
Superficial thick or exophytic hemangiomas, in areas
not easily covered by clothing
Astigmatism or visual axis obstruction potentially
Periocular hemangiomas
resulting in permanent amblyopia, strabismus
Facial hemangiomas >5 cm in diameter or segmental PHACE syndrome (posterior fossa anomalies,
facial hemangiomas hemangioma, arterial lesions, cardiac
abnormalities/coarctation of the aorta, eye
Lumbosacral or perineal hemangiomas abnormalities)

Ulceration, scarring, residual skin changes


 High risk of ulceration
 Tethered spinal cord
 Lipomyelomeningocele
 Genitourinary abnormalities
Airway hemangioma ± "beard area" skin Life-threatening airway obstruction, usually between
hemangiomas 4-6 wk of lif
Multifocal (≥5) infantile hemangiomas Hepatic hemangiomas: can be asymptomatic or
result in abdominal compartment syndrome, heart
failure, and hypothyroidism
Adapted from and reprinted with permission from Toliefson MM, frieden IL Early growth of infantile
hemangiomas: what parents' photographs tell us. Pediatrics. 2012;130(2);e314-e320.

American academy of pediatrics 589


American Academy of Pediatrics PREP 2015

Item C191B. Treatment Options for Infantile Hemangioma


Treatment Comments
Topical
• Corticosteroids Clobetasol for small periorbital lesions, but
risk of glaucoma, cataracts

• Timolol Limited data, ophthalmologic


preparation available
Oral
• Corticosteroids Concern for systemic side effects

• Propranolol Monitor for arrhythmias, hypotension,


bronchospasm, hypoglycemia
Vincristine Vesicant, neurotoxicity, constipation
• Alpha interferon Blood, skin, neurologic and hepatic
sequelae
Intralesional corticosteroids Used for localized lesions
Procedural
• Pulsed dye laser May be useful for ulcerating lesions and to
decrease redness
• Surgery For large lesions in locations without
cosmetic concerns
Multimodal therapy Systemic agents and procedure

Infantile hemangiomas are vascular tumors of benign endothelial-like cells exhibiting specific
histochemical markers that differentiate them from other vascular birthmarks. These tumors
undergo rapid proliferation during the first year of life, usually reaching their maximum size by 6
to 9 months of age. Recent data indicate that the majority of growth occurs during the first 8
weeks after birth, with an accelerated growth phase between 5.5 and 7.5 weeks. This rapid
growth is followed by slow involution over years. Infantile hemangioma is seen more commonly
in girls, non-Hispanic whites, premature infants, children who are products of multiple gestation,
and children of mothers of advanced age. Lesions may be small and localized, or larger and
segmental. Some IHs are considered high risk because of ulceration, bleeding, functional
impairment (particularly involving vision or airway), congestive heart failure, or disfigurement
(Item C191A). While it has traditionally been taught that these lesions involute without sequelae,
nearly 70% of untreated infants in one study had residual skin changes after involution,
suggesting that treatment might be indicated for more infants. While most IHs are not noted at
birth, photographic evidence indicates that two-thirds of patients had a precursor skin lesion
visible at birth, although many of these could not be differentiated from other typical benign
neonatal lesions.

American academy of pediatrics 590


American Academy of Pediatrics PREP 2015

Rational treatment of IH is complicated by the fact that there are no FDA-approved treatment
modalities, inconsistent definitions of high-risk hemangiomas, evidence usually based on case
studies rather than controlled trials, and no standardized approach to the use of the various
agents. Possible treatments include topical or oral agents as well as the procedures described in
Item C191B. Intralesional corticosteroids are often used for focal hemangiomas, but would not
be appropriate for the large segmental lesion seen in the child in this vignette. There are case
reports of successful treatment of IH with interferon, but serious side effects (blood
abnormalities, spastic diplegia) limit its use. Surgery generally is reserved for lesions that do not
respond to less invasive therapy or that present an immediate health risk, as well as those in
noncosmetically important areas. Watchful waiting is not a good option for this child because of
the high risk of functional impairment.

PREP Pearls
• Infantile hemangiomas are benign tumors of endothelial origin that are generally noted
shortly after birth.
• Infantile hemangiomas typically undergo rapid growth in the first few weeks to months
of life followed by slow involution.
• Hemangiomas at risk for ulceration, bleeding, functional impairment, or significant
disfigurement should be referred early for specialist treatment.
• Propranolol has shown good efficacy in the treatment of infantile hemangioma, but it is
currently not approved for this use by the US Food and Drug Administration.

American Board of Pediatrics Content Specification(s)


• Understand the natural history of strawberry hemangiomas

Suggested Reading
• Chen TS, Eichenfield LF, Friedlander SF. Infantile hemangiomas: an update on
pathogenesis and therapy. Pediatrics. 2013;131(I):99-108. doi:10.1542/peds.2012-1128.
• Drolet BA, Frommelt PC, Chamlin SL, et al. Initiation and use of propranolol for
infantile hemangioma: report of a consensus conference. Pediatrics. 2013;131(1):128-
140. dok10.1542/peds.2012-1691.
• Toilefson MM, Frieden I). Early growth of infantile hemangiomas: what parents'
photographs tell us. Pediatrics. 2012:130(2);e314-e320. doi:10.1542Ipeds.20I1-3683.

American academy of pediatrics 591


American Academy of Pediatrics PREP 2015

Item 192
You were awarded a 1-year intramural grant for research and wish to conduct a study to
investigate the role of intrauterine devices as a risk factor for pelvic inflammatory disease. You
must decide whether to do a case-control or a cohort study. After comparing the advantages of
each type of study, you determine that a case-control study would be the better option.

Of the following, the BIGGEST advantage of this type of study design for this research project is
that it
A. allows for calculation of rates of disease in exposed and unexposed
B. allows for study of multiple potential risk factors
C. is well suited for common conditions
D. is well suited for conditions with a short latency
E. relies on recall or records of past events

American academy of pediatrics 592


American Academy of Pediatrics PREP 2015

Item192 PBLI Preferred Response: B


Case-control studies are used to determine how much more (or less) likely it is that cases with a
disorder have been exposed to the factors being studied than were the controls without the
disorder. An advantage of a case-control study design is that it allows for the study of multiple
risk factors, is relatively quick to conduct, is relatively inexpensive, and requires fewer subjects
than a cohort study. It would fit the needs of the project proposed in the vignette, allowing one to
identify cases of pelvic inflammatory disease (RID) diagnosed during a specified time frame and
to evaluate the subjects for several possible risk factors. Case-control design is also well suited
for the study of rare diseases and conditions with a long latency period. Ideally, one should
identify and enroll all incident cases in a population during a specified period, defining cases
using standard diagnostic criteria that are objective rather than subjective, and noting the severity
of the disease being studied. This study design has disadvantages that include the need to recall
all past exposures being studied, difficulty with controlling for extraneous variables, and the
inability to determine rates of disease in exposed and unexposed subjects. Rates of disease
cannot be calculated in case-control, as this is a retrospective study and the population at risk is
not known.

Cohort studies (prospective or retrospective) involve comparison of a subject group with a


known exposure to that of a control group free of exposure. These studies take a long time to
conduct, are relatively expensive, and loss to follow-up can be a problem. In addition, if the
exposure status of the subjects or controls changes, it results in "contamination!' Cohort study
design is not well suited to conditions with a long latency and for rare conditions. However, the
time interval between the exposure and outcome is clear, and it allows for rates of disease in
exposed and unexposed individuals to be determined. It also allows for study of multiple
outcomes. Prospective cohort design is useful for the study of rare exposures and reduces the risk
of survivor bias, as subjects are followed prospectively after an exposure. A cohort design for the
study described in this vignette would entail gathering data on girls in this practice who have
intrauterine devices in place, which would likely be a very small number, and then following
them over time to determine how many develop PID within the year of the study. Therefore, in
this situation, this type of study would likely yield a limited amount of useful information.

PREP Pearls
• Case-control studies are used to determine how much more (or less) likely it is that case
subjects were exposed to the factors being studied than were controls.
• Cohort studies involve comparison of a group with a specific exposure to that of a control
group free of exposure.

American Board of Pediatrics Content Specification(s)


• Understand the uses and limitations of cohort studies
. Understand the uses and limitations of case-control studies

Suggested Reading
• Jennings 1M, Sibinga E. Research and statistics: understanding and identifying bias in
research studies. Pediatr Rev. 2010;31(0161-162. doi:10.1542/pir.31-4-161.
• Johnson 51.. Research and statistics: a question of time: cross-sectional versus
longitudinal study designs. Pediatr Rev. 2010;31(0:250-251. doi:10.1542/pir.31-6-250.
American academy of pediatrics 593
American Academy of Pediatrics PREP 2015

• Perry-Parrish C, Dodge R. Research and statistics: validity hierarchy for study design and
study type. Pediatr Rev. 2010;31(6):27-29. doi:10.1542/ pir.31-1-27.
• Upadhya K, Rowe P. Research and statistics: case-control studies. Pediatr Rev. 2010;
31(6):70-71. doi:10.1542/pir.31-2-70.

American academy of pediatrics 594


American Academy of Pediatrics PREP 2015

Item 193
A 14-year-old adolescent boy presents to your office for routine follow-up of his severe
persistent asthma. He has had asthma since 4 years of age and is currently well-controlled for the
past month on 2 puffs of daily mometasone 220 µg once a day and montelukast at bedtime. He
also notes that he has symptoms of allergic rhinitis and that his asthma gets triggered by multiple
allergic triggers. He has had approximately 3 courses of oral corticosteroids each year until he
was switched to the current regimen 2 years ago. His parents are concerned that he is only 5 ft, 2
inches in height. The father's height is 6 ft, 1 in and the mother's height is 5 ft, 4 in. His physical
examination reveals sexual maturity rating 3 pubic and axillary hair. The parents ask what can be
done to ensure maximal growth, while maintaining his current degree of asthma control.

Of the following, the BEST next step would be to


A. discontinue mometasone; leave him on montelukast alone
B. refer to an allergist for evaluation of appropriateness of allergen
C. refer to an allergist for evaluation of appropriateness of anti-immunoglobulin E therapy
D. refer to an endocrinologist for evaluation of growth
E. switch to combination of lower-dose inhaled steroid and long-acting (3-agonist therapy

American academy of pediatrics 595


American Academy of Pediatrics PREP 2015

Item 193 Preferred Response: D


The adolescent in the vignette should undergo an assessment by an endocrinologist regarding his
growth.

The National Institutes of Health guidelines for asthma recommend daily inhaled corticosteroid
therapy (ICS) as the medication of choice in all patients with persistent asthma (step 2 or higher).
In general, ICSs are efficacious, well-tolerated, and safe at the recommended dosages. The dose-
response curve for ICS treatment begins to flatten for many measures of efficacy at low to
medium doses. While most benefit is achieved with relatively low doses, the risk of adverse
effects increases with the dose of ICS, and even more importantly, with cumulative total
corticosteroid dose (both systemic and inhaled). High doses of ICS (more than 1,000 µg/day in
children) are more likely to have systemic and local adverse effects. Additionally, patients who
receive high ICS doses are more likely to receive systemic corticosteroid doses, further
increasing the risk of adverse effects. However, when used in the approved therapeutic dose
ranges, the most commonly encountered adverse effects are local thrush and dysphonia caused
by mucosal irritation, local immunosuppression, and vocal cord myopathy, respectively. These
local adverse effects can be minimized by use of a spacer with a metered-dose inhaler (MDT)
ICS and by "swishing and spitting" after ICS use.

The bigger concern with long-term use of ICS is growth delay, with a recent study suggesting a
decrease in height of up to half an inch compared to placebo, These children had mild-moderate
asthma and were on therapy with inhaled budesonide (approximately 400 pg/day) for a total of
about 4 to 6 years. Their height was evaluated about 5 years after the study was stopped and
compared to the nedocromil (16 mg) or placebo arm. The adolescent in the vignette was on
multiple courses of systemic corticosteroids, raising the possibility of growth suppression as an
adverse effect. In addition, he has had poorly controlled chronic severe persistent asthma in the
past. Poorly controlled asthma is also linked to poor growth.

This adolescent has finally achieved good control of his asthma on a moderate to high dose of
ICS (mometasone). This dose will likely enable minimization of risk of exacerbations and need
for systemic corticosteroids, while facilitating lung growth and prevention of loss of lung
function. Mometasone furoate, similar to newer ICS, such as fluticasone propionate and
ciclesonide, has greater anti-inflammatory potency, but less potential for adverse effects, owing
to its low systemic bioavailability and extensive hepatic first-pass metabolism. Montelukast, a
leukotriene antagonist as a single agent, is an alternative option in mild persistent asthma (step
2), but would not be recommended in moderate-to-severe persistent asthma as a stand-alone
controller agent. Discontinuation of mometasone may therefore result in deterioration of asthma
control. Consideration of allergen immunotherapy and antiimmunoglobulin E therapy to
modulate the allergic inflammatory cascade would be a reasonable add-on option to his current
regimen, and may even help lower the maintenance dose of ICS in the future ("steroid-sparing").
Similarly, switching to a low-dose ICS-long-acting β-agonist (LABA) combination therapy
might be another "steroid-sparing option once his asthma control is stable on this regimen,
particularly if slowing of his height velocity is noted in his endocrinology evaluation and growth
delay is determined. The US Food and Drug Administration currently has a black box warning
label reinforcing the appropriate use of LABAs in the management of asthma. Specifically,
LABA products should not be initiated as first-line or sole therapy, used with worsening
American academy of pediatrics 596
American Academy of Pediatrics PREP 2015

symptoms, or for acute control of bronchospasm. Long-acting β-adrenoceptor agonist therapy


should be stopped once asthma control is achieved and should be maintained with the use of an
asthma controller agent such as ICS.

Other potential adverse effects of cumulative corticosteroid doses include osteoporosis,


osteopenia, and in the older population, ocular problems such as cataracts and glaucoma. The
choice of ICS preparation prescribed depends on the desired level of symptom control in
individual patients balanced by the potential for adverse effects. Issues such as the drug delivery
device, dose level, formulation of the preparation, bioavailability, potency of the ICS, and
deposition, either in the pulmonary system or in the gastrointestinal system need to be kept in
mind. Strategies to minimize adverse effects include using the lowest dose needed to control
asthma, use of less bioavailable formulations, use of spacers or holding chambers with MDI,
rinsing the mouth after inhalation, adding LABA to ICS rather than increasing ICS (if indicated),
monitoring growth, and ensuring age-appropriate dietary intake of calcium and exercise.

PREP Pearls
• The most commonly encountered adverse effects of inhaled corticosteroid therapy (ICS)
are local thrush and dysphonia when used in approved therapeutic doses. These adverse
effects can be minimized by use of a spacer with a metered dose inhaler ICS and by
'swishing and spitting" after ICS use.
• Long-term use of ICS in prepubertal children may be associated with a decrease in height
of up to half an inch; monitoring of growth should be a part of the asthma evaluation.
• The risk of adverse effects is directly related to the cumulative total corticosteroid dose
(both systemic and inhaled); use of ICS at optimal doses that will facilitate attainment of
asthma control, minimize risk of exacerbations, and require systemic corticosteroids is
recommended.

American Board of Pediatrics Content Specification(s)


• Recognize the adverse effects associated with long-term corticosteroid therapy. including
inhaled corticosteroid therapy in patients with asthma, and manage appropriately

Suggested Reading
• Kelly HW, Sternberg AL, Lescher R, et al. Effect of inhaled glucocorticoids in childhood
on adult height. N Engl J Med. 2012;367(10):904-9l2. doi:10.1056/NEJMoa1203229.
• Kramer TM. Balancing the benefits and risks of inhaled long-acting betaagon ists—the
influence of values. N Erre J Med. 2009;360(16):1592-1595. don
10.1056/NEM40810561.
• National Heart, Lung, and Blood institute. National asthma education and prevention
program expert panel report III: guidelines for the diagnosis and management of asthma.
National institutes of Health, US Department of Health and Human Services website.
• United States Food and Drug Administration. FDA drug safety communication: new
safety requirements for long-acting inhaled asthma medications called Long-Acting Beta-
Agonists (LABAs). US Food and Drug Administration website.

American academy of pediatrics 597


American Academy of Pediatrics PREP 2015

Item 194
You are evaluating an 8-year-old girl who presents to your clinic for routine follow-up after a
hospital admission for pneumonia. This was the fourth admission in the past year for left lower
lobe pneumonia. She also has had a chronic cough that has been slowly worsening. The episodes
of pneumonia do not coincide with upper respiratory infections. The cough occurs during all
hours of the day and is not associated with meals or activities. She does not have any other
medical or developmental problems. Her growth parameters are normal, and she is currently
doing well in third grade. Between episodes of pneumonia, she has not had any difficulty
keeping up with peers, and she participates in sports without significant effort-intolerance or
wheezing.
On physical examination, she is well developed and well nourished. Pulse oximetry shows
oxygen saturation of 99% on room air. She has moist and nonerythematous mucous membranes.
Tympanic membranes are clear. She is breathing comfortably and has decreased breath sounds
and dullness to percussion at the left base. No crackles or wheezing. There are no notable
thoracic deformities. Cardiovascular examination shows a regular rate and rhythm, with warm
and well-perfused extremities. No peripheral cyanosis, clubbing, or edema are detected.
Abdomen is soft, nontender, and nondistended.

Of the following, the test MOST likely to reveal a diagnosis is


A. barium swallow
B. chest computed tomography scan
C. echocardiogram
D. sweat chloride test
E. ventilation-perfusion scan

American academy of pediatrics 598


American Academy of Pediatrics PREP 2015

Item 194 Preferred Response: B


The child in this vignette likely has a congenital malformation of the lung, indicated by the
chronic cough and recurrent pneumonia. The best diagnostic test for this condition is high-
resolution computed tomography (CT) of the chest with contrast.

Congenital malformations of the lungs are relatively rare, but must be considered in the
differential diagnosis of an otherwise healthy child with cough or recurrent infection. Types of
developmental anomalies of the lungs include bronchogenic cysts, pulmonary cysts, congenital
cystic adenomatoid malformations (CCAMs) (Item C194), pulmonary sequestration, and
congenital lobar emphysema. Of these, pulmonary sequestrations and CCAMs are the most
common, and result from abnormal embryologic development. A pulmonary sequestration is a
mass of tissue with all pulmonary histologic components and its own systemic blood supply, but
without connection to the tracheobronchial tree. In contrast, CCAMs do communicate with the
tracheobronchial tree, and the blood supply is from the bronchial circulation. Histologically,
CCAMs have growth of terminal bronchioles and elastic tissue, but do not include cartilage or
alveoli.

Pulmonary conditions can be broadly divided into diseases of the airway, parenchyma, pleura,
pulmonary vascular, and cardiovascular. Imaging modalities to work up these conditions include
ultrasonography, magnetic resonance imaging (MRI), ventilation-perfusion scanning, plain
radiography, diagnostic laryngoscopy and bronchoscopy, and CT. Depending on the level of
airway obstruction, diagnostic laryngoscopy and bronchoscopy can be helpful in identifying
areas of narrowing, weakness, or extrinsic compression. In extrinsic airway compression, MRI,
echocardiography, or barium swallow can help identify the source of compression, such as in a
vascular ring. Pulmonary parenchymal pathology, such as pneumonia, abscess, and
developmental anomalies, can be initially evaluated with simple radiography, but CT or MRI
will more likely yield the diagnosis. Ultrasonography is helpful in identifying pleural effusions,
and is a simple, noninvasive test. Ventilation-perfusion scanning can be a useful test if
pulmonary embolism is suspected, but has more recently fallen out of favor, replaced by rapid
sequence CT with contrast or MRI. Lastly, echocardiography can be considered if heart disease
is suspected because it is a common cause of respiratory symptoms.

Echocardiography can also be helpful in diagnosing heart disease or vascular ring, but these
conditions are unlikely given the lack of airway obstruction and seemingly normal
hemodynamics. Cystic fibrosis can be ruled out with a sweat chloride test, but that diagnosis is
unlikely, given the lack of crackles, wheezing, growth failure, or clubbing. Pulmonary embolism
is unlikely in this well-appearing child with no tachypnea or hypoxia, so a ventilation-perfusion
scan is not indicated. When evaluating pulmonary diseases, clinically localizing area of lesion to
airway, parenchyma, vasculature, or pleura can help in selecting the appropriate diagnostic
modality. In pulmonary parenchymal diseases, plain radiography is a good initial modality, but
CT or MRI is more likely to reveal a definitive diagnosis.

American academy of pediatrics 599


American Academy of Pediatrics PREP 2015

PREP Pearls
• Congenital cystic adenomatoid malformations and pulmonary sequestrations are the most
common developmental anomalies of the lungs.
• Computed tomography and magnetic resonance imaging are the most sensitive and
specific tests for pulmonary parenchymal lesions.
• Diagnostic laryngoscopy and bronchoscopy are useful diagnostic modalities for upper
and large airway pathology.

American Board of Pediatrics Content Specification(s)


• Understand the usefulness of various imaging studies in identifying pulmonary diseases

Suggested Reading
• American Academy of Pediatrics. Index of suspicion. Pediatr Rev. 2002;23(0:283-292.
doi:10.1542/pir.23-2-61.
• Finder JD. Cystic adenomatoid malformation and pulmonary sequestration. In: Kliegman
RM, Stanton BF, St. Game JW III, Schor NF, Hehrman RE eds. Nelson's Textbook of
Pediatrics. 19th ed. Philadelphia, PA: Saunders Elsevier: 201i:1464-1466.
• Scouten WT, White J. Visual diagnosis: an adolescent female who has persistent cough.
Pedratr Rev. 2002;23(3):101-105. doi:10.1542/pir.23-3-101.

American academy of pediatrics 600


American Academy of Pediatrics PREP 2015

Item 195
An 8-year-old girl presents with her third episode of acute onset of intermittent yet worsening
vomiting, low-grade fever, and pain with eating over the last 48 hours. The pain is described as
continuous, sharp, midepigastric, and periumbilical with radiation to her back. She is lying on the
examination table and appears restless and uncomfortable.
There is no history of trauma, sore throat, diarrhea, or dysuria. There is a family history of early
myocardial infarction in her father at 45 years of age and a stroke in her maternal grandmother at
53 years of age.

The girl's vital signs are within normal limits, except for a heart rate of 130 beats/min. Her retina
is pale with white-colored blood vessels. Her abdomen is diffusely tender without localization.
Her liver is palpable 3 cm below the costal margin, and her spleen is palpable 2 cm below the
costal margin. There are yellow papules noted on her trunk, buttocks, and on the extensor surface
of her extremities.

Results of her bilirubin, liver transaminases, and complete blood cell count are normal. Her
lipase level is 200 U/L and amylase level is 550 U/L. Triglyceride level is 2,600 mg/dL (29.4
mmol/L). Total cholesterol level is 250 mg/dL (6.47 mmol/L).

Abdominal ultrasonography shows no gallstones and the common bile duct is not dilated. There
is no pseudocyst in the pancreas, but there is a prominent pancreatic head.

Of the following, the condition that BEST explains the patient's history is
A. αl-antitrypsin deficiency
B. cystic fibrosis
C. familial hypertriglyceridemia
D. hereditary pancreatitis
E. uncontrolled diabetes mellitus

American academy of pediatrics 601


American Academy of Pediatrics PREP 2015

Item 195 Preferred Response: C


The girl in the vignette presents with recurrent abdominal pain and acute pancreatitis in the
presence of abnormal clinical examination and laboratory findings, which include xanthomas of
the skin, milky white pallor of the retinal vessels (lipemia retinalis), mild hepatosplenomegaly,
elevated amylase and lipase, hypertriglyceridemia )500 mg/dL) and mildly elevated low-density
lipoprotein (LDL). With a family history of early atherosclerotic disease, she has a typical
presentation for familial hypertriglyceridemia (FH). At this point, a lipid specialist should
become involved in the management of this patient's condition, which will likely include
pharmacologic therapy with a statin drug and dietary interventions. Most patients with severe
hypertriglyceridemia will not significantly improve with dietary intervention alone, An important
focus in pediatric guidelines is to identify children with inherited lipid disorders, who are at
significant risk for premature cardiovascular disease. A teenager with a low-density lipoprotein
cholesterol (LDL-C) level more than or equal to 190 mg/dL (4.9 mmol/L) has an 80% chance of
having familial hypercholesterolemia. Familial heterozygous hypercholesterolemia would
present with a family history of early coronary artery disease (CAD) or dyslipidemia, a small
pale ring around the iris of the eye in adulthood, xanthomas in young adulthood, and a total
cholesterol level between 325 and 350 mg/dL (8.42 and 9.06 mmol/L). In familial homozygous
hypercholesterolemia, the total cholesterol is 500 to 1,000 mg/dL (12.95-25.90 mmol/L) and
patients have CAD before age 20 years and xanthomas on tendons in the first few months after
birth. Familial combined hyperlipidemia is an autosomal dominant disorder that is characterized
by either elevations in triglycerides or LDL-C or both, with a family history of early CAD or
dyslipidemia. Of these patients, 10% to 20% will present with premature CAD.

Atherosclerosis can begin in childhood. Its severity is related to the presence of other
cardiovascular risk factors, such as blood pressure, lipid levels, tobacco exposure, physical
inactivity, diabetes mellitus (DM), and obesity. When combined with an elevated genetic risk,
atherosclerosis can manifest at an early age with myocardial infarction, stroke, peripheral
vascular disease, and ruptured aortic aneurysm. The more common genetic predispositions for
early atherosclerosis seen in childhood include familial heterozygous hypercholesterolemia,
family history of premature coronary heart disease in a close relative (men aged < 55 years and
women aged < 65 years), familial combined hyperlipidemia, familial defective apolipoprotein B,
family history of dyslipidemia, and type 1 DM.

The 2011 National Heart, Lung, and Blood Institute and the American Academy of Pediatrics
screening recommendations encourage universal cholesterol screening at least once between 9
and 11 years of age, as well as between 17 and 21 years of age with a nonfasting sample for non-
high-density lipoprotein cholesterol (HDL-C). If the non-HDL-C is elevated on universal
screening, a full lipid profile is recommended. If there is a positive family history of
cardiovascular disease, a parent has hyperlipidemia, or if other risk factors are present, a full
lipid profile should be performed between 1 and 4 years of age. Total cholesterol levels are
typically low at birth with a rapid rise in the first 2 years after birth, then with another peak at 9
to 11 years of age. Levels decrease in puberty and increase again in early adulthood. Total
cholesterol levels over 200 mg/dL (5.18 mmol/L) warrant further evaluation because they can
increase susceptibility for premature cardiovascular disease in adulthood.

American academy of pediatrics 602


American Academy of Pediatrics PREP 2015

If hypercholesterolemia is found, treatment begins with a 6-month trial of lifestyle modifications,


such as a low-fat diet and increased exercise, with repeat lipid panel to assess response. In cases
of familial hypercholesterolemia or with other risk factors when dietary changes alone are not
sufficient, statins are recommended as first-line pharmacologic treatment and can be used in
patients as young as 10 years of age if the LDL-C level is 190 mg/dL (4.92 mmol/L) or greater.
If a child has an LDL-C level of 190 mg/dL or greater in conjunction with a confirmation of a
severe primary dyslipidemia, a lipid specialist may choose to implement cholesterol lowering
medications from birth to 10 years of age. Statins are tolerated well, safe, and efficacious in
controlled studies; however, lifestyle modification is always preferred over pharmacotherapy if
effective alone. In a child who is 10 to 21 years of age, a statin should be considered after a 6-
month dietary trial if the LDL-C level is greater than 190 mg/dL or if the LDL-C level is 160 to
189 mg/dL (4.14-4.92 mmol/L) in the presence of a positive family history of early heart disease
or at least 1 high level risk factor or 2 moderate-level risk factors. High-level risk factors include
hypertension requiring drug therapy, cigarette smoker, body mass index at or above the 97th
percentile, DM1, DM2, chronic kidney disease, or Kawasaki disease with current aneurysms.
Moderate-level risk factors include hypertension not requiring drug therapy, body mass index at
or above the 95th percentile but less than the 97th percentile, Kawasaki disease with regressed
coronary artery aneurysms, chronic inflammatory disease, HIV infection, or nephritic syndrome.
The goal in cases of childhood heterozygous FH is to achieve an LDL-C level of 130 mg/dL
(3.37 mmol/L) or less. Sometimes combination therapy beyond dietary intervention and statins is
required to attain optimal levels; these combination therapies can include the addition of bile acid
sequestrant, fibrate therapy, cholesterol absorption inhibitors, or niacin, always in consultation
with a lipid specialist. This therapy and monitoring will be lifelong.

Alpha-1 antitrypsin deficiency presents with inflammation of the liver that can lead to cirrhosis
and liver failure, typically in older children and adolescents. In adulthood, these patients develop
lung disease with chronic obstructive pulmonary disease, emphysema, asthma, recurrent
pneumonia, and chronic bronchitis. They do not typically present with pancreatitis.
Patients with cystic fibrosis can develop pancreatitis, but usually present with respiratory
findings, failure to thrive, and digestive symptoms, none of which were seen in the child in this
vignette.

Hereditary pancreatitis is an inherited form of pancreatitis that presents with recurrent episodes
of pancreatitis in numerous close relatives. These patients do not have abnormal lipid values. It
can progress to chronic pancreatitis. People with this condition typically present before the age
of 20 years and have repeated bouts of upper abdominal pain, nausea, diarrhea, vomiting,
malnutrition, and possibly diabetes. They are also at higher risk for pancreatic cancer.
Acute pancreatitis can also occur in the setting of uncontrolled diabetes mellitus. The girl
described in this vignette did not have findings suggestive of DM.

PREP Pearls
• Recurrent pancreatitis with elevated lipids and a family history of early atherosclerotic
disease is indicative of a familial dyslipidemia.
• The 2011 National Heart, Lung and Blood Institute and the American Academy of
Pediatrics screening recommendations encourage universal cholesterol screening at least
once between 9 and 11 years of age, as well as between 17 and 21 years of age.
American academy of pediatrics 603
American Academy of Pediatrics PREP 2015

• If there is a positive family history of cardiovascular disease, a parent with a known


hyperlipidemia, or other risk factors, a full lipid profile should be performed between 1
and 4 years of age.
• if hypercholesterolemia is found, treatment begins with lifestyle modifications, such as a
low-fat diet and increased exercise.
• In cases of familial dyslipidemias in which dietary changes alone are not sufficient,
Matins are recommended as first-line pharmacologic treatment for children as young as 8
years of age.

American Board of Pediatrics Content Specification(s)


• Recognize the clinical features associated with lipoprotein disorders
• Plan the appropriate immediate and long-term management of lipoprotein disorders,
while considering the long-term prognosis

Suggested Reading
• European Association for Cardiovascular Prevention & Rehabilitation: Reiner 2,
Catapano AL, De Backer G, et al; ESC Committee for Practice Guidelines (CPG) 2008-
2010 and 2010-2012 Committees. ESC/EAS guidelines for the management of
dyslipidaemias: the task force for the management of dyslipidaemias of the European
Society of Cardiology (ESC) and the European Atherosclerosis Society (EAS). Eur f
Heart. 2011;32:1769-1818. doi:10.1093/eurheartjIebr158.
• Kavey RE, Allada V, Daniels SR, et al. Cardiovascular risk reduction in high-risk
pediatric patients: a scientific statement from the American Heart Association Expert
Panel on Population and Prevention Science. Circulation. 2006:114(241:2710-238.
doi:10.1161/ CIRCULATIONAHA.106.179568.
• National Heart, Lung, and Blood Institute, Expert Panel on Integrated Guidelines for
Cardiovascular Health and Risk Reduction in Children and Adolescents. Expert panel on
integrated guidelines for cardiovascular health and risk reduction in children and
adolescents: summary report. Pediatrics. 2011;128(suppl 5):5213-5256.
doi:10.1542/peds.2009-2107C.
• Psaty BM, Rivara FP. Universal screening and drug treatment of dyslipidemia in children
and adolescents. JAMA. 2012;307(31:257-258. doi:10.1001/jama.2011.1916.

American academy of pediatrics 604


American Academy of Pediatrics PREP 2015

Item 196
You are seeing a 4-year-old girl in your office for a health supervision visit. The patient received
a renal transplant from her father 6 weeks ago and is on a steroid-free protocol. She is doing well
after transplantation, with no concerns from the transplant team. Her current medications include
tacrolimus, mycophenolate mofetil, atenolol, and oral magnesium and phosphorus supplements.
She is afebrile with a respiratory rate of 18 breaths/min, heart rate of 78 beats/min, and blood
pressure of 114/60 mm Hg. Her physical examination shows a postsurgical abdominal scar that
is healing well with no inflammation or discharge noticed. Her parents inquire about her planned
immunization schedule.

Of the following, the MOST appropriate vaccination plan for this patient is
A. inactivated and live attenuated vaccines may be given 6 months after transplantation
B. inactivated and live attenuated vaccines may be given 1 year after transplantation
C. inactivated vaccines may be given at this visit
D. inactivated vaccines may be given 6 months after transplantation
E. inactivated vaccines may be given today and live attenuated vaccines 1 year after
transplantation

American academy of pediatrics 605


American Academy of Pediatrics PREP 2015

Item 196 TE Preferred Response: D


Immunosuppressive medications are indicated in patients who have (1) malignancy, (2) allograft
transplant, and (3) connective tissue and autoimmune disorders. These medications given after
allograft transplantation to prevent or treat organ rejection decrease the numbers and/or function
of T and B lymphocytes.

Children and adolescents being considered for solid organ transplantation should therefore
receive immunizations recommended for their age at least 2 weeks (preferably at least 1 month
for live-virus vaccines) before transplantation, because the vaccines will have a better
immunogenic response before transplantation.

American academy of pediatrics 606


American Academy of Pediatrics PREP 2015

Item 197
A 10-year old boy presents to the emergency department with the complaint of increasing
lethargy and an unsteady gait. Past history demonstrates that he was well until 1 year ago, when
he underwent an emergency laparotomy for bilious vomiting and small intestinal obstruction. At
surgery, he was found to have intestinal malrotation with midgut volvulus. He underwent a
corrective procedure that included resection of 2 ft of necrotic bowel in the midjejunum, along
with lysis of Ladd bands. Since that time, he has experienced intermittent bouts of postprandial
abdominal distension, bloating, and diarrhea. Physical examination demonstrates a somnolent,
thin boy with a body mass index of 15. Aside from a well-healed, midline abdominal surgical
scar, no other physical abnormalities are found. Initial laboratory data include the following:

• Hemoglobin, 10.8 g/dL (108 g/L)


• Hematocrit, 33% (0.33)
• White blood cell, 12,000/µL (12 x 109/L), with a differential within reference range
• Platelets, 175 x 103/ µL (175 x 109/L)
• Reticulocyte count, 1.5% (0.015)
• Mean corpuscular volume, 82 pm' (82 IL)
• Sodium, 143 mEq/L (143 mmol/L)
• Potassium, 3.5 mEq/L (3.5 mmol/L)
• Chloride, 110 mEq/L (110 mmol/L)
• Bicarbonate, 14 mEq/L (14 mmol/L)
• Glucose, 85 mg/dL (4.7 mmol/L)
• Blood urea nitrogen, 14 mg/dL (5.0 mmol/L)

Of the following, the test that is MOST likely to demonstrate the cause of this boy's symptoms is
measurement of serum
A. D-lactic acid
B. folic acid
C. thiamine
D. vitamin B1
E. vitamin E

American academy of pediatrics 607


American Academy of Pediatrics PREP 2015

Item 197 Preferred Response: A


In patients who have undergone abdominal surgery, the new onset of neurologic symptoms
should suggest D-lactic acid acidosis related to bacterial overgrowth syndrome. In patients with
suspected malabsorption, an acid stool pH suggests carbohydrate excretion. Lipid malabsorption
may be associated either with diarrhea or with constipation.

In malnourished states associated with underlying bowel disorders, including Crohn disease and
celiac disease. The lactose tolerance test has largely been supplanted by the breath hydrogen test
to identify patients with lactose intolerance, a common problem in patients with bowel disease,
malnutrition, and chronic diarrhea. Inflammatory bowel disease (IBD) serologies would also be
appropriate in this clinical setting, particularly if there were a family history of IBD, or in the
presence of other signs and symptoms of Crohn disease. However, negative serologies do not
rule out IBD, especially in pediatric patients.

PREP Pearls
• In patients who have undergone abdominal surgery, the new onset of neurologic
symptoms should suggest D-lactic acid acidosis related to bacterial overgrowth
syndrome.
• In patients with suspected malabsorption, an acid stool pH suggests carbohydrate
excretion.
• Lipid malabsorption may be associated either with diarrhea or with constipation.

American Board of Pediatrics Content Specification(s)


• Formulate an age-related differential diagnosis for malabsorption
• Recognize clinical situations in which bacterial overgrowth may play a role in
malabsorption

Suggested Reading
• Green PH, Jabri B. Celiac disease. Annu Rev Med. 2006;57:207-221.
doi:10.1146/annurev.med.57.051804.122404
• Jacob R, Zimmer K-P, Schmitz J, Nairn HY. Congenital sucrase-isomaltase deficiency
arising from cleavage and secretion of a mutant form of the enzyme. I Clin Invest.
2000;106(2):281-287. doi:10.1172/JCI9677.
• Montalto M, Santoro L, D'Onofrio F, et al. Classification of malabsorption syndromes.
Dig Dis. 2008;26(2):104-111. doi:10.1159/000116767.
• Pietzak MM, Thomas DW. Childhood malabsorption. Pediatr Rev. 2003; 24(6):195-206.
doi:10.1542/pir.24-6-195.
• Robayo-Torres CC, Quezada-Calvillo R, Nichols BL. Disaccharide digestion: clinical
and molecular aspects. Clin Gastroenterol Hepatol. 2006; 4(3):276-287.
doi:10.1016/j.cgh.2005.12.023
• Zhang DL, Jiang ZW, Jiang J, Cao B, Li 75. D-lactic acidosis secondary to short bowel
syndrome. Postgrad Med 2003;79(928):110-112. doi:10.1136/ pmj.79.928.110.

American academy of pediatrics 608


American Academy of Pediatrics PREP 2015

Item 198
A 13-month-old boy is brought to the office by his mother because of concerns about an unusual
white reflection noted in his right eye (Item Q198) on recent photographs. She has also noted this
eye intermittently turning inward. The child is otherwise healthy.
Family history is remarkable for the patient's maternal grandfather requiring enucleation of his
right eye as an infant because of an abnormality. A physical examination shows an abnormal red
reflex in the right eye with slight esotropia. No other abnormalities are noted.
You refer the child for an ophthalmology examination, which reveals a large, elevated, round,
yellow mass in the right eye and a small mass in the left eye.

Of the following, the MOST likely secondary tumor type that may occur with this disease is
A. astrocytoma
B. leukemia
C. neuroblastoma
D. osteosarcoma
E. Wilms tumor

American academy of pediatrics 609


American Academy of Pediatrics PREP 2015

Item198 TE Preferred Response: D


Retinoblastoma is a malignant intraocular eye tumor that develops in the retina typically before
age 5 years. It can be unilateral or bilateral. Patients with a family history of retinoblastoma
(approximately 10%) or who have bilateral or multifocal retinoblastoma are more likely to have
hereditary retinoblastoma, whereas unilateral and unifocal retinoblastoma tumors are more likely
to be sporadic. Hereditary retinoblastoma is caused by germline heterozygous RBI gene
mutations on chromosome 13q14 that are transmitted in an autosomal dominant manner, with the
retinoblastoma tumor developing after a second somatic mutation or "hit" occurs in the RBI
gene. About 40% of patients with retinoblastoma have hereditary retinoblastoma. The remainder
of cases (60%) are sporadic. All patients with hereditary retinoblastoma have a 50% chance of
passing the gene change on to their children.

The clinical diagnosis of retinoblastoma is based on a thorough examination of the fundus of the
eye using an ophthalmoscope revealing a white pupillary reflex known as Leukocoria. Another
common presentation is strabismus that can coincide with or be preceded by leukocoria. If 1 eye
appears affected, then the other eye should be thoroughly assessed for evidence of a
retinoblastoma as well. Staging and definitive diagnosis is supported by magnetic resonance
imaging (MRI) and ultrasonography. A head MRI should be included to evaluate for the
presence of a pinealoblastoma, which is a retinoblastoma in the pineal gland (if present, this is
called trilateral retinoblastoma and can occur at the time of diagnosis or subsequent to the
diagnosis of the retinoblastoma). All patients with retinoblastoma should have a medical genetics
evaluation for germline RBI mutations.

Preservation of sight is preferred, but not always possible. Treatment choices include
enucleation, cryotherapy, chemotherapy, combination cryotherapy-chemotherapy, and radiation
therapy. External beam radiotherapy should ONLY be used if absolutely necessary because of
the increased risk of second primary cancers with this form of therapy. It is also advisable to
avoid the use of computed tomography as an imaging technique because of the increased
radiation exposure associated with this modality. Most patients require the involvement of
multiple specialists: ophthalmology, oncology, pathology, and radiation oncology. If the tumor
has not spread beyond the eye, the cure rate is greater than 90%.
If a child is at risk for hereditary retinoblastoma based on a known RBI germline mutation, a
personal history of unilateral retinoblastoma, or a family history of retinoblastoma, he or she
should have an eye examination every 3 to 4 weeks until 1 year of age and frequent surveillance
until 3 years of age. Between 3 and 7 years of age, an eye examination should be conducted
every 3 to 6 months, then annually to biannually for the remainder of the individual's life.
Patients with hereditary retinoblastoma are susceptible to second primary cancers, most
commonly pineal gland tumors, osteosarcomas, soft tissue sarcomas, or melanomas. These
typically appear in adolescence or adulthood. More than 50% of individuals with second primary
tumors have a history of external beam radiation therapy. There are not currently standardized
screening protocols for patients with a history of retinoblastoma; however, careful attention
should be paid to any symptoms or signs that may suggest a secondary cancer, such as bone pain
or masses. Some centers advocate for total body MRIs at routine intervals, but this is not an
established screening protocol.

American academy of pediatrics 610


American Academy of Pediatrics PREP 2015

Certain genetic disorders carry a higher predisposition for specific types of cancers.
Astrocytomas are more commonly seen in neurofibromatosis type 1 or tuberous sclerosis.
Leukemia can be seen in Fanconi anemia, ataxia-telangiectasia, Bloom syndrome, and trisomy
21. Neuroblastoma can be seen in hereditary neuroblastoma because of ALK gene mutations,
neurofibromatosis type 1, central congenital hypoventilation syndrome due to PHOX2b gene
mutations, and Hirschsprung disease. Wilms tumor can be seen in Beckwith-Wiedemann
syndrome, WAGR (Wilms tumor-aniridia-genitourinary anomalies-retardation) syndrome,
Denys-Drash syndrome, Frasier syndrome, and isolated hemihyperplasia.

PREP Pearls
• If a child has a history of retinoblastoma, second primary cancers (the most common of
which are osteosarcomas, soft tissue sarcomas, and melanomas) can occur in childhood,
adolescence, or adulthood.
• Retinoblastoma in childhood, which presents with a white pupillary reflex, highlights the
importance of using the ophthalmoscope during each well-child check in infancy and
childhood to look for a normal positive red reflex or any asymmetries suggestive of
potential pathology.
• if there is a family history of retinoblastoma, pay careful attention to the eye examination
of the child during infancy and childhood and refer to ophthalmology to implement high-
risk surveillance protocols to screen for retinoblastoma.

American Board of Pediatrics Content Specification(s)


• Recognize inheritance pattern associated with retinoblastoma and the significance of the
family history in planning management

Suggested Reading
• Canadian Retinoblastoma Society. National Retinoblastoma Strategy Canadian
Guidelines for Care: Strategic th6rapeutique du retinablastome guide clinique canaclien.
Can Ophthalmol. 2009:44(suppL2):S1-88. doi:10.3129/i09-194.
• Lohmann DR, Gallic 81_ Retinoblastoma. GeneReviews, Accessed January 22,2014.
• National Cancer Institute_ Retinoblastoma. National Cancer Institute t,ebsite.

American academy of pediatrics 611


American Academy of Pediatrics PREP 2015

Item 199
The parents of a 12-year-old boy bring him to the emergency department for evaluation of
sudden onset of nausea, vomiting, headache, and slurred speech. He was well when he left for
school in the morning, but when his mother returned home from work today, she found him
asleep. The parents report that the patient has no significant past medical or surgical history. Two
weeks ago, he had symptoms of an upper respiratory infection that have since resolved. No
changes in appetite, weight, or elimination habits are noted. There is no known trauma, although
he plays soccer daily. On physical examination, you note tachycardia, dilated pupils, and
diaphoresis. He laughs inappropriately when questioned. You suggest performing a drug screen.

Of the following, the MOST likely drug abused in this case is


A. atomoxetine
B. dextromethorphan
C. diphenhydramine
D. methylphenidate
E. oxycodone

American academy of pediatrics 612


American Academy of Pediatrics PREP 2015

Item 199 S SBP Preferred Response: B


The boy in this vignette presents with sudden onset of nausea, vomiting, headache, and altered
mental status (sleepiness, slurred speech, and inappropriate laughter) after being home alone.
Whenever there are no physical signs of trauma or findings suggestive of an acute infectious,
metabolic, or central nervous system process, one must consider accidental or purposeful
ingestion of a chemical agent as the cause. Unfortunately, abuse of prescription drugs and over-
the-counter medications is a significant health problem and it is increasing. Next to marijuana
and alcohol, prescription medications are the most common drugs that teenagers use to get high.
Medications that can be purchased over the counter are also often used inappropriately for
recreational purposes. There may be several motivating factors to the nonmedical use of
prescription and over-the-counter medications, such as the wrongful presumption that they are
less addicting or harmful, and fewer concerns about the legal implications. In addition to the
history, the findings on physical examination are key to determining the most likely drug abused.
In this case, dextromethorphan is the most likely drug abused.

It has been estimated that as many as 1 million youth and young adults in the United States
misuse over-the-counter cough and cold medications each year, and reports from other countries
suggest this abuse is prevalent there as well. Dextromethorphan, to which the patient in the
vignette had access, is commonly found in cough and cold medicines, and ingestion should be
suspected when there is an abrupt change in behavior associated with findings of sympathetic
stimulation (tachycardia, mydriasis, diaphoresis), gait disturbance, and euphoria or
hallucinations. The signs of dextromethorphan toxicity are dose dependent, ranging from mild
stimulation to euphoria and hallucinations to a dissociative state and then unresponsiveness. If
co-ingested with mono-amine oxidase inhibitors or other serotonergic agents, the serotonin
syndrome may be produced, which is life threatening. Common street names for
dextromethorphan include Orange Crush, Triple Cs, Skittles, Vitamin D, Dex, and Robo. It can
be swallowed or snorted. Of the 10% of teenagers who reported the use of cough medicine to get
high, only 48% believed it was risky. Diphenhydramine is also easily purchased over the counter
and is commonly used for its antihistamine and antitussive qualities. Rather than diaphoresis, dry
flushed skin would be the manifestation of an overdose.

Abused prescription medications most often belong to the individual or a friend. Nearly 30% of
adolescents report having a friend who abuses prescription stimulants, and almost 20% report
taking prescription medications that were not prescribed to them at least once. Atomoxetine and
methylphenidate are both commonly prescribed for the treatment of attention-
deficit/hyperactivity disorder (ADHD), so teenagers have ready access to these medications.
Although most individuals with ADHD use their medications appropriately, increased
availability raises the potential for use of these drugs for nonmedical purposes or by those
without a proper diagnosis of ADHD. Attention-deficit/hyperactivity disorder medications may
be used inappropriately to help improve concentration or to increase alertness in an effort to
enhance performance on examinations or during times of high productivity demand by students
without ADHD. Misuse of these drugs would present with increased alertness rather than the
sleepiness and slurred speech seen in the boy in the vignette. Oxycodone is another popular
substance of abuse. Estimates suggest that 10% to 20% of teenagers have abused an opioid
prescription pain reliever. Oxycodone ingestion would present with miosis, bradycardia,

American academy of pediatrics 613


American Academy of Pediatrics PREP 2015

hypotension, hypothermia, hyporeflexia, decreased respirations, and impaired mentation. Other


common prescription medications abused include sleep agents and sedatives or anxiolytics.

PREP Pearls
• Dextromethorphan and other agents commonly found in cough and cold medicines
are often used inappropriately for recreational purposes.
• Dextromethorphan overdose manifests as a combination of stimulant, euphoric, and
dissociative effects.
• The nonmedical use of prescription medications is becoming increasingly common
and is associated with a greater likelihood of substance abuse.

American Board of Pediatrics Content Specification(s)


• Recognize the risk of abuse of over-the-counter cough and cold preparations
• Recognize the risk of abuse of prescription medications

Suggested Reading
• Boyd CJ, McCabe SE, Cranford JA, Young A. Adolescents' motivations to abuse
prescription medications. Pediatrics. 2006;118(6):2472-2480. doi:10.1542/peds.2006-
1644.
• Gunn VL, Taha SH, Liebelt EL, Serwint JR. Toxicity of over-the-counter cough and
cold medications. Pediatrics. 2001;108(3)152. doi:10.1542/ peds.108.3.e52.
• Parran TV, Wilford BB, DuPont RL. Prescription drug abuse and addiction: clinical
features, epidemiology, and contributing factors. UpToDate. Available online only
for subscription.
• Rosenbaum C, Boyer EW. Dextromethorphan poisoning: epidemiology.
pharmacology and clinical features. UproDate. Available online only for subscription.
• Schwartz B, Alderman EM. Substances of Abuse. Pediatr Rev. 1997;l8(6):204-215.
doi:10.1542/pir.18-6-204.
• Setlik J. Bond GR. Ho M. Adolescent prescription ADHD medication abuse is rising
along with prescriptions for these medications. Pediatrics. 2009;124(3):875 -880.
doi:10.1542/peds.2008-0931.
• Shehab N, Schaefer MK, Kegler SR, Budnitz DS. Adverse events from cough and
cold medications after a market withdrawal of products labeled for infants. Pediatrics.
2010d 26(6):1100-1107. doi!10.1542/peds.2010-1839.
• Wilford BB, Parran TV, DuPont EL. Prescription drug abuse and addiction:
prevention, identification, and management. UproDate. Available online only for
subscription.

American academy of pediatrics 614


American Academy of Pediatrics PREP 2015

Item 200
An 18-year-old man presents to your clinic for testicular swelling. He also reports dull lower
abdominal pain for the past few weeks. He has been sexually active with 3 female partners in his
lifetime and reports that he uses condoms most of the time. He denies any weight loss or fever.
On physical examination, the right testicle appears noticeably larger than the left with no
discoloration or tenderness. The remainder of the physical examination is unremarkable. You
order testicular ultrasonography, which reveals a well-defined, noncystic solid mass in the right
testicle.
Of the following, the MOST appropriate next test in the evaluation of this mass is
A. bone radionuclide scan
B. carcinoembryonic antigen level
C. computed tomography scan
D. needle biopsy
E. positron emission tomography scan

American academy of pediatrics 615


American Academy of Pediatrics PREP 2015

Item 200 TE Preferred Response: C


The patient in the vignette should have a computed tomography (CT) scan to evaluate the
testicular mass seen on ultrasonography and for abnormalities in the abdominal lymph nodes.
The greatest risk factor for the development of testicular carcinoma is an undescended testicle
that remains uncorrected beyond 1 year of age with an estimated 10- to 50-fold higher risk of
cancer. In prepubertal boys, the tumors are usually yolk sac tumors, gonadoblastomas, or
teratomas, whereas in adolescents, embryonal carcinoma and teratocarcinoma are more common.
Most testicular tumors present as a painless scrotal mass. The lack of symptoms may lead to a
delay in seeking medical attention. A reactive hydrocele or an inguinal hernia may be present at
the time of diagnosis. Fortunately, the majority of pediatric testicular tumors are localized,
however, metastasis can occur most commonly to the lung and the lymph nodes of the
retroperitoneum.

After initial evaluation of the suspected mass by testicular ultrasonography, a CT scan should be
performed to visualize tumor margins and to evaluate the chest and the lymph nodes in the
abdomen. Bone radionuclide scan and positron emission tomography scan are used to look for
areas in the bone and lymph nodes (respectively) with increased metabolic activity suggestive of
inflammation, infection, or malignancy; however, this would not be indicated in the initial
evaluation of suspected testicular cancer.

Carcinoembryonic antigen levels are not generally elevated in the various types of testicular
tumors, therefore this would not be the correct tumor maker. a-fetoprotein (elevated in yolk sac
tumors and embryonal carcinomas) and 13-human choriogonadotropin (sometimes elevated in
embryonal carcinomas) may be useful tumor markers for certain types of testicular cancer. A
needle biopsy should not be performed due to the potential for tumor seeding, which can lead to
cancer recurrence; therefore, radical inguinal orchiectomy with en bloc excision of the spermatic
cord and testicles is recommended for diagnosis and treatment. Depending on the stage, multi-
agent chemotherapy may be indicated.

PREP Pearls
• The greatest risk factor for the development of testicular carcinoma is an undescended
testicle that remains uncorrected beyond 1 year of age with an estimated 10- to 50-
fold higher risk of cancer.
• Most testicular tumors present as a painless scrotal mass. There may be a reactive
hydrocele or an inguinal hernia present at the time of diagnosis.
• A needle biopsy of a testicular mass should not be performed due to the potential for
tumor seeding, which can lead to cancer recurrence, therefore radical inguinal
orchiectomy with en bloc excision of the spermatic cord and testicles is recommended
for diagnosis and treatment.
• After initial evaluation of the suspected mass by testicular ultrasonography, a
computed tomography scan should be performed to visualize tumor margins and to
evaluate the lungs and the lymph nodes in the abdomen.
• α-fetoprotein and β-human choriogonadotropin may be useful tumor markers for
certain types of testicular cancer.

American academy of pediatrics 616


American Academy of Pediatrics PREP 2015

American Board of Pediatrics Content Specification(s)


• Plan the appropriate evaluation of a testicular mass
• Recognize risk factors associated with testicular cancer
• Recognize the laboratory findings associated with a germ cell tumor

Suggested Reading
• Chan E, Wayne C, Nasr A; FRCSC for the Canadian Association of Pediatric Surgeon
Evidence-Based Resource. Ideal timing of orchiopexy: a systematic review, Pediatr Slag
Int. 2014;30(1):87-97. doi:10,10071s00383- 013-34291.
• Nallu A, Mannuel HD, Hussain A. Testicular germ cell tumors: biology and clinical
update. Corr Opin Oncoe. 2013;25(3)166-272. doi:10.1097I CC0.0b013e32835fF3e3.
• Vasdev N, Moon A, Thorpe AC. Classification, epidemiology and therapies for testicular
germ cell tumours. Int f Dev Biot. 2013;57(2-0133-9. doi:10.1387/ndb.I30031nv.

American academy of pediatrics 617


American Academy of Pediatrics PREP 2015

Item 201
You are seeing an 8-year-old boy in your office who, up until recently, had been doing very well
in school and at home. His parents describe him as very bright. He learned to write sentences
before entering kindergarten and has long been using language that is more advanced than that of
his peers. He is fascinated with mathematics and puzzles. Recently, he has been refusing to
participate in school. He often stares off during class and talks back to his teacher; he has been
getting poor grades. He refuses to do schoolwork at home as well. The family denies any acute
stressors at home, and they are not aware of any maltreatment or bullying occurring at school. He
does not have any close friends in school, but there is a friend from chess camp that he likes to
play and talk with.

Of the following, the BEST next step is to


A. evaluate him for an attention-deficit disorder
B. recommend getting him engaged in a sport like soccer after school
C. recommend putting him in more advanced classes or a gifted program
D. refer him to a psychologist for learning disability testing
E. refer him to a therapist for individual psychotherapy

American academy of pediatrics 618


American Academy of Pediatrics PREP 2015

Item 201 Preferred Response: C


The most likely explanation why the boy in the vignette recently refused to participate in school
and his staring off during class is that he is academically gifted, and has been developing apathy
and rebellion from accumulated intellectual boredom in the mainstream school environment.
Very bright children who find themselves repeatedly rehearsing skills that they mastered years
earlier as their sole academic experience may accumulate disdain or despair for the overall
educational system and develop behavior problems in that setting. That the boy in the vignette is
gifted is suggested by advanced early milestones, unusual analytical interests (mathematics and
puzzles), and past strong school performance.

Clinically, it is important to differentiate a child who is failing school because of giftedness from
one who is failing school because of other causes; the most appropriate clinical response is quite
different.

Gifted children with school behavior problems typically respond positively to more advanced
and intellectually stimulating curricula, which is the opposite of what would typically best serve
a child failing school because of a teaming disability, oppositional defiant disorder, or attention-
deficit/ hyperactivity disorder (ADHD). This could mean placing a gifted child in more advanced
classes or enrolling him or her in academically challenging after-school activities and projects.
The solution should be child specific, because two other common characteristics of giftedness
are asynchronous development and difficulty with same-age peer relationships (see Item C201).
For instance, a gifted 8 year old might be proficient in algebra and trigonometry, but might also
prefer to do such work while holding a favorite stuffed animal. Therefore, simply advancing a
gifted child into a more advanced grade level may not be the best solution. Seeking advanced
curricula for a select group of same-age students, without having to advance a child multiple
grades, can help maintain appropriate peer social learning opportunities.

Item C201. Characteristics of Gifted Children


• Asynchrony across developmental domains Advanced language and reasoning skills
Conversation and interests like older children Insatiable curiosity perceptive questions
• Rapid and intuitive understanding of concepts
• Impressive long-term memory
• Ability to hold problems in mind that are not yet figured out
• Ability to make connections between one concept and another
• Interest in patterns and relationships
• Advanced sense of humor for age
• Courage in trying new pathways of thinking
• Pleasure in solving and posing new problems
• Capacity for independent, self-directed activities
• Talent in a specific area: drawing, music, games, math, reading
• Sensitivity and perfectionism
• Intensity of feeling and emotion
From Robinson NM, Olszewski-Kublhus PM, Gifted and talented children: issues for
pediatricians. Pediarr Rev. 1996;17:427-434

American academy of pediatrics 619


American Academy of Pediatrics PREP 2015

The child in the vignette is unlikely to have ADHD because of 3 prior school years with good
performance without noted attention problems.

Enrolling the boy in an after-school sport might be welcomed if he demonstrated interest, but is
unlikely to yield improved school engagement if he is academically bored in that setting.

IQ or academic testing may be of value in helping to document that a child is academically


gifted, but telling the parent to seek “learning disability testing” is wrong message to send to the
family in the vignette. A test like the Wide Range Achievement Test (WRAT) might be used to
document a current level of ability; this patient would likely perform well on this test if he were
motivated to do well on that occasion.

Referring the boy for individual psychotherapy would be appropriate if there were an anxiety,
depressive, or adjustment issue responsible for his poor school performance. Because the most
likely cause of his school problem is giftedness, referral to a therapist would not be appropriate
unless exposure to more challenging curricula failed to resolve the situation.

PREP Pearls
• Academically gifted children can become bored and rebellious in school if they lack any
stimulating challenges in that environment.
• Giftedness should be part of the differential diagnosis for academic failure in school.
• As gifted children tend to have asynchronous domains of development, simply advancing
them ahead in school years may not be the most appropriate approach to meet their
educational needs.

American Board of Pediatrics Content Specification(s)


• Understand the effects of gifted children on family dynamics
• Provide anticipatory guidance with respect to management of a gifted child

Suggested Reading
• Robinson 1" M, Olsiewski-Kubilius PM. Gifted and talented children: issues for
pediatricians. .Pediatr Rev. 1996;17:427-434. doi:10.1542/pir.17-12-427. Rosenberg MD,
Robokos I), Kennedy RF. The gifted child. Pedlar Rev. 2010;31(11:41-43.
doi:10.1542/pir.31-1-41.

American academy of pediatrics 620


American Academy of Pediatrics PREP 2015

Item 202
A 1-year-old girl transfers care to your practice after moving with her family from out of state.
She has a history of heart surgery for tetralogy of Fallot as an infant. She was last seen by a
cardiologist 6 months ago and was scheduled to be seen again after her first birthday. The child
is not taking medications now, but she was prescribed calcium supplements for several months
after surgery. She has had 3 admissions for pneumonia. The family history is significant for
psychiatric illness, including bipolar disorder in her aunt.
Physical examination shows a 1-year-old girl in no distress. Her head circumference is at the
40th percentile, weight is at the 50th percentile, and height is at the 60th percentile. Her ears
appear slightly small. Her chest is clear, and her sternotomy wound is well healed. Her cardiac
examination shows a normal point of maximal impulse. She has a 3/6 systolic murmur at the left
upper sternal border with a 1/6 diastolic murmur in the same location. Her liver is not enlarged.
Her femoral pulses are normal. As her primary care physician, in addition to continued general
pediatric care, you may need to coordinate several other services to follow-up with this patient.

Of the following, the BEST choice of specialists to help you coordinate this child's ongoing care
are
A. endocrinology and pulmonary
B. genetics and immunology
C. immunology and neurology
D. infectious disease and gastroenterology
E. pulmonary and psychiatry

American academy of pediatrics 621


American Academy of Pediatrics PREP 2015

Item 202 TE Preferred Response: B


The child in the vignette has congenital anomalies requiring the care of multiple subspecialists. It
is crucial for the primary care physician to coordinate referral to the appropriate subspecialists
and help coordinate the care between them.

Tetralogy of Fallot falls into the category of conotruncal defects. This is caused by a defect in
neural crest cell migration and causes the great vessel trunk to develop abnormally. The trunk
may not separate at all (truncus arteriosus), may not rotate (transposition of the great arteries), or
does so only incompletely (Tetralogy of Fallot). The same embryonic defect can also cause
complete interruption of the aortic arch.

Patients with any of these cardiac defects need to undergo genetic evaluation for possible 22q11
deletion and be monitored for immunologic dysfunction (severe cellular immunodeficiency with
thymic hypoplasia), as well as hypocalcemia secondary to parathyroid hypoplasia. Of the
subspecialties listed, the combination of genetics and immunology would be the most important,
in addition to cardiology, to help coordinate the care of this patient. Developmental delay and
psychiatric illness may become evident as this child gets older, Psychiatry and neurology
consultations may be needed after the initial evaluations. Cleft palate is another feature of this
syndrome, therefore otolaryngology consultation may be needed.

In patients with conotruncal defects. A high index of suspension needs to be maintained for any
of these other manifestations of 22q11 deletion syndrome. Monitoring calcium concentrations in
the newborn period and early genetic consultation are crucial. Infectious disease consultation
may be needed if the child develops unusual infections. Overall, 22q11 deletion syndrome has a
25% risk of conotruncal defects, 32% risk of cleft palate, 35% risk of renal anomalies, and a 1%
risk of severe immunodeficiency. Hypocalcemia is common in the newborn period, but usually
resolves. The primary care physician plays a key role in coordinating the many specialists who
will help to care for this patient.

PREP Pearls
• The primary care physician plays a key role in coordinating the care of patients requiring
referral to multiple subspecialists because of complex congenital abnormalities.
• Monitor calcium and consider patients with conotruncal defects to be immunosuppressed
until genetic and immunological workup is complete.

American Board of Pediatrics Content Specification(s)


• Understand the importance of a medical home for children with chronic or handicapping
conditions

Suggested Reading
• Doyle D. Hypoparathyroidism. In: Kliegman RM, Stanton BF, St Geme
• III, Schor NF, eds. Nelson Textbook of Pediatrics. 19th ed. Philadelphia, PA: Saunders
Elsevier; 2011;1916-1918.
• Gothelf D. Velocardiofacial syndrome. Odd Adolesc Psychiatr Clinic North Am.
2007;16(3):677-693.

American academy of pediatrics 622


American Academy of Pediatrics PREP 2015

• Srivatava D, Baldwin HS. Molecular determinants of cardiac development. In: Allen HD,
Driscoll DJ, Shaddy RE, Feltes TF, eds. Moss and Adams' Heart Disease in Infants,
Children and Adolescents. 6th ed. Philadelphia. PA: Lippincott Williams & Wilkins;
2001:14-15.

American academy of pediatrics 623


American Academy of Pediatrics PREP 2015

Item 203
A 14-year-old boy who was diagnosed 7 days ago with erysipelas returns to your clinic because
his rash is not improving. On physical examination, the area of erysipelas has spread a little since
diagnosis. He is afebrile, has no systemic signs or symptoms, and is otherwise healthy. On
review of the treatment regimen with the parents, you discover that the family is fasting during
daylight hours for religious purposes and that the boy has not been taking his midday dose as
prescribed.

Of the following, the BEST next course of action in this case would be to
A. admit the boy to the hospital for intravenous antibiotics
B. counsel the family on the risks of fasting for children
C. explain that there are exceptions to fasting for ill people
D. inform the parent that this is medical neglect
E. prescribe an antibiotic that can be taken once or twice a day

American academy of pediatrics 624


American Academy of Pediatrics PREP 2015

Item 203 I-C P Preferred Response: E


The best course of action in the situation described in the vignette is to prescribe an antibiotic
that can be taken once or twice a day. This family has been skipping doses of medication to
comply with their religious practices. In addressing this issue with the family, it is important to
consider the family's cultural beliefs and practices, and negotiate a treatment plan that will enable
adherence while aligning with their religious beliefs.

There is no strict definition of culture; it can refer to race, ethnicity, nationality, religion, or
sexual orientation. Culture determines how an individual views the world. In medical practice,
culture can be a barrier to care. The clinician should work to develop a practice that is culturally
competent by providing an atmosphere where patients and families can participate in shared
decision-making in a relationship based on trust. The first and most important aspect of a
culturally competent practice is good communication. The clinician should create opportunities
for patients and their families to voice their views and perceptions, and the clinician should
become educated regarding the values and beliefs that patients and families hold. The clinician
should work with each family to provide health care that is compatible with the patient's and
family's beliefs and values. It is important to address conflicting views in a professional and
ethical manner. In the LEARN model described by Berlin and Fowkes, it is recommended that
the clinician:
• Listen with sympathy and empathy to the patient's perception of the problem
• Explain the clinician's perception of the problem
• Acknowledge and discuss similarities and differences
• Recommend treatment
• Negotiate agreement

The boy in the vignette has no signs or symptoms of systemic or severe illness and should not be
admitted for intravenous antibiotics when other treatment options are available. The clinician
may wish to discuss the nutritional aspects of fasting with the family; however, this would not
address the treatment that is required for the patient. Some religions allow exceptions to fasting
for medical reasons, but this does not take into account a family's desire to adhere to their
religious practices and preference for treatment options that fit with the family's beliefs are
possible. In the vignette, the mother is giving the child the medication, and has returned to your
office for follow-up, so informing the parent that this is medical neglect would not be
appropriate.

PREP Pearls
• Culture can refer to race, ethnicity, nationality, religion, or sexual orientation.
• A culturally competent practice provides an atmosphere in which patients and families
can participate in shared decision-making.
• The clinician should work with individual families to provide health care that is
compatible with the beliefs and values of the patient and family.

American Board of Pediatrics Content Specification(s)


• Understand the various cultural issues that could affect medical care

Suggested Reading
American academy of pediatrics 625
American Academy of Pediatrics PREP 2015

• Kodjo C. Cultural competence in clinician communication. Pecliatr Rev. 2009;30(457-


64. doi:10.1542/pir.30-2-57.
• Swota A, Hester DM. Ethics for the pediatrician: providing culturally effective health
care. Pediatr Rev. 2011;32(3):e39-e43. doi:10.1542/pir.32-3-e39.

American academy of pediatrics 626


American Academy of Pediatrics PREP 2015

Item 204
A 15-year-old adolescent boy arrives at your office immediately after an injury to the right
shoulder. He landed awkwardly on the shoulder during wrestling practice, and now reports pain
over the right sternoclavicular joint. The boy reports no other symptoms. On physical
examination, there is a noticeable depression of the medial clavicle at the sternoclavicular joint.

Of the following, the MOST appropriate statement regarding this injury is that

A. no treatment is necessary but the boy will have a persistent mild bony deformity
B. the boy should be treated with a figure-eight harness for 4 weeks to 6 weeks
C. the boy should be treated with his arm in a sling for 4 weeks to 6 weeks
D. the boy should undergo elective surgery once the pain and swelling resolve
E. the boy will require additional evaluation and will likely need prompt surgery

American academy of pediatrics 627


American Academy of Pediatrics PREP 2015

Item 204 Preferred Response; E


The physical examination findings suggest that the boy in the vignette has posterior displacement
of his clavicle. With this type of injury, the clavicle is forced posteriorly and can damage the
recurrent laryngeal nerve, brachiocephalic or subclavian blood vessels, the lungs, or the trachea.
He should undergo urgent radiographic evaluation to assess the position of the clavicle and will
likely need prompt surgery.

While most fractures are isolated skeletal injuries, clinicians should be aware that some types of
bone and joint injuries have the potential to affect the vasculature. For example, severe
supracondylar humerus fractures are occasionally associated with tearing of the brachial artery.
Knee dislocations have a high incidence of associated vascular injury and require urgent
evaluation. Rarely, fractures and muscle injuries can be complicated by acute compartment
syndrome, a rapid increase in pressure in a myofascial compartment. Compartment syndrome
can lead to compression of the arteries and nerves in the affected compartment; arterial
compression results in tissue hypoperfusion and damage.

Posterior clavicular displacement can occur because of tearing of the ligaments at the
sternoclavicular (SC) joint or as a result of a proximal clavicle physeal fracture with posterior
displacement of the clavicle shaft. Closure of the clavicular physis often occurs after 20 years of
age, therefore physeal injury is more common in adolescents than ligament disruption. While this
type of injury is rare, the consequences can be severe; therefore, health care providers should be
aware that posterior displacement of the clavicle at the SC joint merits additional evaluation.
The typical mechanism for this type of injury is a high-velocity, posteriorly directed force to the
clavicle. Radiography may show posterior displacement; however, computed tomography (CT)
with contrast is needed to determine whether there is injury to the nerves or blood vessels behind
the clavicle. Posterior clavicle dislocation typically requires reduction of the clavicle under
anesthesia with surgical fixation if the injury appears unstable.

This patient should be treated because of the risk of neurovascular injury. A sling or figure-of-
eight strap would be appropriate for most midshaft and distal clavicle fractures, but would not be
sufficient for this patient. Elective surgery would not be indicated unless radiographic evaluation
showed no risk of injury to the mediastinal structures behind the clavicle.

PREP Pearls
• Children with apparent posterior displacement of the clavicle should undergo urgent
radiographic evaluation, because this type of injury can cause damage to neurovascular
structures in the mediastinum.
• Children with supracondylar fractures of the humerus have significant risk of associated
vascular injury and should be thoroughly evaluated.

American Board of Pediatrics Content Specification(s)


• Recognize the bone and joint injuries that commonly affect the vasculature

Suggested Reading

American academy of pediatrics 628


American Academy of Pediatrics PREP 2015

• Koch MI, Wells L. Proximal clavicle physeal fracture with posterior displacement:
diagnosis, treatment, and prevention. Orthopedics.2012;35(1):e108-e111.
doi:10.3928/01477447-20111122-39.

American academy of pediatrics 629


American Academy of Pediatrics PREP 2015

Item 205
A 2-year-old girl is brought to the emergency department with a 10-day history of crampy
abdominal pain and watery diarrhea that the parents report now contains blood. The family just
returned from a 3-week trip to Mexico where they traveled extensively in the countryside. On
physical examination, her temperature is 38°C, heart rate is 130 beats/min, respiratory rate is 20
breaths/min, blood pressure is 100/60 mm Hg, and weight is 10.6 kg (10th percentile). The
parents report her weight at a recent 2-year-old well-child check was 11 kg. The child appears ill,
but nontoxic. She is interactive with the examiner. Her mucous membranes are moist, and
capillary refill is less than 2 seconds. Her heart rhythm is normal without murmur, and her lungs
are clear to auscultation. Her abdomen is soft, nondistended, and tender to palpation in the lower
quadrants without rebound tenderness.

Of the following, the MOST likely etiology of the patient's illness is


A. enteropathogenic Escherichia coli
B. enterotoxigenic Escherichia roll
C. Entamoeba histolytica
D. Giardia intestinalis
E. Salmonella ser enteritidis

American academy of pediatrics 630


American Academy of Pediatrics PREP 2015

Item 205 Preferred Response: C


The girl in the vignette has crampy abdominal pain and bloody diarrhea after travel to Mexico.
Given her symptoms and travel to a location that likely had poor sanitary conditions, the most
likely cause of her illness is Entamoeba histolytica. This pathogen is found worldwide, most
commonly in tropical regions and resource-limited countries. It is spread by the fecal-oral route.
The incubation period is variable, usually 2 to 4 weeks. Only a minority (10% to 20%) of
infected persons become symptomatic. Illness can be mild with bloody stools, intense abdominal
pain, and fever (amebic dysentery). Severe infection can result in toxic megacolon,
gastrointestinal ulcers, or intestinal perforation. Rare extraintestinal complications include
amebic liver abscess or spread to other organs such as the lungs or brain.

Presumptive diagnosis of intestinal amebiasis can be made by identification of trophozoites or


cysts in the stool. Definitive diagnosis is made by detection of anti-E histolytica antibody by
enzyme immunoassay. Nearly all (95%) patients with extraintestinal amebiasis have positive
serology versus 70% of those with infection limited to the intestinal tract. Recommended
treatment is metronidazole (30-40 mg/kg per day, maximum of 2 g, orally in 3 doses for 7-10
days) or tinidazole (50 mg/kg per day, maximum of 2 g, orally once daily for 3-5 days, 3 years of
age and older). Either treatment is followed by a luminal amebicidal agent such as paromomycin
(25-35 mg/kg per day orally in 3 doses for 7 days) or iodoquinol (30-40 mg/kg per day orally in
3 doses for 20 days). Patients who are asymptomatic, but excreting cysts can be treated with a
luminal amebicidal agent alone.

Diarrhea caused by enteropathogenic Escherichia coli usually is mild, watery, nonbloody, and
common in resource-limited countries. It primarily affects children younger than 2 years of age.
Enterotoxigenic E coil causes a brief (1- to 5-day) diarrheal illness in travelers and young infants
in resource-poor settings characterized by watery, nonbloody diarrhea and abdominal cramps.
Giardia intestinalis is the most common cause of parasitic diarrhea worldwide. It causes a
spectrum of disease, most commonly presenting with watery, nonbloody diarrhea with
abdominal pain, foul-smelling stools, and flatulence. Salmonella serotype Enteritidis can cause
bloody diarrhea and fever; however, travel to resource-limited settings is not a risk factor for
infection caused by nontyphoidal Salmonella species, and the incubation period is less than 72
hours.

American academy of pediatrics 631


American Academy of Pediatrics PREP 2015

PREP Pearls
• Travel to resource-limited settings is a risk factor for intestinal amebiasis (Entamoeba
histolytica infection).
• Entamoeba histolytica can be asymptomatic or cause severe illness with bloody stools,
severe abdominal pain, and fever (amebic dysentery).
• Rare extraintestinal complications of E histolytica infection include amebic liver abscess
or spread to other organs such as the lungs or brain.
• Presumptive diagnosis of intestinal amebiasis is by identification of trophozoites or cysts
in stool; definitive diagnosis is made by detection of anti-E histolytica antibodies with an
enzyme immunoassay.
• Treatment of E histolytica infection is metronidazole or tinidazole, followed by a luminal
amebicidal agent such as paromomycin or iodoquinol. Patients who are asymptomatic,
but excreting cysts can be treated with a lumina' amebicidal agent alone.

American Board of Pediatrics Content Specification(s)


• Understand the epidemiology of amoebiasis
• Recognize the clinical features associated with amoebiasis, and manage appropriately

Suggested Reading
• American Academy of Pediatrics. Amebiasis. In: Pickering LK, Baker CJ, Kimberlin
DW, Long SS, eds. Red Book: 2012 Report of the Committee on Infectious Diseases.
29th ed. Elk Grove Village, IL: American Academy of Pediatrics; 2012:222-225.
• American Academy of Pediatrics. Escherichia coli diarrhea. In: Pickering LK, Baker CJ,
Kimberlin DW, Long SS, eds. Red Book: 2012 Report of the Committee on Infectious
Diseases. 29th ed. Elk Grove Village, IL: American Academy of Pediatrics; 2012:324-
328.
• American Academy of Pediatrics. Giardia intestinalis infections. In: Pickering LK, Baker
CJ, Kimberlin DW, Long SS, eds. Red Book: 2012 Report of the Committee on
Infectious Diseases. 29th ed. Elk Grove Village, IL: American Academy of Pediatrics;
2012:333-335.
• American Academy of Pediatrics. Salmonella infections, In: Pickering LK, Baker CJ,
Kimberlin DW, Long SS, eds. Red Book: 2012 Report of the Committee on Infectious
Diseases. 29th ed. Elk Grove Village, IL: American Academy of Pediatrics; 2012:635-
640.

American academy of pediatrics 632


American Academy of Pediatrics PREP 2015

Item 206
A 4-year-old girl is brought by emergency medical services to your small community emergency
department 30 min after she was struck by a car traveling at approximately 40 mph. The girl ran
out into the street after a rolling soccer ball and was struck by the oncoming car. She was thrown
several feet on impact and had a 1- to 2-min loss of consciousness. Her mother witnessed the
incident and immediately called 911.

On arrival to the emergency department, the paramedics report that the girl's left ankle is
markedly swollen with an obvious deformity, there is a large bleeding laceration over the left
side of her forehead, and there are several bruises across her abdomen.

The girl is moaning with some intermittent gurgling and a small amount of blood coming from
her mouth. She does not answer questions or follow your instructions to move her legs and
squeeze your hands. Paramedics immobilized her entire spine by using a pediatric backboard and
cervical immobilization collar. Her vital signs on arrival to the emergency department show a
heart rate of 160 beats/min, respiratory rate of 40 breaths/min, and a blood pressure of 50/30 mm
Hg.

Of the following, the BEST next step in management is to


A. administer 20 mL/kg of isotonic crystalloid fluid solution intravenously
B. apply a pressure bandage to her left forehead laceration
C. open her airway with a jaw-thrust maneuver
D. perform a chest radiograph
E. transport to a tertiary hospital that has pediatric surgical capabilities

American academy of pediatrics 633


American Academy of Pediatrics PREP 2015

Item 206 TE Preferred Response: C


The young girl in the vignette requires emergent evaluation and management after being struck
by a car. Her clinical findings indicate that she has sustained multiple injuries. The best next step
in her treatment is to open her airway using a jaw-thrust maneuver while maintaining
immobilization of her cervical spine. The inability to establish and maintain a patent airway with
subsequent lack of ventilation and oxygenation is the leading cause of cardiopulmonary arrest in
children. Therefore, the airway is always the first priority in children affected by multisystem
trauma.

Injury results in more deaths in children and adolescents than all other causes combined. An
estimated 1 in 4 children sustains an unintentional injury requiring medical care each year, and
more than 10,000 children in the United States die annually from serious injury. All pediatric
providers must be familiar with the appropriate evaluation of multisystem trauma.

The priorities of assessment and management of injuries in children are the same as those in
adult patients with trauma, though the unique anatomic and physiologic characteristics of
children combine with different injury mechanisms to produce distinct patterns of injury. Failure
to secure the airway, support breathing, and recognize and address hemodynamic instability are
the leading causes of unsuccessful resuscitation in severe pediatric trauma. Consistently using an
efficient, systematic approach in evaluating all children with multisystem trauma-with emphasis
on adequately supporting the airway, breathing, and circulation-can have a significant positive
impact on the ultimate survival and long-term outcomes of these children.

Initial evaluation of all patients with multisystem trauma, including children, should consist of a
rapid primary survey following the A-B-C-D-E sequence, immediate resuscitation if vital
functions are not intact when assessed on the primary survey, a more detailed secondary survey
(which includes a head-to-toe physical examination) following initial resuscitation, and initiation
of definitive care.

During the primary survey phase of the multisystem trauma evaluation, the following elements
must be rapidly and sequentially assessed (and immediately managed, if life-threatening
abnormalities are detected):
1. Airway assessment and maintenance, while maintaining cervical spine immobilization at
all times
2. Breathing and ventilation
3. Circulation with control of bleeding
4. Disability (neurologic status)
5. Exposure/Environmental control (the child's entire body should be exposed by removing
all clothing, while simultaneously taking care to prevent hypothermia)

Regardless of the obvious injuries in a child present-ing with multisystem trauma, the secondary
survey (more detailed, head-to-toe examination) should not begin until the primary survey (using
the ABCDE sequence) has been completed, initial resuscitation to support the ABCDs is
underway, and vital functions are demonstrably stable. In addition to a head-to-toe examination,
the secondary survey should also include a brief medical history and history of the mechanism of
injury for the affected child.
American academy of pediatrics 634
American Academy of Pediatrics PREP 2015

A cervical spine injury should be assumed until proven otherwise in all multiply injured children,
especially those with an altered level of consciousness or evidence of blunt trauma to the head
and neck regions. The entire spine should be protected and immobilization maintained through
the evaluation and management process.

Although isotonic crystalloid fluid solution should be administered to the girl in this vignette
because she is hypotensive after major trauma, assessing and securing the airway should always
be the very first step of the primary survey process for patients with trauma. Likewise, although
bleeding from the left forehead laceration should be controlled during the primary survey, the
airway is the more immediate priority.

Placement of an air splint to support the girl's deformed ankle might be helpful in the overall
management of her condition. However, this should be addressed after the primary survey and
resuscitation to correct more life-threatening problems.
Although this child will indeed require transport to a hospital with pediatric surgical capabilities,
resuscitation efforts must begin immediately, and the girl must be stabilized before she is
transported to a tertiary facility

PREP Pearls
• Inability to establish and maintain a patent airway with subsequent lack of ventilation and
oxygenation is the leading cause of cardiopulmonary arrest in children. The airway is
always the first priority in children affected by multisystem trauma.
• Consistently using an efficient, systematic approach in evaluating all children affected by
multisystem trauma-with emphasis on supporting the airway, breathing, and circulation-
can have a significant positive impact on the ultimate survival and long-term outcomes of
these children.
• Regardless of the obvious injuries in a child presenting with multisystem trauma, the
secondary survey should not begin until the primary survey (using the ABCDE sequence)
has been completed, initial resuscitation to address life-threatening problems is
underway, and stabilization of vital functions has been demonstrated.
• A cervical spine injury should be assumed until proven otherwise in all multiply injured
children. Immobilization of the entire spine should be maintained throughout the
evaluation and initial management process.

American Board of Pediatrics Content Specification(s)


• Plan the appropriate evaluation of multisystem trauma

Suggested Reading
• American Academy of Pediatrics, Pediatric Orthopaedic Society of North America.
Management of pediatric trauma. Pediatrics. 2008;121(4);849-854.
doi:10.1542/peds.2008-0094.
• American College of Surgeons Committee on Trauma. Initial assessment and
management. Advanced Trauma Life Support for Doctors. 8th ed. Chicago, IL: American
College of Surgeons; 2008:148.

American academy of pediatrics 635


American Academy of Pediatrics PREP 2015

• American College of Surgeons Committee on Trauma. Pediatric trauma. Advanced


Trauma Life Support for Doctors. 8th ed. Chicago, IL: American College of Surgeons;
2008:225-240.

American academy of pediatrics 636


American Academy of Pediatrics PREP 2015

Item 207
You are meeting with a first-time mother during her prenatal visit. She tells you that her
pregnancy was unremarkable until 2 months ago, when ultrasonography showed that her
daughter was smaller than expected. Her daughter's growth has been monitored closely since that
time, but continues to be less than anticipated. The mother has had no medical problems during
the pregnancy. She describes smoking 1 pack of cigarettes daily, with no drug or alcohol use.
She did attempt to discontinue smoking, but was unable to stop. She is worried about her
daughter's poor growth and asks if there are any other problems she may face after birth related
to her smoking.

Of the following, you are MOST likely to advise the expectant mother that her daughter is at
increased risk for
A. hearing loss
B. increased attention problems
C. moderate cognitive impairment
D. nicotine withdrawal
E. poor long-term growth

American academy of pediatrics 637


American Academy of Pediatrics PREP 2015

Item 207 Preferred Response: B


Infants whose mothers smoked cigarettes during pregnancy are at an increased risk of attention
problems that extend into adulthood. Epidemiological studies have demonstrated that prenatal
tobacco exposure is associated with many adverse behavioral outcomes that include attention
problems, impulsivity; and hyperactivity. The link between cognition and smoking is much
weaker, with some studies suggesting issues with language, reading, memory, and slightly lower
IQ scores.

Prenatal exposure to nicotine affects the development of neural pathways in the brain. Other
compounds in smoke such as cyanide and bromide may further affect these pathways. These
compounds may play a role in the adverse behavioral outcomes associated with prenatal
smoking. Conversely, an increased risk of congenital abnormalities has not been seen with
prenatal tobacco exposure, with the possible exception of oral facial clefts.

Smoking during pregnancy has been associated with pre-term birth, intrauterine growth
restriction, and low birth weight. The decline in birth weight is correlated to the number of
cigarettes smoked. Prenatal smoking does not appear to impact long-term growth, although some
studies suggest an increased likelihood of childhood obesity, as reflected by a higher body mass
index. Increased rates of sudden infant death syndrome, wheezing, and asthma have also been
demonstrated in infants and children exposed to cigarette smoking during pregnancy. Exposed
newborns do not experience nicotine withdrawal after delivery. Hearing and vision appear
unaffected.

PREP Pearls
• Smoking during pregnancy has been associated with preterm birth, intrauterine growth
restriction, and low birth weight.
• Prenatal tobacco exposure is associated with many adverse behavioral outcomes that
include attention problems, impulsivity, and hyperactivity.
• Increased rates of sudden infant death syndrome, wheezing, and asthma have been
demonstrated in infants and children exposed to cigarette smoking during pregnancy.

American Board of Pediatrics Content Specification(s)


• Recognize the effects of maternal smoking on a fetus

Suggested Reading
• American Academy of Pediatrics, Committee on Substance Abuse, Committee on Fetus
and Newborn. Prenatal substance abuse: short- and long-term effects on the exposed
fetus. Pediatrics. 2013;131(3):e1009-e1024. doi:10.1542/peds.2012-3931.
• Bernstein IM, Mongeon IA, Badger GJ, Solomon L, Heil SH, Higgins ST. Maternal
smoking and its association with birth weight. Obstet Gynecol. 2005;106(5):986-991.
• Bruin JE, Gerstein 1-1C, Holloway AC. Long-term consequences of fetal and neonatal
nicotine exposure: a critical review. Toxicol Sci. 2010,116(2):364374.
doi:10.1093/toxsci/kfq103.
• Morales-Suarez-Varela MM, Bille C, Christensen K, Olsen I. Smoking habits, nicotine
use, and congenital malformations. Obstet Gynecol. 2006;107(1)51-57.
doi:10.1097/01.A0G.0000194079.66834.d5.
American academy of pediatrics 638
American Academy of Pediatrics PREP 2015

Item 208
You are seeing a 4-year-old boy in your clinic for sudden onset of gait imbalance while he was at
preschool. He was playing with other children, when he suddenly could not keep his balance and
sat down. His mother was called to pick him up, and she brought him immediately to your office.
There was no preceding trauma and no recent illness or fever. He denies headache but says he
feels dizzy. On physical examination, his temperature is 37.2°C, blood pressure is 90/56 mm Hg,
heart rate is 96 beats/ruin, and respiratory rate is 18 breaths/min. The boy appears ill, but not
encephalopathic, with pallor and dark circles under his eyes. His neck is supple without
meningismus. The remainder of his physical examination is unremarkable. His neurologic
examination findings are unremarkable. There is no nystagmus. He is able to walk independently
but refuses to take more than 3 steps. While at the office visit, the boy's symptoms improve,
resolving completely by 2 hours after the onset of symptoms.

Of the following, the MOST likely cause of his gait imbalance is


A. acute cerebellar ataxia
B. acute otitis media
C. cerebellar tumor
D. migraine variant
E. vestibular neuritis

American academy of pediatrics 639


American Academy of Pediatrics PREP 2015

Item 208 Preferred Response: D


The boy in the vignette has benign paroxysmal vertigo, a migraine variant. In children with acute
onset vertigo or gait imbalance, with normal findings on tympanic membrane examination, the
neurologic differential diagnosis includes acute cerebellar ataxia, benign paroxysmal vertigo,
brainstem or cerebellar tumor, trauma, or vestibular neuritis. Other causes of gait imbalance
include weakness, intoxication, or metabolic disturbance; there was no indication of any of these
in the history or physical examination. The short duration and normal neurologic examination
findings make benign paroxysmal vertigo the most likely diagnosis in this case.

Acute cerebellar ataxia is characterized by abrupt onset of gait ataxia in a toddler, sometimes
with nystagmus and emesis. Fever, meningismus, and other signs of acute infection are absent.
Symptoms typically last days to weeks and improve spontaneously. The etiology is thought to be
a postviral immunologic reaction. Cerebellar tumors in toddlers present with progressive ataxia
and signs of increased intracranial pressure such as headache and horizontal or vertical gaze
palsy. Common cerebellar tumors in childhood include medulloblastoma, ependymoma, and
astrocytoma. Acute otitis media, caused by viral or bacterial pathogens, is a common cause of
gait imbalance in children. The diagnosis is made when otoscopic examination of the tympanic
membrane shows signs of acute infection. Vestibular neuritis is more common in teenagers, and
is typically caused by viral pathogens, but occasionally bacterial pathogens. Symptoms include
the abrupt onset of vertigo, gait imbalance, or emesis, and may be worsened by abrupt head
movements. Nystagmus, hearing loss, or tinnitus may be present. Symptoms typically last weeks
and resolve spontaneously.

Benign paroxysmal vertigo is treated similarly to migraine. Acetaminophen, ibuprofen, and


antiemetics can be helpful. The symptoms typically resolve with sleep. There may be recurrent
episodes, especially in younger children, but over time the child will often develop symptoms
more typical of migraine headaches.

PREP Pearls
• Benign paroxysmal vertigo is a migraine variant that responds to usual migraine
treatments.
• Imbalance and vertigo in children is commonly caused by middle ear disease.

American Board of Pediatrics Content Specification(s)


• Recognize the clinical findings associated with benign paroxysmal vertigo
• Formulate a differential diagnosis of balance disturbance in patients of various ages
• Recognize the etiology of inner ear infections

Suggested Reading
• Benun J. Balance and vertigo in children. Pediatr Rev. 2011;32(2):84. doi:10.1542/pi
r.32-2 -84 .

American academy of pediatrics 640


American Academy of Pediatrics PREP 2015

Item 209
A 10-year-old, African-American boy presents to your office for a health supervision visit. He is
doing well in school and has no concerns. His review of systems is unremarkable. His mother
has type 2 diabetes mellitus and is worried the child could have it as well. On physical
examination, the child's weight is at the 75th percentile with a body mass index of 21 (90th
percentile). He has no evidence of acanthosis nigricans or any other signs of insulin resistance.

Of the following, the BEST next step in management is to


A. order a 2-hour oral glucose tolerance test because he is overweight and has risk factors to
develop type 2 diabetes mellitus
B. prescribe a home glucometer to be used for intermittent fingerstick glucose checks on the
child
C. reassure the mother that his weight is normal, but he should begin to develop healthy
lifestyle habits
D. reassure the patient's mother that it is rare for a child to develop type 2 diabetes mellitus
E. screen for diabetes mellitus during his teenage years because of his family history

American academy of pediatrics 641


American Academy of Pediatrics PREP 2015

Item 209 Preferred Response: A


The child in the vignette, despite having a weight at the 75th percentile, has a body mass index
(BMI) greater than the 85th percentile. Because the BMI is often overlooked when only height
and weight are measured, the American Academy of Pediatrics recommends the routine use of
BMI to screen for obesity starting at 2 years of age.

Current American Diabetes Association (ADA) guidelines recommend screening for type 2
diabetes mellitus starting at 10 years of age or at onset of puberty, and every 3 years thereafter in
children who are:
 Overweight
o BMI greater than the 85th percentile for age and sex
o weight for height greater than the 85th percentile
o or weight greater than 120% of ideal for height
 and have at least 2 risk factors for diabetes
o Family history of type 2 diabetes mellitus in a first- or second-degree relative
o Higher risk race or ethnicity (Native American, African American, Latino, Asian
American, Pacific Islander)
o Signs of insulin resistance or conditions associated with insulin resistance
(acanthosis nigricans, hypertension, dyslipidemia, polycystic ovary syndrome, or
small-for gestational age birth weight)
o Maternal history of diabetes mellitus or gestational diabetes mellitus during the
child's gestation

This child already has an elevated BMI, a family history of type 2 diabetes mellitus in a first-
degree relative, and is of a higher risk racial background. Therefore screening is recommended.
A fasting glucose level of 126 mg/dL (7.0 mmol/L) or higher leads to a diagnosis of diabetes
mellitus. A 2-hour glucose reading during an oral glucose tolerance test (OGTT) of 200 mg/dL
(11.10 mmol/L) or higher is also diagnostic. In some cases, the fasting plasma glucose level will
be normal, but the postprandial glucose level may be elevated, in which case an OGTT should be
performed if there is a high index of suspicion for type 2 diabetes mellitus. In the absence of
unequivocal symptoms, testing should be repeated to confirm the diagnosis of diabetes mellitus
because there are numerous causes of transient hyperglycemia. However, with unequivocal
symptoms of diabetes mellitus, a random plasma glucose level of 200 mg/dL (11.1 mmol/L) or
higher confirms the diagnosis.

Children who already have evidence of impaired glucose tolerance (fasting glucose, 100-125
mg/dL [5.6-6.9 mmol/L]; 2-hour postprandial glucose on oral glucose tolerance test [OGTT],
140-199 mg/dL [7.8-11.0 mmol/L]) also need more frequent monitoring, with repeat testing
every 1 to 2 years.

Recently, guidelines from the ADA have added hemoglobin Alc as a supplemental screening test
for diabetes mellitus, with a result of 6.5% or greater considered diagnostic of diabetes mellitus
in adults. Although there have been concerns that this screening method would not be very
sensitive or specific in children, recent statements from the Pediatric Endocrine Society Drugs
and Therapeutics Committee now support hemoglobin Alc of 6.5% or greater as a supplemental

American academy of pediatrics 642


American Academy of Pediatrics PREP 2015

screening test in children as well, although there is still considerable controversy concerning its
use.

PREP Pearls
• The American Academy of Pediatrics recommends the routine use of body mass index
(BMI) to screen for obesity in all children beginning at 2 years of age.
• Normal weight children may have an elevated BMI and may require screening for
diabetes mellitus. This will be missed if BMI is not routinely assessed.

American Board of Pediatrics Content Specification(s)


• Understand the utility of the body mass index

Suggested Reading
• American Diabetes Association. Standards of medical care in diabetes-2013. Diabetes
Care. 2013;36 (Suppi 1):S11-S66, doi:10.2337/ dc13-5011.
• Kapadia C, Zeitler P, Drugs and Therapeutics Committee of the Pediatric Endocrine
Society. Hemoglobin Alc measurement for the diagnosis of type 2 diabetes in children.
Int I Pediatr Endocrinol. 2012;2012(1):31. doi:10.1186/1687-9856-2012-31.
• National Center for Chronic Disease Prevention and Health Promotion, Division of
Nutrition, Physical Activity, and Obesity. About BMI for Children and Teens. US
Centers for Disease Control and Prevention.
• Rosenbloom AL, Silverstein JH, Amemiya S. et al. ISPAD Clinical Practice Consensus
Guidelines 2006-2007: type 2 diabetes mellitus in the child and adolescent. Pediatr
Diabetes. 2008;9(5):512-526. doi:10.1111/0399- 5448.2008.00429.x.

American academy of pediatrics 643


American Academy of Pediatrics PREP 2015

Item 210
While examining a full-term newborn, you note that the newborn has sparse hair and lashes, and
marked skin peeling (Item Q210A). He is the product of an uneventful pregnancy and delivery
and has been breastfeeding without difficulty. The mother reports that her brother and nephew
(Item Q210B) also have sparse hair, as well as fragile skin and dental problems.

Of the following, the MOST accurate information you can provide the parents of this newborn is
that
A. avoidance of warm environments is necessary to prevent hyperthermia
B. careful skin lubrication effectively controls the condition
C. the child is unlikely to have an inherited condition as neither parent is affected
D. the condition typically resolves without treatment by age 5 years
E. the only associated abnormalities are related to skin and hair

American academy of pediatrics 644


American Academy of Pediatrics PREP 2015

Item 210 TE Preferred Response: A


The newborn in the vignette has findings consistent with ectodermal dysplasia (ED), including
sparse hair and extensive skin peeling. These newborns have poorly functioning sweat glands,
and therefore are at risk for developing hyperthermia when exposed to excessively warm
environments. Therefore, the most accurate and urgent advice for this family is to provide a safe
thermal environment for their newborn.

There are more than 150 identified forms of ectodermal dysplasia, all of which affect at least 2 of
3 ectodermally derived structures including skin, teeth, and appendages (hair, nails, eccrine and
sebaceous glands). The most common form is hypohidrotic ectodermal dysplasia (HED), which
is most often an X-linked mutation of the EDA gene. Less commonly, HED can be the result of
either autosomal recessive or dominant mutations of the EDAR or EDARRADD genes. The
most common clinical findings of HED are the presence of sparse, lightly pigmented scalp hair,
reduced sweat function, and late appearance of abnormally formed, conical teeth. In addition,
these children develop periorbital hyperpigmentation, saddle nose deformity, frontal bossing,
malar hypoplasia, recessed columellae, prominent low-set ears, concretions of nasal and aural
secretions, absence of dermal ridges, and raspy voice. They typically have normal growth and
development, although up to 20% of children with ED may develop failure to thrive as a result of
gastroesophageal reflux. Usually axillary and pubic hair growth is preserved.

Many anatomic and functional abnormalities have been associated with various forms of
ectodermal dysplasia. Atopic disease is more common in children with ED with one study
demonstrating an increased incidence of asthma, food allergy, and immunodeficiency compared
with the general population. Various syndromes that include ED as one component also have
congenital anomalies such as cleft lip and palate. Vision and hearing are frequently affected.
Skin lubrication is an important component of the supportive care for this condition, but it is only
a part of the comprehensive care needed by these children. They also need psychosocial support,
diligent dental and otolaryngology care, and strict attention to hydration and environmental
temperature. This level of care will be lifelong because the condition does not resolve. The most
common inheritance pattern for HED is X-linked recessive, therefore the parents are unlikely to
have major manifestations of the condition.

PREP Pearls
• Ectodermal dysplasia is characterized by sparse scalp hair, impaired sweating, late
developing conical teeth, periorbital hyperpigmentation, flattened midline facial
structures, and concretions of nasal and aural secretions.
• Because of poor eccrine sweat gland function, these children are susceptible to
hyperthermia, and careful attention to a safe thermal environment is critical.
• There are over 150 forms of ectodermal dysplasia; hypohidrotic ectodermal dysplasia is
the most common form and is most often inherited in an X-linked recessive manner.
• Ectodermal dysplasia is a component of various syndromes and may be accompanied by
conditions such as cleft lip and palate, and hearing and vision impairment.

American Board of Pediatrics Content Specification(s)


• Recognize the clinical findings associated with ectodermal dysplasia

American academy of pediatrics 645


American Academy of Pediatrics PREP 2015

Suggested Reading
• Mark BJ, Becker BA, Halloran DR, et al. Prevalence of atopic disorders and
immunodeficiency in patients with ectodermal dysplasia syndromes. Ann Allergy
Asthma Immunol. 2012;108(6):435-438. doi:10.1016/j. anai.2012.04.010.
• Morelli JG. Ectodermal dysplasia. In: Kliegman RM, Stanton BMD, St. Geme JW III,
Schor NF, Behrman RE, eds. Nelson Textbook of Pediatrics. 19th ed. Philadelphia, PA:
Saunders Elsevier; 2011:2222-2223.
• Wright JT, Grange DT, Richter MR. Hypohidrotic ectodermal dysplasia. GeneReviews.
2013.

American academy of pediatrics 646


American Academy of Pediatrics PREP 2015

Item 211
You are supervising a resident that has just seen a 17-year old adolescent girl for a routine
wellness visit. The resident is concerned that the girl kept her sunglasses on throughout the visit,
and he noted some bruises on her arm. He reports that she has a sexual partner and that they use
condoms regularly. The girl indicated that everything was going well in her life except on
occasion her partner demands to see her cell phone messages. You ask him if he inquired about
possible violence at the hands of her partner. He chose not to address this, as he was afraid it
might exacerbate the situation. You discuss screening for intimate-partner violence with the
resident.

Of the following, the BEST advice in this scenario would be to


A. discuss intimate-partner violence if the patient invites a discussion
B. inform the patient's parents of your concern after she leaves your office
C. refer the patient for mental health screening
D. screen all adolescents for intimate-partner violence and consider involving the parents
E. target screening to patients with clinical evidence of physical abuse

American academy of pediatrics 647


American Academy of Pediatrics PREP 2015

Item 211 I-C P S Preferred Response: D


Dating or relationship violence in adolescents refers to emotional, verbal, physical, or sexual
abuse among romantic and sexual partners. This victimization may antedate intimate partner
violence among adults and early recognition therefore becomes critical in this population. The
American Academy of Pediatrics emphasizes the importance of screening and counseling
adolescents for interpersonal and domestic violence in a culturally sensitive and supportive
manner. The attitude and approach of the screener are important to disclosure. If the adolescent
indicates that parents will be supportive and wishes for them to be brought into the discussion,
then it is permissible to do so.

Because a trusting relationship is extremely important in the provision of further care, discussing
concerns with parents without first getting permission from the adolescent is not recommended.
One should have an understanding of the safety of the home environment and whether the
parents' response will be supportive before deciding on disclosure. As this violence involves
more than physical abuse, many cases would be missed if screening were limited to those with
physical signs of abuse. For example, abusive partner behavior often starts with controlling
activities such as monitoring cellphone use and stalking through social networks. Most
adolescents will not disclose information on violence if not asked. Asking allows the adolescent
to see this as unacceptable behavior and that help can be obtained. Omitting screening for
intimate-partner violence, as with all high-risk behaviors, enables escalation of the activity in the
future. It is important when dealing with adolescents to allocate sufficient time during a visit to
screen them comprehensively.

PREP Pearls
• Dating or relationship violence in adolescents refers to emotional, verbal, physical, or
sexual abuse among romantic and sexual partners.
• It is important to screen all adolescents for relationship violence, but care should be taken
to investigate the availability of social support before disclosure of this abuse to a family
member.

American Board of Pediatrics Content Specification(s)


• Recognize and apply ethical principles regarding the issue of intimate-partner violence

Suggested Reading
• Exner-Cortens D, Eckenrode J. Rothman E. Longitudinal associations between teen
dating violence victimization and adverse health outcomes. Pediatrics. 2013;131(1):71-
78. doi:10.1542/peds.2012-1029.
• Institute of Medicine, National Academy of Sciences. Clinical preventive services for
women: closing the gaps. Institute of Medicine website.
• Liebschutz JM, Rothman EF. Intimate-partner violence: what physicians can do. N Engl J
Med. 2012;367(22):2071-2073. doi:10.1056/NEJMp1204278.
• Lin AJ, Raymond M, Catallozzi M, Ryan 0, Rickert VI. Relationship violence in
adolescents. Adolesc Med State Art Rev. 2007;18(3):530-543.
• Sikes A, Walley C, Hays DG. A qualitative examination of ethical and legal
considerations regarding dating violence. J lnterpers Violence.
• 2012;27(8):1474-1488. doi:10.1177/0886260511425791.
American academy of pediatrics 648
American Academy of Pediatrics PREP 2015

• Silverman JG, Raj A, Clements K. Dating violence and associated sexual risk and
pregnancy among adolescent girls in the United States. Pediatrics. 2004;114(2):e220-
e225. Accessed February 21, 2014.
• Thackeray JD, Hibbard R, Dowd MD, the Committee on Child Abuse and Neglect, the
Committee on Injury, Violence, and Poison Prevention. Clinical report: intimate partner
violence: the role of the pediatrician. Pediatrics. 2010;125(5):1094-1100.
doi:10.1542/peds.2010-0451.

American academy of pediatrics 649


American Academy of Pediatrics PREP 2015

Item 212
A 1-year-old infant is brought to your office for evaluation. The parents state that the infant was
"born with asthma." He has been noted to have wheezing for the past year, wheezing once to
multiple times a day. The wheezing is occasionally related to exertion and to a full feed. He has
been prescribed 2 puffs of a short-acting β-agonist with a spacer for these episodes, but the
parents do not use it consistently, as it seems to help him only when he has an underlying cold.
He was noted to spit up when he was younger, but this has improved since he began ambulating.
He has never had to seek urgent treatment for his wheezing and has never received treatment
with steroids. His height and weight are at the 50th percentile. His physical examination is
unremarkable and his lungs are clear on examination.

Of the following, the MOST appropriate test to order is


A. allergy testing
B. barium swallow
C. echocardiogram
D. pH probe
E. sweat chloride

American academy of pediatrics 650


American Academy of Pediatrics PREP 2015

Item 212 Preferred Response: B


The appropriate test to order for the infant in this vignette is a barium swallow in order to
evaluate for the possibility of gastroesophageal reflux and congenital abnormalities of the
tracheobronchial tree and thoracic structures as the cause of the wheezing. Infantile wheezing
that does not respond to inhaled bronchodilators and occurs daily, rather than intermittently,
points to etiologies other than asthma. Barium swallow may also help in identifying swallowing
dysfunction, aspiration syndromes, and some cases of tracheoesophageal fistula. The history of
his symptoms being related to feeding, on occasion, and improving with ambulation suggests
resolving gastroesophageal reflux.

The etiology of wheezing in infants and young children can be approached clinically based on
severity of onset and mechanism of airway narrowing (Item C212). New onset acute wheezing
may suggest asthma, infection, or sudden airway obstruction. Chronic or episodic wheezing may
be caused by congenital abnormalities, cardiac disease, aspiration syndromes (such as
gastroesophageal reflux and dysphagia), vocal cord dysfunction, impaired immunologic
defenses, and underlying pulmonary disorders. Localized or monophonic wheezing, croup-like
cough or stridor, recurrent pneumonias, feeding difficulties, and lack of response to
bronchodilator therapy indicate a cause of wheezing other than asthma. A chest radiograph
should be considered as an initial test in children with new onset wheezing of undetermined
etiology or chronic persistent wheezing not responding to therapies. Apart from providing a good
image of the large airways, it can help differentiate between diffuse and focal disease. The
presence of generalized hyperinflation implies diffuse air trapping and airway disease (eg,
asthma, cystic fibrosis, primary ciliary dyskinesia, and aspiration), while localized findings
suggest structural abnormalities or foreign body aspiration. A chest radiograph may reveal
parenchymal lung disease, atelectasis, and possibly bronchiectasis, as well as cardiomegaly,
enlarged pulmonary vessels, pulmonary edema, or other signs of cardiac failure. They can also
detect the presence of mediastinal masses or enlarged lymph nodes, and may suggest the
presence of vascular rings (eg, right-sided aortic arch).

The pH probe may assist in the establishment of a diagnosis of gastroesophageal reflux.


However, it is challenging to perform in an outpatient setting and would not pick up intrathoracic
congenital abnormalities, such as a vascular ring, which are important causes of wheezing in an
infant unresponsive to bronchodilators. While "cardiac asthma" can present with daily wheezing,
particularly related to exertion and feeding, the chronicity of the symptoms, as well as the
absence of other physical findings suggestive of underlying cardiac disease and cardiac failure,
make it less likely in this case. An echocardiogram is therefore not the best choice in this
scenario. Cystic fibrosis can present with respiratory symptoms such as recurrent wheezing. The
absence of diarrhea, nasal polyps, clubbing, and growth parameters that are reassuringly in the
50th percentile make cystic fibrosis less likely. A sweat chloride test would therefore not be the
most appropriate test at this time. A positive allergy test may provide supplemental information
regarding the presence of sensitization and, as a criterion of the asthma predictive index, predict
the risk of persistence of wheezing beyond 5 years of age. However, viral respiratory infections
are more common triggers of asthma and reversible wheezing in infants and young children than
allergens; therefore, allergy testing would not be routinely indicated.

American academy of pediatrics 651


American Academy of Pediatrics PREP 2015

PREP Pearls
• Presence of wheeze from birth or perinatal period, or wheezing that does not respond to
bronchodilators, occurs daily, varies with position or feeding, and is associated with
minimal cough should prompt consideration of diagnoses other than asthma.
• A chest radiograph may serve as an initial screening test to evaluate an infant or young
child with persistent wheezing unresponsive to bronchodilators.

American academy of pediatrics 652


American Academy of Pediatrics PREP 2015

• Congenital structural abnormalities of the tracheobronchial tree and intrathoracic


structures can mimic asthma by presenting as wheezing in infants and young children.

American Board of Pediatrics Content Specification(s)


• Plan the appropriate clinical and diagnostic evaluation of wheezing of various etiologies
• Plan appropriate management for wheezing of various etiologies

Suggested Reading
• Chipps BE. Evaluation of infants and children with refractory lower respiratory tract
symptoms. Ann Allergy Asthma Immunol. 2010;104(41:279-283.
doi110.1016/j.anai.2009.11.002.
• National Heart, Lung, and Blood Institute. National asthma education and prevention
program expert panel report III: guidelines for the diagnosis and management of asthma.
National Institutes of Health, US Department of Health and Human Services website.
• Park MK. Congenital heart defects. In: Park MK, ed. The Pediatric Cardiology
Handbook. 4th ed. Philadelphia, PA: Mosby Elsevier; 2010:93-181,

American academy of pediatrics 653


American Academy of Pediatrics PREP 2015

Item 213
A 1-week-old patient is brought to the pediatric clinic by her mother for concerns that, since
birth, the newborn is breathing too fast and intermittently stops breathing. The mother states that
her newborn does not seem to be in any distress between the episodes of fast breathing. These
periods last as long as 5 seconds, at which point she will resume breathing spontaneously at a
rapid rate and then slow to a more normal rate. These episodes occur more often when she is
asleep and are not associated with any color change. The newborn is a product of a full-term,
uncomplicated pregnancy and delivery. She went home with the mother on day 2 after birth. She
takes a 3-oz bottle of formula every 3 hours and usually makes 4 wet and 2 dirty diapers every
day. She does not have any vomiting. Her growth parameters are normal.

Vital signs show a temperature of 37°C, heart rate of 140 beats/min, respiratory rate of 45
breaths/min, and blood pressure of 80/40 mm Hg. Pulse oximetry shows oxygen saturation of
100% on room air. Physical examination shows a well-developed, well-nourished baby. Anterior
fontanelle is open, soft, and flat. Pupils are equal and reactive. The newborn has moist and
nonerythematous eye, oral, and nasal mucous membranes. Cardiovascular examination reveals a
regular rate and rhythm and a soft, grade 1/6 systolic ejection murmur. She has warm and well-
perfused extremities, with strong and equal femoral pulses. She is breathing comfortably at a
timed rate of 45 breaths/min. She has no notable thoracic deformities. During inspiration, her
chest moves in, while her abdomen protrudes. Lungs are clear to auscultation with good air entry
bilaterally. Abdomen is soft, non-tender, and nondistended with no enlarged organs.

Of the following, the MOST appropriate course of action is to


A. admit to hospital for observation
B. obtain capillary blood gas
C. obtain chest radiograph
D. reassure the mother
E. refer to pediatric cardiologist

American academy of pediatrics 654


American Academy of Pediatrics PREP 2015

Item 213 Preferred Response: D


The newborn in the vignette is exhibiting periodic breathing, which is a normal respiratory
pattern in healthy full-term newborns in the first month after birth and the MOST appropriate
course of action is to reassure the mother. Periodic breathing is characterized by normal rhythmic
breathing interrupted by brief (< 10 seconds) intermittent periods of respiratory pause,
alternating with fast breathing. These episodes, which resolve in the first few months after birth,
are not associated with distress or altered gas exchange, and thus require no further evaluation or
treatment.

Regulation of respiration is a complex process involving central respiratory controllers, sensors,


receptors, and effectors. Voluntary control of respiration, which requires some level of
consciousness, originates in the cerebral motor cortex. Subconscious control of respiration
occurs in the pre-Botzinger complex (preBotC) in the brainstem, and controls the automaticity of
breathing much in the same way as does the pacemaker of the heart. This group of neurons is
also responsible for various patterns of breathing, including sighing and gasping. Other nuclei in
the pons, including the apneustic center and the pneumotaxic center, are responsible for
regulating the preBotC. Derangements in the fine tuning of these mechanisms of respiratory
regulation can result in (A) Cheyne-Stokes respiration, which are periods of apnea alternating
with hyperventilation, (B) apneustic breathing, characterized by prolonged inspiratory gasps, or
(C) generally shallow respirations and hyperventilation, as is seen with global central nervous
system dysfunction. The stimulus to breathe is also dependent on central chemoreceptors, which
respond with exquisite sensitivity to changes in cerebrospinal pH, and peripheral chemoreceptors
in the carotid bodies, which respond to changes in the arterial pressure of oxygen (PaO2). These
receptors lead to an increase in respiratory stimuli in metabolic acidosis and hypoxia, and
prevent hypoventilation. Lastly, stretch receptors in the lungs feed back to the controllers in the
brain to breathe more shallowly and rapidly in conditions of decreased compliance, such as
pulmonary edema, interstitial lung diseases, and pneumonia.

The clinical sign of tachypnea carries a wide differential diagnosis, and depends greatly on age,
premorbid conditions, and scenario. Tachypnea has both pulmonary and nonpulmonary causes.
Cardiac disease in children commonly presents with respiratory signs and symptoms. Left
ventricular failure and resulting left atrial hypertension and dilation can lead to pulmonary edema
and increased alveolar and interstitial lung water. This leads to decreased lung compliance and
rapid shallow breathing, and sometimes "cardiac wheezing" caused by the extrinsic compression
of small airways by interstitial edema. Metabolic acidosis from decreased cardiac output can
decrease the pH of the cerebrospinal fluid, which causes further tachypnea. If a cardiac cause of
tachypnea is suspected, the evaluation can include chest radiography, echocardiography, or blood
gas measurement where indicated. Metabolic acidosis from other causes, for example diabetic
ketoacidosis or severe diarrhea, can lead to so-called Kussmaul breathing, which is rapid and
deep.

Early manifestations of sepsis and other inflammatory conditions include tachycardia and
tachypnea. Other toxic, metabolic, and CNS conditions can also cause tachypnea, either through
metabolic acidosis or derangement of regulation of respiration, such as aspirin ingestion,
hyperammonemia, liver disease, or global CNS dysfunction. These conditions can be
investigated with measurement of electrolytes, ammonia level, and liver function, or
American academy of pediatrics 655
American Academy of Pediatrics PREP 2015

neuroimaging where indicated. Pulmonary conditions of decreased lung compliance, such as


pneumonia, alveolar hemorrhage, pulmonary edema, interstitial fibrosis, and acute respiratory
distress syndrome, usually cause rapid shallow breathing. Plain radiography can be performed to
elucidate lung pathology in these conditions. Lastly, tachypnea is an early sign of pulmonary
embolism, which can be a life-threatening condition. In older children and others with risk
factors for thrombosis, a high index of suspicion must be maintained. Rapid sequence computed
tomography with contrast is the best initial test if pulmonary embolism is suspected.

The newborn in this vignette has periodic breathing, as opposed to one of the aforementioned
disorders because she was born at full term, and is otherwise healthy and neurologically normal.
In addition, the breathing is not associated with distress or any change in color that would
indicate abnormal gas exchange. A soft-grade 1/6 systolic ejection murmur is likely normal in
the newborn period, as is the breathing pattern of the chest moving in while the abdomen
protrudes. Thus, pulmonary conditions such as pneumonia, pulmonary hemorrhage, or
pulmonary edema are not likely, so chest radiography or a capillary blood gas measurement is
not indicated. Cardiac conditions are not likely because the extremities are well perfused and
pulses are equal, so neither observation in the hospital nor referral to a cardiologist is warranted.
The complicated mechanisms of respiratory regulation are not yet mature in normal, full-term
newborns, and asymptomatic periodic breathing does not require evaluation or treatment.

PREP Pearls
• Periodic breathing occurs in normal newborns, and consists of periods of respiratory
pause lasting 10 seconds or less, followed by episodes of tachypnea.
• Tachypnea can be caused by pulmonary conditions or nonpulmonary conditions, such as
central nervous system, cardiac, inflammatory, or metabolic conditions.
• Cardiac disease in children often manifests with respiratory signs and symptoms.

American Board of Pediatrics Content Specification(s)


• Recognize normal breathing patterns in patients of various ages
• Recognize the various factors that influence respiratory rate
• Plan the appropriate clinical and diagnostic evaluation of tachypnea of various etiologies

Suggested Reading
• Bloomfield D. Tachypnea. Pediatr Rev. 2002;23(8):294-295. doi:10.1542/ pir.23-8-294.
• Carlo WA. Respiratory tract disorders. In: Kliegman RM, Stanton BF, St. Geme JW III,
Schor NF, Behrman RE, eds. Nelson's Textbook of Pediatrics. 19th ed. Philadelphia, PA:
Saunders Elsevier; 2011:579-580_
• Carroll JL, Marcus CL, Loughlin GM. Disordered control of breathing in infants and
children. Pediatr Rev. 1993;14(2):51-65. doi:10.1542/pir.14-2-51.
• Sarnaik AP, Heidemann SM. Regulation of respiration. In: Kliegman RM, Stanton BF,
St. Geme JW III, Schor NF, Behrman RE, eds. Nelson's Textbook of Pediatrics. 19th ed.
Philadelphia, PA: Saunders Elsevier; 2011:1421.

American academy of pediatrics 656


American Academy of Pediatrics PREP 2015

Item 214
A 15-year-old patient in your practice has a 10-month history of fatigue, headaches, and diffuse
achiness. There has been no night sweats, fever, rash, or joint swellings noted. They live in an
urban area in the northeastern United States, and there has been no history of travel.
Physical examination shows no abnormalities. Temperature is 37.1°C, pulse rate is 76 beats/min,
blood pressure is 112/68 mm Hg, and respiratory rate is 18 breaths/min.
Laboratory results show the following:
• White blood cell count, 9,700/µL (9.7 x 109/L), with 55% neutrophils, 38% lymphocytes,
6% monocytes, and 1% eosinophils
• Erythrocyte sedimentation rate, 3 mm/h
• C-reactive protein, 0.10 mg/L (0.95 nmol/L)
• Epstein-Barr virus serology
 Viral capsid antigen (VCA) Immunoglobulin G, positive
 VCA Immunoglobulin M, negative
 Epstein-Barr nuclear antigen, positive
• Lyme enzyme immunosorbent assay, 1.27 (negative, < 0.91; equivocal, 0.91-1.09;
positive, > 1.09)

Of the following, the BEST next step in evaluating this patient's condition is to
A. begin treatment with doxycycline
B. explain that Epstein-Barr virus infection can cause a false-positive Lyme serology
C. obtain a Western blot for Lyme disease
D. perform a lumbar puncture to evaluate for central nervous system Lyme disease
E. repeat the enzyme immunosorbent assay for Lyme

American academy of pediatrics 657


American Academy of Pediatrics PREP 2015

Item214 PBLI Preferred Response: C


The diagnosis of Lyme disease requires an understand-ing of the individual's exposure to the
causative agent (Borrelia burgdorferi) or epidemiological risk, the likelihood the symptoms are
consistent with Lyme disease, and only then, the results of laboratory testing.

Laboratory testing for Lyme disease consists of an initial screening test (Lyme enzyme
immunosorbent assay [EIA) or fluorescent antibody [FA] test) that, if positive or equivocal,
should be evaluated by a confirmatory test (Lyme Western blot). The advantage of a screening
test is that it is relatively rapid and inexpensive and sufficiently specific that a negative test
implies absence of the condition, which in this vignette is Lyme disease. A positive test is,
however, not sufficiently sensitive for the condition (ie, there may be a false-positive result) and
should be confirmed with the more sensitive Western blot. The Western blot for Lyme antibodies
is not more specific than the screening tests. A similar sequence of testing is used in HIV testing
with a screening rapid, inexpensive EIA test that, if positive or equivocal, is confirmed with a
more sensitive assay (HIV Western blot or nucleic acid detection test).

In this case, the child lives in an urban area and has not described travel that would potentially
expose him to ticks that can transmit Lyme disease. Additionally, the findings of fatigue,
headache, and diffuse achiness in the absence of erythema migrans, migratory large joint
arthritis, Bell palsy, or other objective clinical features of Lyme disease are not specific for this
condition.

Obtaining a Western blot for Lyme antibodies would be the next step in addressing the
significance of the positive Lyme EIA result. If it were negative, additional evaluation or
treatment for Lyme disease would not be indicated in this case. Beginning doxycyline at this
time would not be indicated given the uncertainty of the diagnosis. Acute EBV infection may
cause a false-positive Lyme EIA result, but the serology in this case points to past, not active,
EBV infection. Without further evidence of objective neurological findings suggestive of Lyme
disease (eg, cranial neuropathy, paresthesias, aseptic meningitis) or a confirmed serology, a
lumbar puncture is not indicated in this case. Repeating the same test is not likely to determine
the validity of the result.

In addition to appreciating the sensitivity and specificity of a test, it is important to understand


the concept of a predictive value in interpreting a test result. The positive predictive value
represents the likelihood that a patient with a positive test has the condition and the negative
predictive value that he does not have the disease. These findings depend on the prevalence of
the condition, as well as the sensitivity and specificity of the test,

PREP Pearls
• A good screening test is a rapid, inexpensive assay and has a good specificity (ie, a
negative test is reliable). A positive or equivocal test should generally be confirmed with
a more sensitive confirmatory assay. Testing for Lyme or HIV antibodies employs such a
system with a screening enzyme immunosorbent assay as the first line evaluation,
followed by more sensitive testing of a positive or equivocal result.

American academy of pediatrics 658


American Academy of Pediatrics PREP 2015

• The positive predictive value represents the likelihood that a patient with a positive test
has the condition and the negative predictive value that he does not have the disease.
These findings depend on the prevalence of the condition, as well as the sensitivity and
specificity of the test.

American Board of Pediatrics Content Specification(s)


• Understand positive and negative predictive values
• Understand sensitivity and specificity and how to apply them to test results

Suggested Reading
• Carvajal DN, Rowe PC. Research and statistics: sensitivity, specificity, predictive values,
and likelihood ratios. Pedia tr• Rev. 2010;31(14:511-513. doi:10.1542/pir.31-12-511.
• Neuspiel DR. Weighing the evidence: positive predictive values. AAP Grand Rounds.
2003:10(3):38. http://aapgrandrounds.aappub/ications.org/ content/10/3/38.extract.
• Moyer V. Weighing the evidence: SpPl n and SnNOut. AAP Grand Rounds.
2003;9(6):65. http://aapgrandrounds.aappublications.org/ content/9/6/65.1,full.

American academy of pediatrics 659


American Academy of Pediatrics PREP 2015

Item 215
A 10-year-old girl presents to your office with complaints of dysuria and gross hematuria for the
last 2 days. She describes her urine as "red with blood clots in it" She denies fever, frequency, or
flank pain. She has a temperature of 38.5°C, a respiratory rate of 16 breaths/min, heart rate of 74
beats/min, and blood pressure of 98/62 mm Hg. Her physical examination is significant only for
mild suprapubic pain with no findings of flank pain, organomegaly, guarding, or rebound
tenderness. Her growth parameters are normal.

Of the following, the pathogen that is MOST commonly associated with the girl's symptoms is
A. adenovirus
B. Candida species
C. Enterococcus species
D. Escherichia coil
E. Staphylococcus saprophyticus

American academy of pediatrics 660


American Academy of Pediatrics PREP 2015

Item 215 Preferred Response: D


Bright red hematuria is usually indicative of lower urinary tract bleeding, whereas glomerular
hematuria (as in nephritis) is usually described as cola, tea, or brown colored. An underlying
cause is more frequently identified in patients presenting with gross hematuria. A detailed
history, physical examination, and urinalysis usually provide dues to the underlying cause of
gross hematuria.

The presence of blood clots with dysuria indicates cystitis in the patient in the vignette. Cystitis,
or inflammation of the urinary bladder, may occur alone (uncomplicated cystitis) or in
association with pyelonephritis (complicated cystitis). One of the most important risk factors for
the development of acute cystitis, especially in female patients, is sexual inter-course. Risk
factors for cystitis along with pyelonephritis include anatomic or physiologic abnormalities
associated with incomplete bladder emptying, urinary catheters, and associated diagnoses such as
malignancy or diabetes.

Organisms from the intestinal tract that colonize the periurethral area and urinary tract (eg,
bacteria, fungi, viruses, parasites) are the usual etiologic agents for cystitis. Gram-negative
bacteria, such as Escherichia tali, account for nearly 90% of uncomplicated cystitis cases and
would be the most likely pathogen for the girl in this vignette. Clean-catch or catheterized urine
specimen should be obtained for culture prior to initiation of antibiotic therapy.
Uncomplicated hemorrhagic cystitis, reported with adenovirus infection, is not common in
children. Staphylococcus saprophyticus, Enterococcus, and Candida are also rare etiologic agents
of cystitis.

Children with asymptomatic gross hematuria usually require a detailed evaluation to determine
the underlying cause. The workup in such cases includes urinalysis (to differentiate between red
blood cells and heme pigments), urine calcium-creatinine ratio (hypercalciuria), serum creatinine
and C3 (nephritis), renal ultrasonography (congenital anomalies), and hemoglobin
electrophoresis (sickle cell disease).

PREP Pearls
• Bright red hematuria with dysuria, with or without the presence of blood clots, indicates
cystitis.
• Cystitis, or inflammation of the urinary bladder may occur alone (uncomplicated cystitis)
or in association with pyelonephritis (complicated cystitis).
• Gram-negative bacteria, such as Escherichia coil, account for nearly 90% of
uncomplicated cystitis cases.

American Board of Pediatrics Content Specification(s)


• Formulate a differential diagnosis of gross hematuria

Suggested Reading
• Azzarone G, Liewehr S, O'Connor K. Cystitis. Pediatr Rev. 2007;28:474-476.
doi:10.1542/pir.28-12-474

American academy of pediatrics 661


American Academy of Pediatrics PREP 2015

• Reidy KJ, Rio MD. In: Hematuria. American Academy of Pediatrics Textbook of
Pediatric Care. 19th ed. Mclnerny TIC, Adam HM, Campbell DE, Kamat DP, Kelleher
KJ, eds. Elk Grove Village, IL: American Academy of Pediatrics; 2009;1566-1570.
• Subcommittee on Urinary Tract Infection, Steering Committee on Quality Improvement
and Management. Urinary tract infection: clinical practice guideline for the diagnosis and
management of the initial UTI in febrile infants and children 2 to 24 months. Pediatrics.
2011;128(3):595. doi:10.1542/peds.2011-1330.

American academy of pediatrics 662


American Academy of Pediatrics PREP 2015

Item 216
A 9-year-old boy is brought to your office by his parents, who are worried about his nutritional
status. The child was diagnosed with autism spectrum disorder at age 4 years. During his last
office visit 14 months ago, he weighed 24 kg and his body mass index was approximately 17.
Over the past year, he has frequently refused to eat meals, and he avoids solid foods. His diet is
composed of mostly water, apple juice, and some mashed potatoes. His current body mass index
is 15. You recommend an increase in his daily energy intake, to include a liquid nutritional
supplement. He returns to the office 2 weeks later, having lost an additional kg of body weight.
On physical examination, he appears cachectic. Vital signs show a resting heart rate of 55
beats/min and a blood pressure of 80/55 mm Hg. You elect to admit him to the hospital for
nutritional rehabilitation. Initial laboratory studies include the following:

• Complete blood cell count, within normal limits


• Sodium, 139 mEq/L (139 mmol/L)
• Potassium, 3.5 mEq/L (3.5 mmol/L)
• Chloride, 108 mEq/L (108 mmol/L)
• Bicarbonate, 22 mEq/L (22 mmol/L)
• Glucose, 95 mg/dL (5.3 mmol/L)
After discussing the situation with his parents, you begin nasogastric feeding, which initially
provides 2,100 kcal per day. Three days after commencing this regimen, he appears weak with
difficulty getting out of bed. Laboratory studies are ordered, and these demonstrate the
following:
• White blood cell count, 3,500/µL (3.5 x 109/L)
• Hemoglobin, 13.5 g/dL (135 g/L)
• Sodium, 135 mEq/L (135 mmol/L)
• Potassium, 2.8 mEq/L (2.8 mmol/L)
• Chloride, 105 inEq/L (105 mrnol/L)
• Bicarbonate, 22 mEq/L (22 mmol/L)
• Calcium, 7.5 mg/dL (1.88 mmol/L)
• Phosphorus, 1.8 mg/dL (0.58 mmol/dL)
• Magnesium, 1.0 mEq/L (0.5 mmol/L)
• Glucose, 115 mg/dL (6.4 mmol/dL)

Of the following, you are MOST likely to recommend


A. continuous enteral feedings
B. increasing calorie intake to 2,500 kcal per day
C. reducing calorie intake to 900 kcal per day
D. supplementing enteral feedings with calcium and electrolytes
E. total parenteral nutrition

American academy of pediatrics 663


American Academy of Pediatrics PREP 2015

Item 216 Preferred Response: C


The boy described in the vignette manifests protein-energy malnutrition secondary to an autism-
related behavioral eating disorder. He must be hospitalized because of continued weight loss and
physical findings that indicate severe malnutrition. The boy has started receiving enteral feedings
via the nasogastric route with a total intake of 2,100 kcal/per day. This level of energy
consumption represents the approximate daily recommended intake for an ambulatory 50 kg
individual (when using the calculation of 100 kcal/kg for the initial 10 kg body weight, 50
kcal/kg for the second 10 kg, and 20 kcal/ kg for each kg of body weight above 20 kg). However,
3 days after instituting this regimen, he develops muscle weakness and fatigue, along with
hypocalcemia, hypokalemia, hypomagnesemia, and hypophosphatemia. These findings suggest
the development of refeeding syndrome, which is the consequence of overly aggressive
nutritional management of the malnourished patient. Feedings should be introduced more slowly
in this clinical setting. Following correction of electrolyte deficits, feedings should be restarted at
a rate that provides about 50% of targeted intake. Using a calculation based upon this child's
"normally nourished" weight 14 months ago, he should receive approximately 900 kcal per day.
Refeeding syndrome is a well-described phenomenon that develops as a consequence of specific
metabolic events that accompany aggressive energy supplementation. Patients particularly at risk
include those with the following conditions:
• Anorexia nervosa
• Kwashiorkor or marasmus
• Chronic malnutrition
• Chronic alcoholism
• Prolonged fasting
• Prolonged intravenous hydration
• Metabolic stress (eg, chronic disease) and nutrient depletion

During periods of starvation, insulin levels fall, promoting release of glucose and free fatty acids
for energy. Thyroid hormone levels also are reduced resulting in a lower metabolic rate, and
insulin like growth factor 1 levels fall, reducing protein synthesis. When glycogen stores are
exhausted, upregulation of gluconeogenesis from protein catabolism leads to water, vitamin, and
mineral depletion. A sudden, marked increase in carbohydrate intake, as with refeeding, will
raise serum insulin and lower serum glucagon levels.

Increased cellular glucose uptake then promotes intracellular movement of phosphate, potassium,
and magnesium, while increased utilization of vitamins (including thiamine) and adenosine
triphosphate lead to a deficiency state. Clinically, multiple organ systems may be affected,
resulting in muscle weakness, seizures, cardiac arrhythmias, hypotension, and ileus. To avoid the
refeeding syndrome, nutritional rehabilitation of patients who are at high risk for refeeding
syndrome should proceed as follows:
1. Correct electrolyte abnormalities.
2. Institute oral or enteral nutrition.
a. Start at a calculated intake of about 50% targeted energy consumption.
b. Provide vitamin and trace mineral supplements to meet age-appropriate
recommended daily intakes.
c. Advance energy and volume intake slowly.
3. Closely monitor vital signs, intake and output, and serum electrolyte levels.
American academy of pediatrics 664
American Academy of Pediatrics PREP 2015

The decision to institute bolus versus continuous feedings via the enteral route will depend on
the patient's clinical status and gastrointestinal motility. Severely malnourished subjects may
benefit from a slow, continuous infusion rate to avoid problems created by rapid hormonal
changes, leading to marked fluid and electrolyte shifts. Severe malnutrition is frequently
associated with electrolyte disturbances and alterations in gastrointestinal (GI) motility,
including gastroparesis. Great care must be taken before instituting an oral or enteral feeding
program. In severely malnourished patients before and during refeeding, correction of fluid and
electrolyte abnormalities should be achieved via the intravenous route, and not by adding
electrolyte to the enteral feeding regimen. A period of parenteral nutrition may be required in
those situations in which GI function is significantly compromised. For the boy described in the
vignette, the initial decision to employ enteral feedings was appropriate, and either the bolus or
continuous route would have been an acceptable alternative. However, because this patient's
gastrointestinal tract has already tolerated bolus enteral feedings, a significant reduction in
energy intake should represent the initial modification in the nutrition support program.

When deciding upon an appropriate nutritional support regimen for the hospitalized patient, a
simple but dear axiom should always prevail: "If the gut works, use it." Parenteral nutrition
should be reserved only for those situations in which GI motility or absorptive function are
compromised. Even in those cases, the availability of either partially or completely hydrolyzed
formulas has greatly enhanced the ability to provide adequate nutrition via the enteral route.
In general, peripheral venous nutrition is employed in patients who are unable to receive oral or
enteral nutrition for periods from 3 to 7 days duration and total parenteral nutrition is instituted
when patients must consume nothing by mouth (nil per os) for more than 7 days. Although
enteral feedings pose fewer significant risks than parenteral nutrition, including lower risks of
infection, fluid and electrolyte disturbances, and catheter malfunction, care must be taken to
avoid problems related to infusion rate and composition. Item C216, page C-185, lists the most
common tube feeding-related complications, including those metabolic problems associated with
refeeding syndrome. Many of the mechanical tube-related complications have been obviated by
the development of softer feeding tubes (polyurethane) that pose a far lower risk for perforation
and mucosal irritation.

Item (216). Most Common Complications of Enteral Nutrition


• Mechanical
- Malposition (tube placement into the trachea, pneumothorax)
- Blockage
- Sinusitis, otitis media
- Laryngeal ulceration
- Tracheoesophageal fistula - Variceal rupture
- Duodenal perforation
• Gastrointestinal
- Diarrhea or constipation - Abdominal distension
- Gastroesophageal reflux, esophagitis - Pulmonary aspiration
• Metabolic (including refeeding syndrome)
- Vitamin, mineral, trace element deficiencies
American academy of pediatrics 665
American Academy of Pediatrics PREP 2015

- Essential fatty acid deficiency (with low-fat formulas)


- Hyperglycemia
- Hyper- or hypokalemia
- Hypophosphatemia
- Hypomagnesaemia

PREP Pearls
• To prevent refeeding syndrome in a malnourished patient, the initial dietary regimen
should provide approximately 50% of calculated energy requirements.
• In addition to standard serum electrolytes (sodium, potassium, calcium), magnesium and
phosphorus levels should be monitored during refeeding.
• Vitamin supplements should be given during refeeding of malnourished patients because
clinical deficiency states (eg, thiamine) may develop during the refeeding period.

American Board of Pediatrics Content Specification(s)


• Identify the clinical and laboratory features associated with refeeding syndrome

Suggested Reading
• ASPEN Board of Directors and the Clinical Guidelines Task Force. Guidelines for the
use of parenteral and enteral nutrition in adult and pediatric patients. 'PEN 1 Pa renter
Enteral Nutr. 2002;26(1 Suppl):1SA138SA. doi:10.1177/0148607102026001011.
• Bankhead R, Boullata J, Brantley S, et al. Enteral nutrition practice recommendations.
WEN J Pa renter Enteral Nutr. 2009;33(4;122-167. doi:10.1177/0148607108330314.
• Fisher M, Simpser E, Schneider M. Hypophosphatemia secondary to oral refeeding in
anorexia nervosa. Int J Eat Disord. 2000; 28 (2):181-187. 10.1002/1098-
108X(200009)28:2<181::A1D-EAT7>3.0.0O2-K. Fuentebella J, Kerner JA. Refeeding
syndrome. Pediatr Clin North Am. 2009;56(5)120/-1210. doi:10.1016/j.pc1.2009.06.006.

American academy of pediatrics 666


American Academy of Pediatrics PREP 2015

Item 217
At birth, a full-term neonate is noted to have a L4 to L5 myelomeningocele. The neonate was
born to a 28-year-old mother with inconsistent prenatal care. The baby is transferred to your
neonatal intensive care unit. On physical examination, he has visible head enlargement with a
bulging fontanelle. He has severe foot deformities and bilateral hip dislocation. Head
ultrasonography confirms hydrocephalus, and neurosurgery is consulted. The mother is
concerned about any other systemic abnormalities that may be present.

Of the following, the MOST appropriate next step in evaluation should be


A. chromosome analysis
B. electroencephalogram
C. hearing screen
D. ophthalmologic evaluation
E. urologic evaluation

American academy of pediatrics 667


American Academy of Pediatrics PREP 2015

Item 217 TE Preferred Response: E


The patient in the vignette has myelomeningocele with common associated pathology in the
skeleton, skin, gastrointestinal, central nervous system, and genitourinary tracts. The next most
appropriate evaluation is to assess the urologic system. Myelomeningocele is a form of neural
tube defect (NTD) resulting from failure of the posterior closure of the neural tube. This most
commonly presents in the lumbosacral, thoracolumbar, and lumbar region and occurs between
the 3rd and 4th weeks of gestation. The incidence is 2 to 4 per 10,000 births. In most cases,
myelomeningocele is a multifactorial process resulting from a combination of environmental,
dietary, and polygenetic factors. Maternal folate deficiency and maternal anticonvulsant use
increases the risk.

Spinal dysraphism can clinically manifest as lower extremity weakness, tight heel cords,
decreased sphincter tone, sensory loss, gait abnormalities, bowel and bladder dysfunction, and
orthopedic deformities including clubfeet and dislocated hips. Often the integrity of the skin is
compromised with an evident open bony defect, fistulous tract, dimple, lipoma, hairy tuft, or
palpable mass overlying the region. A thorough neurologic examination can delineate the
functional level of the lesion and offer prognosis for ambulatory potential in the future. The
lower the lesion, the more likely the patient will be able to ambulate. Flaccid paralysis, absence
of deep tendon reflexes, decreased ability to sense touch and pain, and leg asymmetry is
commonly noted below the level of the lesion. A low sacral myelomeningocele will affect bowel
and bladder control, but not routinely impair the motor function. Higher lesions will typically
affect motor skills.

Patients with myelomeningocele have a higher risk for hydrocephalus in association with Chiari
II malformation (60%-95% higher risk versus patients without myelomeningocele), thus
neuroimaging at birth is very important to look for associated brain anomalies. Many patients
require ventriculoperitoneal shunts for the hydrocephalus. Patients with myelomeningocele have
a higher risk for seizures (25%). If present, an electroencephalogram (EEG) should be obtained.
However, screening EEGs are not routinely done.
Careful evaluation of the genitourinary system, initially as well as periodically, is extremely
important in the long-term management of myelomeningocele. Patients are highly prone to
vesicoureteral reflux, hydronephrosis, bladder dyssynergy, elevated bladder pressures, and
neurogenic bladder or bowel (80%). Because progressive renal insufficiency is a major cause of
morbidity and mortality in patients with myelomeningocele, a thorough urodynamic evaluation
including renal ultrasonography, cystometrograms, serum electrolytes, periodic urine cultures,
and vesiculo urethrogram are necessary to recognize and prevent renal damage. Urinary
continence and bladder pressure control are the primary goals of the urologic team. Most patients
also require bowel control programs that include low-fat, high-fiber diets, stool softeners, and
intermittent enemas.

Tethering of the spinal cord can be seen at birth or may develop over time, necessitating surgical
intervention to prevent further neurologic damage. Of patients with myelomeningocele, 80%
have normal intelligence, and better outcomes are seen in patients who had ventriculoperitoneal
shunts placed early to treat hydrocephalus. A multidisciplinary team is important in the care of
patients with myelomeningocele including a pediatrician, neurosurgeon, urologist, orthopedist,
neurologist, physiatrist, and therapists.
American academy of pediatrics 668
American Academy of Pediatrics PREP 2015

Folate is intrinsically important in the etiology of myelomeningocele and prevention of neural


tube defects; however, its mechanism in prevention is not well understood. Flour, pasta, rice, and
cornmeal are routinely fortified with folate in the United States. Folk acid supplementation
during pregnancy can reduce the risk of NTDs in the general population by 50%. In a woman
with a prior pregnancy with an NTD, folate supplementation during a subsequent pregnancy can
reduce the risk of repeat NTD by 70%. The Centers for Disease Control and Prevention
recommend that all women of childbearing age take a minimum of 0.4 mg of folic acid each day
because many pregnancies are not discovered until after the 4th week of gestation. Women with
a history of a NTD in themselves or a prior pregnancy are recommended to take 4 mg of folic
acid daily periconceptually, as well as during the first 3 months of the pregnancy. For women
who have had a child with a NTD, the recurrence rate in subsequent pregnancies is 2% to 4%.
Although myelomeningocele is clearly documented to be a polygenic and multifactorial problem,
chromosomes are rarely abnormal unless the myelomeningocele is present in conjunction with
multiple congenital anomalies. An EEG is only needed if the patient presents with seizures.
Hearing screens are typically normal. Ophthalmologic evaluation is not an immediate need in the
neonatal period, though patients are at risk for strabismus and should have an evaluation early in
life.

PREP Pearls
• Careful evaluation of the genitourinary system in conjunction with a pediatric urologist,
initially as well as periodically, is extremely important in the long-term management of
myelomeningocele.
• Folic acid supplementation during pregnancy can reduce the risk of neural tube defects
(NTD) in the general population by 50% and in that of a woman with a prior pregnancy
with a NTD by 70%.
• Patients with myelomeningocele are at risk for hydrocephalus in association with Chiari
II malformation.
• A multidisciplinary team is commonly involved including a pediatrician, neurosurgeon,
urologist, orthopedist, neurologist, physiatrist, and therapists.

American Board of Pediatrics Content Specification(s)


• Recognize other abnormalities commonly associated with myelomeningocele

Suggested Reading
• Joseph DB. Current approaches to the urologic care of children with spina bifida. Curr
Urol Rep. 2008;9(2):151-157.
• Liptak GS, Dosa NP. Myelomeningocele. Pediatr Rev. 2010;31:443-450.
doi:10.1542/pir.31-11-443.
• Kliegman RM, Behrman RE, Jenson Ha, Stanton BF, eds. Nelson Textbook of Pediatrics.
18th ed. Philadelphia, PA: Saunders Elsevier; 2007.
• US Preventive Services Task Force. Folic acid for the prevention of neural tube defects:
US Preventive Services Task Force recommendation statement. Ann Intern Med.
2009;150(9):626-631. doi:10.7326/0003-4819- l50-9-200905050-00009.

American academy of pediatrics 669


American Academy of Pediatrics PREP 2015

Item 218
The parents of a 4-month-old female infant arrive at your clinic for a new patient visit. They are
seeking a second opinion because the infant's recent hearing evaluation has confirmed a severe
sensorineural hearing loss. They see no signs of developmental problems, so they are hesitant to
accept the recommendation to access early childhood intervention services now.

Of the following, your BEST advice today is that


A. a multidisciplinary team approach is best and you will help coordinate that care
B. their daughter is too young to benefit from services now
C. they should learn American Sign Language because it is the best means of
communication
D. they should move to a city that has a school for the deaf in order for their daughter to be
educated
E. this degree of hearing impairment forebodes significant learning problems in the future

American academy of pediatrics 670


American Academy of Pediatrics PREP 2015

Item 218 TE SBP Preferred Response: A


The infant in the vignette confirmed severe sensorineural hearing loss and despite the fact that
she is not yet exhibiting any developmental delay, she should be referred for early intervention
now. A multidisciplinary team approach is most successful and the primary care physician
should serve as the coordinator of that care.

Hearing impairment is common with significant hearing loss occurring in 1 to 2 per 1,000
newborns. Delayed language development, behavior and psychosocial problems, and academic
underachievement may result. Early detection (before 9 months of age) and intervention with
effective treatment, such as speech therapy and amplification, improve language, cognitive, and
health outcomes. The US Preventive Services Task Force and the Joint Committee on Infant
Hearing recommend that all newborns be screened before 1 month of age; all those who fail this
screening should have comprehensive audiologic assessment by 3 months of age; and
individualized intervention for those with confirmed hearing impairment should begin by 6
months of age.

It is important to recognize that hearing loss is often a hidden handicap. Hearing-impaired infants
may achieve the early language milestones of smiling, cooing, and gesturing at the age-
appropriate time. Therefore, it is not uncommon for parents to raise similar concerns as those in
the vignette. Children with less severe hearing loss can appear to have relatively normal
development and thus remain undetected and untreated. The first step is to confirm the hearing
loss and diagnose any medical conditions, and then treat the underlying cause when possible.
Education and collaboration with parents and family members is key to successful outcomes. All
infants with any degree of bilateral or unilateral permanent hearing loss are eligible for early
intervention services. All children with a permanent hearing impairment should be followed by a
multidisciplinary team that may include audiologists, otolaryngologists, speech pathologists,
geneticists, educational specialists, a primary care provider, other medical specialists as needed
for comorbid conditions, and a care coordinator. The child's vision should also be evaluated to
ensure that he or she has the maximal opportunity to learn and develop communication skills.
The parents may need to be reassured, sometimes even convinced, that their child is not too
young to benefit from these services.

Multiple factors contribute to a family's plan regarding management and they should share in
decision making. Research has found no significant differences in developmental outcomes by
method of communication, so a range of options should be offered to families. Total
communication, as a method of instruction, is a combination of oral (speech, lip reading, and the
use of residual hearing) and manual (signing and fingerspelling) communication. Some families
may choose only oral or only manual communication rather than a combination of both. For
manual communication, several signing systems are in use today. American Sign Language
(ASL) is a visual-spatial language, is not phonologically based like English, and is the official
language of the culture of the deaf in the United States. American Sign Language has its own
rules of semantics, syntax, pragmatics, and vocabulary. Fingerspelling is a manual alphabet of 26
distinct hand positions used to represent each letter in the English alphabet. Signing Exact
English is a system that uses components of ASL, but attempts correct English usage.

American academy of pediatrics 671


American Academy of Pediatrics PREP 2015

If the patient's family desires access to spoken language, hearing aids are the primary form of
amplification used and can improve hearing, but do not necessarily restore it to normal.
Language outcomes are improved if amplification is initiated before the age of 6 months. Some
patients may not benefit from conventional hearing aids (air conduction devices), so a device that
transmits sound directly through the skull (bone conduction) may be tried. Cochlear implants
(surgically implanted prosthetic devices that electrically stimulate the cochlear nerve) may be
selected to use in patients with severe to profound bilateral sensorineural hearing loss and little
or no benefit from hearing aid use for 6 months.

Hearing loss or deafness does not affect one's intellectual abilities; however, children who are
hard of hearing or deaf generally have some difficulties learning language and communication
skills, and benefit from special education services and adaptations of the classroom environment.
The family should not feel compelled to move to a new city with a school for the deaf to educate
their daughter. The Individuals with Disabilities Education Act, formerly the Education of the
Handicapped Act Public Law 94-142, guarantees the opportunity to develop intellectually,
socially, and psychologically in a school and community that is inclusive yet focused on meeting
individual needs. Classrooms should be adapted with technology, interpreters, and trained
educators so hearing-impaired children can learn effectively. Hearing impairment alone does not
forebode significant learning problems.

PREP Pearls
• A multidisciplinary team approach to education for hearing-impaired children is most
successful and the pediatrician's role in coordinating that care is very important.
• Evidence is lacking to determine a preferred mode of communication, so a range of
options should be offered to families of hearing-impaired children.
• Classrooms should be adapted for hearing-impaired students to reach their full potential.

American Board of Pediatrics Content Specification(s)


• Understand the major approaches to education for hearing impaired children

Suggested Reading
• American Academy of Pediatrics, Joint Committee on Infant Hearing. Year 2007 position
statement: principles and guidelines for early hearing detection and intervention
programs. Pediatrics. 2007;120(4):898-921. doi:10.1542/peds.2007-2333.
• Bachmann KR, Arvedson JC. Early identification and intervention for children who are
hearing impaired. Pediatr Rev. 1998;19(5):155-165. doi:10.1542/pir.19-5-155.
• Feldman HM. Evaluation and management of language and speech disorders in preschool
children. Pediatr Rev. 2005;26(4):131-142. doi:10.1542/pir.26-4-131.
• Gifford KA, Holmes MG, Bernstein HH. Hearing loss in children. Pediatr Rev.
2009;30(6):207-215. doi:10.1542/pir.30-6-207.
• Joint Committee on Infant Hearing of the American Academy of Pediatrics, Muse C,
Harrison J, et al. Supplement to the JC1H 2007 position statement: principles and
guidelines for early intervention after confirmation that a child is deaf or hard of hearing.
Pediatrics. 2013;131(41:e1324-e1349. doi:10.1542/peds.2013-0008.
• Petrou 5, McCann D, Law CM, Watkin PM, Worsfold S, Kennedy CR. Health status and
health-related quality of life preference-based outcomes of children who are aged 7 to 9
American academy of pediatrics 672
American Academy of Pediatrics PREP 2015

years and have bilateral permanent childhood hearing impairment. Pediatrics.


2007;120(5):1044-1052. doi:10.1542/peds.2007-0159.
• Watkin P, McCann D, Law C, et al. Language ability in children with permanent hearing
impairment: the influence of early management and family participation. Pediatrics.
2007;120(3):e694-e701. doi:10.1542/ peds.2006-2116.

American academy of pediatrics 673


American Academy of Pediatrics PREP 2015

Item 219
A 13-year-old adolescent girl presents to your clinic for a routine visit. She was diagnosed with
acute lymphoblastic leukemia at 5 years of age, was treated with chemotherapy, and has been in
complete remission. Her chemotherapy regimen included corticosteroids, vincristine,
doxorubicin, asparaginase, cytarabine, methotrexate, and 6-mercaptopurine. Her mother is
concerned about the long-term effects of the treatment her daughter received.

Of the following, this patient is MOST likely to be at increased risk for


A. cardiovascular disease
B. deep vein thrombosis
C. glucose intolerance
D. infertility
E. short stature

American academy of pediatrics 674


American Academy of Pediatrics PREP 2015

Item 219 Preferred Response: A


The adolescent in this vignette is most likely to have problems with cardiovascular disease as a
late sequela of her exposure to chemotherapy, in particular doxorubicin.
With the overall cure rate for childhood cancer now approximately 80%, patients are
experiencing the late effects of their chemotherapy. The farther out the child is from initial
diagnosis and treatment, the lower the risk of recurrent or second malignancy, and the higher the
likelihood of morbidity from other causes. Pediatric cancer survivors can develop cardiovascular
disease after receiving chemotherapy or radiation, and it is the second leading cause of morbidity
and mortality after cancer recurrence. Pediatric cancer survivors have an increased risk of
premature atherosclerosis, stroke, coronary artery disease, and heart failure.

Although asparaginase and the presence of central venous catheters can increase the risk for
thrombosis around the time of treatment, it would be unlikely to cause deep venous thromboses
years after treatment has ended. Glucose intolerance can be seen in patients who receive high
doses of corticosteroids for long periods, but again this should be reversible once the patient has
stopped taking the medication. Infertility can be a concern for cancer survivors, more frequently
after treatment with alkylating agents, such as cyclophosphamide, and in older children who
received treatment during puberty. Short stature and endocrine dysfunction are commonly seen
in childhood cancer survivors who have received cranial, spinal, or total body radiation therapy.
This was often included in treatment protocols for acute lymphoblastic leukemia before
intrathecal chemotherapy became standard of care.

PREP Pearls
• Pediatric cancer survivors can develop cardiovascular disease after receiving
chemotherapy or radiation, and it is the second leading cause of morbidity and mortality
after cancer recurrence.
• With the overall cure rate for childhood cancer now approximately 80%, patients are
experiencing the late effects of their chemotherapy.

American Board of Pediatrics Content Specification(s)


• Understand the risks, side effects, and late sequelae associated with various
chemotherapeutic drugs

Suggested Reading
• Armstrong GT, Liu Q, Yasui Y, et al. Late mortality among 5-year survivors of
childhood cancer: a summary from the Childhood Cancer Survivor Study. J Clin Oncol.
2009;27(14):2328-2338. doi:10.1200/ JC0.2008.21.1425.
• Chow EJ, Liu W, Srivastava K, et al. Differential effects of radiotherapy on growth and
endocrine function among acute leukemia survivors: a childhood cancer survivor study
report. Pediatr Blood Cancer. 2013;60(1):110-5. doi:10.1002/pbc.24198.
• Dengel DR, Kelly AS, Zhang L, Hodges JS, Baker KS, Steinberger J. Signs of early sub-
clinical atherosclerosis in childhood cancer survivors. Pediatr Blood Cancer.
2013;61(3):532-537. doi:10.1002/pbc.24829.

American academy of pediatrics 675


American Academy of Pediatrics PREP 2015

• Leisenring WM, Mertens AC, Armstrong GT, et al. Pediatric cancer survivorship
research: experience of the Childhood Cancer Survivor Study. J Clin Oncol.
2009;27(14):2319-2327. doi:10.1200/)C0.2008.21.1813.
• Lipshultz SE, Cochran TR, Franco VI, Miller TL. Treatment-related cardiotoxicity in
survivors of childhood cancer. Nat Rev Clin Oncol. 2013;10(12):697-710.
doi:10.1038/nrclinonc.2013.195.

American academy of pediatrics 676


American Academy of Pediatrics PREP 2015

Item 220
Yesterday there was a tragedy in your neighborhood, a drive-by shooting in which 2 adolescents
killed a third adolescent. You are anticipating community knowledge of this event will affect
your patients. To prepare your office staff to handle questions appropriately about this event, you
decide to share information about predictors of community violence in children.

Of the following, the MOST appropriate statement to make in this situation is that
A. community violence by itself is not a significant predictor that a child in that area will
become socially violent themselves
B. early onset of aggression in childhood is unlikely to predict those who may commit future
social violence
C. individual mental health counseling is effective at reducing future socially induced
violence
D. parental involvement with children can effectively reduce socially induced violence
E. socially induced aggressive behavior in children is generally unrelated to genetic or
biological factors

American academy of pediatrics 677


American Academy of Pediatrics PREP 2015

Item 220 I-C S Preferred Response: D


Adolescent violence is multifactorial, and as such does not have a neat and simple explanation as
to why it occurs. Common elements include biological factors (eg, brain changes from
abuse/neglect and substance abuse), psychological factors (eg, caregiver-modeled violence and
poor coping skills), and social factors (eg, neighborhood crime and lack of family responsivity).
Both acute and chronic factors can be identified as well. Violent acts can be borne out of static
factors like being a male with a history of academic failure, and out of dynamic factors such as
suffering an acute loss or misattributing hostile intent to another's immediate actions.

Despite the multifactorial origin of violence, the preferred means to address adolescent violence
depends on core parenting and psychiatric treatment principles. The most family-supportive and
consistently endorsed intervention is increased parental involvement with their children. Parental
supervision, monitoring, engagement, and support of all realms of the child's life is central to
multisystemic therapy, the most evidence-based treatment for juvenile offenders. For conduct
disorder problems that are not yet at the level of legal charges being filed, increased parental
involvement with re-establishment of parental authority is a cornerstone of behavior
management training interventions. As such, the office staff in this vignette should learn that
promoting positive parental involvement with their children is key to reducing socially induced
violence.

Other effective strategies for reducing violence include social skills training in the
elementary/middle school years, group anger management training, family therapy, increased
school supervision and structure, individualized education program (IEP) evaluations to address
academic underachievement, and safety measures regarding access to guns and other weapons.
Community violence is a significant predictor of social violence among adolescents. Early-onset
aggression is a significant risk factor for future violent behaviors in adolescence. Biological
factors, like male sex and genetic vulnerability, are well-recognized contributors to violent
behaviors. Although mental health counseling can be very helpful for treating psychiatric
conditions, individual counseling that lacks family or community involvement is not a very
effective strategy for reducing adolescent social violence.

PREP Pearls
• Violent behavior in adolescents is multifactorial in origin, and has both chronic and acute
risk factors.
• Parental engagement in all aspects of their children's lives is a recommended intervention
in the treatment of aggressive youth.

American Board of Pediatrics Content Specification(s)


• Understand the effects of societal violence on children

Suggested Reading
• Barclay RP, Hilt RJ. Youth violence: assessment and treatment planning in primary
practice. Pediatr Ann. 2014;43(1):38-44. doi: 10.3928/0090448120131223-16.

American academy of pediatrics 678


American Academy of Pediatrics PREP 2015

• Bern at DH, Oakes JM, Pettingell SL, Resnick M. Risk and direct protective factors for
youth violence, results from the national longitudinal study of adolescent health. Am I
Prey Med. 2012;43(2 suppl 1):S57-S66. doi; 10.1016/j.amepre.2012.04.023.
• Tarter RE, Kirisci L, Vanyukov M, et al. Predicting adolescent violence: impact of family
history, substance use, psychiatric history, and social adjustment. Am I Psychiatry.
2002;159(9):1541-1547. doi:10.1176/appi. ajp.159.9.1541

American academy of pediatrics 679


American Academy of Pediatrics PREP 2015

Item 221
A 3-day-old, 32-week-gestation newborn has a murmur. The baby is currently on room air with
oxygen saturation of 97% and is tolerating feeds and voiding well. Heart rate is 130 beats/min,
respiratory rate is 32 breaths/min, and blood pressure is 80/47 mm Hg. The newborn is in no
distress. The chest examination shows clear breath sounds bilaterally. The cardiac examination
shows a normal point of maximal impulse, and there is a 2/6 systolic murmur at the left upper
sternal border that radiates to the scapula on the same side. The liver size is normal. The
cardiologist sees the patient and tells you that the echocardiogram shows a patent ductus
arteriosus (PDA). The PDA is smaller in size than the branch pulmonary arteries.

Of the following, the BEST course of action for this newborn at this time is
A. observation only
B. furosemide given intravenously
C. indomethacin for 3 doses
D. oxygen 100% by nasal cannula at 2 L
E. surgical ligation

American academy of pediatrics 680


American Academy of Pediatrics PREP 2015

Item 221 Preferred Response; A


The newborn described in the vignette is premature but not in distress, and has no signs of
congestive heart failure such as tachypnea, enlarged liver, or tachycardia. There is no evidence of
a large diastolic runoff, that is, the pulse pressure on the newborn's blood pressure is not
widened. The newborn is also tolerating feedings and is in room air. The patent ductus arteriosus
(PDA) is smaller than the branch pulmonary arteries on echocardiography and is therefore not
characterized as large. The best course of action for this newborn is observation.

In this patient, given this combination of findings, the risks associated with indomethacin
administration or surgical closure do not need to be undertaken. There is no evidence that the
PDA is hemodynamically significant and it may close spontaneously. Surgical closure holds the
risk of thoracic duct and recurrent laryngeal nerve injury. Indomethacin holds the risk of renal
and intestinal damage.

The criteria for conservative management with diuresis versus treatment with indomethacin or
surgical ligation differ between centers. If a premature infant has prolonged respiratory distress,
requires ventilation for more than 7 days, has a large PDA, and has a large pulse pressure,
treatment would be indicated because the incidence of chronic lung disease increases with time.
There is a slight increase in mortality with surgical ligation, but this may be necessary in patients
with renal dysfunction or concern for necrotizing enterocolitis (NEC). For a moderate-sized PDA
with minimal respiratory support, there is no difference in outcome between conservative
management with diuretics and indomethacin.

PREP Pearls
• Often a patent ductus arteriosus (PDA) will close spontaneously.
• Risks and benefits must be assessed for each infant, considering gestational age, intestinal
perfusion, lung disease, and renal function, as well as size of PDA and the infant's
clinical status.

American Board of Pediatrics Content Specification(s)


• Plan appropriate initial management of patent ductus arteriosus (PDA) in an infant born
prematurely

Suggested Reading
• Bernstein D. Patent ductus arteriosus. In: Kliegman RM, Stanton BF, St Geme 1W III,
Schor NF, eds. Nelson Textbook of Pediatrics. 19th ed. Philadelphia, PA: Saunders
Elsevier; 2011:1559-1560.
• Hamrick SEG, Hansmann G. Patent ductus arteriosus of preterm infant. Pediatrics.
2010;125(5)1020-1030. doi:10,1542/peds.2009-3506.
• Mirea L, Sankaran K, Seshia M, et al. Treatment of PDA and neonatal mortality and
morbidities/morbidities: adjustment for treatment selection bias. I Pediatr.
2012;16(4):689-694. doi:10.1016/j.jpeds.2012.05.007.

American academy of pediatrics 681


American Academy of Pediatrics PREP 2015

Item 222
You are on a committee charged with decreasing the infection rate in your hospital system. The
committee has identified hospital-acquired methicillin-resistant Staphylococcus aureus (MRSA)
infection rates in intensive care units as a focus for improvement. A recently published article
demonstrated that cleansing each patient with antibacterial soap daily decreased the incidence of
MRSA infections in an intensive care unit (ICU). Your hospital system does not currently use
this practice. The committee plans to evaluate the use of this practice using the Langley model of
quality improvement (ie, Plan-Do-Study-Act (PDSA).

Of the following, the MOST appropriate first step would be to


A. develop a procedure for ICU staff to use antibacterial soap to cleanse patients
B. implement a policy to cleanse ICU patients with antibacterial soap daily throughout the
hospital system
C. implement a policy to cleanse patients daily with antibacterial soap in a single ICU
D. review MRSA infection rates in an ICU that already uses antibacterial soap at another
hospital in your town
E. wait for more evidence that using antibacterial soap decreases MRSA infection rates

American academy of pediatrics 682


American Academy of Pediatrics PREP 2015

Item 222 TE BPLI SBP Preferred Response: A


The most appropriate first step in this scenario would be for hospital personnel to plan and
develop a procedure for the intensive care unit (ICU) to implement the use of antibacterial soap
to cleanse patients. The planning of the quality improvement (QI) initiative is the first step in the
Langley model of improvement.

Quality improvement is a process of enacting incremental change, and measuring the effects of
these changes over time. The Plan-Do-Study-Act (PDSA) model starts with asking what are we
trying to accomplish, how will we know that any changes are improvements, and what changes
will result in improvements. Then, a PDSA cycle is started. In the "Plan" phase, the QI
objectives are defined and a plan is made. The plan should include identifying the staff
responsible for making changes, defining what changes will be made, and when the changes will
occur. The "Do" phase of the project includes implementation of the changes. Any problems that
are encountered are documented so they can be addressed. During the "Study" phase of the
project, the team analyzes the process and the outcomes, and records what was learned from the
cycle. During the "Act" phase, the team determines whether to adopt, adapt, or abandon the
changes based on the "Study" phase results. The results of the first cycle can then be inserted
into the next PDSA cycle.

The first step in the PDSA model is to plan a strategy, therefore it would be incorrect to
immediately implement a new policy to cleanse patients in a single ICU or throughout the
hospital system. In the PDSA model, the approach should be a step-by-step process. Change
would be implemented initially on a small scale, such as in a single ICU, before making it a
system-wide policy. It may be beneficial to look at other units that may already be using
antibacterial soap to cleanse patients; however, this would then be part of the background for the
project and would not necessarily be equivalent to the new process developed in your hospital. If
your QI goal is to decrease the incidence of MRSA infections, then waiting and taking no action
on the issue pending published research data would be inappropriate.

PREP Pearls
• The "Plan" phase of a Plan-Do-Study-Act cycle should include quality improvement (QI)
objectives, define the staff responsible for changes, the specific changes to be made, and
a timeline.
• The "Do" phase of a QI project includes implementation of changes and documentation
of problems.
• In the 'Study" phase of a QI project, the process, outcomes, and lessons learned are
analyzed.
• The "Act" phase of a QI project consists of deciding whether to adopt, adapt, or abandon
the changes after reviewing the "Study" phase results.

American Board of Pediatrics Content Specification(s)


• Identify the components of the Langley Model for Improvement: Plan-Do-Study-Act

Suggested Reading
• Leonard MS. Patient safety and quality improvement: medical errors and adverse events.
Pediatr Rev. 2010;31(0151-158. doi:10.15421pir.31-4-151.
American academy of pediatrics 683
American Academy of Pediatrics PREP 2015

• Schriefer J. Leonard MS. Patient safety and quality improvement: an overview of Ql.
Pediatr Rev. 2012;33(8):353-360. doi:10.1542/pir,33-8-353.

American academy of pediatrics 684


American Academy of Pediatrics PREP 2015

Item 223
You are a seeing 3-day-old patient admitted to the hospital for hyperbilirubinemia. During your
rounds, the patient's parents ask how their child's discomfort could be minimized during lancing
of the heel to obtain blood.

Of the following, the method MOST likely to be effective is


A. distraction with a toy
B. feeding a sucrose solution
C. hearing a parent's voice
D. rocking and holding
E. swaddling the child

American academy of pediatrics 685


American Academy of Pediatrics PREP 2015

Item 223 Preferred Response: B


Providing infants with a sucrose solution is most likely to be effective in minimizing discomfort
during blood draws. Many studies have shown that for brief procedures, administration of a
sugar solution to an infant can decrease behaviors associated with pain. Sweeter sugars appear to
confer greater benefit in reducing pain; for example, administration of a sucrose solution has
been shown to decrease pain in multiple studies, whereas lactose solutions have not been shown
to have a positive effect. The exact mechanism by which sugar solutions mitigate the pain
response is not known, but may be related to the release of endogenous opioids.

Over the last decade, the medical community has developed an increased awareness of the need
to address procedure-related pain in infants. Historically, several barriers have interfered with the
provision of adequate analgesia to younger children, including the misconceptions on the part of
health care providers that young infants do not perceive pain. The perception that pharmacologic
treatments are too risky for younger patients and that nonpharmacologic therapies are ineffective
have also contributed to inadequate pain management

Although infants are not capable of reporting pain directly, facial expressions, cry duration, and
physiologic parameters can indicate that an infant is experiencing pain. The use of
pharmacologic and nonpharmacologic treatments for pain management can decrease behaviors
indicative of pain in the youngest patients. For brief, minimally invasive procedures,
nonpharmacologic methods of pain control may be sufficient. However, for longer, more
invasive procedures, topical or systemic analgesics are indicated.

Distraction with a toy has not been shown to mitigate the perception of pain in older infants and
is unlikely to benefit the neonate in the vignette. Hearing a parent's voice, rocking, and holding
have not been showed to decrease pain during procedures, but may help to calm an infant
afterwards. There are limited data to suggest that swaddling may help decrease pain-associated
behaviors in young infants, but the data supporting the use of sucrose solutions are more robust.

PREP Pearls
• The use of pharmacologic and nonpharmacologic treatments for pain management can
decrease behaviors and physiologic changes indicative of pain in infants.
• Several studies have shown that for brief, minimally invasive procedures, administration
of a sucrose solution to an infant can decrease behaviors associated with pain.

American Board of Pediatrics Content Specification(s)


• Understand the appropriate use of nonpharmacologic pain management modalities

Suggested Reading
• Cramton RE, Gruchala NE. Managing procedural pain in pediatric patients. Curr Opin
Pediatr. 2012;24(4):530-538. doi:10.1097/ MOP.0b013e328355b2c5.
• Harrison D, Beggs S. Stevens B. Sucrose for procedural pain management in infants.
Pediatrics. 2012:130(5}:918-925. doi:10.1542/peds.2011-3848.

American academy of pediatrics 686


American Academy of Pediatrics PREP 2015

Item 224
A 5-year-old boy with acute lymphoblastic leukemia on maintenance chemotherapy is admitted
to the hospital with a 4-week history of watery diarrhea, mild crampy abdominal pain and
occasional vomiting. The parents report that he has lost 2 kg over the last month. On physical
examination, his temperature is 37°C, heart rate is 130 beats/min, respiratory rate is 20
breaths/min, blood pressure is 100/60 mm Hg, and weight is 15 kg (5th percentile). The child is
tired appearing but interactive with the examiner. His mucous membranes are mildly dry and
capillary refill is 2 seconds. Examination of the oropharynx, heart, and lungs is unremarkable.
His abdomen is soft, nondistended, and nontender to palpation, and there is no organomegaly.
There are no skin rashes or lesions. Examination of the stool reveals an organism (Item Q224).

Of the following, the MOST likely etiology of the patient's illness is


A. Cryptococcus neoformans
B. Cryptosporidium species
C. Endolimax nana
D. Entamoeba coli
E. Giardia intestinalis

American academy of pediatrics 687


American Academy of Pediatrics PREP 2015

Item 224 Preferred Response: B


The immunocompromised boy in the vignette has a protracted illness characterized by watery
diarrhea, abdominal pain, vomiting, and weight loss without fever, Examination of his stool
reveals Cryptosporidium species, intracellular parasites that stain red using a modified acid-fast
stain. Cryptosporidium hominis and Cryptosporidium parvum are the primary species that infect
humans and are transmitted by the fecal-oral route. C parvum can infect cattle and other
mammals as well. Outbreaks of Cryptosporidium, which is chlorine tolerant, have been
associated with contaminated drinking and recreational water. The organism is found in every
area of the United States and worldwide.

In immunocompetent patients, infection caused by Cryptosporidium can be asymptomatic, but


usually causes watery, nonbloody diarrhea lasting 1 to 2 weeks. Abdominal pain and cramping,
nausea and vomiting, fever, and weight loss caused by dehydration also can occur. In
immunocompromised patients, infection can be severe and life threatening. Rare complications
include infection of the lungs, biliary tract, and disseminated disease. In immunocompetent
patients, clearance of oocysts from stool usually occurs within 2 weeks of illness resolution.
Oocyst shedding in those with immunodeficiency can last for months.

Cryptococcus neoformans causes pulmonary disease and is likely to disseminate to bones, skin,
and the central nervous system in patients with impaired T-cell immunity. Gastrointestinal
cryptococcosis infection is rare. Endolimax nano and Entamoeba coil are nonpathogenic
intestinal protozoa, even in immunocompromised hosts. Giardia intestinalis is the most common
cause of parasitic diarrhea worldwide. It most commonly results in watery, nonbloody diarrhea
with abdominal pain, foul-smelling stools, and flatulence. Patients with impaired immunity can
have chronic infection resulting in weight loss and failure to thrive. Under microscopic
examination, the Giardia trophozoites are binucleated and have the appearance of a face (Item
C224).

PREP Pearls
• in immunocompetent patients, infection caused by Cryptosporidium can be
asymptomatic, but usually causes watery, nonbloody diarrhea lasting 1 to 2 weeks.
• In immunocompromised patients, Cryptosporidium infection can be severe and life-
threatening.
• Cryptosporidium species are intracellular parasites that stain red using a modified acid-
fast stain.
• Outbreaks of Cryptosporidium, which is chlorine tolerant, have been associated with
contaminated drinking and recreational water.

American academy of pediatrics 688


American Academy of Pediatrics PREP 2015

American Board of Pediatrics Content Specification(s)


• Understand the epidemiology of Cryptosporidium infection
• Recognize the clinical features associated with Cryptosporidium infection, including
Cryptosporidium diarrhea in an immunocompromised host

Suggested Reading
• American Academy of Pediatrics. Amebiasis. In; Pickering LK, Baker CJ, Kimberlin
DW, Long SS, eds. Red Book: 2012 Report of the Committee on Infectious Diseases.
29th ed. Elk Grove Village, IL: American Academy of Pediatrics; 2012:222-225.
• American Academy of Pediatrics. Cryptococcus neoforrnans infections. In: Pickering
LK, Baker CI, Kimberlin DW, Long SS, eds. Red Book: 2012 Report of the Committee
on Infectious Diseases. 29th ed. Elk Grove Village, IL: American Academy of Pediatrics;
2012:294-296.
• American Academy of Pediatrics. Cryptosporidium. In: Pickering LK, Baker CJ,
Kimberlin DW, Long SS, eds. Red Book: 2012 Report of the Committee on Infectious
Diseases. 29th ed. Elk Grove Village, IL: American Academy of Pediatrics; 2012:296-
298.
• American Academy of Pediatrics. Giardia intestinalis infections. In: Pickering LK, Baker
CJ, Kimberlin DW, Long SS, eds. Red Book: 2012 Report of the Committee on
Infectious Diseases. 29th ed. Elk Grove Village, IL: American Academy of Pediatrics;
2012:333-335.
• US Centers for Disease Control and Prevention. Parasites: cryptosporidium. US Centers
for Disease Control and Prevention website. Updated January 16, 2013.
• US Centers for Disease Control and Prevention. Parasites: nonpathogenic (harmless)
intestinal protozoa. US Centers for Disease Control and Prevention website.
http://www.cdc.gov/parasites/nonpathprotozoat Updated December 14, 2012.

American academy of pediatrics 689


American Academy of Pediatrics PREP 2015

Item 225
A 10-year-old boy presents to the emergency department after being thrown off his bike and
hitting the right side of his head on the sidewalk. This occurred after he hit a "speed bump" while
biking at full speed. He seemed "dazed" for a few seconds initially, but he has been alert and
talking to his mother during the car ride to the emergency department. On physical examination,
the boy has tenderness and a hematoma over the right temporoparietal region of his head. The
remainder of his initial physical examination findings, which include a complete neurologic
examination, are unremarkable. On subsequent examination 30 min later, the boy seems much
more lethargic, his speech is slurred, and his responses to very simple questions seem delayed.
His mother tells you that before your reassess-"lent, he said he "had a really bad headache."

Of the following, the MOST likely explanation for the boy's current symptoms is
A. basilar skull fracture
B. concussion
C. epidural hematoma
D. subarachnoid hemorrhage
E. subdural hematoma

American academy of pediatrics 690


American Academy of Pediatrics PREP 2015

Item 225 TE Preferred Response: C


Given his history of injury and the clinical findings, an epidural hematoma (EDH) is the most
likely explanation for the signs and symptoms displayed by the boy in the vignette. Epidural
hematoma should be suspected in children who experience head trauma, including injuries that
initially seem relatively minor, and then display progressive symptoms such as worsening
headache, lethargy, vomiting, and focal neurologic signs. Epidural hematomas develop from
bleeding into the space between the skull and
dura mater. Although typically arising from
shearing of the middle meningeal artery, they
may also result from venous bleeding. Epidural
hematomas most commonly result from a blunt
trauma mechanism, with falls being the most
frequent cause. Although 60% to 80% of
children with EDH will have an associated
skull fracture, up to 40% will not. While
classic teaching indicates that patients with
EDH experience a loss of consciousness at the
time of injury, then have a lucid period, and
then clinically deteriorate, this clinical
presentation is actually quite rare. Only about
20% of children with EDH experience loss of
consciousness and most present with more
subtle signs, including persistent vomiting and
headache after head trauma. Recent studies
estimate that more than 30% of patients with
EDH are alert with normal neurologic
examination findings at the time of diagnosis.

As EDHs expand, however, affected children


may clinically deteriorate with increased
lethargy or focal neurologic findings, as well as
frank signs of herniation. Epidural hematomas
(Item C225A) appear as lens-shaped
convexities that generally do not cross suture lines on computed tomography of the brain. All
children found to have EDHs require emergent neurosurgical consultation and very close
monitoring. Some clinically stable children with small EDHs may be treated nonoperatively,
while those with larger hematomas, midline shifts, or significant clinical symptoms must
undergo emergent operative management.

Basilar skull fractures (BSFs) can occur anywhere along the base of the skull, but typically
involve the petrous portion of the temporal bone. Basilar skull fractures can have unique clinical
presentations providing clues that are often readily apparent on physical examination. These
findings include hemotympanum, cerebrospinal fluid (CSF) otorrhea or rhinorrhea, periorbital
ecchymosis ("raccoon eyes"), and postauricular ecchymoses (Battle sign). However, they can
also occur in the absence of these clinical findings. Basilar skull fractures are associated with
intracranial injury, as well as an increased risk of meningitis. The boy in the vignette has no
American academy of pediatrics 691
American Academy of Pediatrics PREP 2015

clinical findings that are specifically indicative of a BSF. Also, the scalp hematoma that is
present over his right parietal skull suggests this area as the site of traumatic impact, rather than
the skull base.

Concussion is a clinical syndrome, following head trauma, involving an alteration of brain


function, typically affecting memory and orientation. Symptoms of concussion may include loss
of consciousness, headache, vomiting, memory loss, emotional lability, and dizziness. These
symptoms generally improve gradually over time. Although the symptoms displayed by the boy
in the vignette could be the result of concussion, the presence of a large hematoma over his right
parietal scalp and his rapid clinical deterioration are more concerning for the diagnosis of EDH.

Subarachnoid hemorrhage (SAH) resulting from head trauma typically occurs in patients with
more severe mechanisms of injury. Subarachnoid hemorrhage results from tearing of the small
vessels of the pia mater resulting from significant blunt trauma and associated shearing forces.
The hemorrhage occurs in a space communicating with other CSF-containing spaces, therefore
problems arising from mass effect that occur with EDHs and subdural hematomas (SDHs) rarely
occur with SAH. Subarachnoid hemorrhage often occurs in association with other intracranial
injuries. Associated signs and symptoms may include loss of consciousness, severe headache, or
signs of meningeal irritation. The clinical picture displayed by the boy in the vignette is more
consistent with that of EDH than SAH

Subdural hematomas (Item C225B) are caused by injury to the bridging cortical veins between
the dura and arachnoid membrane, usually because of shearing forces associated with sudden
acceleration and deceleration of the head. These lesions are more common in younger children.
Abusive head trauma is a common cause of SDH (Item C225C) in infants and young children.
Subdural hematoma is not typically associated with overlying skull fracture. Unlike EDH, an
SDH is generally associated with underlying parenchymal brain injury, and hemorrhages can be
bilateral. Subdural hematomas appear as crescent-shaped areas of hemorrhage on computed
tomography of the brain, and can cross suture lines. Affected children may present with
alteration or loss of consciousness, seizures, irritability, vomiting, lethargy, and signs and
symptoms of increased intracranial pressure (ie, bulging fontanelle, bradycardia, irregular

American academy of pediatrics 692


American Academy of Pediatrics PREP 2015

respirations). Mortality of children presenting with acute SDH is high, ranging from 10% to
20%. The clinical presentation for the boy in the vignette is much more consistent with that of
EDH than SDH.

Magnetic resonance image demonstrating a subdural hemorrhage.

CTscan demonstrates subdural hemorrhage left posterior and left lateral that resulted from child
abuse.

PREP Pearls
• Epidural hematoma (EDH) should be suspected in children who experience head trauma,
including injuries that initially seem relatively minor, and then display progressive
symptoms such as worsening headache, lethargy, vomiting, and focal neurologic signs.
• Only about 20% of children with EDH experience loss of consciousness, and most
present with more subtle signs, including persistent vomiting and headache after head
trauma.
• All children found to have EDHs require emergent neurosurgical consultation and very
close monitoring.

American Board of Pediatrics Content Specification(s)


• Recognize the signs and symptoms associated with closed-head trauma leg,
hemotympanum, ecchymoses, postauricular bruise

American academy of pediatrics 693


American Academy of Pediatrics PREP 2015

Suggested Reading
• Quayle KS, McQuillen KK, Lee P. Head trauma. la: Strange GR, Ahrens W, McQuillen
KK, et al, eds. Pedratric Emergency Medicine: Just the Farts. New York, NY: McGraw-
Hill; 2004:39-45
• Schunk JE, Schutzman SA. Pediatric head injury. Pediatr Rev.2012;33(9):398-4
doi:10.1542/pir.33-9-398.
• Wing R, James C. Pediatric head injury and concussion. Emerg Med Ciin North Am.
2013;31(3}:653-675. doi:10.1016/j.emc.2013.05.007.

American academy of pediatrics 694


American Academy of Pediatrics PREP 2015

Item 226
You are seeing a 2-week-old newborn in your office following discharge from the neonatal
intensive care unit after treatment for indirect hyperbilirubinemia. The newborn was born at term
after an unremarkable pregnancy, with prenatal screening notable for maternal blood type O
positive and antibody negative. The newborn was jaundiced at less than 24 hours after birth.
Evaluation at that time included blood type B positive and direct antiglobulin test positive.
Intense phototherapy and intravenous immunoglobulin therapy was required, with the total
bilirubin peaking at 18.2 mg/dL (311.3 µmol/L). At the time of hospital discharge, the
hemoglobin was 12 g/dL (120 g/L), the total bilirubin was 10.2 mg/ dL (174.5 umol/L), and the
direct bilirubin was 0.9 mg/dL (15.4 pion). The mother asks if there are complications related to
the jaundice that she needs to worry about.

Of the following, the most likely complication to be seen in this infant is


A. cholestatic liver disease
B. hearing loss
C. kernicterus
D. late anemia of infancy
E. malignant melanoma

American academy of pediatrics 695


American Academy of Pediatrics PREP 2015

Item 226 TE Preferred Response: D


The newborn in this vignette has hemolytic disease of the fetus and newborn (HDFN) due to
ABO incompatibility and is at increased risk of late anemia of infancy. Mothers with blood type
O naturally have antibodies against A and B red blood cell (RBC) surface antigens. These
antibodies are immunoglobulin G type and cross the placenta. Although 15% of pregnancies are
at risk for ABO incompatibility, it occurs in only 4% of at risk newborns. Affected newborns
commonly present with hyperbilirubinemia within the first 24 hours after birth that may progress
to levels causing kernicterus if treatment is not initiated.

Intensive phototherapy is the initial treatment for hyperbilirubinemia caused by HDFN. Affected
newborns whose bilirubin values continue to rise in spite of optimized phototherapy or whose
values are close to the exchange transfusion threshold may be treated with intravenous
immunoglobulin (IVIG). Although the efficacy of IVIG has not been proven, IVIG is felt to
block antibody receptors on RBCs and inhibit hemolysis. Double volume exchange transfusion is
reserved for those newborns whose total serum bilirubin values reach the established threshold.
Kernicterus and late anemia of infancy are the 2 major complications of HDFN. Early and
effective management of hyperbilirubinemia can prevent the development of kernicterus. Late
anemia of infancy is seen in the first month after birth and is attributed to continued destruction
of the newborn's RBCs by the circulating maternal immunoglobulin. Intrauterine or exchange
transfusion may further exacerbate the anemia by suppressing erythropoiesis. At risk newborns
should be monitored by serial hematocrit and reticulocyte counts after discharge from the
hospital. Multiple dose treatment with IVIG has been suggested to decrease the ongoing
hemolysis, decreasing the need for transfusion for late anemia of infancy.

The newborn in this vignette is unlikely to have kernicterus, as the majority of cases in term
newborns occur when the serum bilirubin values exceed 20 mg/dL (342.1 µmol/L). Infants with
high bilirubin values have brainstem auditory evoked response abnormalities, but these adverse
effects appear to reverse with treatment of the hyperbilirubinemia. Limited studies have not
demonstrated any increased risk of skin cancer, including melanoma following treatment with
phototherapy. Although up to 13% of infants with HDFN may develop cholestatic liver disease,
it is independently associated with intrauterine RBC transfusion or Rhesus D alloimmunization.

PREP Pearls
• Kernicterus and late anemia of infancy are the 2 major complications of hemolytic
disease of the fetus and newborn.
• The late anemia of infancy associated with hemolytic disease of the fetus and newborn is
seen in the first month after birth and is attributed to continued destruction of the
newborn's red blood cells by the circulating maternal immunoglobulin.

American Board of Pediatrics Content Specification(s)


• Recognize the association of ABO or Rh incompatibility with progressive or severe
anemia in infants of various ages

Suggested Reading

American academy of pediatrics 696


American Academy of Pediatrics PREP 2015

• ip 5, Chung M, Kulig I, O'Brien R, Sege R, Gicken 5, Maisels Mb Lau I and


Subcommittee on Hyperbilirubinemia. An evidence-based review of important issues
concerning neonatal hyperbilirubinemia. Pediatrics. 2004; 214(1): e130-e153.
• Ross MB, de Alarcon P. Hemolytic disease of the fetus and newborn. NeoReviews.
2013;14(2):e83-e88. doi:10.1542/neo.14-2-e83.
• Watchko if, Tiribelli C. Bilirubin-induced neurologic damage-mechanisms and
management approaches. NEJM. 2013;369(21):2021-2030.
cloi:10.1056/NEIMra1308124.

American academy of pediatrics 697


American Academy of Pediatrics PREP 2015

Item 227
You are evaluating a 5-year-old girl who has no chronic medical problems and is struggling with
reading. Her early development was appropriate, and she does not have difficulties paying
attention. Her physical examination findings are unremarkable. Her hearing and vision test
results are unremarkable. Her father thinks he had dyslexia as a child, but was never formally
diagnosed. An educational psychologist has diagnosed the girl as having dyslexia. The parents
want to know how to proceed with treatment for the girl. They also ask whether her 1-year-old
brother is at risk for dyslexia.

Of the following, the next BEST recommendation is


A. encourage parents to read aloud at home
B. evaluate the girl for trace mineral deficiencies
C. obtain an electroencephalogram for the girl
D. refer the brother for developmental pediatrics evaluation
E. refer the girl for vision therapy

American academy of pediatrics 698


American Academy of Pediatrics PREP 2015

Item 227 TE Preferred Response: A


Dyslexia is a learning disability that typically presents in early elementary school-aged children
who have difficulties in reading, spelling, writing, or remembering words. It is a disorder of
language that has a multifactorial etiology including genetic and environmental factors. Dyslexia
can also occur in children with primary neurologic or genetic disorders, such as epilepsy or
neurofibromatosis type 1. Although dyslexia cannot be prevented, the best way to promote
language development and reading skills is for parents to read aloud to the dyslexic child.

When there is concern for a learning disability, the pediatrician should evaluate for:
• medical causes of learning difficulties (eg, attentional or sleep disorders, seizure disorder)
• dysmorphic features or abnormal neurologic examination findings that may suggest a
neurogenetic syndrome
• sensory deficits that could compound learning difficulties (eg, hearing impairment,
strabismus, amblyopia, or refractive errors)

A learning disability is best diagnosed by a trained, experienced professional, such as an


educational psychologist, developmental pediatrician, or neuropsychologist. An individualized
education plan should be developed, ideally involving members of a multidisciplinary team
including educational therapists, psychologists, teachers, parents, and health care professionals as
needed.

For the girl in the vignette who has no clinical signs of dietary deficiencies or seizures,
laboratory testing and electroencephalography would not be indicated. Vision therapy for
dyslexia is not supported by evidence-based research. The girl's younger brother should be
monitored for signs of language delay or reading difficulties, but he does not require a
developmental pediatrics evaluation at this time.

PREP Pearls
• Dyslexia is a disorder of language and not a visual disorder.
• Vision therapy is not recommended for treatment of dyslexia.

American Board of Pediatrics Content Specification(s)


• Recognize appropriate educational settings for patients with learning disabilities, and the
various strategies utilized in those settings to circumvent weaknesses

Suggested Reading
• Erik von Hahn L. Specific learning disabilities in children: clinical features. UpToDate.
Available online only for subscription.
• Handle SM, Pierson WM, Section on Ophthalmology, et al. Learning disabilities,
dyslexia, and vision. Pediatrics. 2011;127(3):e818-e856. doi:10.1542/peds.2010-3670.

American academy of pediatrics 699


American Academy of Pediatrics PREP 2015

Item 228
A 16-year-old adolescent girl with type 1 diabetes mellitus since 2 years of age is in your clinic
for routine follow-up. She currently sees both you and a pediatric endocrinologist every 3
months to discuss diabetes care. Her hemoglobin A1c is currently 7.4%. She is very satisfied
with her current medical care but recognizes that she will eventually need to see an adult
endocrinologist. You begin to plan for this transition.

Of the following, the BEST next step in this process would be to


A. not bring up the subject of transition to a new physician, because this may upset the
patient and result in poor control of her diabetes mellitus in the teenage years
B. plan to continue to care for this patient in your office, because internal medicine
physicians may not be comfortable managing juvenile-onset diabetes mellitus
C. plan to make an appointment with an adult endocrinologist after she turns 18 years old
D. provide a checklist of shared responsibilities for diabetes care between the young adult
and family members to encourage the development of independent activity
E. refer her to an adult endocrinologist now because of complicated medical issues she may
soon face

American academy of pediatrics 700


American Academy of Pediatrics PREP 2015

Item 228 TE I-C SBP Preferred Response: D


A well-timed transition from child- to adult-oriented health care is specific to each person and
ideally occurs between 18 and 21 years of age. Coordination of patient, family, and provider
responsibilities optimizes the adolescent's ability to assume adult roles and activities.
Recent data show that for some conditions, such as diabetes mellitus, outcomes can worsen in
the years after transition to adult care. In 1 study, adolescents and young adults with type 1
diabetes mellitus who made a transition from pediatric to adult care were 2.5 times more likely to
have chronically high blood glucose levels. In another study, admission rates for diabetes
mellitus-related hospitalizations increased during this transition period. Similar findings have
been reported in other chronic diseases after transition, such as in patients with sickle cell
disease, with higher mortality seen after the transition.

To ensure good continuity of care and prevent this worsening of outcomes, a gradual approach to
transition has been suggested that slowly transfers responsibility for care from the parents to the
patient (Item 0228). First, by 12 to 13 years of age, office transition policies should be discussed
with patients and families. By 14 to 15 years of age, a transition plan should be jointly developed
with the patient, family, and physician. At 16 to 17 years of age, the plan should be continually
reviewed and updated. By 18 to 21 years of age, a transition should be made to the adult
healthcare model. The initial plan can start with a checklist of responsibilities, clearly noting
what should be done by the adolescent and what should be done by the parent. Adolescent
responsibilities can start with more straightforward tasks, such as making appointments and
refilling medications. Over time, the adolescent should assume more responsibility for personally
reviewing and managing their care, such as diabetes mellitus complications and necessary
follow-up. Discussing the costs of medications, and how they will be paid for into adulthood is
very important. Finally, the patient and family should work with their pediatrician or specialist to
identify an adult care provider, and ideally the patient would initiate a request to transfer records
and request a consult. This ensures that visits will not be missed and that the adolescent patient is
ready to participate in the adult healthcare model.

For adolescents with type 1 diabetes mellitus, the American Diabetes Association and National
Diabetes Education Program offer guidelines and checklists on how to best structure this
transition. Additional resources to help patients with any chronic condition to make a transition
are available at www.gottransition.org.

For the 16-year-old adolescent girl in this vignette, making a checklist of shared responsibilities
that will be reviewed and revised over time will help foster her independence and increase her
likelihood of achieving a successful transition.

It would not be appropriate to avoid the subject of transition to adult care, because effective
planning can make the process much easier and more successful. It is also not appropriate to
suggest that the adult healthcare model cannot serve the needs of patients with type 1 diabetes
mellitus.

Referring the patient to an adult specialist too early will likely lead to many missed appointments
if the transition does not go smoothly and waiting to refer the patient until she is 18 years of age
without a detailed transition plan will likely have similar results.
American academy of pediatrics 701
American Academy of Pediatrics PREP 2015

PREP Pearls
• Coordination of patient, family, and provider responsibilities optimizes the adolescent's
ability to assume adult roles and activities.
• A successful transition to adult health care should involve gradually transferring
responsibility for care from the parents to the patient.
• Many organizations offer guidelines and resources regarding the transition of adolescent
patients with specific medical conditions to adult health care.

American Board of Pediatrics Content Specification(s)

American academy of pediatrics 702


American Academy of Pediatrics PREP 2015

• Provide appropriate guidance to transition adolescents from pediatric to adult health care

Suggested Reading
• American Academy of Pediatrics, American Academy of Family Physicians, American
College of Physicians. Transitions Clinical Report Authoring Group, Cooley WC,
Sagerman PJ. Supporting the health care transition from adolescence to adulthood in the
medical home. Pediatrics. 2011;128(1):182-200. doi:10.1542/peds.2011-0969.
• Center for Health Care Transition Improvement. Got transition homepage. The National
Alliance to Advance Adolescent Health website.
• Lotstein DS, Seid M, Klingensmith G, et al. Transition from pediatric to adult care for
youth diagnosed with type 1 diabetes in adolescence. Pediatrics. 2013;131(4):e1062-
e1070. doi:10.1542/peds.2012-1450.
• Nakhla M, Daneman D, To T, Paradis G, Guttmann A .Transition to adult care for youths
with diabetes mellitus: findings from a Universal Health Care System. Pediatrics.
2009;124(6):e1134-e1141. doi:10.1542/peds.2009-0041.
• National Diabetes Education Program. Pediatric to adult health care transition planning
checklist. National Institutes of Health website. Accessed January 13, 2014.
• Peters A, Laffel L; American Diabetes Association Transitions Working Group. Diabetes
care for emerging adults: recommendations for transition from pediatric to adult diabetes
care systems. Diabetes Care. 2011;34(11):2477-2485. doi:10.2337/dc11-1723.

American academy of pediatrics 703


American Academy of Pediatrics PREP 2015

Item 229
A 3-year-old boy visits your office for a new patient visit. His mother reports that he has had
very dry, scaly skin since infancy that has not improved with trials of various over-the-counter
moisturizing creams. His mother reports that her brother also had very scaly skin (Item Q229),
and both she and her brother have small cataracts that do not interfere with vision. On physical
examination, the boy's skin is remarkable for a widespread fine brown scale, especially
prominent in flexor joint creases, on the sides of the neck, the flanks, and in front of the ears. The
palms and soles are not affected. You also detect a small corneal opacity in his right eye.

Of the following, in addition to referral to ophthalmology, the BEST next treatment to prescribe
for this child is
A. calcineurin inhibitor
B. high-potency topical corticosteroid
C. topical antibiotic ointment
D. topical retinoid
E. urea-containing emollient

American academy of pediatrics 704


American Academy of Pediatrics PREP 2015

Item 229 Preferred Response: E


Ichthyosis occurs in various forms, all involving abnormal skin keratinization, barrier function,
and desquamation that produce the characteristic scaly skin of these disorders. Treatment is
aimed at moisturizing and softening the skin and promoting desquamation. Urea cream is one of
the most effective products to treat ichthyosis and therefore would be the most appropriate initial
treatment for the child in the vignette.

Ichthyosis is often categorized genetically and can be autosomal dominant, autosomal recessive,
X-linked, syndromic, or sporadic. Recent evidence implicates mutations of the filaggrin gene,
also important in atopic dermatitis, with autosomally inherited ichthyosis. The most common
form is ichthyosis vulgaris, an autosomal dominant condition with onset after the newborn period
(often during the first year of life or early childhood). It is characterized by fine white or tan
scales most prominent on the lower legs, while sparing the antecubital and popliteal fossae.
Patients also demonstrate hyperlinearity of the palms and soles. They frequently have comorbid
atopic dermatitis and keratosis pilaris.

The patient in the vignette demonstrates characteristic findings of X-linked ichthyosis, which is
associated with an absence of cholesterol sulfate sulfhydrolase. The onset of this form of
ichthyosis can occur at birth with widespread erythema and desquamation. As the patient
matures, he develops brown, polygonal scales in the flexures, preauricular, lateral neck and flank
areas. There is no palmar or plantar involvement. Corneal opacities are common both among
patients (50%) and carriers (30%), and patients also have an increased risk of testicular cancer.
Lamellar ichthyosis is a rare condition (incidence of about 1:300,000), but is among the most
common autosomal recessive forms of ichthyosis. It is one of the causes of "collodion baby; in
which the newborn is covered by a membrane and exhibits ectropion and small, misshapen ears.
Hair may be sparse and palms and soles are thickened; teeth and mucosal surfaces are normal.
Scales are large, dark, rectangular, and adherent to the entire surface, including flexor creases.
The other common type of collodion baby occurs with congenital ichthyosiform erythroderma.
These infants demonstrate erythroderma, which is not seen in lamellar ichthyosis. Scales are
finer and whiter in color. There is hyperkeratosis of the knees, elbows, ankles, palms, and soles.
Hair is sparse and nails are dystrophic.

In addition to these more common forms of ichthyosis, a vast array of syndromes exist in which
ichthyosis is a component. In these syndromes, ichthyosis can be associated with disorders of the
eyes, bones, central nervous system, growth, and development. Affected children may also have
sensorineural hearing loss and may be born prematurely.

The goal of treatment for most children suffering from ichthyosis is to support skin barrier
function and reduce pruritus by use of moisturizing agents. Among the most effective classes of
products to achieve moisturization, as well as keratolysis, are urea and lactic acid creams.
Topical retinoids have been useful for some patients, but are irritating and expensive to apply
over a large area; they are not first-line therapy for ichthyosis. Topical steroids or calcineurin
inhibitors may be useful, especially for patients who also have atopic dermatitis, but would be
second-line therapy for ichthyosis. Steroids should be used at the lowest potency that is effective.
Topical antibiotics are not indicated unless there is a secondary bacterial infection.

American academy of pediatrics 705


American Academy of Pediatrics PREP 2015

PREP Pearls
• The ichthyoses are a set of disorders with abnormal skin keratinization and barrier
function, characterized by scaling.
• Ichthyoses may be autosomal dominant, recessive, X-linked, sporadic, or syndromic.
• Urea and lactic acid creams are effective first-line treatments for ichthyoses.
• The most common form of ichthyosis is the autosomal dominant ichthyosis vulgaris,
characterized by white scale prominent on the lower extremities.
• X-linked ichthyosis presents with brown adherent scale on the neck, flank, face, and
flexor creases, and is often associated with lens opacities.

American Board of Pediatrics Content Specification(s)


• Recognize the clinical findings associated with ichthyosis

Suggested Reading
• Krowchuk DP, Mancini Al. Ichthyosis. In: Krowchuk DP, Mancini AJ, eds. Pediatric
Dermatology: A Quick Reference Guide. 2nd ed. Elk Grove Village IL: American
Academy of Pediatrics; 2011:441-448.
• Epps RE. Atopic dermatitis and ichthyosis. Pediatr Rev. 2010;131(7):278285.
doi:10.1542/pir.31-7-278.
• Fleckman P, Newell BD, van Steensel MA, Yan AC. Topical treatment of ichthyoses.
Dermatol Then 2013;2611):16-25. dal: 10.1111/j.1529-8019.2012.01526.x.
• Morelli JG. Disorders of keratinization. In: Kliegman RM, Stanton BMD, St. Geme JW
Ill, Schor NF, Behrman RE, eds. Nelson Textbook of Pediatrics. 19th ed. Philadelphia,
PA: Saunders Elsevier; 2011:2267-2273.

American academy of pediatrics 706


American Academy of Pediatrics PREP 2015

Item 230
The mother of a 15-year-old adolescent girl is concerned because her daughter's school
performance has been deteriorating. The girl has been irritable with occasional outbursts,
especially when her mother enters her bedroom. Her mother has noticed a lot of art supplies in
her room, but no artwork being produced. She also notices some stains on her daughter's hand. In
talking to the girl alone, she admits to using inhalants.

Of the following, the MOST likely consequence if this girl continues this behavior is
A. cardiac arrhythmias
B. chronic diarrhea
C. encephalopathy
D. pneumonia
E. pneumothorax

American academy of pediatrics 707


American Academy of Pediatrics PREP 2015

Item 230 S Preferred Response: C


Young adolescents who experiment with drugs may begin with inhalant abuse, but most will
move on to other substances and discontinue this practice. However, some may continue use and
go on to suffer the consequences of chronic use. The most significant effect of chronic use of
inhalants is on the central nervous system with widespread and long-lasting changes, including
deficits in cognition, vision, hearing, and motor function. Other organs that may be damaged
include the heart, lungs, liver, kidneys, and bone marrow.

Cardiac arrhythmias with sudden death may occur with only a one-time use of inhalants.
Diarrhea is not a side effect, but nausea, vomiting, headaches, and dizziness often occur after
each use. Respiratory symptoms may include choking from aspiration of vomitus. Suffocation
may occur if a plastic bag over the head is used to inhale fumes. Asphyxiation is possible
because of high concentration of gases that can displace oxygen with repeated inhalations.
Pneumonia and pneumothorax are not direct consequences of chronic inhalant use.
There are 4 categories of inhaled substances: volatile solvents (eg, paint thinners), aerosols (eg,
propellants in spray paint), gases (eg, butane lighters), and nitrites. Nitrites are a class of
inhalants that are abused to enhance sexual pleasure and performance. They act by relaxing
muscles and dilating blood vessels, rather than by acting on the central nervous system like other
inhaled substances. Nitrites can be associated with unsafe sexual practices that increase the risk
of contracting and spreading infectious diseases like HIV/ AIDS and hepatitis. The following
signs may alert an adult to the use of inhalants by youth: the presence of an odor on the breath,
contact dermatitis around the lips, rags or containers with the volatile substance, or marks on the
skin from paint.

PREP Pearls
• Inhalant abuse often goes undetected, because the effects can be transient.
• Recurrent inhalant use causes damage to multiple organ systems, the most significant of
which is the central nervous system.

American Board of Pediatrics Content Specification(s)


• Identify the major physiologic consequences associated with inhalant use/abuse
• Understand the variety of substances used as inhalants

Suggested Reading
• Crocetti M. Inhalants. Pediatr Rev. 2008;29(0;33-34. doi:10.1542/pir. 29-1-33.
• National Institute on Drug Abuse. Research report series: inhalant abuse. National
Institutes of Health website.

American academy of pediatrics 708


American Academy of Pediatrics PREP 2015

Item 231
An otherwise healthy 2-year-old child, who has a documented milk allergy, is admitted to the
hospital for chronic constipation. The child was given a milk and molasses enema per the orders
of the resident team. The child had an anaphylactic reaction that required treatment with
epinephrine and diphenhydramine, and transfer to the pediatric intensive care unit for
observation. The multidisciplinary team undertakes a root cause analysis to determine if latent or
active errors may be uncovered.

Of the following, the MOST likely to represent an active error would be


A. the electronic medical record did not alert the resident regarding milk allergy when the
order was placed
B. the medical student was too intimidated to point out the history of milk allergy to the
attending physician
C. the overworked nurse had the nurse's aide administer the enema
D. the pharmacist could not find the documentation regarding the milk allergy
E. the resident who wrote the order for the milk and molasses enema disregarded the history
of milk allergy

American academy of pediatrics 709


American Academy of Pediatrics PREP 2015

Item 231 SBP Preferred Response: E


In this scenario, active error was committed by the resident who ordered the milk and molasses
enema in this child with a history of milk allergy. Active errors occur at the point of interface
between humans and a complex system, are generally readily evident (eg, pushing an incorrect
button, ignoring a warning signal), and almost always involve someone at the frontline. "Active"
and "latent" errors are unearthed during root cause analysis (RCA), a structured method used to
analyze serious adverse events. A central principle of RCA in health care (and in industry where
it was originally developed) is to identify underlying "system" problems that increase the
likelihood of errors. The goal of the RCA is to identify active errors (that occur at point of
contact between humans and some aspect of a larger system) and latent errors (the less apparent
problems within systems that contribute to adverse events). Active errors are sometime referred
to as errors at the "sharp" end and are noticed first because they are consequent to the actions of
the worker. Latent errors are those that are present distant from the adverse event (the "blunt"
end) and may be secondary to equipment design and organizational flaws. Examples of latent
errors in this vignette include the lack of electronic medical record alert, the medical student
feeling too intimated to point out the error, the overworked nurse had the nurse's aide administer
the enema without informing the aide of the milk allergy, and the pharmacist failing to find
documentation of milk allergy.

Root cause analysis is reported to be one of the most widely used approaches to improve patient
safety, though data to support its effectiveness is limited. The goal of the RCA is to prevent
future harm by eliminating the latent errors that underlie adverse events. The process of RCA
occurs within a framework of a prespecified protocol beginning with data collection and
reconstruction of the event through reviews of records and participant interviews. A
multidisciplinary team then analyzes the sequence of events leading to the error, with the goal of
identifying "how" the event occurred (identification of the active error) and "why" it occurred
(identification of latent errors). Since 1997, the Joint Commission has mandated use of RCA to
analyze sentinel events (such as wrong site surgery). As of April 2007, 26 states have mandated
reporting of serious adverse events (increasingly using the National Quality Forum's list of
"never events:' http: //psnet.ahrq. gov/primer. aspx?primerID=10).

PREP Pearls
• Root cause analysis is a structured method of analysis of serious adverse events.
• An active error occurs at the point of contact of the human with the complex healthcare
system.
• Latent errors refer to health care system flaws that occur remote to the event, but are
contributory to the adverse event. Factors that may lead to latent errors include those
pertaining to institutional, regulatory, work environment, team environment, staffing,
task-related, and patient characteristics.

American Board of Pediatrics Content Specification(s)


• Understand and apply root cause analysis to determine the factors contributing to an error

Suggested Reading

American academy of pediatrics 710


American Academy of Pediatrics PREP 2015

• Agency for Healthcare Research and Quality. Root cause analysis. US Department of
Health and Human Services website. Updated October 1, 2012. Accessed November 28,
2013.
• Wu AW, Lipshutz AK, Pronovost PJ. Efficiency and effectiveness of root cause analysis
in medicine. JA MA. 2008;299(6):685-687. doi:10.1001/ jama.299.6.685.

American academy of pediatrics 711


American Academy of Pediatrics PREP 2015

Item 232
A 12-month-old boy is brought to the emergency department because of bluish discoloration of
the skin. The mother first noticed the child's lips turning slightly blue 2 days ago. His lips and
mucous membranes have become progressively blue and now involves the fingernails and
extremities. The mother did not seek medical attention sooner because he otherwise seems like
his normal self. There is no cough, runny nose, difficulty breathing, red mucous membranes, or
rash. He has been fussier for the past week because he is teething. He has been normally active
and eating and drinking well, and he does not fatigue easily. He is a product of a full-term
delivery with no problems. He has been thriving and has never been hospitalized. His
immunizations are up to date, and he has no allergies. Medications include multivitamin drops,
as well as as-needed doses of acetaminophen and topical benzocaine for teething pain.
Vital signs show a temperature of 37.5°C, heart rate of 140 beats/min, respiratory rate of 30
breaths/min, blood pressure of 90/50 mm Hg, and SpO2 of 85% on room air. Physical
examination shows a generally well-developed, well-nourished child in no apparent distress.
Oral mucous membranes are moist and not erythematous, but have a blue discoloration. He is
breathing comfortably, and his lungs are clear to auscultation bilaterally. His heart has a regular
rate and rhythm with no rubs, gallops, or murmurs. Extremities are warm and well perfused with
no edema. There is obvious cyanosis of lips, nail beds, and skin. There is no digital clubbing.

Of the following, the MOST likely cause of his cyanosis is


A. anemia
B. left to right intracardiac shunt
C. methemoglobinemia
D. right to left intracardiac shunt
E. ventilation-perfusion mismatch

American academy of pediatrics 712


American Academy of Pediatrics PREP 2015

Item 232 Preferred Response: C


The child in the vignette has methemoglobinemia, marked by bluish discoloration of the lips and
mucous membranes, oxygen saturation of 85%, and no evidence of primary pulmonary
dysfunction. In this case, the methemoglobinemia is likely because of the topical benzocaine that
was given for teething.

Pulse oximetry (SpO2) is the most common technique used to monitor oxygenation. Pulse
oximetry is noninvasive, therefore it is used to monitor children during surgery, transport,
procedural sedation, and critical illness. Pulse oximetry estimates the saturation of hemoglobin
with oxygen by measuring the differential light absorption of oxyhemoglobin and
deoxyhemoglobin at 660 nm (red) and 940 nm (infrared) wavelengths. This technique requires
pulsatile flow to detect oxyhemoglobin, estimating arterial saturation (Sa O2). Changes in arterial
pressure of oxygen (PaO2) higher than 80 mm Hg are not likely to reflect changes in Sa O2
because of the shape of the hemoglobin-oxygen dissociation curve (Item C232). Also, the same
PaO2 level may lead to different SpO2 levels, because the affinity of hemoglobin to oxygen is
dependent on various factors. For example, acidosis, higher temperature, and elevated levels of
2,3-bisphosphoglyceric acid (2,3-BPG) lowers the affinity of hemoglobin to oxygen, favoring its
unloading in tissues. In contrast, alkalosis, lower temperature, and lower 2,3-BPG increase the
affinity of hemoglobin to oxygen, favoring its uptake.

Pulse oximetry should not be the only method of monitoring patients with primary ventilatory
failure, such as neuromuscular conditions and central nervous system depression, because
patients with satisfactory SpO2 may have dangerous levels of hypercarbia if they are receiving
supplemental oxygen. Other limitations of pulse oximetry include decreased signal from motion,
external light, anemia, and decreased perfusion and pulsatility.

Methemoglobin is the oxidized, ferric (Fe3+) state of hemoglobin, as opposed to the normal
ferrous (Fe2+) state. It does not have oxygen-carrying capacity; and thus does not unload oxygen
to the tissues. The oxidation of hemoglobin to methemoglobin is caused by topical anesthetics,
gastroenteritis, exposure to nitric oxide, or ingestion of nitrite-containing well water.
Methemoglobin levels higher than 20% can lead to symptoms of fatigue, dizziness, and nausea.
Life-threatening manifestations, such as arrhythmias, seizures, and altered mental status, may
occur at levels greater than 45%. Treatment includes removal of the inciting agent, and
administration of methylene blue, which is a cofactor of nicotinamide adenine dinucleotide
phosphate (NADP)-methemoglobin reductase. Pulse oximetry may provide inaccurate
information about SpO2 in methemoglobinemia because pulse oximetry recognizes all types of
hemoglobin as either oxyhemoglobin or deoxyhemoglobin. Methemoglobin absorbs light in both
the oxygenated and deoxygenated wavelengths. As a result, the reported SpO2 by pulse oximetry
is typically around 85%, irrespective of the true SpO2 Inaccurate information may also be
provided by pulse oximetry in the presence of carboxyhemoglobin because it absorbs light in the
same (red) wavelength as oxyhemoglobin, leading to overestimation of the oxygen saturation.
Diagnosis of methemoglobinemia and carboxyhemoglobinemia is best established by co-
oximetry, which involves blood gas analysis of oxyhemoglobin, deoxyhemoglobin,
methemoglobin, and carboxyhemoglobin levels.

American academy of pediatrics 713


American Academy of Pediatrics PREP 2015

Anemia does not cause cyanosis. On the contrary, central cyanosis is usually visible when 5 g/dL
(50 g/L) of hemoglobin is desaturated, so it can be more readily seen at a higher percentage of
oxygen saturation if the patient has polycythemia. A left-to-right intracardiac shunt alone does
not cause cyanosis. A right-to-left intracardiac shunt is not likely in this case because the child is
well nourished and has no murmur indicative of a shunt. Ventilation-perfusion mismatch can
cause cyanosis if areas of the lung are perfused but not ventilated, such as in pneumonia,
atelectasis, and pulmonary edema, but in this case, the patient is breathing comfortably and has
clear lungs.

Methemoglobinemia and carboxyhemoglobinemia are conditions in which pulse oximetry does


not reliably represent the arterial oxygen saturation and can be best diagnosed by co-oximetry.

PREP Pearls
• Pulse oximetry is an accurate, noninvasive method of monitoring oxygenation.
• SpO2 may not provide an accurate representation of true arterial oxygen saturation in
methemoglobinemia and carboxyhemoglobinemia.
• Acidosis, higher temperature, and elevated levels of 2,3-bisphosphoglyceric acid lowers
the affinity of hemoglobin to oxygen, favoring its unloading in tissues.

American Board of Pediatrics Content Specification(s)


• Understand the correlation between PaO, and oxyhemoglobin concentration
• Understand the value and limitations of pulse oximetry

Suggested Reading
• Mack E. Focus on diagnosis: co-oximetry, Pediatr Rev. 2007;28(473-74.
doi:10.1542/pir.28-2-73. Moussa S. Huffman C. Index of suspicion. Pediatr Rev.

American academy of pediatrics 714


American Academy of Pediatrics PREP 2015

2009;30(6):217-222. doi:10.1542/pir.30-6-217, Voter KZ, McBride IT. Back to basics:


diagnostic tests of lung function. Pediatr Rev. 1996;17(453-63. doi:10.1542/pir.17-2-53.

American academy of pediatrics 715


American Academy of Pediatrics PREP 2015

Item 233
A 15-year-old adolescent boy in your practice has sustained open fractures of his radius and
ulnar bones after falling off his bicycle on a dirt trail. He has a history of urticarial reaction to
penicillin. He is being prepared to be taken to the operating room for debridement and
stabilization. You are asked by the orthopedist to recommend antibiotic coverage for him.

Of the following, the BEST choice of therapy is


A. cefazolin and gentamicin
B. piperacillin- tazobactam
C. to await culture results prior to instituting antibiotics
D. vancomycin and ciprofloxacin
E. vancomycin and nafcillin

American academy of pediatrics 716


American Academy of Pediatrics PREP 2015

Item 233 Preferred Response: D


In a dirty wound such as the open fracture described for the adolescent boy in this vignette,
prompt institution of antibiotic therapy in combination with surgical debridement, profuse
irrigation, and stabilization of the radius and ulna are important components of treatment. In this
setting, polymicrobial contamination with skin and soil organisms including Staphylococcus
aureus, streptococci, enteric gram-negative organisms, and anaerobes need to be considered. Of
the regimens listed, vancomycin and ciprofloxacin provide broad spectrum coverage for these
types of organisms and avoids use of a β-lactam drug in a patient with a history of urticarial
reaction to penicillin. In a dirty wound, antibiotic therapy should not be delayed while awaiting
results of cultures. Additionally, this patient's tetanus vaccine status should be reviewed, and if it
is more than 5 years or unknown duration since his last dose, a booster dose of tetanus vaccine
should be given.

A first generation cephalosporin such as cefazolin in combination with gentamicin provides


reasonable broad spectrum coverage, except for community-acquired, methicillin-resistant S
aureus (MRSA). Additionally, cross reaction between penicillin and a first generation
cephalosporin would be a concern given the boy's history of urticarial reaction to penicillin.
Piperacillin-tazobactam provides good broad spectrum coverage for this kind of injury, except
for MRSA, but it is a penicillin. Vancomycin and nafcillin as a combination do not provide
gram-negative coverage, and again, nafcillin is a penicillin derivative.

Other soft tissue wounds may arise from abrasions, lacerations, crush, puncture, or bite injuries.
Prompt cleaning of such wounds with tap water, sterile water, or sterile saline is effective in
preventing infection. Removal of any foreign body or material is also important. Prophylactic
antibiotics may be indicated for bite wounds to the hands, feet, and possibly the face. Topical
antibiotic ointment may be used for an abrasion. If a wound infection occurs, antibiotic coverage
should be directed at skin flora (Staphylococcus, Streptococcus) and other possible organisms
based on the nature of the injury (eg, Pasteurella multocida with dog or cat bites).

PREP Pearls
• Open fracture wounds and bite wounds to the hands, feet, and possibly the face require
prompt, broad spectrum antibiotic overage.
• Most wounds in children should be managed with irrigation and do not require antibiotic
prophylaxis. If they become infected, coverage for skin flora (Staphylococcus and
Streptococcus) and other possible organisms, based on the nature of the injury, should be
considered.

American Board of Pediatrics Content Specification(s)


• Plan the appropriate management of a skin infection resulting from a wound

Suggested Reading
• Sagerman Pt Wounds. Pediatr Rev 2005;26(443-49. doi:10.1542/pir.26-2-43. Spiro DM,
Zonfrillo MR, Meckler GD. Wounds. Pediatr Rev. 2010;31(8):326-344.
doi:10.1542/pir.31-8-326.

American academy of pediatrics 717


American Academy of Pediatrics PREP 2015

Item 234
A 6-year-old boy presents because of diarrhea for the past year. The child was toilet trained at
age 3 years, but he has been soiling his underwear for the past year. During meals, he frequently
complains of abdominal pain, and his abdomen becomes distended. He has experienced no
fevers, other illnesses, or weight loss. Physical examination demonstrates a well-developed child
whose height is 43 cm (50th percentile) and weight is 19 kg (25th percentile). He has a
protuberant abdomen and normal bowel sounds with a fullness noted in the left lower quadrant.
Rectal examination is deferred because of a large amount of soft stool in the child's underwear.

Of the following, the MOST likely cause of this child's symptoms is


A. celiac disease
B. fecal overflow incontinence
C. giardiasis
D. Hirschsprung disease
E. irritable bowel syndrome

American academy of pediatrics 718


American Academy of Pediatrics PREP 2015

Item 234 Preferred Response: B


In the vignette, a 6-year-old boy presents with a year-long complaint of abdominal pain during
meals and daytime fecal soiling. His growth and development are normal, and additional history
demonstrates that he was toilet trained at 3 years of age. Although a thorough history is required
as part of his complete evaluation, with particular emphasis on the time after birth of meconium
passage and his subsequent stooling pattern, the information in this vignette strongly suggests a
diagnosis of encopresis secondary to chronic constipation. The physical finding of a stool mass
in his left lower quadrant, plus the presence of semiliquid stool in the boy's underwear, further
suggest that the 'diarrhea" reported by the parents represents overflow fecal incontinence.

Encopresis is defined as fecal soiling in children who are beyond the age of toilet training.
Functional fecal incontinence in children may be further classified as either primary, affecting
children who have never been toilet trained, or secondary, as in the vignette, presenting in the
child where incontinence returns after successful toilet training. In most patients, considering
each child's neurodevelopmental status, this diagnosis may be applied to soiling episodes that
occur in the child older than 4 years of age. In 90% of cases, encopresis occurs as the
consequence of chronic constipation in a child who demonstrates stool-withholding behavior.
Although encopresis is estimated to affect 1% to 2% of children under the age of 10 years, with
an 80% male predominance, a complaint of constipation and related problems may account for
more than 25% of all visits to a pediatric gastroenterologist. In otherwise healthy children,
incontinence is most likely to represent fecal overflow around a retained stool mass. Soiling may
be reported as diarrhea by some parents; however, a pattern of stool-withholding behavior will
usually be elicited, indicating a "nonorganic" diagnosis of functional constipation-related
encopresis. The typical history for the child with secondary fecal incontinence includes the
following characteristics:
• The child does not have the urge to defecate immediately prior to soiling.
• Encopresis occurs almost exclusively during the day, most commonly in the afternoon or
when returning home from school.
• Overflow incontinence occurs around a stool mass, and therefore the child intermittently
passes a very large bowel movement that often clogs the toilet.

Fecal incontinence in children may also occur in the setting of a number of organic diagnoses,
including anorectal malformations and myelomeningocele. In most cases, these problems will be
clearly evident from the history and physical examination. Other conditions, including tethered
cord, chronic malabsorption (eg, cystic fibrosis, celiac disease), or an endocrinopathy, should be
considered in any child who presents without a typical history of constipation. Item C234 lists
some of the physical findings that may be associated with an organic cause of fecal incontinence.

The other diagnoses presented in the response choices are clearly not suggested by the boy's
history and physical examination. The finding of a stool mass on physical examination rules out
any chronic infectious process such as giardiasis. The boy's normal growth and development,
with "diarrhea" and soiling developing at the age of 5 years, are inconsistent with a diagnosis of
celiac disease. Because he was toilet trained at age 3 years, without an apparent history of
abnormal defecation before age 5 years, Hirschsprung disease is also very unlikely. In infants
and toddlers, irritable bowel syndrome (also referred to as "toddler's diarrhea") is most often
associated with high carbohydrate-containing diets or consumption of excessive amounts of fluid
American academy of pediatrics 719
American Academy of Pediatrics PREP 2015

(particularly fruit juices), with its onset usually before 3 years of age. The finding of a stool mass
on physical examination is inconsistent with this diagnosis.

Item C234. Physical Findings Associated With Organic Constipation


• Abdominal distension
• Abnormal lumbosacral spine curvature or pigmentation
• Absent cremasteric reflex
• Absent or delayed lower extremity deep-tendon reflexes
• Anal abnormalities
- Absent anal wink
- Anterior anal displacement
- Patulous anal sphincter
- Hypertonic anal sphincter ("Milkmaid's sign") with empty rectal vault
• Decreased lower extremity tone and/or strength
• Midline pigmentary abnormalities of the lower spine
• Occult blood in stool
• Poor growth and weight gain
• Presacral dimple covered by a tuft of hair
• Sacral agenesis

Adapted and reprinted with permission. Baker 55, Di Lorenzo C, Liptak GS, et al. Evaluation
and treatment of constipation in infants and children: recommendations of the North American
Society for Pediatric Gastroenterology, Hepatology and Nutrition. I Pediatr Gastroenterat and
Nutr 2006; 43:e4 -el 3.

PREP Pearls
• In a child presenting with diarrhea, the physical finding of a left lower quadrant (fecal)
mass suggests a diagnosis of functional fecal incontinence.
• Encopresis occurs with an 80% male predominance.
• In the child with encopresis, soiling episodes rarely occur during sleep.

American Board of Pediatrics Content Specification(s)


• Distinguish between encopresis and delayed bowel training
• Recognize the clinical features associated with fecal overflow incontinence

Suggested Reading
• Baker SS, Di Lorenzo C, Liptak GS, et at. Evaluation and treatment of constipation in
infants and children: recommendations of the North American Society for Pediatric
Gastroenterology, Hepatology and Nutrition. J Pediatr Gastroenterol and Nutr
2006;43(3):el-e13. doi: 10.1097/01.mpg.0000233159.97667.c3.
• Bekkali NL, van den Berg MM, Dijkgraaf MG, et al. Rectal fecal impaction treatment in
childhood constipation: enemas versus high doses oral PEG. Pediatrics.
2009;124(6):e1108-e1115. doi:10.1542/peds.2009-0022.

American academy of pediatrics 720


American Academy of Pediatrics PREP 2015

• Borowitz SM, Cox DJ, Sutphen IL, Kovatchev B. Treatment of childhood encopresis: a
randomized trial comparing three treatment protocols. / Pediatr Gastroenterol Nutr.
2002;34(4):378-384.
• Di Lorenzo C, Benninga MA. Pathophysiology of pediatric fecal incontinence.
Gastroenterology. 2004;126(1 suppl 1):S33-S40. doi:10.1053/j. gastro.2003.10.012.
• Nurko S, Scott SM. Coexistence of constipation and incontinence in children and adults.
Best Pratt Res Clin Gastroenterol. 2011;25(1):29-41. doi:10.1016/j.bpg.2010.12.002.
• Reid H, Bahar RI. Treatment of encopresis and chronic constipation in young children:
clinical results from interactive parent-child guidance. Clin Pediatr (Phila).
2006;45(2):157-164. doi:10.1177/000992280604500207.

American academy of pediatrics 721


American Academy of Pediatrics PREP 2015

Item 235
A 30-year-old mother delivers a full-term neonate with a birth weight of 2,930 g. On physical
examination, the neonate has short limbs, narrow thorax, dark gray sclera, significant bowing of
the lower and upper extremities, and a head that appears proportionately larger than the rest of
the body with frontal bossing. The baby has shallow orbits, a small nose, and a low nasal bridge
(Item Q235A).

Radiologic imaging reveals short, thick, ribbon-like,


poorly mineralized long bones with bowing, multiple
fractures, callus formation, and beaded ribs. (Item
Q235B) (Item Q235C). Craniofacial radiographs show
poorly mineralized calvarium.

American academy of pediatrics 722


American Academy of Pediatrics PREP 2015

Of the following, this neonate is MOST at risk for


A. aortic root dilatation
B. genitourinary anomalies
C. intellectual disability
D. pseudoglioma
E. respiratory failure

American academy of pediatrics 723


American Academy of Pediatrics PREP 2015

Item 235 TE Preferred Response: E


The patient in the vignette has osteogenesis imperfecta (OI) type III, or progressively deforming
OI. This disorder typically presents at birth with significant bowing of the extremities and flexed
hips, thin cortices, thin beaded ribs, severe osteoporosis, Wormian bones in the skull, codfish
vertebrae, multiple fractures, blue or grey sclerae, small-for-gestational-age weight and length,
and relative macrocephaly. The skull is unusually soft with a triangular-shaped face. Skeletal
survey and clinical examination are diagnostic. Neonates can sustain fractures and easy bruising
by simple caregiver handling. Some affected individuals do not survive early infancy because of
pulmonary failure resulting from the sheer number and severity of the rib fractures. In infants
who do survive early infancy, the patient will have long-term needs for mobility assistance
because of the severe bone deformity and marked bone fragility. Affected individuals have
extremely short stature, with some adults reaching a length of less than 3 ft. Intellect is typically
normal. Hearing loss, both conductive and sensorineural, typically develops after puberty
because of fractures in the middle ear bones. Hearing should be closely monitored every 3 to 5
years beginning at age 5 years. Patients should be screened via clinical examination and history
for basilar impression, which is an upward protrusion of the top of the spine into the base of the
skull, causing neurologic symptoms such as headache, double vision, poor coordination, or limb
weakness.

All patients with OI type III have dentinogenesis imperfecta. Dental care should focus on
maintenance of primary and permanent dentition, gingival health, and correction of
malocclusion. Patients require multidisciplinary involvement from orthopedics, otolaryngology,
rehabilitation specialists, pediatric dentists, medical geneticists, and primary care physicians.
Bisphosphonates have been widely used in children with severe OI, but their role in amending
the natural history of OI is still under investigation. On the most recent Cochrane Collaboration,
this therapy was found to have a positive impact on bone density, but did not reduce fracture
incidence.

There are 4 different OI subtypes associated with COL1A1 or COL1A2 autosomal dominant
gene mutations. They are as follows: OI type 1, classic nondeforming CH with blue sclerae; OI
type II, perinatally lethal OI; OI type III, progressively deforming OI; and OI type IV, common
variable OI with normal sclerae. The most common type is the classic nondeforming OI with
blue sclerae. This subtype is mild, has a normal lifespan, can attain near-normal stature, has
ligamentous laxity, and does not typically have bony deformity present OI type II is a perinatally
lethal condition with neonates rarely surviving beyond 1 week of age. Most die from pulmonary
insufficiency as a direct result of a narrow thorax, multiple rib fractures, or a flail chest.
Osteogenesis imperfecta is commonly diagnosed with a pertinent family history, fracture history,
typical physical examination findings, radiographic findings, and molecular genetic testing, or
type 1 collagen biochemical analysis.

With OI types II and III, the most common cause of neo-natal mortality is respiratory distress
secondary to pulmonary insufficiency. Genitourinary anomalies and pseudoglioma are not seen
in association with OI. Aortic root dilation has been rarely reported in OI , but is not a common
manifestation.

American academy of pediatrics 724


American Academy of Pediatrics PREP 2015

PREP Pearls
• With osteogenesis imperfecta (OI ) types Hand III, the most common cause of neonatal
mortality is respiratory distress secondary to pulmonary insufficiency from the narrow
thorax and multiple rib fractures.
• A clinical diagnosis of OI is supported by the presence of a number of consistent features
including multiple fractures associated with little to no trauma, short stature, bony
deformity (except in type blue or grey sclerae, dentinogenesis imperfecta, ligamentous
laxity, triangular-shaped facies, and progressive hearing loss after puberty.
• Patients require multidisciplinary involvement from orthopedics, otolaryngology,
rehabilitation specialists, pediatric dentists, medical geneticists, and primary care
physicians.
• In any patient presenting with multiple fractures, one must exclude the possibility of child
abuse.

American Board of Pediatrics Content Specification(s)


 Recognize the clinical findings associated with osteogenesis imperfecta

Suggested Reading
• Ben Amor IM, Glorieux FH, Rauch F. Genotype-phenotype correlations in autosomal
dominant osteogenesis imperfecta. I Osteoporos.
• 2011;2011:540178. doi:10.4061/2011/540178.
• Jenny C. Evaluating infants and young children with multiple fractures. Pediatrics.
2006;118(3):1299-1303. doi:10.1542/peds.2006-1795.
• Phillipi CA, Remmington T, Steiner RD. Bisphosphonate therapy for osteogenesis
imperfecta. Cochrane Database Syst Rev. 2008;(4):CD005088.
doi:10.1002/14651858.CD005088.pub2.
• Steiner RD, Adsit J, Basel D. COL1A1/2-related osteogenesis imperfecta. Gene Reviews.
Accessed January 22, 2014.

American academy of pediatrics 725


American Academy of Pediatrics PREP 2015

Item 236
The parents of a healthy 3.5-year-old girl report problems with toilet training. They began toilet
training at 2 years of age. After 3 months of training, the patient learned to urinate in the toilet.
Now she still has a few accidents during the day, especially when she is busy playing. The
mother admits that she is less diligent about reminding the patient to void frequently because the
mother is caring for the new baby in the family. The patient also continues to have nocturnal
enuresis 3 or 4 times per week. She has been healthy and has grown well. A review of systems
and findings on the physical examination are negative.

Of the following, your BEST anticipatory guidance is that


A. female children are more likely to be delayed in urinary continence
B. occasional daytime and nocturnal accidents are normal at this age
C. organic etiologies for primary nocturnal enuresis are found in 7% of children
D. the patient is being defiant and better discipline is needed
E. primary nocturnal enuresis occurs in 30% of 5 year olds

American academy of pediatrics 726


American Academy of Pediatrics PREP 2015

Item 236 I-C Preferred Response: B


Toilet training concerns are very common. Although parents are often concerned when complete
continence has not been achieved after several months, the child in the vignette is displaying
normal progression towards toilet training.

There is no consensus regarding recommendations for the best method (parent versus child-
oriented) or timing of toilet training, so there is a wide variation in parental expectations and
performance. Many factors influence the optimal time for initiating this process, including the
neurodevelopmental status of the child, parental and cultural beliefs, plus the social and family
environment. Some suggested clues that imply toilet training readiness are as follows: the child
must be able to follow directions and communicate need for assistance, be aware of urges and
have sphincter control, have the desire to remain clean and dry, and have the necessary motor
skills to walk, sit on the toilet, and to dress and undress. Readiness should be based on these
language, social-emotional, and motor milestones rather than on chronologic age.

There is marked concordance in the sequential acquisition of individual toilet training skills
between girls and boys. The average length of time to complete toilet training is 6 to 7 months.
Although an earlier age at initiation of intensive training is correlated with a younger age at
completion, the duration of this training period is longer. In general, girls reach toilet training
milestones a few months earlier than boys. Most children in the United States achieve bowel and
bladder control between 24 and 48 months of age. Occasional daytime accidents will occur,
especially when children are preoccupied, similar to the girl in the vignette. This micturition
deferral is the most common cause of intermittent wetting in preschool children. Frequently,
children will exhibit signs of regression of milestones with family stressors, such as when a new
sibling is born, yet this is not something for which the parents should discipline the child. These
temporary setbacks do not constitute failure. Parents should be reminded that this developmental
stage correlates with the child's increased desire for independence and control. Incentives and
positive rewards should be offered for successes, but punishment is not warranted when the child
has an "accident." Consistency by all caregivers is extremely important.

Primary diurnal (daytime) enuresis should be evaluated if the child is older than 4 years of age.
Nighttime dryness is often achieved months to years later than daytime bladder control. Primary
nocturnal enuresis is present in 20% of 5-year-old children and is more common in those with a
positive family history. Organic causes are found in only 2% to 3% of cases. Secondary enuresis,
as defined by the recurrence of enuresis after a period of 6 months of dryness, should be
evaluated as well. A thorough history should be collected and a physical examination performed,
along with obtaining a family history, social history, and urinalysis.

PREP Pearls
• Toilet training readiness should be based on the attainment of necessary language, social-
emotional, and motor milestones rather than on chronologic age.
• Bowel and bladder control is attained by most children between 24 and 48 months of age.
• The average length of time to complete toilet training is 6 to 7 months, so occasional
accidents should be expected and the child should not be punished.

American Board of Pediatrics Content Specification(s)


American academy of pediatrics 727
American Academy of Pediatrics PREP 2015

• Understand the factors that contribute to readiness for toilet training


• Recognize the normal age ranges in boys and girls during which bowel and bladder
control occurs
• Plan the appropriate management of delayed toilet training

Suggested Reading
• American Academy of Pediatrics. Toilet training guidelines: clinicians—the role of the
clinician in toilet training. Pediatrics. 1999;103(6 Pt 2]:13641366. Accessed January
15,2014.
• Blum NJ, Taubman B, Nemeth N. Relationship between age at initiation of toilet training
and duration of training: a prospective study. Pediatrics. 2003:111(4 Pt 11:810-814.
• Brazelton TB, Christophersen ER, Frauman AC, et al. Instruction, timeliness, and
medical influences affecting toilet training. Pediatrics. 1999;103(6 Pt 21:1353-1358.
• Howell DM, Wysocki K, Steiner MI. Toilet training. Pediatr Rev. 2010;31(0:262-263.
doi:10.1542/pir.31-6-262.
• Michel RS. Toilet training. Pediatr Rev. 1999:20(71:240.245. doi:10.1542/ pir.20-7-240.
• Robson WI.. Diurnal enuresis. Pediatr Rev. 1997;18(121:407-412. doi:10.1542/pir.18-12-
407.
• Schmitt BD. Nocturnal enuresis. Pediatr Rev. 1997;18(61:183-191 doi:10.1542/pir.18-6-
183.
• Schum TR, Kolb TM, McAuliffe TL, Simms MD, Underhill RL, Lewis M. Sequential
acquisition of toilet-training skills: a descriptive study of gender and age differences in
normal children. Pediatrics. 2002;109(3):E48. doi:10.1542/peds,109.3.e48.

American academy of pediatrics 728


American Academy of Pediatrics PREP 2015

Item 237
A child that you follow in your practice has been diagnosed with factor XIII deficiency, a rare
congenital bleeding disorder. The family has consented to participate in a multi institutional
study looking at the efficacy of a new, longer-acting factor XIII concentrate product compared to
the existing product; however, the study may take years to accrue an adequate number of
patients. The medical student working with you asks why this is important.

Of the following, the MOST appropriate response would be that


A. increasing the sample size improves the ability to detect adverse events
B. a larger sample size decreases the power of a study
C. the larger the detectable difference in effect, the larger the sample size required
D. the smaller the sample size, the less likely a type 1 error is made
E. the smaller the sample size, the less likely a type 2 error is made

American academy of pediatrics 729


American Academy of Pediatrics PREP 2015

Item 237 Preferred Response: A


Increasing the sample size improves the ability to detect adverse events. For studies in which the
difference in measured effect is small, a larger sample size is required to detect a statistically
significant difference.

Power is the statistical probability that a study will not make a type 2 error, which is to accept a
null hypothesis and conclude that there was no effect when there actually was one. Having a
larger sample size increases the power of the study. A larger sample size reduces the likelihood
of concluding that there is no significant difference when in fact there was a true difference, also
known as a type 1 error.

PREP Pearls
• Increasing sample size improves the ability to detect adverse events.
• For studies in which the difference in measured effect is small, a larger sample size is
required to detect a statistically significant difference.
• Power is the statistical probability that a study will not mistakenly accept a null
hypothesis and conclude that there was no effect when there actually was one,
• Having a larger sample size increases the power of the study.
• A larger sample size reduces the likelihood of making a type 1 or type 2 error.

American Board of Pediatrics Content Specification(s)


• Understand how sample size affects the power of a study
• Understand how sample size may limit the ability to detect adverse events

Suggested Reading
• Bonis PAL. Glossary of common biostatistical and epidemiological terms. UpToDate.
Available online only for subscription.
• Haas JP. Sample size and power. Am Infect Control. 2012;40(81:766-767.
doi:10.1016/),ajic.2012.05.020.
• Merrifield A, Smith W. Sample size calculations for the design of health studies: a review
of key concepts for non-statisticians. NSW Public Health Bull. 2012; 213(7-81:142-147.
doi:10.1071/NB11017.
• Rind DM. Proof, p-values, and hypothesis testing. UpToLtate. Available online only for
subscription.

American academy of pediatrics 730


American Academy of Pediatrics PREP 2015

Item 238
You have seen a 9-year-old girl in your office for various medical complaints repeatedly over the
past year. Last week, the mother brought her daughter in with a complaint of abdominal pain and
muscle cramps. In previous visits, the girl was reported to have muscle twitching and "staring
spells." Today, she reports that her daughter has been having headaches, dizziness, and
diminished appetite. Over the past year, you have performed radiography to evaluate specific
musculoskeletal complaints and blood tests, including a complete blood cell count, liver function
tests, electrolytes, calcium, and an erythrocyte sedimentation rate, all of which were normal. You
have been unable to come up with clear medical reasons for various complaints that have
occurred; you now believe this could be a case of factitious disorder by proxy.

Of the following, the MOST appropriate next step would be to


A. call child protective services to report the mother for possible medical abuse of her child
B. refer her to a neurologist to be evaluated for dizziness and headaches
C. tell the patient and her mother that stress and worry can cause many types of physical
symptoms
D. tell the patient that you do not think her physical symptoms are real
E. tell the patient's mother that you do not think the physical symptoms are real

American academy of pediatrics 731


American Academy of Pediatrics PREP 2015

Item 238 Preferred Response: C


Factitious disorder by proxy is a psychiatric diagnosis applied to a caregiver who falsifies or
fabricates a child's symptoms to serve an underlying caregiver need. This is quite similar to
Munchausen syndrome by proxy, though this latter diagnosis implies more specifically that
healthcare professionals have an active role in the initiation and perpetuation of the syndrome
through medical tests and procedures. Factitious disorder by proxy more specifically refers to the
adult caregiver's pathology.

When working with patients and families, it is often difficult to know with certainty when
parental anxiety regarding a child's symptoms has moved into the pathological range. Children
may have legitimate medical problems that get exaggerated in severity in parent reports (which is
not by itself pathological), and parental worries about "missing something" may either be
reasonable, or may venture into the unreasonable territory including pushing for inappropriate
invasive studies or procedures. Preventing injury to the child in a scenario of exaggerated
parental anxiety or frank symptom falsification requires provider recognition that a child might
be harmed psychologically or physically by unnecessary medical care. "Proving" the parental
psychological reasons for why this is happening or proving that symptom falsification is
occurring is usually not important. Communication among providers in a care team, and
declining to order unreasonable tests or procedures is what will prevent harm to the child.

Providing a parent with a rational physiological explanation for her child's reported symptoms
(ie, stress will cause changes in autonomic nervous system tone, which in turn, causes stomach
aches, muscle cramps, headaches, diminished appetite, etc) is a way to take parental concerns
seriously, avoid an unnecessary challenge of the treatment alliance, and to advocate for
appropriate care for the child. Once the factitious dis-order is recognized, treatment to address
the family's anxiety/ stress will enable both the child and parent to receive support that does not
create medical risk

Calling child protective services to report medical abuse is necessary in clear and egregious
situations, but when a provider is uncertain and the child appears to be in no imminent danger, a
report to child protective services may do more harm than good. At that point, the parent is
highly likely to break off the treatment alliance, and if there is not enough evidence to prove
medical abuse occurred, then the parent is free to repeat the whole cycle with a new set of care
providers and with a more covert approach. The decision to report must balance the legal and
ethical mandated reporting obligations with the risk to the child and potential disruption of the
therapeutic alliance.

Referring this child to neurology for evaluation of headache/dizziness risks further splitting the
treatment team's message, and risks initiating more unnecessary medical tests/ procedures.
Unless one has clear reasons for why a new referral is appropriate, such as clear migraine
symptoms, avoiding unnecessary referrals is a key part of the treatment plan.
Telling either the patient or the parent that you do not believe the symptoms are "real" is
invalidating, a challenge to the treatment alliance, and sets up adversarial interactions going
forward. Parents or children may increase their efforts to "prove" that symptoms are real or to
find another specialist. Stating that symptoms are not "real" is also inappropriate, because

American academy of pediatrics 732


American Academy of Pediatrics PREP 2015

somatic complaints typically do have physiologic origins (ie, gastrointestinal distress from
increased bowel contractions/motility because of anxiety or stress impact on autonomic nerves).

PREP Pearls
• Stress and anxiety are important contributory factors in the symptoms of a somatic
disorder, or in what can evolve into a factitious disorder by proxy when parental desires
for obtaining medical provider attention takes over.
• Providing a parent a rational biological explanation for how stress/ anxiety can generate
physical symptoms helps move families away from endless requests for testing and
toward a focus on appropriate treatment and rehabilitation.
• Regular health care team communication prevents different providers from initiating
opposing assessment and treatment strategies, and is an essential step in managing a
factitious disorder.

American Board of Pediatrics Content Specification(s)


• Understand the components of a management plan for factitious disorder (Munchausen
syndrome) by proxy

Suggested Reading
• Silber TJ. Somatization disorders: diagnosis, treatment and prognosis. Pediatr Rev.
2011;32(2):56-64. doi:10.1542/pir.32-2-56.
• Squires 1E, Squires RH. A review of Munchausen syndrome by proxy. Pediatr Ann.
2013;42(4):67-71. doi: 10,3928/00904481-20130326-09.

American academy of pediatrics 733


American Academy of Pediatrics PREP 2015

Item 239
A previously healthy 4-year-old child arrives at the emergency department with lethargy, cool
extremities, and decreased level of consciousness after a 1-week viral illness. On physical
examination, the capillary refill time is 4 seconds, heart rate is 180 beats/min, blood pressure is
60/40 mm Hg, respiratory rate is 40 breaths/min, and axillary temperature is 37°C. The bedside
monitor shows sinus tachycardia, and shallow breathing. The child is minimally responsive to
intravenous catheter placement. The peripheral pulses are thready, lungs are difficult to
auscultate, and heart rate is regular. No murmur or rub is heard, but there is a S3 gallop. The liver
is 5 cm below the right costal margin. A chest radiograph shows cardiomegaly and pulmonary
edema.

Of the following, the BEST next step in management is administration of


A. adenosine
B. atropine
C. esmolol
D. immunoglobulin
E. 100% oxygen

American academy of pediatrics 734


American Academy of Pediatrics PREP 2015

Item 239 Preferred Response: E


Cardiogenic shock is uncommon in previously healthy children and diagnosis requires a high
index of suspicion. The child in the vignette has myocarditis. There is a problem with myocardial
function, and he is experiencing acute pump failure, which causes poor perfusion, hepatomegaly,
and a S3 gallop on examination. The most likely cardiac rhythm on presentation will be sinus
tachycardia. Patients in this situation do not tolerate large boluses of intravenous fluid as would
often be done initially for a child in shock. Of the choices listed, oxygen administration is the
best next step in management to provide maximal oxygenation to the tissues. The myocardium,
other organs (especially brain and kidney), and the skeletal muscles are receiving less than
adequate perfusion. As the perfusion worsens, acidosis will occur. Providing supplemental
oxygen is not the only intervention that is needed, but of the choices given, it is the best and the
one that is likely to be helpful.

The other choices would not be useful in the initial care of a patient in cardiogenic shock.
Adenosine is used to block the atrioventricular node in the setting of reentrant supraventricular
tachycardia. If given to a patient who is acidotic, in shock with dilated atria, and sinus
tachycardia, it could cause atrial fibrillation, and prolonged atrioventricular block. Heart block
may already occur spontaneously with progressive, fulminant myocarditis. Atropine is used to
treat bradycardia as a vagolytic agent. Esmolol is an intravenous (β-blocker used to treat
tachyarrhythmias, it would be dangerous for the child in the vignette because he is dependent on
his sinus tachycardia for cardiac output. Slowing the heart rate suddenly and lowering the blood
pressure would make the peripheral perfusion much worse. The patient will likely require
inotropic support instead. After the acute resuscitation and treatment phase, patients with
cardiomyopathy may be given β-blockers as long-term therapy, but not on first presentation
when unstable. The same is true for intravenous immunoglobulin, which may be used if the
patient is suspected to have a viral myocarditis or Kawasaki disease with shock. Intravenous
immunoglobulin would be too great a fluid challenge if given when the patient first presents.

PREP Pearls
• The diagnosis of cardiogenic shock is suspected when the patient has a gallop and
enlarged liver. Sinus tachycardia is the most common rhythm. Large fluid boluses may
cause decompensation.
• Oxygen and inotropic support are indicated in the treatment of cardiogenic shock.

American Board of Pediatrics Content Specification(s)


• Plan appropriate management of cardiogenic shock in children of various ages

Suggested Reading
• Towbin J. Myocarditis. In: Allen HD, Driscoll DJ, Shaddy RE, Feltes eds. Moss and
Adams Heart Disease in Infants, Children and Adolescents. 6th ed. Philadelphia, PA:
Lippincott Williams & Wilkins; 2001:1197-1215.
• Turner D, Cheifetz 1. Shock. In Kliegman RM, Stanton SF, St Geme JW III, Schor NF,
eds. Nelson Textbook of Pediatrics. 19th ed. Philadelphia, PA: Saunders Elsevier;
2011:305-314.

American academy of pediatrics 735


American Academy of Pediatrics PREP 2015

Item 240
The mother of an 8-year-old patient with type 1 diabetes mellitus calls your office requesting that
you measure a homocysteine level on her daughter. She presents you with an internet article
showing results of a study of 2,000 adults diagnosed with type 1 diabetes mellitus in childhood.
Eighteen percent of those patients with a mean homocysteine level of greater than 2.03 ± 1 mg/L
(15 ± 1 umol/L) develop coronary artery disease by the time they are 35 years of age, compared
with 16% of those whose levels were less than 1.76 ± 1 mg/L (13 ± 1 umol/L). The odds ratio is
1.3 (1.01-6) and P value is .05. Normal homocysteine levels are 0.7 to 2.0 mg/ dL (5-15 umol/L).

Of the following, the BEST recommendation you would make to the mother is
A. do not measure homocysteine levels because of the odds ratio confidence interval range
B. do not measure homocysteine levels because the results are not clinically relevant
C. do not measure homocysteine levels because there was no control group without diabetes
mellitus
D. measure homocysteine levels because of the clinical importance of coronary artery
disease
E. measure homocysteine levels because the results of the study are significant

American academy of pediatrics 736


American Academy of Pediatrics PREP 2015

Item 240 Preferred Response: B


Results are considered statistically significant when the P value is less than .05 or the 95%
confidence interval does not include zero. A statistically significant result indicates that the
results are highly unlikely to be the result of chance. A statistically significant result does not,
however, signify causality or clinical importance, as is the case in the vignette. Clinical
significance refers to an effect significant enough to alter practice or decision making. This can
involve prognosis, effectiveness, cost, risk, benefit, or other factors. A result that is not
statistically significant may still have clinical significance, but the studies may not have been
large enough to show a statistically significant difference. Practitioners should keep this in mind
when there are few or only low power studies regarding a clinical question.

The fact that a research paper shows a statistically significant association between a risk factor
and a disease does not in itself allow one to conclude that the risk factor caused the disease.
Causal relationships between risk factors and disease must also be assessed to rule out
confounding factors that cause the appearance of a significant association that does not truly
exist.

To interpret the results of an association study, clinicians should consider whether the results are
generalizable and have been observed in other studies. True causal associations will be seen in
multiple studies.

For the clinician caring for the child in the vignette, the best choice would be not to measure
homocysteine levels because the results are not clinically relevant. The study was highly
powered with 2,000 individuals, and it was statistically significant with a P value of .05.
However, the difference between the 2 groups was less than 0.3 mg/L and both results are within
the accepted normal range for the homocysteine levels. This is also a single study in an adult
population. It is not known whether the results of this study will be reproducible or are
generalizable to other populations, including the pediatric population.

In the vignette, 18% of patients with increased homocysteine levels developed coronary artery
disease versus 16% in the group with lower homocysteine levels. The result presented is a small
difference identified in a single study. The levels of homocysteine in both groups are within
normal limits, so the interpretation of any homocysteine results would be difficult for the
clinician. Although the participants in the study were all diagnosed with diabetes in childhood,
these participants are now adults and homocysteine levels obtained in the pediatric population
may not be comparable.

Not measuring homocysteine levels because of the odds ratio confidence interval range would
not be appropriate, because the confidence interval does not contain 1. Therefore the results
demonstrate a statistically significant association between homocysteine levels and coronary
artery disease.

Not measuring homocysteine levels because there was no control group without diabetes mellitus
would not be appropriate, because this was an observational study with a target population of
patients with diabetes mellitus.

American academy of pediatrics 737


American Academy of Pediatrics PREP 2015

To measure homocysteine levels because of the clinical importance of coronary artery disease
would be incorrect, because there is no clear evidence that homocysteine levels predict coronary
artery disease in children.

To measure homocysteine levels because the results of the study are significant would also be
incorrect. Although the results are statistically significant, the clinical significance of
homocysteine levels in children is unknown, and both sets of homocysteine levels in the study
are within normal range and would therefore be difficult to interpret.

PREP Pearls
• Statistically significant study results are unlikely to be the result of chance.
• Statistical significance does not imply clinical significance.
• Clinically significant results should be generalizable and reproducible.
• Clinically significant study results should have an effect that suggests an alteration in
practice or decision making.

American Board of Pediatrics Content Specification(s)


• Distinguish statistical significance from clinical importance

Suggested Reading
• Gharzarian SR. Research and statistics: distinguishing statistical significance from
clinical importance: the value of the P value. Pediatr Rev. 2011;32(2):73-74.
doi:10.1542/p1r.32-2-73.
• Rosenberg El, Bass PF 3rd, Davidson RA. Arriving at correct conclusions: the
importance of association, causality, and clinical significance. South Med J.
2012;105(3):161-166. doi:10.1097/SMJ.0b013e31824b9a19.

American academy of pediatrics 738


American Academy of Pediatrics PREP 2015

Item 241
An 11-year-old boy goes to your office for follow-up 1 week after a fall onto his right shoulder.
He arrived at the emergency department of a local hospital with pain over the midshaft of the
right clavicle. Radiography showed no evidence of clavicle fracture, but an abnormal finding
involving the proximal humerus was incidentally noted. The emergency department physician
obtained dedicated radiographs of the humerus that the family brought for your review (Item
Q241).

The boy has no history of fractures. On physical examination of the upper extremity, the boy has
no tenderness to palpation around the clavicle, shoulder, or proximal humerus, and has full range
of motion of the right shoulder.

Of the following, the MOST appropriate initial management is


A. biopsy of the lesion under fluoroscopic guidance
B. magnetic resonance imaging of the proximal humerus
C. referral to orthopedic surgery for incision and drainage
D. referral to orthopedic surgery for resection
E. repeat radiography if child develops pain at the site of the lesion

American academy of pediatrics 739


American Academy of Pediatrics PREP 2015

Item 241 Preferred Response: E


The radiograph for the child in the vignette is diagnostic for a unicameral bone cyst (UBC).
Treatment is not indicated because he does not have symptoms attributable to the cyst and
because the cyst is not in a location where pathologic fracture could lead to serious
consequences.

Unicameral bone cysts, also known as simple cysts, typically occur during the first 2 decades of
life. These benign lesions are more common in boys and are typically located in the proximal
humerus or proximal femur. Most UBCs are discovered when affected individuals present with
pain following pathologic fractures; however, they may be also be found incidentally, as with
this patient. On plain radiography, UBCs appear as radiolucent lesions with a rim of sclerosis,
adjacent to the epiphysis. The "fallen leaf sign:' the presence of a bone fragment that has fallen to
the bottom of the cyst, is a pathognomonic finding. Unicameral bone cysts begin at the epiphysis
and move towards the shaft of the bone as a child grows. Eventually, the cysts resolve without
treatment.

An aneurysmal bone cyst (ABC) can have a similar radiographic appearance to a UBC, but
ABCs tend to occur in the metaphysis or diaphysis. ABCs are blood-filled cysts and patients
generally present with pain. Patients with ABCs should be referred to orthopedic surgery for
management.

Fibrous cortical defects (also called nonossifying fibro-mas) are very common lesions, occurring
in up to 40% of children; the femur is the most common location. Fibrous cortical defects rarely
cause pathologic fracture and are usually discovered incidentally. On radiographs, fibrous
cortical defects appear as well-delineated lucent lesions surrounded by sclerosis. Fibrous cortical
defects only require treatment in the rare case that the lesion occupies at least 50% of the bone
diameter.

Fibrous dysplasia refers to an area of bone that has been replaced by fibrous tissue. These
radiolucent lesions are classically described as having a "ground glass appearance." Single
lesions are typically found incidentally, but may be associated with pain or pathologic fracture.
No treatment is needed in asymptomatic patients, unless the lesions occur in a location where
pathologic fracture could have serious consequences (eg, the proximal femur).
Malignant tumors, such as Ewing sarcoma and osteosarcoma, destroy adjacent bone and lead to
irregularity of the cortex and periosteal reaction, whereas benign lesions tend to be well
circumscribed.

Biopsy is not needed for the boy in this vignette because his radiograph demonstrates the classic
findings of a UBC. Incision and drainage may be indicated for osteomyelitis, but not for this
noninfectious entity. Steroid injections are the first-line treatment for UBCs that are especially
large, in high-risk locations of the bone, or for a child experiencing repeated pathologic fractures.
Operative treatment is indicated for UBCs that do not resolve with corticosteroid injection.
Magnetic resonance imaging is not indicated because this lesion is well characterized on plain
radiography.

American academy of pediatrics 740


American Academy of Pediatrics PREP 2015

PREP Pearls
• Unicameral bone cysts typically arise from the epiphysis and are most common in the
proximal humerus and proximal femur.
• Aneurysmal bone cysts usually occur in the diaphysis or metaphysis.
• Malignant tumors, such as Ewing sarcoma and osteosarcoma, destroy adjacent bone and
lead to irregularity of the cortex and periosteal reaction.

American Board of Pediatrics Content Specification(s)


• Formulate a differential diagnosis of a bone cyst

Suggested Reading
• Gereige R, Kumar M. Bone lesions: benign and malignant. Pediatr Rev. 2010;31(9):355-
362. doi:10.1542/pir.31-9-355
• Sarwark JF, LaBella C. Pediatric Orthopaedics and Sports Injuries: A Quick Reference
Guide. Elk Grove Village, IL: American Academy of Pediatrics; 2010.

American academy of pediatrics 741


American Academy of Pediatrics PREP 2015

Item 242
A neonate born at term is noted to have a faint petechial rash and marked hepatosplenomegaly.
Growth parameters are at the fifth percentile for gestational age. No other significant findings are
noted on physical examination. The maternal history is unremarkable. Prenatal laboratory studies
for hepatitis B virus, syphilis, and human immunodeficiency virus are negative, and the mother
is rubella immune. A computed tomography brain scan of the neonate is performed (Item Q242).

Of the following, the MOST likely etiology of this infant's illness is


A. cytomegalovirus
B. herpes simplex virus
C. rubella virus
D. Toxoplasma gondii
E. Treponema pallidum

American academy of pediatrics 742


American Academy of Pediatrics PREP 2015

Item 242 Preferred Response: D


The newborn infant described in the vignette has a petechial rash, hepatosplenomegaly, and a
computed tomography of the head showing diffuse intracranial calcifications, suggestive of
congenital infection caused by Toxoplasma gondii. Only 10% to 30% of infants with congenital
toxoplasmosis are symptomatic at birth. However, a large number of infants that appear
asymptomatic will later be found to have visual, hearing, or learning impairments. The classic
triad of chorioretinitis, hydrocephalus, and intracranial calcifications is rare. Symptomatic
newborns with toxoplasmosis can present with various clinical manifestations, including
temperature instability, rash (maculopapular or petechial), hepatosplenomegaly, jaundice,
generalized lymphadenopathy, pneumonitis, microcephaly, meningoencephalitis, seizures, and
chorioretinitis. Untreated congenital toxoplasmosis leads to chorioretinitis in up to 85% of cases
by young adulthood and can result in vision loss. Thrombocytopenia is a common laboratory
finding. Cerebrospinal fluid abnormalities such as pleocytosis, hypoglycorrhachia, and elevated
protein can be present.

Toxoplasmosis acquired after birth is often asymptomatic. However, immunocompromised


individuals frequently develop severe disease. Clinical manifestations can range from a flulike or
mononucleosis like illness to severe complications such as myocarditis, pericarditis, pneumonia,
and hepatitis. Lymphadenopathy is common. Central nervous system infection (eg, encephalitis,
brain abscesses) is rare, typically occurring in immunocompromised patients. Isolated ocular
disease can occur with both congenital and acquired infection.

Approximately 22% of the US population 12 years of age and older has been infected with T
gondii, a protozoan parasite found throughout the world, most commonly in hot and humid
climates at low altitudes. The tissue form of the parasite can be transmitted to humans by eating
undercooked meat (pork, venison, lamb) or contact with contaminated items used in the
preparation of these foods. Animal to human transmission can occur by accidental ingestion of
oocysts excreted in cat feces. Cats acquire infection by eating infected birds and rodents. Human-
to-human transmission of toxoplasmosis occurs in congenital infection or rarely through
transplantation of infected organs or receipt of infected blood.

In immunocompetent hosts without severe or persistent infection, antimicrobial therapy is


unnecessary, because the disease is self-limited. For children and adults in whom treatment is
required, pyrimethamine and sulfadiazine in combination with folinic acid (reduces
pyrimethamine hematologic toxicity) is recommended. If sulfonamides are not tolerated,
pyrimethamine can be used in combination with clindamycin, azithromycin, or atovaquone. For
the treatment of isolated retinitis, trimethoprim-sulfamethoxazole monotherapy has been shown
to be as effective as pyrimethamine and sulfadiazine. Systemic corticosteroids may be used in
patients with ocular or central nervous system disease.

Congenital infection caused by cytomegalovirus (CMV) is symptomatic in 10% of newborns and


can present with a petechial rash, hepatosplenomegaly, and central nervous system
manifestations, as described for the patient in the vignette. However, the intracranial
calcifications of CMV are characteristically periventricular rather than diffusely distributed.
Other manifestations of symptomatic congenital CMV infection can include microcephaly,
chorioretinitis, intrauterine growth restriction, jaundice, and hearing loss.
American academy of pediatrics 743
American Academy of Pediatrics PREP 2015

Congenital herpes simplex virus (HSV) infection is uncommon, occurring in approximately 1 of


100,000 deliveries. Congenital HSV can present with cutaneous and central nervous system
manifestations, in addition to hepatomegaly; however, skin abnormalities typically include
vesiculobullous lesions, ulcerations, or eschars, and not petechiae, as described for the patient in
the vignette. Neither congenital rubella nor syphilis is a likely cause of illness for the newborn
described in the vignette because maternal laboratory studies were negative for these pathogens.
Manifestations of congenital rubella infection typically include ophthalmo-logic (cataracts,
microphthalmos, retinopathy, glaucoma), neurologic (meningoencephalitis, microcephaly), and
cardiac (patent ductus arteriosus, peripheral pulmonary artery stenosis) abnormalities.

Hepatosplenomegaly, pneumonitis, bone disease, rash, and thrombocytopenia also can occur.
Congenital syphilis can present with edema, snuffles, hepatosplenomegaly, rash, osteochondritis,
lymphadenopathy, pneumonia, anemia, and thrombocytopenia. Central nervous system infection
can be present at birth in congenital syphilis, but intracranial calcifications are not characteristic.

PREP Pearls
• Most newborns with congenital toxoplasmosis are asymptomatic at birth, Symptoms
presenting in newborns with toxoplasmosis can include temperature instability, rash
(maculopapular or petechial), hepatosplenomegaly, jaundice, generalized
lymphadenopathy, pneumonitis, microcephaly, meningoencephalitis, seizures, and
chorioretinitis.
• The classic triad of chorioretinitis, hydrocephalus, and diffuse intracranial calcifications
is rarely seen in congenital toxoplasmosis.
• Pyrimethamine and sulfadiazine in combination with folinic acid is recommended as
first-line therapy for toxoplasmosis.

American Board of Pediatrics Content Specification(s)


• Understand the epidemiology of Toxoplasma gondii
• Identify the clinical features associated with congenital and acquired Toxoplasma gondii
infestation, and manage appropriately

Suggested Reading
• American Academy of Pediatrics. Cytomegalovirus infection. In: Pickering LK, Baker
Cl, Kimberlin DW, Long 55, eds. Red Book: 2012 Report of the Committee on Infectious
Diseases. 29th ed. EIk Grove Village, IL: American Academy of Pediatrics; 2012:300-
309.
• American Academy of Pediatrics. Herpes simplex. In: Pickering LK, Baker Kimberlin
DW, Long 55, eds. Red Book: 2012 Report of the Committee on Infectious Diseases,
29th ed. Elk Grove Village, IL: American Academy of Pediatrics; 2012:398-408.
• American Academy of Pediatrics. Rubella. In: Pickering LK, Baker CI, Kimberlin DW,
Long 55, eds. Red Book: 2012 Report of the Committee on Infectious Diseases. 29th ed.
EIk Grove Village, IL: American Academy of Pediatrics; 2012:629-634.
• American Academy of Pediatrics. Syphilis. In: Pickering LK, Baker a Kimberlin DW,
Long 55, eds. Red Book: 2012 Report of the Committee on Infectious Diseases. 29th ed.
Elk Grove Village, IL: American Academy of Pediatrics; 2012:690-703.
American academy of pediatrics 744
American Academy of Pediatrics PREP 2015

• American Academy of Pediatrics. Toxoplasma gondii infections. In: Pickering LK, Baker
CI, Kimberlin DW, Long SS, eds. Red Book: 2012 Report of the Committee on
Infectious Diseases. 29th ed. Elk Grove Village, IL; American Academy of Pediatrics;
2012;720-728.
• Marquez L. Levy ML, Munoz FM, Palazzi DL. A report of three cases and review of
intrauterine herpes simplex virus infection. Pediatr Infect Ott 1. 2011;30(4:153-157.
doi:10.1097/INF.0b013e3181f55a5c.
• US Centers for Disease Control and Prevention. Parasites: Toxoplasmosis floxoplasma
infection). US Centers for Disease Control and Prevention website.

American academy of pediatrics 745


American Academy of Pediatrics PREP 2015

Item 243
A healthy 7-week-old, full-term infant is brought to the emergency department by his mother
because of swelling over the right side of his scalp. The mother tells you that yesterday evening
the baby rolled off of a sofa while his aunt, who was babysitting, was changing his diaper. The
infant has been active and feeding well, but this morning, the mother noticed an area of swelling
over the right side of his head that seemed tender to touch.
A complete physical examination reveals a well-appearing, interactive baby with a 3 x 3 cm
hematoma over the right parietal region of his head. There are no other remarkable findings on
physical examination.
Computed tomography of the head reveals a small, nondepressed linear fracture of the right
parietal skull with overlying soft tissue swelling. No associated intracranial injury is noted.

Of the following, the BEST next step in management is


A. admission to the pediatric inpatient unit for a period of observation
B. discharge to home after providing appropriate anticipatory guidance
C. emergent neurosurgical consultation
D. initiation of prophylactic anticonvulsant therapy
E. perform a skeletal survey

American academy of pediatrics 746


American Academy of Pediatrics PREP 2015

Item 243 Preferred Response: E


The young infant in the vignette is well appearing, but is found to have a nondepressed linear
fracture of his right parietal skull. Although linear skull fractures may certainly arise from
unintentional trauma in infants and children, the reported history of injury in this case is
questionable because a 7-week-old infant would not typically be able to roll over independently.
Abusive head trauma must be included in the differential diagnosis, and a skeletal survey should
be obtained to investigate for other musculoskeletal injuries.

Skull fractures generally arise from direct impact to the calvarium. Linear fractures are the most
commonly occur-ring type of skull fracture in children, accounting for about 75% of cases;
depressed and basilar skull fractures occur less commonly. The parietal region of the skull is
involved most frequently in children with linear skull fractures, and 15% to 30% of these
children have an associated intracranial injury. In general, infants are at higher risk for skull
fracture than older children, probably because their skulls are thinner. Skull fractures in infants
may result from short distance falls; typically, about half of infants with skull fracture have fallen
from less than 4 to 5 ft.

Linear, parietal skull fractures, like those seen in the infant in the vignette, are the most common
type of skull injury seen after head trauma from both unintentional and inflicted mechanisms.
Most linear skull fractures are associated with an overlying scalp hematoma or soft tissue
swelling that is apparent on physical examination. Associated soft tissue swelling may not be
seen immediately after the injury. The clinical findings associated with accidental and inflicted
linear skull fractures can be essentially identical, therefore children with these injuries should be
evaluated with careful consideration of the history provided by the child's care-givers. Linear
skull fractures, although not specific for child abuse, are seen relatively frequently in children
who are victims of child abuse. Inflicted injury should be suspected if the reported history is not
consistent with clinical findings, if the mechanism reported is not consistent with the child's
developmental abilities, if no history of trauma is given, if there has been a delay in seeking
treatment for the injury, or if there are other inconsistencies or changes in the history relayed by
the caregivers.

Most isolated linear skull fractures heal without complication. No specific therapy is needed for
these types of fractures, other than appropriate pain management.

Hospital admission for observation would certainly be appropriate for the infant in the vignette,
especially given his very young age. However, obtaining a skeletal survey to evaluate for other
potential inflicted musculoskeletal injuries would be the best next step in the management of his
condition because of the implausibility of the reported injury mechanism.

Discharging this infant from the hospital before a full evaluation for inflicted injury would not be
appropriate. Well-appearing children with linear skull fractures and no associated intracranial
injury may be considered for dis-charge after a period of observation in the emergency
department if child abuse is not suspected.

American academy of pediatrics 747


American Academy of Pediatrics PREP 2015

For isolated linear skull fractures without associated intracranial injuries, emergent neurosurgical
consultation is not required. Neurosurgical consultation is necessary for all depressed skull
fractures, basilar skull fractures, and skull fractures with associated intracranial injury.
There is no role for prophylactic anticonvulsant therapy for well-appearing infants and children
with isolated linear skull fractures.

PREP Pearls
• Linear, parietal skull fractures are the most common type of skull injury after head
trauma from both unintentional and inflicted mechanisms.
• Skull fractures in children younger than 2 years of age without a plausible history of
injury should raise the clinician's concern and prompt a more thorough evaluation for
inflicted trauma.
• Well-appearing children with linear skull fractures and no associated intracranial injury
may be considered for discharge after a period of observation in the emergency
department if child abuse is not suspected.

American Board of Pediatrics Content Specification(s)


• Recognize the significance of a linear skull fracture in infants

Suggested Reading
• Greenes DS. Neurotrauma. In: Fleisher GR, Ludwig 5, eds. Textbook of Pediatric
Emergency Medicine. 6th ed. Philadelphia, PA: Lippincott Williams & Wilkins;
2010;1422-1447.
• Schunk 1E, Schutzman SA. Pediatric head injury. Pediatr Rev. 2012;33(9):398-410.
doi:10.1542/pi r. 33 -9-398.

American academy of pediatrics 748


American Academy of Pediatrics PREP 2015

Item 244
Your clinic has just purchased a fingerstick hemoglobin Alc machine. The machine does not
require venipuncture and it gives results in 6 minutes, allowing you to obtain and discuss the
results immediately with the patient. Your hospital laboratory director informs you that the
standard deviation of this fingerstick hemoglobin Alc test is 0.3.
You are concerned about the accuracy of the test. To test the machine's accuracy, you decide to
send a patient to the main laboratory, where his hemoglobin A1 c is reported at 7.5 by a highly
accurate method. You then perform a fingerstick hemoglobin Alc in your office.

Of the following, based on the standard deviation of the test, you expect that 95% of the time
your machine will report a result for this patient MOST likely between
A. 5.3 and 9.3
B. 6.5 and 8.5
C. 6.6 and 8.4
D. 6.9 and 8.1
E. 7.2 and 7.8

American academy of pediatrics 749


American Academy of Pediatrics PREP 2015

Item 244 PBLI Preferred Response: D


Standard deviation (SD) shows how much variation or dispersion exists from the average (mean)
or expected value. A low SD indicates that the data points tend to be very close to the mean; high
SD indicates that the data points are spread out over a large range of values.
A normal distribution has most of its values near the mean, with progressively fewer values
found farther from the mean. One SD above and below the mean encompasses 68.2% of the
values. Two or 3 SDs represent 95.4% or 99.7% of data values surrounding the mean,
respectively.

For the scenario described in the vignette, the expected hemoglobin Alc value is 7.5 with an SD
of 0.3. Therefore 2 SDs, encompassing 95% of expected values, would include the mean ± 0.6,
or 6.9 to 8.1. In this case, it would be very unlikely for the machine to report a fingerstick
hemoglobin Al c level of 6.6 (-3 SD) or 8.4 (+3 SD), because this would occur in only 0.3% of
the time. Levels lower than 6.6 or higher than 8.4 would be reported even less frequently in this
scenario.

PREP Pearls
 Standard deviation (SD) is a measure of the variability of individual values around the
mean or average value.
 When data are grouped closely together, the SD is small. When data are highly variable,
the SD is large.

American Board of Pediatrics Content Specification(s)


• Understand standard deviation in the interpretation of results

Suggested Reading
 Smith TIC, Johnson SB. Research and statistics: distribution, variability, and statistical
significance. Pediatr Rev. 2010;31(10):431-432, doi:10.1542/ pir.31-10-431.
 Soyemi K. Choosing the right statistical test. Pediatr Rev. 2012;33(5):e38-e44.
doi:10.1542/pir.33-5-e38.

American academy of pediatrics 750


American Academy of Pediatrics PREP 2015

Item 245
You are asked to attend the cesarean delivery of a term newborn due to breech presentation. The
prenatal history is unremarkable, except for a remote family history of neurofibromatosis. The
delivery is complicated by a difficult breech extraction requiring extensive manipulation of the
newborn. She emerges limp with a weak cry, responding well to vigorous stimulation that
consists of rubbing of the back and flicking of the feet. Examination in the delivery room is
notable for an extensive area of hyperpigmentation of the posterior trunk (Item Q245).

Of the following, the most likely diagnosis is


A. bruising
B. dermal melanosis
C. melanocytic nevi
D. nevus of Ito
E. nevus of Ota

American academy of pediatrics 751


American Academy of Pediatrics PREP 2015

Item 245 Preferred Response: B


The extensive area of hyperpigmentation of the posterior trunk seen on the newborn in this
vignette is dermal melanosis. Dermal melanosis, or Mongolian spots, is the most common form
of hyperpigmentation of the skin seen in the neonatal period, occurring in 96% of black, 46% of
Hispanic, and 10% of white newborns. Melanocytes trapped in the dermal layer of the skin
present as macular slate gray pigmentation. Common locations are the buttocks and sacral area,
although the back, shoulders, and extremities may be involved. Dermal melanosis tends to fade
over time, but lesions distal from the lumbosacral area may persist. No treatment is needed,
although inborn errors of metabolism such as GM1 gangliosidosis and Hurler syndrome should
be considered when extensive dermal melanosis is seen.

The presence of dermal melanosis should be well documented in the newborn examination, as it
may later be confused with bruising. Without adequate documentation, subsequent examiners
may raise the suspicion of child abuse. Bruising of the back and soles of the feet may be seen
after vigorous stimulation of a newborn in the delivery room, especially if a lower platelet count
is present. The distribution and outline of the hyperpigmentation of the newborn in this vignette
is not consistent with bruising.

The nevus of Ota is a congenital melanocytic lesion that is located on the face in the distribution
of the first or second divisions of the trigeminal nerve on the periorbital region, temple, forehead,
and nose (Item C245A). Affected patients are at risk of melanoma and glaucoma. The nevus of
Ito is a similar lesion that appears on the shoulder, neck, and upper arm. Congenital melanocytic
nevi (CMN) may present at birth and are due to disrupted migration of melanocytes (Item
C245B). They are generally brown to black in color and less than 1.5 cm in diameter, although
giant CMN may be 20 cm or more in diameter and contain significant hair on the surface.
Patients with CMN are also at risk for melanoma. Consultation with a pediatric dermatologist
should be considered with these lesions.

American academy of pediatrics 752


American Academy of Pediatrics PREP 2015

PREP Pearls
• Dermal melanosis, or Mongolian spots, is the most common form of hyperpigmentation
of the skin seen in the neonatal period, occurring in 96% of black, 46% of Hispanic, and
10% of white newborns.
• The presence of dermal melanosis should be well documented in the newborn
examination, as it may later be confused with bruising. Without adequate documentation,
subsequent examiners may raise the suspicion of child abuse.

American Board of Pediatrics Content Spec ification(s)


• Recognize the clinical characteristics of dermal melanosis (Mongolian spots)

Suggested Reading
• Krowchuk DP, Mancini AJ, eds. Mongolian Spots. Pediatric Dermatology: A Quick
Reference Guide. 2nd ed. Elk Grove Village, IL: American Academy of Pediatrics; 2011:
647 pp.
• McLaughlin MR, O'Connor NR, Ham P. Newborn skin: part II: birthmarks. Am Fam
Physician. 2008;77(1):56-60. Accessed March 17,2014.
• Swetman GL, Lee DH, Benjamin LT. Nevi in the newborn. NeoReviews.
2011;12(4):e207-e215. doi:10.1542/neo.12-4-e207.

American academy of pediatrics 753


American Academy of Pediatrics PREP 2015

Item 246
A 15-year-old girl sustains a severe traumatic brain injury in a snowboarding accident. She is
transported to a pediatric intensive care unit. After 3 days, her parents are told she has severe
brain injury and is unlikely to recover meaningful neurologic function. Her neurologic
examination shows sluggishly reactive pupils, absent corneal reflexes, and absent gag reflex. She
takes intermittent spontaneous breaths but depends on the ventilator to maintain respiratory
function. Her parents are considering discontinuing mechanical ventilation because of the
severity of her brain injury and poor prognosis. They ask if it is possible to donate her organs in
this situation.

Of the following, the BEST response to the parents' question is that


A. brain death criteria are met and so she could be an organ donor
B. brain death criteria are not met and so she cannot be an organ donor
C. if cardiac death does not occur, she could be an organ donor
D. if cardiac death occurs, she could be an organ donor
E. organ donation requires both cardiac and brain death criteria be met

American academy of pediatrics 754


American Academy of Pediatrics PREP 2015

Item 246 Preferred Response: D


While the girl in the vignette is in a coma, her physical examination shows the presence of some
brainstem reflexes, so she does not meet criteria for brain death. If ventilator support were
discontinued and she was unable to continue effective ventilation independently, and she had a
cardiac arrest and died, her organs could be donated. This is referred to as donation after cardiac
death.

Organ donation after cardiac death occurs in a hospital setting, where organs can be surgically
recovered as soon as possible after cardiac death and cessation of organ perfusion. It is inevitable
that brain death criteria will be met after cardiac death occurs, therefore it is not necessary to
perform a brain death examination in that situation.

Pediatric brain death criteria have been established and updated as recently as 2012. There has to
be a known and irreversible cause of coma, absence of hypotension, hypothermia, metabolic
disturbances, and sedating effects of medication. Two brain death examinations by different
examiners, done 12 to 24 hours apart, must confirm the absence of brainstem reflexes including
the reflex to breathe. Ancillary studies, such as cerebral blood flow studies or
electroencephalography, are not necessary but can be used, especially in circumstances where the
examination cannot be performed safely.

If neither brain death nor cardiac death has occurred, then the person cannot be an organ donor.
This can have severe emotional consequences for families already devastated by the death of
their child. When severe brain injury has occurred, but brain death has not, and the person does
not recover significantly in 4 weeks, the patient can be described as in a persistent vegetative
state. In this state, often wake and sleep cycles occur, with spontaneous eye opening and closure
and even spontaneous swallowing. The person may be able to visually track, cry, or vocalize.
However, they do not have purposeful interactions or meaningful communication. Case reports
have described brief episodes of recovery from persistent vegetative state, but in the vast
majority of cases, significant neurologic function does not return. Family members sometimes
differ in their wishes for their child who is in a persistent vegetative state. Adolescents may have
voiced their own desires before their injury and these should be taken into account in decision
making. Consultation with an ethics committee is often very helpful.

PREP Pearls
• Organ donation can occur after either brain death or cardiac death.
• Two brain death examinations by different examiners, performed 12 to 24 hours apart,
are necessary to confirm brain death.

American Board of Pediatrics Content Specification(s)


• Recognize and apply ethical principles surrounding the issue of brain death

Suggested Reading
Antommaria, AHM. Ethics for the pediatrician: conceptual and ethical issues in the declaration
of death. Pediatr Rev. 2010;31(10);427-430. doi:10.1542/pir.31-10-427.

American academy of pediatrics 755


American Academy of Pediatrics PREP 2015

Nakagawa TA, Ashwal S, Mathur M, Mysore M. Guidelines for the determination of brain death
in infants and children: an update of the 1987 task force recommendations—executive summary.
Ann Neural. 2012;71(4):573-585. doi:10.1002/ana.23552.

American academy of pediatrics 756


American Academy of Pediatrics PREP 2015

Item 247
While examining a full-term newborn, you note an oval, smooth, yellow, raised lesion on the
scalp and face (Item Q247). There is no hair over the scalp portion of the lesion. The remainder
of the physical examination is unremarkable.

Of the following, the MOST accurate statement is


A. close developmental monitoring is indicated for children with this lesion
B. electrocautery of the lesion will prevent long-term consequences
C. excision of the lesion is necessary because of a high risk of malignant transformation
D. gastrointestinal abnormalities are commonly associated with this lesion
E. the lesion will regress spontaneously by 8 years of age

American academy of pediatrics 757


American Academy of Pediatrics PREP 2015

Item 247 Preferred Response: A


The newborn in the vignette has a nevus sebaceous, also called nevus sebaceous of Jadassohn, a
common hamartoma involving the epidermis, hair follicles, and sebaceous and apocrine glands.
A nevus sebaceous usually appears at birth, or shortly thereafter, as a yellow, alopecic, raised
oval lesion with fine papules on the surface. It most commonly is located on the scalp, but can
extend to the face or, less commonly, the neck or trunk. It tends to remain fairly constant
throughout childhood, but androgen stimulation during puberty can cause the lesion to become
larger, more raised, and verrucous in appearance. The lesions do not typically regress with age.
Nevus sebaceous may be an isolated finding, or it may occur as part of a constellation of
findings, such as the neurocutaneous disorder linear nevus sebaceous syndrome (LNSS)
(Schimmelpenning-Feuerstein-Mims syndrome, Solomon syndrome). The characteristic triad of
findings for LNSS is a midline nevus sebaceous, seizures, and intellectual disability. Although
this child does not have a midline lesion, monitoring for developmental and neurologic
symptoms would be prudent. Other organ systems may be involved in LNSS including the
ophthalmologic, cardiovascular, skeletal, and urogenital systems. Gastrointestinal lesions are
infrequently noted.

For years, standard teaching was that sebaceous nevi have a high rate of malignant
transformation and therefore should be excised prophylactically during adolescence. However,
recent studies indicate that only a small percentage of lesions develop tumors, and the majority
of these tumors are benign. Basal cell cancer is the most common malignancy associated with
nevus sebaceous, occurring in fewer than 2% of lesions. Therefore, experts have now called into
question the need to excise these lesions. If treatment is desired, because of aesthetic or
malignancy concerns, surgical excision is the appropriate approach. Electrocautery does not
effectively treat this lesion.

PREP Pearls
• Nevus sebaceous presents at birth as an oval, yellow-orange, slightly raised, and well-
circumscribed area of alopecia typically occurring on the scalp. and less commonly on
the face, neck, or trunk.
• Nevus sebaceous may occur as part of the linear sebaceous nevus syndrome, a triad of
nevus sebaceous, seizures, and intellectual disability.
• Because malignant transformation occurs in less than 2% of nevus sebaceous lesions,
prophylactic excision is not universally required.

American Board of Pediatrics Content Specification(s)


• Recognize the clinical findings associated with sebaceous nevus

Suggested Reading
 Aguayo R, Pallares J, Casanova JM, et al. Squamous cell carcinoma developing in
Jadassohn's sebaceous nevus: case report and review of the literature. Dermatol Surg.
2010;36(10:1763-1768. doi:10.1111/j.152.4- 4725.2010.01746.x.
 Brandling-Bennett HA, Morel KD. Epidermal nevi. Pediatr Clin N Am. 2010;57(5)1177-
1198. doi:10.1016/j.pd.2010.07.004.

American academy of pediatrics 758


American Academy of Pediatrics PREP 2015

 Menascu S, Donner EJ. Linear nevus sebaceous syndrome: case reports and review of the
literature. Pediatr Neural. 2008;38(3):207-210. doi:10.1016/j.pediatrneuro1.2007.10.012.

American academy of pediatrics 759


American Academy of Pediatrics PREP 2015

Item 248
A resident you have been working with has seen a number of teenage girls who have pelvic
inflammatory disease with interesting presenting symptoms. The resident would like to share this
with other pediatricians by publishing a case series.

Of the following, the BIGGEST advantage of this type of scholarly activity is the ability to
A. calculate incidence rates
B. determine causal agents
C. generate etiologic hypotheses
D. generate preventive hypotheses
E. study high-frequency conditions

American academy of pediatrics 760


American Academy of Pediatrics PREP 2015

Item 248 PBLI Preferred Response: C


When deciding on the type of study design to use in a research project, it is important to
understand the advantages and limitations of each such design. The vignette refers to a proposed
case series study to share presenting symptoms of girls with pelvic inflammatory disease. There
is no plan to manipulate a study factor or randomize subjects, rather it is observational work and
falls into the subtype called a descriptive study design. These studies describe individual health
characteristics and are useful in generating etiologic hypotheses, for health planning purposes
and for allocating resources. They have the advantage of being inexpensive and relatively easy to
conduct.

Calculation of incidence rates requires that a number of new cases be detected in a given
population over a defined period, and thus cannot be calculated with the available information.
Determining causal agents would require a comparative study design and this type of study can
only be conducted after the generation of etiologic hypotheses. Only after possible etiologic
factors are determined can preventive hypotheses be generated. Low-frequency conditions are
best studied with a case-control study design. The lack of a comparison group in a case series
negates the ability to make any causal inference.

PREP Pearls
• "Case series" is a descriptive study design that is inexpensive and convenient, but with
limited uses.
• A case series is best used for generation of etiologic hypotheses, health planning
purposes, and allocation of resources.

American Board of Pediatrics Content Specification(s)


• Understand the uses and limitations of case reports/series and anecdotal evidence

Suggested Reading
• Jennings JM, Sibinga E. Research and statistics: understanding and identifying bias in
research studies. Pediatr Rev. 2010;31(4):161-162. doi:10.1542/pir.31-4-161.
• Johnson SL. Research and statistics: a question of time: cross-sectional versus
longitudinal study designs. Pediatr Rev. 2010;31(6):250-251. doi:10.1542/pir.31-6-250.
• Perry-Parrish C, Dodge R. Research and statistics: validity hierarchy for study design and
study type. Pediatr Rev. 2010;31(6):27-29. doi:10.1542/ pir.31-1-27.
• Upadhya K, Rowe P. Research and statistics: case-control studies. Pediatr Rev. 2030;
31(6):70-71. doi:10.1542/pir.31-2-70.

American academy of pediatrics 761


American Academy of Pediatrics PREP 2015

Item 249
An 18-year-old young man is seen for evaluation of chronic rhinosinusitis. He has a history of
sneezing, rhinorrhea, and nasal congestion during the last 2 spring seasons. He takes aspirin on
occasion for headaches associated with the congestion. He has a history of asthma that was
seasonal and mild in the past, but has worsened this year with almost daily asthma symptoms and
1 hospitalization this past fall. He denies a history of nose bleeds, pneumonias, or bronchitis. He
has not had chronic diarrhea or failure to thrive. He admits to recreational drug use, primarily
marijuana. On physical examination, his weight is at the 50th percentile and his height is at the
90th percentile. You observe fleshy, pedunculated polyps in the posterior end of both nasal
cavities. His nasal mucosa appears edematous and he has mucoid drainage bilaterally. You also
notice a cobblestone appearance at the back of his throat and mucoid postnasal drainage. His
heart sounds are heard prominently in the left side of his chest and his lungs are clear. The
remainder of his physical examination is unremarkable. Laboratory data shows:
• White blood cell count, 5,000/µL (5.00 x 109/L), with 55% neutrophils, 3% bands, 35%
lymphocytes, 5% monocytes, and 2% eosinophils
• Hemoglobin, 14 g/dL (140 g/L)
• Platelets, 200 x 103/µL (200 x 109/L)

Of the following, the MOST likely diagnosis is


A. aspirin sensitivity
B. Churg-Strauss syndrome
C. cocaine use
D. cystic fibrosis
E. rhinitis medicamentosa

American academy of pediatrics 762


American Academy of Pediatrics PREP 2015

Item 249 Preferred Response: A


The young man in the vignette has bilateral nasal polyps. He also has a history of seasonal
allergic rhinitis with symptoms worsening in the spring, and seasonal asthma that has now
become perennial. Nasal polyps may occur in conjunction with chronic rhinitis and sinusitis and
should be considered in the differential diagnosis of patients who present with constant nasal
congestion or anosmia. Of the conditions mentioned in the vignette, aspirin sensitivity is the
most likely pathogenic cause of this young man's polyps. Aspirin-exacerbated respiratory disease
(AERD), previously referred to as the "aspirin triad; consists of nasal polyps, acetylsalicylic acid
intolerance, and asthma, and is identified in 13% to 40% of patients with nasal polyposis.
Aspirin-exacerbated respiratory disease rarely occurs before the adolescent years. When
associated with asthma, patients with nasal polyposis excrete increased urinary leukotriene E4
(LTE4), suggesting that overproduction of cysteinyl leukotrienes may play a pathogenic role. In
patients with nasal polyposis and AERD, aspirin desensitization, followed by long-term daily
aspirin maintenance treatment, may be considered. Though eosinophils are a consistent finding
in nasal polyp tissue, allergy does not appear to predispose to polyp formation and only 10% to
15% of patients with allergic rhinitis also have nasal polyps.

Churg-Strauss syndrome occurs in middle-aged men and is a rare systemic vasculitis occurring
in patients with asthma and blood eosinophilia. The lungs, skin, and nervous system are the most
common sites of involvement, although many other organs are affected frequently. In addition to
the absence of eosinophilia of greater than 10%, there is no evidence of nasal septal perforation
in this patient classically seen in this condition, secondary to vasculitis.
While a history of marijuana use may prompt exploration of other recreational drug use such as
cocaine, the absence of nasal septal perforations make it less likely. Polyps are typically not
found with cocaine abuse, and the nasal mucosa in habitual cocaine users appears inflamed
(erythematous) with a watery discharge. Rarely, nasal collapse, septal perforations, palatal
retractions, and pharyngeal wall ulceration have all been reported.

Cystic fibrosis (CF) is to be suspected in any child younger than 10 years of age with nasal
polyps, particularly bilateral. Nasal polyps may occur in up to 50% of children with CF. The
older age of the patient, absence of other findings (gastrointestinal and respiratory), and normal
growth parameters make this less likely.

There is no history of overuse of nasal decongestants or evidence of irritation of the nasal


mucosa, making rhinitis medicamentosa less likely. Rhinitis medicamentosa is seen following
repetitive and prolonged use of topical a-adrenergic nasal decongestant sprays, such as
oxymetazoline and phenylephrine, and occasionally after the recreational use of cocaine, as
previously mentioned. Patients may develop rebound congestion, tachyphylaxis, and reduced
mucociliary clearance because of the loss of ciliated epithelial cells, and on rare occasions, nasal
septal perforation. The nasal mucous membranes may appear "beefy-red" and boggy, with
punctate bleeding and areas of increased tissue friability, along with profuse, stringy, mucoid
discharge.

PREP Pearls
• The finding of nasal polyps in a child younger than 10 years of age should prompt
consideration of cystic fibrosis as an associated condition.
American academy of pediatrics 763
American Academy of Pediatrics PREP 2015

• Aspirin-exacerbated respiratory disease is a triad consisting of nasal polyps,


acetylsalicylic acid intolerance, and asthma seen in adolescents and adults, and may
warrant consideration of aspirin desensitization therapy.
• While eosinophils are a consistent finding in nasal polyp tissue, only 10% to 15% of
patients with allergic rhinitis also have nasal polyps.

American Board of Pediatrics Content Specification(s)


• Recognize the clinical findings associated with nasal polyps in patients of various ages
• Identify conditions associated with nasal polyps in patients of various ages

Suggested Reading
• National Institute on Drug Abuse. Cocaine: abuse and addiction. National Institutes of
Health website.
• Wallace DV, Dykewicz MS, Bernstein DI, at al. The diagnosis and management of
rhinitis: an updated practice parameter. J Allergy Clin Immurrol. 2008;122(2 Suppl):S1-
S84. doi:10.1016/j.jaci.2008.06.003.

American academy of pediatrics 764


American Academy of Pediatrics PREP 2015

Item 250
You are taking care of a newborn in the neonatal intensive care unit who has had bluish
discoloration since birth 1 hour ago. He is the product of an uneventful pregnancy and normal
spontaneous term vaginal delivery. Apgar scores were 7 at one min and 8 at five min. Vital signs
show a temperature of 37°C, pulse rate of 140 beats/min, blood pressure of 70/30 mm Hg,
respiratory rate of 30 breaths/min, and SpO2 of 75% on room air. Physical examination shows a
well-appearing newborn who has cyanotic lips, trunk, and extremities. He has good tone, normal
cry, and moves all extremities. He is breathing comfortably, and lungs are clear to auscultation
bilaterally. He has warm and well-perfused extremities. His heart has a regular rate and rhythm.
When he is placed on 100% supplemental oxygen, his pulse oximetry reading does not change.

Of the following, the MOST likely cause of his cyanosis is


A. amniotic fluid aspiration
B. anomalous left coronary artery from pulmonary artery
C. pneumonia
D. transposition of the great arteries
E. ventricular septal defect

American academy of pediatrics 765


American Academy of Pediatrics PREP 2015

Item 250 Preferred Response: D


The newborn in the vignette has cyanosis from transposition of the great arteries (TGA), which is
the most common cyanotic congenital heart lesion presenting at birth. As the newborn was
breathing comfortably and placement in 100% oxygen did not improve his oxygen saturation, the
cyanosis is more likely cardiac in origin. Nonpulmonary causes of cyanosis are listed in Item
C250.

Cyanosis is a bluish discoloration of the skin that is visible centrally in the presence of more than
5 g of deoxygenated hemoglobin per dL of blood. When deoxygenated systemic venous or
pulmonary arterial blood is shunted to the systemic arterial system without getting oxygenated, it
can originate from a pulmonary cause or a nonpulmonary cause. Intrapulmonary shunting is
typically caused by ventilation-perfusion mismatching in which areas of the lung are perfused
but not ventilated. This can be caused by pneumonia, increased diffusion barrier, atelectasis, and
pulmonary edema. The oxygen saturation in most instances of intrapulmonary shunting can be
improved by increasing the fraction of inspired oxygen (FiO2). By increasing the alveolar
pressure of oxygen (PO2), a higher driving force for diffusion of oxygen into the pulmonary
capillaries will increase the arterial PO2. In contrast, increasing the FiO2 in a patient with an
intracardiac shunt without significant lung disease will not compensate for amounts of blood that
are directly shunted right to left, and the patient will remain cyanotic. Thus, increasing the FiO2
can help differentiate these 2 causes.

Transposition of the great arteries is the most common form of cyanotic congenital heart disease
that presents at birth. Oxygenated pulmonary venous blood is not readily able to enter the
systemic circulation because it is ejected back into the pulmonary circulation from the left
ventricle. In order for survival, mixing must occur at the atrial level, ventricular level, or via the
ductus arteriosus to allow oxygenated blood to be sent to the systemic circulation. An infusion of
prostaglandin E, is necessary for newborns with TGA to maintain ductal patency. Other
conditions that can present with cyanosis at birth include pulmonary valve atresia and severe
Ebstein malformation. Chest radiography can be helpful in distinguishing among these
conditions because TGA may show evidence of increased pulmonary blood flow, whereas
pulmonary atresia and Ebstein malformation typically show decreased pulmonary blood flow.
Primary pulmonary hypertension in the newborn with right-to-left shunting across the ductus
arteriosus can also cause cyanosis. This condition, in the absence of structural heart disease, can
be caused by sepsis, asphyxia, meconium aspiration, diaphragmatic hernia, and pulmonary
hypoplasia. Pulmonary hypertension can also be a cause of cyanosis because of decreased
pulmonary blood flow. It can be primary in nature or secondary to increased pulmonary vascular
congestion from heart failure, left ventricular outflow tract obstruction, or large left-to-right
shunts.

Anomalous left coronary artery from pulmonary artery causes coronary insufficiency when the
pulmonary vascular resistance decreases several weeks after birth, but does not cause cyanosis.
The newborn's lungs were clear, and his oxygen saturations did not improve with 100% oxygen,
therefore a pulmonary cause of cyanosis, such as pneumonia or amniotic fluid aspiration, is less
likely. Isolated ventricular septal defects do not typically present with cyanosis at birth.
Pulmonary causes of cyanosis should be distinguished from nonpulmonary causes. Especially in
the newborn period, this can be accomplished by administering 100% oxygen. If a cardiac cause
American academy of pediatrics 766
American Academy of Pediatrics PREP 2015

of the cyanosis is suspected, prostaglandin E, should be started to maintain patency of the ductus
arteriosus.

Item 0250: Nonrespiratory Causes of Cyanosis


• Cardiac Defects
o Decreased pulmonary blood flow
o Admixture lesions
o Congestive heart failure (pulmonary congestion)
• Primary Pulmonary
o Hypertension of the Newborn
• Central Nervous System Disease
o Intracranial hemorrhage
o Maternal sedative administration
o Meningitis
• Methemoglobinemia
• Hypoglycemia
• Sepsis
• Cold
Reprinted with permission. Tingelstad I. Consultation with the specialist nonrespiratory
cyanosis. Pediatr Rev. 1999;20;30:150

PREP Pearls
• Pulmonary causes can often be distinguished from cardiac causes of cyanosis by
administering 100% oxygen.
• Transposition of the great arteries is a common cardiac cause of cyanosis presenting at
birth.
• Cyanosis is a bluish discoloration of the skin caused by 5 g/dL of deoxygenated
hemoglobin.

American Board of Pediatrics Content Specification(s)


• Identify the common extrapulmonary causes of cyanosis
• Plan the appropriate clinical and laboratory evaluation of cyanosis

Suggested Reading
• Lees MH, King DH. Cyanosis in the newborn. Pediatr Rev. 1987;9(2):3642.
doi:10.1542/pir.9-2-36.
• Silberbach M, Hannon D. Presentation of congenital heart disease in the neonate and
young infant. Pediatr Rev. 2007;28(4):123-131. doi:10.1542/ pir.28-4-123.
• Tingelstad I. Consultation with the specialist: nonrespiratory cyanosis. Pediatr Rev.
1999;20(10:350-352. doi:10.1542/pir.20-10-350.

American academy of pediatrics 767


American Academy of Pediatrics PREP 2015

Item 251
A 16-year-old adolescent boy presents to the emergency department after being in a fight with
another boy in the schoolyard. During the altercation, he punched the boy in the mouth and
comes for evaluation of a laceration over the knuckles. He is otherwise well, and his
immunizations are reportedly up to date. He is alert and in no acute distress. Findings on physical
examination are unremarkable except for a 1.5-cm laceration over his third to fourth
metacarpophalangeal joints. After cleaning the wound, you decide to give him a dose of
parenteral antibiotic before discharge on an oral regimen for 5 days.

Of the following, the BEST choice for the parenteral antibiotic is


A. ampicillin-sulbactam
B. cefazolin
C. clindamycin
D. nafcillin
E. vancomycin

American academy of pediatrics 768


American Academy of Pediatrics PREP 2015

Item 251 Preferred Response: A


A clenched fist injury, such as described for the adolescent boy in this vignette, poses a risk for
infection with skin organisms (eg, Staphylococcus, Streptococcus) and human oral flora
including streptococcal species and anaerobes including Eikonella corrodens, Bacteroides and
Peptostreptococcus species. Up to 10% to 15% of human bite wounds become infected and in
this location may spread to the joint capsule or deeper into the tendon spaces of the hand. Wound
irrigation is an important part of the initial management.

Given the spectrum of bacteria implicated in human bite infections, ampicillin-sulbactam, of the
agents listed, provides the most complete coverage for staphylococci, streptococci, and oral
anaerobes. It does not provide coverage for community-acquired, methicillin-resistant
Staphylococcus aureus (MRSA), but this has only been rarely reported in human bite infections.
Clindamycin does provide coverage for MRSA and other gram positive organisms, but is not
effective against Eikenella corrodens, which has been shown to be an important component of
human bite wound infections. In a patient with penicillin allergy, clindamycin plus trimethoprim-
sulfamethoxazole would be the recommended alternative regimen. Nafcillin and vancomycin
cover staphylococci and streptococci, but not oral anaerobes.

In addition to acute bacterial infections, human bites can transmit hepatitis B if the biting
individual is a chronic carrier or actively infected with the virus. HIV transmission in this setting
is much less likely, but has been described in several case reports.

PREP Pearls
• Clenched fist injuries pose a 10% to 15% risk of becoming infected.
• Infections from human bites are caused by skin organisms (Staphylococcus,
Streptococcus) and oral flora (Streptococcus species and oral anaerobes including
Eikenella corrodens, Bacteroides, and Peptostreptococcus species).

American Board of Pediatrics Content Specification(s)


• Plan the appropriate management of a human bite

Suggested Reading
• American Academy of Pediatrics. Bite wounds. In: Pickering LK, Baker CJ, Kimberlin
DW, Long SS, eds. Red Book: 2012 Report of the Committee on Infectious Diseases.
29th ed. Elk Grove Village, IL: American Academy of Pediatrics; 2012:205-206.
• Barrett J, Revis DR Ir. Human bites. Medscape. http://emedicine.
medscape.com/article/218901-overview. Updated February 11, 2014.

American academy of pediatrics 769


American Academy of Pediatrics PREP 2015

Item 252
You are evaluating a newborn in the well-baby nursery. He was born at 37 weeks' gestational age
after an uncomplicated pregnancy and weighs 3.2 kg. He is breastfeeding well. His physical
examination is remarkable for a systolic murmur and several white skin patches. You order
echocardiography, which shows cardiac rhabdomyomas. When you tell the parents the child's
diagnosis, they reveal to you that the paternal grandfather has the same disorder. The parents ask
what the chance is of another child having this same disorder.

Of the following, the BEST response to the parents' question is


A. <1%
B. 25%
C. 50%
D. 75%
E. 100%

American academy of pediatrics 770


American Academy of Pediatrics PREP 2015

Item 252 Preferred Response: C


The newborn in the vignette has tuberous sclerosis complex (TSC), manifested by ash leaf spots
and cardiac rhabdomyomas. Tuberous sclerosis complex results from a mutation in either the
TSC1 gene on chromosome 9 or the TSC2 gene on chromosome 16. About two-thirds of cases
are caused by de novo mutations, but when inherited, TSC is transmitted in an autosomal
dominant fashion with variable expressivity. This can mean that a parent might not even know
they have tuberous sclerosis. In the vignette, the newborn as well as his paternal grandfather are
affected, which implies that the father also carries the mutation. The chance of a subsequent
child having TSC would be 50% in this case.

Neurocutaneous syndromes are genetic disorders affecting the brain, skin, and eye. The most
common are tuberous sclerosis, neurofibromatosis, and Sturge-Weber syndrome. More rare ones
include ataxia telangiectasia, incontinentia pigmenti, and von Hippel-Lindau disease. Tuberous
sclerosis and neurofibromatosis type 1 are inherited in an autosomal dominant pattern, but two-
thirds and one-half of cases, respectively, are caused by de novo mutations. Sturge-Weber
syndrome is nonheritable. The rarer neurocutaneous syndromes have variable inheritance
patterns: ataxia telangiectasia is autosomal recessive, incontinentia pigmenti is X-linked
dominant, and von Hippel-Lindau is autosomal dominant.

PREP Pearls
• Most cases of tuberous sclerosis complex result from de novo gene mutations.
• When inherited, tuberous sclerosis is transmitted in an autosomal dominant fashion.

American Board of Pediatrics Content Specification(s)


• Recognize the inheritance pattern of neurocutaneous hamartoses (eg, neurofibromatosis)

Suggested Reading
• Owens J, Bodensteiner JB. Tuberous sclerosis complex: genetics, clinical features, and
diagnosis. UpToDate. Available online only for subscription.

American academy of pediatrics 771


American Academy of Pediatrics PREP 2015

Item 253
A mother comes into your pediatric office to verify her child's antihypertensive medication. Her
son was recently discharged from the hospital, and a new prescription for clonidine was called
into the local pharmacy. After she got home, after picking up the prescription, she thought the
pills looked different than the clonidine pills that he was taking previously. She did not give the
new pills to her son. She shows you the bottle that is labeled as Klonopin®.

Of the following, this scenario is BEST defined as a(n)


A. adverse drug event
B. near miss event
C. never event
D. sentinel event
E. serious reportable event

American academy of pediatrics 772


American Academy of Pediatrics PREP 2015

Item 253 SBP Preferred Response: B


The filling of the prescription in this vignette with Klonopin* instead of clonidine is a near miss
event. The Joint Commission defines a near miss event as "any process variation which did not
affect the outcome, but for which a recurrence carries a significant chance of serious outcome."
The incorrect medication was identified before being given to the patient and no harm occurred.
If incorrect filling of a clonidine prescription with Klonopin• recurs, this or another patient may
take the incorrect medication and experience serious harm.

The incorrect filling of the prescription in the vignette could also be called a medical error.
Medical errors are defined by the Institute of Medicine as "the failure to complete a planned
action as intended or the use of a wrong plan to achieve an aim?' A medical error does not need
to result in harm to the patient. The most common type of medical error is a medication error.
Medication errors can occur anywhere between the ordering and administration of a drug,
including the incorrect filling of the prescription.

An adverse drug event (ADE) is an injury to the patient due to a medication. The incorrect filling
of the prescription in the vignette is not an ADE, but a potential ADE, as it was intercepted
before the patient took the incorrect medication.

An unexpected occurrence involving death, serious physical injury, or serious psychological


injury, or the risk thereof, is defined as a sentinel event. All sentinel events require root cause
analysis, development of an action plan, implementation of the improvements, and monitoring of
the improvements. Near miss events that are not identified may recur, with the outcome being a
sentinel event.

A serious reportable event is defined by the National Quality Forum (NQF) as an error in
medical care that is identifiable, preventable, and has serious consequences for the patient. These
have also been called "never events," as they should never happen in a hospital. The NQF has
identified 28 events that are considered serious reportable events, including wrong side surgery
and contaminated drugs.

PREP Pearls
• A medical error is defined by the Institute of Medicine as the failure to complete a
planned action as intended or the use of a wrong plan to achieve an aim."
• A near miss event is defined by the Joint Commission as "any process variation which
did not affect the outcome but for which a recurrence carries a significant chance of
serious outcome."
• A sentinel event is an unexpected occurrence involving death, serious physical injury, or
serious psychological injury, or the risk thereof.

American Board of Pediatrics Content Specification(s)


• Understand and apply the definition of a medical error
• Understand and apply the definition of a near miss event
• Understand and apply the definition of a sentinel event

Suggested Reading
American academy of pediatrics 773
American Academy of Pediatrics PREP 2015

• Kuhn LT, Corrigan JM, Donaldson MD, eds. To Err is Human: Building a Safer Health
Care System. Washington, DC; National Academy Press; 1999.
• Leonard MS. Patient safety and quality improvement: medical errors and adverse events.
Pediatr Rev. 2010;31(4)151-158. doi:10.1542/pir.31-4-151.
• Morrell RC. lCA1-30 revises standards to help reduce medical errors. APSF Newsletter.
2001;16(2).

American academy of pediatrics 774


American Academy of Pediatrics PREP 2015

Item 254
A 2-week-old female newborn arrives at your office for follow-up of bilateral branchial cleft
cysts, failed newborn hearing screen, external auditory canal stenosis, and preauricular pits noted
in the newborn nursery. The newborn is otherwise developing normally. She is feeding well. She
can regard her mother's face at short distances and has appropriate Moro, suck, and grasp
reflexes. The mother has already scheduled a follow-up appointment with audiology and
otolaryngology to further assess her potential hearing loss.

Of the following, the MOST appropriate study to assess for other associated anomalies is
A. barium swallow
B. brain magnetic resonance imaging
C. echocardiogram
D. renal ultrasonography
E. skeletal survey

American academy of pediatrics 775


American Academy of Pediatrics PREP 2015

Item 254 Preferred Response: D


The newborn in the vignette has classic branchio-oto-renal (BOR) syndrome, which is an
autosomal dominant disorder. Of the choices listed, renal ultrasonography is most likely to find
associated abnormalities. Branchio-oto -renal is a spectrum of disorders that includes BOR and
branchio-otic syndrome. Classic features include ear malformations (inner, outer, or middle)
associated with hearing loss, branchial cleft cysts or fistulae, and renal anomalies. Cup-ear
deformity and preauricular pits and tags are common. Some patients will evolve to end-stage
renal disease later in life, so it is imperative that physicians recognize this association because
progressive kidney disease can be initially silent. Branchio-otic syndrome is similar to BOR
syndrome, but without the renal manifestations. There is significant variability in the severity,
existence, and type of malformations even within the same family. This is known as variable
expression in genetics. This is an autosomal dominant disorder, therefore affected individuals
have a 50/50 risk for passing it on to their children.

Otologic abnormalities are present in 90% of patients with BOR syndrome. This typically
includes some variant of hearing loss (mixed, conductive, or sensorineural) with abnormalities of
the pinnae, external auditory canal, and middle and inner ear. These patients should have a
complete auditory acuity assessment and computed tomography of the temporal bones. Renal
defects could be unilateral or bilateral. The estimated incidence of renal anomalies is close to
70%, though the true incidence is unclear because not all physicians recognize the necessity to
screen with renal ultra-sonography, serum urea nitrogen, creatinine, and urinalysis. Anomalies
could include renal agenesis, hypoplasia, dysplasia, ureteropelvic junction obstruction, calyceal
cysts, pelviectasis, hydronephrosis, and vesicoureteral reflux. Second branchial arch anomalies,
which include branchial cleft cysts and fistulae, are best assessed by examination of the neck, as
well as computed tomography of the neck if a mass is noted.

Patients with BOR do not have associated cardiac lesions, abnormalities of the brain, or tracheal
and intestinal anomalies, so echocardiography, brain magnetic resonance imaging, and barium
swallow are not indicated in the child in this vignette. Branchio-oto-renal syndrome is not
associated with skeletal dysplasia that would manifest any abnormal findings on a skeletal
survey. Thus, renal ultrasonography is the most appropriate study to look for kidney anomalies
that might not otherwise be visible or palpable on clinical examination alone. These anomalies
can progress to end-stage renal disease, therefore it is essential that the physicians recognize this
association to prevent or ameliorate progressive kidney disease that may need medications or
surgical treatments to optimize the patient's long-term prognosis. Multidisciplinary involvement
would include audiology, otolaryngology, urology, nephrology, and early intervention. These
patients do not typically have intellectual deficit. Early intervention would be necessary to
optimize communication and educational measures related to the patient's hearing loss.

PREP Pearls
• Any patient with a branchial cleft cyst or fistulae should have a thorough examination
and auditory assessment of the ear as well as renal ultrasonography, serum urea nitrogen,
creatinine, and urinalysis to assess for associated abnormalities consistent with branchio-
oto-renal syndrome or branchio-otic syndrome.
• Annual audiometry screening is recommended to look for any progressive hearing loss.

American academy of pediatrics 776


American Academy of Pediatrics PREP 2015

• Renal assessment should include laboratories to assess renal function and prevent
progression to end-stage renal disease. Semiannual or annual examination by an urologist
or a nephrologist may be necessary should any evidence of decreased renal function or
significant abnormalities of the kidney system be present.

American Board of Pediatrics Content Specification(s)


• Recognize the clinical features associated with branchial cleft anomalies, and plan
appropriate evaluation and management

Suggested Reading
• Chang El-I, Menezes M, Meyer NC, et al. Branchio-oto-renal syndrome: the mutation
spectrum in EYA I and its phenotypic consequences. Hum Mutat. 2004;23(6):582-589.
doill0.1002/humu.20048.
• Smith RJH. Branchiootorenal spectrum disorders. Gene Reviews.

American academy of pediatrics 777


American Academy of Pediatrics PREP 2015

Item 255
The mother of a 9-year-old girl wants her daughter evaluated for possible sexual abuse. The
parents are separated and the patient visits her father every other weekend. The mother has
noticed some discharge on the underwear since the patient returned from visiting her father this
weekend. The parents are in a custody battle and the mother wants evidence for court. The
patient is tearful and hesitant to be examined. The physical examination is unremarkable. You
review your ethical obligations for confidentiality and privacy of personal health information in
reporting child abuse cases.

Of the following, the MOST correct statement is that


A. documentation should be limited in cases where a parent is the possible perpetrator and
may view medical records
B. in order to consult another health care professional to assist in the investigation, parental
authorization is required
C. it is never permissible for the pediatrician to withhold information from either biologic
parent
D. it is not mandatory that you report this case to child protective services because the
examination is normal
E. state agencies designated to investigate child abuse reports may receive protected health
information important to the investigation without legal guardian authorization

American academy of pediatrics 778


American Academy of Pediatrics PREP 2015

Item 255 SBP P I-C Preferred Response: E


The medical record is a medicolegal document and physicians need to be familiar with state and
federal laws regarding disclosure of a child's protected health information. The Federal Health
Insurance Portability and Accountability Act (HIPAA) of 1996 addresses the security and
privacy of patient health data and has significantly affected how patient information is shared.
The HIPAA privacy rules apply to the physician of record. In general, HIPAA permits disclosure
of information without authorization by the legal guardian for the purposes of treatment or
intervention, and investigation of matters that relate to abuse or neglect, public health, and safety.
Mandatory reporting laws state that once a practitioner has reasonable cause to suspect child
abuse or neglect, he or she is obligated to report this to Child Protective Services (CPS). This
holds true even when the physical examination findings are unremarkable. In cases of sexual or
physical abuse, CPS usually contacts police to investigate for possible criminal behavior.
Alleged abuse is best investigated by trained health care professionals, along with child life
specialists, social workers, and law enforcement. The history and physical examination findings
should be carefully documented in detail, both for use by legal agencies and the physician in the
event of future court testimony. Parents of the child should be informed that the child abuse
report is being made unless the safety of the child cannot be guaranteed. The child's security is a
priority and once the safety of the child is ensured, then the parents may be informed.
It is permissible to withhold the child's protected health information from the parent if there is
concern that the parent is the abuser or is protecting an abuser. Therefore, one should not limit
documentation out of concern that the perpetrator will have access to the medical record.
Protected health information may be shared with other health care professionals consulted for the
purpose of investigation of the case without parental permission. In addition, the health care
provider may disclose protected health information to CPS or law enforcement agencies
designated to investigate child abuse reports without parent or legal guardian authorization.

PREP Pearls
• Physicians need to be familiar with state and federal laws regarding disclosure of a child's
protected health information.
• Once child abuse or neglect is suspected, a practitioner is obligated to report to Child
Protective Services (CPS}.
• Disclosure of protected health information without authorization by the legal guardian is
permitted for the purposes of treatment or intervention, and investigation of matters that
relate to abuse or neglect, public health, and safety.

American Board of Pediatrics Content Specification(s)


• Recognize and apply ethical principles regarding the issues of physical and mental abuse
• Recognize and apply ethical principles regarding child abuse (eg, confidentiality,
reporting)

Suggested Reading
• American Academy of Pediatrics Committee on Child Abuse and Neglect. Policy
statement: child abuse, confidentiality, and the health insurance portability and
accountability act. Pediatrics. 2010;125(1):197-201. doi:10.1542/peds.2009-2864.
• Asnes AG, Leventhal IM. Managing child abuse: general principles. Pediatr Rev.
2010;31(2):47-55. doi:10.1542/pir.31-2-47.
American academy of pediatrics 779
American Academy of Pediatrics PREP 2015

• Endom EE. Child abuse: social and medicolegal issues. UpToDate. Available online only
for subscription.
• Flaherty EG, Stirling J Jr, American Academy of Pediatrics, Committee on Child Abuse
and Neglect. Clinical report: the pediatrician's role in child maltreatment prevention.
Pediatrics. 2010;126(4):833-841. doi:10.1542/ peds.2010-2087.
• Fisher MA. Ethics for the pediatrician: caring for abused children. Pediatr Rev.
2011;32(7):e73-e78. doi:10.1542/pir.32-7-e73.

American academy of pediatrics 780


American Academy of Pediatrics PREP 2015

Item 256
A 10-month-old female infant presents to your office with fatigue and pallor. The mother reports
that her daughter has a persistent cough but no runny nose or fever. The infant's past medical
history reveals 2 viral upper respiratory illnesses in the past month. Chest radiographs obtained
at that time showed diffuse bilateral opacities. The infant previously had 3 other viral illnesses
with persistent coughing. She drinks 28 oz to 32 oz of cow's milk based formula per day, as well
as age-appropriate solid foods. In the office, her temperature is 37.7°C, pulse rate is 120
beats/min, respiratory rate is 35 breaths/min, and blood pressure is 90/45 mm Hg. She is at the
50th percentile for weight and height. On physical examination, the child is pale and sleeping
comfortably. She has anicteric sclerae and no hepatosplenomegaly or lymphadenopathy. The
remainder of the physical examination is unremarkable. Chest radiographs show persistent
patchy infiltrates bilaterally.

The following are the results of the infant's laboratory tests: Complete blood cell count:
• White blood cell count, 9,500/µL (9.5 x 109/L), with normal differential
• Hemoglobin, 5.0 g/dL (50 g/L)
• Mean corpuscular volume, 63 µm3 (63 fL)
• Platelet count, 510,000 x 103/µL (510 x 109/L)
• Reticulocyte count, 1.5%
• Absolute reticulocyte count, 0.06 x 106/µL (reference range, 0.029-0.099 x 106/ µL)
• Ferritin, 9 ng/mL (20 pmol/L, reference range, 10-95 ng/mL [22-213 pmol/L])
• Stool for occult blood, negative

Of the following, the test MOST likely to detect the cause of anemia in this patient is
A. bronchoalveolar lavage
B. creatinine clearance
C. direct antiglobulin test (direct Coombs)
D. mycoplasma titers
E. serum immunoglobulins

American academy of pediatrics 781


American Academy of Pediatrics PREP 2015

Item 256 Preferred Response: A


The infant in the vignette has a presentation concerning for idiopathic pulmonary hemosiderosis
(IPH), a rare but possibly fatal form of alveolar hemorrhage that can present as iron deficiency
anemia. The incidence of IPH is 0.24 to 1.26 per million and it occurs primarily in children
younger than 10 years of age. Clinically, it is characterized by the triad of hemoptysis, iron
deficiency anemia, and pulmonary infiltrates on chest radiograph. The diagnosis is confirmed by
the pathognomonic finding of hemosiderin-laden macrophages on bronchoalveolar lavage. The
causes of IPH are unclear, but it has been associated with various mechanisms. There may be an
allergic component based on the frequent association between IPH and cow's milk allergy.
Siblings have been reported as having IPH, which might suggest a genetic component. Idiopathic
pulmonary hemosiderosis has also been described in children exposed to certain infections.
Lastly, the association with celiac disease (ie, Lane-Hamilton syndrome), glomerulonephritis,
and rheumatoid arthritis seem to support an autoimmune cause of IPH. The mainstay of
treatment for IPH is corticosteroids. For refractory patients, other immunosuppressants may be
indicated.

Hemoptysis is seen only 44% of the time, whereas dyspnea and anemia are present 68% and
64% of the time, respectively, at diagnosis. Half of the cases with anemia are severe, with
hemoglobin less than 7 g/dL (70 g/L). Therefore, one must have a high index of suspicion for
IPH in a patient who presents with iron deficiency anemia and recurrent respiratory symptoms.
Untreated IPH can lead to progressive fibrotic changes in the lungs and eventually fatal
restrictive lung disease or hemorrhage.

Creatinine clearance would not be useful in diagnosing IPH. Renal disease as a cause of anemia
would not usually present as iron deficiency, but more likely anemia of chronic disease with a
ferritin that is elevated as an acute phase reactant. The direct antiglobulin is useful in diagnosing
autoimmune hemolytic anemia, which would be accompanied by an elevated reticulocyte count.
Mycoplasma infection can be associated with an autoimmune hemolytic anemia, though this is
less likely given the lack of reticulocytosis and the young age of this patient. Iron deficiency (as
suggested by the low ferritin) is not a typical feature of autoimmune hemolytic anemia. Serum
immunoglobulins may be useful in the evaluation of a patient with recurrent infections, but is not
likely to be pathognomonic for IPH or other causes of iron deficiency anemia.

PREP Pearls
• Idiopathic pulmonary hemosiderosis is a rare but possibly fatal form of alveolar
hemorrhage that can cause iron deficiency anemia, primarily in children younger than 10
years of age.
• Hemoptysis is seen only 44% of the time, whereas dyspnea and anemia are present 68%
and 64% of the time, respectively, at diagnosis.
• The mainstay of treatment for idiopathic pulmonary hemosiderosis is corticosteroids. For
refractory patients, other immunosuppressants may be indicated.

American Board of Pediatrics Content Specification(s)


• Recognize clinical findings associated with hemosiderosis
• Identify the risk factors associated with hemosiderosis

American academy of pediatrics 782


American Academy of Pediatrics PREP 2015

Suggested Reading
• Hendrickx GF, Somers K, Vandenplas Y. Lane-Hamilton syndrome: case report and
review of the literature. EurI Pediatr. 2011;170(12):1597-1602. doi:10.1007/500431-011-
1568-5.
• Poggi V, Lo Vecchio A, Menna F, Menna G. Idiopathic pulmonary hemosiderosis: a rare
cause of iron-deficiency anemia in childhood. Pediatr Hematol Oncol. 2011;33(4):e160-
e162. doi:10.1097/ MPH.0b013e318212a6df.
• Taytard J, Nathan N, de Blic J, et al; French RespiRare' group. New insights into
pediatric idiopathic pulmonary hemosiderosis: the French RespiRare' cohort. Orphanet I
Rare Dis. 2013;8(0:161. doi:10.1186/1750-1172-8-161.

American academy of pediatrics 783


American Academy of Pediatrics PREP 2015

Item 257
The mother of a 12-year-old boy who is in your office for his annual health supervision visit tells
you that she is worried about how well he is going to handle his parents' upcoming divorce. The
boy looks annoyed and uncomfortable when his mother brings up this subject, but he says
nothing and stares at the floor. The mother states that both parents have made a commitment to
work cooperatively on parenting him despite their separation. The mother has heard that divorce
can be very hard on children and is wondering how they can help their son during this process.
So far, no significant behavioral problems have occurred, although he is noted to be more often
sad and easily annoyed than he used to be. You have known him for a number of years and have
had no prior concerns about his emotional or physical development. He is active with a local
swim team, and he sings in both the school and community choirs.

Of the following, the MOST likely statement to be helpful for this young man is to
A. emphasize that he needs to remain just as engaged with his friends and activities
B. emphasize that his time with each parent be maintained "fairly," close to a 50-50
distribution
C. recommend holding off from strenuous exercise right now because it is emotionally
draining
D. send him to see a therapist, even if it means he will miss some of his afterschool activities
E. send him to summer camps that are specifically designed for children coping with a loss

American academy of pediatrics 784


American Academy of Pediatrics PREP 2015

Item 257 Preferred Response: A


Children who are experiencing a parental divorce, like the 12 year old in this vignette, have
many changes happening in their life. In addition to their feelings about the situation, they
typically experience changes in where they live and how their parents relate to them. Therefore,
one of the best ways to promote resilience in the case of a major life alteration like a parental
divorce is to promote as much consistency as possible in other areas of the child's life.
Continuing to attend the same school, participating in the same sports and extracurricular
activities, and maintaining contact with the same friends are all helpful ways to promote
resilience.

The way that the parents relate to their children should stay as consistent as possible. Parents
relaxing rules and indulging a child's material wishes in an attempt to compensate for the child's
emotional distress typically leads to difficulties on returning to previous limits and expectations.
Divorcing parents may off-load disagreements on to their children, treating the child like a
commodity that has to be divided equally like other assets. The principle of maintaining a "fair"
50-50 split of a child does not take into account the child's perspective of what might be best for
them. Ideally children should have their external social and school structures continue
uninterrupted, which may require an unequal custodial split because of parent work schedules
and living locations. Prolonged, adversarial custody arrangements have been associated with
poorer child adjustment.

Maintaining regular physical exercise has been found to help with coping and resilience.
Discontinuing a current age-appropriate social activity because of the emotional strain of
parental divorce would not support coping and resilience.

Therapy could be helpful for children who are having emotional or behavioral difficulties
associated with the changes resulting from parental divorce, but only when these difficulties
approach the dysfunctional range and are affecting day-to-day life. Some degree of emotional
distress is expected, such as sadness and being more easily annoyed, during the transitional phase
of divorce. Most children do not need professional help.

Sending the child away to a summer camp as an intervention to promote resiliency would disrupt
their normal social supports and would not be preferred. If such a camp were successful at
promoting new relationships, this too would result in new peer and adult support "losses" when
that camp ends.

PREP Pearls
• Children going through difficult transitional stressors, like a parental divorce, benefit
from maintaining as much of their school and social structure and routines as possible.
• Adversarial custody arrangements in a divorce have been associated with poorer child
outcome. A parenting plan that incorporates a child's best interests is more likely to help
with resilience.

American Board of Pediatrics Content Specification(s)


• Understand the concept of resilience and counsel parents on how to raise resilient
children
American academy of pediatrics 785
American Academy of Pediatrics PREP 2015

Suggested Reading
• American Psychological Association. The road to resilience. http://www.
apa.org/helpcenter/road-resitience.aspx. Accessed February 20, 2014. Kleinsorge C,
Covitz LM. Impact of divorce on children: developmental considerations. Pediatr Rev.
2012;31147-155. doi:10.1542/pir.33-4-147.

American academy of pediatrics 786


American Academy of Pediatrics PREP 2015

Item 258
A 10-year-old boy comes to your office for an initial visit. He has been in foster care for several
years, but is reunited with his mother. He had heart surgery at 2 years of age, but the patient does
not have any records from that procedure. As far as his mother knows, he has never been
hospitalized for respiratory illnesses such as asthma or pneumonia. He does not take any
medications. He has decreased exercise tolerance.
On physical examination, he is a slender boy in no acute distress. His weight is at the 10th
percentile for his age, and height is at the 50th percentile. His heart rate is 100 beats/ min and
respiratory rate is 28 breaths/min. Oxygen saturation on room air is 89%. His blood pressure is
95/65 mm Hg in the right arm, and his blood pressure is 115/90 mm Hg in the right leg. His neck
veins are not distended. His chest shows clear and equal breath sounds. There is a well-healed
sternotomy scar. His heart examination shows a normal S1 with a very loud P2 component to S2.
There is a 2/6 soft systolic murmur at the right midsternal border that does not radiate to either
axilla. There is no diastolic murmur. There is no organomegaly on abdominal examination. The
femoral pulses are 2+ without any radial femoral delay. His hemoglobin level is 17 g/dL (170
g/L).

Of the following, the MOST likely diagnosis is


A. coarctation of the aorta with poor systemic circulation
B. large residual ventricular septal defect with pulmonary over circulation
C. moderate-sized patent ductus arteriosus with left to right shunt
D. pulmonary hypertension with Eisenmenger syndrome
E. pulmonic stenosis with decreased pulmonary blood flow

American academy of pediatrics 787


American Academy of Pediatrics PREP 2015

Item 258 Preferred Response: D


This patient in the vignette has evidence of pulmonary hypertension (PHTN) and Eisenmenger
syndrome. Cardiac causes for PHTN include lesions that cause excessive pulmonary blood flow
over a long period of time. This typically can occur with a large ventricular septal defect (VSD)
that goes unrepaired.

At some point in childhood, depending on the amount of excess blood flow that has occurred
from a left-to-right shunt, the pulmonary vasculature will become damaged enough that the
pulmonary vascular resistance will increase, and the shunt will reverse from the right ventricle to
the left ventricle. In that case, the patient will have developed Eisenmenger syndrome. The
findings on examination would be a loud pulmonic component to S2 (or P2) and desaturation
with decreased exercise tolerance. In light of this patient's previous unknown cardiac surgery at
age 2 years, the most likely explanation for his physical findings and desaturation is
Eisenmenger syndrome.

The other major cardiac causes of PHTN are lesions that do not allow the pulmonary vasculature
to empty effectively, such as pulmonary vein stenosis and mitral stenosis. This causes pulmonary
venous hypertension, which eventually causes increased pulmonary vascular resistance if left
untreated.

A patient with a large residual VSD with pulmonary over-circulation will have elevated
pulmonary artery pressures, but will not have elevated pulmonary vascular resistance initially (or
there would not be increased blood flow to the lungs). Once there is vascular damage from
excess pulmonary flow, the pulmonary vascular resistance will increase and the patient will
develop PHTN. In the patient in the vignette, we are not told of a murmur that would suggest a
VSD.

Patients with coarctation will have left ventricular failure if there is severe obstruction. When the
pulmonary vascular resistance continues to be normally elevated in newborns, a component of
PHTN may exist if the infant is acidotic. In a 10-year-old patient, severe coarctation would not
be expected to cause PHTN. We are not told that the patient has decreased pulses or
hypertension.

A patent ductus arteriosus (PDA) will shunt from higher to lower pressure. A shunt that is left to
right or from the aorta to the pulmonary artery is evidence that the pulmonary artery pressure is
less than systemic. The patient in this vignette has the risk of developing PHTN, but the murmur
of a PDA was not heard on examination.

Pulmonic stenosis would cause desaturation, but would also cause a loud murmur in the left
upper sternal border, rather than just a loud P2, especially if it were severe.

Noncardiac causes of pulmonary hypertension include chronic lung disease, sickle cell anemia,
postviral inflammation, collagen vascular disease, medications, or primary angiopathy of the
pulmonary vascular bed. It is progressive, but the time course is dependent on the underlying
cause. Aggressive pulmonary vasodilator therapy may be required for this type of pulmonary
hypertension. The presentation may be subtle with mild exercise intolerance, elevated
American academy of pediatrics 788
American Academy of Pediatrics PREP 2015

hemoglobin, or mild desaturation. Evidence of elevated right ventricular pressure may be found
on the echocardiogram. Cor pulmonale refers to the right ventricular dysfunction that occurs
when the pulmonary vascular resistance is elevated in the absence of congenital heart disease.

PREP Pearls
• Pulmonary hypertension [PHTN) can be primary or secondary, early onset or late onset.
• Eisenmenger syndrome is part of a secondary PHTN complex caused by congenital heart
disease and history of excessive pulmonary blood flow.
• Cor pulmonale refers to the right ventricular dysfunction that occurs when the pulmonary
vascular resistance is elevated in the absence of congenital heart disease.

American academy of pediatrics 789


American Academy of Pediatrics PREP 2015

Item 259
A 17-year-old adolescent girl who is an elite level soccer player presents to your office for
evaluation of dyspnea and a feeling of throat tightening with exercise. This problem had an
insidious onset about 9 months ago. Her symptoms resolve within 3 minutes to 5 minutes of
exercise cessation. Her mother relates that the girl's previous physician treated her with inhaled
albuterol initially, and subsequently prescribed montelukast and an inhaled corticosteroid. She
did not see any improvement while taking these medications. The girl is otherwise healthy and
has no symptoms when she is not exercising. On physical examination, her respiratory rate is 12
breaths/min. She has no nasopharyngeal edema and her lungs are clear to auscultation.

Of the following, the treatment MOST likely to improve the girl's symptoms is
A. additional athletic training
B. nasal saline irrigation
C. oral amoxicillin-clavulanate
D. oral prednisone
E. speech therapy

American academy of pediatrics 790


American Academy of Pediatrics PREP 2015

Item 260
A healthy 22-month-old girl with no drug allergies presents to the emergency department for
repair of a 0.6-cm simple laceration located just above the medial aspect of her right eyebrow.
She sustained this laceration 1 hour ago, when she tripped while running inside her home and hit
her head on the corner of a coffee table. According to her mother, she experienced no loss of
consciousness and has been behaving normally since the injury occurred. The child appears very
well and is interacting playfully with her mother when you enter the room, but she becomes
extremely anxious and screams loudly whenever you approach her. Despite your gentle
approach, reassurance, and multiple attempts to calm her by using distraction strategies, she cries
and struggles to escape from her mother's arms as you perform a quick physical examination and
apply LET gel (4% lidocaine, 0.1 % epinephrine, and 0.5% tetracaine) to anesthetize the small
laceration. You determine that placement of 2 simple interrupted sutures will be required to
appropriately repair the girl's laceration. The girl's mother asks if you could order something to
"keep her calm" during the anticipated laceration repair.

Of the following, the MOST appropriate option for sedation in this patient is
A. intramuscular ketamine
B. intramuscular morphine
C. intranasal midazolam
D. intravenous etomidate
E. intravenous propofol

American academy of pediatrics 791


American Academy of Pediatrics PREP 2015

Item 260 Preferred Response: C

Morphine is a narcotic analgesic that provides effective pain relief and may have a sedating
effect in some children, especially when used in higher doses. For the young girl in the vignette,
the pain associated with the brief laceration repair should be well controlled by the topical
anesthetic that was applied. An anxiolytic agent, rather than a systemic analgesic like morphine,
is indicated.

Intravenous etomidate and intravenous propofol are both fast-acting and potent sedatives that
would achieve deeper levels of sedation than that needed for the patient in the vignette. These
agents should be administered only by providers who are skilled in their administration.
Numerous nonpharmacologic interventions, including imagery, hypnosis, relaxation, modeling,
parental presence, provision of information, and distraction, have been shown to be effective for
helping children cope with medical procedures. These interventions should certainly be used
before or in combination with anxiolytic medications. Preparation and communication are crucial
elements to incorporate in the planning and performance of distressing procedures for all
children. The use of sedation does not eliminate the need to properly prepare the child and
caregivers, and to use calming communication and nonpharmacologic strategies throughout the
procedure to reduce patient distress.

PREP Pearls
• Procedural sedation and analgesia involves the use of pharmacologic agents to provide
anxiolysis, analgesia, sedation, and motor control during procedures and diagnostic tests.
• Minimal sedation allows patients to respond normally to verbal commands and does not
impair cardiorespiratory function.
• The use of sedation does not eliminate the need to properly prepare the child and
caregivers, and to use calming communication and nonpharmacologic strategies
throughout the procedure to reduce patient distress.

American Board of Pediatrics Content Specification(s)


• Understand the appropriate use of minimal sedation (anxiolytic)

Suggested Reading
• Caglar D, Kwun R, Haines CJ. Pediatric procedural sedation and analgesia in the
emergency department. Pediatr Emerg Med Rep. 2011;16(8):93.
• Koh IL, Palermo P. Conscious sedation: reality or myth? Pediarr Rev. 2007;28(7)1:243-
248. doi:10.1542/pir.28-7-243.

American academy of pediatrics 792

You might also like